You are on page 1of 659

For More Visit : www.LearnEngineering.

in

n
g.i
rin
ee
gin
En
arn
Le
w.
ww

For More Visit : www.LearnEngineering.in


For More Visit : www.LearnEngineering.in

esjs I;kjs nksLrksa] ;fn vki jkds'k ;kno lj dh Online Class ysuk pkgrs gSa rks gekjs Offical You Tube Channel
Rakesh Yadav Readers Publication ( http://www.youtube.com/c/RakeshYadavReadersPublication)
dks Subscribe djsaA rFkk lHkh Chapters dh Video/ Purchase/Rent djsa vkSj vius vki dks Coaching dh HkhM+
ls cpk;saA
¶'Digital India' ds Concept dks viuk;s¸ vkSj ?kj cSBs Hkkjr ds loZJs"B Teacher 'Rakesh Yadav Sir (Wizard

.in
of Maths)' ls i<+sa] vkSj vklkuh ls ?kj cSBs Govt. Job esa selection ysAa

 ge gj Subject dh o gj Exam dh Online Class You Tube ij Provide djkrs gSaA

ng
 lHkh One Day Competitive Exams dh Online Coaching ds fy, Hkkjr ds ,d ek=k fo'oluh; laLFkku
Rakesh Yadav Readers Publication ls if<+;]s D;ksfa d ge Toppers vkSj Teachers nksuksa dks i<+krs gSaA

eri
 gekjs ;gk¡ Pendrive Classes Hkh Available gSa] ftUgsa vki gels [kjhn ldrs gSaA

e
gekjs ;gk¡ Coaching Institutes esa Video Classes Hkh Provide dh tkrh gSa (Offline o VAST ekè;e ls)A
gin
 All our Video Classes are different for : SSC/CDS/NDA/IBPS/SBI/DMRC/SSC-JE/ and other
one day examinations, each.
ge Hkkjr ds fofHk Coaching Centers ij Expert Teachers Hkh Provide djkrs gSaA dksbZ Hkh Coaching
En

Center ge ls Best Teachers ds fy, lEidZ dj ldrk gSA

gekjs ;gk¡ gj Hkk"kk esa Video Classes Available gSA


arn

fo|kFkhZ thou esa dke;kch ds fy, 3 ckrsa lnSo ;kn j[ksa%


1- D;k i<a+s % Rakesh Yadav Readers Publication dh Books i<a+sA
Le

2- fduls i<+sa % Rakesh Yadav Sir ds ekxZn'kZu esa i<+sa oks Hkh dsoy Online Classes ds ekè;e lsA
w.

3- D;k uk i<+s a% fdlh Hkh nwljs Publication dh Book uk i<+saA


ww

You can purchase All our books at To follow

www.rakeshyadavpublication.com Rakesh Yadav sir :


To purchase all our video classes in pendrive call at rakesh.yadav0011@gmail.com,
9268468484, 9268668686, 9868946424 https://twitter.com/rakeshyadavsir
For More Visit : www.LearnEngineering.in
For More Visit : www.LearnEngineering.in

.in
ng
eri
e
gin
En
arn
Le
w.
ww

For More Visit : www.LearnEngineering.in


For More Visit : www.LearnEngineering.in

n
g.i
Dedicated to

rin
My Beloved Daughters
Unnati Yadav & Pragati Yadav

ee
gin
t hr usd h bPNk l Hkh esagksrh gS
] exj t hr usd sfy , r S
; kj h d j usd h bPNk
cgqr d e y ksxksaesagksrh gS
A
En
arn

Key To Success
Le

Preparation = (Purpose + Principle + Planning + Practice


+ Perseverance + Patience + Pride) = Success
w.
ww

For More Visit : www.LearnEngineering.in


For More Visit : www.LearnEngineering.in

Published by

Rakesh Yadav Readers Publication Pvt. Ltd.

Address : H.no. 1425, Outram lane


near B.B.M. Depot GTB Nagar Delhi-110009

.in
Contact us: 92-686-686-86, 92-684-684-84

Edition-Ist 2016

ng
Copyright © publisher

eri
No part of this publication may be reproduced or distributed in any form or by any
means. Electronic Mechanical, Photocopying, Recording, Scanning or otherwise or stored

e
in a database, or retrieval system without the prior written of permission the publisher
gin
• Disclaimer
En

Information contained in this work has been obtained by Rakesh Yadav Readers
Publication Pvt. Ltd. from sources believed to be reliable, However neither Rakesh
Yadav Readers Publication Pvt. Ltd. or not Its Author guarantee the accuracy or
arn

completeness of any information published herein and neither Rakesh Yadav Readers
Publication Pvt. Ltd. or Its authors shall be responsible for any error, omissions or
damage arising out of use of this information, this is published with understanding
Le

that Rakesh Yadav Readers Publication Pvt. Ltd. and Its author are supplying information
but are not attempting to render engineering or other professional services, if such
w.

services are required, the assistance of an appropriate professional should be sought.


ww

All disputes subject to New Delhi Jurisdiction only

Editor-in-chief
Karan Chaudhary

For More Visit : www.LearnEngineering.in


For More Visit : www.LearnEngineering.in

n
g.i
Preface

rin
Nothing thrills a writer more than the success of his book. With this book,

ee
I hope to reach a much wider section of the student community and others,
who relentlessly compete for various Government – jobs.
gin
I am thankful to Almighty and my family (My parents, brother, wife,
daughters and son), who extended their help in various invisible ways.
En

I sincerely hope, the book ADVANCE MATHS will meet a good response.
I would humbly appreciate suggestions, doubt, etc. concerned with this book
arn

at the following.
Le

Email: rakesh.yadav0011@gmail.com
Whatsapp @–+91- 9868946424
w.
ww

Author
Rakesh Yadav

For More Visit : www.LearnEngineering.in


For More Visit : www.LearnEngineering.in

Content
1. Unit Digit ......................................................................................................................... 1 – 6

n
2. Number of Zeroes .......................................................................................................... 7 – 12

g.i
3. Factor ......................................................................................................................... 13 – 17
4. Divisibility .................................................................................................................... 18 – 28
5. Remainder Theorem.................................................................................................... 29 – 45

rin
6. A.P & G.P. ................................................................................................................... 46 – 57
7. Power Indices & Surds ................................................................................................ 58 – 78

ee
8. Simplification .............................................................................................................. 79 – 97
9. Linear Equation in two variables ................................................................................. 98 –100
gin
10. Polynomials .............................................................................................................. 101– 105
11. Algebric Identities .................................................................................................... 106 – 162
12. Trigonometry Identities ............................................................................................. 163 – 234
En

13. Maximum and Minimum value of Trigonometric function ........................................... 235 – 242
14. Height and Distance ................................................................................................ 243 – 268
arn

15. Trigonometry Circular Measure of Angles ................................................................. 269 – 280


16. Mensuration 2–D ..................................................................................................... 281 – 361
17. Mensuration 3–D ..................................................................................................... 362 – 440
Le

18. Quadrilaterals .......................................................................................................... 441 – 468


19. Polygon ................................................................................................................... 469 – 474
w.

20. Lines and Angles ..................................................................................................... 475 – 490


21. Triangles ................................................................................................................. 491 – 504
ww

22. Congruence and Similarity ....................................................................................... 505 – 537


23. Centre of Triangle .................................................................................................... 538 – 562
24. Circle, Chords and Tangents .................................................................................... 563 – 626
25. Co-Ordinate Geometry ............................................................................................ 627 – 650

For More Visit : www.LearnEngineering.in


For More Visit : www.LearnEngineering.in

CHAPTER

01
UNIT DIGIT
Last Digit of number is called Unit Ex.6: Find the unit digit at the Ex.14: Find the unit place (67354)1237
Digit product of all the odd prime Sol. (67354)1237

ir
numbers. unit digit = (4)1237 = (4) odd power
1234

v.iSn
sol. The prime numbers are 3, 5, 7, So, unit digit = 4
11, 13, 17, ...... etc. Ex.15: Find the unit place (3259)1214
unit digit Now we know that if 5 is multi- Sol. (3259)1214
In This no. 4 is unit digit. plied by any odd number it al- unit digit = (9)1214 = (9)even power

dnag
The unit digit of the ways gives the last digit 5. So unit digit = 1
R e s ulta nt v a lue d e p e n d s u p on the required unit digit will ne Ex.16: Find the unit place (6734)312
The unit digits of all '5', Sol. (6734)312
p ar tic ip ating numbers. Ex.7: Find the unit digit of 584 × 328
unit digit = (4)312 = (4)even

eYari
Ex.1: 23 + 34 + 46 + 78 = 181, unit × 547 × 613
unit digit = 6
digit of 181. Sol. The unit digits = 4, 8, 7, 3
Rule of (2, 3, 7 and 8) 
Sol. \ unit digit = 1 multiplying the unit digits
unit digit when 'N' is raised to
It is clear that the unit digit of =4×8×7×3
a power
the Resultant value 181 de-
pends upon the unit digits 3,
4, 6, 8
snhe
= unit digit = 2
Ex.8: Find the unit digit of the prod-
uct of all the even numbers
If the value of the power is
Power
3 + 4 + 6 + 8 = 21 Sol. The even number are
kgei
unit 1 2 3 4
So, units digit = 1 2, 4, 6, 8, 10, 12, ....... etc. digit or or or or
Ex.2: What is the unit digit of ? Now we know that if '0' is multi- 4n+1 4n+2 4n+3 4n+4
31 × 37 × 36 × 46 × 89 plied by any number it always
ERna

2 2 4 8 6
sol. 31 × 37 × 36 × 46 × 89 gives the last digit 0. so the re-
Unit digit = 1, 7, 6, 6, 9 quired unit digit will be 0. 3 3 9 7 1
multiply the unit digits = 1 × 7 Ex.9: Find the unit digit 4!
7 7 9 3 1
×6×6×9 Sol. 4! = 4 × 3 × 2 × 1 = 24
aBryn

 1×7=7 unit digit = 4 8 8 4 2 6


 7 × 6 = 42 Ex.10: Find the unit digit 5! here n  Natural No.
 2 × 6 = 12 Sol. 5! = 5 × 4 × 3 × 2 × 1 = 120 If those number which unit
 2 × 9 = 18 unit digit = 0 digit 2, 3, 7 and 8. 
* Factorial 5 and more than 5
Les

unit digit = 8 all unit digit have cyclicity 4


Ex.3: What is the unit digit of? express gives unit digit 0. Ex.18: Find the unit place 335
wa. th

31 × 33 × 37 × 39 × 43 Unit digit when 'N' is Raised to a power Sol. 335 = 332 × 33


unit digit of 0, 1, 5 and 6 has Break the power form of 4n
Sol. 31 × 33 × 37 × 39 × 43 (34)8 × 3³ = (......1) × (...7)
any power (odd or even) no
multiply the unit digits change unit place = 1 × 7 = 7
wwM

=1×3×7×9×3 Ex.19: Find the unit place (127)39


Ex.11: (3765)437
unit digit = 7 Sol. (127)39
unit digit = (5)437 = 5
unit place = (7)39
Ex.4: What is the unit digit of ? Ex.12: (6736)32567 = (7)36 × (7)3 = (74)9 × (7)3
91 × 93 × 95 × 96 × 97 × 98 unit digit = (6)32567 = 6 = (...1) × (....3)
Sol. multiply the unit digit Ex.13: (32541)325 unit place = 1 × 3 = 3
1×3×5×6×7×8=0 unit digit = (1)325 = 1 Ex.20: Find the unit place (678)562
Ex.5: Find the unit digit of 135 × 136 * 4 and 9  Sol. (678)562
× 170 4 9 unit digit = (8)562
Sol. The unit digits = 5, 6, 0 even odd even odd = (8)560 × (8)2
multiply the units digit power power power power = (84)140 × (8)2
=5×6×0 unit unit = (...6) × (...4)
= unit digit = 0 digit  6 4 digit 1 9 unit digit = 6 × 4 = 24 = 4

Rakesh Yadav Readers Publication Pvt. Ltd. 1

For More Visit : www.LearnEngineering.in


For More Visit : www.LearnEngineering.in

Ex.21: Find the unit place (327)640 Ex.25: Find the unit digit of (37)105
Sol. (327)640 Sol. (37)105 (5)215 + (4)326 + (6)213 + (7)112
unit digit = (7)640 unit digit = (7)105
640 is multiple of 4 = (7)104 × 71
then = (74)160 = (74)26 × 71
Same even Same (74)28
unit digit = (1)160 = 1 = (...1)26 × 7 power
Ex.22: Find the unit digit of (2137)753 unit digit = 1 × 7 = 7
=5 + 6 + 6 + 1
Sol. (2137)753 Ex.26: Find the unit place
unit digit = 18 = 8
unit digit = (7)753 (23)21 × (24)22 × (26)23 × (27)24 ×
Ex.28: Find the unit place of
= (7)752 × 71 (25)25
= (74)188 × 71 Sol. (23)21 × (24)22 × (26)23 × (27)24 × 1255 848

r
= (....1) × 7 (25)25 
311 1618
unit digit = 1 × 7 = 7 unit digit = (3)21 × (4)22 × (6)23 ×

Sni
Ex.23: Find the unit digit of (13)2003 (7)24 × (5)25 1255 848

gv.i
Sol. (13)2003 Break the power multiple of 4 Sol. 
311 1618
unit digit = (3)2003 320 × 31 × 422 × 623 × (74)6 × 525
= 32000 × 33 (23 )48
(3  4)55

ridna
= (34)500 × 33 = = + 4 18
even same same 311
(2 )
= (....1)500 × 27 power digit digit
= 1 × 27 = 27
=3×6×6×1×5 355  455 2144
unit digit = 7 = +

eeYa
Ex.24: Find the unit digit of (22)23 unit digit = 0 311 272
Sol. (22)23 Note:- unit digit = even × 5 = '0' = 344 × 455 + 272
unit digit = (2)23 Ex.27: Find the unit place unit digt = (.....1) × (......4) +6
= (2) × 2 = (24)5 × 8
20 3 (235)215 + (314)326 + (6736)213 +
geisnh = 4 + 6 = 10,
= (....6)5 × 8 (3167)112
unit digit unit digit = 0
unit digit = 6 × 8 = 48 = 8

EXERCISE
Enak

1. Find the unit digit of 584 × 389 10. The last digit of the number ob- 18. The digit in unit’s place of the
R

× 476 × 786 tained by multiplying the num- product (2153)167 is :


(a) 7 (b) 3 (c) 4 (d) 6 bers 81 × 82 × 83 × 84 × 85 × 86 (a) 1 (b) 3 (c) 7 (d) 9
2. Find the unit digit of
aryn

× 87 × 88 × 89 will be 19. The digit in unit’s place of the


641 × 673 × 677 × 679 × 681 (a) 0 (b) 9 (c) 7 (d) 2 product
(a) 9 (b) 3 (c) 6 (d) 7
Les B

11. Find the units digit of the (2464)1793 × (615)317 ×(131)491 is


3. Find the unit digit of expression 256251 + 36528 + 22853
(5627)153 × (671)230 (a) 0 (b) 2 (c) 3 (d) 5
(a) 4 (b) 3 (d) 6 (d) 5 20. What will be the unit digit in the
(a) 7 (b) 9 (c) 3 (d) 1
12. Find the units digit of the product of 7105 ?
4. Find the unit digit of
wa. th

(3625)333 × (4268)645 expression 55725 + 735810 + 22853


(a) 5 (b) 7 (c) 9 (d) 1
(a) 6 (b) 3 (c) 4 (d) 0 (a) 4 (b) 0 (c) 6 (d) 5
21. what is the number of unit place
5. Find the unit digit 13. Find the units digit of the expression
in (329)78
(3694)1793 × (615)317 × (841)941 111 + 122 + 133 + 144 + 155 + 166.
(a) 1 (b) 7 (c) 9 (d) 3
wwM

(a) 5 (b) 3 (c) 4 (d) 0 (a) 1 (b) 9 (d) 7 (d) 0


6. Find the unit digit of 22. unit digit of the number (22) 23
14. Find the last digit of the number
(795 – 358) is:
13 + 23 + 33 + 43 .... + 993.
(a) 7 (b) 3 (c) 4 (d) 0 (a) 4 (b) 6 (c) 6 (d) 8
(a) 0 (b) 1 (d) 2 (d) 5
7. Find the unit place of 15. Unit digit in (264) +(264)103 is:
102 23. The unit digit in the product
(17)1999 + (11)1999 – (7)1999 (122)173 is:
(a) 0 (b) 4 (c) 6 (d) 8
(a) 0 (b) 1 (c) 2 (d) 7 (a) 2 (b) 4 (c) 6 (d) 8
16. Un it d i g i t [(251 ) 98 +(2 1) 29 –
8. Find the unit digit of 24. The unit digit in the sum of (124)372
36 × 47 × 63 × 74 × 82 × 95 (106)100+(705)35–(16)4+259] is
(a) 1 (b) 4 (c) 5 (d) 6 + (124)373 is :
(a) 6 (b) 9 (c) 0 (d) 2
9. Find the unit digit of 111! (facto- 17. The unit digit in the expansion of (a) 5 (b) 4 (c) 2 (d) 0
2008
rial 111). (2137)754 is 25. The last digit of (1001) + (1002) is:
(a) 0 (b) 1 (c) 5 (d) 3 (a) 1 (b) 3 (c) 7 (d) 9 (a) 0 (b) 3 (c) 4 (d) 6

Rakesh Yadav Readers Publication Pvt. Ltd. 2

For More Visit : www.LearnEngineering.in


For More Visit : www.LearnEngineering.in

26. Find the unit digit in the product: 36. Find the last digit of (1 0!) 5 + (10 0 !) 4 + (10 00 !) 3 +
(4387)245 × (621)72. 222888 + 888222 (10000!)2 + (100000!)
(a) 1 (b) 2 (c) 5 (d) 7 (a) 2 (b) 6 (c) 0 (d) 8 (a) 45939 (b) 00929
27. The unit digit of the expression 37. Find the last digit of 3232 32 (c) 20929
256251 + 36528 + 7354 is (d) Can't determined
(a) 6 (b) 5 (c) 4 (d) 0 (a) 4 (b) 8 (c) 6 (d) 2
38. Find the last digit of the expres- 45. The unit digit of the following
28. The unit’s digit in the product 771 expression (1!)99 + (2!)98 + (3!)97 +
× 663 × 365 is sion:
(4!)96 +...... (99!)1 is:
(a) 1 (b) 2 (c) 3 (d) 4 12 + 22 + 32 + 42 + ...... + 1002.
(a) 0 (b) 4 (c) 6 (d) 8 (a) 1 (b) 3 (c) 7 (d) 6
29. The last digit of 340 is
(a) 1 (b) 3 (c) 7 (d) 9 39. Find the unit digit of 1 + 22 + 33
1 46. The unit digit of (12345k)72 is 6.
30. The digit in unit’s place of the + ......1010. The value of k is:

ir
number (1570) 2 + (1571) 2 + (a) 9 (b) 7 (c) 0 (d) N.O.T. (a) 8 (b) 6 (c) 2

v.iSn
(1572)2 + (1573)2 is : 40. Find the unit digit of (d) all of these
(a) 4 (b) 1 (c) 2 (d) 3 1324 × 6857 + 2413 × 5768 + 1234 + 47. The unit digit of the expression
31. The unit digit in 3 × 38 × 537 × 5678. (1!)1! + (2!)2! + (3!)3! + ..... +(100!)100!
1256 is
(a) 4 (b) 7 (c) 0 (d) 8

dnag
(a) 4 (b) 2 (c) 6 (d) 8 (a) 0 (b) 1 (c) 2 (d) 7
41. The unit digit of the expression 48. The last digit of the expressioin
32. The unit digit in the product
(2467)153 × (341)72 is 1 2  3  4  5  6  7  8  9 10 4 × 92 × 43 × 94 × 45 × 96 × ...... ×
(a) 1 (b) 3 (c) 7 (d) 9 100 499 × 9100 is :

eYari
33. The unit digit in the product (a) 4 (b) 6 (c) 9 (d) 1
(a) 7 (b) 9 (c) 8 (d) N.O.T.
(6732)170 × (6733)172 × (6734)174 × 49. The last digit of the expression
42. Find the unit digit of the expres-
(6736)176 4 + 92 + 43 + 94 + 45 + 96 + ...... +
sion 8889235! + 2229235! + 6662359!
(a) 1 (b) 3 (c) 4 (d) 5 499 + 9100 is:
+ 9999999!.
34. Find the unit digit of the prod-
uct of all the prime number be-
tween 1 and 99999
(a) 9 (b) 7 (c) 0 (d) N.O.T.
(a) 5 (b) 9
(d) None of these
snhe
(c) 3

43. The last digit of the following ex-


(a) 0 (b) 3
(d) None of these
(c) 5

50. The unit digit of 234 × 345 × 456 ×


kgei
35. Find the unit digit of the prod- pression is: (1!)1 + (2!)2 + (3!)3
7
uct of all the elements of the set + (4!) + .... + (10!)10
4
56 × 678 × 789 is:
which consists all the prime (a) 4 (b) 5 (c) 6 (d) 7 (a) 0 (b) 5
ERna

numbers greater than 2 but less 44. The last 5 digits of the following (c) Can't be determined
than 222. expression will be
(a) 4 (b) 5 (c) 0 (d) N.O.T. (d) None of these
(1!)5 + (2!)4 + (3!)3 + (4!)2 + (5!)1 +
aBryn

ANSWER KEY
1. (d) 6. (c) 11. (b) 16. (b) 21. (a) 26. (d) 31. (d) 36. (c) 41. (c) 46. (d)
2. (a) 7. (b) 12. (c) 17. (d) 22. (d) 27. (d) 32. (c) 37. (c) 42. (b) 47. (d)
3. (a) 8. (a) 13. (b) 18. (c) 23. (a) 28. (d) 33. (c) 38. (a) 43. (d) 48. (b)
Les

4. (d) 9. (a) 14. (a) 19. (a) 24. (d) 29. (a) 34. (c) 39. (b) 44. (b) 49. (a)
5. (d) 10. (a) 15. (a) 20. (b) 25. (b) 30. (a) 35. (b) 40. (a) 45. (c) 50. (a)
wa. th

SOLUTION
wwM

1. (d) 584 × 389 × 476 × 786 unit digit = 9 = 5 × (84)161 × 81


unit digit 4, 9, 6, 6 3. (a)(5627)153 × (671)230 = 5 × (6)161 × 8
Multiplying the unit digit unit digit (7)153 × (1)230 unit digit = 5 × 6 × 8 = 240 = 0
=4×9×6×6 = (7)152 × 71 × 1 5. (d)(3694)1793× (615)317 × (841)941
unit digit = 6 = (74)38 × 7 × 1 unit digit (4)1793 × (5)317 × (1)941
2. (a) 641 × 673 × 677 × 679 × 681 = (....1)38 × 7 4odd power = 4
unit digit = 1, 3, 7, 9, 1 unit digit = 1 × 7 = 7
5n = 5
Multiply the unit digit 4. (d) (3625)333 × (4268)645
4 × 5 × 1 = 20
=1×3×7×9×1 unit digit (5)333 × (8)645
Hence, unit digit = 0
= 21 × 9 = 189 = 5 × (8)644 × 81

Rakesh Yadav Readers Publication Pvt. Ltd. 3

For More Visit : www.LearnEngineering.in


For More Visit : www.LearnEngineering.in

6. (c)795 – 358 13. (b) 111 + 122 + 133 + 144 + 155 + 166 17. (d) (2137)754
= 792 × 73 – 356 × 32 unit digit = (1)1 + (2)2 + (3)3 + = (7)754 will give unit digit
= (74)23 × 343 – (34)14 × 9 (4)4 + (5)5 + (6)6 unit digit
1 754
= (.....1)23 × 3 – (.....1)14 × 9 Sum of unit digit = 1 + 4 + 7 + 6 7 = 7 7 754 divide by 4=
2 9 4
unit digit = (.....3) – (......9) 7 = 49
+ 5 + 6 = 29 3 4 So, remainder is 2
7 = 343 3
= 13 – 9 = 4 unit digit = 9 4 2
7= 9
7 = 2401 1
7. (b)(17)1999 + (11)1999 – (7)1999 5
7 = 16807 7
14. (a) 13 + 23 + 33 + 43 .... + 993
unit digit = (7)1999 + (1)1999 – (7)1999 & will
Sum of cube of natural no. repeat
 (7)1999 – (7)1999 gives = 0
Then, unit digit = 1 Unit Place = 9
 n (n  1) 2  99(99  1) 2

r

=  
 =   18. (c) (2153)167
8. (a)Unit digit = 36 × 47 × 63 × 74 ×  
 2  2

Sni
  unit digit = 3167
82 × 95
The unit digit of 36 = 34 × 32 = 9 unit digit

gv.i
2
 99 100  31  3  3
The unit digit of 47 = 4 =  
 = (99 × 50)2
 2  32  9  9
The unit digit of 63 = 6
33  27  7

ridna
The unit digit of 74 = 1 = (4850)2
The unit digit of 82 = 4 34  81  1
Unit digit = 0
The unit digit of 95 = 94 × 91 = 9 This cycle will continue
15. (a) (264)102 + (264)103

eeYa
multiply the unit digits = 9 × 4  divide the power of 3 by 4
×6×1×4×9 unit digit
167
41  4 4  remainder is 3
unit digit = 6  4
9. (a)111! = 1 × 2 × 3 × 4 × 5 × .... 42  16  6 33  7
geisnh
× 100 × 111 Unit digit = 7
43  64  4
Since there is product of 5 and 19. (a) (2464)1793 × (615)317 × (131)491
2 hence it will give zero as the Rule:When 4 has odd power, then unit 41  4  4
unit digit. digit is: 4
42  16  6
Enak

Hence the unit digit of 111! is 0 (zero). When 4 has even power, then unit 43  64  4
10. (a) 81 × 82 × 83 × 84 × 85 × 86 digit is 6 So odd power of 4 will have 4 as unit
× 87 × 88 × 89 (264)102 +(264)103 digit and even power will have 6 as
R

Unit digits = 1, 2, 3, 4, ........,9 unit digit 5 and 1 have same unit dig-
 
Product of unit digits its respectively
102
(4) + (4)103
aryn

= 1 × 2 × 3 ×............×9 491
1793 317 × (13 1)
(2 4 6 4 ) ×(6 1 5 )
Because 5 multiply any even no.  
Les B

Then (even power) (odd power) odd power


we gets unit digit = 0 unit digit 6 + 4 = 10  0
11. (b) 256251 + 36528 + 22853 unit digit 4 ×5 ×1 = 20
Alternate :
wa. th

6251 528 853


unit digit = (5) + (6) + (2)  (264)102 + (264)103  20  0 unit digit
20. (b) 7105
852 1  (264)102 (1 + 264)
unit digit = (....5)+ (....6)+ (2)×2  71  7  7
 (264)102 × 265 72  49  9
= (....5) + (....6) + (24)213 × 2 
wwM

Multiplication of 5 & 2 = 0  73  343  3


= 5 + 6 + (6)213 × 2
Sum of unit digit= 5 + 6 + 6 × 2 Hence, unit digit is 0.  74  2401  1
16 (b) [(251)98 + (21)29 – (106)100 + Divide power of 7 by 4
= 5 + 6 + 12 = 23
Hence, unit digit = 3 (705)35 – (16)4 + (259)] 105
 remainder = 1  71 is left
12. (c) 55725 + 735810 + 22853 4
unit place of 1,5 and 6 will
unit digit = (5)725 + (3)5810 + (2)853 remain same unit digit = 7
= (...5) + (34)1452 × 32 + (24)213 × 21 21. (a) (329)78
= [(1)98+(1)29–(6)100+(5)35–(6)4+9]
= 5 + (1)1452 × 9 + (16)213 × 21  If power of 9 is odd, then unit digit
= [1 + 1 – 6 + 5 – 6 + 9]
Sum of unit digit = 5 + 1 × 9 + 6 number be 9. If power is even then
× 2 = 5 + 9 + 12 = 26  16 – 12 = 4 unit digit number be 1.
unit digit = 6 Hence, unit digit = 4 Hence, unit digit = 1

Rakesh Yadav Readers Publication Pvt. Ltd. 4

For More Visit : www.LearnEngineering.in


For More Visit : www.LearnEngineering.in

22. (d) (22)23 27. (d) 5 always gives unit digit 5


Result unit digit and 6 always gives unit digit 6 Result Unit digit
1
2 2 2 unit digit 7
1
= 7 7
2
2 4 4 Cycle 2
= 49 9
1
  7
2
3
8 8 3 3 3
completes 3
343
2
4
16 6
2 7 = 3
3  9  9
5 Cycle 4
2 32 2 3
 7 = 2401 1
3 27  7
So divide power of 22 by 4 4
3  81  1 Hence, unit digit = 7
23
= remainder 3 33. (c) (6732)170 × (6733)172 × (6734)174
4 25
6251
+ 36
528
+ 72
54
× (6736)176
2³ = 8
unit digit = (2)170 × (3)172 × (4)174 × (6)176
unit digit = 8 56251 + 6528 + 354 = (24)42 × 22 × (34)43 × (4)174 × (6)176

ir
23. (a) (122)173 54/ = r = 2 = (...6) × 4 × (...1) × (...6) × (...6)

v.iSn
4
Unit digit unit digit 5 + 6 + 32 Multiplication of unit digit
1
2  2  2 = 5 + 6 + 9 = 20 = 0 = 6 × 4 × 1 × 6 × 6 = 864
2
2  4  4 Hence, unit digit = 0 Hence, unit digit = 4
3 Cycle
2  8  8 28. (d) 771 × 663 × 365 34. (c) The set of prime number S

dnag
4
2  16  6
= {2, 3, 5, 7, 11, 13, .....}
  
5
2  32  2 unit place 73 63 31 Since there is one 5 and one 2 which
2173 = 24×43+1 = 24×43×2 = 1643×2    gives 10 after multiplying mutually,

eYari
= 643×2 = 6×2 = 12 unit digit  3 × 6 × 3 = 54 it means the unit digit will be zero.
unit digit = 2  4 Hence, unit digit = 0
24. (d) (124)3 72 (124)
37 3
29. (a) 340 : 35. (b) The set of required prime
number = The set of reqired
  40

4
3 72

When 4 has odd power then unit


digit is 4 when 4 has even power
4
37 3

1
snhe
Divide = 4  remainder = 0

3 3
Unit digit
prime number
= {3, 5, 7, 11, .....}
Since there is no any even number is
the set so when 5 will multiply with
3
kgei
then unit digit is 6
1
2 Cycle any odd number, it will always give 5
4  4  4 3  9  9
3
as the last digit.
2 3  27 7
4  16  6 Hence the unit digit will be 5.
ERna

4
3 3  81 1 36. (c) The last d ig it of the
4  64  4
4 Hence, unit digit of 340 of com- expression will be same as the
4  256  6
372 373 pleting all cycle = 1 last digit of 2888 + 8222.
4 4
  30. (a) Now the last digit of 2888 is 6 and
aBryn

6 + 4 = 10 2
(1570) + (1571) + (1572) +
2 2
(1573)
2 the last digit of the 8222 is 4.
last (unit) digit = 0      6 + 4 = 10.
25. (b) (1001)2008 + 1002 unit digit 2
0 + 1
2
+ 2
2
+ 3
2
Hence, unit digit = 0
    
Unit digit  12008 + 1002 0 + 1 + 4 + 9 = 14 37. (c) Find the last digit of 23232
unit digit = 4
Les

Unit digit will be 1 in case of 1 32


respective of power 31. (d) 3 But 232 = 232 × 32 × 32 ....... × 32 times
wa. th

×38 × 53 7 × 125 6
 1+ 1002 = 1003  32

unit digit (last digit) = 3 × 24 28 48 232 = 24 × 8 × (32 × 32 ....... × 31 times)


× ×
26. (d) unit place
Note:- Always multiply only unit digit  32
232 = 2
4n
1
7  7  7 of first no. to second and product's where n = 8 × (32 × 32 ....... × 32 times)
wwM

7
2
 49  9 unit digit no. with 3rd no. Again prod- Again 24n = (16)n  unit digit is 6, for
Cycle uct of last's unit digit to fourth and so
7
3

 3 343 every n  N
4
on. Hence, the required unit digit = 6
7  2401  1 Hence, unit digit = 8
(4387)245 × (621)72 38. (a) Sum of square natural
 153 72
n n  12n  1
(7)245 × (1)72 32. (c) (2467) × (341)
number =
 6
(7)4×61+1 × 1 (7) 153 × (1)72 Here, n = 100

(7)4×61×7 × 1 100 101 201
 [153/4= remainder = 1] = = 338350
6
(1)61×7 × 1
unit digit = 7  71 × 1 = 7 Then,Unit digit = 0

Rakesh Yadav Readers Publication Pvt. Ltd. 5

For More Visit : www.LearnEngineering.in


For More Visit : www.LearnEngineering.in

39. (b) Find the unit digit of 11 + 22 So, the unit digit of 8889235! is equal 46. (d) unit digit (12345k)72 = 6
+ 33 + ......1010. to the unit digit of 89235! if we put the value of k = 2, 6, 8
The unit digit of 11 = 1 Now, the unit digit of 89235! equal to the Then we get unit digit = 6
The unit digit of 22 = 4 unit of 84 (since 9235! is divisible by 47. (d) (1!)1! + (2!)2! + (3!)3! + .....
4), which is 6. +(100!)100!
The unit digit of 33 = 7
unit digit of (888)9235! = (8)4n = 6 unit digit (1!)1! = 11 = 1
The unit digit of 44 = 6
unit digit of (222)9235! = (2)4n = 6 unit digit (2!)2! = (2)2 = 4
The unit digit of 55 = 5 unit digit of (666)2359! = (6)anypower unit digit (3!)3! = (6)6 = 6
The unit digit of 66 = 6 =6 unit digit (4!)4! = (4)24 = 6
The unit digit of 77 = 3 unit digit of (999)9999! = (9)even power unit digit (5!)5! = (0)120 = 0
The unit digit of 88 = 6 =1 ......................................................

r
The unit digit of 99 = 9 Thus the unit digit of the ex- ......................................................
The unit digit of 1010 = 0 pression is 9. (  6 + 6 + 6 + 1 = 19)

Sni
unit digit (100!)100!
Thus the unit digit of the given 43. (d) The unit digit of the given ex-  = (0)1×2×3×.........×100 = 0

gv.i
expression will be 7. pression will be equal to the unit
digit of the sum of the unit dig- Sum of unit digit = 1 + 4 + 6 + 6
( 1 + 4 + 7 + 6 + 5 + 6 + 3 + 6
its of every term of the expres- + 0 + 0 + 0 = 17
+ 9 = 47)
sion. unit digit = 7
(a) The unit of 324 is 1

ridna
40.
Now, The unit digit of (1!)2 = 1 48. (b) 4 × 92 × 43 × 94 × 45 × 96 ×
The unit digit of 857 is 8 ...... × 499 × 9100
The unit digit of (2!)2 = 4
The unit digit of 413 is 4 unit digit 41 = 4
The unit digit of (3!)3 = 6
The unit digit of 768 is 1 unit digit 92 = 1

eeYa
The unit digit of (4!)4 = 6
So the resultant value of the unit digit 43 = 4
unit digits The unit digit of (5!)5 = 0 unit digit 94 = 1
=1×8+4×1+4+8 The unit digit of (6!)6 = 0 unit digit 45 = 4
= 8 + 4 + 4 + 8 = 24 Thus the last digit of the (7!)7, .......................................................
geisnh
(8!)8, (9!)9, (10!)10 will be zero. .......................................................
Thus the unit digit of the whole
So, the unit digit of the given unit digit 499 = 4
expression is 4.
expression = 7 unit digit 9100 = 1
41. (c) Since in the numerator of the Then multiply the unit digit
(  1+4+6 + 6 + 0 + 0 + 0 + 0 + 0 = 17)
product of the expression there 4 × 1 × 4 × 1 × 4 × 1 ...... 4 × 1
Enak

will be 2 zeros at the and thase 44. (b) The last digit of (1!)5 = 1
Pair of 4 × 1 (4) is equal 50
two zeros will be cancelled by 2 The last digit of (2!)4 = 16
we can say this expression = 450
zeros of the denominator. Hence The last digit of (3!)3 = 216 Then, unit place = 6
The last digit of (4!)2 = 576
R

finally we get a non-zero unit 49. (a) 4 + 92 + 43 + 94 + 45 + 96 +


digit in the expression. The last digit of (10!)5 = 00000 ...... + 499 + 9100
The last digit of (100!)4 = 00000 unit digit 41 = 4
aryn

Now, 1 2  3  4  5  6  7  8  9 10 (1000!)3 = 00000 unit digit 92 = 1


100 unit digit 43 = 4
(10000!)2 = 00000
Les B

unit digit 94 = 1
1 21  31  22  51  21  (100000!)1 = 00000
unit digit 45 = 4
1 1 3 2 1 1 Thus the last 5 digits of the .......................................................
= 3  7 2 3  2 5 given expression = 00929
5²  2² .......................................................
[  1 + 16 + 216 + 576 + 120 + unit digit 499 = 4
wa. th

1 28  34  52  71 00000 + 00000 + 00000 + unit digit 9100 = 1


= 00000 + 00000 + = 00929] Then,
52  22 Sum of unit place
45. (c) (1!)99 + (2!)98 + (3!)97 + (4!)96
= 1 × 2 × 34 × 7
6
+...... (99!)1 4 + 1 + 4 + 1 + 4+ 1.......4 + 1
wwM

Therefore, the unit digit of the unti digit (1!)99 = (1!)99 = 1 Pair of 4 + 1 (5) is equal to 50
given expression will be same as We can say this expression
unit digit (2!)98 = 1 × 2 = (2)98 = 4
that of 1 × 26 × 34 × 7. = 50 × 5 = 250
unit digit (3!)97 = 1 × 2 × 3 = (6)97 = 6 unit digit = 0
Now, The unit digit of 1 × 2 6 × 34 × 7
unit digit=(4!)96 =1 × 2 × 3 × 4=(4)96 = 6 4 5 7 8
is 8. 50. (a) 23 × 34 × 456 × 56 × 67 × 789
unit digit=(5!)95=1×2×3×4× 5 = (0)95 = 0
(  the product of unit digits of We know unit digit of
......................................................
1, 26, 34, 7 si 1 × 4 × 1 × 7 = 28) 4 81
...................................................... 23 = 2 = 2
10! same unit digit (99!) 1 = (1×2....99) unit digit of 56 = 5
7

Hence, the unit digit of is 8.


100 = (0)1 = 0 Then we know that when even
42. (b) First of all we find the unit Then, Sum of unit digit = 1 + 4 number is multplied by 5 then
digit individually of all the four + 6 + 6 + 0 + 0 +.... 0 = 17 we get unit place = '0'
terms, unit digit = 7 So, last digit = 0

Rakesh Yadav Readers Publication Pvt. Ltd. 6

For More Visit : www.LearnEngineering.in


For More Visit : www.LearnEngineering.in

CHAPTER

02
NUMBER OF ZEROES
Number of zeroes in an Expression Sol. 1 × 3 × 5 × 7 × 9 ........ 97 × 99 *Alternate method is easier

ir
zero:- zero will be formed by 2 and 5 In this series the number of zero than first.
Ex. 10 = 2 × 5 and the end of the product is "0".

v.iSn
Ex.6 Find the largest power of 2
100 = 22 × 52 Because there is no even number that can contained in:-
1000 = 23 × 53 present in this series while it is 1 × 2 × 3 × 4 ......... 22 ?
 We can say that for 'n' number necessary to be 2 and 5 for the Sol. 1 × 2 × 3 × 4 ......... 22
of zeroes at end of the product. Zero

dnag
2 22
We need exactly 'n' The highest power of k that can
2 11
combinations of 5 and 2 exactly divided n! we divide n by k,
2 5
Ex.1 Find the number of zeroes at n by k2, n by k³ and so on till we get 2 2 19

eYrai
the end of the product:- n  2 1
 x  equal to 1 an then add up as. 0
5 × 7 × 9 × 2 × 11 k 
 Number of 2's = 11 + 5 + 2 + 1= 19
Sol. 5 × 7 × 9 × 2 × 11
n   n   n   n  n  Hence, there are 19 times 2
In this product we see   +   +   +  4  +...+  x  involved as a factor in 22!
Number of 2's = 1
Number of 5's = 1
Number of pair 2's and 5's = 1
k   k²  
       k³
snhe
  k  k 

Ex.4 Find the largest power of 5
contained in 124!
Ex.7 Find the largest power of 5 that
can contained in
1 × 2 × 3 × 4 ......... 41 × 42
 Number of zero = 1 Sol. 5 42
kgei
124  124 
Ex.2 Find the number of zeroes at Sol.     2  =24 + 4 = 28 5 8
 5  
  5  9
the end of the product:- 12 × 27 5 1
× 63 × 113 × 1250 × 24 × 650 [We cannot do it further since 0
ERna

Sol. 12 × 27 × 63 × 113 × 1250 × 24 124 is not divisible by 53] Hence, there are 9 times 5 involved
× 650 He nc e, the re are 2 8 time s 5 as a factor in 42!
alternate as a factor in 124! Ex.8 Find the largest power of 7
Break the numbers form of 2
Alternate:- that can exactly divide 777!
and 5 multiple
aBryn

In this series 27, 63 & 113 Divide successive quotients till Sol. 7 777
are not multiple of 2 & 5. we get 0 as the last quotient 7 111
7 15 128 (add up all the
 The multiple of 2 & 5 are 5 124 quotients)
7 2
12, 1250, 24 & 650 5 24 28 (add up all 0
 12 = 2 × 2 × 3 = 22 × 3 the quotients) Thus the highest power of 7 is 128
Les

4
 1250 = 2 × 5 × 5 × 5 × 5 = 21× 54 by which 777! can be divided.
Ex.5 Find the largest power of 3
wa.th

 24 = 2 × 2 × 2 × 3 = 23 × 3 Ex.9 Find the number of zeroes at


that can divide 270!
650 = 2 × 5 × 5 × 13 = 21 × 52 × 13 the end of the product 10!
 270  270  270  270  Sol. 10! = 1 × 2 × 3 × 4 × 5 × 6 × 7 ×
22 × 3 × 27 × 63 × 113 × 21× 54 × Sol.   2  3  4 
 3  
  3  
 3  
 3  8 × 9 × 10
23 × 3× 21 × 52 × 13
wwM

Number of 2's = 7 270 


  5  = 90 + 30 + 10 + 3 + 1 = 134 2×5
Number of 5's = 6  3 

* In this type expression it is
 Number of pair 2's and 5's = 6 Hence, there are 134 times
3 involved as a factor in 270! clear that in any factorial
there are 7 two's and 6 fives. value, the number of 5's will
Alternate:-
Hence we shall be able to Divide successive quotients till we alway s be les ser then the
form only 6 pairs of 2 and 5, get 0 as the last quotient numb er of 2' s In this
Hence there will be 6 zeroes 3 270 condition. all we need to do is
at the end of the product of 3 90 to count the number of 5's
numbers. 3 30 Number of 5's = 2
134 (add up all the Number of 2's = 5
Ex.3 Find the number of zeroes at 3 10 quotients)
3 3 But pair of 2's and 5's are =2
the end of the product:- Then,
3 1
1 × 3 × 5 × 7 × 9 ........ 97 × 99 0 Number of zero's = 2

Rakesh Yadav Readers Publication Pvt. Ltd. 7

For More Visit : www.LearnEngineering.in


For More Visit : www.LearnEngineering.in

Alternate:- Ex.14 Find the number of zeroes at 57 = (1× 5)7 = 17 × 57 =7


Highest power of 5's in 10! the end of the product 1012 = (2 × 5)12 = 212 × 512 = 12
512 × 513 ............1120 1517 = (3 × 5)17 = 317 × 517 = 17
10
=2 Sol. 1 × 2 × 3 ........ 511 × 512 × 2022 = (4 × 5)22 = 422 × 522 = 22
5 513 .......1120 – 1 × 2 × 3 ........ 2527 = 27 27
(5 × 5) = 5 × 5 27
= 54
Number of 5's = 2 511 Number of 5's power
Then number of zeroes = 2 = 7 + 12 + 17 + 22 + 54 = 112
Ex.10 Find the number of zeroes at 5 1120
5 224 Then number of zeroes at the
the end of the product 100! end of product = 112
Sol. 5 100 5 44 277
5 8 Ex.18 Find the number of zeroes at
5 20 24 (add up all the 5 1 the end of the product

r
5 4 quotients) 0 11 × 22 × 33 × 44 ........ 100100
0

i
.iSn
Number of 5's = 20 + 4 = 24 5 511 Sol. Count the power of 5's
5 102 55 = 5
then number of zeroes = 24
5 20 126 1010 = 10
Ex.11 Find the number of zeroes at
5 4 1515 = 15
the end of the product 1000!.

agv
0 . .
Sol. 1 × 2 × 3 × ........ 999 × 1000 Number of zeroes = 277 - 126 . .
5 1000 = 151

ridn
. .
5 200 Ex.15 Find the number of zeroes at . .
5 40 249 the end of the product 100100 = 100
5 8

eeYa
5 1 15 × 25 × 35 ........ 325 5 + 10 + 15 ....... + 100
0 Sol. In this ty pe ev ery se cond it is an a.p.series
The highest power of Number terms has power of 2's. It we use a.p. formula
5's = 200 + 40 + 8 + 1= 249 means power of 2's more than
l–a
then number of zeroes at the
geisnh
that of 5 So count the power number of term = +1
end of the product = 249 of 5's d
Ex.12 Find the number of zeroes at power of 5's = total power of 5's l = last term of a.p.
the end of the product 55 = (1× 5)5 = 15 × 55 =5 a = first term of a.p.
1 × 3 × 5 × 7 ......... 73 × 1024 105 = (2 × 5)5 = 25 × 55 =5 d = common difference
Enak

Sol. 1 × 3 × 5 × 7 ......... 73 × 1024 155 = (3 × 5)5 = 35 × 55 =5 100  5


Number of 5's from 1 to 73 205 = (4 × 5)5 = 45 × 55 =5 number of term = 1
5
5 73 255 = (5 × 5)5 = 55 × 55 = 10
R

= 20
5 14 305 = (6 × 5)5 = 65 × 55 =5
16 (add up all the sum of nth term of a.p.
5 2 quotients) Number of 5's power = 35
aryn

0 then number of zeroes at the end n


Total number of 5's = 14 + 2 = 16 = 2a  n – 1 d 
of the product = 35 2
we know that
Les B

Ex.16 Find the number of zeroes at


1024 = 210 20 
the end of the product = 2  5  20  1 5 
number of 2 = 10 2 
11 × 22 × 33 × 44 ........ 2828
number of pairs (2 and 5) = 10 Sol. count the numbe of 5's  1010  19  5
then number of zeroes 10 power of 5's = total power of 5's
wa.th

Ex.13 Find the number of zeroes at 55 = (1× 5)5 = 15 × 55 =5  10 105 = 1050


the end of the product 1010 = (2 × 5)10 = 210 × 510 = 10 but 2525 = (5 × 5)25 = 525 × 525
12 × 13 × 14 ......... 84 1515 = (3 × 5)15 = 315 × 515 = 15 5050 = (2 ×5 × 5)50 =250 × 550 × 550
Sol. In this types expression first 2020 = (4 × 5)20 = 420 × 520 = 20 7575 = (3 × 5 × 5)75 = 375 × 575 × 575
wwM

for us complete the sereis 2525 = (5 × 5)25 = 525 × 525 = 50 100100 = (4 × 5 × 5)100 = 4100 × 5100 × 5100
1 × 2 × 3 ........ 11 × 12 × 13 Number of 5's power then number of 5's power
....... 84 - 1 × 2 × 3...... 11 = 25 + 50 + 75 + 100
= 5 + 10 + 15 + 20 + 50
Number of zero (1 to 84) = 250
= 100
5 84 Then number of zeroes at the then number of total zeroes
5 16 19 end of product = 100 at the end of product
5 3 = 1050 + 250 = 1300
Ex.17 Find the number of zeroes at
0 Ex.19 Find the number of zeroes at
Number of zero (1 to 11) the end of the product
a = 13 , b = 24 , c = 35, ....., z = 2628 the end of the product
5 11
a × b × c × d ....... × z 10 × 20 × 30 ........ 80
5 2 2
0 Sol. Count the number of 5's Sol. 101 × 1 × 101 × 2 × 101 × 3....101
Number of zeroes = 19 – 2 = 17 power of 5's = total power of 5's × 8 = 108 [1 × 2 × 3 ....... 8]

Rakesh Yadav Readers Publication Pvt. Ltd. 8

For More Visit : www.LearnEngineering.in


For More Visit : www.LearnEngineering.in

from 1 to 8, number of 0's = 1 Sol. 101 × 1 × 101 × 2 × 101 × 3....101 number of 0's = 20 + 4 = 24
 Only one pair (2 & 5) × 100 and 10100 , here number of zero
then total number of 0's = 10100 [1×2×3×......100] = 100
=1+8=9 from 1 to 100 number of 0's
total number of 0's
Ex.20 Find the number of zeroes at 5 100
the end of the product 5 20 = 24 + 100 = 124
24 (add up all the
10 × 20 × 30 ........ 1000 5 4 quotients) then number of zeroes = 124
0

EXERCISE

ir
v.iSn
1. Find the number of zeroes at 15. Find the number of zeroes at 26. Find the number of zeroes at
the end of the product 47! the end of the p roduct the end of the product
(a) 8 (b) 9 (c) 10 (d) 11 21×22×23.......59×60 3200×1000×40000×32000×15000
2. Find the number of zeroes at (a) 14 (b) 4 (c) 10 (d) 12 (a) 15 (b) 2 (c) 14 (d) 16
16. Find the number of zeroes at

dnag
the end of the product 125! 27. Find the number of zeroes at
(a) 25 (b) 30 (c) 31 (d) 28 the end of the product the end of the product
3. Find the number of zeroes at 35 × 36 × 37 × ......89 × 90 20×40×7600×600×300×1000
the end of the product 378! (a) 21 (b) 7 (c) 14 (d) 20 (a) 11 (b) 10 (c) 2 (d) 3

eYrai
(a) 93 (b) 90 (c) 75 (d) 81 17. Find the number of zeroes at 28. Find the number of zeroes at
4. Find the number of zeroes at the end of the product the end of the product 100! +
the end of the product 680! 41 × 42 ..............109 × 110 200!
(a) 163 (b) 169 (c) 170 (d)165 (a) 26 (b) 9 (c) 17 (d) 25 (a) 24 (b) 25 (c) 49
(d) N.O.T
5.

6.
Find the number of zeroes at
the end of the product 1000!
(a) 200 (b) 249 (c) 248 (d) 250
Find the number of zeroes at
snhe
18. Find the number of zeroes at
the end of the product
140!×5×15×22×11×44×135
(a) 34 (b) 35 (c) 36 (d) 37
29. Find the number of zeroes at
the end of the product
11 × 22 × 33 × 44 .......1010
kgei
the end of the product 500! 19. Find the number of zeroes at (a) 10 (b) 15 (c) 5
the end of the product (d) N.O.T
(a) 100 (b) 124 (c) 120 (d) 125
25! × 32! × 45! 30. Find the number of zeroes at
7. Find the number of zeroes at
the end of the product
ERna

the end of the product 1132! (a) 10 (b) 23 (c) 22 (d) 7


20. Find the number of zeroes at 100! × 200!
(a) 280 (b) 271 (c) 281 (d) 272 (a) 49 (b) 24 (c) 73
8. Find the number of zeroes at the end of the product
1140! × 358! × 171! (d) N.O.T
the end of the product 1098! 31. Find the number of zeroes at
aBryn

(a) 280 (b) 270 (c) 271 (d) 262 (a) 282 (b) 325 (c) 411 (d) 370
21. Find the number of zeroes at the end of the p roduct
9. Find the number of zeroes at 5×10×15×20×25×30×35×40×45×50
the end of the product 2346! the end of the product 1 ×22 ×
33 × 44 × 55 ........ 4949 (a) 8 (b) 12 (c) 10 (d) 14
(a) 580 (b) 583 (c) 575 (d) 580 32. Find the number of zeroes at
10. Find the number of zeroes at (a) 225 (b) 250 (c) 240 (d) 245
the end of the product
22. Find the number of zeroes at
Les

the end of the product 2700! 2 × 4 × 6 × 8 × 10 ........ 200


(a) 673 (b) 670 (c) 669 (d) 675 the end of the product
wa.th

(a) 49 (b) 24 (c) 25 (d) 50


11. Find the number of zeroes at 1001× 992 × 983 × 974 .....1100
33. Find the number of zeroes at
the end of the product 10 × 15 (a) 970 (b) 1124(c) 875 (d) 975
the end of the product
× 44 × 28 × 70 23. Find the number of zeroes at
1 × 3 × 5 × 7 ........99 × 64
(a) 2 (b) 3 (c) 4 (d) 5 the end of the product
(a) 23 (b) 6 (c) 0 (d) 5
wwM

12. Find the number of zeroes at 11! × 22! × 33! × 44! .......1010!
34. Find The No. zero at the end of
the end of the p roduct (a) 51 (b) 10 (c) 5! + 10! the product of 2222 × 5555
12×5×15×24×13×30×75 (d) N.O.T (a) 222 (b) 555 (c) 777 (d) 333
(a) 4 (b) 5 (c) 2 (d) 3 24. Find the number of zeroes at 35. Find the number of zeroes at
13. Find the number of zeroes at the end of the product the end of the product
the end of the p roduct 22 × 54 × 42 × 108 × 610 × 1512 × 814 × 10 + 1 00 + 1 00 0 +
2×4×6×.......48×50 2016 × 1018 × 2520 .....100000000
(a) 6 (b) 12 (c) 7 (d) 5 (a) 98 (b) 90 (c) 94 (d) 100 (a) 8 (b) 28 (c) 0 (d) 1
14. Find the number of zeroes at 25. Find the number of zeroes at the 36. Find the number of zeroes at
the end of the p roduct end of the product the end of the product
1×3×5×7×9×11........99×101 3200+1000+40000+32000+15000 101 × 102 × 103 × 104 ........1010
(a) 24 (b) 5 (c) 2 (d) 0 (a) 15 (b) 13 (c) 2 (d) 3 (a) 10 (b) 55 (c) 50 (d) 45

Rakesh Yadav Readers Publication Pvt. Ltd. 9

For More Visit : www.LearnEngineering.in


For More Visit : www.LearnEngineering.in

37. Find the number of zeroes at 38. Find the number of zeroes at (8123 - 8122- 8121) (3223 - 3222 - 3221)
the end of the product the end of the product (a) 1 (b) 2 (c) 0 (d) 3
21×52×23×54×25×56×27×58×29×510 (3123 - 3122 - 3121) (2121 - 2120 -2119) 40 Find the number of zeroes at the
(a) 30 (b) 25 (a) 1 (b) 0 (c) 119 (d) 120 end of the product
(c) 55 (d) 50 39. Find the number of zeroes at 5× 10 × 15 ........ 75
the end of the product (a) 11 (b) 15 (c) 10 (d) 18

ANSWER KEY
1. (c) 5. (b) 9. (b) 13. (a) 17. (c) 21. (b) 25. (c) 29. (b) 33. (b) 37. (b)

r
2. (c) 6. (b) 10. (a) 14. (d) 18. (d) 22. (b) 26. (d) 30. (c) 34. (a) 38. (a)
3. (a) 7. (c) 11. (b) 15. (c) 19. (b) 23. (c) 27. (a) 31. (a) 35. (d) 39. (b)

i
.iSn
4. (b) 8. (c) 12. (b) 16. (c) 20. (c) 24. (a) 28. (a) 32. (b) 36. (b) 40. (a)

agv
SOLUTION

ridn
11.(b) 10 × 15 × 44 × 28 × 70
1. (c) 5 47 6. (b) 5 500
2 × 5 ×3× 5 × 2 × 2 ×11× 2

eeYa
5 9 5 100
5 1 5 20 124
0 × 2 ×7× 2 ×5×7
5 4
No. of zeroes = 9 + 1 = 10 0 In this expression
No. of zeroes = 100 + 20 + 4 No of 2's = 6
2. (c) 5
geisnh
125 = 124 No. of 5's = 3
5 25 Pair of 2's and 5's = 3
5 5 31 7. (c) 5 1132
So, No of zeroes = 3
5 1 5 226
5 45 12.(b) 1 2 × 5 × 1 5 × 2 4 × 1 3 × 3 0 ×7 5
0
Enak

5 9 281 2 × 2 ×3× 5 ×3× 5 × 2 × 2


No of zeroes 5 1
= 25 + 5 + 1 = 31 0 × 2 ×3×13× 2 ×3× 5 × 5 × 5 ×3
R

No. of zeroes = 226 + 45 + 9 + 1 No. of 2's  6


3. (a) 5 378 No. of 5's  5
= 281
5 75
Pair of 2's and 5's = 5
aryn

5 15 93 8. (c) 5 1098
5 3 5 219 No. of zeroes = 5
5 43 13.(a) 2×4×6×.......48×50
Les B

0 271
5 8  2×1×2×2×2×3 ........ 2×24×2×25
No. of zeroes = 75 + 15 + 3 = 93 1
 225 (1×2×3×4×.......25)
0
4. (b) 5 680 No. of zeroes = 219 + 43 + 8 + 1 Ther e ar e many 2 's I n This
series we count 5's
wa.th

5 136 = 271
5 27 169 9.(b) 5 5 25
2346
5 5 5 469 5 5
5 1 5 93 5 1 6
0 5 18 583
wwM

0
No. of zeroes = 136 + 27 + 5 + 1 5 3
No. of 5's = 5 + 1 = 6
= 169 0 Then No. of zeroes = 6
No. of zeroes = 469 + 93 + 18 + 3
14. (d) 1×3×5×7×9×11........99×101
= 583
5. (b) 5 1000 10.(a) 5 2700 Ther e is no 'zer o' in this
5 200 expression because there is no
5 40 5 540
249 5 108 even present here.
5 8 673
5 21 15.(c) 21×22×23.......59×60
5 1
0 5 4 1×2×3.....19×20×21×22×23......59×60
0
No. of zeroes = 200 + 40 + 8+ 1 No. of zeroes = 540 + 108 + 21 + 4 - 1×2×3......20
= 249 = 673

Rakesh Yadav Readers Publication Pvt. Ltd. 10

For More Visit : www.LearnEngineering.in


For More Visit : www.LearnEngineering.in

5 60 No. of zeroes In 25! = 5 + 1 = 6 5051 = (5×5×2)51 = 551 × 551


5 12 No of zeroes In 32! = 6 + 1 = 7 2576 = (5×5)76 = 576 × 576
5 2 14 5 20 No. of zero In 45! 9 + 1 = 10
5 4 4 Then no of zeroes
0 0 Total No. of zero = 6 + 7 + 10= 23 = 1 + 26 + 51 + 76 = 154
No. of zeroes 1 to 60 = 12 + 2 = 14 20.(c)
5 1140 Total number of zeroes
No. of zeroes 1 to 20 = 4 5 358
5 228 = 154 + 970 = 1124
No. of zeroes 21 to 60 = 14 - 4 = 10 5 71
5 45 23 (c) Count the No. of 5's
16. (c) 35 × 36 × 37 × ......89 × 90 283 5 14 87
5 9 5 2 55! and 1010!
1×2×3×4.....34×35×36......89×90 5 1
0 No. of 5's = 5! + 10!
–1×2×3×.........×33×34 0 Then ,

ir
5 171 No. of zeroes = 5! + 10!
No. of zeroes 1 to 90 = 18 + 3 = 21
5 34

v.iSn
5 90 24.(a) Count the No. of 5's
5 6 41
5 18 5 1 The n
21
5 3 54 × 108 × 1512 × 2016 × 2520
0
0 No. of zeroes in 1140! = 4 + 8 + 12 + 16 + 40

dnag
No. of Zeroes 1 to 34
= 228 + 45 + 9 + 1 = 283 = 80
5 34
No. of zeroes in 358! So, No. of zero = 80
5 6
5 1 7 = 71 + 14 + 2 = 87 25.(c)

eYrai
0 No. of zeroes in 171! 3200
No. of Zeroes = 6 + 1 = 7 = 34 + 6 + 1 = 41 1000
No. of zeroes 35 to 90 = 21 – 7 Total No. of zeroes= 283+87+41 40000
= 14 = 411 32000
17.(c) 41 × 42 ..............109 × 110
1×2×3×4.....40×41×42......109×110

-1×2×3.....40
only the Fives
Thus,
snhe
21.(b) The Fives will be less than the
two's Hence, we need to count
+ 15000
91200
No. or zero = 2
kgei
55 = (5× 1)5 = 5 26.(d) 3200 × 1000 × 40000 × 32000
No. of zeroes 1 to 110 = 22 + 4 = 26 10 = (5 × 2)10 = 10
10 × 15000
No. of zeroes 1 to 40 =8 + 1 = 9 No. of zero's 2 + 3 + 4 + 3 + 3
1515 = (3 × 5)15 = 15
ERna

5 110 5 40 2020 = (4 × 5)20 = 20 = 15


5 22 5 8 2525 = (5 × 5)25 = 50 But 1500 = 3 × 5 × 100
5 4 26 5 1 9
0 0 3030 = (5 × 6)30 = 30 Here 5 is present
No. of zeroes 41 to 110 = 26–9 3535 = (5 × 7)35 = 35 When 5 is multiply by ev en
aBryn

= 17 4040 = (5 × 8)40 = 40 number, then unit digit '0' is


18.(d) 140! would have 28 + 5 + 1 = 34 4545 = (5 × 9)45 = 45 get.
5 140 5 + 10 + 15 + 20 + 50+ 30 + 35 + 40 + 45 Then,
5 28 No. of Fives = 250 No. of Total zero = 15 + 1 =16
5 5 34 Then, 27.(a) 20 × 40 × 7600 × 600 × 300
Les

5 1 Number of zeroes = 250 × 1000


Now Remaining part
wa.th

22.(b) The Five will be less than the


5 × 15 × 22 × 11 × 44 × 135 No. of zeroes = 1 + 1 + 2 + 2 + 2 + 3
two's Then count the number of
5× 3 × 5 × 2 × 11 × 11 × 2 × 2 × = 11
five
100 1× 95 6 × 90 11 .........10 91 × 5 96 28.(a) 100! + 200!
11 × 5 × 27
No. of 2's = 3 (1+6+11......91+96) using sum of No. of zeroes In 100! = 20 + 4 = 24
wwM

No. of 5's = 3 A.P. No. of zeroes In 200! = 40 + 8 + 1


Pair of (2 & 5) = 3 a = 1, d=5 = 49
Remaining part of the Expression
would have 3 zeroes When you add the two Number
96  1
Total No. of zeroes = 34 + 3 = 37 No. of term = +1 = 20 (One with 24 zeroes and the
5 other with 49 zeroes at It's end)
19.(b) 5 25 5 32
5 5 5 6 20 The Total No. of zeroes = 24
5 1 6 5 1 7 Sn = [2×1+19×5]
2 29.(b) 11 × 22 × 33 × 44 .......1010
0 0
= 10 [2 + 95] = 970 Count the Number of 5's
5 45 But 55 no of fives = 5
5 9
10 1001=(5×5×4)1 = 51 × 51 1010 No. of Fives = 10
5 1
7526 = (5×5×3)26 = 526 × 526 No. of zeroes = 5 + 10 = 15
0

Rakesh Yadav Readers Publication Pvt. Ltd. 11

For More Visit : www.LearnEngineering.in


For More Visit : www.LearnEngineering.in

30.(c) 100! 200! No of 5's = 24 = Pair of (2 × 5) = 25


Then No. of zeroes = 24 No of zero = 25
5 200
5 100 5 40 33.(b) 1×3×5×7.......99×2 6 38.(a) (3123 - 3122 - 3121) (2121 - 2120 -2119)
5 20 5 8 Here No. of 5 is more than no.of  3121 (32 -31 - 30) 2119 (22 - 21 - 20)
24 49
5 4 5 1 2 then count the number of 2  (3121)(2119) (9-3-1) (4 - 2 - 1)
0 0 No. of 2's = 6  (3121)(2119) (5) (1)
No. of zeroes In 100! = 24 Now No. of zero = 6 = 51 × 2119 × 3121
No. of zeroes In 200! = 49 34.(a) 2222 × 5555 No. of 5's = 1
No. of 2's = 222 No. of 2's = 119
When you multiply two numbers
(One with 24 zeroes and the No. of 5's = 555 Pair of (2 & 5) = 1
other with 49 zeroes at It's end). No. of 2's are less than Number No. of zero = 1

r
The Resultant Total No. of zeroes of 5's 39.(b) (8123 - 8122- 8121) (3223 - 3222 - 3221)

i
.iSn
= 24 + 49 = 73 Pair (2's & 5's) = 222  8121 (82 - 81 - 1) 3221 (32 - 31 - 1)
31.(a) 5 × 10 × 15 × 20 × 25×.....×50 No. of zero = 222
 8121 (64 - 9) 3221 (9 - 4)
5×1×5×2×5×3×5×4 ......5×10 35.(d)10+100+1000+ .....100000000
10  8121 × 55 × 3221 × 5
510 (1×2×3×4 .......10)

agv
100 = 8121 × 3221 × 5 × 11 × 5
The two will be less than the
1000
.............. = (23)121 × 3221 × 52 × 11
fives hence we need to count ..............

ridn
only the two's = 11 × 52 × 2363 × 3221
100000000
1 to 10 no of 2's 111111110 No. of 2's = 363
No. of 5's = 2

eeYa
2 10 This there is only one zero at the
2 5 end of result Pair of (2 & 5) = 2
2 2 8
36.(b) 101 × 102 × 103 × 104 ........1010 No . of zero = 2
2 1
10(1 + 2 + 3........10) = 1055 40.(a) 51 × 1 × 51 × 2 × 51 × 3 ....... 51 × 15
0
No. of 2's = 5 + 2 + 1 = 8
geisnh = 515 (1 × 2 × 3 ........ 15)
Then No. of zeroes = 8 10 10  1 each term multiple of 5 So power
1+2+3....10=  55
32.(b) 2 × 4 × 6 × 8 × 10 ........ 200 2 of 5's more than 2 then count the
= 2×1×2×2×2×2×3 ........... 2×100 No. of zero = 55 number of 2 from 1 to 15.
= 2100 (1×2×3×.........100)
Enak

37.(b) 21 × 52 × 23 × 54 × 25 × 56 × 27 × 58 2 15
We count No of 5 × 29 × 510 2 7
2 3 11
5 100  2(1+3+5+7+9) × 5 (2+4+6+8+10)
2 1
R

5 20  225 × 530
24 0
5 4 Number of 2's are less than the number of (2 and 5) pairs = 11
0
Number of 5's
aryn

then number of zeroes = 11


Les B
wa.th
wwM

Rakesh Yadav Readers Publication Pvt. Ltd. 12

For More Visit : www.LearnEngineering.in


For More Visit : www.LearnEngineering.in

CHAPTER

03
FACTOR
Factor A number which divides So,
a given number exactly is called fac- * This m ethod is easy f or

ir
Total number of even Factor
tor (or divisor) of that given num- smaller number but for larger of 24 = 6

v.iSn
ber and the given number is called number its a probelm So use
for alternate method Alternate
a multiple of that number.
Alternate 2 24
Ex. 1, 2, 4, and 8 are factors of 8 2 2
because 8 is perfectly divisible of 1, 8 = 2 × 2 × 2 = 23

dnag
2 6
2, 4 and 8 Number of Total factors 3 3
Factors and Multiple 1
=3+1=4 24 = 23 × 31

eYrai
Ex. Factors of 35 = 1, 5, 7, 35 Ex.2 Find the total number of fac- Number of even factor = 3×(1+1)
Ex. Factors of 24 = 1, 2, 3, 4, 6, 8, tors of 240 =3×2=6
12, 24 Sol. 2 Ex.5 Find the number of even fac-
240
Same, 2 120 tor of 60.
Multiple of 2 = 2, 4, 6, 8, 10, .........
Multiple of 7 = 7, 14, 21, 28, 35 ........
* 1 is a factor of every number
2
2
3
5
60
30
15
5
snhe Sol. 2 60
2 30
3 15
5 5
kgei
* every number is a factor of it- 1 1
self 240 = 2 × 2 × 2 × 2 × 3 × 5 60 = 2 × 2 × 3 × 5
= 2 4 × 31 × 51 = 2 2 × 31 × 51
*
ERna

every number, except 1 has at


No. of even factor = 2 × (1+1) × (1+1)
least 2 factor Total Factors
=2×2×2=8
* eve ry numbe r has infinite = (4 + 1) × (1 + 1) × (1 + 1)
No. of odd factor
number of its multiples = 5 × 2 × 2 = 20
aBryn

Ex.6 Find the number of odd factors


* every number is a multiple of Ex.3 Find the total number of fac- of 40.
itself tors of 500. Sol. 40 = 1, 2, 4, 5, 8, 10, 20, 40
Number of Factors Sol. Odd factors = 1, 5
Let N be the composite number 2 500 Number of odd factors = 2
2 250 Alternate
Les

and a, b, c,.. be its prime factors


5 125 2 40
and p, q, r be the indices (or pow-
wa.th

5 25 2 20
ers) of a, b, c, i.e, if N can be
5 5 2 10
expressed as N = ap.bq.cr then to- 1
tal number of factors of N = (p + 1)
5 5
1
× (q + 1) × (r + 1) 500 = 2 × 2 × 5 × 5 × 5
40 = 2 × 2 × 2 × 5
wwM

If a is even prime factor, b and c = 22 × 53 = 2 3 × 51


are odd prime factors No. of odd Factors= 1 × (1 + 1)
The number of even factors No. of factors = (2 + 1) × (3 + 1)
=1×2
= (P) × (q + 1) × (r + 1) = 3 × 4 = 12 =2
The number of odd factors Ex.7 Find the number of factors,
Number of even Factor
number of even factors and
= (1) × (q + 1) × (r + 1)
Ex4. Find the numb er of ev en number of odd factors of 180
Ex.1 Find the total number of fac- Sol. 180 = 2 × 2 × 3 × 3 × 5
tors of 8. factors of 24.
= 22 × 3 2 × 51
Sol. 8 = 1, 2, 4 and 8 are Perfectly Sol. Factor of 24 = 1, 2, 3, 4, 6, 8, 12, 24 Total Number of factors
divisible Even Factor of 24 = 2, 4, 6, 8, = (2 + 1) × (2 + 1) × (1 + 1)
So number of factors = 4 12, 24, = 3 × 3 × 2 = 18

Rakesh Yadav Readers Publication Pvt. Ltd. 13

For More Visit : www.LearnEngineering.in


For More Visit : www.LearnEngineering.in

Number of even factors  Sum of even factors = (a1 + a2 sum of all factors
= 2 (2 + 1) × (1 + 1) ... ap) × (bº + b1 + b² .... bq) × (cº = (20 + 21 + 22 + 23) × (30 + 31 + 32)
= 2 × 3 × 2 = 12 + c1 + c² ..... cr) × (50 + 51)
Number of odd factors  Sum of odd actor = (aº) × (bº + = 15 × 13 × 6 = 1170
= 1 × (2 + 1) × (1 + 1) b1 + b² .... bq) × (cº + c1 + c²......cr) Sum of even factors
=1×3×2=6 =(21 + 22 + 23) × (30 + 31 + 32)
Ex10. Find the sum of all factors of 8.
Ex.8 Find the number of factors, × (50 × 51)
Sol.factors of 8 = 1, 2, 4, 8
number of even factors and = 14 × 13 × 6 = 1092
Sum of factors = 1 + 2 + 4 + 8
number of odd factors of 360. sum of odd factors
= 15
Sol. 2 360 = 20 × (30 + 31 + 32) × (50 × 51)
2 180
This method is easy for smaller
= 1 × 13 × 6 = 78

r
2 90 number but for larger number
Ex13.find the sum of all factors ,

i
.iSn
3 45 its a probelm So use for alter-
sum of even factors and sum of
3 15 nate method
odd factors of 100.
5 5 Alternate
1 8 = 23 2 100

agv
360 = 2 × 2 × 2 × 3 × 3 × 5 sum of all factors = (20 + 21 + 22 + 23) 2 50
= 2 3 × 32 × 51 = 1 + 2 + 4 + 8 = 15 5 25

ridn
Total number of factors
5 5
0
(a = 1, where a = real number) 1 100 = 22 × 52
= (3 + 1) × (2 + 1) × (1 + 1)

eeYa
= 4 × 3 × 2 = 24 Ex.11find the sum of all factors , Sum of all factors
Number of even factors sum of even factors and sum of = (20 +21 + 22) × (50 + 51 + 52)
= 3 × (2 + 1) × (1 +1) odd factors of 24. = 7 × 31 = 217
= 3 × 3 × 2 = 18 Sol. factors of 24 = 1, 2, 3, 4, 6, 8, 12, 24 Sum of even factors
Number of odd factors
geisnh
sum of factors = 1 + 2 + 3 + 4 + 6 = (21 + 22) × (50 + 51 + 52)
+ 8 + 12 + 24 = 60 = 6 × 31 = 186
= 1 × (2 + 1) × (1 + 1)
sum of even factors Sum of odd factors
=3×2=6
= 2 + 4 + 6 + 8 + 12 + 24 = 56 = (20) × (50 + 51 + 52)
Ex.9 Find the number of factors,
= 1 × 31 = 31
Enak

number of even factors and Sum of odd factors = 1 + 3 = 4


number of odd factors of 100 Alternate Prime Factorisation
Sol. 2 24 Prime Factorisation : If a natural
2 100
R

2 12 number is expressed as the product


2 50
2 6
5 25 of prime numbers (factors) then the
3 3
aryn

5 5 factorisation of the number is called


1 24 = 23 × 31
1 its prime factorisation.
Les B

sum of all factors (i)72


100 = 2 × 2 × 5 × 5 = 22 × 52
= (20 + 21 + 22 + 23) × (30 + 31)
Total no. of Factor
= (1 + 2 + 4 + 8) × (1 + 3) 2 72
= (2 + 1) × (2 + 1) = 3 × 3 = 9 2 36
= 15 × 4 = 60
No. of even factor= 2 × (2 + 1) 2 18
wa.th

=2×3=6 Sum of even factors 3 9


No. of odd factor = 1 × (2 + 1) = (21 + 22 + 23) × (30 + 31) 3 3
=1×3=3 = (2 + 4 + 8) × (1 + 3) 1
= 14 × 4 = 56
Sum of factors 72 = 2×2×2×3×3
wwM

Sum of odd factos


72 = 23 × 32
Let N be the composite number = (20) × (30 + 31) = 1 × 4 = 4
and a, b, c,... be its prime factors number of prime factors = 3 + 2 = 5
Ex12.find the sum of all factors ,
and p, q, r be the indices (or (ii) 540
sum of even factors and sum of
powers) of a, b, c, i.e. if N can be odd factors of 360. 2 540
expressed as N = ap.bq.cr 2 270
Sol.
2 360 3 135
then the sum of all the divi- 3 45
2 180
sors (or factors) of N 2 90 3 15
= (aº + a1 + a² .... ap) × (bº + b1 + 3 45 5 5
b² ....bq) × (cº + c1 + c² ...... cr) 3 15 1
360 = 23 × 32 × 51
If a is even prime factor and b 5 5 540 = 2 × 2× 3× 3× 3× 5
and c odd prime factors then 1 = 2 2 × 33 × 51

Rakesh Yadav Readers Publication Pvt. Ltd. 14

For More Visit : www.LearnEngineering.in


For More Visit : www.LearnEngineering.in

No. of prime factor = 2 + 3 + 1 211 × 34 × 515 × 74 Sol. 64 ×86× 108× 1210


=6 Total No. of prime factors  (2×3)4 × (2×2×2)6 ×(2×5)8×(2 ×2×3)10
(iii) find the number of prime factor = 11 +4 + 15 + 4 = 34  24×34 × (23)6 × 28 × 58 × (22 ×3)10
23 × 57 × 214 × 108 Ex14.The Number of prime Factors In  24 × 34 × 218 × 28 × 58 × 220 × 310
Sol.23 × 57 × 214 × 108 the expression 64 ×86× 108× 1210 is  24+18+8+20 ×34+10 ×58
= 250 × 314 × 58
23 × 57 × (3 × 7)4 × (2 × 5)8 (a) 48 (b) 64
Total No. of prime factor
2 3 × 5 7 × 34 × 74 × 28 × 58 (b) 72 (d) 80 = 50 + 14 + 8 = 72

EXERCISE

ir
1 Find the number of Factors of 8. Find the No. of Prime Factor of (a) 18,9,9 (b) 18,10,8

v.iSn
1728 536 (c) 18,8,10 (d) 18,12,6
(a) 28 (b) 29 (c) 30 (d) 31 (a) 4 (b) 5 (c) 6 (d) 3 15. For the Number 760
2. Find the Number of Factor of 9. Find the No. of prime Factor of (i) The sum and Number of all
1420 1044 factors

dnag
(a) 12 (b) 13 (c) 14 (d) 15 (a) 4 (b) 5 (c) 10 (d) 9 (ii) The Sum and Number of
10. Find The No. of prime factor of even factors
3. Find the Number of Divisors of
(56)20 × (36)31 × (42)13 × (13)21 (iii) The Sum and Number of
10800
(a) 240 (b) 242 (c) 264 (d) 248 odd factors

eYrai
(a) 30 (b) 60 (c) 120 (d) 180
11. Find the total Number of Prime 16. For The Number 96
4. Find the No. of Prime Factor of (i) Sum and number of all
240. Factors of
factors
(a) 4 (b) 5 (c) 6 (d) 8 217 ×631 × 75× 1011 × 1110 × 2112
(ii) The sum and Number of
(a) 142 (b) 144 (c) 140 (d) 146
5. Find the No. of prime factor.
(30)26 × (25)51 × (12)23
(a) 249 (b) 250 (c) 255 (d) 260
12.

13.
(a) 3
snhe
Find the prime Factors 210
(b) 4 (c) 5
Find the sum of odd factors
(d) 6
even factors
(iii) The sum and Number of
odd factors
17. For the Number 270
6. Find the No. of Prime Factor
kgei
of544 (i) The sum & Number of all
(30)15 × (22)11 × (15)24
(a) 16 (b) 18 (c) 20 (d) 22 Factor
(a) 110 (b) 115 (c) 120 (d) 125
14. For the Number 2450 find (ii) The sum & Numb er of
7. Find the No. of Prime Factor
ERna

(i) Number of all factors even factor


180
(ii) Number of even factors (iii) The sum & Number of odd
(a) 4 (b) 5 (c) 6 (d) 7 Factor
(iii) Number of odd factors
aBryn

ANSWER KEY
1. (a) 3. (b) 5. (a) 7. (b) 9. (b) 11. (c) 13. (b)
2. (a) 4. (c) 6. (b) 8. (a) 10. (c) 12. (b) 14. (a)
Les
wa.th

SOLUTION

1. (a) = 7 × 4 = 28
2 1728 3. (b) 2
wwM

10800
2 864 2 (a) 2 1420 2 5400
2 432 2 710 2 2700
2 216 5 355 2 1350
2 108 71 71 3 675
2 54 1 3 225
3 27 3 75
3 9 1420 = 2 × 2 × 5 × 71 5 25
3 3 5 5
= 22 × 51 × 711
1 1
1728 = 2×2×2×2×2×2×3×3×3 No. of factors
= 26 × 33 = (2 + 1) × (1+1) × (1+1) 10800 = 2×2×2×2×3×3×3×5×5
No. of factors = (6 + 1) × (3 + 1) = 3 × 2 × 2 = 12 = 24 × 23 × 52 = 24 × 33 × 52

Rakesh Yadav Readers Publication Pvt. Ltd. 15

For More Visit : www.LearnEngineering.in


For More Visit : www.LearnEngineering.in

No. of factors = (4 + 1) (3 + 1) (2 + 1) No. of prime factor 15.


2 760
= 5 × 4 × 3 = 60 =2+2+1=5 2 380
4. (c) 2 240 10. (c) (56)20 × (36)31 × (42)13 × (13)21 2 190
2 120 5 95
2 60  (2× 2 × 2 × 7 )20 × (2²×3²)31 × ( 2 × 3 19 19
2 30 1
× 7 )13 × (13)21
3 15 760 = 2×2×2×5×19
5 5  (2³ ×7)20 × (262)×(3)62 × ( 2 × 3 ×
1 = 23×5 1×19 1
13 21
240 = 2 × 2 × 2 × 2 × 3 × 5 7 ) × (13) (i) Numbe of factor
60 20 62 62 13 13 13
= 24 × 31 × 51  2 × 7 × 2 × 3 × 2 × 3 ×7 = (3+1)×(1+1)×(1+1)

r
No. of prime factor = 4+1+1=6 ×1321
= 4×2×2=16
260 + 62+13 × 362+13 × 720+13 × 1321

i

.iSn
5. (a) (30)26 × (25)51 × (12)23
Sum of factor
Break The form of prime factor  2135 × 375 × 733 × 1321
 (21×31 ×51)26 × (5×5)51 × (2×2×3)23 Number of prime factors = (2 + 21+22+23) × (50 +51) × (190 +191)
0

= 135 + 75 + 33 + 21 = 264 = (1+2+4+8) × (1+5) × (1+19)

agv
 226 × 326 ×526 × 5102 × 246 × 323
11. (c) 217 ×631 × 75× 1011 × 1110 × 2112 = 15×6×20 = 1800
 226+46 × 326+23 × 526+102  217 × (2×3)31 × 75 × (2×5)11 ×1110 ×

ridn
(ii) Number of even factor
 272 × 349 × 5128 (3×7)12
= 3 × (1 + 1) × (1 + 1)
No. of prime factors  217 × 231 × 331 × 75 × 211 × 511 × 1110

eeYa
× 312 × 712 = 3 × 2 × 2 = 12
 72 + 49 + 128 = 249
 217+31+11 × 331+12 × 511 × 75+12 × 1110 Sum of even factor
6. (b) (30)15 × (22)11 × (15)24  259 × 343 ×511 × 717 × 1110 =(2 +22 + 23) × (50 + 51) × (190 +191)
1
 (2×3×5)15 × (2×11)11 × (3×5)24 Total No. of Prime Factors
geisnh = 14 × 6 × 20 = 1680
 215 × 315 ×515 × 211 ×1111 ×324 ×524 = 59 + 43 + 11 + 17 + 10
(iii) Number of odd factors
 215+11 ×315+24 ×515+24+1111 = 140
= 1 × (1+1) × (1+1)
 226 ×339 ×539 ×1111 12. (b) 2 210
=1×2×2=4
Enak

No. of Prime factor 3 105


5 35 Sum of odd factors
26 + 39 + 39 + 11 = 115 7 7
= (2 ) × (50 + 51) × (190 + 191)
0
1
R

7. (b) 2 180 = 1 × 6 × 20 = 120


2 90 210 = 21 × 3 1 × 51 × 71
3 45 16. 96 = 2×2×2×2×2×3
aryn

=1+1+1+1=4
3 15 = 25 ×31
5 5 13. (b) 544 = 2 × 2 × 2 × 2 × 2 × 1 7
Les B

1 = 25 × 171 (i) Number of all factor


Sum of odd factors = (5+1) × (1+1) = 6 × 2 = 12
180 = 2 × 2 × 3 ×3 × 5
= (20) × (170 + 171) Sum of all factor
=22 × 32 ×51
= 1 × (1 + 17) = (20 +21 + 22 +23 +24 +25 )×(30 +31)
No. of prime Factor = 2+2+1 = 5
wa.th

= 1 × 18 = 18
8. (a) 2 536 = (1+2+4+8+16+32)×(1+3)
2 268 14. (a) 2 2450 = 63 × 4 = 252
5 1225
2 134 (ii) Number of even factor
5 245
67 67
wwM

7 49 = 5 × (1+1) = 5 × 2 = 10
1
7 7
1 Sum of even factor
536 = 2×2×2×67 = 23 × 671
No. of prime factor = 3+1 = 4 2450 = 2 × 5 × 5 × 7 × 7 = (21 + 22 +23 +24 +25 )×(30 +31)
2450 = 21 × 52 × 72 = (2+4+8+16+32) × (1+3)
9. (b) 2 1044
2 522 Number of Factor = (1 + 1) (2 + 1) (2 + 1) = 62 × 4 = 248
3 261 (iii) Number of odd factor
= 2 × 3 × 3 = 18
3 87 = 1× (1 + 1) = 1 × 2 = 2
29 29 Number of even Factor = 1 × (2 + 1 ) × (2 + 1)
1 Sum of odd factor
=1×3×3=9
= (20) × (30 + 31)
1044 = 2×2×3×3×29 Number of odd factor = 1(2+1)×(2+1)
=1×4=4
= 22 × 32 × 291 =3×3=9

Rakesh Yadav Readers Publication Pvt. Ltd. 16

For More Visit : www.LearnEngineering.in


For More Visit : www.LearnEngineering.in

17.Sol. 2 270 = (1+1)×(3+1)×(1+1) Sum of even factor


3 135 = 2×4×2=16 = (21)×(30+31 +32 +33) × (50 + 51)
3 45
Sum of all factor = 2 × 40 × 6 = 480
3 15
5 5 = (20 +21)×(30 +31 +32+33)×(50+51) Number of odd factors
1 = 3 × 40 × 6 = 720 = 1×(3+1)×(1+1)
270 = 2×3×3×3×5 (ii) Number of even factor = 4×2 = 8
= 21×33×51 = 1 × (3+1) × (1+1) Sum of odd factors
(i) Number of all factor =1×4×2=8 =20×(30+31+32+33)×(50+51)
= 1×40×6 = 240

ir
v.iSn
dnag
eYrai
snhe
kgei
ERna
aBryn
Les
wa.th
wwM

Rakesh Yadav Readers Publication Pvt. Ltd. 17

For More Visit : www.LearnEngineering.in


For More Visit : www.LearnEngineering.in

CHAPTER

04
DIVISIBILITY
Rule of Divisibility (i) I f any numb er is made b y 55 is not divisible by 7 So,
repeating a digit 6 times the 68734 is not divisible by 7

r
* Divisibility by 2 If Last number will be divisible by 3, Ex.3: C he ck to se e if 2 49 83 is

i
.iSn
d ig i t o f t h e n u m b e r i s 7, 11, 13, 21, 37 and 1001 etc. divisible by 7
divisible by 2 (ii) A six digit number if formed by
Divisibility by 4 If Last re pe ating a thre e digit Sol. 2498 3
two digits of the number are number; for example, 256, 256 –6 ×2

agv
divisibile by 4 or 678, 678 etc. Any number of 249 2
Divisibility by 8 If Last this form is always exactly –4 ×2

ridn
three digits of the number are divisible by 7, 11, 13, 1001 etc. 24 5
divisible by 8 Some important points –10 ×2
Divisibility by 16 If Last

eeYa
14
four digits of the number are (a) If a is divisible by b then ac is
divisible by 16 also divisible by b. 14 is divisible by 7, therefore 24983
Divisibility by 32 If Last (b) If a is divisible by b and b is is also divisible by 7
five digits of the number are divisible by c then a is divisible Ex.4: Check to see if 65432577 is
divisible by 32
geisnh
by c. divisible by 7
* Divisibility of 3 All such (c) If n is divisible by d and m is Sol. When any number is made of
numbers the Sum of whose divisible by d then (m + n) and more than five digits then we
digits are divisible by 3 (m – n) are both divisible by d. check divisiblity by 7 another
This has an im portant
Enak

Divisibility of 9 All such rule


numbers the Sum of whose implication. Suppose 48 and Step I. First for we make pair of 3
digits are divisible by 9 528 are both divisible by 8. digits from right side (last)
* Divisibility by 6 A number Then (528 + 48) as well as
65 432 577
R

is d iv is ib le b y 6 If it is (528 – 48) are divisible by 8


Step II. Add alternate pairs
simultaneously divisible by 2 Ex.1: Chec k to s ee if 20 3 is
= 65 + 577 = 642
aryn

and 3 divisible by 7
Step III. Substract from remaning
* Divisibility by 5 If Last Sol. 20 3 (3rd) pair = 642 –432 = 210
Les B

digit (0 and 5) is divisible by 5 –6 ×2 If d if fe re nc e is d iv is ib le b y 7


Divisibility by 25 If Last 14 therefore number is also divisible
two digits of the number are by 7
Step I. Double the last digit = 3 × 2
divisibile by 25 Here difference = 210
=6
wa.th

Divisibility by 125 If Last 210 is divisible by 7. Therefore


Step II. Substract that from the rest
three digits of the number are 65432577 will be divisible by 7.
of the Number = 20 – 6
divisible by 125 Note:- We can use First rule of
= 14
* Divisibility by 7 Double divisibility by 7 but when a number
Step III. Chec k to s ee if the
the last digit and subtract it
wwM

difference is divisible by 7. 14 has more than 5 digits this rule is


from the remaining leading easy for solve problem.
is divisible by 7 therefore 203
truncated number. If the result Ex.5: Check to see if 23756789765
is also divisible by 7
is divisible by 7, then so was is divisible by 7
Ex.2: Check to see if 68734 is
the original number. Sol. 23 756 789 765
divisible by 7
* Divis ibil ity by 11 The Step I. Add alternate pair
difference of the sum of the Sol. 6873 4
765 + 756 = 1521
digits in the odd places and the –8 ×2
686 5
23 + 789 = 812
sum of digits in the even places
–10 ×2 Step II. Substract pairs
is ' 0' or multip le of 11 is
divisible 67 6 1521 – 812 = 709
* Divisibility by 3, 7, 11, 13, 21, –12 ×2 709 is not divisible by 7 therefore
37 and 1001 55 23756789765 is not divisible by 7

Rakesh Yadav Readers Publication Pvt. Ltd. 18

For More Visit : www.LearnEngineering.in


For More Visit : www.LearnEngineering.in

Ex.6: If 5432*7 is divisible by 9, then Alternate:-


15  x
the digit in place of * is 2.............................2 =2+4+3+x+5+1=
9
(a) 0 (b) 1 (c) 6 (d) 9 This number has 99 digits. First
(1st) and last (99th) term is 2 (given) 18
5  4  3  2  x  7 21  x x would be 3 =
Sol. (c) = middle terms (2nd to 98th) is assume 4. 9
9 9 3rd to 98th term = (4 ............ 4)
Put the value of 'x'. So the So, x = 3
differece between odd and
num be r is c om pletely Ex.12. 271 + 272 + 273 + 274) is divisible
even place of terms (3rd to 98th) = 0
divisible by 9. Put x = 6 by
Remaining Terms
(a) 9 (b) 10 (c) 11 (d) 13
21  6 27 1st, 2nd and 99th (last)
= = ='0' remainder Sol.(b) Expression
9 9 Here,
= 271 (1 + 2 + 4 + 8)

ir
1st term = 2
Pro perty : A numb er is = 271 × 15 = 271 × 3 × 5
2nd terms = 4

v.iSn
completely divisible by 9 it the Which is exactly divisible by 10.
s um of the digits of the Last terms = 2
Ex.13. A 4 digit number is formed
num be r is c om pletely Difference of odd and even place of by repeating a 2-digit number
divisible by 9 and give no the remaning term such of 2525, 3232, etc.Any
(2 + 2) – 4 = 0

dnag
remainder. number of this form is always
Ex.7: When 335 is added to 5A7, the So, If the middle digit be 4, then exactly divisible by :
result is 8B2. 8B2 is divisible 24442 or 244442 etc are divisible (a) 7 (b) 11 (c) 13
by 3 . What is the lar ge st by 11. (d) Sm alle st 3 -d ig it p rime

eYrai
posisble value of A? Ex.10: Both the end digits of a 100 number
(a) 8 (b) 2 (c) 1 (d) 4 digit number N are 2. N is Sol.(d) Let the unit digit be x and
divisible by 11 then all the ten's digit be y.
Sol. 5 A 7 middle digits are:
3 3 5  Number
(a) Only 4 (b) Only 2
8 B 2
 A  1, 2, 3, 4, 5 &
B  5, 6, 7, 8, 9
snhe
(c) Only 3 (d) Any digit
Sol. (d) A number is divisible by 11
if the difference of the sum of digits
=1000y + 100x + 10y + x
= 1010y + 101x = 101 (10y + x)
Clearly, this numb er is
divisible by 101, which is the
kgei
8B2 is exactly  8 + B + 2 = at odd and even places by either sm alle st thr ee -d ig it p rime
multiple of 3 zero or multiple of 11. number.
 B = 5 or 8  A = 1 or 4 If the middle digit be any digit then Ex. 14: A six digit number is formed
21 11 12 or 23 33 33 32 e tc are
ERna

Ex.8: If * is a digit such that 5824* by repe ating a thre e digit


is d iv is ib le b y 11 , then * divisible by 11. number; for example, 256,
equals : Alternate:- 256 or 678, 678 e tc. Any
(a) 2 (b) 3 (c) 5 (d) 6 2.............................2 number of this form is always
This number has 100 digits. First exactly divisible by:
aBryn

Sol. (c) 5 8 2 4 * (1st) and last (100th) term is 2 (given) (a) 7 only (b) 11 only
middle terms (2nd to 99th) is assume (c) 13 only (d) 1001
 5 +2 + * = 8 + 4
any digit. Sol. (d) The number (x y z x y z) can
7 + * = 12
2rd to 99th term = (n ............ n) be written, after giv en
* = 12 - 7 = 5
differece between odd and corresponding weightage of
Les

Property: A number will be even places (2rd to 99th) = 0 the places at which the digits
exactly divisible by 11 when the
wa.th

Remaining Terms occur, as 100000 x + 10000y


difference of the sum of odd place 1st, and 100th (last) + 1000z + 100+ 10y + z
digits and even place digits is zero Here, = 100100x + 10010y + 1001z
or divisible by 11. 1st term = 2 = 1001 (100x + 10y + z)
Ex.9: Both the end digits of a 99 Last terms = 2 Since 1001 is a factor, the
wwM

digit number N are 2 N is Difference of odd and even place of number is divisible by 1001.
divisible by 11 then all the the remaning terms As the number is divisible by
middle digits are: (2 – 2) = 0 1001, it will also be divisible
(a) 1 (b) 2 (c) 3 (d) 4 So, If the middle digit be any digit, by all three namely, 7, 11 and
Sol. (d) A number is divisible by 11 then (2..........2) is divisible by 11. 13 and not by only one of these
Ex.11: If the number 243x 51 is because all three are factors
if the difference of the sum of digits
divisible by 9 then the value of of 1001.
at odd and even places by either
the digit marked as x would be: Ex .15: Which of the following
zero or multiple of 11. number will always divide a
If the middle digit be 4, then (a) 3 (b) 1 (c) 2 (d) 4
Sol. (a) 243x 51 is divisible by 9 six–digit number of the form
24442 or 244442 etc are divisible xyxyxy (where 1  x  9, 1  y
divisiblty of 9 = sum of digit
by 11.  9)?
divisible by 9

Rakesh Yadav Readers Publication Pvt. Ltd. 19

For More Visit : www.LearnEngineering.in


For More Visit : www.LearnEngineering.in

(a) 1010 (b) 10101 Ex.23: In a division sum, the divisor


dividend
(c) 11011 (d) 11010 is 10 times the quotient and
Sol. (b) Number = xy xy xy 6 27 4 Quotient 5 times the remainder. If the
= xy × 10000 + xy × 100 + xy divisor 24 re mainde r is 4 6, the
= xy (10000 + 100 + 1) 3 Remainder dividend is:
= xy × 10101 Sol. Remainder = 46
6 × 4 + 3 = 27
Ex.16. 47 is added to the product of Divisor = 5 × 46 = 230
divisor × quotient + remainder
71 and an unknown number. The = dividend 230
new number is divisible by 7 giving Ex.20: In a pr oble m involv ing Quotient =  23
10
the quotient 98. The unknown division, the divisor is eight
times the quotient and four Dividend = divisor × quotient
number is a multiple of

r
times the remainder. If the + remainder
(a) 2 (b) 5 (c) 7 (d) 3
= 230 × 23 + 46

i
.iSn
Sol. (d) Let the unknown number remainder is 12, then the
dividend is: = 5290 + 46 = 5336
be x.
(a) 300 (b) 288 (c) 512 (d) 524 Ex.24:On dividing 397246 by a
 71 × x + 47 = 98 × 7 Sol. (a) Remainder = 12 certain number, the quotient

agv
 71x = 686 – 47 = 639 Divisor = 4 × 12 = 48 is 865 and the remainder is
639 48 211. Find the divisior.
 x= =9=3×3

ridn
Quotient = =6
71 8 397246 – 211
Ex.17. When an integer K is divided Dividend = divisor × quotient Sol. Divisor = = 459
865

eeYa
by 3, the remainder is 1, and + remainder = 48 × 6 + 12
when K + 1 is divided by 5, there = 288 + 12 = 300 Ex.25: A number when divided by
remainder is 0. Of the following, Ex.21: The divisor is 25 times the 14 leaves a remainder of 8,
a possible value of K is quotient and 5 time s the but when the same number
remainder. If the quotient is is divided by 7, it will leave the
(a) 62 (b) 63 (c) 64 (d) 65
geisnh
16, the dividend is : remainder:
Sol. (c) Take option (d)
(a) 6400 (b) 6480 (a) 3 (b) 2 (c) 1
Whe n 64 is divide d by 3 ,
(c) 400 (d) 480 (d) Can't be determined
remainder = 1
Sol. (b) Divide nd = divisor × Sol. 14 N x
Whe n 65 is divide d by 5 ,
Enak

quotient + Remainder
remainder = 0 According to Question
Ex.18. If n is a whole numb er Divisor = 16×25 = 5×R 8 Remainder
N = 14x + 8
greater than 1, then n² (n²–1)
R

1 According to the question,


is always divisible by:  R= ×16×25 0 1
5
(a) 16 (b) 12 (c) 10 (d) 8
aryn

Dividend
Sol.(b) n² (n² –1) = n² (n+1) (n–1) N 14x +8
1 =
Now, we put values n = 2,3..... 7 7
Les B

= [(16×25)×16] + × 16×25
When n = 2 5
8
 n² (n²–1) = 4×3×1=12, which = [16 × 25 × 16] + 80 = 6480 Remainder = = 1
7
is a multiple of 12 Ex.22: In a division problem, the
Ex.26: If a number is divided by 102
When n= 3. divisor is 4 times the quotient
and leaves remainder 91. If
wa.th

n² (n²–1) = 9×4×2=72, and 3 times the remainder. If


remainder is 4, the dividend is: this number is divided by 17
Which is also a multiple of 12. the remainder
(a) 36 (b) 40 (c) 12 (d) 30
etc. Sol.
Sol. (b)
Ex.19. If n is ev en, (6 n – 1) is dividend
wwM

divisible by Divisor Dividend Quotient


102 M P: Quotient
(a) 37 (b) 35 (c) 30 (d) 6 –
Remainder divisor
Sol.(b) When n = 2.
According to the question 91 Remainder
6n – 1 = 6² – 1 = 36 – 1 = 35 We know that
When, n = an even number, 12 Dividend 3 divisor×quotent+ remainder =divided
an – bn is always divisible by (a²
– M = 102 × P + 91
– b²).
Relation among divisor, dividend 4
176  P91
quotient and remainder : Divide nd is (Div is or × Now = M =
17
Quotient) + Remainder
27 = (12 × 3) + 4 17  6P 91
 27 is divided by 6 then M= +
6 = 40 17 17

Rakesh Yadav Readers Publication Pvt. Ltd. 20

For More Visit : www.LearnEngineering.in


For More Visit : www.LearnEngineering.in

91 Then = 0 10 24
R=  R=6 Remainder = = 7
17 17
Note: If we have to find remainder (225 Q + 70) Ex.34:When a number is divided
of those term which divide previous 15 certain divisor, remainder is
term we will take remainder of it
2 35 but another no. is divided
and divide by this term and we have 10  100
to get. = = by the same divisor
15 15 remainder is 27. If the sum of
91 both number is divided by the
R=6 Remainder = 10
17 same certain divisor
Alternate:-
Ex.27: If a number is divided by 84 remainder is 20. Find the

ir
and leaves remainder 37. If 2 2
certain divisor
this number is divided by 12.
Remainder 70
= 

v.iSn
15 15 Sol. N1 = Dx + 35 .... (i)
Sol. Then the remainder 84 is
N2 = Dy + 27 ......(ii)
divisible by 12 10 Here N1 = First no.

7 2
10
2
100 N2 = Second no.
70
=   =

dnag
So, remainder = = 1 = 
12 15 D = certain divisor
15 15
Ex.28: A number when divided by x & y = Quotient
899 gives a remainder 63. If Remainder = 10
(i) + (ii)
the same number is divided Ex.32:If a number is divided 36 and

eYrai
According to the question
by 29, the remainder will be: leaves remainder 23. If cube
(a) 10 (b) 5 (c) 4 (d) 2 of this number is divided by N1+N2 D(x +y)+62
12. Then what is the =
Remainder 63 D D
Sol. (b) = remainder.
29
 remainder = 5
29

Ex.29: A number when divided by


Sol. 36 N snhe Q
Here divisor is same

Then Remainder = D 62 1
23 – 42
kgei
296 gives a remainder 75.
When the same number is N = 36 Q + 23 20
divided by 37 the remainder cube of number
will be Remainder = 20
= N³ = (36 Q + 23)³
ERna

(a) 1 (b) 2 (c) 8 (d) 11 Quotient = 1


0 –1 Dividend = 62
Remainder 75
Sol. (a) = Divisor = 62 – 20 × 1 = 42
37 37 Now, (36 Q + 23)3
aBryn

remaindder = 1 Divisor = 42
12
Ex.30:A number being divided by 52 3 Alternate:
gives remainder 45. If the =
 –1
=
–1
12 N1 N2 , N1+N2
number is divided by 13, the 12
Remainder = 12 – 1 = 11 D D D
remainder will be:
Les

(a) 5 (b) 6 (c) 12 (d) 7 Alternate:-


–1
wa.th

Sol. (b) since 13 is factor of 52. So  R1 R2 R3


divide its remainder by 13 Remainder = 23 3
 
12 D = R1+ R2– R3
45 3
Remainder = = 6 –1 –1
13 =   = Then divisor = 35 + 27 – 20
wwM

12 12
Ex.31: If A number is divided by 225 = 42
a remainder at 70. But when Remainder = 12 – 1 = 11 Successive Division : If the quotient
a square of the number is Ex.33:Two number when divided by in a division is further used as a
divided by 15. What is the 17. Leave remainder 13 and dividend for the next divisor and again
remainder? 11 respectively if the sum of the latest obtained divisor is used as
those two numbers is divided a dividend for another divisor and so
Sol. 225 N Q on, then it is called then " successive
by 17 the remainder will be
division" i.e, if we divide 150 by 4, we
Sol. N1 (First Number) = 17x+13 get 37 as quotient and 2 as a
70
N2 (Second no.) = 17y + 11 remainder then if 37 it divided by
N = 225 Q + 70
another divisior say 5 then we get 7
Square of number = N² (N1+N2 ) 17(x  y ) 13  11
= + as a quotient and 2 remainder and
= (225 Q + 70)² 17 17 17 again if we divide 7 by another divisior

Rakesh Yadav Readers Publication Pvt. Ltd. 21

For More Visit : www.LearnEngineering.in


For More Visit : www.LearnEngineering.in

say 3 we get 2 as quotient and 1 as a 2 + 1 Alternate II.


Alternate :
remainder i.e, we can represent it as × 4 37 1
following +
5 1 5 9 4
4 150 × 1
5 37 2 1×5+4=9
+ 3
3 7 2 Remainder 4
2 9 × 4 + 1 = 37
1 × Number = 37
Now you can see that the 3 + 1 Now, divided b y 5 and 4
quotient obtained in the first division
Step 1. (1 + 3) × 4 = 16 successively
behaves as a dividend for another
Step 2. (16 + 3) × 5 = 95
divisor 5. Once again the quotient 7 5 37 7
Step 3. (95 + 1) × 2 = 192

r
is treated as a dividend for the next 35
divisor 3. Thus it is clear from the Step 4. (192 + 1) = 193

i
.iSn
2 Remainder
above discussion as Or
Dividend Divisior Quotient Remainder
Number = (((1 + 3) × 4 + 3) × 5 4
7 1
+ 1) × 2 + 1 4
150 4 37 2

agv
= ((16 + 3) × 5 + 1) × 2 + 1 3 Remainder
37 5 7 2
7 3 2 1 = 96 × 2 + 1
Remainder = 2, 3

ridn
So the 150 is successively divided Ex.37: Find the smallest no. which
Number = 193
by 4, 5, and 3 the corresponding one successive divided 5, 3
remainders are 2, 2 and 1. Ex.36: A numbe r when d ivid ed
and 7 give remainder 2, 1 and

eeYa
Ex.35: The least possible number succes sive ly by 4 and 5 2 respectively
when successively divided by leaves remainders 1 and 4
2, 5, 4, 3 gives respective re spec tive ly. When it is Sol. 5 142 2
remainders of 1, 1, 3, 1 is : successively divided by 5 and 3 28 1
(a) 372 (b) 275 (c) 273 (d) 193
geisnh
4 the respective remainder 7 9 2
Sol. The problem can be will be
1
expressed as Sol. The least number × in this
case will be determind as 1×7+2=9
2 A 9 × 3 + 1 = 28
Enak

follows
5 B 1 28 × 5 + 2 = 142
4 C 1 Remainder 4 X
Number = 142
3 D 3 5 y –1 Alternate:-
R

E 1
1–4 5 + 2
So it can be solved as ×
aryn

y=5×1+4=9
((((( E × 3) + 1)4 + 3 ) 5 + 1)2 + 1) =A 3 + 1
(where A is the re quir ed X = 4 × y + 1 = 4 × 9 + 1 = 37 ×
Les B

number) Now 7 2
+
So for the leas t possib le 5 37  (2  7)  3  1  5   2
number E = 1 (the least   
4 7 –2
positive integer) = (28 × 5) + 2
wa.th

then A= ((((( 1 × 3) + 1) × 4 + 1–3 Number = 142


3) 5 + 1)2 + 1) He re, the resp ective E x. 38: A leas t number when
[ Since at E = 0, we get a two remainder are 2, 3 successively divided by 2, 3, 5
digit number] it leaves the respective
Alternate:-
wwM

So it can be solved as remainder 1, 2 and 3. What


Successive
2 193 1 will be the remainder if this
Divisor Remainder
5 96 1 number will be divided by 7 ?
4 + 1
4 19 3 Sol. 2 53 1
3 4 1 ×
3 26 2
1 5 + 4
Number = (4 + 5) × 4 + 1 = 36 + 1 = 37 5 8 3
D=1×3+1=4
1
C = 4 × 4 + 3 = 19 5 37
B = 19 × 5 + 1 = 96 Step. I 5×1+3=8
4 7–2
A = 96 × 2 + 1 = 193 Step. II 8 × 3 + 2 = 26
1–3 Step. III 26 × 2 + 1 = 53
So Number= 193 Remainder = 2, 3 So the least number = 53

Rakesh Yadav Readers Publication Pvt. Ltd. 22

For More Visit : www.LearnEngineering.in


For More Visit : www.LearnEngineering.in

According to the question, quotient is divided by 5. We got


53 is divided by 7 then Sol.
3 + 1 a remainder of 3. What will be
remainder = 4 × the remainder if the number
Ex.39: Find the smallest no. which is divided by 65?
when successive divided by 2 + 1
Sol. 13 + 1
4, 5 and 6 give remainder 2,
1 and 1. Also find sequence Number = (1  2) 3  1 ×
of remainder if the sequence = 9 + 1 = 10 5 3
+
of divisor is reverse.
According to question, Number = (3  5)13  1
Sol. 4 146 2
10 = 8 × 13 + 1 = 105
5 36 1 Remainder = = 4

ir
6 According to the question,
6 7 1

v.iSn
Ex.42:A number divided by 13 leaves 105
1 Remainder = = 40
a remainder 1 and if the 65
6×1+1=7
7 × 5 + 1 = 36 BINOMIAL THEOREMREM

dnag
36 × 4 + 2 = 146
Number = 146 * Statement of the theorem:-
According to the question, According to the theorem, it is possible to expand any power of x +y
Now divisor is 6, 5 and 4 into a sum of the form

eYrai
n n n n n
Then successive remainder (x + y) = c 0 xn y0 + c 1 xn–1 y1 + c 2 xn–2 y2 + ..... + c n–1 x1 yn–1 + c n x0yn
n

6 146
5 24 – 2 form ula is obtaine d by
n
4 4–4
1–0
Remainder = 2, 4 and 0
Where each
snhe
specific positive integer
known as
ck

bino mial
is a substituting 1 for y, so that
it inv olve s only a s ingle
variable, In this form, the
formula reads
Ex.40: A number when divided
kgei
successively by 6, 7 and 8, it co efficient. (Whe n an n n
(1 + x)n = c 0 x0 + c 1 x1+
leaves the respective expone nt is zero, the
n
corre sponding power c 2 x 2 + ..... + nc n– 1
remainders of 3, 5 and 4, what n–1 x
ERna

will be the last remainder expression is taken to be 1


n
when such a least possible and this multiplicative factor c n xn
number is divided is often omitted from the or equivalently
successively by 8, 7 and 6. term. Hence one often sees
n
the rig ht side written as n
aBryn

(1 + x)n =  c k xk.
Sol. 6 537 3 k=0
n
7 89 5 c0 xn + ....) This formula Ex. (i) (x + y)³ = x³ + 3x²y + 3xy² + y³,
8 12 4 (ii) (x + y)4 = x4 + 4x³y + 6x²y² +
is also referred to as the
1 4xy³ + y4y
binomial formula or the
Les

binomial identity. Using Some important points


Step. I 1 × 8 + 4 = 12
1. The powers of x start at n and
wa.th

Step. II 12 × 7 + 5 = 89 summation notation, it can


be written as decrease by 1 in each term until
Step. III 89 × 6 + 3 = 537
they reach 0 (with {{{1}}} often
least number = 537 n
n
c k xn – k yk = unwritten);
Now we divide 5 37 (x + y)n = 
k=0 2. The powers of y start at 0 and in-
wwM

successively by 8, 7 and 6.
crease by 1 until they reach n;
8 537 n
n
c k xkyn–k 3. The nth row of pascal's Triangle
7 67 1 
k=0 will be the coefficients of the ex-
6 9 4 Remainder The final expression follows panded binomial when the terms
1 3 from the previous one by the are arranged in this way;
So, 3 is the last Remainder. symmetry of x and y in the 4. The number of terms in the
Ex.41:A number when divided by 3 f ir st e xp re ss ion, and b y expansion before like terms
leaves a remainder1. When comparison it follows that are combined is the sum of
the quotient is divided by 2. the sequence of binomial the coefficients and is equal
It leaves a remainder1. What coefficients in the formula to 2n, and
will be the remainder when is symm etric al. A sim ple 5. there will be (n + 1) terms
the number is divided by 6? variant of the b inom ial in the e xp re ss ion af te r

Rakesh Yadav Readers Publication Pvt. Ltd. 23

For More Visit : www.LearnEngineering.in


For More Visit : www.LearnEngineering.in

combining like terms in the Sol. (c) 341 + 782 pletely divisible by 23 so 23
expansion.  (Equalising the power) will completely divide this
The binomial theorem can  341 + (72)41
number
be applied to the powers of  341 + 4941, 3 + 49 = Com- * (an - bn) is always divisible by
any binomal. for example. (a - b) where n  odd power
(x + 2)³ pletely Divisible by 52 Hint
= x³ + 3x³ + 3x²(2) + 3x(2)² + 2³ 52 = 1, 2, 4, 13, 26, 52
So, 17 is not the factor of 52 (a3+b3) = (a-b) (a2+ab+b2)
=x³ + 6x² + 12x + 8.
* For a binomial involving sub- hence this number will be
traction, the theorem can be completely divisible by 17 * (an - bn) is always completely
applied by using the form (x Ex.45. (49)15 – 1 is exactly divisible divisible by ( a - b), (a + b)
by: where n  (even power)

r
– y)n =(x + (–y))n. This has the
effect of changing the sign of (a) 50 (b) 51 (c) 29 (d) 8 Hint

i
.iSn
every other term in expan- Sol.(d) xn – an is exactly divisible by
sion: (x – a) if n is odd. (a2 – b2 ) = (a-b) (a+b)
(x – y)³ = (x + (–y))³  (49 ) 15 – (1) 15 is e xac tly Ex.47 Which of the following will not

agv
= x³ + 3x² (–y) + 3x (–y)² + (–y)³ divisible by 49–1 = 48, that is a
divide 2310 – 1024 completly.
= x³ – 3x²y + 3xy² – y³ multiple of 8.
(a) 3 (b) 5 (c) 7 (d) 4
Ex.46: Which of the following com-

ridn
* (an + bn ) is always divisible Sol. 1024 is the value of 210
by (a + b) when n  odd pletely divide
(2947 + 2347 + 1747) and
power

eeYa
HINT (a) 21 (b) 22 (c) 23 (d) 24 2310 - 210  (23 - 2) and (23 +
2) is completely divisible
3 3 2
a +b = (a+b) (a +ab+b )
2
2947 1747 2347
Sol. (c) (23 - 2) = 21 = 1, 3, 7, 21
Ex.43:Which of the following num- 23
(23 + 2) = 25 = 1, 5, 25
ber will not completly divide
geisnh 47 47
29 + 17 will be completely
Hence this number is not
the (29)37 + (17)37 \ divisible by 46 or its factor (2
divisibe by 4.
(a) 2 (b) 11 (c) 23 (d) 46 and 23) and 23 47 is com-
Sol. (b) (29 37 + 17 37), (29 + 17)
= 46 Completely divisible by
Enak

n n n n
n
(a+b)
n
(a-b)
n n
(a-b) (a+b)
46 = 1, 2, 23, 46
n odd n odd n even n even
This will be completely divis- n n n n It can’t
is perfectly (a-b) (a-b) is perfectly
n
ible by all the factors of 46 n
(a+b)
R

is perfectly divisble be deter-


So 11 will not divide the divisble
by (a +b) divisble by mined
given number. by (a -b) (a +b),(a - b)
aryn

2 2
Ex.44: Which of the following will not 3 3 3 3
(a-b) 2 2
(a+b) (a –b ) a+b= .....
completely divide (341 + 782) ? 2 2
(a–b) (a²+b²+ab) (a+b) (a-b)
(a+b) (a-ab+b)
Les B

(a) 4 (b) 52 (c) 17 (d) 26

EXERCISE
wa.th

1. In a division sum, the divisor is is the dividend ? r es pe ctiv ely. Whe n it is


10 times the quotient and 5 (a) 169943 (b) 159963 successively divided by 5 and
times the remainder. If the (c) 159943 (d) 159953 4 the respective remainders
wwM

remainder is 48, the dividend is: 4. The sum of 20 odd natural will be:
(a) 808 (b) 5008 number is equal to : (a) 4,1 (b) 3,2 (c) 2,3 (d) 1,2
(c) 5808 (d) 8508 (a) 210 (b) 300 (c) 400 (d) 240 7. 461+462+463+464 is divisible by :
2. The divisor is 321, the quotient 5. When a number is divided by
(a) 3 (b) 10 (c) 11 (d) 13
11 and the remainder 260. 56, the remainder obtained is
29. What will be the remainder 8. (325 + 326 + 327 + 328) is divisible
Find the dividend. by :
(a) 3719 (b) 3971 when the number is divided by
8? (a) 11 (b) 16 (c) 25 (d) 30
(c) 3791 (d) 3179
3. In a division sum, the divisor (a) 4 (b) 5 (c) 3 (d) 7 9. The least number, which must
is 5 times the remainder and 6. A numb er whe n divide d be added to 6709 to make it
the quotient is 6 times the successively by 4 and 5 leave exactly divisible by 9, is
remainder which is 73. What the re mainde r 1 and 4 (a) 5 (b) 4 (c) 7 (d) 2

Rakesh Yadav Readers Publication Pvt. Ltd. 24

For More Visit : www.LearnEngineering.in


For More Visit : www.LearnEngineering.in

10. If 78*3945 is divisible by 11 20. If the number 4 8 3 2 7 * 8 is number is divided by 12 is


where * is a digit, then * is divisible by 1 1, the n the (a) 3 (b) 4 (c) 6 (d) 8
equal to : missing digit (*) is 32. The expression 8 n – 4n, where
(a) 1 (b) 0 (c) 3 (d) 5 (a) 5 (b) 3 (c) 2 (d) 1 n is a natural numb er is
11. When a number is divided by 21. A number, when divided by always divisible by
357 the remainder is 39. If 136, leaves remainder 36. If (a) 15 (b) 18 (c) 36 (d) 48
same number is divided by 17, the same number is divided by 33. (461 +462 +463) is divisible by
the remainder will be : 17, the remainder will be (a) 3 (b) 11 (c) 13 (d) 17
(a) 0 (b) 3 (c) 5 (d) 11 (a) 9 (b) 7 (c) 3 (d) 2 34. When an integer K is divided
22. Two numbers, when divided by by 3, the remainder is 1, and
12. A number when divided by 6
17, leaves remainder 13 and 11 when K + 1 is divided by 5, the
leaves remainder 3. When the remainder is 0. Of the following,

ir
re sp ec tive ly . If the s um of
square of the same number is a possible value of K is:
those two numbers is divided

v.iSn
divided by 6, the remainder is : (a) 62 (b) 63 (c) 64 (d) 65
by 17, the remainder will be :
(a) 0 (b) 1 (c) 2 (d) 3 (a) 13 (b) 11 (c) 7 (d) 4 35. A number when divided by 91
13. When a number is divided by 23. A number, when divided by gives a remainder 17. When
893, the remainder is 193. 221, leaves a remainder 64. the same number is divided by

dnag
What will be remainder when What is the remainder if the 13, the remainder will be :
it is divided by 47 ? same number is divided by 13? (a) 0 (b) 4 (c) 6 (d) 3
(a) 3 (b) 5 (c) 25 (d) 33 (a) 0 (b) 1 (c) 11 (d) 12 36. A number when divided by 280
14. A number divided by 13 leaves 24. When a number is divided by leaves 115 as remainder. When

eYrai
a r em aind er 1 and if the 387, the remainder obtained is the same number is divided by
48 . If the sam e num be r is 35, the remainder is:
quotient, thus obtaine d, is
divided by 43, the remainder (a) 15 (b) 10 (c) 20 (d) 17
divide d by 5 , we g et a
obtained will be ? 37. A certain number when divided
remainder of 3. What will be
the remainder if the number
is divided by 65 ?
(a) 28 (b) 16 (c) 18 (d) 40
(a) 0 (b) 3
25. When snhe
two
(c) 5
num be r
(d) 35

separately divided by 33, the


ar e
by 175 leaves a remainder 132.
When the s am e numb er is
divided by 25, the remainder is:
(a) 6 (b) 7 (c) 8 (d) 9
r em aind er s ar e 21 and 2 8
kgei
15. Which of the following number respectively. If the sum of the 38. Which one of the following will
is NOT divisible by 18 ? two number is divided by 33, the completely divide by 571 + 572 + 573
(a) 54036 (b) 50436 remainder will be ? (a) 150 (b) 160 (c) 155 (d) 30
ERna

(c) 34056 (d) 65043 (a) 10 (b) 12 (c) 14 (d) 16 39. Which of the following numbers
16. If n is an integer, then (n3 - n) 26. (271 + 272 + 273 + 274) is divisible will always divide a six-digit
is always divisible by : by : number of the form xyxyxy
(a) 4 (b) 5 (c) 6 (d) 7 (a) 9 (b) 10 (c) 11 (d) 13 (where 1 < x < , 1 < y < 9)?
aBryn

27. When 'n' is divisible by 5 the (a) 1010 (b) 10101


17. A 4 digit number is formed by
remainder is 2. What is the (c) 11011 (d) 11010
repeating a 2 digit number
remainder when n2 is divided 40. A positive integer when divided
such as 2525, 3232, etc. Any by 5 ? by 425 gives remainder 45.
number of this form is always (a) 2 (b) 3 (c) 1 (d) 4 When the same num ber is
exactly divisible by: 28. A number when divided by 49 divided by 17, the remainder
Les

(a) 7 only (b) 11 only leaves 32 as remainder. The will be


wa.th

(c) 13 only (d) Sm alle st 3 number when divided by 7 will (a) 5 (b) 2 (c) 11 (d) 13
digit prime number have the remainder as: 41. A number x when divided by 289
18. If two num be rs are e ac h (a) 4 (b) 3 (c) 2 (d) 5 leaves 18 as the remainder.
divided by the same divisor, the 29. When a number is divided by The same number when divided
36, the remainder is 19. What by 17 leaves y as a remainder.
wwM

remainders are respectively 3


will be the remainder when The value of y is
and 4. If the sum of the two the number is divided by 12 ?
numbers be divided by the (a) 5 (b) 2 (c) 3 (d) 1
(a) 7 (b) 5 (c) 3 (d) 0
same divisor, the remainder is 42. When n is divided by 6, the
30. In a division sum, the divisor
remainder is 4. When 2n is
2. The divisor is : is 10 times the quotient and 5
divided by 6, the remainder is:
(a) 9 (b) 7 (c) 5 (d) 3 times the remainder. If the
re mainde r is 4 6, the n the (a) 2 (b) 0 (c) 4 (d) 1
19. A number when divided by 5 43. In a division sum, the divisor
leaves remainder 3. What is dividend is :
(a) 4236 (b) 4306 is 3 times the quotient and 6
the re mainde r when the times the remainder. If the
(c) 4336 (d) 5336
square of the same number is 31. When a number is divided by r em aind er is 2, the n the
divided by 5 ? 24, the remainder is 16. The dividend is :
(a) 1 (b) 2 (c) 3 (d) 4 re mainde r when the s am e (a) 50 (b) 48 (c) 36 (d) 28

Rakesh Yadav Readers Publication Pvt. Ltd. 25

For More Visit : www.LearnEngineering.in


For More Visit : www.LearnEngineering.in

44. In a divison sum, the divisor (c) 0 (d) 5 (c) Both 10 & 20
is 12 times the quotient and 5 46. The quotie nt whe n 10 100 is (d) Neither 10 nor 20
times the remainder. If the divided by 575 is : 49. If (17)41 + (29)41 is divided by 23.
re mainde r is 3 6, the n the Find the remainder
(a) 1025 (b) 275
dividend is : (a) 1 (b) 6 (c) 0 (d) 12
(c) 275 ×1025 (d) 225 ×1075 50. If (3)41 + (7)82 always divisible by
(a) 2706 (b) 2726 47. The remainder obtained when (a) 10 (b) 49 (c) 52 (d) 44
(c) 2736 (d) 2262 233 + 313 is divided by 54 51. If m n - n m = (m + n); (m, n)
(a) 0 (b) 1  prime numbers, then what
45. For any integral value of n, 32n
(c) 3 (d) C.N.D can be said about m and n:
+ 9n + 5 when divide by 3 will
5 5
48. (19 + 21 ) is divisible by (a) m, n are only even integers
leave the remainder
(b) m, n are only odd integers

r
(a) Only 10 (b) m is even and n is odd
(a) 1 (b) 2

i
.iSn
(b) Only 20 (d) none of these

ANSWER KEY

agv
1. (c) 6. (c) 11. (c) 16. (c) 21. (d) 26. (b) 31. (b) 36. (b) 41. (d) 46. (c)
2. (c) 7. (b) 12. (d) 17. (d) 22. (c) 27. (d) 32. (d) 37. (b) 42. (a) 47. (a)
3. (c) 8. (d) 13. (b) 18. (c) 23. (d) 28. (a) 33. (a) 38. (c) 43. (a) 48. (c)

ridn
4. (c) 9. (a) 14. (d) 19. (d) 24. (c) 29. (a) 34. (c) 39. (b) 44. (c) 49. (c)
5. (b) 10. (d) 15. (d) 20. (d) 25. (d) 30. (d) 35. (b) 40. (c) 45. (b) 50. (c)
51. (c)

eeYa
SOLUTION
geisnh
1. (c) Remainder = 48
Remainder 29 = 325 (1 + 3 + 9 + 27)
Divisor = 48 × 5 = 240 5. (b) =
8 8 = 325 × 40 = 324 × 120
Enak

240 Remainder = 5 Now, check with option


Quotient =  24
10 6. (c) Numb er is divide d Only, check with option
Dividend = 240 × 24 + 48 successively Only 30 can divide this.
R

Remainder 9. (a) 6709


= 5760 + 48
4 37 1  6+7+0+9=22
= 5808 5 9 4
aryn

[9 - (divisibility property)
2. (c) Div id end = Divisor × 1
Sum of digits must be divisible
Quotient + Remainder 5×1+4=9
Les B

by 9]
= 321 × 11 + 260 9×4+1= 37
So 22 + 5 = 27 is divisible by 9
= 3531 + 260 = 3791 Number is 37
5 is answer
3. (c) Remainder = 73 5 37 2
Quotient = 6×73 = 438 4 7 3 10. (d) 78 * 3945
wa.th

1
Divisor = 5×73 = 365 Odd place : 7 + * + 9 + 5 = 21 + *
Dividend = 365 × 438 + 73 Remainder is 2, 3 Even place : 8 + 3 + 4 = 15
7. (b) 461 + 462 + 463 + 464 (21+ *) –(15)= either 11 or 0
= 159943
= 461(40 + 41 + 42 + 43)
wwM

4. (c) 1, 3, 5, 7 .......20th term (21 + *) – 15 = 11


= 461 (1 + 4 + 16 + 64) 21 + * = 26
a = 1, d = 2, n = 20
= 461 × 85 *=5
n = 460 × 4 × 85
sum = [2a + (n - 1)d] 39
2 = 460 × 340 11. (c) Remainder of number =
17 17
20 = 460 × 34 × 10
 remainder = 5
= [2 ×1+(20-1)2] Now, check with option
2 12. (d) Shorcut Method
Only, check with option
= 10[2×1 + 19 ×2] = 400 Let number is: 9 (Gives remain-
Only 10 can divide this. der 3 when divided by 6)
Alternate :
8. (d) (325 + 326 + 327 + 328)
The sum of first n odd natural 92 81
numbers = n2 = 202 = 400 = 335 (30 + 31 + 32 + 33) Now = 6  Remainder = 3
6

Rakesh Yadav Readers Publication Pvt. Ltd. 26

For More Visit : www.LearnEngineering.in


For More Visit : www.LearnEngineering.in

Remainder of no. 193 22. (c) (dividend = divisor × quotient n² 72 49


13. (b) 47
= 47 + remainder) = =  remainder= 4
5 5 5
 remainder = 5 First no. = (17 × n) +13
Let 'n' = 1 remainder of no. 32
14. (d) 28. (a) =
13 105 1 7 7
 (17 × 1) + 13
5 8 3  Remainder = 4
1  30
5×1+3=8 Second no.= (17 × n) + 11 remainder of no. 19
29. (a) =
13 × 8 + 1 = 105 = (17 × 1) + 11 = 28 12 12
 remainder = 7
remainder = 105 ÷ 65 According to question
30. (d)
Remainder = 40

ir
30  28 58 Quotient : Divisor : Remainder
15. (d) A number will be divisible by =  remainder = 7

v.iSn
18 if it is divisible by 2 and 9 17 17 1 : 10

Clearly we can see 65043 is not Alternate:- 5 : 1


divisible by 2. Because unit Divisor = Rem aind er 1 + 1 : 10 : 2
digit of 65043 is 3 so this will Remainder 2 – Remainder 3

dnag
not be divsible by 18 ×23 ×23 ×23
17 = 13 + 11 – Remainder 3
16. (c) (n³ – n) and n is any integer. 23 : 230 : 46
Remainder 3 = 24 – 17 = 7
put n = 2 so, 2³ – 2 = 6 Dividend = ( Divisor × Quotient)
+ Remainder

eYrai
It will be always divisble by 6 Remainder of no. 64
23. (d) = = (230 × 23) + 46= 5336
( Put n = 2,3,4...) 13 13
17. (d) Smallest 3 digit prime num-  remainder = 12 Remainder of no. 16
31. (b) =
ber is '101' 12 12

18.
xyxy is always divisible by 101
Hence, 101 Will be the divisor.
(c) Shortcut Method
24. (c)


43
snhe
Remainder of no.

remainder = 5
48
= 43 = 4 is remainder
32. (d) 8n – 4n
n = 1,2,3..............(n is a natu-
kgei
ral number)
divisor = Rem aind er 1 + 25. (d) first no. = (33 × n) + 21
Put, n = 2,
Remainder 2 – Remainder 3 Let no = 1 expression = 8² – 4² = 64 – 16 = 48
= 3 + 4 – 2 = 7 – 2= 5 = (33 × 1) + 21 = 54
ERna

 8n – 4n is divisible by 48
19. (d) Let no. be 8 Second no.= (33 × n) + 28 48 is completely divisible by 4
2
8 64 = (33 × 1) + 28 = 61 so fn is divisible 4
 = 5 33. (a) (461 + 462 + 463)
5 According to question
aBryn

= 4 remainder = 461(40 + 41+ 42)


54  61 115 = 461(1 + 4 + 16) = 461 × 21
Alternate:- 
33 33 Now check the options
2
Remainder Only 3 divides it. So '3' is answer
Remainder =  16 Remainder
5 34. (c) Always do these types of
Alternate:-
Les

2
question by options to save time
3 9 Divisor = Rem aind er 1 +
=   =
wa.th

=4 Pick up the option and follow


5 5 Remainder 2 – Remainder 3 the question instruction
20. (d) 48 3 27 * 8 33 = 21 + 28 – Remainder 3 take option (c)
odd place  4 + 3 + 7 + 8 = 22 Remainder 3 = 16 64  Divide 3 it gives remain-
der 1
wwM

Even place  8 + 2 + * = 10 + * 26. (b) (271 + 272 + 273 + 274)


Now add 1 to 64
Difference should be either = 271(20 + 21 + 22 +23)
zero or 11,22,33 ................etc. = 271(1 + 2 + 4 + 8) 65
 remainder '0' it satisfies
 22 – (10 + *) = 11 = 271× 15 = 270×30 5
So, k = 64 this is answer
22 – 10 – * = 11 It is divisible by 10
12 – * = 11 Remainder of no. 17
n 35. (b) =
*=1 27. (d)  remainder 2 13 13
5 remainder = 4
Remainder of no. 36 If we put n = 7 Then it satisfies
21. (d) = Remainder of no. 115
17 17 above situation 36. (b) =
35 35
 remainder = 2 So n = 7 Remainder = 10

Rakesh Yadav Readers Publication Pvt. Ltd. 27

For More Visit : www.LearnEngineering.in


For More Visit : www.LearnEngineering.in

Remainder of no. 132 43. (a) 47. (a) We know that (an + bn) is
37. (b) = Remainder : Divisor : Quotient
always divisible (a + b) then.
25 25
where n  odd power
remainder = 7 3 : 1
(233 + 313) is Always divisible
38. (c) 571 + 572 + 573 1 : 6 by (23 + 31) = 54
= 571 (50 + 51 + 52) So remainder is '0'
1 : 6 : 2
= 571 (1 + 5 + 25) 48. (c) (a n + b n ), is alway s
= 571 × 31 = 570 × 155 ×2 ×2 ×2
divisible by (a + b)
Check with option, Actual 2 12 4 When n  odd power
So 155 is answer Dividend = (Divisor × Quotient) (19 + 21) = 40
39. (b) Number = xyxyxy + remainder Factor of 40 (1, 2, 4, 5, 10, 20,

r
= xy × 10000 + xy × 100 + xy 40) is divisible by (195 + 215)
= (12 × 4) + 2 = 50

i
.iSn
= xy (10000+ 100 +1) then options 1 0 & 20 is
= xy(10101) 44. (c) divisible
Hence, option (B) will divide an- 49. (c) (a n + b n ), is alway s
Remainder : Divisor : Quotient
swer divisible (a + b)

agv
12 : 1
Alternate: 1 : 5 When n is odd power
You can as sum e (1 2121 2, 12 : 60 : 5
Then,

ridn
343434..........) any number divis- ×3 ×3 ×3 (1741 + 2941) is always divisible
ible by option, So that number
by (17 + 29) = 46

eeYa
is divisible by exactly that's the 36 180 15
answer factor of 46 (1, 2, 23, 46)
Dividend = (divisor × quotient) + Re- So, (1741 + 2941) is perfectly
Remainder of no. 45
40. (c) = mainder divisible by 23
17 17
hence, Remainder '0'
geisnh
= (180 × 15) + 36
 remainder = 11
= 2736 50. (c) 341 + 782
Remainder of no. 18 Equalising the power
41. (d) = 45. (b) 32n + 9n + 5
17 17
Put n = 1 341 + (72)41 = 341 + 4941
 remainder = 1
Enak

 32×1 + 9× 1 + 5 341 + 4941 is always divisible


n (3 + 49) = 52
42. (a) 6 = remainder 4  9 + 9 + 5  23
So 52 is divisible by (341 + 782)
23
R

10   remainder = 2 51. (c) mn - nm = m + n


If n = 10  3
6 Consider m = 2 and n = 5,
Note: value of n can be 1,2,3,4,
aryn

 remainder = 4 (matched) n = 10 then


.............
20 2 5 - 52 = 5 + 2
Les B

100
2n = 2 × 10  46. (c) 10 ÷ 575
6 7=7
2100 5100 100 25 25 75 25
 remainder = 2 2 ×5 2 .2 .5 Thus option (a) and (b) are
575 wr ong and op tion (c) is
Note : Always put value in these type
= 2 75  1025 correct.
wa.th

of questions.
wwM

Rakesh Yadav Readers Publication Pvt. Ltd. 28

For More Visit : www.LearnEngineering.in


For More Visit : www.LearnEngineering.in

CHAPTER

05
REMAINDER THEOREM
Ex:- What remainders can be Ex:- + 6 -1 Ex:- + 13 -2
possible when 25 is divided by 7

ir
+4 -3

v.iSn
55 13
25 7 15
7 Ex:- + 5 -1
15 13 0 15 13 1
7 25 3 0 or 15

dnag
21 167 13 -2
Actual
+4 Remainder 6 * When 13 is divided by 15, then
* '0' is the smallest divisible the multiple of 15 which is less
7 25 4 number when 0 is divided by

eYrai
than 13 is 0. which is 0 th
-28 any number always remainder multiple of 15. Hence actual
or -3 Negative will be 0 remainder will be +13 and for
Remainder
Ex:- 0 7 0 0 the neg ative remainder we
Remainder is always positive but
have to see the multiple of 15
some times we use negative
remainder for our convenience if 25
is divided by 7 then actual remainder
*
7 -0
0 snhe
when 0 is divided by 7, then 0th
multiple of 7 is (7 × 0 = 0) then
which should be greater than
13, Now 15 is the multiple of 15
greater than 13, so remainder
kgei
will be + 4 but – 3 can be used for 0 is subtracted from 0, we will will be - 2
convenience for actual remainder get zero.
Ex:- When 45 is divided by 14 then Ex:- + 2 -1
multiple of 7, less than 25 is 21 hence
actual remainder will be + 4 and for
ERna

14 45
negative remainder we have to see the 3 2
-42
multiple of 7 greater than 25, which is +3 Actual or 3
28 so – 3 will be the remainder Remainder
3 2 0
aBryn

Ex:- What will be the remainder 14 45


4 -0
when 37 is divided by 9 -56 +2 Actual
- 11 Negative Remainder
9 37 4 Remainder
-36 +3 -11 3 2 1
+1 Actual or -3
Les

Reminder Negative
-1
Remainder
wa.th

45
9 37 5 14 Ex:-
-45
-8 Negative Ex:- + 1 -7 +4 -3
Remainder
wwM

When 37 is divided 9, then the 4


73
multiple of 9 smaller than 37 is 36. 8 7
Hence actual remainder will be +1
Ex:- + 3 -1 7 4 0
It we want a negative remainder we 0
have to see the multiple of 9 greater +4
than 37 which is 45, hence – 8 will 111
7 4 1
be the negative Remainder. 4
-7
0 -3
+1 -8 Ex:- , Remainder = 0
100 TYPE – 1
100 0 0
37 0 Ex.1 what will be the remainder
9 0 Remainder when 23 × 34 is divided by 9

Rakesh Yadav Readers Publication Pvt. Ltd. 29

For More Visit : www.LearnEngineering.in


For More Visit : www.LearnEngineering.in

Alternate II Sol.
Sol. 23  34
-4 -2 43  83
9
23× 34 21
when 23 is divided by 9 the 9 = -4×- 2 =8
remainder is 9 +1 -20 + 20 -1

+5 -4
+5 -4 +7 -2 43 83
21 21
23 Wheather remainder is + ve
9 23 34 or ne gative smaller remainder
9 9 should be used for the e asier
When 34 is divided by 9, the
calculation. If 43 is divided by 21, the

r
remainder is Now this time we have used smaller remainder will be (+1) and
negative remainder. If 23 is divided

i
.iSn
+7 -2 If 83 is divided by 21 the smaller
by 9, the remainder will be -4 and if remainder will be – 1,
34 is divided by 9, the remainder
34 will be - 2 As there is (×) sign in the
+1 -1

agv
9 pr oc es s, the p roduct of the 1  1
  1  21
remainders is (+8) As the product 21
is less than divisor so there is no 43 × 83

ridn
+5 +7 need to divide it again.  20
then =5×7= 35 Ex.3 What will be the remainder
Alternate III

eeYa
34 121  93
23 × when
9 8
-4 +7 +1 -7 +5 -3
The sign will be the same
23 × 34
be twee n re mainde rs as in the
geisnh
process. For Ex (23 × 34). Here we 9 121 93
see that the sign b/w 23 and 34 is 8 8
= -4 × 7 = - 28
(×), So, the sign b/w remainders By using smaller remainders
Af te r dividing 2 3 by 9
will b e (×). I f the pr od uc t of remainder - 4 has taken and after
Enak

remainder is greater than divisor, dividing 34 by 9, remainder (+7) has -3


+1
we have to divide it again to get the take n. N ow the p roduct of the 1 3
remainder re mainde rs are (-2 8). We will   2
121 + 93 8
R

In this process when 23 is neglect the (-ve) sign and again will 8
divided by 9, remainder +5 has been get the remainder by dividing fist
used and when 34 is divided by 9 process. After that we will put (-ve) =8–2=6
aryn

remainder +7 has been used we can sign. It the remainder is negative, In this operation we have used
se e that the s ig n be twee n the then we will get (+ve) remainder by (+ve) sign. So the same sign (+) will
Les B

proces is (×), then the product of adding divisor into it. be used b/w the remainders. (1 –3) = (–2)
remainders is (5 × 7) = 35, the remainder is (-ve). So, to get
Which is greater than 9. Now actual remainder we have to add 8
again we have to divide 35 by 9 we -4 +7
hence actual remainder will be 6.
wa.th

will get + 8 or - 1 as remainder. If


23 × 34 Ex.4 What will be the remainder
the remainder is negative (-1) it = - 28
should be deducted from divisor, so 9 130  147
we will ge t positive (+v e) Negelecting (-ve) sign when
11
wwM

remainder) +1
Sol. +9 -2 +4 -7
= 28 Now dividing by general
+5 +7 9
process 130 147
23 × 34 = 5× 7 11 11
9 9 = +1 (Again putting (-ve) sign)
By using smaller remainder
=–1=9–1
+8 -1 (To get (+ve) remainder) = 8 -2 +4
2  4
35
Same Remainder in each process 130 + 147  2
= = +8 (Remander) 11
9 Ex.2 What will be the remainder 11
or
9 -1 = 8 when 43 × 83 is divided by 21? So, remainder is 2

Rakesh Yadav Readers Publication Pvt. Ltd. 30

For More Visit : www.LearnEngineering.in


For More Visit : www.LearnEngineering.in

Ex.5 When 127 × 139 × 12653 × 79 Ex.10 When 1+ 2 + 3 + 4 + 5 ..... + 1000


4 = 1×2×3×4 = 24
× 18769 is divided by 5, the is divided by 10 the remainder 12
remainder will be. will be = Remainder = 0
+2 -1 -2 -1 -1 Hence all the factorial next
It is sign of Factorial. to will be completly divisible by 12
1 1
127 139 12653  79 18769 So, '0' will be the remainder is each
2 1×2=2
5 case
3 1×2×3=6
4 1 × 2 × 3 × 4 = 24 1 2  6 9
Divisiblity of 5 can be examined by 5 1 × 2× 3 × 4 × 5 = 120   9
dividing the last digit of the number 12 12
O value is 1
Remainder = 9
2  1 2  1 1 4

ir
  4 +1 –9 Ex.12 Which of the following will
5 5 1 completly divide

v.iSn
1 =
Hence remainder is 4 10 1+ 2+ 3+ 4+ 5+ 6.........1000
1
Ex.6 What will be the remainder +2 –8 (a) 10 (b) 9
when 127  139  12653  79 2 (c) 12 (d) 8
2 = 1×2 =
10

dnag
 18769 is divided by 5 Sol. In such type of question you
+6 –4 can take the help of
Sol. 6
3 = 1×2×3 = Options to save y our
+2 –1 –2 –1 –1 10 valuable time

eYrai
127  139  12653  79  18769 +4 -6 Option 'b'
5 4 = 1×2×3×4 = 24 +1 +2 -3 -3 +3 0 0 0
10
2 –1 – 2 –1–1 –3 1+ 2+ 3+ 4+ 5+ 6+ 7..... 1000
1
= = –3 120

= 5–3=2
5 5

Ex.7 What will be the remainder


when 195×1958×1975×170 is
snhe
5 = 5×4×3×2×1= 10
=remainder =0
Value of 5 is 120 which is completly

9

1 2  3  3  3
9
0
kgei
divided by 19. divisible by 1 0, Hence the Hence 0 is the remainder
reamainder (In the same way) will Hence this numb er is
+5 +1 –1 –1 be 0. divisible by 9
ERna

= 195 ×1958 ×1975 ×170 In the same way


19 * The number is divided by 10 to
6 = 6×5×4×3×2×1 = 720 get unit digit
5 1 –1 –1 10 * The number is divided by100 (10 )
2

= =5 = Reamainder = 0 to get last two digits


19
aBryn

* The number is divided by[(10)3] to


Ex.8 What will be the remainder * 7, 8......... 1000 is divided by 10, 0
get last three digit
when 1750 × 1748 × 1753 × will be the remainder in each case. * This process will continues as it is
70 × 35 is divided by 17 So by using smaller remainder
Sol. –1 –3 +2 +2 +1 +1 +2 -4 +4 0 0 0 Last Two Digit ( v fU
re 2 v a
d)
Les

1+ 2+ 3+ 4+ 5+ 6.......... 1 000
1 Ex.13 Find the last two digit of the
1750 1748 1753  70  35 product
wa.th

10
17
23 × 13999 × 497 × 73 × 96
–1 –3  2  2 1 1 2 4  4 3
= 12   = 3 Sol. This num be r should b e
17 10 10
divided by 100 to get last two
Ex.11 What will be the remainder
wwM

Hence Remainder is 12 digit.


when 1+ 2+ 3+ 4+ .......... 1000
1
Ex.9 What will be the remainder
is divided by 12 ? 123×13999×497×73×96 24
when (1750 + 1748 + 1752 +
70 + 35) is divided by 17 ? 1 -11 100
25
Sol. –1 –3 +1 +2 +1 1=1 = 1 In such type of process we
12 simplify the operation firstly.
1750  1748  1752  70  35 +2 -10 The number by which we
17
2 = 1×2 = 2 simplify the operation, the
12 same number is multiplied in
–1 – 3  1  2  1 0 +6
= = =0 -6 the last. In this case 96 and
17 17 100 are simplified. So, we
Hence remainder is 0 3 = 1×2×3= 6
12 multiply by 4 in the last

Rakesh Yadav Readers Publication Pvt. Ltd. 31

For More Visit : www.LearnEngineering.in


For More Visit : www.LearnEngineering.in

Ex.16 87× 92 × 194 × 44 Find the Sol. In such type of Operations,


-2 -1 -3 -2 -1
last two digits ? the power is simplified in such a way
23 that the difference b/w divisor and
123×1399×497×73×24
87×92×194×44 the number made by breaking of
25 Sol. power is minimum So, the number
100
25 near to 9 should be 8 or 10
12
  12 Simplifying by 4 8
25 9
-13 -2 -6 -6 10
 25  12  13
To ge t last two d ig it we 87×23×194×44 -1
3 6
multiply it by 4. 25 23 6

 
2 6
(8)

r
13 × 4 = 52 9 9 = 9
13  2  6  6

i
.iSn
So, the last two digit is 52
25  1
1
6
( 5 and 2)  1 
* divisibility of 25  last 2 +1 +11 9 9
So Remainder = 1

agv
digits divisible by 25
26×36
Ex.14 39 × 55 × 57 × 24 × 13872 × Ex.21 What will be the remainder
25
when 221 isadivided by 9

ridn
9871 Find the last two digits
1  11
 11 -1
11 6 25 7
221  2 
3
39×55×57×24×13872×9871

eeYa
Sol. = So, Actual last two digits Sol.  (8)
7

100 20 9 9 = 9
5 =11×4 = 44
7
Simplifying two times by 4 Ex.17 What will be the remainder

 1
 1 = 9 - 1 = 8
and 5. So, to get last two digit when 25 isadivided by 13 9
geisnh
we have to multiply 20 (4 × 5) Sol. 12 -1 = -1 Ex.22 What will be the remainder
when 222 is divided by 9
-1 +1 +2 +1 +2 +1 25
13 -1 +2
= 13 - 1 = 12 3 7
39×11×57×6×13872×9871 222  2   2
Enak

5 Remainder is always positive Sol.   (8)7 ×2


9 9 = 9
48
* divisibility of 5  last 1 digit  25 
R

7
Ex.18 Find the remainder ?  1 2
divisible by 5 13 
-1 9
1  1  2  1  2  1 4
aryn

   4 48 1  2 2
5 5 (25)   1  1    2
48

9 9
Les B

 5  4 1 13
In such type of operations we try  92  7
So, actual last two digits to get the multiple of divisor near to Ex.23 What will be the remainder
1 × 20 = 20 (2 and 0) the dividends actual number So, that when (35)37 is divided by 9 ?
the difference b/w then will be 1. In
Ex.15 173 × 192 × 99 × 96 find the
wa.th

37
this case the multiple of 13 near to
Sol.
 35 
last two digits 25 is 26. and the difference b/w 25 9
and 26 is 1 and power of even So, the
173×192×99×96 24 The multiple of 9 near to 35
Sol. remainder will be (+ve) is 36
100
wwM

25 13

Ex.19
 36 Find the remainder?
-1
Simplifying by 4 37
1
7 37
  1
(35)
-2 -8 -1 -1 +1 9
9
173×192×99×24 (36)13   113  1
Remainder = 9 - 1 = 8
Sol.
25 7 Ex.24 What will be the remainder
In this operation the multiple when 740 is divided by 400
–2811 16 of 7 near to 36 is 35 and the difference Sol.
  = 16
25 25 between 36 and 35 is 1 71 = 7
72 = 49
So, Actual last two digit 218
Ex.20 Find the remainder? 73 = 343
= 16 × 4 = 64 (6 and 4) 9 4
7 = 2401

Rakesh Yadav Readers Publication Pvt. Ltd. 32

For More Visit : www.LearnEngineering.in


For More Visit : www.LearnEngineering.in

4 10 20 5
7 40 7  Sol. 64
If
8
is simplified by 4 we get
2
.
400 400 65 Now 5 is d iv id ed b y 2 we g et
10 10 66

 2401 
1 1
remainder 1, In means that the
400 400
6
2 =64 divisor should be multiplied by
(power has broken in such a remainder to get actual remainder
way that 74 = 2401, which is
near to the 2400 a multiple 268 5
  1 4  4
of 400) 2
65
Ex.25 What will be the remainder
when 242 is divided by 33 11
(Actual remainder)
22   26 

ir
32  Ex.30 What will be the remainder
242
when 235 is by 10 \

v.iSn
Sol. 33 65
33 34
32 and 34 are near to the 33 +4 -1 235 235 34

the difference is 1. Hence 4   1


11
Sol.   2×2
11
 10 25 2×5
Power is to be broken in such 4 × (64)

dnag
= 65
a way that we can a get 32 65 This Fraction is simplified by 2
and 34
4  1 4 17
   22 
2 =2
1
65 65 234
 

eYrai
2
2 =4 5 5
23 = 8 remainder = 65 - 4 = 61
24 = 16 Ex.28 What will be the remainder -1
5
2 = 32 when 419 is divided by 33


22  240 4   2 
33

33
5 8

Sol.
419
33
snhe 33
34
32
(4)
= 5

 1
17

17

419 = (22)19 = 238   1 = 5 - 1 = 4


kgei
+4 -1 5
238 5
Actual Remainder = 4 × 2 = 8
4 ×(32)
8 So  2 = 32
33
ERna

33 As this number was simplified


5 7 by 2, So to get actual remainder
4   1
4 1
8 3
2 2 35 8  2 
 4    we have to multiply it by 2
33 33 33 33
aBryn

remainder = 4 +8 -1 Ex.31 What will be the remainder


Ex.26 What will be the remainder 8   1
7 when 5500 is divided by 500
7
when 355 is divided by 82  8 ´ (32) =
33 5500 53  5497
Sol. 33 Sol. 
31 = 3 500 125  4
8  1
Les

2
3 = 9   8 3 497
33 = 27 33 5 ×5
=
wa.th

4 53×4
3 = 81
81 remainder =33 - 8 = 25 53 = simplifying by 125
355 33 352
 82
82 82 83
TYPE - 2 +1
Ex.29 When 20 is divided by 8 the
wwM

497 497
33   34 
13 remainder will be  (5 ) =
(+ 1) =1
 Sol. 8 20 2 4 4
82
-16 Actual Remainder = 1 × 125
4 Remainder = 125
+27 -1
+1 Ex.32 What will be the remainder
27 ×(81)  27  1  27 5 when 37100 is divided by 7 ?
= 20 5
82 82 = = =1
8 2 2 2
remainder = 82 - 27 = 55 
33
Ex.27 What will be the remainder When 20 is divided by 8 we Sol.  37 
100
 2 
100
2  23 
 
when 268 is divided by 65 ? get '4' remainder. 7 7 7

Rakesh Yadav Readers Publication Pvt. Ltd. 33

For More Visit : www.LearnEngineering.in


For More Visit : www.LearnEngineering.in

you are seeing that after three steps


58 5  5  5  5  5  5  5  5
3
2 =8
the cycle of remainders is repeating, 
2100 remainder is far greater which is generaly know as 'Pattern 7 7
than 7, So, we have to divide method So break the power of 3 3 3 3
remainder again. multiple of 3    
77 75 2 25  25  25  25 9  9
+2 +1 11 11  11   4
 7 7
 2×(8)
33
7 7
So we see that the cyclic period of
7 remainder is 6, since after 6 steps
1 2
33   the remainder start repeating Now
2   1 25
 113
  121 we divide the power by 6

r
7 
7 5622  53
5135  

i
.iSn
2  1 2 = 
=  25
(1)  2 1  2 7 7
7 7  2
7 7 1
So, Remainder = 2
Ex.34 Find the remainder when 5135 

agv
Cyclicity:- Happening again and 22

again In the same order or period


is divided by 7. 56
   125
Sol. The Remainder when 5 1 is =
7

ridn
Ex.33 Find the remainder when
1
1177 is divided by 7 5 22
divided by 7 =  R 5 . 1 6 6
11 77 7  

eeYa
Sol. 7 7
7 4
Remainder = 6
The Remainder when 11 1 is 
52 25 Ex.35 Find the Remainder when
11  R4 143321 is divided by 5
divided by 7 =  4 7 7 Sol. when 143 is divided by 5 we
7
geisnh
6 get remainder 3 thus 143321
4 4  is divide d by 5 the n
  53 125 re mainde r 3 321 . this
11  11 4  4 16  R6
112    2 7 7 re mainde r is v er y larg e
7 7 7
Enak

divisor so again divided


54 5  5  5  5
4 4 4  321 321

   7 7 143  3
11  11  11 4  4  4 64 5 5
R

113    1 3  3
7 7 7   The remainder when 3 1 is
= 25  25 9 31
 R 2
aryn

4 4 4 4 7 7 divided by 5  R3
    5
11  11  11  11 4  4  4  4 55 5  5  5  5  5
Les B

114   =  31
7 7 7 7  R  3
5
2 2 3 3 5 32 9
      R4
16  16 25  25  5 9  5 5 5
wa.th

4 =  R 3
7 7 7 33 27
 R 2
4 4 4 4 4 56 5  5  5  5  5  5 5 5
= 
     7 7 34 81
1111111111 4  4  4  4  4   R 1
wwM

115   –3 – 3 – 3
5 5
7 7
   –27 35  243  R  3
2 2 4 = 25  25  25 = = –6 5 5
   7
7 36 729
16  16  4 4  4  R4
  2  R = –6 + 7 = 1 5 5
7 7 4  80
57 5  5  5  5  5  5  5 3321 3  3
4 4 4 4 4 4  
7 7 5 5
     
11  11  11  11  11  11 1 3
116  3 3 3 2
7  
    80
64  64 1  1 25  25  25  5 9  6
 5 
 34   3
  1
7 7 7 7 5

Rakesh Yadav Readers Publication Pvt. Ltd. 34

For More Visit : www.LearnEngineering.in


For More Visit : www.LearnEngineering.in

80 f (1) = 0

 1 1 3
3
3

1
 f (x) = x29 - x26 - x23 + 1
5 5 10

Remainder = 3 
 8  4 f(1)= 1 - 1 - 1 + 1 = 0
7 f (1) = 0,
Ex.36 find the remainder when 36773
divide by 80 we can say (x - 1) is a factor
10

6773 
1 4

1 4 of f (x)
3 7 7 x+1=0
Sol. 
80 x=-1
Remainder = 4
we know that = 34 = 81 x29 – x26 - x23 + 1
Ex.38 What will be the remainder -1-1+1+1=0
36773 36772  31 when [48 + (62)117] is divided (x + 1) is a factor of f (x)

ir
80 80 by 9 ? Both (x + 1) & (x - 1) is a factor
Sol. of x29 - x26 - x23 + 1

v.iSn
4 1693
+3 -1

3   31 Ex.42 If ( x – 2) is a factor of
48+(62)
117 Polynomial x2 + kx + 4. Find
80
9 the value of k.
Sol. (x - 2) is a factor of x2 + kx+4

dnag
1 117
3   1
3 1 2 when (x - 2) = 0
    =2
1693 9 9 9 x=2

 81  31 f (2) = (2)2 + 2k + 4 = 0
80 Hence Remainder = 2
2k = – 8

eYrai
Ex.39 when [ 51 + (67)99 ] is divided k = –4
1693


1 13
3
3

by 68, find the remainder Ex.43 If (x + 1) & (x – 1) are the
80 80 -17 -1 Factor of the Polynomial ax3
Hence Remainder = 3 + bx2 + 3x + 5. find the value
Ex.37 Find the R em aind er

32 32
32
when divided by 7.
of
Sol.


68 snhe
51+(67)99

17   1
99


17  1 18

Sol.
of a and b
If (x – 1) is factor of f(x)
then,
x–1=0
kgei
32
32 68 68 68 x=1
Sol. 32   18 f(x) = ax3 + bx2 + 3x + 5
7 f(1)=a(1)3 + b(1)2 + 3(1) + 5 =0
Remainder = 68 - 18 = 50
a + b = –8 ..... (i)
ERna

32 32


4  Remainder of Algebraic Function If (x + 1), is a factor of f (x)
7
Whe n F (x) is d ivided by Then,
(x-a) the remainder is F (a) (x + 1) = 0
4 = 22
 (x - a) is a factor of F (x) x=–1
aBryn

232 32


2  then f (a) = 0 f(–1) = a (–1)3 + b (–1)2 + 3 (-1)
Ex.40 Is (x – 2) a factor of f (x) +5=0
7
x2 + x - 5 ? –a+b–3+5=0
64 32 Sol. (x - 2) = 0 –a + b = –2

2  a - b = 2 ..... (ii)
x=2 from (i) & (ii)
Les

7
x value f (x) aa =-3 , b = -5
wa.th

 23  8
F (2) = (2)2 + (2) - 5 Ex.44 Find the remainder when x 3
32
263
 21  =4+2-5 + 5x2 + 7 is divided by (x - 2)
 =6-5=1  0 Sol. x–2=0
7 x=2
(x - 2) is not a factor of
wwM

1 x2 - x + 5 f (x) = x3 + 5x2 + 7
 If F (2) = 0, we can say (x - 2), f (2) = (2)3 + 5 (2)2 + 7
32 = 8 + 20 + 7 = 35

 2  3 21
 21    8  21
 21 
32
it is a factor of f (x)
Ex.41 x29 - x26 - x23 + 1
Remainder = 35
7 7 Ex.45 Find the remainder when x2
(a) (x - 1) but not (x + 1)
– 7x + 15 is divided by x – 3
32 (b) (x + 1) but not (x - 1) Sol. x–3=0

1  2 
232
(c) both (x + 1) & (x - 1) x=3
7 7 (d) Neither (x + 1) not (x - 1) Put the value of x = 3
Again  23  8 Sol. (c) If (x - 1), is a factor then, F (x) = x2 – 7x + 15
f(x) = 0, F (3) = (3)2 – 7 (3) + 15
10
230  22  2   4
3
and x – 1 = 0 = 9 – 21 + 15 = 3
 x=1 Remainder 3
7 7

Rakesh Yadav Readers Publication Pvt. Ltd. 35

For More Visit : www.LearnEngineering.in


For More Visit : www.LearnEngineering.in

Ex.46 x51 + 16 when divided by x + 1 +4 +4


find the Remainder. =13 444 34
39
Sol. (x + 1) = 0 10+102 4+4 8
54 = =
x = -1 6 6 6
52
f (x) = x51 + 16 2 Remainder Remainder = 8 - 6 = 2
f(-1) = (-1)51+16=-1 + 16 = 15 Ex.50 What will be the remainder
Remainder = 15 when 123456789 is divided +4 +4 +4
Ex.47 If x2 + 4x + k when divided by by 8 ? 101+102+103
x - 2 leave remainder 2x. Sol. (Divisibility Rule) = 4+4+4 = 12
6 6 6
find the value of k. 21 = 2 Last digit divisibile
Sol. x2 + 4x + k by 2 Remainder = 0

r
x-2=0 22 = 4 Last two digits The remainder will be zero

i
.iSn
x=2 divisibile by 4
f (x) = 2x (0) after each three number So the
23 = 8 Last three digits
f (2) = 2 + 2 = 4 remainder is 0 upto the 99 th term.
divisibile by 8
f (2) = (2)2 + 4 ×2 + k = 4 24 =16 Last four digits So the remaining 10100 term will be

agv
4+8+k=4 divisibile by 16 divided by 6 to get the remainder
25 = 32 Last five digits

ridn
K = -8 divisibile by 32 +4 +4 +4 +4 +4 +4
So, for the divisibility of 8 the
TYPE – 3 last three digit of the number

eeYa
101+102+103 + 101+102+103 ...
Ex.48 777777........... 129 Times is should be divisible by 8. In
this way we get 5 as the +4 +4
divided by 37 the remainder
remainder
will be ? ..... 1099+ 10100
Sol. If any number is made by 123456789
geisnh
repeating a digit 6 times the 8 6
number will be divisible by
7, 11, 13 and 37. 8 789 98
72
So, 777777 ........... 126 times +4
69
Enak

is divisible by 37 because 64
126 is the multiple of 6. So, 5 Remainder 10 100
=4
the remaining three digits 6
So, the remainder is 5.
R

will be divided by 37 to get the Ex.51 What w i l l b e r e m a i n d e r


remainder Hence, the remainder is 4.
w h e n 123456789101112
aryn

13141516 divided by 16. Ex.53 What will be the remainder


77777777......126 Times,777
 Sol. For the divisibility of 16, the when 101 + 102 + 103 + ........
37
last four digits of the number 1032 is divided by 6 \
Les B

should be divisible by 16. In


37 777 21 777
=0 this way we ge t 12 as
74 37 Sol. +4 +4 +4 +4 +4 +4
remainder
37
37 Remainder =0 12345678910111213141516 101+102+103 +.... 1030+1031+1032
wa.th

16 6
Hence , the numb er is 16 1516 94
divisible by 37. 144 0 will be the remainder of ter
Ex.49 When 444444444 is divided by 76 each three term. So, o will
wwM

13 the remainder will be ? 64 be the remainder up to 30th


Sol. 4 is repeating 9 times in this 12 Remainder term
number As we know that Hence the remainder is 12.
any numb er r ep eating 6 Ex.52 101 + 102 + 103 + ........ 1099 + +4 +4
times is divisible by 13. So 10100 when divided by 6, the
31 32
the remaining three digit remainder will be \  10 +10
will be divided by 13 to get the 1 2 3 99 100 6
will be divided by 13 to get the Sol. 10  10  10  ........10  10
remainder 6
+4 44 8
  =2
444444,444 444 6 6
 10 = Remainder = 4
13 13 Hence remainder is 2
6

Rakesh Yadav Readers Publication Pvt. Ltd. 36

For More Visit : www.LearnEngineering.in


For More Visit : www.LearnEngineering.in

EXERCISE
1. Find the Remainder when 77 18. The Remainder when (2) 243 is 33. Find the R emainde r when
× 85 × 73 is divided by 9 divided by 32 is 1+ 2+ 3+ 4....... 50 is divided
(a) 1 (b) 2 (c) 4 (d) 7 (a) 8 (b) 4 (c) 10 by 12 is
2. Find the Remainder when 273 (d) None of these (a) 2 (b) 8 (c) 7 (d) 9
+ 375 + 478 + 657 + 597 is 19. Find the Remainder when (59)28 34. Find the Remainder when 9111
divided by 25 is divided by 7 is divided by 11
(a) 5 (b) 10 (c) 9 (d) 8 (a) 2 (b) 4 (c) 6 (d) 1 (a) 2 (b) 9 (c) 7 (d) 6
3. Find the Remainder when 1330 20. Find the Remainder when 4177 35. Find the Remainder when 52450

ir
× 1356 × 1363 × 1368 × 1397 is is divided by 17 is divided by 126
divided by 13 (a) 2 (b) 1 (c) 6 (d) 4 (a) 5 (b) 25 (c) 125 (d) 1

v.iSn
(a) 7 (b) 9 (c) 11 (d) 8 21. Find the Remainder when 2 49 36. Find the Remainder when 401012
4. Find the Remainder when 2327 is divided by 7 is divided by 7
+ 2372 + 2394 + 4624 + 4650 (a) 1 (b) 2 (c) 3 (d) 4 (a) 5 (b) 4 (c) 3 (d) 2
is divided by 23 22. Find the Remainder when (51203 37. Find the Remainder when 101

dnag
(a) 12 (b) 14 (c) 13 (d) 10 + 249 )is divided by 17 + 10 2 + 10 3 + ......... 10 100 is
5. Find the Remainder when 6732 (a) 4 (b) 5 (c) 6 divided by 6
is divided by 68 (d) None of these (a) 4 (b) 6 (c) 2 (d) 3
23. Find the R emainde r when

eYrai
(a) 67 (b) 66 (c) 1 (d) 0 38. Find the Remainder when 10 1
6. Find the Remainder when 9999 1234567891011121314is + 102 + 103 + ......... 101000 + 101001
is divided by 100 divided by 8 is divided by 6
(a) 99 (b) 98 (c) 1 (d) 3 (a) 4 (b) 2 (c) 6 (d) 3 (a) 4 (b) 6 (c) 2 (d) 3
24. Find the R emainde r when
7. Find the Remainder 197 130 is
divided by 196
(a) 1 (b) 195 (c) 7 (d) 5
snhe
41424344454647484950 is
divided by 16
(a) 2 (b) 12 (c) 6 (d) 8
39. Find the R emainde r when
666666 ......... 134times is
divided by 13
(a) 1 (b) 3 (c) 11 (d) 9
8. Find the Re mainder 6 36 is
kgei
divided by 215 25. Find the R emainde r when 40. Find the R emainde r when
21222324252627282930 is 555555 ......... 244times is
(a) 214 (b) 6 (c) 5 (d) 1
divided by 8 divided by 37
9. Find the Remainder 75 7575 is
ERna

(a) 5 (b) 2 (c) 3 (d) 4 (a) 18 (b) 5 (c) 36 (d) 0


divided by 37
26. Find the R emainde r when 41. Find the R emainde r when
(a) 1 (b) 36 (c) 3 (d) 7
919 2939 4959 6979 899 is 777777 ......... 363 times is
10. Find the Remainder 43 197 is divided by 11
divided by 7 divided by 16
aBryn

(a) 3 (b) 13 (c) 11 (d) 8 (a) 0 (b) 7 (c) 1 (d) 3


(a) 42 (b) 41 (c) 1 (d) 6 42. Find the R emainde r when
11. Find the Remainder when 17200 27. Find the R emainde r when
888888 ......... 184 times is
is divided by 18 313 2333 43536 3738 39 is
divided by 37
(a) 17 (b) 16 (c) 1 (d) 4 divided by 4
(a) 1 (b) 8 (c) 36 (d) 7
12. Find the Remainder when (1213 (a) 1 (b) 2 (c) 3 (d) N.O.T.
Les

43. Find the R emainde r when


+ 2313) is divided by 11 28. Find the Remainder when
999999999 is divided by 13
wa.th

(a) 2 (b) 1 (c) 0 (d) 3 1234.... 41digits is divided by 8


(a) 8 (b) 11 (c) 5 (d) 12
13. F i nd t he r e m a in d e r wh e n (a) 1 (b) 2 (c) 3 (d) 4
44. Find the Remainder when 7 99
(7 19 + 2) is divided by 6 29. Find the Remainder when is divided by 2400
(a) 3 (b) 1 (c) 5 (d) 2 1234....... 81digits is divided by 16 (a) 1 (b) 49 (c) 343 (d) 7
wwM

14. Find the Remainder when 3 21 (a) 13 (b) 8 (c) 1 (d) 7 45. Find the Remainder when 31989
is divided by 5 is 30. Find the Remainder when 8 77 is divided by 7
(a) 3 (b) 2 (c) 1 (d) 4 is divided by 17 (a) 2 (b) 6 (c) 4 (d) 5
15. Find the Remainder when 2 31 (a) 8 (b) 9 (c) 13 (d) 7 46. Find the Remainder when 54124
is divided by 5 31. Find the R emainde r when is divided by 17
(a) 1 (b) 2 (c) 3 (d) 4 1+ 2+ 3+ 4....... 100 is divided (a) 4 (b) 5 (c) 3 (d) 15
16. Find the Remainder when 2591 by 5 is 47. Find the Remainder when 21875
is divided by 255 (a) 0 (b) 1 (c) 2 (d) 3 is divided by 17
(a) 225 (b) 128 (c) 127 (d) 64 32. Find the R emainde r when (a) 8 (b) 13 (c) 16 (d) 9
17. Find the Remainder when 51203 1+ 2+ 3+ 4....... 100 is divided 48. Find the Remainder when 83261
is divided by 7 by 6 is is divided by 17
(a) 4 (b) 2 (c) 1 (d) 6 (a) 3 (b) 4 (c) 2 (d) 1 (a) 13 (b) 9 (c) 8 (d) 2

Rakesh Yadav Readers Publication Pvt. Ltd. 37

For More Visit : www.LearnEngineering.in


For More Visit : www.LearnEngineering.in

49. Find the R emainde r when 57. If x3 + 6x2 + 4x + k is exactly 64. On dividing (x3 - 6x + 7) by (x + 1),
32 divisible by (x + 2) then value then remainder is :
32  is divided by 9
32
of k is : – (a) 2 (b) 12 (c) 0 (d) 7
(a) 4 (b) 7 (c) 1 (d) 2 (a) -6 (b) -7 (c) -8 (d) -10 65. 5 2
If (x - 9x + 12x - 14) is divided
50. Find the R emainde r when 58. Value of k for with (x - 1) is a by (x - 3), the remainder is :
32 factor of (x 3 - k) is :
32  is divided by 7
32

(a) - 1 (b) 1 (c) 8 (d) - 8


(a) 184 (b) 56 (c) 2 (d) 1
4 3 2
(a) 4 (b) 7 (c) 2 (d) 1 59. If x100 + 2x99 + k is divisible by (x 66. When (x – 3x + 2x – 5x + 7)
51. Find the R emainde r when + 1), then the value of k is: i s d i v i d e d b y ( x –2 ) , t h e n
34 35 (a) 1 (b) -3 (c) 2 (d) -2 remainder is :
33  is divided by 7
60. 3 2
If (x - 5x + 4p) is divisible by (x (a) 3 (b) - 3 (c) 2 (d) 0
(a) 5 (b) 4 (c) 6 (d) 2

r
+ 2), then the value of p is 67. 3 2
If 5x + 5x - 6x + 9 is divided by
52. Find the R emainde r when (a) 7 (b) -2 (c) 3 (d) -7 (x + 3), then remainder is :

i
.iSn
888222 + 222888 is divided by 5 61. If (x - a) is a factor of (x3 – 3x2 a (a) 135 (b) -135(c) 63 (d) -63
(a) 0 (b) 1 (c) 3 (d) 4 + 2a2x + b), then the value of b
53. Find the Remainder when 68. If (x11 + 1) is divided by (x + 1),
is: then remainder is :
22225555 + 55552222 is divided by 7

agv
(a) 0 (b) 2 (c) 1 (d) 3 (a) 2 (b) 0 (c) 11 (d) 12
(a) 0 (b) 2 (c) 4 (d) 5
62. If x 3 + 3x 2 +4x + k contains
54. Find the R emainde r when 69. If 2x3 + 5x2 - 4x - 6 is divided by

ridn
(x + 6) as a factor, the value of
2x + 1, then remainder is :
5051 is divided by 11
52 k is:
(a) 6 (b) 4 (c) 7 (d) 3 (a) 66 (b) 33 (c) 132 (d) 36 13

eeYa
63. If (x + 2) and (x - 1) are the (a)  (b) 3
55. The Remaidner when (20)23 is 3
divided by 17 is factors of (x3 + 10x2 + mx + n),
(c) -3 (d) 6
(a) 11 (b) 3 (c) 6 the values of m and n are :
70. If x 3 + 5x 2 + 1 0k leaves
(d) Can't determind (a) m = 5, n = -3
remainder -2x when divided by
geisnh
(b) m = 17, n = -8
56. If (x - 2) is a factor of (x2 + 3qx - x2 +2, then the value of k is:
2q), then the value of q is : (c) m = 7, n = -18
(a) -2 (b) -1 (c) 1 (d) 2
(a) 2 (b) -2 (c) -1 (d) 1 (d) m = 23, n = -19
Enak

ANSWER KEY
1. (b) 8. (d) 15. (c) 22. (d) 29. (a) 36. (d) 43. (b) 50. (a) 57. (c) 64. (b)
R

2. (a) 9. (a) 16. (b) 23. (b) 30. (b) 37. (a) 44. (c) 51. (b) 58. (b) 65. (a)
3. (b) 10. (c) 17. (a) 24. (c) 31. (d) 38. (c) 45. (b) 52. (a) 59. (a) 66. (b)
4. (b) 11. (c) 18. (a) 25. (b) 32. (a) 39. (a) 46. (a) 53. (a) 60. (a) 67. (d)
aryn

5. (c) 12. (a) 19. (b) 26. (c) 33. (d) 40. (b) 47. (b) 54. (d) 61. (a) 68. (b)
6. (a) 13. (a) 20. (c) 27. (c) 34. (b) 41. (b) 48. (d) 55. (a) 62. (c) 69. (c)
Les B

7. (a) 14. (a) 21. (b) 28. (a) 35. (b) 42. (b) 49. (a) 56. (c) 63. (c) 70. (c)
wa.th

SOLUTION

1. (b) +5 +4 +1 3. (b)
wwM

77×85 × 73 +4 +4 -2 +3 +6 R = (-3) = 3
9 +3 +3
1330 × 1356 × 1363 × 1368 × 1397
5  4 1 20 16  36 9
  =2 13 = 13 = R = 9
9 9 +3 13
4. (b)
2. (a) -2 0 +3 +7 -3 +4 +3 +2 +1 +4
4  4  2  3  6 16  36

273+ 375 + 478 + 657 + 597 13 13 2327 + 2372 + 2394 + 4624 + 4650
25
* Av oid '–' (N eg ativ e) s ig n. 23
2  0  3  7  3 5 Norm ally divid ed 36 b y 13 4  3  2  1  4 14
 =5 
remainder = -3. Now use '-' 23 23
25 25
(Negative) sign R = 14

Rakesh Yadav Readers Publication Pvt. Ltd. 38

For More Visit : www.LearnEngineering.in


For More Visit : www.LearnEngineering.in

-1 13. (a) +1 +1 +4
5. (c) +2
32 19 3 67 2 67
6732 1
 
1 719 + 2
 1 2 2
 
 2 8

4
68 68 68 6 6 7 7
R= 1 1 2 3 67

6. (a) -1 =   1 4
6 6 7
99
99 R=3
99 1 1
  
1 4 4
321  
100 100 100 14.(a) 7 7
5
R = 100 - 1 = 99 R=4

ir
 3  9 2243 2243
+1

v.iSn
7. (a) Break The power multiple of 2 18.(a) 
130 32 9
197130 1 1 form
 
-1 +3  23  8
196 196 196

dnag
R=1 2 10 1 10 -1
3 3 9 3
   
636 5 5 81 81 81
8. (d) 3
1 1
 
2 8
215 10      
1 3 9 9 9 9

eYrai
 We know that 63 = 216 
 So break the power multiple 3 5 Remainder = 9 – 1 = 8
+1 1 3 3 19. (b) +3
 
5 5


215
3 12
6   216
215
12

15. (c)
R=3
231
5
snhe 5928 328
7
 33 = 27

7

33  3
7

9
kgei
12

 1 
1  22  4 -1 +3
215 215 -1 9

R=1 27  3
ERna


2 15 1 15
 21 7
9. (a) +1 2   2  4
7575 5 5 9
757575 1
  
1
15
 1  3  1 3  3
aBryn

37 37 37 1 2 2 7 7 7
 1 2
 
R=1 5 5 5 R=7-3=4
10. (c) R=5-2=3 20. (c) +7
+1 2591 38
16.(b) 4177 777 72  71 
255
Les

197
1 1  
43197 = =  28 = 256 17 17 17
wa.th

7 7
7 8
73
 27
Now 2  -2
R=1 255 38 38
11. (c) +1 +1  49 7 2
 
7
17 17
wwM

200
17200 1 1
   There will be no effect of -ve
73
 256 128
18 18 18  sign because the power is even
255 9
R=1 238  7 24  22  7
1
73
128 128    
12. (a) +1 +1   17 17
255 255
Remainder = 128  24  16
1213+ 2313
11 17.(a) +2 -1
203
13 13
51 203 2
 1  1 
1 1 2
 7

7
9
16  4  7  1  28  1 28
9

11 11 11 
 23  8 17 17 17
R=2

Rakesh Yadav Readers Publication Pvt. Ltd. 39

For More Visit : www.LearnEngineering.in


For More Visit : www.LearnEngineering.in

-6 26. (c) 919293949596979899 877


30. (b)
16 17
28 28 (Avoid -ve sign) Last '4' digits 9 8 9 9
   83  512
17 17
Remainder = 16 9899
Now use -ve sign +2
96
R = - (-6) = 6 29 3 25 2 25

2 49 16  8   8  512  64
21. (b) 2 8 3
139 17 17
7 128
+1 11 25

Remainder = 11 2  64

r

16
17
313233........3839

i
16

.iSn
 23  21 8 2
   27. (c)
4 4
6
2  2  64 1
7 7
divisibility by 4  The last '2'   
17
1
16
2 1 2 2 digits divisible by 4
 

agv
 = Last '2' digits 39 1 –4
7 7 7
6 6
R=2 39 16  2  64 1  2 4
 

ridn
R= 
22. (d) 0 4 17 17
R=3

eeYa
1  2  4 8
51203  249 12345..........41 digits  
17 28. (a) 17 17
8
R = 17 - 8 = 9
51 is divisible by 17 So (51)203 is From 1 to 9 = 9 digits
divisible by 17 then remainder Remainder = 41 - 9 = 32 digits 31. (d) 1+ 2+ 3+............ 100
geisnh
'0', Now only divide 249 5
32
-1 Number   16
2 +1
12 12
1, 2, 3, 4, ..........9/10 11 ....... 41
249 24  21 16  2
 digits 1=1 =
1,
R=1
Enak

   5
17 17 17 Total Number = 9 + 16 = 25
1 2 3 4 ........ 23 24 25 +2
12
1 2 1 2 2 Last '3' digits = 425
 
R

 
17 17 17 425 2=1×2 = 2 , R = 2
Remainder  5
R=2 8
aryn

23. (b) 1234567891011121314 R=1 +1


8 29. (a) 1234.......81digits
Les B

3=1×2×3 = 6 , R = 1
 divisibitity by 8  The Last 16 5
Three digits are divisible by 8 from 1 to 9 = 9 digits
So Now last 3 digits 314 divide Remainder digits = 81 - 9 = 72 -1
by 8 we get remainder digits
wa.th

72 4=1×2×3×4= 24 , R = -1
314  36 5
 , R=2 Number 
8 2
24.(c) 41424344.........4950
1, 2, 3, .....9/ 10 11 .. 81 digits 5=5×4×3×2×1 = 120 , R = 0
Total Number = 9 + 36 + 45 5
wwM

16 5, 6, 7 ............ 100 is all


1, 2, 3, .......... 43 44 45
divisibility by 16  The last
Last '4' digits 4445 divide by 16 pe rf ec t divisible by 5 .So
Four digits are divisible by 16
we get remainder remainder '0'
No : Last '4' digits 4 9 5 0
4445 +1 +2 +1 -1 0 0 0 0
4950 R = 13
=R=6 16
16
25. (b) 21222324252627282930 16 4445 1+ 2+ 3+ 4+ 5+ 6+......... 99+ 100
=
32 5
8 124
Last '3' digits 930
112 1  2 1 1 3
930 125  
Remainder = 5 5
8 112
R=2 13 Remainder R=3

Rakesh Yadav Readers Publication Pvt. Ltd. 40

For More Visit : www.LearnEngineering.in


For More Visit : www.LearnEngineering.in

1+ 2+ 3+........ 100 34.(b) -2 101  102  103  104.......10100


32. (a) 37. (a)
6 9111 9
111
6
 
+1 11 11 10
101  R4
111 111 6
1 =1 =
1
=R=1  2 2
   4 4 +2
6 11 11
+2 Avoid -ve sign 10  100 8
101  102  
22 6 6
2111 25  21  
2 =1×2 = 2 = R = 2  R=2
6 11 11 10  100  1000
101  102  103 

ir
0  2  32 5
6
4  4  4 12

v.iSn
3 =1×2×3 = 6 = R = 0 -1 
6

6
6
32
22
2 Remainder = 0
4 =4×3×2×1 = 24 = R = 0   '0' will be the remainder of each
6 11 three terms

dnag
+1 +2 0 0 0 0
1
22
2
So '0' will be the remainder of 99th
1 2 2
    term.
1+ 2+ 3+ 4+ 5+ ......... 100 11 11 11
+4
= Now use -ve sign

eYrai
6
R=–2 10100
Actual Remainder = 11 – 2 = 9  R4
1 2 3 6

6 6 52450 * 10n is divided by 6. We always
35. (b)
R=3

33. (d)
1+ 2+ 3+........ 50
12
126
 53  125
snhe -1
get remainder 4, where n =
natural number
1 2 3 1000
38. (c) 10  10  10 ..........10  10
1001

6
kgei
+1 3 816 2 816 +4
5  5
 125  25
1 126 126
1 =1 = =R=1
12 10
ERna

8
1  25 1 25 25 101  R4
+2     6
126 126 126 +4 +4
2 R = 25
2 =1×2 = =R=2
12
aBryn

36. (d) -2 10  100 8


101  102  
+6 6 6
1012 1012 R=2
3 =1×2×3 =
6
=R=6
40 2
  +4 +4 +4
12 7 7
12
- ve s ig n will b e no e ff ec t 10  100  1000
Les

0 101  102  103 


because power is even 6
wa.th

4 =1×2×3×4 = 24 = R = 0
1012
2 4  4  4 12
12   
6

6
12 7
R=0
0 2 8 3
wwM

'0' will be remainder of each


337 three terms so '0' will be the
5 =1×2×3×4×5 =120 = R = 0
3
2  21
12   remainder of 999th.
12 7
+4 4
+1 +2 +6 0 0 0 +1
101000  101001 4  4 8
1+ 2+ 3+ 4+ 5+ ......... 50 337 337  
=  8  2  1  2 6 6 6
6 7 7
R=2
1 2  6 9
 9 1 2 2
12 12   39.(a) 666666.......134 times
7 7
R=9 R=2 13

Rakesh Yadav Readers Publication Pvt. Ltd. 41

For More Visit : www.LearnEngineering.in


For More Visit : www.LearnEngineering.in

* If any digit is made by repeating 43. (b) 999999999  34  81


a 6 times. The Number will be -4
13
divisible by 3, 7, 11, 13, 37, 39.
6 times '9' is divisible by 13,
So, 666666 ......... 132 times is 4 31 31

divisible by 13. because 132 is


r em aind er will be
Remaining digits divide by 13
0,
 3   81
the multip le of 6. So, The 17 17
we get Remainder 31 31
Remaining 2 digits will be. 4 4
divide de d b y 13 to g et the 999     
17 17
Remainder 13
431
R = 11 Avoid -ve sing. 
666666........132times 66 17
 44. (c) 799
13  42  16

r
2400 2
15 15
4  41 16 4
 
66

i
.iSn
 =R=1 1
7 7 
13 17 17
72  49 15
4
40. (b) 555555......244 times
 1
 4 1 4
 =  4
73  343 17 17 17

agv
37
5 55 55 5. .. .. .2 40 tim es is 74  2401 Now use -ve sign = – (–4)
divisible by 37 becaused any Remainder = 4

ridn
Break the power multiple of '4'
digit is made by repeating 6 form 47.(b) +4
time s. The num be r will b e 24
4
7   73

eeYa
divisible by 37. 875 875

2400
21  21
555555........240 times,5555 17 17
24
37 875


2401  343  4 42  16
5555 2400 17
Remainder
geisnh
437
37 +1 4 2
 41
 
24 24
17
37 5555 15 2401  343 1  343
 
 -1
37 2400 2400
Enak

185
437 437
185 16  41 1  4
 
1 343 343
5 Remainder   
2400 2400 17 17
R

R=5 R = 343 1 4 4


 
41. (b) 777777.............363Times 45. (b) 31989 17 17
aryn

11 7 R = 17 – 4 = 13
48. (d) -2
 33  27
Les B

777777.............360 Times,777

11 -1 261
83261 83

777 3 663 663 17 17

11 3  27
  261 261
2  2
wa.th

 
Remainder = 7 7 7 
663 17 17
42. (b) 888888.........184times
 1
 1 Avoide -ve sign.
37 7 65
R=7–1=6 2261 24  21
 
wwM

888888.........180 times,8888  
 46. (a) +3 17 17
37
8888  24  16
124
 54 124
54
37   -1
17 17
124
3
 
65 65
37 8888 16 2 1 2
74 17    
31  3 17 17
148
148 32  9 1  2
  2
8 Remainder 17
33  27
Now use sign -(-2) = 2
 So Remainder = 8 3 4  81 Remainder = 2

Rakesh Yadav Readers Publication Pvt. Ltd. 42

For More Visit : www.LearnEngineering.in


For More Visit : www.LearnEngineering.in

32 35
49. (a) 3232 +4 51. (b) 3334
   
9 7
32 2 16 16 -2
7 7 49
   
Cyclicity
 
+5 9 9 9 34
we solve 3334 33

321 +1 34
7 7
R5  2
9 5 5

416 43  41 64  4
 7
+5 +5    No effect of -ve sign. Because
9 9 9
power is even.

ir
322 32  32 5  5 25 5 11
   R7 1  4  1 4  4 234 23  21  
9 9 9 9

v.iSn
 
9 9 9 7 7
+5 +5 +5
R=4
+1
32
32
32 3
32  32  32 5  5  5 50. (a) 32  11 11

dnag
9

9

9 7  8  2  1  2  1 2
7 7 7
125 When 32 is divided by 7 then
 R8 Remainder 4 2
9 
So, 32 32 is divided b y 7

eYrai
+5 +5 +5 +5 7
remainder = 432 35 11
34 35 3
33  2 2   22
32 32 Now :-
32 4


32  32  32  32 5  5  5  5
 
4  7

7

7
9 9 9
-2 -2

25  25 2  2 4
7
4 = 22

2 
snhe
232 32
+1

11
8  4  1 4  4
 
kgei
   ,R  4 7 7 7 7
9 9 9
+5 +5 +5 +5 +5 -2 -2 5 R=4
64 32
2 
ERna

 888222  222888
325 32  32  32  32  32 25  25  5 7 52. (a)
  5
9 9 9
 23  8 888222 222888
2  2 5 20  
,R  2 5 5
aBryn

  32
9 9

2 63
 21 
3222 2888
+5 +5 +5 +5 +5 +5 -2 -2 -2 7  
5 5
326 32  32  32  32  32  32 25  25  25 1 55 222
4
  3  32 2 4
9 9 9     
Les

32

2 2  2 8 2  3 21
 21   8 21
 21 
32
5 5
wa.th

  ,R  9  8  1 
9 9 7 7 1 9 1
 
After this It repeated so, 5 5
32
Cyclicity = 6

1  2 
232
4 1 4 1 5
   
wwM

6 7 7
32 5 5 5 5
So, R 1 Again 
9 23  8 Thus the remainder is zero.
32 10 Alternatively:
230  22  2   4
32 5 3
Now 32
32
  
 326  322 
  [To check the divisibility by 5
  
7 7 just see the sum of the unit
5 2
32
2
32 digits which is 10 (=4+6)

1  32  
32  1
  8222  4 units digit
9 9 10
10 1  4  1  4
 8  4 
322  7 7 and 2888  6 units digit
 R7 7
9 Hence it is divisible. So there
Remainder = 4
(above explain In Solution) is no remainder]

Rakesh Yadav Readers Publication Pvt. Ltd. 43

For More Visit : www.LearnEngineering.in


For More Visit : www.LearnEngineering.in

22225555  55552222 +1 61.(a) x-a=0  x=a


53.(a)  x3 - 3x2 a + 2a2 x + b = 0
7
26 5 5 1  a3 - 3a2 a + 2a2 a + b = 0
+3 +4
 3  3   3  a3 - 3a3 + 2a3 + b = 0
11 11  b =0
22225555 55552222 +1 62.(c) (x + 6) is a factor of f (x)

7 7 The f (x) = 0
5555 2222 5 x+6=0
3  4  243  3 x = -6
7 7 11 f(x) = x3 + 3x2 + 4x + k
3 1851 740 2
3  3 2 3
4   4 5
f(-6)=(-6)3+ 3 (-6)2 + 4×-6 + k = 0

r
7

7  1  3  1 3 
3 = -216 + 108 - 24 + k = 0
k = 132

i
11 11 11

.iSn
-1 +1 63.(c) (x + 2) & (x - 1) are the factor
Remainder = 3
55. (a) of x3 + 10x2 + mx + n, so put x = -
1851 740
27 9 64 16 +3 2 and x = 1 respectively
+10

agv
  (-2)3 + 10(-2)2 + m (-2) + n = 0
7 7
23 23
33  32
7 7  -8 + 40 - 2m + n = 0
1
1851
9 1
740
16 20 3
 
27  9

 
   -2m + n + 32 = 0

ridn
7 7 17 17 17 17
 -2m + n = -32 ......(i)
-2 +5
9 16 Put x = 1

eeYa
  (1)3 + 10(1)2 + m.1 + n = 0
7 7 7 2 3 3
10  9 10  10  9 100  90
     1 + 10 + m + n = 0
9  16 7 17 17 17  m + n = - 11 .......(ii)

7 7 3 solve (i) and (ii) we get
2  5  8  5  40
Remainder = 0
geisnh
  m =7
52
17 17 17 and
 5051   Av oid (-) sign nor mally
54. (d) ,we break = 5051  n = - 18
11 divide 40 by 17, Remainder 6, 64. (b)x + 1 = 0  x = - 1
Now use negative sign R = -6
Enak

6 put the value of x = -1 in


 R = 17 - 6 = 11 equation
56. (c) (x - 2) is factor of x2 + 3qx - 2q
51 51
R = x3 - 6x + 7 = (-1)3 - 6(-1) + 7
50  6 So, x - 2 = 0  x = 2
=-1+6+7
R

11 11 Put, x = 2 in x2 + 3qx - 2q and


Remainder = 12
+3 equate to zero.
65. (a)x - 3 = 0  x = 3
aryn

 (2)2 + 3 × q × 2 - 2q = 0
put of value of x = 3 in x5 - 9x2 +
2 25 1 25  4 +6q - 2q = 0  4 + 4q =0
6 6 36 6 12x - 14
   
Les B

1+q=0  q=-1
11 11 57. (c)Factor  x + 2 = 0  x = -2 R = (3)5 - 9(3)2 + 12(3) - 14
25 5
5 (put) R = 243 - 81 + 36 - 14
3 6 3 6
   (-2)3 + 6(-2)2 + 4(-2)+k = 0 R = 279 - 95
11 11 R = 184
 -8 + 24 - 8 + k = 0
wa.th

 3  243 5
 k = -8 66. (b)x - 2 = 0  x = 2
+1 58.(b) x - 1 = 0  x = 1 (put) put the value of x = 2 in x4 - 3x3
 13 - k = 0 + 2x2 - 5x + 7
5 5
 1-k=0  k= 1 R = 24 - 3(2)3 + 2(2)2 - 5(2) + 7
wwM

 243  6  
1 6

6
59.(a) x + 1 = 0  x = -1 (put) R = 16 - 24 + 8 - 10 + 7
11 11 11
 x100 + 2x99 + k = 0 R = 31 - 34 = - 3
51
50 6  (-1)100 + 2(-1)99 + k = 0 67. (d)x + 3 = 0  x = - 3
Now 
11 11  1 - 2 + k = 0  -1 + k = 0 put te value of x = -3 in 5x3 +5x2
52  k=1 - 6x + 9
 5051 
52
6 60.(a) x + 2 = 0  x = -2 (put) R = 5(-3)3 + 5(-3)2 - 6(-3) + 9
11 R = -135 + 45 + 18 + 9
 x3 - 5x2 + 4p = 0
+3  (-2)3 - 5(-2)2 + 4p = 0 R = -135 + 72 = -63
 -8 - 20 + 4p = 0 68. (b) x + 1 = 0  x = - 1
2 26 26
6   36  -28 + 4p = 0  4p = 28 put the value of x = -1 in x11 + 1

11 11  p=7 R = (-1)11 + 1 = - 1 + 1 = 0

Rakesh Yadav Readers Publication Pvt. Ltd. 44

For More Visit : www.LearnEngineering.in


For More Visit : www.LearnEngineering.in

70. (c) x2 + 2 = 0  x2 = -2
1 1 5 4
69. (c)2x + 1 = 0  x = - R  2    6 Put the value of x2 = -2 in x3 +5x2
2 8 4 2
+ 10k
1 1 5 R = x2. x + 5x2 + 10k = -2x (given)
put the value of x = - in 2x3 + R  26
2 4 4  (-2) x + 5. (-2) + 10k = -2x
5x2 - 4x - 6  -10 + 10k = 0
4  k=1
3 2 R   4  3
 1  1  1 4
R  2
 2   5 
 2   4 
 2   6
     

ir
v.iSn
dnag
eYrai
snhe
kgei
ERna
aBryn
Les
wa.th
wwM

Rakesh Yadav Readers Publication Pvt. Ltd. 45

For More Visit : www.LearnEngineering.in


For More Visit : www.LearnEngineering.in

CHAPTER

ARITHMETIC PROGRESSION
06
& GEOMETRIC PROGRESSION
Arithmetic Progression The Ist A.P (3, 9, 15, 21....) +
n
(ii) Sn  2a  n  1 d  2nd AP (8, 13, 18, ........)
2

r
A.P :- Quantities are said to be In 33rd Term lies, In first A.P

i
.iSn
arithmetic progression when they 10  which 17th term of first A.P
 2  5  10  1 7  The n
Increase or decrease by a common 2 
difference. = 5 [ 10 + 63 ] = 5 × 73 = 365 T17 = a + (n - 1) d
Some m or e exam ples of Ex. 2 Find the sum of the 10 terms a=3

agv
Ar ithm etic P rogr es sion are as of the following series -11, -8, -5, - d=6
follows 2, .... T17 = 3 + 16 × 6

ridn
1, 2, 3, 4, 5, 6, 7,... Sol. -11, -8, -5, -2, .... T17 = 99
3, 7, 11, 15, 19, 23 ....... a = - 11, d = 3, n = 10 We can say that
T33 = 99

eeYa
19, 17, 15, 13, 11, ........ n
Sn = [2a + (n-1)d] Ex.5 Find the sum to 100 terms of
–10, -4, 2, 8, 14, 20, ...... 2 the series
40, 37.5, 35, 32.5, 30, ...... 1 + 4 + 6 + 5 + 11 + 6 + ......
5 , 12 , 19 , 26 , 33.......... 10
S10 = [2×(-11)+(9)3] (a) 7400 (b) 7550
2
geisnh (c) 7600 (d) 7500
+7 +7 +7 +7 = 5× (-22 +27) = 25
a1 a2 a3 a4 a5 Sol. (c) 1 + 4 + 6 + 5 + 11 + 6 + ......
Ex. 3 If t1+t5+ t10+ t15+ t20+ t24= 225
d = Common difference (where t1)  first term of A.P, The series is a combination of
= a2 - a1 = 12 - 5 = 7 T5  fifth term of A.P) two APS.
(1 + 6 + 11 + ..... 50th term) +
Enak

a1  First Term find the sum of


(4 + 5 + 6 + ........50th term)
a 2  Second Term t1+t2+ t3+ .......... t24
Sum of series
Tn = a+(n-1)d a1 = 1 a=4
R

Tn =a+(n-1) d d1 = 5 d=1
Sol. t1= first term = a
t5 = a + (5 - 1) d = a + 4d n = 50 n = 50
Tn  nth term of A.P.
aryn

a  First Term of A.P. t10= a + 9d 50 50


d  common difference t15= a + 14d S100 = [2 × 1 + 49 × 5] + [2 ×
2 2
Les B

t20= a + 19d
4 + 49 × 1]
L= a+(n -1) d t24= a + 23 d
= 25 [2 + 245] + 25 [8 + 49]
a + a + 4d + a +9d + a + 14d + a +
L  Last term/ nth term of A.P. = 25 [247] + 25 [57]
19d + a + 23d = 225
Sum of n term. 6a + 69d = 225 = 25 [247 + 57]
wa.th

n = 25 × 304 = 7600
Sn  2a  n  1 d  2a+23d = 75 Ex.6 Find the value of
2
1 - 2 - 3 + 2 - 3 - 4 + ....+up to
n 24 
S24  2a  24  1 d  100 terms
or Sn  a  l  2 
wwM

2 (a) –626 (b) – 622


Ex. 1 5, 12, 19, 26 .............. T10 S24  122a  23d (c) – 624 (d) – 628
(i) Find the value of t10 Sol.(a) Series is a combination of
Put value of 2a + 23d = 75 three APS.
(ii) Sum of 10 term. S24 = 12 [ 75] = 900 (1 + 2 + 3 ...... 34 terms) -
Sol. 5, 12, 19, 26 .... T10 Ex. 4 Find the 33 rd term of the a=1
sequence. d=1
d = 12 - 5 = 7
3, 8, 9, 13, 15, 18, 21, 23 ....... (2 + 3 + 4 ......33 terms) -
a=5 (a) 93 (b) 992 (c) 105 (d) 83 a=2
n = 10 Sol. (b) 3, 8, 9, 13, 15, 18, 21, 23 d=1
T10 = a + (n - 1) d ....... (3 + 4 + 5 ........33terms)
= 5 + (10 - 1) 7 The ab ov e se ries is a a=3
= 5 + 63 = 68 combination af two APS d=1

Rakesh Yadav Readers Publication Pvt. Ltd. 46

For More Visit : www.LearnEngineering.in


For More Visit : www.LearnEngineering.in

Sum of series Ex.9 The sum of the second and the Ex.11 How many term of an AP
fifth term of an AP is 8 and that must be taken for their sum to
34 33 of the third and the seventh
= [2 × 1 + 33 × 1] - [2 × 2 be equal to 120 if its third term
2 2 term is 14. Find the eleventh is 9 and the difference between
term. the seventh and the second
33
+ 32 × 1] - [2 × 3 + 32 × 1] (a) 19 (b) 17 (c) 15 (d) 16 term is 20 ?
2
Sol. (a) T2 + T5 = 8 (a) 6 (b) 9 (c) 7 (d) 8
33 33 T3 + T7 = 14 Sol. (d) T3 = a + 2d = 9
= 17 [2 + 33] - [4 + 32] - T2 = a + d
2 2 T7 - T2 = (a + 6d) - (a + d) = 20
[6 + 32] T5 = a + 4d 5d = 20
T2 + T5 = a + d + a + 4d = 8 d=4
33 33

ir
= 17 × 35 - ×36 - ×38 2a + 5d = 8 ..........(i) T3 = a + 2 × 4 = 9
2 2 T3 = a + 2d

v.iSn
a=1
= 17 × 35 - 33 × 18 - 33 × 19 T7 = a + 6d
= 595 - 594 - 627 = – 626 T3 + T7 = a + 2d + a + 6d = 14 n
120 = [2 × 1 + (n - 1)4]
Ex.7 It the sum of first 11 terms of 2a + 8d = 14 ..........(ii) 2
A.P is equal to sum of first 19 (ii) – (i)

dnag
terms of that A.P. find the sum (2a + 8d) - (2a + 5d) = 14 - 8 n
120= ×2 [1 + (n - 1)2]
of first 30 terms of that A.P. 3d = 6 2
Sol. According to question d=2 120 = n [1 + 2n - 2]
S11 = S19

eYrai
Put the value d equation (ii) 120 = n (2n - 1)
2a + 8 × 2 = 14 Take Option (d) , Put the value
11 [2a + 10d]  19 [2a+18d]
2 a=-1 n=8
2 T11 = a + 10d
22a + 110 d = 38a + 342 d = 8 (2 × 8 - 1) = 120
16a = -232d
2a = -29d
2a+29d=0
= - 1 + 20
= 19
snhe
= -1 + 10 × 2

Ex.10 A number 20 is divided Into *


 n=8
Useful Results
If the same quantity be added
kgei
Four Parts that are in AP such to, or subtracted from,all the
30  that the product of the first and terms of an AP, the resulting
S30  2a  30  1 d 
2  fourth is to the product of the terms will from an AP, but with
ERna

= 15 [2a + 29d] second and third is 2 : 3. Find the same common difference as
Put value the Largest parts. before.
(a) 12 (b) 4 (c) 8 (d) 9
2a+29d=0 Sol. (c) Let the 4 terms of A.P. * If all the terms of an AP be
aBryn

S30 =15 × 0 = 0 a - 3d, a - d, a + d, a + 3d multiplied or divided by the same


Sum of first 30 terms of that (a - 3d) + (a + 3d) + a - d + a + d = 20 quantity, the resulting terms will
A.P = 0 4a = 20 form an AP, but with a new
a=5 common difference, which will be
Total No. of Term the multiplication/division of the
a  3da  3d  2
Les

old common difference. (as the


last term  First term
a  da  d 3 case may be)
wa.th

 1
commondifference
a 2  9d2 2 * If a1, a2, a3, a4, a5,.......an are in
 A.P then
Ex.8 Find the Total No. of terms a 2  d2 3
Between. 300 to 600 which the arithmetic mean (A.M)
Put the value of a
wwM

are exactly divisible by 4.


25  9d2 2  a1  a2  a 3  .....a n 
Sol. 304, 308, 312 ........... 596 =  
  n
d = 4, a = 304, last terms 25  d2 3  
= 596 75 - 27d = 50 - 2d2
2
* The arithmetic mean (A.M) of
l a 25 = 25d2
No. of Terms  1 odd numbers of consecutive
d d2 = 1
terms is the middle most term
596  304 d=1
itself.
 1 a - 3d = 5 - 3 × 1 = 2
4 * If a and b are in A.P then
a-d=5-1=4
292 a+d=5+1=6
 1 ab
4 a + 3d = 5 + 3 × 1 = 8 A.M. =
= 73 + 1 = 74 2
Largest part = 8

Rakesh Yadav Readers Publication Pvt. Ltd. 47

For More Visit : www.LearnEngineering.in


For More Visit : www.LearnEngineering.in

* If you have to assume 3 terms Sum of an Infinite geometric 15×


1

progression when r < 1 = (21+2+3+4+5 )1/5  2 5

in AP, assume them as


a -d, a, a +d or a,a + d, a +2d = 23 = 8
a
For assuming 4 terms of an AP S  Ex.6 The seventh Term of a G.P 8
1 r times the Fourth Term. What
we use; a - 3d, a-d, a +d and a
+3d will be the first term when its
1
For assuming 5 terms of an AP, Ex. 2 16, 8, 4, 2, 1, .........  fifth term is 48 ?
2 (a) 3 (b) 6 (c) 2 (d) 4
take them as:
find sum of G.P Sol. (a) T7 = 8 T4
a-2d, a-d, a, a+d, a+2d.
Sol. 16, 8, 4, 2 ........  ar6 = 8 × ar3
Geometric Progression a = 16 r3 = 8

r
8 1 r=2
Geometric Progression (G.P) GP

i
r=

.iSn
 T5 = ar4 = 48
 Guantitier are said to be In geo- 16 2
a × (2)4 = 48
metric progression when they In- r<1
a × 16 = 48
crease or decrease by a constant
a 16 16 First term a = 3

agv
factor S     32
1  r 1  1 1/ 2 Ex.7 What will be the sum of n
Ex. 3, 6, 12, 24, ............ terms of the series
2

ridn
Ex. 3 Find the value of 8 + 88 + 888 + ........ ?
a1 a2
a2 8 10n1  9n
Common ratio (r)  a
1 1 1 
 3  9  27 .......... 
   

eeYa
1 25   (a)
81
nth term of G.P  Sol. First Solve this part
n-1 1 1 1 8 10n 1  10  9n
 
Tn =ar   ......... (b)
3 9 27 81
Where a = First terms of G.P
geisnh n -1
It is an Inifinite G.P. (c) 8 (10 - 10)
r = common ratio (d) 8 (10n + 1 - 10)
n  no. of Term. 1
a= Sol. (b) 8 + 88 + 888 + ............. n
Sum of G.P 3 = 8 (1 + 11 + 111 + ..........n)
Enak

r n
1  1 8
sn  a where r > 1 9 1 = [9 + 99 + 999 ........n]
r 1 9
r= 1 3
R

or, 3 8
= [(10-1)+(100-1)+(100-1).....n]
9
 where r < 1
aryn

n
1 r 
sn a Where r <1 1 1
8
1 r = [(101+102+103...10n) –
9
Les B

a 3 1
S    3 
Ex. 1 3, 6, 12, 24 ........ T10 1 r 1 1 2 2 (1+1+1....n)]
Find 3 3 
(i) T10 (10th term of G.P) The n It is a G.P
(ii) S10 (Sum of First 10 terms
wa.th

Put the value of


8 
n
of G.P) 10 10  1  n 
 
Sol. 3, 6, 12, 24, ........T10 1 1 1 1 = 9 
  ......  
 10  1 
3 4 27 2
6
r= =2 1
wwM

3 = 25 2
5 8 10n 1  10 
= 9   n
a=3 Ex. 4 Find the Geometric mean of  9 
T10 = 3.2 (10 - 1) = 3 × 29 3, 9, 27. 10n 1 –10 – 9n 
8  
= 3 × 512 = 1536 (a) 9 (b) 27 (c) 3 (d) 81 =
Sol. (a) G.M = (a1.a2.a3. )1/3 9 9
r n 1
Sn = a r 1 = (3 × 9 × 27)1/3 8 n 1

r 1  10  10  9n 
= (31 × 32 × 33 )1/3
81
3 210  1 10 = (36)1/3 = 32 = 9 Ex.8 (666 ....n digit)2 + (888.....n digit)
S10     3 2 1
 Ex. 5 Find the G.M of 2, 4, 8, 16,32 is equal to
2 1 1
(a) 4 (b) 8 (c) 16 (d) 32
= 3×(1024 – 1) Sol. (b) (2×4×8×16×32)1/5 4
(a) (10n - 1)
= 3 × 1023 = 3096 = (21 × 22 × 23 × 24 ×25)1/5 9

Rakesh Yadav Readers Publication Pvt. Ltd. 48

For More Visit : www.LearnEngineering.in


For More Visit : www.LearnEngineering.in

Then sides of square


4 1000 1000
(b) (10 2n
- 1) S    3000m 4a1 = 4, a1 = 1
9 2 1/ 3
1 4a2 = 8, a2 = 2
3
4 4a3 = 16, a3 = 4
(c) (10n -1)n -1 - 1) Ex.11 The side of a square is 16 cm 4a4 = 32, a4 = 8
9 Infinite squares are made by
Where a 1, a 2, a 3, a 4 sides of
4 joining the mid points of the
square. Calculate the area of square 1, 2, 4, 8
(d) (10n + 1)
9 This is a G.P.
all the squares made.
Sol. (b) (666.........n digit)2 + (888....n We can say that If perimeter
Sol.
digit) of square In G.P. then their
16
In this type question we take D H C sides will be in G.P.

ir
n = 1 then Ex.13 The sides of a right angle 

v.iSn
(6)2 + 8 = 44 are 6, 8 and 10 cm respectively.
we take option (b) put the n = 1 A new right angle  is made
16 E G 16
4 4 by joining the mid-points of all
(102 - 1) = 99× = 44 the sides . This process contin-
9 9

dnag
8 2 ues for infinite then calculate
4 the area of all the  so made.
So, (102n - 1) 8 F
9 A 16 B
Area of large square of ABCD Sol.
Ex.9 A ball is thrown from a height

eYrai
of 500m on the ground. The = (16)2 = 256
ball bounce 4/5 times of I st Square of made of mid-point
ev er y last b ounc e then (EFGH) EF2 = AE2 + AF2 4 10
6
calculate the total Distance

Sol.
the ball It stop.

400
AE =
16
2

16
snhe
 8 (E, midpoint of AD) 5 3

8
500 m
AF =  8 (F, mid point of AB)
kgei
m 320
2
400 320 Area of Large right angle tri-
2 2
EF = 8   8  8 2 1
×4/5 angle   6  8  24
ERna

×4/5 4/5
2
2

(500 + 400), (400 + 320), .......


Area of EFGH = 8 2    128 A1 A2 A3
  
900, 720, ......  256 + 128 + ........  24 6 3/2............
aBryn

a = 900 a = 256
r = 128/256 = 1/2 A1  24  Ar ea of fir st r ig ht
720 4
r  angle triangle
900 5 256 256
S    512 cm2 A2 = 6  Area of second right
a 900 900 1 1 1
S    2 2 angle triangle
1  r 1  4 1/5
Les

5 * The figure m ad e by the A2 6 1


wa.th

joining the mid-points of a r  


= 900 × 5 = 4500 m A1 24 4
Ex .10 A b all is thrown fr om a square and the area of thus
height of 600 m on the ground. made square is half of the ac- 24 24 4 32cm2
the ball bounce 2/3 times of Its tual square S    24 
1 34 3
1
wwM

every Last Bounce then calcu- Ex.12 A number of Squares are 4


late the Total Distance the ball described whose perimetres
till It stop. are in G.P. Then their sides will * The figure made by the joining
be In the mid-points of a right angle
Sol. 400
(a) A.P (b) G.P triangle is right angle triangle
600 m
m 800 (c) H.P and the area of thus made tri-
400
3 th
(d) None of these 1
Sol. (b) Perimeter of square = 4 × angle is of the
×2/3 4
×2/3
side actual triangle.
a = (600 + 400) = 1000 According to question Ex.14 If A is the sum of the n terms
800 2 Let perimetres of square in of the series 1 + 1/4 + 1/16 +
1000, 400 + , ......... r  G.P. 4, 8, 16, 32 ......... ... and B is the sum of 2n terms
3 3

Rakesh Yadav Readers Publication Pvt. Ltd. 49

For More Visit : www.LearnEngineering.in


For More Visit : www.LearnEngineering.in

of the series 1 + 1/2 + 1/4 + Ex.16 Two numbers A and B are a/b =b/c =c/d = .....=1/r
...., then find the value of A/B such that their GM is 20% Conversely, a series of quantities
(a) 1/3 (b) 1/2 (c) 2/3 (d) 3/4 Lower than their AM. Find the in continued proportion may be
Ratio B/W the numbers. represented by x, xr, xr2 ......
1 1 (a) 3 : 2 (b) 3 : 1
Sol. (c) A = 1 + +
4 16
+........n term * For assuming 3 terms of GP
(c) 4 : 1 (d) 2 : 1
a=1 assume them as
Sol. (c) GM of A and B = (AB)1/2
a, ar, ar2 or a/r, a, ar
1 A B
r= AM of A and B = * for assuming 4 terms of GP
4 2
assume them as
Let A.M = 100
  1 n  1 
n
a a
11  Then G.M = 80
   1 , ,ar, ar 3

r

 4    
 
  4 A.M : GM r3 r

i
.iSn
A = Sn = 1 = 3 ....(i) 100 : 80
1 5:4
* If a1, a2, a3, a4, a5,.......an are in G.P
4 4
the geometric mean G.M
A .M 5
1 1 1  = (a1, a2, a3, a4, .......an)1/n

agv
B=1+ + + .......2n term G.M 4
2 4 8 * If a and b are two term in G.P
a=1 then
A  B

ridn
1 2 5 G.M = ab
r= 1/2

2   4
AB
*

eeYa
To f ind the sum of f ir st n
B = S2n
A B 5 natural numbers
  1  2n  n
 Let the sum be denoted by S;
 1  1 2 AB 4
11
   
  
  then
 2  4 Uses c & d form
= = ...(ii)
geisnh S = 1 +2 + 3 + ....+n, is given by
1 1
1 A  B  2 ab 5  4 n n +1
2 2 
A  B  2 ab 5  4 S=
(i) / (ii) 2
2
* To find the sum of the squares
Enak

2 A B
A/B =
3
 
2 
9
of the first n natural numbers
A B 1
Ex.15 Find the sum to n terms of  Let the sum be denoted by S;
then
R

the series
11 + 103 + 1005 + ......... A B 3 S= 12 + 22 + 32 + ........n2
 This is given by:
A B 1
aryn

10
(a) 10n  1  n 2
  
9 Again Use c & d  n n +12n +1 
S=  
Les B

A 3 1 
 6 

10
(b) 10n – 1  n 2
  
9 B 3 1 * To find the sum of the cubes of
10 the first n natural numbers.
(c) 10n  1  n
  A 4 2
Let the sum be denoted by S:then
wa.th

9  
B 2 1 S=13 + 23 + 33 + ......+n3
10 Take Square Both sides
(d) 10n – 1  n
  2
9  n n +1 
A 4   
Sol. (b) 11 + 103 + 1005 + ......... S=  
 2
wwM

B 1  
10 + 1 + 100 + 3 + 1000 + 5 + .......
(101 + 102 + 103 + .....10n) + (1+3+5+......n) A:B=4:1 Thus, the sum of the cubes of
the first n natural numbers is
  Useful Result of G.P equal to the square of the sum
It is a G.P It is sum of odd no.
n
* If all the ter ms of a GP b e of these numbers.

Sn = a r 1
 S=n 2 multiplied or divided by the * To find the sum of the first n
r 1 same quantity, the resulting odd natural numbers.
terms will form a GP with the S = 1 + 3 + 5 + ..... + (2n - 1)  n2
10 10 n  1
  n 2 same common ratio as before.
Total sum = * To find the sum of the first n
10  1 * If a, b, c, d, ......are in GP they
are also in continued proportion, even natural numbers.
10
= (10n - 1) + n2 since, by definition, S = 2 + 4 + 6+....+2n  n (n+1)
9

Rakesh Yadav Readers Publication Pvt. Ltd. 50

For More Visit : www.LearnEngineering.in


For More Visit : www.LearnEngineering.in

EXERCISE

1. Find t10 and s10 for the following 9. Find t24 and s24 for the following 15. How many terms are there In
series. series. the A.P 20, 25, 30, ........130?
1, 8, 15, 22............... 9 (a) 22 (b) 23 (c) 21 (d) 24
(a) 64,325 (c) 64,318 18, 9, , ...... 16. Find the Ist terms an A.P. whose
2
(c) 57,325 (d) 57,318 8th and 12 th ter ms are
23
2. Find t20 and S20 for the following 1   1  respectively 39 & 59
series (a) 18   ,36 1 – 22 
2   2  (a) 5 (b) 6 (c) 4 (d) 3
2, 8, 14, 20, ............. 22 17 There is an AP 1, 3, 5, .......
1 

ir
(a) 116,1172(c) 110,1180 1  
(b) 18   ,36 1 – 224  which term of this AP is 55. ?
(c) 116,1180(d) 110,1172 2   

v.iSn
(a) 27th (b) 26th (c) 25th (d) 28th
3. Find t24 and s24 for the following 24
1   1  18. F ind the 15 th te r m of the
series. (c) 18   ,36 1 – 224 
2  sequence 20, 15, 10, ..........
3, 13, 23, 33 ............  
(a) 233,2842 (b) 230,2832 23
(a) -45 (b) -50 (c) -55 (d) 0

dnag
1   1  19. A number 15 is divided In three
(c) 230,2842 (d) 233,2832 (d) 18   ,36 1 – 224 
4. Find t18 and s22 for the following 2    parts which are In A.P and The
series. 10. Find t30 and s30 for the following sum of their squares is 83.
series – 3, 1, 5, 9,......... series. Find the Smallest No.

eYrai
64, 16, 4, 1 ........ (a) 5 (b) 3 (c) 6 (d) 8
(a) 65,858 (b) 60,850
(c) 60,868 (d) 65,850   1 30  20. The sum of the first 16 terms of
64 
1 – 
 

  an A.P. whose first terms and
5. Find t28 and s48 for the following  4 
series. 1   thir d te rm are 5 and 1 5
(a) 26 ,
30, 33, 36, 39 ......
(a) 111,4834 (b) 111,4824
(c) 121,4824 (d) 121,4834
4
snhe
1–
1
4
  1 30 
64  
respectively is
(a) 600 (b) 765 (c) 640 (d) 680
21. The Number of terms of the
6. Find t30 and s30 for the following 1 – 
    series 54 + 51 + 48 + ..... such
kgei
series. 1  4  
(b) 26 , that the sum is 513 is
4 1
If 36, 34, 32, 30...... 1 (a) 18 (b) 19
(a) 22,210 (b) –22,210 4
(c) Both a and b (d) 15
ERna

(c) –22,220 (d) 22, –210   1 30  22 A man receives ` 60 for the
7. Find the t 20 & S 20 for the 64 
1  
   

 4  first week and ` 3 more each
following series 1  
(c) 26 , 1 week than the preceding week.
2, 8, 32, ...... 4 1– How much does he earn by the
aBryn

4
2 20 20th week ?
(a) 239, 4 –1    1 30 
(a) ` 1770 (b) ` 1620
3 64   
1  
   (c) ` 1890 (d) ` 1790
4 20 1  4  
4 –1 (d) 26 , 1 23. How many terms are there In
(b) 240,
3  4 1 the G.P 5, 20, 80, 320 ........
Les

4
2 21 11 Find the sum of all numbers 20480?
wa.th

(c) 238, 4 –1  (a) 5 (b) 6 (c) 7 (d) 8


3 divisible by 6 In 100 to 400
(a) 12450 (b) 12550 24. A boy agrees to work at the rate
2 20
(d) 241, 4 –1
  (c) 12400 (d) 12456 of one rupee on the first day,
3 two rupees on the second day,
12. How many natural Numbers
8. Find t7 and s7 for the following
wwM

between 300 to 500 are nultiple four rupees on the third day and
series.
of 7 ? does on. How much will the boy
1, 3, 9, 27 ......
(a) 29 (b) 28 (c) 27 (d) 30 get if he start working on the
1 7 13. Find the v alue of the 1st of February and finishes on
(a) 729, 3 –1
 
2 Expression 1 - 6 + 2 - 7 + 3 - 8 + the 20th of February ?
1 8 ......... to 100 terms (a) 220 (b) 220 - 1
(b) 243, 3 –1
  (a) -250 (b) -500
19
(c) 2 - 1 (d) 219
2
(c) -450 (d) -300 25. If the Fifth term of a G.P. is
1 7 14. How many terms of the series 81 and first term is 16,
(c) 729, 3 1
 
2 -12, -9, -6, ......must be Taken what will be the 4 th terms of
1 8 that the sum may be 54 ? the G.P?
(d) 243, 3 1
  (a) 15 (b) 14 (c) 18 (d) 12 (a) 36 (b) 18 (c) 54 (d) 24
3

Rakesh Yadav Readers Publication Pvt. Ltd. 51

For More Visit : www.LearnEngineering.in


For More Visit : www.LearnEngineering.in

26. The 4th and 10th term of a GP the equilateral triangles, if side of (a) 1080 (b) 720
are 1/3 and 243 respectively. the largest equilateral triangle is (c) 900 (d) 620
Find the 2nd term. 24 units. 40. The sum of an infinite GP
(a) 3 (b) 1 (a) 288 units (b) 72 units whos e comm on r atio is
(c) 1/27 (d) 1/9 (c) 36 units (d) 144 units numerically less than 1 is 32
27. The 7th and 21th term of an 34. Find the v alue of the and the sum of the first two
AP are 6 and -22 respectively. expression terms is 24. What will be the
Find the 26th term. 1 – 6 + 2 – 7 + 3 – 8 + ..... to third term ?
(a) -34 (b) -32 (c) -12 (d) -10 100 terms (a) 2 (b) 16 (c) 8 (d) 4
28. The sum of 5 number in AP is (a) –250 (b) –500 41. W ha t wi ll b e t he v al ue o f
30 and the s um of their (c) –450 (d) –300 n1/2. n1/4. n1/8. ......... 
squares is 220. Which of the 35. Find the sum of the Integers (a) n2 (b) n (c) n3/2 (d) n3
between 1 and 200 that are 42. Determine the first term of the

r
following is the third term?
(a) 5 (b) 6 (c) 8 (d) 10 multiples of 7 ? geometric progeression, the

i
.iSn
29. The sum of the first four terms (a) 2742 (b) 2842 sum of whose first term and
of an AP is 28 and sum of the (c) 2642 (d) 2546 third term is 40 and the sum
first eight terms of the same 36. After striking a floor a rubber of the second term and fourth
AP is 88. Find the sum of the ball rebounds (7/8) th of the term is 80.

agv
first 16 terms of the AP? height fr om which it has (a) 12 (b) 8 (c) 16 (d) 4
(a) 346 (b) 340 (c) 304 (d) 268 fallen. Find the total distance 43. It is G.P 32, 4, 8, n and 2 and
that it travels before coming to G.M is 8, find the value of n

ridn
30. Find the number of terms of the
rest, if it is gently dropped from (a) 2 (b) 4 (c) 8 (d) 16
1 1 1 a height of 420 meters. 44. Find the Arithmetic mean of
series , , , ........ 729
81 27 9 (a) 2940 (b) 6300 the following series

eeYa
(a) 11 (b) 12 (c) 10 (d) 13 (c) 1080 (d) 3360 20, 23, 26, 29
31. A man saves ` 100 in January 37. Jack and Jil were playing 59 49
2014 and increases his saving mathematical puzzles with each (a) 49 (b) (c) (d) 59
by ` 50 every month over the other. Jill drew a square of sides 8 2 2
45. If the arithmetic mean of the
previous month. What is the
geisnh
cm and then kept on drawing
annual saving for the man in squares inside the squares by number x1, x2, x3 .......xn, is x ,
the year 2014 ? joining the mid points of the then the arithmetic mean of
(a) ` 4200 (b) ` 4500 squares. She continued this the number ax1+ b, ax2+ b, ax3+
(c) ` 4000 (d) ` 4100 process indefinitely. Jill asked jack b, .......axn+ b, where a and b are
Enak

32. What is the maximum sum of to determine the sum of the areas two constants, would be ?
the terms in the Arithmetic of all the squares that she drew. If
progression Jack answered correctly then what (a) x (b) na x  nb
1 1 would be his answer ? (c) a x (d) a x  b
R

25, 24 , 24, ....... 1, (a) 128 (b) 64 (c) 256 (d) 32 46. A boy draws n squares with
2 2 38. If the m th term of an AP is
1 sides 1, 2, 3, 4, 5, .... in inches.
aryn

(a) 637 (b) 625 1 1 The average area covered by


2 and nth term is then find these n squares will be :
n m
1
Les B

th
(c) 662 (d) 650 the sum to mn term  n 1 
2 (a) 
 2 
33. An equilateral triangle is drawn by 1 1  
joining the midpoints of the sides
(a)
4
mn  1 (b)
4
mn  1
 n  1   2n  1 
of another equilateral triangle. A 1 1 (b) 
 2    
 3 
wa.th

third equilateral triangle is drawn (c)


2
mn  1 (d) 2 mn  1 
inside the second one joining the 1

midpoints of the sides of the second 39. The first and the Last terms  n  1   2n  1 
of an A.P. are 107 and 253. If (c)   
 
equilateral triangle, and the  2  3 
process continues infinitely. Find there are five terms in this
wwM

the sum of the perimeters of all se quence , find the s um of  n  1   2n  1 


sequence  2   1
(d)   
   3 

ANSWER KEY
1. (a) 6. (b) 11. (a) 16. (c) 21. (c) 26. (c) 31. (b) 36. (b) 41. (b) 46. (b)
2. (c) 7. (a) 12. (a) 17. (d) 22. (a) 27. (b) 32. (a) 37. (a) 42. (b)
3. (d) 8. (a) 13. (a) 18. (b) 23. (c) 28. (d) 33. (d) 38. (c) 43. (d)
4. (a) 9. (d) 14. (d) 19. (b) 24. (b) 29. (c) 34. (a) 39. (c) 44. (c)
5. (b) 10. (a) 15. (b) 20. (d) 25. (c) 30. (a) 35. (b) 40. (d) 45. (d)

Rakesh Yadav Readers Publication Pvt. Ltd. 52

For More Visit : www.LearnEngineering.in


For More Visit : www.LearnEngineering.in

SOLUTION

1. (a) 1, 8, 15, 22, .......... = 2 × 238 = 239   1  30 


This is A.P series n
64 1  
  

S20 =
a r 1
  (r  1)  4 
a=1 =
r 1 1
d = (8 - 1) = 7 1
4
T10 = 1 + (10 - 1) 7 = 1 + 63 = 64 2 420  1
  2 20 st
= = 4 -1
  11. (a) I Term = a = 102 (where is
4 1 3 the Ist term greater than 100
10

ir
S10 = [2 × 1 + 9 × 7] 8. (a) 1, 3, 9, 27 ...... that is divisible by 6)
2

v.iSn
a=1 Last term less than 400, which
= 5 [ 2 + 63] = 5 × 65 = 325
3 is divisible by 6 is 396.
2. (c) 2, 8, 14, 20 r 3
a=2 1 then.
T7 = 1(3)7 - 1 = 36 = 729 102 + 108 + 114 ......396

dnag
d=6
T20 = 2 + 19 × 6 = 116 a = 102, d = 6
137 –1 1 7
S20 = 10 [2 × 2 + 19 ×6] S7 = = 3 –1
  No. of term
3 –1 2
= 10 [4 + 114] = 1180

eYrai
Last term  First term
9 = 1
3. (d) 3, 13, 23, 33 ............ 9. (d) 18, 9, , ...... difference
2
a=3
a = 18 396  102
d = 10  1
T24 = 3 + 23 × 10 = 233
S24 = 12 [2 × 3 + 23 × 10]
= 12 [6 + 230]
r=
9

18 2
1
snhe
1 
24 1
1 
23 
294
6
6

 1  49  1  50
kgei
= 12 × 236 = 2832 T24 = 18   = 18  
2  2 
4. (a) – 3, 1, 5, 9,......... 50
1  r n  S50  2 102  49  6
a=–3 2
= a  1  r  r  1
ERna

S24
d = 1 – (-3) = 1 + 3 = 4   = 25 [204 + 294] = 12450
T18 = -3 + 17 × 4 = 65 12. (a) First multiple of 7 term (300
  1  24 
S22 = 11 [2 × -3 + 21 × 4] 18 1     to 500) = 301
   2  
aBryn

= 11 [-6 + 84] = 78 × 11 = 858   Last multiple of 7 term (300 to


5. (b) 30, 33, 36, 39 ...... 1 500) = 497
1
a = 30 2
301, 308, ......... 497
d=3  1  24
 497  301
T28 = 30 + 27 × 3 = 111 18 1     No. of term = 1
Les

  2   1  7
S48 = 24 [2 × 30 + 47 × 3]    = 36 1- 24 
wa.th

1  2 

= 24 [60 + 141] = 4824 2 196
  1 = 28 + 1 = 29
6. (b) 36, 34, 32, 30...... 10. (a) 64, 16, 4, 1 ........ 7
a = 36 a = 64 13. (a) 1 - 6 + 2 - 7 + 3 - 8 + ......
d = (34 - 36) = -2
wwM

16 1 to 100
T30 = 36 + 29 × -2 d= 
64 4 (1 + 2 + 3 ....... to 50 term)
= 36 - 58 = -22
1 
30 1
1 
29
[Where a = 1, d = 1, n = 50]
S30 = 15 [2 × 36 + 29 × -2] T30 = 64   = 64  
4 4 - (6 + 7 + 8 ... to 50 term)
= 15 [72 - 58]
[Where a = 6, d = 1, n = 50]
= 15 [14] = 210 3 1 1
= 4   Both series are In A.P.
7. (a) 2, 8, 32, ...... 429 426
a=2  1  30
 Use the formula for sum of an
64 
1  
 
 A.P.
8 
r= =4  4   r 1
2 S30 = 1 n 
T20 = arn - 1 = 2(4)20 - 1 = 2 × (4)19 1 Sn  2a  n  1 d 
4 2 

Rakesh Yadav Readers Publication Pvt. Ltd. 53

For More Visit : www.LearnEngineering.in


For More Visit : www.LearnEngineering.in

= 25 (2 × 1 + 24 × 1) - 25 (2 ×6 + 24 × 1) 18. (b) 20, 15, 10, .......... 22. (a) First week = a = 60
a = 20 second week = 63
= 25 (2 + 24) - 25 (12 + 24)
d = -5 Therefore
= 25 × 26 - 25 × 36 T15 = 20 + 14 × -5 60, 63, 66 ......20th week
a = 60
= 25 (26 - 36) = 25 × -10 T15 = 20 - 70
d=3
= - 250 T15 = -50 n = 20
19. (b) Let three term In A.P. S20 = 10 [2 × 60 + 19 × 3]
14. (d) -12, -9, -6, ......
a - d, a , a + d = 10 [120 + 57] = 1770
Sn = 54 A.T.Q 23. (c) 5, 20, 80, 320 ........ 20480
a = - 12 a - d + a + a + d = 15 a=5
3a = 15 20

r
d=3
a=5 r= =4
5

i
.iSn
n=? and Tn = 20480
n  (a - d)2 + (a)2 + (a + d)2 = 83 Tn = arn - 1
Sn  2a  n  1 d  a2 + d2 - 2ad + a2 + a2 + d2 + 2ad 20480 = 5 (4)n-1
2 
4096 = (4)n - 1

agv
= 83
n  3a2 + 2d2 = 83 212 = 22 (n - 1)
54 =  2  12  n  1 3 212 = 22 n - 2
2 

ridn
Put the value a = 5 same base comparision the
108 = n [-24 + 3n - 3] 3 × (5)2 + 2d2 = 83 power.
75 + 2d2 = 83 12 = 2n - 2

eeYa
108 = n [3n - 27]
2d2 = 8 n=7
36 = n [n - 9] 24. (b) First day = 1
d=2
Take option (d) n = 12, satisfy IInd =2
Then a - d = 5 - 2 = 3
= 12 × 3 = 36 a=5 IIIrd =4
geisnh
So : n = 12 a+d=5+2=7 Boy does the work = 20 day
15. (b) 20, 25, 30, ........130 Smallest No. = 3 then 1, 2, 4, 8, .........
20. (d) a1 = 5 = a  (i) a=1
a = 20 a3 = 15 = a + 2d  (ii) r=2
Enak

d=5 (ii) - (i)


last term = l = 130 2d = 10 a rn  1
 
d=5 Sn = r 1
l = a + (n + 1) d r 1
5, 10, 15 .......
R

130 = 20 + (n - 1) 5 a=5 1 220  1


110 = (n - 1) 5 d=5 =
 
= 220 - 1
2 1
aryn

22 = n - 1 n = 16
25. (c) T5 = ar5-1 = ar4 = 81
n = 23 16
Les B

S16 = 2  5  15  5  8 10  75  a  16 given


16. (c) T8 = 39 2 
= 680 16 × r4 = 81
T12 = 59
21. (c) 54 + 51 + 48 + .......
T8 = a + 7d = 39 ...(i) a = 54, d = - 3 81
r4 =
wa.th

T12 = a + 11d = 59 ...(ii) 16


n
(ii) - (i) Sn = [2a + (n - 1) d] 3
2 r=
4d = 20 2
d=5 n
513 = [2 × 54 + (n - 1)(- 3)] T4 = ar 4-1 = ar3
wwM

Put the value of d (i) equation 2


3
3  27
a + 7 × 5 = 39 n = 16 ×    16 
513 = [108 - 3n + 3] 2  8
a=4 2
17. (d) 1, 3, 5, ....... 1026 = n (111 - 3n) T4 = 54
a=1 342 = n (37 – n)
This is a quadratic equation n 1
d=2 26. (c) T4 = ar3 = ........(i)
has two values. 3
nth of AP = 55
In this cond ition we help T10 = ar9 = 243........(ii)
55 = 1 + (n - 1) 2 option.
(ii) , (i)
54 = (n - 1) 2 Option Both n = 18, And n = 19,
27 = n - 1 Satisfy this eauation. ar 9 243
Then (c) option. 
n = 28th ar 3 1/ 3

Rakesh Yadav Readers Publication Pvt. Ltd. 54

For More Visit : www.LearnEngineering.in


For More Visit : www.LearnEngineering.in

r6 = 243 × 3 = 729 Put the value of d, equation (i) 33. (d) A


r=3 2a + 3 × 2 = 14
Put the value r First equation 2a = 8
a=4
1 D E
a × (3)3 = S16 =8 [2 × 4 + 15 × 2]
3
= 8 [8 + 30] = 8 × 38 = 304
1 1
a × 27 = 30. (a) a =
3 81 B F C
24
1 1/27 Perimeter of  ABC
a= d= = -3
81

ir
1/ 81 = 3 × side = 3 × 24 = 72

v.iSn
1 1 Tn = arn-1  DEF
T2 = ar = 3 
81 27 Perimeter  DEF = 3 × 12 = 36
1
27. (b) T7 = a + 6d = 6 ......(i) 729 = (-3)n-1 DE || BC, D & E mid point of
81 AB & AC
T21 = a + 20d = -22 ......(ii)

dnag
(ii) – (i) 729 × 81 = (-3)n-1 1
36 × 34 = (-3)n-1 Then DE = BC
(a + 20d) - (a + 6d) = - 22 - 6 2
14d = -28 310 = (-3)n-1

eYrai
d = -2 comparison 1
Then DF = AC
Put the value d (i) equation 10 = n - 1 2
a + 6 × -2 = 6 n = 11 1
EF = AB
a - 12 = 6
a = 18
T26 = a + 25d = 18 + 25 × -2
snhe
31. (b) January Saves = 100
Febury = 100 + 50 = 150
100 + 150 + .........
2
Therefore,
72, 36, 18, .............. 
kgei
= 18 - 50 = - 32 Sum of 12 months saving 1
28. (d) Let the 5 number of A.P a = 72, r =
S12 = 6[2 × 100 + 11 × 50] 2
a - 2d, a - d, a, a + d, a + 2d
= 6 [200 + 550]
ERna

a - 2d + a - d + a + a + d + a + 2d 72
S  72  2 = 144
= 30 = 6 × 750 = ` 4500 1
1
5a = 30 32. (a) The maximum sum of the 2
a=6 Terms in the A.P. when all 34. (a) 1 – 6 + 2 – 7 + 3 – 8 + .... to
aBryn

(a - 2d)2 + (a - d)2 + a2 + (a + d)2 + (a +2d)2 terms will be positive then. 100 terms
= 220 The ab ov e se ries is a
A.P is combination of Two APs.
a2 + 4d2 - 4ad + a2 + d2 - 2ad +
a2 + a2 + d2 + 2ad+a2 + 4d2 +  (1 + 2 + 3 + ..... + 50 terms)
1 1 1
4ad = 220 25, 24 , 24, 23 , .......1, – (6 + 7 + 8 ....... + 50 terms)
2 2 2
Les

5a2 + 10d2 = 220


 25 [2 × 1 + 49 × 1] – [25 (2 × 6 +
a2 +2d2 = 44
wa.th

l a 49 × 1)]
Put the value a = 6 No. of term = 1  25 [51] – 25 × 61
(6)2 + 2d2 = 44 d
 25 [51 –61] = 25 × – 10 = –250
36 + 2d2 = 44 35. (b) Multiple of 7 from 1 to 200
2d2 = 8 1
 25
wwM

7, 14, .......... 196


d2 = 4 , d = 2  2  1 = 49 + 1 = 50
T3 = a = 6 1/2 l a
No. of Term = 1
29. (c) S4 = 28 d
Sum of series
S8 = 88 l = 196
S4 = 2 [2a + 3d] = 28 50 1 a=7
S50 = [2 × 25 + 49 ×  ]
S8 = 4 [2a + 7d] = 88 2 2 d=7
2a + 3d = 14 .....(i) 51 196  7
2a + 7d = 22 ......(ii) = 25 [50 - 24.5] = 25 × No. of term = 1
2 7
(ii) - (i)
4d = 8 1 189
= 637 No. of term =  1  27  1  28
d=2 2 7

Rakesh Yadav Readers Publication Pvt. Ltd. 55

For More Visit : www.LearnEngineering.in


For More Visit : www.LearnEngineering.in

28 1 a
S28 = [2 × 7 + 27 × 7] 38. (c) Tm = a + (m - 1) d = .....(i) 40. (d) S   32
2 n 1 r
= 14 [14 + 189] = 2842 a = 32 (1 - r) .......(i)
1
36. (b) Tn = a + (n - 1) d = .....(ii) Let 2 Terms of G.P a, ar
m a + ar = 24
321.5625 (i) - (ii) a (1 + r) = 24
420 1 1
m (m - 1)d – (n - 1) d =  24
367.5 n m a= .........(ii)
1 r
×7/8 mn put the value of a
7/8 d [m - 1 - n + 1] 
mn

r
(420+367.5), (367.5+321. 5625), ....... 24
 32 1  r
0 1 r

i
.iSn
mn
d × (m - n) 
787.5, 689.0625, ........... mn 24 = 32 (1 - r) (1 + r)
It is Infinite G.P 1 3
d  = (1 - r2)

agv
a mn 4
S  Put the value of 'd' equation (i)
1 r 3 1
r2 = 1 - =

ridn
a = 787.5 1 1 4 4
a + (m - 1)  
mn n 1
7

eeYa
r= r=
8 1 1 1 2
a + m – 
mn mn n A.T.Q.
787.5 787.5 a + ar = 24
S   1 1 1 a (1 + r) = 24
7 1/ 8 a  
1
geisnh n mn n
8  1
1 1  2  = 24
a 
= 797.5 × 8 = 6300m.  
a=
37. (a) D C mn 3
H
a× = 24
Enak

mn 2
Smn = [2a + (mn - 1)d] a = 16
2
IIIrd Term of G.P = ar2
E G 2
R

mn  1 1  1 
 2   mn  1  
= 16 ×   
2 
 mn mn  2 
4
4 2
aryn

1
mn 1  = 16 × =4
4F   2  mn  1 4
A B 2 mn 
Les B

Area of ABCD = (8)2 = 64 41. (b) n1/2. n1/4. n1/8. ......... 


Area of square of EFGH 1 1 1 1



  4  8 ............ 

2
2  mn  1 n2 

2
= 4 2   32 1 1 1
1
wa.th

a= ,r  
= mn +1 2 4 2
Sam e 2
1
64, 32, 16..........  39. (c) Let the 4 terms of A.P. 2
It is a G.P.
a
wwM

a - 2d, a - d, a, a + d, a + 2d S 
a 1 r
S 
1 r Sum of IInd and IVth term  
=a-d+a+d  
1  
 a  64 , r = = 2a  1 
2  
A.T.Q.  2 
S =  
Ist + Vth = 107 + 253 = 2a 1  1 
 
64 a = 180  2 

1 Sum of all 5 terms
1 S = 1
2 2a + 2a + a = 5a = 5 × 180
1 1 1 
= 128 = 900  
  4  8 ............ 
n2  = n1 = n

Rakesh Yadav Readers Publication Pvt. Ltd. 56

For More Visit : www.LearnEngineering.in


For More Visit : www.LearnEngineering.in

42. (b) Let the 4 terms of G.P 1


8 = 210 5  21/5  n1/5 a x1  x 2  ....  x n   nb
a, ar, ar2, ar3 
8 = 22 × 21/5 × n1/5 n
A.T.Q.
a + ar2 = 40, a (1 + r2) = 40 .....(i) (2n)1/5 = 2
a x1  x 2  x 3  ...  x n  nb
ar + ar3 = 80, ar (1 +r2) = 80 .....(ii) 2n = 32  +
n n
(i) / (ii) n = 16
a 1  r2 = ax +b
   40 44. (c) A.M =
a 1
 a2  a3  a4
2
ar 1  r  80 4 46. (b) Average area
1 1 20, 23, 26, 29,
 12  22  32.....n2

ir
r 2 =
A.M =
20  23  26  29 n
r=2

v.iSn
4
Put the value r in equation (ii)  n n  1 2n  1 
98 49    
a + ar2 = 40  6 
= =
4 2  
a (1 + r2) = 40 n

dnag
45. (d) AM of ax1 + b, ax2 + b, ax3 + b,
a (1 + 22) = 40
....,axn + b is n  12n  1
a (5) = 40 =
6
a=8 ax  b  ax  b  ax  b  ...  axn  b
 1   2   3 

eYrai
43. (d) G.M = (a1 × a2 × a3 × a4 × a5)1/5 n n  1 .2n  1
=
8 = (32 × 4 × 8 × n × 2)1/5 2 3
8 = (25 × 22 × 23 × n × 2)1/5
 ax1  ax2  ax3  ...  axn   b  b  ..  b Hence, (b) is the correct option.
8 = (210 × 2 × n)1/5 n
snhe
kgei
ERna
aBryn
Les
wa.th
wwM

Rakesh Yadav Readers Publication Pvt. Ltd. 57

For More Visit : www.LearnEngineering.in


For More Visit : www.LearnEngineering.in

CHAPTER

07
POWER, INDICES AND SURDS
Before we proceed to exponents are integers and q  0 are called Thus, every natural number, ev-
(Indices) and surds, it is proper ery whole number, every inte-
rational numbers and their set

r
to learn about Real numbers. is denoted by Q. ger, every rational number and

Sni
every irrational number is a real
Number System 1 2 3  6 number.
, ,  , 6  as 6   etc.

gv.i
Ex. Note:-
Natural Numbers: These are the 4 5 7  1
numbers (1, 2, 3, ....etc) that are used are rational numbers. (i) The sum (or difference) of a ra-
for counting. In other words, all posi- tional and an irrational number
The set of rational numbers

ridna
tive integers are natural numbers. is irrational.
encloses the set of integers and
The least natural number is 1 but there fractions.
is no largest natural number. Representation of Rational     32 
E.g. 4  3 , 2  5 ,  2 ,7   etc.

eeYa
The set of natural number is denoted Numbers as Decimals : The
by N. decimal form of a rational num- are all irrational.
Thus, N = {1, 2, 3,......} ber is either terminating or non- (ii) The product of a rational and an
terminating. irrational number is irrational,
Whole Numbers : The set of
numbers that includes all natu-
geisnh e.g. 4 3 ,  2 5 etc. are all irra-
17 21
ral numbers and the number E.g.  4.25,  4.2  terminat- tional.
4 5
zero are called whole numbers. Even and Odd numbers : Inte-
ing (or finite) decimal.
The set of whole numbers is gers divisible by 2 are called
Enak

denoted by W. 16 2 even numbers, while those


 5.3,  0.6  Non-termi- which are not divisible by 2 are
Thus, W = {0, 1, 2, 3, ......} 3 3
Note : Whole numbers are also known as odd integers.
nating (or Recurring) decimal.
Thus, ......, –6, –4,–2, 0, 2, 4, 6,
R

called as “Non-negative Inte- Note: If the denominator of a rational


gers”. ......etc are even integers.
number has no prime factors other And –5, -3, -1, 1, 3, 5, ....etc
aryn

Integers : All the natural num- than 2 or 5, then and only then it is are odd integers.
bers, zero, and the negatives of expressible as a terminating decimal.
Prime numbers:- A number
Les B

natural numbers are called in- Irrational numbers : The num- greater than 1 is called a prime
tegers. bers which when expressed in number, if it has exactly two fac-
I = {......, -3, -2, -1, 0, 1, 2, 3, decimal form are neither termi-
tors, namely 1 and itself.
.....} nating nor repeating decimals
are called “ Irrational num- E.g.: 2, 3, 5, 7, 11, 13, 17, 19, 23,
(i) Set of negative integers
wa. th

bers”. 29, 31, 37, 41, 43, 47, 53, 59,


= {-1, -2, -3, .....} 61, etc.
(ii) Set of non-negative integers e.g. 2 , 3, 50 , 7 ,  etc  2 is the only even number which
= {0, 1, 2, 3...} is prime.
22 Composite numbers:- Com-
wwM

(iii) Set of positive integers Note: The exact value of  is not ,


7 posite numbers are the num-
= {1, 2, 3 .......}
22 bers greater than 1 which are
(iv) Set of non-positive integers as is rational while  is irratio- not prime.
= {0, 1, 2, 3.....} 7 e.g. 4, 6, 9, 14, 15, etc.
Note: ‘0’ is definately a non-negative 22 Note: 1 is neither prime nor composite.
integer as well as a non-positive integer. nal. is the approximate value of There are 25 prime numbers
7
Rational numbers : The between numbers 1 & 100.
 . Similarly 3.14 is not an exact value
numbers which can be ex- of  . Test for Prime Numbers : Let x
pressed in Real numbers : All rational and be a given number and let k be
p irrational numbers together an integer very near to x s.t.
the form of , where p and q form the set of real numbers,
q denoted by R. k x.

Rakesh Yadav Readers Publication Pvt. Ltd. 58

For More Visit : www.LearnEngineering.in


For More Visit : www.LearnEngineering.in

If x is not divisible by any prime n


n
 1 1
number less than k, then x is 5. am  amm..........n times  am  = 4  8   
prime, otherwise, it is not prime.  4 8
6. (ab)n = anbn
E.g.: Check whether 571 is prime or 3 99
not ? n = 12  
a  an 8 8
7.    , b0
clearly, 24 > 571 b  bn
1
Ex. 4 If 22 x –1  ,then x  ?
So, we divide 571 by each prime 8  x – 3
number less than 24 which are n a n , when n is even 
8.  –a   n  1 1
2, 3, 5, 7, 9, 11, 17, 19 and 23  –a , when n is odd  Sol. 2
2 x –1 2 x –1
  2 
8
x –3 
23 
x –3
we find that 571 is not divisible
Remark:- These rules are also true
by any of them. So, 571 is a
when n is negative or fraction. 1

ir
prime number.  22 x –1 
23 x –9

v.iSn
n
Co-Prime Numbers : Two num- n 1
a –n  a 
–1n
bers are co-prime, if their H.C.F 9.  a –1      (22x–1)(23x–9) = 1
a 
(Highest common factor) is 1.  2(2x–1)+(3x–9) = 1
E.g. (2, 3), (3, 13), (5, 7) etc are co- 1 1 1  25x–10 = 1  25(x–2)= 1
  ........n times

dnag
prime numbers. =  25(x – 2) = 2º
a a a
Perfect Numbers : If the sum 10. ap/q = a1/q × p = (a1/q)p is positive  x–2=0  x=2
of divisors of a number exclud- Ex. 5 If 42x + 1 = 8x + 3 then x = ?
ing N itself is equal to N, then N integer, q  0
Sol. 42x + 1 = 8x + 3

eYari
is called a perfect number. = a1/q × a1/q × ....... p times  (22)(2x + 1) = (23)(x + 3)
E.g.: 6, 28, 496, 8128 etc.  If the index of a power is until  24x + 2 = 23x + 9
For 6, divisors are 1, 2 and 3. (i.e. 1) then the value of the
 4x + 2 = 3x + 9
6:1+2+3=6 power is equal to its base, i.e.
( base in both side is same)
28 : 1 + 2 + 4 + 7 + 14 = 28
Note: The sum of the reciprocals of
the divisors of a perfect number


a1 = a, 01 = 0
snhe
am = an  m = n when a  0 , 1
am = bm  a = b
 x=7
Ex. 6 If 3x – 1 + 3x + 1 = 90, then x = ?
Sol. 3x – 1 + 3x + 1 = 90
including that of its own is al-
kgei
Ex1. Solve the following:
ways equal to 2. (i) (5)3 3x
E.g. For 6, divisors are 1, 2 and 3.   3.3x  90
Sol. 5³ = 5 × 5 × 5 = 125 3
(ii) (–6)4
ERna

1 1 1 1 6  3  2 1 12
     2 Sol. (–6)4 = (–6)×(–6)×(–6)×(–6)= 1296 1 
1 2 3 6 6 6  3x 
  3
 = 90
(iii) (–2)5 3 
Type – I Sol. (–2)5= (–2)×(–2)×(–2)×(–2)×(–2) = –32
10
aBryn

Indices and Surds Ex.2 Solve the following expression 3x 


 = 90
(i) (32)–1/5 3
 Let n be a positive integer and a
1
5 –
be real number, then: Sol. ( 3 2 ) – 1 / 5 = ( 2 5 ) – 1 / 5 = 2 5 = 2 –1 3x
 =9
a  a  a  ......  a 3
an = 1
Les

n factors  =  3x = 27
2
wa. th

an is called “nth power of a” or “a  3x = 3³


–2/3
raised to the power n”  1  x=3
(ii) – 
where, a is called the base and  343  TYPE – II
n is called index or exponent of –2/3 –2/3
Surd: If a is rational and n is
wwM

the power an.  1   1 


  –7 –3 
–2/3 
Sol.  – 343   – 3 
E.g. 3² = square of 3, 3³ = cube of 3    7  a pos itive integer and a 1/n
etc. = (–7)–3×–2/3 = (–7)² = 49 = n
a is
 Laws of Indices: (iii) 3–3 + (–3)3
1. am × an = am+n where a  0 and  Irrational, then a is called a n
1 3 1
(m, n)  I Sol. 3 +(–3) = 3   –3 
–3 3 – 27 “surd of order n” or “nth root of a”
3 27
2. am × an × ap ........ = am+n+p+.......
For the surd a , n is called the
n
m –n 1 – 729 728
a if m>n = – surd-index or the order of the
am 1 27 27
 if n>m surd and “a” is called the
3. an a n –m Ex.3: Solve the following:
1 if m = n ( 2 ² + 2 ³ + 2 – ² + 2 –3) radicand. The symbol ' is
4. (a ) = a = (an)m
m n nm Sol. ( 2 ² + 2 ³ + 2 – ² + 2 – 3 ) called the surd sign or radical.

Rakesh Yadav Readers Publication Pvt. Ltd. 59

For More Visit : www.LearnEngineering.in


For More Visit : www.LearnEngineering.in

E.g. 5 is a surd of order 2 or a na kn a k a  a


1 
b
1 
(iii) n n and n = n
Sol.  x    x –   ÷
square root of 5. b b l b l b  y  y 

3
6 is a surd of order 3 or cube m n mn n m
(iv) a = a= a  a b
root of 6. 1  1 
 y    y –  
6  5 is not a surd as 6  5 (v)  n

a m  a 
m /n
 a m 
1/n
 n am  x  x  

is not a rational number. a b

Note: Every surd is an irrational (vi) a  a a  1  1


x   x – 
number but every irrational  y  y
(vii) a  b  ab  a b
number is not a surd.  1  1
and k. a  l. b  kl . a . b  kl. a b
n m n m mn m n y    y – 

r
 In the surd a n b , a and b are  x  x

Sni
called factors of the surd. (viii) a 2b  a b a b
(i) A surd which has unity as  xy  1   xy – 1 

gv.i
2
   
its rational factor (i.e. a = 1) (ix)  a b   a  b  2 ab

 y   y 
is called “pure surd”. a b
2  xy  1   xy – 1 
E.g.: 4
3, 2, 3 3 etc. (x)  a– b   a  b – 2 ab   
 x   x 

ridna
(ii) A surd which has a rational
factor other than unity, the (xi)  a b a – b  = a – b, a
 xy  1 .  xy – 1
b

other irrational, is called “mixed where a and b are positive ya yb

eeYa
rational numbers.  a b
surd”. e.g. 3 5, 2 7, 5 7 3
 xy  1 .  xy – 1
 A surd is in its simplest form if:
n xa xb
 If a is a surd it implies. (i) There is no factor which has nth
(i) a is a rational number. power of a rational number, a b
 xy 1  xy –1
geisnh
n
under the radical sign whose  ×
(ii)a is an irr ational
index is n, ya b
number. (ii) There is no fraction under the
 Quadratic surd: A surd of order radical sign, and x a b
a b
(iii) The index of the surd is the xy  1 xy –1
Enak

2 (i.e. a ) is called a quadratic


surd. smallest possible.

E.g.: E.g. The surd 4 3  54 is not in its


2 = 21/2 is a quadratic surd x a b  x 
a b
R

simplest form since the number   


but 4 = 41/2 is not a quadratic under the radical sign has factor y a b  y 
54 s.t. its index is equal to the
aryn

surd because 4 = 2 is a order of the surd. It simplest


3/2
Ex.8. If x x = (x3/2)x, then the value
rational number. form: of x is:
Les B

Therefore 4 is not a surd. 4


3  54  4 3. 4 54   3  5  5.  3 
4 4 3/2
Sol. xx = (x3/2)x
 Cubic Surd: A surd of order
 Similar or like Surds: Surds 3/2

3(i.e. 3  xx = x(3/2)x
a ) is called a cubic surd. having same irrational factors
wa. th

3 3
are called “similar or like surd”. 3
E.g.: 9 is a cubic surd but 27 is  x3/2 = x
2 2
not a surd because 3 27 = 3 is E.g.: 3 3, 7 3, 3, 3 e tc . ar e
5 3 9
a rational number. similar surds.  x1/2 =  x=
wwM

 Quartic or Biquadratic surd: 2 4


 Unlike surds: Surds having
A surd of order 4(i.e. 4
a ) is non-common irrational factors Ex.9 If xa = yb = zc and y2 = zx then
called a quartic surd. are called “unlike surds”.
1 1
E.g.: 3 3, 5 2, 6 7, etc. are unlike the value of  is:
E.g.: 3 is a quartic surd but 4 81
4 a c
surds.
is not a quartic surd. Sol. Let xa = yb = zc = k
Note: Each surd can be represented  1  1
a b
  x = k1/a, y = k1/b, z = k1/c
on the number line. Ex. 7.  x    x –   ÷
Now,
 y  y 
Important Formulae Based on Surds:
 y² = zx
n
(i) an  a  a
1 
b
1   (k1/b)2 = (k1/c).(k1/a)
 y    y –  
x   is equal to:
n 1 1
(ii) ab  n a .n b  x  
 k2/b = k c a

Rakesh Yadav Readers Publication Pvt. Ltd. 60

For More Visit : www.LearnEngineering.in


For More Visit : www.LearnEngineering.in

x 3x Sol.
1 1 2 Ex.14 22 = 162 , then x is equal to 2x  3y  6– z  k
  
a c b x 3x Then,
Sol. 22  162
m–n l n–l m l–m n 1 1
Ex.10. (a ) ×(a ) × (a ) 1
2x 4 2 3x 2  k x , 3  k y , 6  k –z
Sol. (am – n)l×(an – l)m × (al – m)n  2  (2 )
= a m l – nl ×a nm – l m × a l n– m n x 3x 3 x 2 We know that
 22  24.2  22 2×3=6
= a m l – n l +n m – l m +l n– m n x 3 x 2
 22  22 (Since has base in 1 1 1
= a0 = 1
both sides is equal) k x  k y  k –z
Ex.11. If ax = b, by = c and cz = a, then
the value of xyz is:  2x 2
3x 2
Same base power will be add
1 1
Sol. If ax = b, by = c, cz = a  x = 3x + 2  1

ir
 2x = –2 k x y  k –z
 ax = b

v.iSn
 x=–1 Now,
 (cz)x = b
 czx = b  (by)zx = b Ex.15. If 2x  4y  8z and xyz = 288 1 1 –1
 
 bxyz = b  xyz = 1 1 1 1
x y z
find the value of 2x  4y  6z

dnag
a b b c c a
 xa   xb   xc  1 1 1
 
E x. 12.  b   c   a  is
Sol. x y z x y z =0
x  x  x  2 4 8
equal to: 2x  22y  23z 9  8 
x x –1
2

eYari
Ex.17: If       , Find the
a b b c c a If a x  a y  a z   4   27  3
 xa   xb   xc   
Sol.  b  c   a  value of x.
x  x  x   then, x = y = z
x x –1

=
(x a – b ) ( a + b) × (x b – c ) (b + c) ×
(x c – a ) (c + a)
x (a – b )( a +b ) ×x (b– c )( b +c ) ×x (c– a )( c+ a)
x = 2y = 3z
x = 3z
2y = 3z
snhe Sol.
9  8 
   
 4   27 

2
3

 x 1 
3z a  a – x 
kgei
= x a 2 –b2
x b2 –c 2
x c 2 –a2 y=  
2
–x x –1
=
2
x a –b  b
2 2
–c  c –a2
2 2
xyz = 288 4  8  2
    
ERna

Put the value of x,y in z form 9  27  3


= x0 = 1
1 3z –2 x 3( x –1)
3z   z  288 2 2 2
Ex.13. The value of 2     
1  x b –a  x c – a 3 3 3
z 3  32  2
aBryn

1 1 –2 x 3 x –3 1
+ + is : z =4 2 2
1 x c –b
x a –b
1 x a –c
x b –c
   
Then x = 3z = 4×3 = 12 3 3
1 1 The power will be equal when
Sol. + 3z 3 4
1  x b –a  x c –a 1x c –b x a –b y= = =6 the base is equal.
2 2
Les

1 –2x + 3x – 3 = 1
+ So,
wa. th

a –c x–3=1
1 x  x b –c
1 1 1 x=4
1 1   Ex.18. If 2x = 4y = 8z,
  2x 4y 6z
xb xc xc xa
1  1  Put the value x,y and z 1 1 1 24
xa x a xb xb    , Find the
wwM

2x 4y 6z 7
1 1 1
1   value of z.
+ 2  12 4  6 6  4
x a xb Sol. 2x = 4y = 8z,
1 c  c
x x 1 1 1 2x = 22y = 23z
 
24 24 24 When base is equal then power
xa xb
 + will be equal,
x a  xb  x c x b  xc  xa 1 1
= 3 = x = 2y = 3z
24 8
xc 3z
+ Ex.16: If 2x  3y  6– z , Find the value So, x = 3z, y =
x  x a  xb
c
2
xa  x b  x c 1 1 1 1 1 1 24
 1 of x  y  z   
xa  x b  x c 2x 4y 6z 7

Rakesh Yadav Readers Publication Pvt. Ltd. 61

For More Visit : www.LearnEngineering.in


For More Visit : www.LearnEngineering.in

Put the value of x and y in z form 5


– E.g.: Which is larger 2 of 3 3 ?
331/12  10 3
1 1 1 24  19 Sol. Given surds are of order 2
   –
2
2  3z 4  3z 6z 7 312  10 3 & 3 respectively whose L.C.M is
2 6.
3 Convert each into a surd of
 3 × 10–1 = 0.3
1 1 1 24 10 order 6, as show below :
  
6z 6z 6z 7 Ex.21: The value of expression 1 3
 1/6
2  21/2  22 3
 23/6   23 
1 24  0.6
0
–  0.1
–1
3  is 1/6
6z 7 –1 3 –1  8  68
 3  3  1
 3     – 
1 24 2  2  3

r
1 2 1/6
 32/6   9 

  31/3  33 2
2z 7

Sni
0 –1
 0.6 –  0.1
 69
7 Sol. –1 3 –1

gv.i
z  3  3  1
48  3     –  Clearly,
9 > 6 8 , so 3 3  2 6
2  2  3
Ex.23: Which one is smaller out of
Ex.19: The value of expression

ridna
–1
 1  3
2 and 4
3 ?
n m –1 m –n m n –2 1–  
4 20 12 15  10 
is:  Sol. 3
2 = (2)1/3 = 24/12 = (16)1/12
16m52m n 9m –1  3–1  23   33  2–3   –31

eeYa
4n 20m –112m –n 15m n –2 and 4
3 = (3)1/4 = 33/12 =(27)1/12
Sol. 1 – 10 –9 –9 –3
16m52m n 9m –1 = 3–1 0    (Taking the LCM of surd)
3 .2 – 3 9 – 3 6 2
Hence 2 < 43 3
22n 22m –25m –122m –2n 3m –n 9n  3 2  (3 – n /2 )–2 –  27 
n
1
Ex.22: If
geisnh  Ex.24: Which one is greatest out of
×3m n –25m n –2 3 3m  2 3 27
 3 4 5
2 52m n 32m –2
4m
then the value of (m – n) is: 5, 3, 4 ?
n
 22n 2m –22m –2n –4m × 3m –n m n –2–2m 2 × 9n  3 2  (3 – n /2 )–2 –  27  1 Sol. 3
5  (5)1/3  (5)20/60  (520 )1/60  60 520
Sol. 3m 3

Enak
3 2 27
5m –1m n –2–2m –n 4
3  (3)1/4  (3)15/60  (315 )1/60  60 315
 2–2×30×5–3 3  3  3n – 33n
2n 2
1
 3m 3
 3
3 2 3
1 1 1 5
4  (4)1/5  (4)12/60  (412)1/60  60 412
R

   2n 2 n 3n
4 125 500 3 –3 1
  5 is the greatest.
3
Ex.20: The value of Expression 8  33m 33
aryn

Ex.25: Which of the following relation


 1 
1/4
33n  2 – 33n is correct A= 2 , B= 3 3 ,C= 4 4
Les B

0.3
1/3 1/6
.  . 9 .  0.81
2/3
  3 –3
27
  33m  8
–2 is 1
2
1
3
1
1 Sol. (2) , (3) , (4) 4
 0.9 . 3 .   .243
2/3 –1/2 –1/4
33n  32 – 1 –3
 3  3 LCM 2, 3 and 4 = 12
33m  8
wa. th

convert each into a surd of order


1/4  33n–3m = 3–3 12
 1 
1/3 1/6 2/3
0.3 .  . 9 .  0.81  3n – 3m = –3
 27  1 6

1 4
×
1 3

Sol. –2  m–n=1 (2)2 6
, (3)3 4
, (4)4 3
1
 0.9 . 3 .  .243
2/3 –1/2 –1/4
wwM

TYPE – III
 3 6 4 3

 Comparison of Surds: (i) If two (2)12 , (3)12 , (4)12


1/3 3/4 2/3 surds are of the same order,
 3  1 1/3  81  1 1 1
     3  .  then the one whose radicand is (26 )12 , (34 )12 , (43 )12
 10   3   100  larger, is the larger of the two.
2/3 1/2
  9  1 2 –5/4 1 1 1
    . 3  3 E.g.: 3
19  3 15, 7  5, 3 9  3 7 (64)12 , (81)12 , (64)12
 10   3 
etc. 1 1 1
Break the power of 3 & 10 separately (ii) If two surds are distinct order, clearly (81)12 > (64)12 = (64)12
1 1 3 8
 – 
1 4
– –
we change them into the surds
1
33 3 4 3
 10 3 3
of the same order. 1 1

 4 1 5 2
so, (3)3 > (2)2 = (4)4
3
2– –
2 4

3
This order is the L.C.M. of the
3  10 orders of the given surds. B>A=C

Rakesh Yadav Readers Publication Pvt. Ltd. 62

For More Visit : www.LearnEngineering.in


For More Visit : www.LearnEngineering.in

Ex.26: Arrange the following in 2 It is cleared that


descending order.  10  3  = 13+2 30
2 2
So, It is cleared that >
3 – 2, 4 – 3, 5 – 4, 2 – 1  21  19  23  21

3– 2 3 2
Sol. 3– 2
1

3 2
13+2 22 <13+2 30 <13+2 36 < Then  21  19 > 23  21   
13+2 40 <13+2 42 Ex.31: Arrange the following in
3–2 1
  Then decendinding order.
3 2 3 2 2350,5200,3300,4250
11  2 < 10  3 < 9  4 Sol. 2350,5200,3300,4250
1
Similarly, 4– 3 , < 8 5< 6 7 Power in same form

ir
4 3
Ex.29: Arrange the following in (27)50, (54)50,(36)50,(45)50

v.iSn
1 descending order. (128)50,(625)50, (729)50,(1024)50
5– 4
5 4 So,
8 5, 6 7, 9 4, 4250 > 3300>5200>2350
1 Ex.32: Arrange the following in
11  2 , 10  3

dnag
and 2 – 1 
2 1 descending order.
Sol. We use square method
As we know, if the numerator is 272,536,448,360
2
Power in same form
same then the fraction whose
de nominator is lar ge r the
 
8  5 = 13-2 40
(26)12,(53)12,(44)12,(35)12

eYari
fraction will be lower. Hence the 2 (64)12,(125)12,(256)12,(243)12
correct order of descending is.  
6  7 = 13-2 42 Then
(256)12 >(243)12 >(125)12 >(64)12
 2 –1    3– 2    4– 3   5– 4  2
   448>360>536 >272
Ex .27: Whic h is g re ater than

  
8  5 or 13  10  
9 4

11  2
snhe 
= 13-2 36

2
= 13-2 22
Ex.33: Arrange the following in
descending order.
3
3, 4
4, 6
6, 12
12
kgei
Sol. Rationalisation of Surds:
2

8 5
 
10  3 = 13-2 30 Sol.
1 1 1
3 3 , 4 4 , 6 6 , 1212
1

  8  5  
3
 8– 5 
ERna

8 5 It is cleared that LCM of 3,4,6 and 12 = 12


Then
13-2 22 >13-2 30 >13-2 36 >
1 4 1 3 1 2 1

 13  10 = 3 33 4 , 4 4 3 , 6 6 2 ,1212
13-2 40 >13-2 42
 13  10  ×
aBryn

 13  10 13  10 Then
1 1
(34) 12 ,(43) 12 ,(6²) 12 , 1212
1 1

So, It is clear that 11  2 > 10  3 > 9  4 It is cleared that


3 3 1 1 1 1
 > > 8 5> 6 7 (81) 12 ,(64) 12 ,(36) 12 ,1212
8 5 13  10
Les

Ex.30: Which is greater than 1 1 1 1


    (81) 12 >(64) 12 >(36) 12 >1212
wa. th

Then 8 5 > 13  10
 23  21 or   21  19 
Ex.28: Arrange the following in
Sol. Rationalisation of Surds: So,
ascending order.
1 1 1
8 5 , 6 7 , 9 4 ,  23  21  1
3 3 > 4 4 > 6 2 > 1212
wwM

11  2 , 10  3
 23  21 ×
23  21 TYPE – IV
Sol. In this type questions we use  Rationalisation of Surds: If the
square method 2
= product of two surds is rational,
2 23  21
then each of them is called the
So,  8 5  = 13+2 40
(R.F.) rationalising factor of the
2
 21  19  other.
 6 7  = 13+2 42  21  19 ×   21  19 
E.g.: 5 7 × 7 = 5 7  7 = 5 × 7 = 35
2
 9 4  = 13+2 36 2  7 is a rationalising factor of

 11  2 
2
= 13+2 22
=  21  19  5 7.

Rakesh Yadav Readers Publication Pvt. Ltd. 63

For More Visit : www.LearnEngineering.in


For More Visit : www.LearnEngineering.in

 Rationalising factor (R.F) of the Sol. x = 3– 2 5 –2


1–
1 Ex.38: The value of is:
surd n
a is a n 2 2 5 2
Difference =  3 –  2
R.F. of the surd a  b is

= 3–2=1 5 –2  5 –2   5 –2 
Sol. = ×
a  b . 52  5  2  5 – 2
1
Then,  3 2
E.g.:  3 2  3– 2  x
2

2 2 1 =
 5 –2 
=  3 –  2  3 – 2 1 Ex.36: If x =
5–2 6
, Find the value 5–4

r
54–4 5
  
3 – 2 is a R.F. of  3 2  of
1
=? = 9–4 5

Sni
x 1
Note: The R.F. of a given surd is not
Ex .39: Find the v alue of

gv.i
unique. 1
Sol. x=
Ex.34: If x = 7  4 3 find the value 5–2 6 5 12 30
 .
Difference of nos. 6 6 25 5
1

ridna
of 2
x 2
= 5  – 2 6  Sol.
5

12 30
6 6 25 5

5
6 6

12 6 5 2
25 5

5
Sol. x = 7  4 3
= 25 – 24 = 1

eeYa
TYPE – V
1 1 1 1 SOME USEFUL RESULTS
= then,  =
x 74 3 x 52 6 5–2 6
(Multiplying numberator and (a) If y = x  x  x  .......
Note:- (i) If x =
denominator by conjugate)
geisnh 3 2

1 1  1  4x
 1 7–4 3  3– 2 then, y =
  then 2
x 74 3 7–4 3 x
(ii) If x = 5 – 2 6
Enak

1 7–4 3 Ex.40: y  7  7  7......... ,


=  2
x  7 2 – 4 3 then which of the following is
  then
1
52 6 true ?
x
R

(a) y = 3 (b) 3 < y < 3.5


1 7–4 3 1 (b) y = 7 (d) Greater than 4
= 
x 49 – 48 (iii) If x =
aryn

7–4 3
1  1  4x
1 Sol. y =
1 1 2
Les B

= 7–4 3
x then =
x 74 3 Here, x = 7
Alternate:-
 When the difference between two
R.F. of the surd 1 1 47
a  b is numbers is equal to 1 then then y =
2
wa. th

reciprocal of this number only


a  b .
change their sign. (“+” change into
when difference of square of 1  29
“–”, and “–” change into “+”) =
2
both number a and b =1
2 3
wwM

x = 74 3 Ex.37: Find the value of  29 lies between 5 or 6


2– 3
2 1 5
difference = (7)² – 4 3   Sol.
2 3 2 3 2 3
= ×
So, y 
2
= 3
= 49 – 48 = 1 2– 3 2– 3 2 3
1 6
1 1 (Multiplying numerator and or, y = 3.5
Then  = denominator by conjugate) 2
x 74 3 7 – 4 3 So, 3 < y < 3.5 is correct.
2
Ex.35: If x = 3 – 2 , find the vlaue =
2  3  =
434 3 Ex.41:
4–3 1
1 If y = 12  12  12  ...... ,
of =?
x =74 3 then y = ?

Rakesh Yadav Readers Publication Pvt. Ltd. 64

For More Visit : www.LearnEngineering.in


For More Visit : www.LearnEngineering.in

48918 3612
1  1  4 12 –1  7
Sol. y = y =3 Sol. 5 3612
 5 489
2 2
3
Note:
1  49 583  5 4
y=
2 if y = x – x – x – ....... 24
3 99
5 4 = 54
1 7 and x = n(n + 1) then y = n
= =4 Alternate:
2 3
we have, x = 12 = 3 × 4 = n(n+1) Ex.50: 2 4 23 4 2 3 4....... will be
Note:
 y = 3. equal to
if y = x  x  x  ....... Ex.45: If y  42 – 42 – 42...... ,

ir
3
and x = n(n + 1) then y = (n + 1) Sol. Let x = 2 4 23 4 2 3 4.......

v.iSn
the value of y = ?
Alternate: Sol. We hav e, x = 42 = 6 × 7
we have x = 12 = 3 × 4 = n (n + 1) = n(n+1) x= 2 3 4x (Squaring both sides)
 y=4  y=6
x2 = 23 4x (Cubing both sides)

dnag
Ex.42: If 20  20  20  ...... (c) If y  a a a...... x6 = 23.4x  25.x  x 5  25
then the value of y = ? then y = a x =2
Sol. We have x = 20 = 4 × 5 = n (n + 1)
TYPE – VI

eYari
 y=5 Ex.46: y  5 5 5.......
 Square-root of an Irrational
(b) If y = Sol. y = 5
x – x – x – ........ number:
As we know that,
(d) If y  a a a ......n ,
then y =

Ex.43: If y =
–1  1  4x
2

9 – 9 – 9........
then y = a
snhe 2n –1
2n
(a+b)² = (a²+b²)+2ab

  2  3 
2
5  2 6
(a²+b²) (2ab)
kgei
then which of the following is Ex.47: If y  a a a a , find the 52 2 3
true ? 52 6 =
value of y. (2ab)
(a) y = 3 (b) 2.5< y < 3
ERna

24 –1
(b) y = 9 (d) Greater than 4 24
 a = 2 &b  3
Sol. n = 4, then y  a
& a² + b² = 5
–1  1  4x
Sol. y = 15 2
2 y = a 16  52 6 =  2 3 
aBryn

Here, x = 9
Ex.48: If y  3 3 3 3 3 , find  a+b = 52 6 = 2 3
–1  1  4  9
then y = Examples:
2 the value of y ?
Sol. n = 5 (i) 7 – 4 3  7 – 2 2 3
Les

–1  37
= 25 –1
wa. th

2 a b
Then, y = 3 25
2
 37 lies between 6 or 7 31
& a² + b² = 2² + 3 =7
y = 3 32 2
–1  6
So, y = = 2.5 = 2 – 3   2 – 3 
wwM

2  z 
n

–1  7
(e) If y   m

 
p
ax  ,

(ii) 5 21
for making it 2ab
or, y = =3 (2ab)
2
x z n
So, 2.5< y < 3 is correct. m p

Ex.44:If y = 1 2 – 1 2 – 1 2 – ....... ,
then y = a 5 – 21  2
 
10 – 2  3  7
18 2 2
then the value of y = ?  9 
 12  6 
8

–1  1  4  12
Ex.49: If  
 
  3
54 


 × =
1
10 – 2 3 7 
Sol. y   2
2
12  a  7 & b  3 
 6 
y
–1  49
2
9  8

5
4 3



 a 2  b 2 10



Rakesh Yadav Readers Publication Pvt. Ltd. 65

For More Visit : www.LearnEngineering.in


For More Visit : www.LearnEngineering.in

Ex.55: Find the square root of


1 2 7– 3 2 10
= 7– 3    Ex.58: If x  , find the value of
2 2 139 – 80 3 . 7
Ex.51: Find the square r oot of Sol. 139 – 80 3 1 x  1– x
7  4 3  . (75+64)
25 3  8 1 x – 1– x
2
2 10 7  2 10
Sol. 74 3 = 5 3 –8  Sol. 1  x = 1 
7
=
7
(a²+b²) (2ab)
Square root
2
74 3  5  2
2 1  x   
= 4+3 2  2 3 Then, 5 3 –8   7 

r
2 Square root of both sides,

Sni
= 2  3  5 3 –8
2
Ex.56: Find the square root of  5 2

gv.i
2
1 x =  
= 2 3  = 2 3 8–4 3  7 

Ex.52: Find the square r oot of Sol. 8–4 3

ridna
 5  2
13 – 4 3 = 
2 4–2 3  1  x =  
7 

Sol. 13 – 4 3

eeYa
(a²+b²) (2ab)  5 – 2
= 2 4–2 3 Same as 1 – x =  
7 

13 – 4 3
3+1
= (12+1) 1 x  1– x
2  2 3 1
geisnh
2 3 × 1
1 x – 1– x
2
=  12 – 1  = 2  3 –1 
2 Put the value,

Square root of 5 2 5– 2
Square root of 
Enak

2 7 7
=
Then  12 – 1  = 12 – 1 2 5 2 5– 2
Ex.53: Find the square root of
Then, 2  3 –1  7

7
R

76  10 3 = 2  3 –1  5 2 5– 2 2 5

=
aryn

Sol. 76  10 3 5 2– 5 2 2 2
75+1 Ex .57: I f x  – 3  3  8 7  4 3 ,
5
Les B

2  5 3 1 then find the value of x. =


2
2
= 5 3 1  Sol. x  – 3 38 74 3 Ex.59: If x = 38  5 3 , Find the value
Square root of x.
wa. th

Then = 5 3 1
2
 x  – 3 38 2  3  Sol. x  38  5 3
Multiply and divided by 2
= 5 3 1  x – 3  38 2 3  
wwM

76  2  5  3
x
Ex.54: Find the square of 33 – 4 35 2
 x  – 3  3  16  8 3 2
Sol. 33 – 4 35  5 3 1
(28+5) x   
2 2 7  5
 x  2 
– 3  19  8 3
2
= 2 7– 5  2  5 3 1
2

Square root  x – 3  4 3  Then, x  




2 
2
Then, 2 7– 5   x – 3  4 3   5 3 1
x 
x = 2 2
2 7– 5

Rakesh Yadav Readers Publication Pvt. Ltd. 66

For More Visit : www.LearnEngineering.in


For More Visit : www.LearnEngineering.in

Ex.60: If x 
3
, Find the value of  3 1  
3 –1 x 2 a x 2 b
 2
2  x –2 a x –2 b
3  3 1  3  3 –1 
1 x 2 ab
1 1 2 (b) If = x
Sol. 1  x  1 
3

2 3   
3

3

3
a b
2 2 x a x b
Multiply and divided by 2 3  2
Ex.62: If x  , Find the value of x– a x– b
2
42 3
1 x  1 x 1– x 4 15
4  Ex.63: If = x , the value of
1 1 x 1 – 1 – x 5 3

ir
2
 3 1

v.iSn
1+ x =  2  3 x  20 x  12
  Sol. When x = 
2 x – 20 x – 12
Square Root of Both Sides
3 1 2 2 5  3
2
Then, 1 x 

dnag
 3 1 2 Sol. x
Then, 1 x =   5 3
 2  (Explanation above question 61)
Put the value: x 2 3
3 1  

eYari
1 x  3 3 2 5 5 3
2 1
1–
 2  2 x 2 3
3 3 1 3 –1   also
Ex.61: If x = , Find the value of 1 1– 20 5 3
2 2 2
1x

1– x
1 1x 1 – 1 – x

2 3
2 
2– 3
2
snhe 
x
12

2 5
5 3
2 3 1 2 – 3 1 Ap plying c om pone nd o &
kgei
3
Sol. When x = 2 2 dividendo
2
x  20 3 3  5 3 3 5
3 1 2 3 2– 3 = =–
ERna

1 x    =1 
Then, x – 20 3– 5 5– 3
2 3 3 3– 3
(Explanation above question 61) x  12 33 5
Put the value:
=
2  33 – 3  2 – 33  3 Similarly,
x – 12

5– 3
aBryn

3 1 3 –1 3  3 3 – 3
2 2 x  20 x  12
  
 3 1   3 – 1 6– 2 3 3 3 –362 3 – 3 3 – 3 x – 20 x – 12
1  
 2  1–  2 
 2
= 2
    3 –  3 –3 3 – 5  3  3 5
Les

3 1 3 –1 =
6 6 5– 3
wa. th

 2  2 = = =1
2  3 1 2 – 3 1 9–3 6
2 5 –2 3
2 2 TYPE – VII =  2
 5– 3 
3 1 3 –1 Useful Result:-
wwM

Note: In these type of questions there


2  2
 4 ab is maximum possibility of having
 3 3  
3– 3  (a) If x 
a b answer '2'.
2 2

Rakesh Yadav Readers Publication Pvt. Ltd. 67

For More Visit : www.LearnEngineering.in


For More Visit : www.LearnEngineering.in

EXERCISE

1 10. The simplified form of 19. Simplify :


1. The value of 52 6 – is: 1

52 6 3/2 –3/2  2 –2 2


16  16
 is : 64 3  2


8
0


(a) 2 2 (b) 2 3
4097 1
(c) 1  5 (d) 5 –1
(a) 0 (b) 64 (a) 0 (b) 1 (c) 2 (d)
2

 3 2  16 1
 –  (c) 1 (d) 4097
 2 3 2 – 3 

r
20. The v alue of
2. Simplify : 


2 –5 3


 11. (0.01024)1/5 is equal to : 12 – 140 –

Sni
 
(a) 4.0 (d) 0.04 1 2

gv.i
1 (c) 0.4 (d) 0.00004 – is :
(a)
2
–5 3 (b) 2–5 3
–2
 8 – 60  10  84

(c) 1 (d) 0 – 2 1  (a) 0 (b) 1 (c) 2 (d) 3


12. 64
  3  
  is equal to :

ridna
4  21. Which of the following number is the
0.16 0.04 least ?
3. The value of 243  243
  is
1 2
equal to : 1 0.5 , 0.49 , 3 0.008 , 0.23
(a) 1 (b) 2 (c) (d)

eeYa
2 16
1 (a) (0.5)2 (b) 0.49
(a) 0.16 (b) 3 (c) 3 (d) 0.04
 1 2 1– 2  (c) 3 0.008 (d) 0.23
4. The simplification of 13.
   simplifies to :
 5 3 5– 3 22. Arrange the following in descending or-
 
0.06  0.06  0.06 – 0.05  0.05  0.05
geisnh der : 3 4, 2, 6 3, 4 5
0.06  0.06  0.06  0.05  0.05  0.05 (a) 5 6 (b) 2 5 6
(a) 3 4 6
4  5  2  3
(a) 1 (b) 0.1 (c) 5 – 6 (d) 2 5–3 6
(b) 3
5  3
4  63  2
(c) 0.01 (d) 0.001
Enak

2  3 2– 3 3 – 1 (c) 2  45  3 4  6 3
5. Simplify : 14.
    simplifies to:
2 – 3 2 3 3  1
  (d)2  34  45  6 3
0.05  0.05  0.05 – 0.04  0.04  0.04 23. The value of
R

0.05  0.05  0.002  0.04  0.04 (a) 2– 3 (b) 2 3 0.13 0.07


(a) 1 (b) 0.1 (c) (d)
243  243
aryn

16 – 3 40 – 3 0.25 0.075 0.2


(c) 0.01 (d) 0.001 7  49  343
2 2
Les B

6. Which one of the following is the least?  5  3  5 – 3 3 7 2


15. 

 +
 

 is equal to: (a) (b) (c) 1 3 (d) 2
5 – 3 5  3 7 3 7
3, 3 2, 2 and 3
4   7
24. The value of :
(a) 64 (b) 62 (c) 66 (d) 68
(a) 2 (b) 3 2 (c) 3 (d) 3 3
– 3+ 3+8 7+4 3 is
wa. th

16. (7.5 × 7.5 +37.5 +2.5 × 2.5) is equal to:


7. Which one of the following is the biggest (a) 100 (b) 80 (c) 60 (d) 30 (a) 1 (b) 2 (c) 3 (d) 8
?
3 0.004096 is equal to
3 4 , 4 6 , 6 15 , and 12 245 . –4 25.
 8  3
17. 
 
 simplifies to : (a) 4 (b) 0.4
wwM

34
125  (c) 0.04 (d) 0.004
(a) (b) 46
625 625 2.3  2.3  2.3 – 1
6 15 12 245 (a) (b) 26. is equal to
(c) (d) 16 8 2.3  2.3  2.3  1
625 16 (a) 1.3 (b) 3.3 (c) 0.3 (d) 2.2
8. Simplify : (c) (d)
32 625 27. The ascending order of
4 4 18. The value of (2.89)0.5,2–(0.5)2, 3 and
3 6 9  3 6 9  3
0.008 is
 5   5 
(a) 2–(0.5)2, 3, 3
0.008 , (2.89)
0.5
     12 – 8 3  2 is : 0.5 2
(a) 52 (b) 54 (c) 58 d) 512 5  24 (b) 3
0.008 ,(2.89) , 3 , 2 – (0.5)
0.5 2
9. If 3x+8 =272x+1, the value of x is : (a) 6– 2 (b) 6 2
(c) 3
0.008 , 3 , (2.89) , 2– (0.5)
(a) 7 (b) 3 (c) –2 (d) 1 (c) 6 –2 (d) 2– 6 (d) 2
3 , 3 0.008 , 2 – (0.5) ,(2.89)
0.5

Rakesh Yadav Readers Publication Pvt. Ltd. 68

For More Visit : www.LearnEngineering.in


For More Visit : www.LearnEngineering.in

28. is equal to  1 1  47.


2  2  2  ........    12  12  12  ....... is equal to
37.

 2 3– 5 2 – 3 – 5 
(a) (b) 2 2 (c) 2 (d) 3 (a) 3 (b) 4 (c) 6 (d) 2
2 in simplified form equals to :
3
48. If a = , t hen t he v alue of
29. The value of 3 3.5  3 2.5 (a) 1 (b) 2 (c)
1
2
(d) 0
2

2 2 1  a + 1 – a is :
 3 3.5
 – 3 8.75  3 2.5    is : 38. 
–2
–2  
  is equal to :
–2 (a) 3 (b)
3
2
(a) 5.375 (b) 1
(c) 6 (d) 5 (a) 16 (b) 8 (c) –8 (d) –1 (c) 2 3 (d) 2– 3
30. The value of 39. Which is the greatest among

ir
5 1 5 –1
–3 –3 49. If a = , b= , the value of
   

v.iSn
19 – 17 , 13 – 11 , 7– 5 5 –1 5 1
3  2 2  3 – 2 2 is
 a 2  ab  b 2 
(a) 189 (b) 180 (c) 108 (d) 198 and  5 – 3 ?  
a 2 – ab  b 2  is
 
5 3 3 2 2 (a) (b)

dnag
31. – + is equal to: 19 – 17 13 – 11 3 4 3 5
3 2 5 2 5 3 (a) (b) (c) (d)
(c) (d) 4 3 5 3
7– 5 5– 3
(a) 0 (b) 2 15  2 3 1 
5 3 5– 3 50. 
 –  
 is
(c) 2 10 (d) 2 6 40. If x = and y = , then

eYari
 5 3 6– 3 6 5
5– 3 5 3
equal to
3 3 (x+y) equals:
0.96 – 0.1 (a) 8 (b) 16 (a) – 2 6 (b) – 2 5
32. 2 2 is simplified to
(c) – 2 3 (d) 0
0.96  0.096  0.1 (c) (d) 2  5  3
:
(a) 1.06
(c) 0.86
(b) 0.95
(d) 0.97
2 15

0.25 is equal to
(a) 250
snhe
41. 0.75 × 0.75 – 2 × 0.75 × 0.25 + 0.25 ×

(b) 2500
51. 
(a)
2  7 – 2 10

2
 is equal to

(b) 7
kgei
(c) 2.5 (d) 0.25 (c) (d)
5 2 5
33. When 4  7   is presented in the form of  1 1 1 
perfect square it will be equal to : 52. If x =1 + 2  3 , then the value of
42. 3 
   
 is equal to
ERna

2  3 3 3 3  3
 7 1
 1 
2    (a) 1 (b) 3 x 
 
 is
(a)
2  7 (b) 
 2 2
 (c) 3 3 (d) 3– 3
 x – 1

2 (a) 1  2 3 (b) 2 3
 1  2
aBryn

 43.  1  is equal to
(c)
2



 7 1  


(d)
 3  4   9
  44 2
2 
 
(c) 3  2 (d) 2 3 – 1
53. If m and n(n > 1) are whole numbers such
34. The simplified form of 2.2
8 –    that mn =121, the value of (m – 1)n+1 is
  2 –2   (a) 1 (b) 10
2 7 5   2  
 –
is :   (c) 121 (d) 1000
7 5 12 – 5 12 – 7 
 
Les

1 1 1
(a) 5 (b) 2 (c) 1 (d) 0 54. –  –
(a) 32 (b) 8 (c) 1 (d) 0
wa. th

3– 8 8– 7 7– 6
1 1 1
35.   3 2 2 6 2 3
3 4 4 5 5 6 1 1
44. –  is equal to + =?
6 3 3 1 62 6– 5 5–2
1 1 1
   is (a) 3 (b) 2 (c) 0 (d) (a) 5 (b) 4 (c) 3 (d) 2
6 7 7 8 8 9 3
wwM

45. Greatest among the numbers  2 1 1 


(a) 3 (b) 3 3 
55.     2 – 2 2
 is
3  62 7 6 8– 7 
9, 3, 4 16, 6 80 is equal to
(c) 3 – 3 (d) 5– 3
36. Simplify % (a) 3 9 (b) 3 (a) 0 (b) 2 2
1 1
– (c) 4 16 (d) 6 80 (c) 2 (d) 2 7
100 – 99 99 – 98
–1
46. Given that 3 =1.732, the value of  –2 
+
1

1  1 2  
98 – 97 97 – 96 56. – 
    is equal to :
3 6  2   
is : 
  
1 5 3 – 2 12 – 32  50
+.... +
2– 1 (a) 4.899 (b) 2.551 1 1
(a) 10 (b) 9 (c) 13 (d) 11 (a) (b) 16 (c) – (d) –16
(c) 1.414 (d) 1.732 16 16

Rakesh Yadav Readers Publication Pvt. Ltd. 69

For More Visit : www.LearnEngineering.in


For More Visit : www.LearnEngineering.in

57. The greatest number among 260,348,436 (a) 1, 2 (b) 1, 3


and 524 is 81. 10  25  108  154  225 =?
(c) 2, 1 (d) 2,3
(a) 260 (b) 348 (c) 436 (d) 524 3
8
68. Let 3 a  3 26  3 7  3 63 then
(a) 8 (b) 4 (c) 1/2 (d) 2
58. 3 3 3..... is equal to (a) a < 729 but a > 216 82. The Simplified value of
(b) a < 216
(a) (b) 3 6 2 6 –2 2 2
3 (c) a > 729 – –
(d) a =729 2  2 3 2 – 2– 3 2 2
(c) 2 3 (d) 3 3
59. The greatest among the numbers 6 1 1 (a) 2 6 (b) 2 (c) 3 (d) 0
69. 2    equals to
3 3 2 3 3 –2
0.09, 3 0.064 , 0.5 and 5 6²  7²  8²  9²  10²
83. is equal to@
(a) (b)
(a)  2 3   (b) 
– 2 3  7 4 3 – 4 2 3

r
3
0.09 0.064
(c) 1 (d) 2
(a) 330 (b) 355

Sni
3
(c) 0.5 (d) 43 3
5 (c 305 (d) 366
70. If = A + B , then B – A is

gv.i
60.The greatest of the following numbers 74 3 84. (3x –2y) : (2x +3y) = 5 : 6, then one of
2
0.16, 0.16 , 0.16
  , 0.04 is
(a) – 13 (b) 2 13 2
3x + 3 y 

ridna
(a) 0.16 0.16 (b) (c) 13 (d) 3 3 – 7 the value of  3  is
3 y
(c) 0.04 (d) (0.16) 2 71. The smallest among t he number s  x – 
61. The greatest of the numbers 2250,3150,5100 and 4200
(a) 4200 (b) 5100 1 1

eeYa
2
8, 4
13 , 5 16 , 10 41 is : (c) 3150 (d) 2250 (a) (b) 5 (c) (d) 25
72. Find the value of 25 5
(a) 4
13 (b) 5
16 85. The value of
(c) 10 (d) 2 30  30  30  ....
41 8 1 1 1 1
 
(a) 5
geisnh (b) 1 2 2  3 3  4
+
4 5
1 1 1  3 10
x
62. If 2 = 3 =6 then y –z 
x  y  z 
 is equal (c) 6 (d) 7
1 1 1 1
     
3
to
73. The value of 2 4 23 4 ................... is 5  6 6  7 7  8 8 9
is
Enak

3 1 (a) 2 (b) 22 (c) 23 (d) 25


(a) 0 (b) 1 (c) (d) – 74. 553 +173 –723 +201960 is equal to (a) 2 (b) 4 (c) 0 (d) 1
2 2
(a) – 1 (b) 0 (c) 1 (d) 17 86. The value of
63. 2 3 40 – 4 3 320  3 3 625 – 3 3 5 is equal to
R

n
5  32n 1 72  72  72  .... is
(a) –2 3 340 (b) 0
75. The value of
243 is
n
9  3n –1 (a) 9 (b) 18 (c) 8 (d) 12
aryn

(c) 3 (d) 3
340 660 1
(a) 3 (b) 9 (c) 6 (d) 12
87. The value of
64. The value of 40  9 81 is 76. The simplified value of 1 2  3
Les B

1
(a) 111 (b) 9  3  110  12 12 – 2 5 – 3 is +
1 2  3
is:
(c) 7 (d) 11 (a) 16 (b) 88 (c) 176 (d) 132 (a) 2 (b) 3
77. If 2 n – 1 + 2 n+1 = 320, then the value of
 x – 24  75  50  = 1, then the
wa. th

65. If
75 – 50
n is
(a) 6 (b) 8 (c) 5 (d) 7
(c) 1 (d) 4  3  2 
value of x is 88. The value of the expression
3

(a) (b) 5 78. If 3 2 a 2 , then t he
5 5 5 5  5 5
6  6 6  ...  upto is
wwM

value of a is
(c) 2 5 (d) 3 5 (a) 4 (b) 5 (c) 6 (d) 8 (a) 30 (b) 5
66. Evaluate 79. The value of (c) 3 (d) 2
4 –3 –3 89. The value of
20  12  3 729 – – 81 –2
5 – 3  32 2
  3 2
 is
3 7 5 5 2 2
(a) 198 (b) 180 (c) 108 (d) 189   is:
(a) 2 (b) 3 5  2 2  7 7  5
80. A tap is dripping at a constant rate into a
(a) 1 (b) 0
container. The level (L cm) of the water in
(c) 0 (d) 2 2
the container is given by the equation L = 2 (c) 2 3 (d) 7
67. If a, b are rationals and a 2 + b 3 – 2t, where t is time taken in hours . Then
the level of water in the container at the level 90. If 11 n  112  343 , then the value
= 98 + 108 – 48 – 72 , then the of water in the container at the start is
of n is:
values of a, b are respectively (a) 0cm (b) 1cm (c) 2cm (d) 4cm (a) 3 (b) 11 (c) 13 (d) 7

Rakesh Yadav Readers Publication Pvt. Ltd. 70

For More Visit : www.LearnEngineering.in


For More Visit : www.LearnEngineering.in

ANSWER KEY
1. (a) 10. (b) 19. (c) 28. (c) 37. (c) 46. (d) 55. (d) 64. (c) 73. (a) 82. (d)
2. (c) 11. (c) 20. (a) 29. (c) 38. (a) 47. (b) 56. (a) 65. (b) 74. (b) 83. (a)
3. (b) 12. (a) 21. (c) 30. (d) 39. (d) 48. (a) 57. (b) 66. (c) 75. (b) 84. (d)
4. (c) 13. (c) 22. (a) 31. (a) 40. (a) 49. (b) 58. (b) 67. (a) 76. (c) 85. (a)
5. (c) 14. (c) 23. (a) 32. (c) 41. (d) 50. (c) 59. (d) 68. (a) 77. (d) 86. (a)
6. (b) 15. (b) 24. (b) 33. (c) 42. (b) 51. (c) 60. (b) 69. (d) 78. (a) 87. (c)
7. (a) 16. (a) 25. (b) 34. (d) 43. (d) 52. (a) 61. (d) 70. (c) 79. (a) 88. (c)
8. (b) 17. (a) 26. (a) 35. (c) 44. (c) 53. (d) 62. (a) 71. (b) 80. (b) 89. (b)
9. (d) 18. (c) 27. (b) 36. (d) 45. (a) 54. (a) 63. (b) 72. (c) 81. (d) 90. (d)

ir
v.iSn
SOLUTION

dnag
4. (c)
1. (a) 1  12 256 , 12 216 , 12 225 , 12 245
52 6 – 0.06  0.06  0.06 – 0.05  0.05  0.05
52 6  Biggest = 3 4
0.06  0.06  0.06  0.05  0.05  0.05
1 4 4

eYari
0.06³ – 0.05³ 3 6 9  3 6 9 
3  2 –
   8. (b)  5   5 
 3  2 0.06²  0.06  0.05  0.05²    
a³ – b³
   4 4
2 a ²  ab  b ²  911   911 
 52 6   3  2
3 2  


 a² +b² +2ab = (a+b)²




 
snhe
a – ba ²  ab  b ²
a ²  ab  b ²

5 6 3 

 1


4
 1
5 3 6 

4


1 3 – 2  a–b
kgei
 3 2 –   5 2  5 2 
  So, a = 0.06, b = 0.05     
 3 2 3 – 2   
 0.06 – 0.05  0.01   

  5² × 5²
3 – 2 5. (c) 0.05  0.05  0.05 – 0.04  0.04  0.04
ERna

3 2–    52+2 = 54
   0.05  0.05  0.002  0.04  0.04
 3–2  9. (d) 3x + 8 = 272x+1

 3 2– 3 2  2 2 
0.05 ³ – 0.04 ³ 3x + 8 = (3³)2x+1
3x + 8 = 36x+3
0.05²  0.002  0.04² x + 8 = 6x + 3
aBryn

3 2 (Description: same as above question) 5x = 5, x = 1


– a = 0.05, b = 0.04
2. (c) 2  3 2 – 3  3 
 a–b  0.05 – 0.04  3 –3   2 1 
2–5 3 16 2  16 2  16  3 
 0.01 10. (b)  
  
 

6. (b) 3 , 32 , 2 , 34
   16 2 
   
3 2– 3 – 2 2 3
Les

1 1 1 1  3 
 22 1 
wa. th

 2  32 – 3  32 , 23 , 22 ,
43
 

4  3 
²   4³ 
1
2–5 3 (take LCM of 3 &2) 4³
 4 2
3 2 3 2 4097
, , ,
36 26 26 4 6
6–3 3 –4–2 3 

64
wwM


2 3  2– 3  2–5 3   6 33 , 6 2² , 6 23 , 6 4²
11. (c)
1
(0.01024)5
2–5 3  6 27 , 6 4 , 6 8 , 6 16
= =1 1
2–5 3  3 2 (Least)  (0.45 )5
3.(b)(243)0.16 ×(243)0.04
7. (a) 3 4 , 4 6 , 6 15 , 12 245 51 = 0.4
0.16+0.04
a m  a n  a m n  
0.4 5
 (243)   1 1
1 1
, 4 , 6 , 12 –2
 1 –2
 2430.20  3
4 6 15 245
(take LCM of 3, 4, 12 & 6) 12. (a) 64 3  
  
20
4 
243100
 4 3 2 1
, 12 , ,
1512 24512

1 412 6 –2  1 –2
 5 243 = 3 4³ 3  
   
243 5
 
 12 44 , 12 63 , 1215² , 12 245 4 

Rakesh Yadav Readers Publication Pvt. Ltd. 71

For More Visit : www.LearnEngineering.in


For More Visit : www.LearnEngineering.in

–2  1 –2 2 1
 4 
 
  5 – 3 4 – 15  2 2
4  Similarly:   =
 5

3 19. (c)
64 3  2 –2 8
0
 4 15  
 1 2  1 –2  1 2–2 Thus, the expression.

 
 
   
   = 
  
4  4  4  1
4  15 4 – 15 2
  2 2
  4 33   1
 1 0 4 – 15 4  15    
 


 1
 
   =1  2  
4   
16  15  8 15  16  15 – 8 15
13. (c)  1 2  1
1– 2  16 – 15  1 2
 
 5  3  5 – 3 62
  4² ×  1
 

r
    4 
16. (a) a = 7.5 and b = 2.5

Sni
1
 1 2
 5 –
 
3  1– 2
 5 3
  a × a +2ab + b × b
 1 2
16   = 4 =2

gv.i
 5  3
 5 – 2
  a² + 2ab + b²   
 4
 (a +b)² =(7.5 + 2.5 )²
5 – 3  10 – 6  5  3 – 10 – 6  (10)²  100
 1 1
2 2 20. (a) –

ridna
    5 – 3

–4
8 3 12 – 140  8 – 60
2  5 – 6 17. (a) 
 

2 5 –2 6 125 
  2
2

eeYa
2 –
4
  5 – 6 125 3
4  3  3
5  
10  84
 
 
   
14. (c) 2 
 8    


1

1
2  3 2 – 3 3 – 1
geisnh 12 – 4  35 8– 4  15
 
 2 – 3  2  3  3 1
 5 
4
625
 
    2
2 16 –
 2 2  10  4  21
 2
  3  2– 3
 3 –1 3 – 1
Enak

  18. (c)  
   
2– 3 2 3


3 1

3 – 1 
  12 – 8 3  2  1 1
  5  24  –
12 – 2 35 8 – 2 15
R

 2
4  3  4

3 43–4 3

 3 –1
 

 36  24 – 24 – 16

2
  4 –3 3–1  5  24 10  2 21
aryn

 
6–4 1
Les B

 31– 2 3   
14   5  24 2 2
  
 2   7   5 –2 7 5

2 5 – 24
 14 +

2 2– 3
   1
5  24 5 – 24
wa. th

2 
2 2
 14 + 2 – 3 = 16 – 3

5 
  3 –2 5 3

2 

2 5– 24
2 25 – 24 2
 5 3  5  32
wwM

15. (b)    2 2
 
 5 – 3
 5  3  
2 5–2 6   7   3 2 7 3

5  3  2 15  2 2  1 1
   
 2 3  2 – 2 3  2   

 2 2
5  3 – 2 15

2
 7 – 5  5 – 3
8  2 15 2

8 – 2 15
 2
 3 – 2 2

 2  3 – 2
 7  3
4  15
 1 1 2
 – 
4 – 15  6 –2 7 – 5 5 – 3 7 3

Rakesh Yadav Readers Publication Pvt. Ltd. 72

For More Visit : www.LearnEngineering.in


For More Visit : www.LearnEngineering.in

Rationalizing in above equation.  (x + y) (x² –xy + y²)


25. (b) 3 0.0 04096
 x³ + y³
1 7 5 1
 × –  0.16 (16³ = 4096) 3 3
7 – 5 7 5 5 – 3 
3 3.5  3 2.5
 0.4  0.4 3.5 + 2.5 = 6
5 3 2 7 – 3 
× – +  0.4
5 3 7 3 7 – 3 –3 –3
26. (a)
2.3  2.3  2.3 – 1
30. (d)
3  2 2 +
3 – 2 2

7 5

 5  3 –  7 – 3 2.3  2.3  2.3  1  1 
3
 1 
3
2 2 2 a = 2.3     
   
b=1 3  2 2 3 – 2 2
7 5 – 5 – 3 – 7 3

ir
 a³ – b³ 3
2  

v.iSn

a ²  ab  b ²  1 3 – 2 2
 0     +
 3 – 2 2
21. (c)  a –b  a ²  ab  b ²   3  2 2 
3 
0.5², 0.49, 0.008, 0.23  a ²  ab  b ²  3

dnag
     
 2.3 –1 = 1.3  1 3 – 2 2
0.25 0.7 0.2 0.23 27. (b)   
 3 – 2 2
(2.89)0.5 2–(0.5)2 3 3 0.008  3  2 2 
least 3 3

eYari
1
2.89 2 2– 0.25 3 – 2 2  3  2 2 
22. (a)    
  9–8  +  9–8 
Descending order: 2.89    
0.2 3 3
12 12 12 12
256 125 64 9 1.7
1.75 
3 – 2 2 + 3  2 2
3
4 4
5

0.07 0.13
2 6
3
Assending order :
snhe 1.732
a=3– 2 2

b=3+ 2 2
243
 243   0.2 1.7 1.732 1.75
kgei
23. (a) 0.25 0.075 0.2 a³ + b3  (a + b)(a² + b² – ab)
7  49  343 3 0.008 2.890.5 3 2– (0.5)²
 (3 – 2 2 + 3 + 2 2 )(17 + 17 – 1)
0.130.07
243 28. (c) x= 2  2  .....
ERna

2  (6)(33)
 0.25 2.075 30.2
7 7 7  198
x² = 2 + 2  2  ......
50.20 1 5 3 3 2 2
3 3 3 x² = 2 + x 31. (a) – +
 0.250.1500.6  1 =7 x² –x – 2 = 0 3 2 5 2 5 3
aBryn

7 7 x² – 2x + x – 2 = 0

24. (b) – 3 38 74 3


x(x–2) +1 (x – 2) = 0
(x + 1) (x – 2) = 0 
5

 3 – 2 –
x=2 3 2 3– 2
Shortcut Method
 5 – 2
3 3
+ 2 2  5 – 3
Les

 – 3  3  8 4  3  22 3 When the question is in t his for m   ×


 5 2 5 – 2

 5 3 5 – 3
wa. th

i.e x  x  x ........
2
 – 3 38 2  3 Then factor the x n1 > n2 
15 – 10

3 3  5 – 2 
3–2 5–2
n1 n2
– 3  38 2 3   mi.diff.
2 2  5 – 3
wwM

 +
5–3
So n1 is answer 2 22
 – 3  3  16  8 3 = 15 – 10 –  15 – 6 + 10 – 6

2 2 × 1  15 – 10 – 15  6  10 – 6
 – 3   3  4 ²  24 3
  0

2
29. (c) 3 3.5  3 2.5 32. (c)
0.96³ – 0.1³
0.96²  0.096  0.1²
 – 3 4  3 3 3.52 – 3 8.75  3 2.52  a = 0.96
 b = 0.1
 – 34 3
x = 3 3.5
a³ – b³
 4 =2 y = 3 2.5

a ²  ab  b ²

Rakesh Yadav Readers Publication Pvt. Ltd. 73

For More Visit : www.LearnEngineering.in


For More Visit : www.LearnEngineering.in

1
a – ba ²  b ²  ab   7 – 6 
2 3 5 – 2 3 – 5
 6 7
a ²  ab  b ² 2 6

 a–b 1
2 3 1
 = 8 – 7
 0.96 – 0.1 7 8  
2 6 2
= 0.86
1 –2
33. (c) 4 7 –2

8 – 9
 9– 8 38. (a)   
–2 
82 7 Now put values

2 1
 4– 3 5 – 4 6– 5 7 – 1
  –4
2 –2

r
2 –2
 7  1  2. 7.1 6 8 – 7  9– 8 –2  

Sni

2  9– 3  3– 3
 (–2)4 =16

gv.i
 7  12
2
 
2 36. (d)
1
×
100  99
39. (d)  19  17    19  17 × 
 1  100 – 99 100  99
    7  1 

ridna
 2 
 2 
100  99  19  17  19  17

2
  100  99
1 19  17 19  17 19  17
2 7 5
34. (d) + – Similarly

eeYa
12 – 5 12 – 7 2
7 5
1
Similarly  13  11  13  11
2 7 – 5   99  98
99 – 98 2
 × +
7  5 7 – 5 5  3 
5 3
1
geisnh
  98  97 ....and Largest + (Because, Same Numerator is
7

 12  5 – 98 – 97
dividided by Smallest denominator)
soon
12 – 5
 12  5 Now : expression: 2
40. (a) x 
5 3 5 3  5  3
Enak
  
 5 12  7  5 3 5 3 2
   100  99 – 99 – 98  98  97
 12 – 7 12 

7  Similarly

.. + 2  1
R


2  7 – 5  +
7  12  5 –  100  1  10 + 1 = 11 y=
5 3

 5  3
2 7 5 3 2
aryn

 1 1   x+y
5 37. (c)   
 12  7 
Les B

 
 2 3– 5 2– 3 – 5 
5  3  2 15  5  3  2 15
 
 5  2
 

1
×
 2  3   5  16
8
 7 – 5  12  5 – 12 – 7  2 3 –  5 
 2  3   5  
2
wa. th

 0 41. (d) 0.75 = a, 0.25 = b


2 3 5 2 3 5  a×a–2×a×b+b×b
1
35. (c)    a² – 2ab + b²  (a–b)²
3 4 232 6 – 5 2 6
 (0.75 – 0.25)²
wwM

Similarly
1 4– 3  (0.50)² = 0.2500

4 3

4– 3 1

 2– 3    5 42. (b) 3+
1
+
1
+
1

4 – 3

 2– 3 –  5   2– 3  5 3 3 3 3–3

 1 1 1
1  3+ + –
2– 3 5 3 3 3 3– 3
 4 – 3 
–2 6 1 3 – 3 – 3 – 3
1  3 +
 
Now put the value in question 3  9–3 
Similarly  = 5 – 4 
4 5

1 
 2  3  5 –  2 – 3  5  3
1

3
 3
1

1
=3
3 3 3

5 6
 6– 5 2 6 2 6 3

Rakesh Yadav Readers Publication Pvt. Ltd. 74

For More Visit : www.LearnEngineering.in


For More Visit : www.LearnEngineering.in

 1 3 6 5 1 5 –1
  9 46. (d) 49. (b) a+b= 
 4 4 2 2  5 3 – 2 12 – 32  50 5 –1 5 1
2 2  
8    
43. (d)   2 2   2 2
2 3 6
   
  
5 3 – 22 3 – 4 2  5 2 
 5  1   5 – 1
 5 – 1 5  1
  1/2 3 6
  29     2 
 5 3 – 4 3 –4 25 2

  2 4 212  
8     
2
 5  1
 =
2 5 1
  =3
  1   5 –1 4
 
 
2   3 6 3  3 2 

ir
4    
 3 2 3 2 5 1 5 –1

v.iSn
a.b = × =1
5 –1 5 1
  1/2   3 = 1.732
put value in expression
  9/2 3/2  
  2 .2   2
8    47. (b)
12  12  12  ....... a  b – ab

dnag
   1   a ²  ab  b ²
  2  
   = 2
 2  a ² – ab  b ² a  b – 3ab

 
4 × 3
2
3 –1 9 –1 4
1/2 

eYari
 = = =
8  212/2 2


  
  48. (a) a=
3
50. (c)
3 –3 9–3 3
2
  61   2 3 1 
8  2 2  3  –  


 

 
 

 
[8  8] = 0 Ans.
a+1=

32
snhe
2
+1



 5 3

2
×
6– 3

5 – 3
6


5 

3
×
 5 3 5 – 3 6– 3
kgei
2
3 2 2 6 2 3
44. (c)   6 3 1 6– 5
6 3 3 1 62 2 + ×
42 3  3 1  6 3 6 5 6– 5
ERna

 
 3 2 ( 6  3 )  2 6 3  1 4 4
   
 
 6  3

6  3
 
  3 1

3  1


2  5 – 3  – 3  6  3 + 6 – 5
2 5–3 6–3 6–5
 2 3 6  2 a+1=  3  1
aBryn

   4
 5 – 3 – 6 – 3 6– 5
 62 6

 2

 –2 3
2
2 
3 2  6  3 

6  3  1 
  a 1 =
 3  1 51. (c)  2  7 – 2 10

 3  2  4
Les

 
2 2
wa. th

2 3  6 2  1 a =
3 1  2 5 
 2 –2 5 2
 2
2 2
Similarly,
 12  6  18  6  18  2 3
 2
 5 – 2
wwM

3 –1
 12  2 3 1–a =  2 5 – 2  5
2
put values : 52. (a) x = 1 2  3
 2 32 3 0
45.(a) 3 1 3 –1 1
+  x
39 4 16 6 80 2 2 x –1
3
   
1
91/3 31/2 161/4 801/6 3 1 3 –1  1 2  3 
     1 2  3 –1
2
94/12 36/12 163/12 80 2/12
    2 3 3 – 2
12 94 12 27² 12 16³ 12 80² =  1 2  3 
2 3–2
Square of 81 is largest . So Ans 3
9  3  1 2 3

Rakesh Yadav Readers Publication Pvt. Ltd. 75

For More Visit : www.LearnEngineering.in


For More Visit : www.LearnEngineering.in

53. (d) mn = 121 = 112


–2–1  3 3 625
 m = 11  2 
 1 
n=2  – 
    3× 3 5  5  5  5  3×5 3 5

 (m – 1)n+1
 2  
 
 15 3 5
2+1
 (11 –1)  10³  1000
 1 22  1 4 1
Now put the value in question

1  – 
   – 
   16  43 5 – 16 3 5 + 15 3 5 – 3 3 5
54. (a)
 2  2
3– 8 19 3 5 – 19 3 5
57. (b) 260  (25)12  (32)12 
 348  (34)12  (81)12 (Greatest)  0
1 3 8 3 8
 ×  4 36 3 12
 (4 )  (64) 12
3– 8 3 8 9–8 64. (c) 40  9 81
 524  (52)12  (25)12

r
 3 8  40  9  9

Sni
58. (b) 3 3 3 .............
Similarly,
Shortcut method  40  9  49 7

gv.i
1
 When the question in from
 = 8 7
8 – 7
 n n n ................. 65. (b)
x – 24 75  50 = 1
1 75 – 50

ridna
 
 So n is answer
7 6
7 – 6 3 75 – 50
1 59. (d) 0.09 3 0.064 0.5 3/5
 x – 24 =
75  50

eeYa
 = 6 5
6– 5     2
0.3 0.4 0.5 0.6
=
75 – 50 


1
= 52 60. (b) 0.16 0.16 0.16
2
0.04
 x – 24  75 – 50
5 –2
Put value in question 
geisnh   75  50 – 2 75 50
0.16 0.40 0.0256 0.04  x – 24 = 25
3  8 –  8  7   7  6 61. (d) 2 8 4 13 5 16 1041
125 – 2  5 3  5 2
–  6  5    5  2
     x – 24 =
Enak

81/2 131/4 161/5 411/10 25


810/20 135/20 164/20 412/20
 3 8 – 8 – 7  7  6 – 6 – 5  5 2 125 – 50 6
     x – 24 =
25
R

3+2=5
20 10 20 5 20 4 20 2
55. (d) 8 13 16 41
   
x – 24 =

25 5 – 2 6 
aryn

2 1 1 
+ + +2 – 2 2 20 5 20 5 20 4 20 2 25
62 7 6 8 – 7 64 13 16 41
Les B

28  x –2 6 = 5–2 6
Greatest =
2 6–2 1 7 – 6  x=5
  +  62. (a) 2x = 3y = 6–z = k
62 6–2 7 6 7 – 6
 2 = k1/x; 3 = k1/y = 6 = k –1/z
4
 2×3=6 66. (c) 20 + 12 + 3 729 – – 81
5 – 3
wa. th

1 8 7 1/x 1/y –1/z


+  +2– 2 2 k ×k =k
8– 7 8 7 1 1 1 
  – 4 5  3
  – 9
x y z  2 5 +2 3+ 9 –  
5 – 3 5  3

2  6 –2 + 7 – 6 + 8 7 
wwM

6–4 7–6 8–7 1 1 1


  4
+2– 2 2
 x y z
=0


 5  3  – 9
 2 5 + 2 3+9–  2 
63. (b) 23 40  2  3 2  2  2  5  
 6 –2 7 – 6 8  7 2–2 2
 2 × 2 35
 6 –2 7 – 6 2 2  7 2– 2 2
 2 5+2 3+ 9 – 2 5 – 2 3– 9  0
 43 5
 2 7 67. (a) a 2 + b 3
 43 320
56. (a) = 98 + 108 – 48 – 72
–1 4 × 34445
 2  –2  
 1  
–    4 ×4 3 5
 772 + 33322
 2   

  
 16 3 5 – 22223 – 33222

Rakesh Yadav Readers Publication Pvt. Ltd. 76

For More Visit : www.LearnEngineering.in


For More Visit : www.LearnEngineering.in

 2n–1 × 5 = 320
 7 2 +6 3 –4 3 –6 2 B = 2 3  2 3 = 12
B – A = 12 – (–1) = 13 320
a 2b 3 = 1 22 3  2n–1 = = 64
71. (b) 2 250 5 50
 (2 )  (32) 50 5
a =1
b =2  3  (33)50  (27)50
150  (2)n–1 = (2)6

 5100  (52)50  (25)50 (Smallest) n=7


68. (a) 3 a = 3 26 + 3 7 + 3 63
 4200  (44) 50  (256)50 78. (a) 5 5 × 5³ ÷ 5–3/2 = 5a+2
Take round figure
 51 × 51/2 × 5³ ÷ 5–3/2 = 5a + 2
 3 a < 3 27 + 3 8 + 3 64 72. (c)
30  30  30 ............  51+1/2 + 3 – (–3/2) = 5a+2
 3 a <3 + 2 + 4  51+1/2+3+3/2 = 5a+2
6 × 5  56 = 5a+2
3 a <9

ir

 a+2=6

v.iSn
 a < 9³ 3 a=4
73. (a) x = 2 4 23 4 ........ 
 a < 729
–3 –3
Option A is answer  Squaring both sides 79. (a)
3  2 2 +
3 – 2 2
6 1 1  x² = 2 3 4 23 4 ......

dnag
69. (d) 2   
 1 
3
 
3
3 2 3 3 –2 Now cubing both sides 1
 
 
 
+ 

x6 = 8 ×4x 3  2 2 3 – 2 2
23 3 1 1 5
 x = 32
 2 + – 3 3

eYari
3 3 2 3 2– 3  x5 = 25
x=2

3 – 2 2 + 3  2 2
 
    
2– 3 – 2 3  74. (b) a = 55,
c = – 72
b = 17  2 × 27 + 6 × 3 × ( 2 2 )2
 2  2 3 +
 
  2 × 27 +18 × 8
2  32 – 3
 22 3 +

2 –


3 – 2 – 3


answer = 0
snhe
a + b + c = 55 + 17 – 72 = 0
 a³ + b³ + c³ – 3abc = 0
If (a + b + c) = 0
 54 + 144  198
80. (b) At the start t = 0°
L = 2 – 2°
4–3  2 – 1 = 1 cm
kgei
 
n /5 2n 1
243 3
 22 3 – 2 3  2 75. (b) n n –1
9 3 10  25  108  154  225
81. (d)
38
ERna

43 3 5n /5 2n 1
70. (c) =A+ B 3 3
74 3  2n n –1
3 3 10  25  108  169

 74 3 n 2n 1 2
aBryn

3 3
 2n n –1
2 3 3
10  25  121
 2² 
 3  22 3
3
n 2n 1
3
3n 1

2
 2n n –1  3n –1
2 3 3
Les


2  3  3(3n + 1) – (3n – 1)  33n+1 –3n +1 
10  36
wa. th

2
 3² = 9
 2  3
16 4

43 3
 3  110  12 12 – 25 – 3
76. (c) 
2

2
 2
 = A+ B
wwM

2 3   3  110  2 32 3 – 25 – 3 82. (d)

62 6 2 2 2

43 3
2 3

2– 3
2– 3
= A+ B
  3 1 25 32 3 –15 – 3 2 2 3

2  2 3

2 2

 4  3  1 3 – 15  35 – 3



4  3 32 – 3 = A+ B  2 2 
62

62

2
4–3 2  3 1 3 1 2 1
 4
  3 – 1  5 ² –
   3 

2
2
2
2
 8 – 4 3 + 6 3 – 9 = A+ B
 4 × 2 × 22  176
 2 3 – 1 = A+ B 77.(d) 2n– 1 + 2n + 1 = 320
n–1
 2 (1 + 2²) = 320 
 62  2

 62  2

2
A = –1 and B = 2 3 2  3 1 2  3 1 2 1
 2n–1(1 + 2²) = 320

Rakesh Yadav Readers Publication Pvt. Ltd. 77

For More Visit : www.LearnEngineering.in


For More Visit : www.LearnEngineering.in

2 Alternate 
   3 27
2  6 2 6 2   38 
   x =
   –  3 27  38  6 6 6  ...  
3 
  3 1
  3  1   
(squaring both side)

2 2 1  3  2 2 2 x² = 6 + 6  6  ....
  
   (5)  25 Ans.

2 1 2 1 3  2  x² = 6 + x
85. (a)
  6  6  .... = x 
2 18  2 3  6  2  18  2 3  6  2  1 1 1 1  
    
3 31  + x² – x – 6 = 0
 2 1 3  2 4  3 5  4
x² – 3x + 2x – 6 = 0

r
1 1 1 1 x (x – 3)+ 2(x – 3) = 0

2  2  1 6 5

7 6

8 7

9 8 (x + 2) (x – 3)

Sni
21 x  2, & x = 3
After Rationalizing
So, Answer is = 3

gv.i
   2 –1   3 –  
2  4 –  
3  5 – 4 +

2 2

  6 2 3   2 1
 2

= 89.(b)

3 2
   6 – 5 3 7 5 5 2 2

ridna
– +
5 2 2 7 7 5
  3 2
2
2 3
2
2  
2 1
 7 – 6   8 – 7   9 – 8
3 3 = 3 7 5 2 5 5
9 –1 = 3 – 1 = 2

eeYa
=   ×
5 2 5 2 7  2
 22 22 220 86. (a)
72  72  72  . . . .
2 2 2 2 2
6  7  8  9  10 7 2 2 2 7 – 5
83. (a) 9 × 8 + ×
7 2 7 5 7 5
74 3  42 3
geisnh
1 1
2 2 2 2 2 87. (c) +
6  7  8  9  10 1 2 3 1– 2 3 3 7  5 – 2 – 5 5  7 – 2
 2 2 = 2 2 2 2
(2  3 )  ( 3  1) 1 1  5 –  2  7 –  2
Enak

 +
2 2 2 2 2 1 3 2 1 3 – 2
6  7  8  9  10

2 3  3 1 1 3 – 2 1 3  2 2 2  7 – 5
R

2 2 + 2 2
 62  72  82  92  102 

1 3 – 2     7 –  5
 36+49+64+81+100  330 Ans.
aryn

22 3 2+2 3 = 35 – 14 – 35  10  14 – 10 = 0
3x  2y 5  
Les B

84. (d)  42 3 –2 22 3


2x  3y 6 90. (d) 11 n = 112 + 343
18 x  12y  10 x  15y
 1 11 n = 22227 + 777
8 x  27y
88. (c) 11 n = 4 77 7
wa. th

6 6 6  ........  
x 27
 (2, 3) are the factor of 6.
y 8 11 n = 11 7
If there is ‘+’ in ‘ ’ Answer is Highest
3 x  3y 2 value. n = 7
wwM

 
 3 3y  If there is ‘–’ in ‘ ’, Answer is lowest value. n =7
 x  

Rakesh Yadav Readers Publication Pvt. Ltd. 78

For More Visit : www.LearnEngineering.in


For More Visit : www.LearnEngineering.in

CHAPTER

08
SIMPLIFICATION
• Simplification: 2. Id entity element of
 7

ir
In sim plif ication an Multiplication: ‘1’ is called Ex.2. 99 
The value of  9 is:
expression, we must remove ide ntity element of  9

v.iSn
the brackets strictly in the multiplication as multiplication
 7
order ( ), { }, [] and then we of ‘1’ in any number does not 99 
Sol.  9
must apply the operations:- affect that number.  9
Of, division, Multiplication, e.g. x  1 = x

dnag
Addition and Subtraction.  7
3 Inverse Element of Addition/ 99  
  ×9
• Remember:- ‘BODMAS’ where Negative element of Addition/  9
B stands for bracket, O for of ; Additive Inverse:
D f or d ivis ion; M f or 7

eYrai
The number is called “Additive 99 × 9 + ×9
multiplication, A for addition 9
inverse” of a certain number,
and S for Substraction strictly (100 – 1) × 9 + 7
when it is added to the certain
in this order. 900 – 9 + 7
number and result becomes ‘0’
Note: ‘Of’ means multiplication. = 900 – 2 = 898
• Division Algorithm:-
Dividend = (Divisor  Quotient)
+ Remainder
(zero).
snhe
E.x. (i) x + (-x) = 0
Here (-x) is Additive inverse of x.
(ii) (9) + (-9) is Additive inverse
Ex.3. The value of
1
5
+ 99
44
45
× 9
is:
kgei
2. Modulus or Absolute value :
The absolute value of a real of ‘9’
4 Inverse Element of 1 44
number X is denoted by the Sol. + 99 ×9
symbol |x| and is defined as - Multiplication/Reciprocal 5 45
ERna

E l e m e n t / Mu l t i p l i c a t i v e
x , if x  0 1  44 
 Inverse: 99  45 
+   ×9
|x|  x , if x  0 5  
0, if x  0 The number is called
“Multiplicative inverse” of a
aBryn

1 99  45  44 
E.x : |5| = 5, |-5| = -(-5) = 5 ce rtain number , when the +   ×9
Note: In multiplication and division, product of numb er and 5 
 45 
when both the numbers carry multiplicative inverse is 1.
similar sign, we get positive 1 (100 –1)45  44 
1 +   ×9
sign in the result, otherwise 5 
 45 
E.x. x  1
Les

we get negative sign in the x


result i.e. 1 4500 – 45  44
wa.th

+
(+)  (+) = + 1 5 5
Here , is multiplicative
(+)  (–) = – x
1 4500 – 1
(–)  (+) = – inverse of ‘x’ +
5 5
(–)  (–) = +
wwM

TYPE – I
(+)  (+) = + 1 4500 1
(+)  (–) = – + – = 900
8 5 5 5
(–)  (+) = – Ex.1. The value of 9 × 9 is:
9
(–)  (–) = + 1 791
8 Ex.4. The value of + 999
Important terms 8 792
Sol. 9 × 9
9 × 99
1. Identity element of Addition:  8 1 791
‘0’ (zero) is called identity 9  9 
  ×9 Sol. + 999 × 99
ele ment of addition as   8 792
Addition of ‘O’ in any number 8
9×9+ ×9 1  791 
does not affect that number. 9 = 8  999  792  99
e.g. x + 0 = x ( x  Q)  
81 + 8 = 89

Rakesh Yadav Readers Publication Pvt. Ltd. 79

For More Visit : www.LearnEngineering.in


For More Visit : www.LearnEngineering.in

Ex.7. The value of 9 + 99 + 999 + 92


1 791
=  999  99   99 ...... + 20th term is: Ex.10 The value of 999 × 99 is:
8 792 99
Sol. 9 + 99 + 999 + ...... + 20th term
1 791 = (101–1) + (102 – 1) + (103 – 1) 92
=  1000 – 1 99  Sol. 999 × 99
8 8 ...... (1020 – 1) 99
= (10 1 + 10 2 + 10 3 ....10 20) –
792 (1 + 1 + ...... 20 times)
 92 
=  99000 – 99 999  99 
  99
8 we use G.P formula:  
= 9 9 +9 9 0 0 0 –9 9 = 9 9 0 0 0 a (r n – 1) 92
Sn = 999 × 99 + × 99
TYPE – II r –1 99
(1000 – 1) × 99 + 92

r
Series base question:-
Ex.5. The value of 10 1020 – 1
  99000 – 99 + 92

i
= – 20

.iSn
10 – 1 99000 – 7 = 98993
1 2 3 Ex.11 Find the value of
999 + 999 + 999 +
7 7 7 10 1020 – 1
  3 5 7 19
= – 20    .......... 

agv
4 6 9 4 36 144 8100
999 ......... 999
7 7  1  3 5 7 19
Sol.

ridn
Ex.8. The value of 1 –  +   ..... 
1 2 3  n  1 
 4 36 144 8100
Sol. 999 + 999 + 999 +
7 7 7
1×9 4×9 9×16

eeYa
 2   n  81×100
1 –  + ...... + 1 – 
4 6  n  1
   n  1 
 1 1 1 1 1 1 1 1
999 ......... 999 = - + - + – ..... -
7 7 is: 1 4 4 9 9 16 81 100
1 2  1   2  1 100  1 99
= 99 9+ + 999 + +
geisnh 1 –  + 1 –  + ...... + = 1- = =
7 7 Sol.

 n  1 
 
 n  1  100 100 100
3 6 Ex12. Find the value of
999+ ...... 999 +  n 
7 7 1 – 
 n  1 
  1  1  1  1
Enak

1 2 3 6 1  22  
 1  32  
1  42  
1  52 
= 999 × 6 + + + .... 1 2 n     
7 7 7 7 =1 – +1– ... 1 –
n 1 n 1 n 1  1
R

1 2  3  4  5  6 1 1  92 
.......× 
5994 +  
7 = (1 + 1 +...... n times) –
n 1
 1  1  1
aryn

21 (1 + 2 + 3 .... n) Sol. 1  22 
 1  32 
 1  42 

5994 + = 5994 + 3      
7 n (n  1)
Les B

 Sum of n natural no.=  1  1


= 5997 2
1  52  ..... × 
 1  92 
1 n (n  1)    
1 1
Ex.6. The value of 3 + 33 + ..... =n– ×
3 3 n 1 2 2 2
A - B = (A +B) (A - B)
wa.th

1 n n
=n– =  1  1  1  1
333333 is:
3 2 2 1  2 
 1  2 
 1  3 
 1  3 

       
1 1 1
 1
Ex.9. The value of 2  3 
 ×  1  1  1
wwM

Sol. 3 + 33 + ..... 333333 


3 3 3   1  4 
 1  4 
 . .... 1  

     9
1 1  3  5  999 
= 3 + + 33 + . ... .. 2  5 
 × 2  7 
  ... 2  1001   1
3 3       1  

 9
1  1  3  5
333333+ Sol. 2  3 
 2  5 
 ×  2  7 
 ×   1 3 2 4 3 5
3       = × × × × ×
2 2 3 3 4 4
= 3 + 33 + 333 + 3333 + 33333  999  8 10
1 2 
........  
 ....... 
+ 333333 + ×6  1001  9 9
3
5  9 1003 1003 1 10 5
= 370368 + 2 = × × ......... = × =
= 370370 3 5 7 1001 3 2 9 9

Rakesh Yadav Readers Publication Pvt. Ltd. 80

For More Visit : www.LearnEngineering.in


For More Visit : www.LearnEngineering.in

Ex13. Find the value of Ex.18. Find the value of


 45 
1 1 1 1 1 1 1 1 1 S45 = – 
 2 2  3  44  4
 1 1 1
        ....     ......... 
2 3 4 2 3 4 2 3 4 45 117 3965
34 terms 45
=– [6 + 176] Sol. 1 1 1 1
Sol. 2 + + ..... 
45 117 221 3965
1 1 1 1 1 1 1 1 1 45
        ....34terms =– × 182 = –4095
2 3 4 2 3 4 2 3 4 2 5 9 9 13 13 17 61 65
So, 1 1 1
The value of series = –4095 + + .....+
After 6 Terms series repeat 5 9 9 13 13 17

ir
so, then 1 1
Ex.16 Find the value of + + 1

v.iSn
1 1 1 1 1 1 12 20 61 65
S6  + -   +  = 0
2 3 4 2 3 4 1 1
In the series sum of 6 terms
is 0. so sum of 30 terms
30
+ ......
156 
1
4
( 15 –
1
9
+
1
9

1
13

dnag
(multiple of 6) will be 0 1 1 1 1 1 1 1 1
S30 = 0
Sol. 12
+
20
+
30
+ ..... +
156 + –
13 17
....
61

65
)
Now, solve the remaining 4
term 1 1 1 
3 4 4 5 5 6 12 12  – 

eYrai

4 5 65 

1 1 1 1 1 1 1 1 1 1 1
+ - - = – + – + –
2 3 4 2 12 3 4 4 5 5 6 1 12 3
= × =
4 65 65
 1 1 1

 1  2
1 – 4 
Ex.14. Find the value of 



 2  6
.........

1 1 10
snhe

12 13
TYPE – III
CONTINUED FRACTION
A continued fraction consists of
1  3 
  1  
  1 – 5 
  1  7 
  = – =
kgei
   3     3 13 39 the fractional denom inator s
Ex17. Find the value of
 12  1
1 – 13 
  1 1 1 1 Ex.19 The value of is:
ERna

    .......  1
10 40 88 598 2
1
 1  1  2  2 8
Sol. 5
Sol. 1 – 4 
 1  
   1  
  1 – 5 
  1 1 1 1
   3  3   + + .....+
10 40 88 598 1
aBryn

 6  12  =
1  7 
  1 – 13 
  1
    2
2 5 5 8 8 11 23 26 41/5
3 4 5 3 13 1 1 3 3 3 3
= × × × × × = 1 1 41
4 3 3 5 7 13 7 =
1 1 1 1 1 1 1 5 = 87 = 87
Les

Ex.15. Find the value of 12 – 22 + 32 = –   ... 2


3 2 5 5 8 8 11 41 41
– 42 + 52 – 62 + 72 – 82 ......... 902
wa.th

1 1 Note: These fractions are solved


sol. 12 – 22 + 32 – 42 + 52 – 62 + 72 – 82 . .......
23 26 starting f rom the bottom
towards upside.
.......892 + 902 1 1 1  Ex20. The value of
wwM

=  2 – 26 
 
3  
(1 – 2) (1 + 2) + (3 – 4) (3 + 4) + 1
1 is:
(5 – 6) (5 + 6) + (7 – 8) (7 + 8) 1 12  2 1
= 
 26 
 = 2
.......... (89 –90) (89 + 90) 3   13 1
3
= (– 1) ×( 3) + (–1) × (7) + (–1) 1
Alternate:- 4
×(11) + (–1) × (15) ...... (–1) × (179) 5
 
= –(3 + 7 + 11 + 15 ....... 179)  1 – 1 
1 First Last 
a = 3, d = 4  1
diff. b/w two no. term term  Sol. 1
179 – 3 
  1
2
No. of term = +1 1
4 1 1  3
1 2 1
= 44 + 1 = 45  –  = 4
3 
2 26 
 13 5

Rakesh Yadav Readers Publication Pvt. Ltd. 81

For More Visit : www.LearnEngineering.in


For More Visit : www.LearnEngineering.in

1 Sol. 2 1
 1+ 1 2 Sol. 1+
2 2  0.39 1
5 2 1
3 3 1
2 1
21 1
3 1
3 1
1 1
 1+ 1
1 2
2
68 /5 = 2
2  0.39 1
2 =1+ 1
1 3 1
 1+ 11/ 3 1
5 1

r
2 1
68 1

i
2

.iSn
= 2
1 68 2
 1+  1+ 2  0.39
141 141 6 1
3 1
68 11 =1+ 1 =1+
1

agv
1 1
209 1 2
x  2 1 1
141 = 3 3

ridn
2
Ex.21. The value of 2 2  0.39 2
39 /11
3

eeYa
+ is: 1 1 1
2 2 = =1+ =1+
5 = 1 3 8
6 22 1 1
7 2  0.39 5 5 5
2 39
5 3
3
geisnh 2 2 5 13 5
3 = = 22 =1+ = =1
22 39 2 8 8 8
Sol. 2+ 2 
2 39 100 100
5 9
6 1
7 2 200 100 (b) If =
Enak
2 1 26
5 = = = a
3 222 222 111 1
100 b
c
3
R

=2+ Find the value of a + b + c


2 1
5 Ex23. The value of 1 + is: Sol. Convert the fraction same
6 1
7 1 form of series
aryn

17 /3 1
1
6 1 1
Les B

3 = 26 = 8
=2 + 2
2 1 1 9 9
5 Sol. 1+ =1+
18 1 1
7 1 1
17 1 7/6 1
1 =
wa.th

6 1
3 2
=2+ 1
2 1 1 1
5 =1+ 8
137 /17 6 = 1 + 13 /7
1 Now, comparison both series
7
wwM

3 3 1
=2+ =2+ 1
34 719 7 20 7 1 =
5 =1+ = =1 1
137 137 13 13 13 a 2
1 1
b 1
411 411 Ex24. c 8
=2+ =2 (a) The vlaue of
719 719 a=2
Ex 22. The value of 1 b=1
1+ is:
2 1 c=8
1
2 is : 1 So, a + b + c = 2 + 1 + 8 = 11
2  0.39 1
1 Alternate:-
2 1
3 1
2 1
3 1 26 8
3 =2+
9 9

Rakesh Yadav Readers Publication Pvt. Ltd. 82

For More Visit : www.LearnEngineering.in


For More Visit : www.LearnEngineering.in

a=2 use c & d


9 1
9 1 =1+ ab 53
8 8 =
=1+
8 8 c=1 a –b 5–3
b=1 ab 8 4
1 1
and = = =
1 1 d 8 a–b 2 1
and =
c 8 Then d = 8 a  b 25
c=8 Now a × c + b + d Ex.28. If =
a – b 12
 a + b + c = 2 + 1 + 8 = 11 = 3 × 1 + 1 + 8 = 12
TYPE – IV a
1 17 Find the value of
b

ir
Ex25. If =
1 60 Componendo and dividendo (C. &
a

v.iSn
1 D) It is a theorem on proportions a  b 25
b Sol. =
1 that allows for a quic k way to a – b 12
c
d perform calculations and Reduce use c & d
the amount of expansions needed
Find the value of a × c + b + d
a  b  a – b 25  12

dnag
It is partic ular ly use ful when
Sol. Convert the fraction same
form of series.
dealing with equations involving a  b  a – b = 25 –12
fractions or rational functions.
1 2a 37
a a  b a  kb =

eYrai
60 = 1 1 Ex. , , 2b 13
= b a – b a – kb
17 9 1
3 3 If a, b, c and d are numbers a 37
17 17 =
such that b and d are non- b 13
9

=
3
1
1
8
zero and
Some Points
snhe
a c
= , then
b d Ex.29.
ab 4
= , Find the vlaue of
a–b 1
a
b
1 ab 4
kgei
9 ab cd Sol. =
1. Componendo = a–b 1
b d
use C & D
1 1 a b cd
ERna

= 1 = 1 2. Dividendo = a  b  a – b 4 1
3 3 b d =
1 1 a  b – a  b 4 –1
1 1 a a  kb c  kd
9 /8 1 3. for k  , =
1 b a – kb c – kd 2a 5
8 = ,
aBryn

–b a a  kc
2b 3
Comparision both series 4. for k  , =
a b b  kd a 5
=
1 1 a 16 b 3
1 = 1 Ex.26. If =
a 3 b 3
1 1 x 5
Les

b 1 Ex.30. If =2
1 1 ab x– 5
c 1 Find the value
wa.th

d 8 a–b Find the value of x


a = 3, b = 1, c = 1 and d = 8 a c
Sol. If = x 5 2
 a×c+b+d b d Sol. =
x– 5 1
= 3 × 1 + 1 + 8 = 12
wwM

a  b cd use C & D.


Alternate:- Then =
a – b c –d
17
The value.
a 16
=
x  5  x – 5 = 2  1
60 b 3 x  5 – x – 5 2 – 1
60 9 a  b 16  3 19
=3+ = = 2x 3
17 17 a – b 16 – 3 13 =
2 5 1
a=3 a 5 ab
Ex.27. If = Find x
17 8 b 3 a–b
=1+ =3
9 9 5
a 5
Sol. = ,
b=1 b 3 x=3 5

Rakesh Yadav Readers Publication Pvt. Ltd. 83

For More Visit : www.LearnEngineering.in


For More Visit : www.LearnEngineering.in

x3 1 x 3 –1 x 3  3x 189 x m  3n  m – 3n
Ex31. If = , (x  1, –1) Ex.34. If = Sol. =
x 1 x–1 3x 2  1 61 1 m  3n – m  3n
Find the value of x Find the value of x Use C & D
x3 1 x 3 –1 x 3  3x 189
Sol. = Sol. = x 1 m  3n
x 1 x –1 3x 2  1 61 =
x 1 m – 3n
Use C & D
x 3  1 x 1 square both sides
= x 3  3x  3x 2  1 189  61
x3 –1 x –1  = x 12 m  3n
We use C & d x 3  3x – 3x 2 – 1 189 – 61 
  =
x 1 m – 3n
3 3 250 125 Again use c & d
x  1  x –1  x  1  x – 1 = =

r
128 64
3
x  1 – x 3
– 1 = x  1 – x – 1 2 2
x  1  x –1 = m3n m–3n

i
.iSn
3
 x  1  125 2 2
2x 3 2x  

 x – 1

 =
64
x –1 – x –1 m 3m –m–3n
=
2 2
2 x2 1
  = 2m
 x  1 3  5 3

agv
3
x x 
 
 =   2  2x 6n
=  x – 1  4 
1 1 2
x 1 m

ridn
x3 = x x 1 5 =
x3–x = 0 = 2x 3n
x –1 4
x (x2–1) = 0 3x2n+3n = 2mx

eeYa
4x+4 = 5x–5
However, since x  1, –1, x=9 Ex.37. If a  b : ab = 4:1 a>b
So x = 0 Ex.35. If a+b = 1 Find a : b
2x  y 5 c+d = 1 a b 4
Ex.32. If 2x – y = , find the value Sol. =
4
geisnh d ab 1
a–b = ,
c multiply 2 both denominator
2x
of y find the value of c²–d² sides
Sol. a+b = 1 .......(i) ab 4
=
Enak

2x  y 5 2 ab 1 2
Sol. = d
2x – y 4 a–b = ......(ii)
c ab 2
Use C & D =
(i) / (ii) 2 ab 1
R

2x  y   2x – y   5  4 ab 1


Use C & D
2x  y – 2x – y  5 – 4 =
a – b d/c a  b  2 ab 2 1
aryn

=
4x 9 a  b – ab 2 –1
 2y = ab c
Les B

1 = 2
a–b d a b
2x Use. C & D
  3
2 =
So,  y = 9  a– b 1
cd a
=
wa.th

2x  y 1 c–d b a b 3
Ex.33. If x  2y = , =
2  c+d = 1 (Given) a– b 1
3x+y 1 a Again use C & D
Find the value of 3x – y =
c –d b a 3 1
wwM

=
2x  y 1 b b 3 –1
Sol. = then, (c – d) = Square both sides
x  2y 2 a
2
4x–2y = x+2y Now, c²–d² = (c+d) (c–d) 3 1
3x = 4y = 1×(c–d) = c – d a  
= 2
b  3 – 1
3x 4 b
c²–d² =
y =1 a a 4 2 3
Use C & D =
m  3n  m – 3n b 4–2 3
3x  y 4  1 5 Ex.36. If x =
= = m  3n – m  3n a 2 3
3x – y 4 – 1 3 =
Find 3nx2 + 3n b 2– 3

Rakesh Yadav Readers Publication Pvt. Ltd. 84

For More Visit : www.LearnEngineering.in


For More Visit : www.LearnEngineering.in

Ex.38. If a+b : ab = 4:1 3 1 z c


x  3x  3x 2  1
   = ......(III)
find the value a 3 2 1– z a
Sol. a +b = 4, x  3x  – 3x  1
(I)×(II)×(III)
ab = 1 m 1 a b c
then, square both sides = × × =1
m –1 b c a
ab = 1
Again use C & D 1 x 1 y 1 z
1 So, × × =1
b= 1– x 1– y 1– z
a x 3  3x m
= TYPE – V
Now, a +b = 4 3x 2  1 1

ir
Put the value b 3mx2+m = x2+3x Recurring number

v.iSn
1 x2+3x–3mx2–m = 0
a+ = 4
a 2ab
Ex.40. If x = and b > 1 Impure
a2–4a+1 = 0 Pure
b2  1 Recurring
ax2+bx+c Recurring

dnag
decimal decimal
we use, ax  a –x Number
Find Number
ax – a–x 1. Pure recurring decimals :
–b  b2 – 4ac

x 2b These are recurring decimals

eYrai
2a
Sol. = where the recurrence starts
a b2  1
immediately after the decimal
– 4  –4 ² –411 Use C & D point.
21
a b2  1 Ex: 0. 4444 ......  0.4

=
4  16 – 4
2
x
=
2b snhe
a  x b2  1  2b
=
3.232323....  3.23
0.564564564  0.564
a – x b2  1 – 2b
kgei
4  12 2. Impure recurring decimals:
= 2
Unlike pure rec urring
2 a  x  b 1
=  
 decimals, in these decimals,
a – x  b – 1
ERna

42 3 the recurrence occurs after


= ax b 1 a certain number of digits in
2 =
b –1 the decimal.
a–x
Take positive
Again use C & D Ex: 0.43542542........  0.43542
aBryn

then,
 ax    a–x =b 0.546666.........  0.546
a = 2+ 3 Then, ax
 – a–x  1 recurring number
3 3
a–b b–c Pure recurring decimal
m 1  m 1
Ex39. If x = Find Ex.41. If x = ,y= , number convert into
Les

3
m 1 – m 1 3 ab bc fraction
wa.th

the value of x2+3x–3mx2–m ca


z= 3
ca 0.333.....  0.3 =
x 3
m 1  m 1 3 9
Sol. = 3 1 x 1 y 1 z
1 m 1 – 3 m 1 Find × × 4
1– x 1– y 1– z 0.444.....  0.4 
wwM

Use C & D 9
a–b
3 Sol. x= 45
x 1 m 1 ab 0.454545..... = 0.45 
= 3 99
x –1 m –1 1 ab
= 3.232323....  3.23
cube both sides x a–b
Use C & D 23 23
3
x  1 m1 1 x a =3+ = 3
= 99 99
3 = ......(I)
x – 1 m –1 1– x b
5.564564564  5.564
3 3 3 2 2 1 y b
(a+b) = a +b +3a b+3b a Sam e = .....(II) 564
(a–b)3 = a3 –b3 –3a2b+3b2a 1– y c  5
999

Rakesh Yadav Readers Publication Pvt. Ltd. 85

For More Visit : www.LearnEngineering.in


For More Visit : www.LearnEngineering.in

* the number of of 9's in this 573 – 57 Ex.46 Find the value of


group equals the number of (v) 0.57333...=0.57 3 =
900 0.37  8.56  1.23
digits in the recurring part of
the decimal. 516 43 Sol. 0.37  8.56  1.23
Impure recurring decimal number = =
900 75   
convert into fraction
726  72 37 56 23
456  4 (vi) 0.7266....=0.72 6 = +8 +1 
0.4565656......... = 0.456  900 99 99 99
990
expanding the meaning 654 109 37 56 23
= = =8+1+ + +
Step - 1 900 150 99 99 99

r
Subtract the non-recurring
86 37  56  23
initial part of the decimal (in this (vii) 9.868686...9+0. 86 = 9+ =9+

i
.iSn
case, it is 4) from the number formed 99 99
by writing down the starting digits of 783  78 116
the decimal value upto the digit where (viii)0.783333...0.78 3 = =9+
900 99
the recurring decimals are written for

agv
the first time; (456 - 4) 705 47 17
Step - 2 = = =9+1+
99

ridn
900 60
As many 9's as the number
of digits in the recurring part of the 67  6 61 = 10 + 0.17 = 10.17
(ix) 0.6777...=0.6 7 = =

eeYa
decimal. (in this case, since the 90 90 Ex.47 Find the value of
recurring part '56' has 2 digits, we
write down 2 9's.) These nines have 71 71 2.856  3.74  5.8756
(x) 4.7171...= 4. 71 =4+ =4
to be followed by as many zeroes as 99 99 Sol. 2.856  3.74  5.8756
the number of digits in the non
geisnh 73  00 73 I II III
recurring part of the decimal value.
(xi) 5.00 73 =5+ =5+
(In this case, the non recurring part 9900 9900 × × × × × × × ×
of the decimal value is '4'. Since, 4 2. 8 5 6 5 6 5 6 5
has 1 digit, attach one zeroes to the 73 3. 7 4 7 4 7 4 7 4
=5 5. 8 7 5 6 6 6 6 6
9900
Enak

two nines to get the number to


12.4 7 9 7 0 7 0 5
divide the result of the first step.)
873 – 8
(xii) 0.8737373....=0.8 73 = = 12.47970
456  4 990

R

990 Where
865 173 I.  maximum digits non recurring
42. Convert to fraction. = =
900 180
aryn

(i) 0.22222......... [where ( 5.8756 ) has 3 max. non


(ii) 0.5555 ........ Ex. 43 recurring digit (875)]
Les B

(iii) 0.444747........ 0.56777........ II.  LCM of recurring digits of


(iv) 0.456456456......... Sol 0.567
number (where 2,2 and 1
(v) 0.57333333.......... recurring digits)
(vi) 0.72666666.......... 567  56 511 III.  atleast 3 digits
= =
(vii) 9.868686 900 900
wa.th

Type – VI
(viii) 0.783333...... Ex.44 Number of Digits
(ix) 0.67777....... 0.43542542......
(x) 4.717171........ Ex.48 How many digits are required
Sol 0.43542 to write the counting number
wwM

(xi) 5.00 73 from 1 to 50 ?


(xii) 0.87373......... 43542  43 43499 Sol. No. × digit = total
 =
99900 99900 digit
2 From 1 to 9 =9 × 1 = 9
Sol. (i) 0.222....= 0 . 2 =
9 Ex.45 Find the value of 3
0.037 From 10 to 50 = 41 × 2 = 82
5 total digit = 9 + 82 = 91
(ii) 0.555...=0. 5 = 37 1
Sol. 0.037  = * There are 9 numbers from 1 to 9,
9 999 27 and each number has 1 digit hence,
47 the total number of digits are (1×9)
(iii) 0.474747...=0. 47 = 1 1 = 9. In the same way there are 41
99 3
0.037 = 3
=
27 3 number (50 – 10 + 1) from 10 upto
456 152 50, and the number of digits are (41
(iv) 0.456456....=0. 456 = = = 0.333.....  0.3 ×2) = 82.
999 333

Rakesh Yadav Readers Publication Pvt. Ltd. 86

For More Visit : www.LearnEngineering.in


For More Visit : www.LearnEngineering.in

Hence, From 10 to 99 = 90 × 2 = 180


The total number of digits From100 to 999 = 900 × 3 No. Digit
(9 + 82) = 91 = 2700 2 digit
Ex.49 How many digits are requried From1000 to 8756 = 7757 × 4 3 digit
to write the counting number = 31028 4 digit
from 1 to 672? total digit = 9 + 180 + 2700 +
Sol. No.× digit = total digit 31028 = 33917 Ex. 52 Calculate the number of
Ex.51Calculate the number of digits digits in the prod uct of
From 1 to 9 =9 × 1 = 9
in the product of 411111 × 522222 8232 × 25348
From 10 to 99 = 90 × 2 = 180
Sol. 4 = 22 Sol. 8232 × 25348
From 100 to 672 = 573 × 3 = 1719
= (22)11111 × (5)22222 (am × bm (23)232 × (52)348
total digits
s = 1908
(By equalising power)

ir
Ex.50How many digits are requried = (ab)m)
(By equalising power) (2)696 × (5)696

v.iSn
to write the counting number
= (2)22222 × (5)22222 = (10)22222 = (10)696
from 1 to 8756?
Sol. No. × digit = total digit So, number of digits
From 1 to 9 = 9 × 1 = 9 Number of digits = 22222 + 1 = 696 + 1 = 697
= 22223

dnag
SOLUTION

eYrai
5. On simplification of 12. When simplified, the expres-
1
1. Simplify : 1 sion
2 2 2
1
3 2.644 – 2.356 –1
1 1 1

11
2
1

5
4
5
6.
(a) 1 (b) 4 snhe
0.288
(c) 5 (d) 6
What is the square root of
1002 0.0013  0.00164  30  4
is equal to :
5 
kgei
(a) 1 (b) 1 0.09 ? (a) 1.6 (b) 0.8 (c) 1.0 (d) 0
17 7
(a) 0.3 (b) 0.03
6 21 (c) 0.003 (d) 3.0 1 1 1 1
(c) 1 (d) 1  –
13. When   – 
 is divided
ERna

17 17 7. Find the value of 2 4 5 6

2 3 2 5 3 7 
2. Simplify : 1  0.75  2   –
by   – 
 , the result is:
3  0.75  0.75
   1
1 1 – 0.75   5 9 5 18 
aBryn

4
1
5 (a) 4 (b) 1 (c) 2 (d) 0.25
1 1
8. Find the value of (a) 5 (b) 2
7 4 7 3 10 18
(a)
4
(b) (c) (d) x  441 = 0.02
7 5 7
1 3
(a) 1.64 (b) 2.64
Les

(c) 3 (d) 3
5 7 17 1 (c) 1.764 (d) 0.1764 6 10
wa.th

 of –
3. Simplify : 3 51 5 3 9. By which s mallest numb er 14. The square root of (2722–1282)
2 5 28 2 should 5808 be multiplied so is:
 of –
9 7 5 3 that it bec omes a p erfe ct (a) 256 (b) 200 (c) 240 (d) 144
square ?
wwM

1 1 15. One-third of the square root of


(a) (b) 4 (c) 2 (d) (a) 2 (b) 7 (c) 11 (d) 3
2 4
which number is 0.001?
38 100 (a) 0.0009 (b) 0.000001
4. Assume that 10.   3 125 is equal to :
16 49 (c) 0.00009
13 = 3.605 (approximately)
(d) None of the above
3 7 4
1 3 0 = 11.40 (approximately) (a) 7 (b) 1 (c) (d)
4 100 7
72.9
find the value of 1.3  1300 16. 3 is equal to :
11. By which smallest number 1323 0.4096
 0.013 : must be multiplied, so that it be-
comes a perfect cube ? (a) 0.5625 (b) 5.625
(a) 36.164 (b) 36.304
(c) 37.304 (d) 37.164 (a) 2 (b) 3 (c) 5 (d) 7 (c) 182 (d) 13.6

Rakesh Yadav Readers Publication Pvt. Ltd. 87

For More Visit : www.LearnEngineering.in


For More Visit : www.LearnEngineering.in

1 1 18 21 13 12
1 17 (a) (b) (c) (d)
17. The value of  24. If x = 1
then x  ? 17 17 17 17
1 22 2
3
1 2 33. Simplify:
2 –
7
2 5 3 1      
9 (a) (b) (c) (d)      
5 2 5 2 1 1 1 1 – 1– 1 1– 1 
 1  1  1  1 
12 22 5
1  
 10   10   
 10  10   
(a) (b) (c) (d) 1 9 
1 5 1 1 
 10  10  10  10
22 5 22 25. –   
 –   is equal to

20 5 
4 6 3 2

   

r
1    
18. If x  1 , the n the 9 9 1 1
1  1
    1 – 

i
(a) 0 (b) 1 (c) (d)

.iSn
1 20 10  1   1 
1 
 10      10   
1  10   10  
1 26. The value of 
1
2

agv
2 2 2
0.1  0.01  0.009
2 2 2 (a)
10 0
(b)
90

value of 2x +
7
is :
0.01  0.001  0.0009 10 1 101

ridn
4
(a) 102 (b) 10 20 10 1
(a) 3 (b) 4 (c) 5 (d) 6 (c) 0.1 (d) 0.01 (c) (d)
101 10 0

eeYa
27. The value of
19 1
19. Simplify :  : 1 1 1 1 1 1
43 1 2 2 2 34.      =?
2 0.03
  0.21
      0.065 30 42 56 72 90 110
1
3 2 2 2
1
geisnh0.003  0.021  0.0065 2 1
1
4 (a) 2 (b)
2 3
(a) 0.1 (b) 10 (c) 10 (d) 10 27 9
28. If (102)2 = 10404, then the value
19 43 38 5 6
(a) 1 (b) (c) (d) of (c) (d)
Enak
104.04  1.0404  0.010404
43 19 43 27 55
is equal to
20. Simplify: 35. The value of 1 ÷ [1 + 1 ÷ { 1 + 1
(a) 0.306 (b) 0.0306
÷( 1 +1 ÷ 2)}]
R

1 1   1 1 1 1 1 (c) 11.122 (d) 11.322


8 – 3  1 –  
1 – – 
2  5 1
4   4 2 2 3 6 29. If 4096  64 , then the value of (a) 1 (b) (c) 2 (b)
aryn

8 2
1 1 1 2 40.96  0.4096 + 0.004096 +
36. The simplification of
(a) 4 (b) 4 (c) 9 (d)
Les B

2 6 2 9 0.00004096 up to two place of


3.36 – 2.05  1.33 equals :
decimals is:
21. If 50  *
, then the value of * (a) 7.09 (b) 7.10 (a) 2.60 (b) 2 .6 1
* 1
12 (c) 7.11 (d) 7.12
wa.th

2 (c) 2.64 (d) 2.64


30. The least number that must be
is: subtracted from 63522 to make 37. (0.2 × 0.2 +0.01) (0.1 × 0.1 + 0.02 )–1
25 4 the result a perfect square is :
5 41 41 9
(a) (b) (c) 4 (d) 25 (a) 18 (b) 20 (c) 24 (d) 30 (a) (b) (c) (d)
2 25
wwM

3 12 4 5
22. Find the sum of the following : 31. By which s mallest numb er
should 20184 be multiplied so 38. The value of
1 1 1 1 1 1 1 1 that it bec omes a p erfe ct
      
9 6 12 20 30 42 56 72 square? 5  11  19  29  49
(a) 2 (b) 3 (c) 5 (d) 6
1 1 (a) 3 (b) 9 (c) 7 (d) 5
(a) (b) 0 (c) (d) 1
2 9 1
32. If 2=x + , the n the 7
23. The value of 25–5[2+3 (2–2 (5– 1 39. The value of 3 is equal to
1 875
3 )+5)–10 ]÷4 1
3
(a) 5 (b) 23.25 4 1 1 1 1
(c) 23.75 (d) 25 value of x is : (a) (b) (c) (d)
3 15 4 5

Rakesh Yadav Readers Publication Pvt. Ltd. 88

For More Visit : www.LearnEngineering.in


For More Visit : www.LearnEngineering.in

2  1 1 1 1 1 1 
40. 2
is simplified to 47. 
      
 1 2
3.5 5.7 7.9 9.11 11.13 13.15  2 –1
2  0.39 3 11
3
2
is equal to 54. The value of
2 1
3 3
1
3 2 2 7 2 3 
(a) (b) (c) (d) 1
45 25 45 15 3
1 3
(a) (b) 2 48. The number of digits in the
3
square root of 625686734489 is
(c) 6 (d) None of these 38 109 116
(a) 4 (b) 5 (c) 6 (d) 7 (a) (b) (c) 1 (d)
109 38 109
49. There are some boys and girls

ir
1 2 4 1 1 3
– –  –   in a room. The square of the
41. 2 3 5 3 5 4 is simplified to 55.

v.iSn
number of the girls is less than 8  57  38  108  169  ?
1 2 4 1 1 4
 –  – – the square of the number of
2 3 3 3 5 5 boys by 28. If there were two (a) 4 (b) 6 (c) 8 (d) 10
more girls, the number of boys 56. If the number p is 5 more than
10 3 would have been the same as q and the sum of the squares of p

dnag
(a) – (b) – that of the girls. The total num- and q is 55, then the product of p
3 10
ber of the boys and girls in the and q is
(c) 1 (d) – 2 room are (a) 10 (b) – 10 (c) 15 (d) – 15
42. The simplification of (a) 56 (b) 14 (c) 10 (d) 7

eYrai
2 1
4 – 1
0.63  0.37  0.80  y ield s the 50. If 2 0.014  0.14x =0.014× 0.14 2 y . 57. 7 2 ÷ is equal to
1 1 1
3 1 2
result is: 2 7 2
1
x 1
find the value of y ,
(a) 1.80

(c) 1.79
(b) 1.81

(d) 1.80 (a) 0.000196


(c) 0.0196
snhe (b) 0.00196
(d) 0.196
(a) 1 (b)
1
2
5–

(c)2
5

(d)
1
3
kgei
43. The square root of 0.4 is : 58. If part of a
 2 2 th
0.1 – 0.01   
(a) 0.8 (b) 0.6 
51.   + 1 is equal to  
0.0001   
 
ERna

 
(c) 0.7 (d) 0.9  
(a) 1010 (b) 110  5 
44. The square root of (c) 101 (d) 100 4 – 
 1 1 
 1 
100 – 1100 – 2100 – 3...100 – 200
aBryn

7  3 57 – 3 5 52.
100  99 98 ... 3 21


3 
1 
 2 
(a) 4 (b) (c) 3 5 (d) 2 is equal to  4 
5
journey takes ten minutes,
100 3
 11 15 2  11 15 2 (a) then to complete th
of that
4   4 –  99  98  97  ..  3  2  1 5
Les

45.   –  is equal
 15 71  15 71 journey. it will take
wa.th

to: 1 (a) 80 minutes (b) 50 minutes


(b) – (c) 48 minutes (d) 60 minutes
(a) 1 (b) 2 (c) 3 (d) 4 99  98  97  ...  3  2  1
(c) 0
13 59. is equal to
46. is equal to 1 1
wwM

48 2 4 –2
(d) – 7 4 1
99  98  97  ...  3  2  1
1 1 1 1
1 3 1 2
(a) (b) 1 1 2 7 1
1 53. The value of 1  2
3 2 1 1
1 1 1 5–
1 1 1 5
16 8 1
1 (a) 1 (b) 4 (c) 3 (d) 2
1
1 1 2
(c) (d) 1 1 1
1 1 3 60. a –b

b –a is equal to
3 3 1 2 1 2
1 1
1 1 21 17 34 8
1 1 (a) (b) (c) (d) (a) a – b (b) b – a
1 2
4 13 3 21 5 (c) 1 (d) 0
8

Rakesh Yadav Readers Publication Pvt. Ltd. 89

For More Visit : www.LearnEngineering.in


For More Visit : www.LearnEngineering.in

61. Find the sum of


1 (a) 0.9 (b) 0. 9
 1   2   3   n  (a) 2 (b)
1 –   1 –   1 –   .....  1 –  2
 n  1  n  1  n  1  n  1 (c) 0.0 9 (d) 0. 09
1 (c) 3 – 2 (d) 0
n
73. 0. 001 is equal to
(a) n (b)
2
4 – 0.04 1 1
67. The value of is close (a) (b)
1 4  0.4 1000 999
(c) (n+1) (d)
2
n  1 to
1 1
62. The square root of 33 – 4 3 5 (a) 0.4 (b) 0.8 (c) 1.0 (d) 1.4 (c) (d)
99 9
is : 68. If 0.05  0.5  a = 0.5 × 0.05 ×

r
p
a 74. 1. 27 in the form is equal to

i
.iSn
(a)  2 7  5
  (b)   72 5
b , then
b
is equal to q

(c)   7 – 2 5 (d)  2 7 – 5  (a) 0.0025 (b) 0.025 127 73


(a) (b)
(c) 0.25 (d) 0.00025 100 100

agv
63. What number must be added to
69. The value of (1001)3 is
the expression 16a2 –12a to 14 11
make it a perfect square ? (a) 1003003001 (c) (d)

ridn
11 14
(b) 100303001
9 3 13 (c) 100300301
(a) (b) (c) (d) 16 75. 8.3 1 +0. 6 +0.00 2 is equal to

eeYa
4 2 2 (d) 103003001
70. Find the Value of (a) 8. 912 (b) 8.9 12
64. If x [–2{–4(–a)}]+5 [–2{–2 (–a)}]
=4a, then x = ? (c) 8.97 9 (d) 8.9 79
5. 6 + 7. 3 +8. 7 +6. 1
(a) – 2 (b) – 3 (c) – 4 (d) – 5
76. The difference of 5. 76 & 2. 3
65. If a = 64 and b =289, then the
geisnh
(a) 27.8 (b) 27.7
1 (c) 27.08 (d) 27.07 (a) 2. 54 (b) 3. 73
value of 2 is
 a  b – b – a
 71. Find the value of 6. 74 +7. 32 (c) 3. 46 (d) 3. 43
Enak
1/2
(a) 2 (b) 2 (a) 14.06 (b) 14.07 77. The value of (0. 63 + 0. 37 )
(c) 4 (d) –2
66. The simplified value of (c) 13.06 (d) 13.07 100
(a) 1 (b)
R

72. Which of the following number 99


3 2 4 3 6
– + greatest of all ? 99 100
3 6 6 2 3 2 (c) (d)
aryn

0.9, 0. 9 , 0.0 9 , 0. 09 100 33

ANSWER KEY
Les B

1. (a) 9. (d) 17. (d) 25. (a) 33. (c) 41. (b) 49. (b) 57. (c) 65. (a) 73. (b)
2. (a) 10. (b) 18. (c) 26. (b) 34. (d) 42. (b) 50. (b) 58. (c) 66. (d) 74. (c)
3. (c) 11. (d) 19. (d) 27. (b) 35. (b) 43. (b) 51. (d) 59. (a) 67. (b) 75. (c)
wa.th

4. (c) 12. (a) 20. (a) 28. (d) 36. (d) 44. (d) 52. (c) 60. (c) 68. (b) 76. (d)
5. (c) 13. (a) 21. (d) 29. (c) 37. (a) 45. (d) 53. (c) 61. (b) 69. (a) 77. (b)
6. (a) 14. (c) 22. (a) 30. (a) 38. (a) 46. (d) 54. (a) 62. (d) 70. (a)
7. (a) 15. (d) 23. (c) 31. (d) 39. (d) 47. (d) 55. (a) 63. (a) 71. (b)
8. (d) 16. (b) 24. (b) 32. (b) 40. (d) 48. (c) 56. (c) 64. (b) 72. (b)
wwM

Rakesh Yadav Readers Publication Pvt. Ltd. 90

For More Visit : www.LearnEngineering.in


For More Visit : www.LearnEngineering.in

SOLUTION

1. (a) According to the question,


5 7 17 1 1
 × –   4
1 3 51 5 3 0.25
1  2 5 28 2
2  × – 8. (d) According to the question,
1
3 9 7 5 3
2
4 x  441 = 0.02
1 5 7 1 7 1
5  – –
3 15 3 9 3 x
 

ir
2 2 8 2  = 0.02
1 4– – 441
9 3

v.iSn
 1 9 3
2
1
3 7–3 x 2
2  =
54 4 21 100
9
  2
5 8–6

dnag
2
42
9 x =

1 100
 1 4. (c) According to the question,
2 Squaring both sides.
1

eYrai
35 1.3  1300  0.013
2  x = 0.1764
9
130 130
  1300  9. (d) According to the question,
1 100 10000
3 5808
 1
1
18  15
2

9

11.4
10
snhe
 36.05 

 1.14 + 36.05 + 0.114


11.4
100
2 1936
2 968
2 484
2 242
11 121
kgei
1 11 11
 1  37.304 1
18 5. (c) According to the question,
1 Factors are:
ERna

33
2 2
2.644 –  2.356  3, 2, 2, 2, 2, 11, 11
1 33
 1  1 0.288 Smallest number is = 3
33  18 51
10. (b) According to the question,
33 2.644  2.356  2.644 – 2.356
aBryn

 38
0.288 100
51  33 84 11   3 125
   1 a²–b² =  a+b  a–b 
51 51 17  16 49

2. (a) According to the question, 5  0.288 2 10


=5  5
Les

 
2 2 0.288 4 7
1  1
wa.th

3 35 6. (a) According to the question,


1 1 2 7
4 54   5
1 Square root of 0.09 = 0.09 = 0.3 4 10
5
7. (a) According to the question, 7 3
wwM

2 18  =1
 1  1 3 4 4
15 9  15  0.75  2
1   0.75   0.75   1 11. (d) According to the question,
9 1 – 0.75  

24  18 42 7 3 1323
3 3 3
   0.75  1 –  0.75  3 441
24 24 4   3 147
1 – 0.75 1 – 0.75 7 49
3. (c) According to the question,
7 7
5 7 17 1  a³–b³ =  a – b  a² +b² + ab 1
 of –
3 51 5 3 3 3
2 5 28 2 0.75   1 –  0.75  Factors are: 3, 3, 3, 7, 7, 7
 of – 
9 7 5 3 0.25  Smallest number is = 7

Rakesh Yadav Readers Publication Pvt. Ltd. 91

For More Visit : www.LearnEngineering.in


For More Visit : www.LearnEngineering.in

12. (a) According to the question,


729 13 7
–1  3  1000  +
1 1 1 4096 4 4
1002  0.001 3 – 0.0016 4  30   
5
 4 9
20
  10  =5
16 4
4
 10 × 0.1 – 0.2 × 1 + 19. (d) According to the question,
5  5.625
17. (d) According to the question, 19 1
4  
 10 × 0.1 – 0.2 + 43 1
5 1 17 2
  1
1 22 3
2 4 1

r
3  1
 1–  1
10 5 2 – 4

i
.iSn
7
5 –1 4 9 19 1
  
5 43 1
1 17 2

agv
8   4
3
 = 1.6 1 22 5
5 3
9

ridn
13. (a) According to the question, 2 –
7 19 1
 
1 1 1 1 43 5

eeYa
–  – 1 17 2
2 4 5 6   19
 2 5 3 7 7 22
–  – 3 
5 19 19 38
5 9 5 18   
43 43 43
5
30 – 15  12 – 10
geisnh
 
17
20. (a) According to the question,
60 15  7 22
 36 – 50  54 – 35 1  1  1 1  1 1 1 
8  3  1  1   

5 17

22
=1
 2  4  4 2 2 3 6 
90   
22 22 22
Enak

17 90 1 18. (c) According to the question, 17 13 5 1  3 1 1  


  = 5         
60 5 10 2  4 4 2  2 3 6  
1
14. (c) According to the question, If x = 1
R

1
1 17 13 5 1  9  2  1  
2 2 1       
 272  – 128  1  2  4 4 2  6  
aryn

1
1
  272  128   272 – 128  2
13 5 1 
Les B

17
 2
  4  4  2  1 
 a²–b² =  a+b  a–b  1   
 x 1
1
1
 400  144 2 17 13 5  2
1   
 4 
wa.th

 20×12 = 240 3 2 4
15. (d) According to the question,
1 17 13 3
x = 1  
 4 
1
x =0.001 3 2 4 
1
3
wwM

5
17 16
x = 0.003  
5 2 4
Squaring both sides x= 1
8
x = 0.000009 34  16 18
 =
13 4 4
 Number is = 0.000009  x=
16. (b) According to the question, 8 9 1
 =4
72.9 7 2 2
 3  Value of 2x + is
0.4096 4 21. (d) According to the question,
50 x
729  10000 13 7  
3  2× + x 1
 8 4 12
4096  10 2

Rakesh Yadav Readers Publication Pvt. Ltd. 92

For More Visit : www.LearnEngineering.in


For More Visit : www.LearnEngineering.in

25 1
 50   x² 0.01  0.0001  0.000081  2x 
 4
2 0.0001  0.000001  0.00000081 1
12  1
 x² = 25 × 25
0.010181
1
 x = 25  2  x 
 4
22. (a) According to the question, 0.00010181 1
13
1 1 1 1 1 1 1 1
         100 = 10
9 6 12 20 30 42 56 72 13
27. (b) According to the question,  2x
2 33 4 4 5 5 6 6 7 7 8 8 9 17

1 1 1 1 1 1 1 1 1 1 1 (0.03)²  (0.21)²  (0.065)²


            13

ir
(0.003)²  (0.021)²  (0.0065)²
9 2 3 3 4 4 5 5 6 6 7  x 2
17

v.iSn
1 1 1 1 1 0.0009  0.0441  0.004225
     
7 8 8 9 2 0.000009  0.000441  0.00004225 34  13 21
 x   x=
23. (c) According to the question. 17 17
0.049225

dnag
 25–5 [2 +3 (2 –2 (5–3 )+5)–1 0]÷4   100 = 10 33. (c) According to the question
0.00049225
 25–5 [2 +3 (2 –2 ×2 +5 )–1 0 ]÷4
28. (d) According to the question,        
 25 – 5 [2 + 3 (2 – 4 + 5) – 10] ÷ 4        
 1 1 1  1   1  1 1  1 
104.04  1.0404  0.010404

eYrai
 25 – 5 [2+3 × 3– 10] ÷ 4 
 1
 10  
 
 
1
 10  
 
 
1
 10  
 
 
1 
 10   

 10   10   10   10  
 25 – 5 [11– 10] ÷ 4 10404 10404 10404
  
5 100 10000 1000000    
 25 –    


100  5
4
4

102
10
 snhe
102
100

 10.2+1.02+0.102 = 11.322
102
1000
÷
1 




1
 10 

1
10
  1 –
 
 


1
 10 

1
10




 
kgei
29. (c) According to the question, 1 111
Let 1   a
95  1 101
= 23.75 40.96  0.4096  0.004096  0.00004096
 10 
4 10
4096 4096 4096 4096
ERna

 100

10000

1000000

100000000
1 2 1 5
24. (b) x  =  = 1 91
1 x 2 64 64 64 64 1  b
5   
2  1 101
2 10 100 1000 10000 10 
10
aBryn

25. (a) According to the question  6.4+0.64+0.064+0.0064 = 7.11


30. (a) According to the question, a ²  b²
9 1 1  5 1 1 
As we know that the square of  [ a ²  b ²  (a  b )(a  b )]
a b
      252 is which is near the value
 20  5 4  6 3 2 
of 63522 (a  b )(a  b )
Les

 63522 – x = 63504 
a b
wa.th

 1  1  5 5  
9 x = 18
 20   5   4   6  6  31. (d) According to the question,
 (a – b)
    
111 91
4 20184  
9 1 1  3 5046 101 101
    0
wwM

 20
5 2 1682
 4  29 841 20
29 29 
1 101
9 1 1
 
 5  34. (d) According to the Question
20 4 
Factors are 2, 2 3 , 2 , 29, 29
1 1 1 1 1 1
 It should be multiplied by = 6      
9 9 30 42 56 72 90 110
  =0
20 20 32. (b) According to the question,
1 1 1 1 1 1 1 1 1 1 1 1 1
26. (b) According to the question, If 2  x             
1 5 6 6 7 7 8 8 9 9 10 10 11
1
(0.1)²  (0.01)²  (0.009)² 1 1 1 11  5 6
3
 (0.01)²  (0.001)²  (0.0009)² 4    
5 11 55 55

Rakesh Yadav Readers Publication Pvt. Ltd. 93

For More Visit : www.LearnEngineering.in


For More Visit : www.LearnEngineering.in

35. (b) According to the question, 39. (d) According to the question, 44. (d) According to the question,

1  1  1  1  1  1  1  2  7
 3
875
 7  3 5 7 – 3 5 
   1 
 1  1  1  1  1   1  2      4 =2
   1 1 49 – 45
 3 
  3  125 5 45. (d) According to the question,
 1  1  1  1  1  
  2  40. (d) According to the question,
2 2
2
 11 15   11 15 
  2  4   – 4 – 
 1 1  1  1   2  15 71   15 71 
  3  2 × 0.39
2
3

r
2
 5 3  11 15 11 15  11 15 11 15 
1 11 3

i
 4   4 – 4  – 4  

.iSn
  3   15 71 15 71 15 71 15 71
2
 3  2
 1  1  5  2
6 × 0.39  11   15 
  2  4  2  

agv
3
11  15   71 
8  5
 1  2
 5 

ridn
8  22
 71 15 
2  0.39 2   2   4
36. (d) According to question, 39  15 71 

eeYa
3.36 – 2.05  1.33 2 46. (d) According to the question,
 3.363636 ...– 2.050505... +  22
2
1.333333... 100 1 1
 
 2.646464... 1 1
geisnh 200 100 3 3
  1 4
 2.64 222 111 1 1
1 9
37. (a) According to the question, 41. (b) According to the question, 2
4
 0.2  0.2  0.01 0.1  0.1  0.021 –
1
2 4 1 1 3
–  –  
Enak

 2 3 5 3 5 4
1
0.2  0.2  0.01 1 2 4 1 1 4  13
 –  – – 9
 2 3 3 3 5 5 3  Satisfied
0.1  0.1  0.02 48
R

13
–30 – 40  48 – 20  12  45
0.04  0.01 47. (d) According to the question,
 60
aryn

0.01  0.02 
15  20 – 40  10 – 6 – 24
 1 1 1 1 1 1 
0.05 5 30   3.5  5.7  7.9  9.11  11.13  13.15 
Les B

 =
0.03 3 15 –30 –3
   1 1 1 1 1 1 1 1 1 1 1 1 1 
38. (a) According to the question, 60 25 10  2  3 – 5  5 – 7  7 – 9  9 – 11  11 – 13  13 – 15 
42. (b) According to the question,
wa.th

5  11  19  29  49
 0.63  0.37  0.80 1 1 1  1  5 – 1
 – 
 0.6363...+0.3737....+0.8080... 2  3 15  2  15 
 5  11  19  29  7
 1.81 1 4 2
wwM

  
 5  11  19  36 43. (b) According to the question, 2 15 15

 0.4 48. (c) According to the question,


 5  11  19  6
4  625686734489 =6
 5  11  25 
9 NOTE: For counting the digits of
square root we make pairs first.
 5  11  5 2 Then the digits will be equal to num-

3 ber of pairs.
 5  16
 0.66666..... 49. (b) Let the number of boys = x
 9 =3  0.6 the number of girls = y

Rakesh Yadav Readers Publication Pvt. Ltd. 94

For More Visit : www.LearnEngineering.in


For More Visit : www.LearnEngineering.in

According to the question, Second part


1
 1 1 1 1
 x² – y² = 28 .........(i) 1  1 
1 1 1 1
1 1 2 2
 x = y+2 1 5 1 1
3 1 2 2
1 8 1 24
 x – y = 2 ..........(ii) 5 5–
5 5
From eq. (i)
1 1 1 1
 x² – y² = 28  1 = 1 = 1
= 24
13 34 2
8  1  2 2
1 5 53 53
21 21 2
 (x + y) (x – y) = 28 13 24 24

ir
 (x + y)×2 = 28 54. (a) According to the question 1 53

v.iSn
= 130 =
 x + y = 14 130
1 2 7 13 53
2 –1 –
50. (b) According to the question, 3 11 3 11 According to question,
= 1  1
3 3

dnag
2 0.014  0.14x =0.014×0.14 2 y 1 3 53 53
 3
3 10 = 
1 65 130
3
Squaring both sides 3
53 130 130
=  = =2

eYrai
 0.014×0.14x = (0.014)² × 65 53 65
77 – 39 38 33
(0.14)² × y   
33 33 109 5
10 58. (c) 4 –
3 1
x 1
33
 y

x
= 0.014 × 0.14

 109
38
snhe 3
2
1
1
4
= 0.00196
kgei
 y 55. (a) According to the question 5
 4
1
51. (d) According to the question 1
 8  57  38  108  169 1
3
ERna

9
0.1 ² – 0.01 ² 
  0.0001
 +1
  8  57  38  108  13 4

5 5
8  57  38  121 = 4 – = 4–
aBryn

0.1  0.010.1 – 0.01   1


1
   +1 1 1
0.0001  4 31
  8  57  38  11 3
9 9
0.11  0.09 
 +1 8  57  49
5 5
0.0001 = 4– =4–
Les

  9 40
8  57  7 8  64 1
 99 + 1 = 100
wa.th

31 31
 88  16 =4
52. (c) According to the question 5  31 31 1
56. (c) According to the question = 4– =4– =
40 8 8
 100 –1100 – 2100 – 3............100 – 200 p–q=5 ......(i)
According to question
100  99 98 ....... 3 21
wwM

p² + q² = 55 .......(ii)
1
99  98  97  .......1  0  –1  –2 ....... –100  (p–q)² = p² + q² – 2pq part = 10 minutes
 100  99  98  .....3  2 1 8
(5)² = 55 – 2pq
1 part = 80 minutes
=0 25 = 55 – 2pq
3 3
53. (c) According to the question 2pq = 30  pq = 15 part = 80 × = 48 minutes
5 5
1 57.(c) Take a first part 59. (a) Take first part
 1
1 60 – 7
1 2 1 30 1
1 4 – – 53 1 1 29 9
1 14 4 – 2 –
7
2 7 2
1 = = 49  16 = 65 7 4 7 4
1 11 7 8 = = 7 8
2 3 1  1 1
1 2 7 2 7 14 3 1 
3 2 7 2 7

Rakesh Yadav Readers Publication Pvt. Ltd. 95

For More Visit : www.LearnEngineering.in


For More Visit : www.LearnEngineering.in

62. (d) 33 – 4 3 5
116 – 63
=
3 2  6 – 3 – 4 3  6 – 2
28 = 33 – 2 × 2 35 3 4
53 14 53
= 49  16 = × =
28 65 13 0 = 33 – 2 × 2 × 7 5
14
+
6  3 – 2
Take second part = 28 + 5 – 2 × 2 7  5 1

1 2 2 = 2 3 – 6 –3 2+ 6 +3 2–2 3
1
1
=
2
1
 2 7   5 – 22 7  5
=0
2 1 2
1 2 =
2
1
25 – 1 2 7  5 67. (b)
4 – 0.04

r
5– 5 4  0.4
2
5

i
33 – 4 35 = 

.iSn
1
 2 7 5  4 – 0.2 3.8 38 19
1
= 1
= = = = = = 0.8
2
5 2
1 
  2 7 5  4  0.6 4.6 46 23

agv
2 53 63. (a) (a – b)² = a² + b² – 2ab 68. (b) 0.05  0.5  a = 0.5×0.05× b
24 24 16a²–12a
Squaring both sides

ridn
=
1
=
1
=
53
3  3 2 0.05×0.5×a = 0.5 × 0.5 × 0.05 ×
24 1 06  24 13 0 = (4a)² – 2×4a× + 
  
2 2 2  0.05 × b

eeYa
53 53 a = 0.5 × 0.05 × b
According to the question  3 2 9
Number be added = 
   = a 5 5
53 53 53 130
2  4 
b 10  100
 =  = 1 =1 64. (b) x [–2{–4 (–a)}]+5 [–2{–2(–a)}]
130 130 130 53
geisnh
= 4a a 1
1 1 x [– 2{4a}] + 5[– 2{2a}] = 4a
 = 0.025
60. (c)  b 40
a –b b –a
1 2 1 2 x [– 8a] + 5[– 4a] = 4a
69. (a) (1001)3
– 8ax – 20a = 4a
Enak

1 1 = 1001 × 1001 × 1001


= 
a –b – a –b  – 8ax = 24a
1 2 1 2  = 1002001 × 1001
24a = 1003003001
R

a –b a–b
1 2 1 2 x= –
= a –b

a –b = a–b =1 8a
1 2 2 1 1 2 70. (a). 5. 6 + 7. 3 + 8. 7 +6. 1
x=–3
aryn

   2 
1  3  65. (a) a = 64 , b = 289
1–
61. (b)   1–
+  
+ 1–
 
+.. 6 3 7 1
 n  1
 n 1  n 1 = 5+ +7+ +8+ +6+
Les B

9 9 9 9
a =8, b = 17
 n 
1 –
.........+  
 1  6  3  7  1
 n  1 = 26 +  
2  9
n 1–1 n 1–2 n 1– 3

 a  b – b – a
  
wa.th

= 
 
+ 
 
+ 
 
 ..........
 n 1   n 1   n 1  17
1 = 26 +
= 9
n  1 – n   8  17 – 17 – 8 2 
+ 
 
 8 8
 n 1  = 26 + 1 + = 27 = 27. 8
wwM

1 1
9 9
= 25 – 9 2 = 5 31/2 = 2 2

=
n

n –1

n –2
.......+
1   71. (b) 6. 74 +7. 32
n 1 n 1 n 1 n 1
3 2 4 3 6 74 32
1 66. (d) – + = 6+ +7+
= (n+(n–1)+(n – 2)...........1) 3 6 6 2 3 2 99 99
n 1
3 2 6– 3 4 3 6– 2 74  32 106

 n n  1 
 =  –  =13+ =13+
99 99
1+2+3.....n =  6 3 6– 3 6 2 6– 2

 2  7 7
6 3– 2 =13+1+ =14
1  n n  1  n +  99 99
   3 2 3– 2
n 1
 2  2
 = 14. 07

Rakesh Yadav Readers Publication Pvt. Ltd. 96

For More Visit : www.LearnEngineering.in


For More Visit : www.LearnEngineering.in

72. (b) 0.9 = 0.9 3 14 76 3


=1+ = =5+ -2-
0. 9 = 0.999 ........ 11 11 99 9
0.0 9 = 0.09999....... 75. (c) 8.3 1 + 0. 6 +0.00 2
 76 3 
xx x xx x =3+ 
 99  9 
0. 09 = 0.090909........  
8. 31 1 11 1 =8.979999
0. 66 6 66 6
0.9 is the greatest of all = 3. 43
0. 00 2 22 2 = 8.979
8.97 9 99 9
1 77. (b) 0. 63 +0. 37
73. (b) 0. 001 =
999 76. (d) (5. 76 ) – (2. 3 )
63 37 100

ir
27  76   3 = + =
99 99 99

v.iSn
74. (c) 1. 27 = 1 + 5  99  -
 2  9 

99    

dnag
eYrai
snhe
kgei
ERna
aBryn
Les
wa.th
wwM

Rakesh Yadav Readers Publication Pvt. Ltd. 97

For More Visit : www.LearnEngineering.in


For More Visit : www.LearnEngineering.in

CHAPTER

LINER EQUATIONS 09
IN TWO VARIABLES
• Linear Equations in Two Vari-  x =2 , y = 1 is a solution of 2x+5y (ii) an infinite number of solutions,
ables: An equation of the form =9
a1 b1 c 1

r
ax + by + c = 0 where a, b, c • System of Linear Equa- if  
a 2 b2 c 2

Sni
 R(real numbers) and a  0, tio ns :
b0 Consistent System :- A system a1 b1 c1

gv.i
and x, y are variables is called a consisting of two simultaneous (iii) no solution, if  
linear equation in two variables. a 2 b2 c 2
linear equations is said to be
Examples : Each of the follow- consistent, if it has at least one Homogenous System of Equa-

ridna
ing equations is a linear equa- solutions. tions:
tion : Inco ns is tent Sys tem : A The system of equations a1x +
(i) 4x + 7y = 13 system consisting of two simul- b1y
(ii) 2x - 5y = 36 taneous linear equations is = 0; a2x + b2y = 0 has

eeYa
said to be inconsistent, if it has (i) only solution x = 0, y = 0, when
(iii) 3x  7y  2 a1 b1
no solution at all.

The condition a  0, b  0, is of- E.g.: Consider the system of equa- a 2 b2
ten denoted by a2 + b2  0 tions: x + y = 9 & 3x + 3y = 5.
geisnh (ii) an infinite number of solutions
Note: The graph of a linear equation Clearly, there are no values of x
a1 b1
ax + by + c = 0, is a straight line. and y whic h m ay when 
Solution of linear equation : Any pair simulatneously satisfy the given a 2 b2
of values of x and y which satisfy equations. So, the system given • The graphs of a1x + b1y + c1 = 0,
Enak

the equation ax + by + c = 0, is called above is inconsistent. a2x + b2y + C2 = 0 will be :


its solution. Conditions for Solvability : (i) Parallel, if the system has no
E.g.: show that x = 2 and y = 1 is a The system of equations a1x + Solution ;
solution of 2x + 5y = 9 b1y + c1 = 0, a2x + b2y + c2 = 0 (ii) Coincident, if the system has
R

Sol: Substituting x = 2 and y = 1 in has : infinite number of solutions ;


the given equation, we get LHS a1 b1 (iii) Intersecting, if the system has a
aryn

= 2  2 + 5  1 = 9 = RHS (i) a unique solution, if  unique solution.


a 2 b2
Les B

EXERCISE

1. If 11x -13 = -2x + 78, then x =? 5. If 2x + 3y = 12 and 3x - 2y = 5, 8. The cost of 2 sarees and 4 shirts
wa. th

(a) 7 (b) 8 (c) 6 (d) 4 then x and y must have the is Rs. 16000 while 1 saree and 6
2. If 2x + 3y = 29 and y = x + 3, values : shirts cost the same. The cost of
what is the value of x ? (a) 2 and 3 (b) 2 and -3 12 shirts is :
(a) 5 (b) 6 (c) 4 (d) 7 (c) 3 and -2 (d) 3 and 2 (a) Rs. 12,000 (b) Rs. 24,000
wwM

3. If 2x + 3y = 5 and x = -2, then the 6. The equations ax + b = 0 and cx + (c) Rs. 48,000
value of y is : d = 0 are consistent, if : (d) Can’t be determined
1 (a) ad = bc (b) ad + bc = 0 9. The system of equations kx - y =
(a) (b) 3 (c) 1 (d) 9 (c) ab - cd = 0 (d) ab + cd = 0 2 and 6x - 2y = 3 has a unique
3
4. The value of x + y in the solution 7. The equations 2x + y = 5 and x + solution when :
2y = 4 are (a) K = 0 (b) K  0
x y 5
of the equations + = and (a) consistent and have infi- (c) K = 3 (d) K  3
4 3 12 nitely many solutions 10. The value of y in the solution of
x (b) consistent and have a
+ y =1
2 unique solution. the equation 2x+y = 2x-y  8 is :
1 5 3 (c) inconsistent 1 1 3
(a) (b) 2 (c) (d) (d) none of these (a) 0 (b) (c) (d)
2 2 2 4 2 4

Rakesh Yadav Readers Publication Pvt. Ltd. 98

For More Visit : www.LearnEngineering.in


For More Visit : www.LearnEngineering.in

11. The solutions of the equations 14. The value of k for which the sys- 17. The number of solutions of the
3x  y 1 2x  y  2 3x  2y 1 tem of equations x + 2y = 5, 3x 1
  is + ky + 15 = 0 has no solution, equations x   2 and 2xy - 3y
3 5 6 y
is:
(a) x = 2, y = 1 (b) x = 1, y = 1 = -2 is :
(c) x = -1, y = -1 (d) x = 1, y = 2 (a) 6 (b) -6 (c) 2 (d) 4
(a) 0 (b) 1 (c) 2
12. If x + 2y  3, x > 0 and y > 0, then 15. The equations 2x - 5y = 9 and 8x
(d) None of these
one of the solutions is : - 20y = 36 have : 18. If 2a + 3b = 17 and 2a+2 - 3b+1 = 5,
(a) x = -1, y = 2 (a) no common solution then:
(b) x = 2, y = 1 (b) exactly one common solution (a) a = 2, b = 3 (b) a= -2, b = 3
(c) x = 1, y = 1 (c) exactly two common solu- (c) a = 2, b = -3(d) a = 3, b = 2
(d) x = 0, y = 0

ir
tions 19. The solution to the system of
13. A purse contains 25 paise and 10 equations

v.iSn
(d) more than two common so-
paise coins. The total amount in |x + y|= 1 and x - y = 0 is given by:
lutions
the purse is ` 8.25. If the num-
16. The difference between two 1 1
ber of 25 paise coins is one- third (a) x y  (b) x  y  
numbers is 5 and the difference 2 2

dnag
of the number of 10 paise coins
in the purse, then the total num- between their squares is 65. The 1 1
larger number is : (c) x y or x  y  
ber of coins in the purse: 2 2
(a) 30 (b) 40 (c) 45 (d) 60 (a) 9 (b) 10 (c) 11 (d) 12 (d) x = 1, y = 0

eYari
ANSWER KEY
1. (a) 3. (b) 5. (d) 7. (b) 9. (d) 11. (b) 13. (d) 15. (d) 17. (d) 19. (c)
2. (c) 4. (d) 6. (a) 8. (b)
snhe
10. (a) 12. (c) 14. (a) 16. (a) 18. (d)
kgei
SOLUTION
ERna

1. (a) 11x - 13 = -2x + 78  cost of 12 shirts = (Rs.2000  12)


1 3
 11x + 2x = 78 + 13 x  y 1  = Rs. 24000
2 2 9.(d) For a unique solution, we must
 13x = 91 5.(d) 2x + 3y = 12 (i)
aBryn

have
91 3x - 2y = 5 (ii)
 x 7 (i)  2 + (ii)  3, we get ; x = 3 a1 b1
13 
putting x = 3 in (i), we get 2  3+3y a 2 b2
2.(c) Putting y = x + 3 in 2x + 3y = 29, = 12
we get,
Les

 3y = 6  y = 2 k 1  1
2x + 3(x + 3) = 29  2x + 3x + 9 =    k   6   k  3
 x = 3 and y = 2
wa. th

29 6 2  2
6.(a) The equations are consistent if
20 a b 10.(a) 2x+y = 2x-y = 8 =2
3/2

 5x = 29 - 9 = 20  x  4 
5 c d
3.(b) Putting x = -2 in 2x + 3y = 5, we 3
wwM

i.e. ad = bc x+y= .......(i)


get ; 2
7.(b) 2x + y = 5 (i)
-4 + 3y = 5  3y = 5 + 4 = 9
x + 2y = 4 (ii) 3
9 On solving we get, x = 2, y = 1 and x - y = ........ (ii)
y  3 2
3 Thus (b) is true
4.(d) Given equations are : 8.(b) Let cost of 1 saree = Rs. x & (i) – (ii)
3x + 4y = 5 (i) and cost of 1 shirt = Rs. y 2y = 0  y = 0
x + 2y = 2 (ii)  2x + 4y = 16000 ........(i)
(i) - 2  (ii): x = 5 - 4 = 1 and x + 6y = 16000 .......(ii) 3x - y +1 2x + y + 2
11.(b) =
 from (ii) 2y = 2 - x = 2 - 1 Multiplying (ii) by 2 and 3 5
=1 substracting (i) from it, we get,
1  5(3x - y +1) = 3(2x + y +2)
y  8y = 16000  y = 2000
2

Rakesh Yadav Readers Publication Pvt. Ltd. 99

For More Visit : www.LearnEngineering.in


For More Visit : www.LearnEngineering.in

 9x - 8y = 1 .........(i)  y(2x - 3) = -2 ......(ii)


1
and x  y  y  3x (ii)
3x – y  1 3x  2y  1 3 1
and = putting y = in (ii)
3 6 putting y = 3x in (i), we get : 2x
= 2(3x - y + 1) = (3x + 2y + 1) 5x + 6x = 165  11x = 165  x =
15 2x  3
 3x - 4y = -1 ........(ii)  2  2x - 3 = -4 + 2x. this
 from (ii), y = 3x = 3  15 = 45 2x
(i) - 2  (ii):- gives 1=0
 Total number of coins in the
(9 - 6)x - 8y + 8y = 1 - (-2)  x =1 purse This is impossible So, there is no
putting x = 1 in (i) we get, 9  1 - 8y = x + y = 15 + 45 = 60 solution.
=1 14.(a) a1x + b1y + c1 = 0 18.(b) 2a + 3b = 17 and

r
 8y = 8  y = 1 a2x + b2y + c2 = 0 will have no 2a+2 - 3b+1 = 5  22.2a - 3.3b = 5

Sni
x =1, y = 1. solution if
  4.2a - 3.3b = 5
12. (c) Here we will go through a1 b1 c 1 1 2 let 2a = x & 3b = y then

gv.i
    k  6
options. a 2 b2 c 2 3 k x + y = 17 ...........(i)
in option (a) x < 0 and 4x - 3y = 5 ............(ii)
15.(d) The given equations are 2x - 5y
in option (d) x = 0 3  (i) + (ii), we get

ridna
= 9 and 8x - 20y = 36  2x - 5y
hence (a) and (d) can’t be the 7x = 56  x = 8  2a = 8 = 23 
=9
required answer because both
Thus, there is one equation in two a=3
does not satisfy the given condi-
variables. So, the given equations

eeYa
tion i.e. x > 0. putting x = 8 in (i), we get
have an infinite number of solu-
Now option (b) x = 2, y = 1, then tions. y = 17 - 8 = 9  3b = 9 = 32 
x + 2y = 2 + 2(1) = 4 which is > 3 16.(a) Let the numbers be x and y. b=2
clearly, values of option (b) do not Then,
geisnh  a = 3 and b = 2.
satisfy x + 2y  3 x - y = 5 and x2 - y2 = 65
19.(c) Note that |a| = 1 means a = 1
option (c) x = 1, y = 1, then x + 2y x 2  y 2 65 or a = -1
= 1+ 2 = 3  3    x  y  13
x y 5 So, |x + y|=1  x + y = 1 or -(x + y)
So, x =1, y = 1 is one of the
Enak
=1
solving x - y = 5 and x + y = 13,
solutions.  (x + y) = – 1
we get;
13. (d) Let the number of 25 paise x = 9 and y = 4 solving x + y = 1, x - y = 0, we get
coins be x & that of 10 paise coins
R

 larger number = 9 1 1
be y, then: x= and y =
17.(d) 2 2
aryn

25 10 solving x + y = – 1, x – y = 0, we
x y  8.25 1 1 1
100 100 x  2  2x y  ........( i ) get x = –1/2 and y = – 1/2
y y 2x
Les B

 5x + 2y = 165 (i) 1
and 2xy - 3y = -2  x=y= 
2
wa. th
wwM

Rakesh Yadav Readers Publication Pvt. Ltd. 100

For More Visit : www.LearnEngineering.in


For More Visit : www.LearnEngineering.in

CHAPTER

10
POLYNOMIALS
• Polynomials : An expression Nature of Roots • If the two roots α &β be equal in
of the form p(x ) = a0 + a1x + a2 The value of x at which value of magnitude and opposite in sign,

ir
x 2 + ......+ an xn , where an  0, equation will be zero. then b = 0

v.iSn
is called a polynomial in 1. Roots are imaginary : • If a,b,c are rational number and
x of degree n. b² – 4ac  0
a  b is one r oot of the
Here a0, a1, a2, .....an are real 2. Roots are real:
quadratic equation, then the
numbers and each power of x b² – 4ac  0

dnag
other root must be conjugate
is a non-negative integer. b² –4ac  0
e.g. a  b and viceversa
(i) 2x + 7 is a polynomial in x of Ex.1Find the Quadratic equation
different equal

eYari
degree 1. whose one root is 3  3
 
(ii) 2y2 - 5y + 7 is a polynomial in b²–4ac>0 b²–4ac=0 Sol. If one root is 3  3 then second
y of degree 2.
root will be 3  3
(iii) 3u3 +
3 2
7
u - 8u + 7 is a poly-

nomial in u of degree 3.
b²–4ac

snhe
rational irrational

perfect square a + b a– b
Sum of root

  
= 3  3 + 3 – 3 =6 
kgei
2 3 3 Sum & product of root:- Product of root
(iv) 5t4 – t  3t2  is a poly-
7 8 Let there are two roots named
nomial in t of degree 4.  & , then

= 3 3  3 – 3  =6
ERna

using,
–b  b² – 4ac
(v)  x 5 , 1
, 2
5
x  3 x 3x 1
etc. =
2a
& x2 (sum of root) x + (product of
root) = 0
 x2 –6x + 6 = 0
aBryn

are not polynomials. –b – b² – 4ac


= Ex.2: Two roots of equation 2x2 – 7x
• Polynomials of Various De- 2a
+ 12 = are
grees : Sum of root:-
(1) Linear Polynomial : A polyno- α β
–b α &β then, find β + =?
 = α
Les

mial of degree 1 is called a lin-


a
ear polynomial. Sol. 2x2 – 7x + 12 = 0
wa. th

Product of root:
A linear polynomial is of the On comparing with standard
form p(x) = ax + b, where a  0 c equation ax2 + bx + c = 0
 =
e.g. a a =2, b = –7, & c = 12
then, ax² + bx + c = 0 can be –b 7
wwM

7 α +β =  α +β =
3x  7 ,   
2x  5 ,  x   etc. written as: a 2
 3 b c c
 x² + x+ =0 α β = α β = 6
(2) Quadratic Polynomial : A a a a
polynomial of degree 2 is called α β α2  β2
 –b  c  β + = αβ
a quadratic polynomial.  x² –  a  x + =0 α
  a
It is of the form p(x) = ax2 + bx αβ2 – 2αβ
+ c, where a  0 x²–(sum of root)x + product =
αβ
of root=0
2

E.g. (2x +7x - 9), 3x  2x  7 , 2
 • If the roots  &  be recipro-
7 
2

  –26
= 2
y 2
 7y  5  etc. cal to each other then a = c. 6

Rakesh Yadav Readers Publication Pvt. Ltd. 101

For More Visit : www.LearnEngineering.in


For More Visit : www.LearnEngineering.in

(2) A quadratic polynomial whose (iii) (x n - an) is divisible by (x + a)


49 only for even values of n.
– 12 49 – 48 1 zeros are  and β is given by
= 4 = = (iv) (x n + an) is never divisible by (x
6 46 24
p(x ) = {x2 - (  + β )x +  β } - a)
Ex.3: Find the product of the root (3) If  , β and γ are the zeros • H.C.F & L.C.M of Polynomi-
of the equation x 2 – 3  0 of p(x) = ax3 + bx2 + cx + d, then, als :
Sol. On comparing this equation with Divisor : A polynomial p(x) is
b
ax2 + bx + c = 0 (i)  +β + γ =  called a divisor of another poly-
a nomial f (x ) = p(x ).g(x ) for
a = 1, b = 0 & c = – 3
c some polynomial g(x ).
c (ii) ( β +β γ + γ  )  • H.C.F. or (G.C.D.) of Polyno-

r
Product of root αβ = = – 3 a
a mials : A polynomial h(x ) is

Sni
• Byquadratic Polynomial : A d called the H.C.F. or G.C.D of
(iii)  βγ = 
polynomial of degree 4 is called a two or more given polyno-

gv.i
a biquadratic polynomial. (4) A cubic polynomial whose ze- mials, if h(x ) is a polynomial
It is of the form P(x) = ax4 + bx3 of heighest degree dividing
ros are  , β and γ is given by
+ cx2 + dx + e where a  0 each one of the given polyno-

ridna
p(x) mials.
E.g. (3x4 + 7x3 - 4x2 + 6x + 11), (4t4
- 7t3 + 6t2 - 11t + 9) etc. = {x 3 - (  + β + γ )x 2 + (  β • Remark : The coefficient of
(3) Cubic Polynomial : A polyno- +β γ + γ  ) x - β γ } heighest degree term in H.C.F

eeYa
mial of degree 3 is called a cu- is always taken as positive.
Factor Theorem : The Condi-
bic polynomial. e.g. What is the HCF of (x + 3)2 (x -
tion that (x - a) is a factor of a
It is of the form P(x) = ax3 + bx2 2)3 and (x - 1) (x + 3)(x - 2)2 ?
polynomial f(x), if and only if
+ cx + d, where a  0 Sol. p(x ) = (x + 3)2 (x - 2)3
geisnh
f (a) = 0
E.g. (4x 3 – 2x 2 + 7x + 9) , Thus, (x - a) i s a factor q(x ) = (x - 1) (x + 3) (x - 2)2

2 
2y 3 – 5y 2 – 8 etc. of f (x)  f (a) = 0. We see that (x + 3) (x - 2)2 is
such a polynomial that is a
Value of a Polynomial at a Remarks : (i) (x + a) is a factor
common divisor and whose de-
Enak

of polynomiual p(x) if and only


given point: gree is heighest among all com-
if p(-a) = 0
If P(x) is a polynomial in x and mon divisors.
(ii) (ax - b) is a factor of a polyno-
if  is any real number, then

R

L.C.M. of Polynomials : A
the value obtained by put- b
mial p(x), if p   0 polynomial p(x ) is called the
ting x =  in P(x) is called the a L.C.M. of two or more given
aryn

value of P(x) at x =  (iii) (ax + b) is a factor of a polyno- polynomials, if it is a polyno-


The value of P (x) at x =  is
Les B

 b mial of smallest degree which


denoted by p(  ). mial p(x), if p    0 is divided by each one of the
e.g. Let p(x) = 3x2 - 2x + 7. then  a
given polynomials.
p(2) = (3  22 - 2  2 +7) (iv) (x - a) (x - b) are factors of a
e.g. Find the L.C.M of (x - 3) (x +
= (12 - 4 + 7) polynomial p(x) if p(a) = 0 and
wa. th

4)2 and (x - 3)3 (x + 4) :


= 15 p(b) = 0.
Sol : p(x ) = (x - 3) (x + 4)2
p(-1) = [3  (-1)2 - 2(-1) + 7] • Remainder Theorem : If a
polynomial f (x) of degree n  1, q(x ) = (x - 3)3 (x + 4)
= (3+2+7) = 12
is divided by (x - a), then the we make a polynomial by
wwM

Zeros of a Polynomial : A real taking each factor of p(x )


number  is called a zero of the remainder is f (a).
e.g. Let f (x ) = x 3 + 3x 2 - 5x + 4 be and q(x).
polynomial p(x), if p(  )= 0
divided by (x -1). Find the re- If a factor is common in both,
Note : 1. If  and β are the mainder. then we take the factor which
zeros of p(x) = ax 2 + bx + c, Sol. Remainder = f (1) has highest degree in p(x )
a  0, then. = 13 + 3  12 - 5  1+ 4 = 3 and q(x ).
Important Results :  LCM = (x - 3)3 (x + 4)2
b
(i)  β  (i) (x n - an) is divisible by (x - a) Note : For any two polynomials
a
for all values of n. p(x ) and q(x )
c (ii) (x n + an) is divisible by (x + a) p(x)  q(x ) = (Their H.C.F.)
(ii)  β only when n is odd.
a  (Their L.C.M.)

Rakesh Yadav Readers Publication Pvt. Ltd. 102

For More Visit : www.LearnEngineering.in


For More Visit : www.LearnEngineering.in

• Factorisation of Polynomials (iii) (x + y)2 + (x - y)2 = 2 (x 2 + y2)


(iv) (x + y)2 - (x - y)2 = 4 xy 1
: To express a given polynomial = (x + y + z) [(x - y)2 + (y - z)2
as the product of polynomials, (v) (x + y)3 = x 3 + y3 + 3x y (x + y) 2
each of degree less than that (vi) (x - y)3 = x 3 - y3 - 3x y (x - y) +(z -x)2]
of the given polynomial such (vii) x 2 - y2 = (x + y) (x - y) (xii) x 2 + y2 + z2 - x y - yz - zx
that no such a factor has a fac- (viii) (x 3 + y3 )= (x + y) (x 2 + y2 - x y)
tor of lower degree, is called (ix) (x 3 - y3 )= (x - y) (x 2 + y2 + x y) 1
factorisation. = [(x - y)2 + (y - z)2 + (z - x )2]
(x) (x + y + z)2 = (x2 + y2 + z2 + 2 (xy 2
• Formulae for Factorisation: + yz + zx)]
(xiii) x 4 + x 2y2 + y4 = (x 2 + x y + y2)(x
(i) (x + y)2 = x 2 + y2 + 2x y (xi) (x 3+ y 3 + z 3 - 3xyz = (x + y + z) 2
(ii) (x - y)2 = x 2 + y2 - 2x y (x 2 + y 2 + z 2 - x y - yz - z x) - x y + y2)

ir
v.iSn
EXERCISE

dnag
1. If f (x) is divided by (3x + 5), the (d) neither (x - 1) nor (x + 1) 16. If  and β are the zeros of x 2 + 3x +
remainder is : 9. The value of expression (9x 2 +
7, then the vaue of (  + β ) is :

eYari
3  3 4
(a) f   (b) f   12x + 7) for x   is : (a) -3 (b) 3 (c) 7 (d) -7
5  5 3
17. If  and β are the zeros of 2x 2 +
(a) 7 (b) 0 (c) - 7 (d) 18
5  5 10. When (x 3 - 2x 2 + px - q) is di- 3x -10, then the value of  β is:

2.
(c) f  
11
3
(d) f   
 3
If (x + 1) is divided by (x + 1),
the remainder is :
snhe
vided by (x 2 - 2x - 3), the re-
mainder is (x - 6). The values of
P and q are :
5
(a) 
2
(b) 5 (c) - 5 (d) 
3
2
kgei
2
(a) 0 (b) 2 (a) p = - 2, q = -6 18. If common factor of x + bx + c
(c) 11 (d) 12 (b) p = 2, q = -6 and x 2 + mx + n is (x + a), then
3. When (x - 3x + 2x 2 - 5x + 7)
4 3 (c) p = - 2, q = 6 the value of a is :
ERna

is divided by (x - 2) , the (d) p = 2, q = 6 cn cn


remainder is : 11. If (x - a) is a factor of (x 3 - 3x (a) (b)
bm bm
2
(a) 3 (b) -3 (c) 2 (d) 0 a + 2a 2x + b), then the value
cn c 1
4. If (x -2) is a factor of (x 2 + 3qx - of b is : (c) (d)
mb bm
aBryn

2q), then the value of q is : (a) 0 (b) 2 (c) 1 (d) 3


19. (x 4 + 5x 3 + 6x 2) is equal to :
(a) 2 (b) - 2 (c) 1 (d) - 1 12. If x100 + 2x 99 + K is divisible by
(a) x (x + 3) (x 2 + 2)
5. The value of  for which the (x + 1), then the value of K is :
(b) x 2 (x + 3) (x + 2)
expression x3 + x2 -5x +  will (a) – 3 (b) 2 (c) – 2 (d) 1
13. If the polynomial f (x ) is such (c) x 2 (x -2) (x - 3)
Les

be divisible by (x -2) is : (d) x (x 2 + 3) (x + 2)


that f (-1) = 0, then a factor of
(a) 2 (b) - 2 (c) - 3 (d) 4
wa. th

f(x) is : 20. The factors of (x 4 + 625) are :


6. If (x + 1) and (x - 2) be the (a) (x 2 - 25) (x 2 + 25)
factors of x 3 +(a + 1)x 2 - (b - 2)x (a) – 1 (b) x – 1
(c) x + 1 (d) –1 – x (b) (x 2 + 25) (x 2 + 25)
- 6, then the value of a and b
14. If x 3 + 5x 2 + 10K leaves remain- (c) (x 2 -10x + 25)(x 2 + 5x + 24)
will be :
wwM

der -2x when divided by x 2 + (d) do not exist


(a) 2 and 8 (b) 1 and 7
2, then the value of k is : 21. The factors of (x 4 + 4) are :
(c) 5 and 3 (d) 3 and 7
(a) - 2 (b) 1 (c) - 1 (d) 2
(a) (x 2 + 2)2
7. The polynomial (x 4 - 5x 3 + 5x2 -
15. Which of the following is a (b) (x 2 + 2) (x 2 - 2)
10x + 24) has a factor as :
polynomial ? (c) (x 2 + 2x + 2)(x 2 - 2x + 2)
(a) x + 4 (b) x - 2
(d) None of these
(c) x + 2 (a) x 2 - 3x + 2 x +7 22. (x + y)3 - (x - y)3 can be factor-
(d) None of these
ized as :
8. (x 29 - x 25 + x13 -1) is divisible by: 1
(b) x  (a) 2y (3x 2 + y 2)
(a) both (x -1) & (x + 1) x (b) 2x (3x 2 + y 2)
(b) (x - 1) but not by (x + 1) (c) x 7/2 - x + x 3/2 (c) 2y (3y 2 + x 2)
(c) (x + 1) but not by (x - 1) (d) None of these (d) 2x (x 2 + 3y 2)

Rakesh Yadav Readers Publication Pvt. Ltd. 103

For More Visit : www.LearnEngineering.in


For More Visit : www.LearnEngineering.in

23. The H.C.F. of x 2 - x y - 2y2 and 28. The H.C.F of 2(x 2 - y2) and 5(x 3 (c) a = 6, b = 8
2x 2 - x y - y2 is : - y3) is : (d) a = 5, b = 8
(a) (x + y) (a) 2(x 2 - y2) (b) (x - y) 33. Factories : (x 8 + x 4 y 4 + y 8)
(b) (x - y) (c) (x + y) (d) (x 2 + y2) (a) (x 2 + x y + y2) (x 2 - x y + y2)
(c) (2x - 3y) 29. The L.C.M of (2x 2 - 3x + 2) and (x 4 -x 2 y2 + y4)
(d) None of these (x 3 - 4x 2 + 4x ) is : (b) (x 2 + x y - y2) (x 4 - x 4 y 4 + y 4)
24. The H.C.F. of (x 3 + x 2 + x + 1) (a) x (2x 2 + 1) (x 2 + 2) (c) (x 2 + x y + y2)2 (x 4 - x 2 y 2 + y 4)
and (x 4 - 1) is: (b) x (2x + 1) (x - 2)2 (d) (x 2 - x y + y2)2 (x 4 - x 4 y 4 - y 4)
(a) (x 2 – 1) (x 2 + 1) (c) x (2x 2 + 1) (x - 1)2
(b) (x + 1) (x 2 - 1)  6 y6 
(d) x (2x + 1) (x 2 - 1) 34. Factorise :  x  27 
(c) (x + 1) (x 2 + 1) 30. The L.C.M of (a3 + b3) and (a4 -  

r
(d) (x 2 + 1) (x + 1) (x 3 + 1) b4) is :

Sni
25. The L.C.M of the polynomials X  2 y2   4 x 2y2 x 2y6 
(a) (a3 + b3) (a2 + b2) (a - b) (a)  x  3   x  3  9 

and Y, where X = (x + 3)2 (x - 2)    

gv.i
(b) (a3 + b3) (a + b) (a2 + b2)
(x + 1)2 and Y = (x + 1)2 (x + 3) (x
(c) (a + b) (a2 + ab + b2) (a3 + b3)  2 y2   4 x 2 y2 y4 
+ 4) is given by :
(d) (a 3 + b3) (a2 - b2) (a - b) (b)  x  3  x –  
(a) (x - 2) (x + 4) (x + 3)2 (x + 1)2    3 9 

ridna
31. If Polynomials 2x 3 + ax 2 + 3x -
(b) (x + 1) (x - 2) (x + 3) (x + 4)
5 and x3 + x2 - 2x + a are di-
(c) (x - 2) (x + 1) (x + 3)2 (x + 4)  2 y2   4 x 2y2 x 2y 4 
vided by (x – 2), the same re-
(d) (x - 2) (x + 1)2 (x + 3) (x + 4) (c)  x  3  x 
 3

9 

mainder are obtained. Find the   

eeYa
26. The L.C.M of (x + 2)2 (x - 2) and
value of a :
(x 2 - 4x - 12) is :  2 y2   4 x 2y2 y 4 
(a) 3 (b) - 9 (c) - 3 (d) - 5
(a) (x + 2)(x – 2)
32. If the polynomial f (x) = x 4 - 2x (d)  x  3  x 
 3

9


(b) (x + 2) (x - 2) (x - 6) 3
   
+ 3x 2 - ax + b is divided by
geisnh
(c) (x + 2) (x -2)2 (x - 1) and (x + 1), the remain- 35. Factorise : (x 4 + x 2 + 25)
(d) (x + 2)2 (x - 2) (x - 6) ders are 5 and 19 respectively. (a) (x 2 + 3x + 5)(x 2 + 3x – 5)
27. The H.C.F. of (x 2 - 4), (x 2 - 5x - The values of a and b are: (b) (x 2 + 5 + 3x) (x 2 + 5 – 3x)
6) and (x 2 + x -6) is : (c) (x 2 + x + 5) (x 2 – x + 5)
(a) a = 8, b = 5
Enak

(a) 1 (b) (x - 2) (d) None of these


(b) a = 5, b = 6
(c) (x + 2) (d) (x 2 + x - 6)
ANSWER KEY
R

1. (d) 5. (b) 9. (a) 13. (c) 17. (c) 21. (c) 24. (c) 27. (a) 30. (a) 33. (a)
aryn

2. (a) 6. (b) 10. (c) 14. (b) 18. (a) 22. (a) 25. (a) 28. (b) 31. (c) 34. (b)
3. (b) 7. (b) 11. (a) 15. (d) 19. (b) 23. (d) 26. (d) 29. (b) 32. (d) 35. (b)
Les B

4. (d) 8. (b) 12. (d) 16. (a) 20. (d)

SOLUTION
wa. th

4q = - 4  q = -1 6= 0
5
1.(d) 3x + 5 = 0  x   5.(b) (x - 2) is a factor of or a + b = 8 ......(i)
3
polynomial and 2a - b = -5 ......(ii)
wwM

 5 f (x ) = x 3 + x 2 - 5x +  (i) + (ii) 3a = 3  a = 1
So, remainder is f     From equation (i)
 3  f (2) = 0  23 + 22 - 5  2 +
2.(a) Remainder = f (-1) =0 b=8-1=7
= (-1)11 + 1  a=1&b=7
 12 – 10 +  = 0   = – 2
= -1 + 1 = 0 7.(b) Since x = 2 makes the given
6.(b) Since (x + 1) & (x - 2) are the expression zero, so, (x - 2) is
3.(b) Remainder = f (2) factors of its factor.
= 24 - 3 (2)3 + 2(2)2 -5  2 +7 f (x ) = x3 + (a + 1)x 2 - (b - 2) 8.(b) Since x = 1 makes x 29 - x 25 +
= 16 - 24 + 8 - 10 + 7 = - 3 x-6 x 13 - 1 zero, so (x - 1) is its
4.(d) Since (x - 2) is a factor of f (x )  f (-1) = 0 and f (2) = 0 factor. And x = -1 does not
= x2 + 3qx - 2q or -1 + (a + 1) + (b - 2)-6 = 0 make it zero
 f (2) = 0  22 + 3q  2 - 2q = 0 and 8 + 4(a + 1) - (b - 2)  2 - so (x + 1) is not its factor.

Rakesh Yadav Readers Publication Pvt. Ltd. 104

For More Visit : www.LearnEngineering.in


For More Visit : www.LearnEngineering.in

9.(a) f (x) = 9x 2 + 12x + 7  from (i) and (ii)  H.C.F. = (x - y)


2 ab - c = ma - n 29.(b) 2x 2 - 3x + 2 = 2x 2 - 4x + x -
 4  4  4 cn 2 = 2x (x - 2) + 1 (x - 2) = (x -
 f     9     12     7
 3  3  3  a (b  m)  c  n or a  2) (2x + 1)
bm
= 16 - 16 + 7 = 7 19. (b) x 4 + 5x 3 + 6 x 2 = x 2 (x 2 + 5x x 3 - 4x 2 + 4x = x (x 2 - 4x + 4)
10.(c) + 6) = x (x - 2)2
x = x 2 (x 2 + 3x + 2x + 6)  L.C.M. = x (x - 2)2 (2x + 1)
= x 2 [x (x + 3) + 2(x + 3)] 30.(a) a3 + b3 = (a + b) (a2 - ab + b2)
x2  2x  3 x3  2x2  px  q
= x 2 (x + 3) (x + 2) a4 - b4 = (a - b) (a +b) (a2 + b2)
x3  2x2  3x
   20.(b) Do not exist  L.C.M. = (a - b) (a + b)
21. (c) x 4 + 4 = (x 2)2 + (2)2 + 4x 2 - 4x2 (a2 -ab + b2) (a2 + b2)
 p  3 x  q  remainder

ir
= (x + 2)2 - (2x)2 = (a - b) (a3 + b3) (a2 + b2)
 (p + 3)x - q = x - 6

v.iSn
= ( x 2 + 2x + 2) (x 2 - 2x + 2) 31.(c) f (x ) = 2x 3 + ax 2 + 3x - 5
 p + 3 = 1 and q = 6
or p = - 2 and q = 6 22. (a) Using formuale , a3 - b3 g(x ) = x 3 + x 2 - 2x + a
11.(a) let f (x ) = x 3 - 3x 2a + 2a2x + = (a - b) (a2 + b2 + ab) By remainder theorem,
b  (x + y)3 - (x - y)3 = [(x + y) - (x f (2) = 2(2)3 + a(2)2 + 3  2 - 5

dnag
- y)] + [(x + y)2 + (x - y)2 + (x +
 (x - a) is a factor of f (x ) = 17 + 4a
y) (x – y)]
 f (a) = 0  a3 - 3a3 + 2a3 + b = 0 and, g(2) = 23 + (2)2 - 2  2 + a
= 2y [2(x 2 + y2) + (x 2 - y2)]
 b=0 = 2y (3x 2 + y 2) =8+a

eYari
12.(d)  x 100 + 2x 99 + k 23.(d) x 2 - x y - 2y2 = (x 2 - y2) - (x y  17 + 4a = 8+a
= f (x) (let) is divisible by (x + 1) + y2)  3a = - 9 or a = - 3
 f (-1) = 0 = (x + y) (x - y) - y (x + y)
32.(d) By remainder theorem,
 1-2+k=0  k=1 = (x + y) (x - y - y)
13.(c) Since x = -1 makes f (x ) zero,

14.(b)
So (x + 1) is its factor.
snhe
= (x + y) (x - 2y)
2x 2 - x y - y2 = (x 2 -x y)
+ (x 2 - y2)
f (1) = 5 .......(i) [ x - 1 = 0 
x = 1]
and f (-1) = 19 .....(ii) [  x +
1 = 0  x = -1]
kgei
x 5 = x(x - y) + (x + y) (x - y)
x2  2 x3  5x2 10k = (x - y) (x + x + y) Now, from (i) 1 - 2 + 3 - a + b
= (x - y) (2x + y) =5
x3  2x
ERna

  Clearly, no factor is common, or b - a = 3 ...... (iii)


So, H.C.F = 1 from (ii) 1 + 2 + 3 + a + b = 19
5x2  2x 10k
24.(c) x 3 + x 2 + x + 1 = x2 (x + 1) +1 or a + b = 13 ......(iv)
5x2 .  10 (x + 1)
  (iii) + (iv) 2b = 16 or b = 8
aBryn

= (x + 1) (x2 + 1)
 2x  10k  10  Remainder Now from (iv), a = 13 - 8 = 5
x 4 - 1 = (x 2 - 1) (x 2 + 1)
but given, remainder = - 2x  a = 5, b = 8
= (x + 1) (x - 1) (x 2 + 1)
 - 2x + 10k - 10 = -2x 33.(a) x 8 + x 4y4 + y8
 Required H.C.F = (x + 1) (x 2 + 1)
 10k = 10 25.(a) X = (x + 3)² (x – 2) (x+1)² = x 8 + 2x 4 y4 + y8 – x 4y 4
Les

 k=1 Y = (x + 1)² (x + 3)(x + 4) = (x 4 + y 4)2 - (x 2y2)2


wa. th

15.(d) For polynomial, each power So, LCM = (x – 2) (x + 4) (x = (x 4 + y 4 + x 2y2) (x 4 + y4 - x


of x must be a non-negative +3)² (x + 1)² 2 2
y)
integer. 26.(d) x 2 - 4x - 12 = x 2 - 6x + 2x - 12 = [(x 2 + y2)2 - (x y)2] (x 4 - x 2y2 +
b 3 = x (x - 6) + 2(x - 6) y4)
16.(a)   β      3
= (x + 2) (x - 6)
wwM

a 1 = (x 2 + x y + y2) (x 2 - x y + y2) (x 4 - x
and other is (x + 2)2 (x -2) 2 2
y + y4)
c 10
17.(c)  β   5  L.C.M = (x + 2)2 (x – 2) (x - 6) 3
a 2 27.(a) x 2 - 4 = (x + 2) (x -2) y6  y2 
18.(a) Let f (x ) = x 2 + bx + c x 2 - 5x - 6 = x 2 - 6x + x - 6 34.(b)
6
x 
27
 x2   3
  

and g (x) = x 2 + mx + n  3 
= (x -6) (x + 1)
 (x + a) is a common factor of and x 2 + x - 6 = x 2 + 3x - 2x - 6  2 y2   4 x 2y2 y 4 
f (x ) and g(x) = (x + 3) (x - 2) =  x  3   x  3  9 
 f (-a) = 0 and g (–a) = 0    
Clearly, ther is no common factor.
or a2 – ba + c =0 and a2 – ma + 35.(b) x 4 + x 2 + 25 = (x 2)2 + (5)2 + 10x
So, H.C.F = 1. 2
n=0 - 9x2
28.(b) 2(x 2 - y 2) = 2(x - y) (x + y)
 a2 = ab - c...(i) and a2 = ma - and 5(x 3 - y3) = 5(x - y) (x 2 + = (x 2 + 5)2 - (3x )2
n .....(ii) y 2 + x y) = (x 2 + 5 + 3x) (x 2 + 5 - 3x)

Rakesh Yadav Readers Publication Pvt. Ltd. 105

For More Visit : www.LearnEngineering.in


For More Visit : www.LearnEngineering.in

CHAPTER

11
ALGEBRIC IDENTITIES
• An alg ebraic identity is an
1 1
algebraic equation which is true 2. If x – – a, then 8. If x+ =1, then x³ = –1

r
for all values of the variable (s). x x

Sni
Important Formulae: 1 1
1. (a+b)2 = a2 + b2 + 2ab (i) x²+ =a2+2 9. If x+ =–1, then x³ =1,
x2 x

gv.i
2. (a–b)2 = a2 + b2 – 2ab
3. (a+b)2 = (a – b)2 + 4ab 1 1 1
(ii) x4+ = b²–2, where b = a²+2 10. If x+ = 3 then x³+ = 0  x6
4. (a–b)2 = (a + b)2 – 4ab x4 x x³

ridna
5. a2–b2 = (a + b) (a–b) = –1 or x6+1 = 0
1
6. (a+b)3 = a3 + b3 + 3ab(a+b) e.g. x – =3, then 11. If ax + by = m and bx –ay = n
7. (a–b)3 = a3 – b3 – 3ab(a–b) x then, (a²+b²) (x² + y²) = m² + n2

eeYa
8. (a³+b³) = (a + b) (a2–ab+b2) 1 Note : If the sum of squares of real
9. (a³–b³) = (a – b) (a²+ab+b²) x²+ =3² + 2 = 11 and numbers be zero, then each number
x2
10. (a+b+c)² = a² + b ² +c ² + is equal to zero i.e.
2(ab+bc+ca) 1 if (x – a)² + (y – b)² + (z – c)² = 0,
x4+ = 11²–2= 119. then
x4
11. a³+b³+c³–3abc
geisnh
x – a = 0  x = a,
= (a+b+c) (a²+b²+c²–ab–bc–ca)
1 y – b = 0  y = b and
1 3. If x4+ = a, then z–c=0  z=c
= ( a + b + c ) [ ( a – b ) 2+ ( b – c ) 2 x4
2 or if x ²+ y² + z² = 0, then x = 0, y
1
Enak

+ (c–a)²] =0&z=0
(i) x²+ = a  2 =b
Note:- x2 Based on Increasing power
a³+b³+c³–3abc = 0, 1 1
Ex.1 If x   3 , find the value of
R

If {(i) a + b + c = 0 (a  b  c) (ii) x+ = b 2 x
x
or
1
(ii) a²+b²+c²–ab–bc–ca = 0 1 x²  ?
aryn

(ii) x– = b–2 x²
1 x
1
Les B

12. a²+b²+c²–ab–bc–ca = Sol. x 3


2 1 x
e.g. x4+ 4 =119
[(a–b)²+(b–c)²+(c–a)2] x Squaring both sides,
13. If ax2 +bx +c = 2
1  1 2
 x²+ = 119  2 =11   x     3
wa. th

–b  b² – 4ac x2  x
0, then, x =
2a 1
x+ = 11  2 = 13 1 1
Some Important Results x  x²   2 x   9
x² x
1 1
wwM

1. If x + = a, then 1
x x– = 11 – 2 =3  x²  9–27
x x²
1
(i) x² + 2 = a²–2 1 1 1
x 4. If x+ =2, then x =1 (If x   a then x ²  = a²–2)
x x x²
1
(ii) x 4+ 4 = b 2 – 2 wher e 1 1
x 5. If x+ = –2, then x = –1 Same x 4  = (a² – 2)2 – 2
1 x x4
b = a²– 2 0 e.g. x + = 3, 3 Alternate:-
x 1  1  1
Then, 6. x³+ x  
= x   –3   1
x³  x   x x 3
1 1 3
x
x2+ =3²–2 = 7, and x4 + 4 1  1  1
x² x 7. x³– x – 
= x –  +3   1
= 49–2=47 x³  x  x x²  = 3² – 2 = 7

Rakesh Yadav Readers Publication Pvt. Ltd. 106

For More Visit : www.LearnEngineering.in


For More Visit : www.LearnEngineering.in

1 1 1 1
Ex.2 If x   5 , find the (i) x ²  x2  = 6² – 2 = 34 Sol. x 3
x x² x2 x
1 1 Cube both sides,
(ii) x 4  x4  = 34² – 2 = 1154
x4 x4  1
3
3

1 x    3
1  x
Sol.(i) x  5
x x8  = 1154²– 2 = 1331714
x8 1  1
1 x³   3  x   x    27
x²  = 5² – 2 = 23 1 x³ x x
x² Ex.5 If x –  3 , Find the value of
x 1
1  Put the value of x  3
(ii) x  5 1 1 x

ir
4
x (i) x ²  (ii) x  4
x² x

v.iSn
1 1
x²  x³   3  3 = 27
= 23 1 x³
x² Sol. x – 3
Again squaring both sides x
1
squaring both side, x³   27 – 9  18

dnag
2
 2 1  2 x³
 x  2    23   1
2
 x  2
1
x –   3 (If x 1 a then x ³  = a³ – 3a)
 x +x = x³
1 1
x4   2  x 4  4  529 Alternate:

eYari
x4 x 1 
x²  – 2 x   9 Here, a = 3
1 x² x
x4   529 – 2 1
x4 1 x³   3³ – 3  3
x²   9  2 = 11 x³

x4 
1
x4
1
 527

1  1
snhe
squaring both sides,
2
2
x³ 
1

 18
Ex.3 If x   4 , find the (i) x ²   x²    11
kgei
x x ²  x²  1
Ex.8 If x   4 , Find the value of
1 1 1 x
(ii) x 4  4 x4   2 x²   121
x x4 x² 1
ERna

x³ 
1 x³
Sol.(i) x   4 x4 
1
 121 – 2  119
x
x4 1
1 Sol. x 4
x²  = 4² – 2 = 14 1 1 x
aBryn

x² (If x – = a then x ²   a²  2
x x² 1
1 x³  = 4³ – 3×4 = 64 – 12
(ii) x  4 4 1 2 x³
x and x  4  a ²  2  – 2 )
x = 52
1 Same as:
x²  
Les

= 14 1
x² Ex.6 If x –  4 , Find the value of
1
wa. th

Again squaring both sides x


 If x   3, 4, 5, 6 , then
x
1 4 1
1 (i) x ²  (ii) x 
4
x  4  196 – 2 x² x4 1
x ³ 18, 52, 110, 198
x x³
1
wwM

1 Sol. x– 4
4
x  4  194 x 1
x Ex.9 If x –  4 , then the value of
1 x
1 x²  = 4² + 2 = 18
Ex.4 If x   6 , Find the value of x² 1
x x³ –
1 x³
2 1 4 1 x4  = 18² – 2 = 322
(i) x  (ii) x  x4 1
x2 x4 Sol. x– 4
1 x
8 1 Ex.7 If x   3 , Find the value of
(iii) x  x Cube both sides,
x8
3
1 1  1 3
Sol. x 6 x³  x –   4
x x³  x 

Rakesh Yadav Readers Publication Pvt. Ltd. 107

For More Visit : www.LearnEngineering.in


For More Visit : www.LearnEngineering.in

1 1 1  1
x³ – – 3  x   x –   43 Ex.11 If x+ = 4, x² + = 5²+2 = 27 ......(i)
x³ x x x x2

1 1 1
Put the value of x – 4 Find the value of x5 + x³– = 5³+3×5 = 140 .....(ii)
x x5 x³
 (i) × (ii)
1 Sol. x+ =4
x³ – – 3  4  64 x 1
x³ x5 – = 27×140 – 5 = 3775
1 x5
1 x²+ = 4²–2 = 14 ....(i)
x³ –  64  12 x² 1
x³ Ex.14 If x   3 , Find the value of
1 x

r
1 x³+ = 4³–3×4 = 52 ...(ii)
x³ –  76 x³

Sni
x³ 1
(i) × (ii) x6  .
x6

gv.i
1 1 1
(If x– =a then x³– = a³+3a) x5 + = 14×52 – 4 = 724 1
x x³ x5 Sol. x 3
Alternate: x
1

ridna
Here, a = 4 Ex.12 If x – = 4 find the value of 1
x x3   33 – 3  3 = 18
1 x3
x³– = 4³+3×4 = 64+12 1
x³ x5 – Squaring both side,

eeYa
= 76 x5 2
 3 1  2
1 1  x  3   18
Sol. x– =4  x 
* Same as if x– = 3,4,5,6 x
x
1  1 
1
geisnh x6   2  x 3   3   324
1 x²+ 2 = 4²+2 = 18 x 6
x 
Then x³– = 36, 76, 140, 234 x

......(i) 1
1 x6   324 – 2
Ex.10 If x+ = 5 find the vlaue of 1 x6
Enak

x x³– = 4³+3×4 = 76 .....(ii)



1
1 (i) × (ii) x6   322
5
x+ 5 x6
x
R

 2 1    
x  2  x³ –  = 18×76 1 1
1  x   x ³  x  a ,then x 6  6
Sol. x+ = 5 x x
aryn

x 1 1 = (a³ – 3a)² – 2
x5 – +x – 5 =18×76
1 x x
Les B

x²+ = 52 – 2 = 23 ....(i)
x² 1 1
Put the value of x – =4 Ex.15. If x   3 , Find the value of
x x
1
x³+ = 5³–3×5 = 110 ...(ii) 1
x³ 1  1 x7  .
wa. th

(i) × (ii) x5 – = 18×76 – x –  x7


x5  x
 2 1   3 1  = 1368–4 = 1364 1
Sol. x 3
 x  2   x  3  = 23×110 x
 x   x   1
* If  x –  = a, then
wwM

 x 1
1 1 x2  = 3² – 2 = 7
x5 + +x + 5 = 2530 x2
x x 1
x5 – = (a²+2) (a³+3a)–a
1 x5 1
x4  = 7² – 2 = 47 ....(i)
Put the value of x + =5 x4
x 1
Ex.13 If x– =5 then find the value of
1 x 1
x3  = 3³ – 3 × 3 = 18 ....(ii)
x5 + =2530–5 = 2525 x3
x5 1
x5– Multiply (i) and (ii)
1 x5
* If x+ = a  3 1   4 1 
x
1 x  3  ×x  4 
1 Sol. x – =5  x   x 
Then x5 + = (a²–2)(a³–3a)–a x
x5 = 47 × 18

Rakesh Yadav Readers Publication Pvt. Ltd. 108

For More Visit : www.LearnEngineering.in


For More Visit : www.LearnEngineering.in

1 1 1 1
 x7  x   = 18 × 47 x 4 x2   4²  2  18
x x7 x x2
1 Adding 2 both sides,
1 x2 
Put the value of x  3 = 4² – 2 = 14
x x2 1
x2   2  18  2
Subtract 2 both side x2
1
x7   18  47 – 3 1 2
x7 x2  – 2 = 14 – 2  1
x2  x    20
1  x
x7   843  1
2
x7  x –  = 12 1
 x x  20

ir
1 1 x
 x  a,then x 7  7

v.iSn
x x 1
x–  12 1 3
x x3 
x3
  20  – 3  20
=  a 2 2

– 2  – 2  a 3 – 3a  – a We know that,
1
1  1  1 x3   20 20 – 3 20

dnag
1 x2 – 2 =
x   x –  x3
Ex.16 If x –  4 , Find the value of x  x  x
x
1 1
x2 – x3   17 20 ...(i)
1 = 4 12 x3
x7 – 7 . x2

eYari
x
1 1
1 Ex.18. If x – = 3 find the value of x³ When x – =4
Sol. x– 4 x x
x Then,
1
x2 
1
x2
= 4² + 2 = 18
+

Sol. x –
x3
1
=3
snhe x3 –
1
x3
= 4³ + 3 × 4 = 76 ...(ii)

Multiply (i) and (ii)


1 x
kgei
x 4  4  182 – 2  322 ...(i)
x 1  3 1   3 1 
x² + = 3² + 2 = 11 x  3  ×x – 3 
 x   x 
1 x2
x3 –  43  3  4  76 ...(ii)
ERna

x3 Adding 2 both sides = 17 20 × 76


Multiply (i) and (ii) 1
x² + + 2 = 11+ 2 1
x2 x6 – = 1292 20
 4 1   3 1  x6
x  4  ×x – 3 
aBryn

 x   x   1
2

 x   = 13 2 1
= 322 × 76  x Ex.20 If x   27 , Find the value
x2
1 1
x7 – x  – = 24472 1 1
x x7 x+ = 13 of x 
x x
Les

1  1 Now,
x7 –
wa. th

= 24472 +  x –  1
x7  x Sol. x2   27
1 3
x2
1
x³ +

=  13  –3× 13
x7 – = 24472 + 4 = 24476 Adding 2 both sides,
x7
1 1
wwM

x³ + = – x2   2  27  2
1 1
x –  a , then x 7 – 7 x ³ 13 13 3 13 x2
 x x
1 2
x³ + =  1
=  a 2 2

 2  – 2  a  3a   a
3
x ³ 10 13  x    29
 x
1 1
Ex.17. If x   4 , find the value of Ex.19 If x –  4 , Find the value of 1
x x x  29
x
1 1
x2 – x6 – 2 1
x2 x6 Ex21. If x   31 , find the value
x2
1  x  1  x – 1  1 1
Sol. x2 – =   Sol: x– 4 of x –
x2  x  x x x

Rakesh Yadav Readers Publication Pvt. Ltd. 109

For More Visit : www.LearnEngineering.in


For More Visit : www.LearnEngineering.in

1 1 1
Sol. x2   31 x2   14 x4   2  324
x2 x2 x4
Substract 2 both sides, Again adding (2) both side 2
 2 1 
2 We get,  x  2  = (18)2
 1  x 
 x –   29 1
 x x2   2  14  2
x2 1
x2   18 ....(i)
1 x2
x –  29 1
2
x  2 Again subtracting (2) from both
 x     4
 x side,
4 1 We get,
Ex.22 If x   23 , find the value

r
x4 1
x 4 1
x x2  – 2  18 – 2

Sni
1 x2
of x  Now,
x

gv.i
1
1 x2  – 2  16
4 1 x3   43 – 3  4 x2
Sol. x  4  23 x3
x 2

ridna
 1 2
By adding (2) both side, 3 1  x –    4
x  3  52  x
We get, x
1 1 1
x4  x– 4

eeYa
 2  23  2 Ex.24 If x  =5, find the value of
x4 x x
2
Now,
 2 1  1
 x  2  = (5)2 x2 – 1
 x  x2 x3 –  43  3  4
geisnh x3
1 1
x2   5 ....(i) Sol. x =5 1
x2 x x3 –  76
x3
Adding 2 both side 1 2
x2 5 – 2 = 23 1
Enak

We get, x2 Ex.26 If 2x   4 , Find the


7x
1 Subtract 2 both sides,
x2  252 1
x2 2
value of 49x 
R

1
2
x  2 – 223 – 2 4x 2
 1
2
x
x    7 1
aryn

 x 2 Sol. 2x  4
 1 7x
x – 
  21
 x
Les B

1 7
x  7
x Multiply by both sides
1 2
x –  21
4 1 x 7 1  7
Ex.23 If x   194 , find the value
 2x    4
x4 We know that, 2 7x  2
wa. th

1 1  1  1
3
of x  . x2 – 2 = 
x – x   1
x3 x  x  x 7x   14
2x
1 = 5 21 Squaring both sides,
wwM

Sol. x4   194
x4 1  1 
2
2
Ex.25 If x 4   322 , Find the value  7x    14 
By adding (2) both side, x4  2x 
We get,
1 1 1
of x 3 – 49x 2   2  7x 
1 x3
x 4  4  2  194  2 4x 2 2x
x
1 = 196
Sol. x4   322
1 x4 1
x 4  4  2  196 49x 2   196 – 7
x By adding (2) both side, 4x 2
2
We get,
 2 1  2 1
 x  2   14  1 49x 2   189
 x  x4   2  322  2 4x 2
x4

Rakesh Yadav Readers Publication Pvt. Ltd. 110

For More Visit : www.LearnEngineering.in


For More Visit : www.LearnEngineering.in

Ex.27 If 4x 
1
 5 , Find the
Ex.29If x  y  z  7 , x 2  y2  z 2  19 a 2
– ab  b 2  = 2 ....(i)
3x Find xy + yz + zx = ?
2
a 2
 ab  b 2  = 4( As Given)..(ii)
2 1 Sol. x  y  z  
value of 9x  (ii) – (i)
16x 2 2ab = 2
x ²  y ²  z ²  2  xy  yz  zx 
1 ab = 1
Sol. 4x  5 2 Ex.32 If a4 + a2b2 + b4 = 12,
3x  7  19  2  xy  yz  zx 
a² – ab + b² = 4, Find the value
3 49 – 19 = 2  xy  yz  zx  of ab.
Multiply by both sides Sol. a 4  a 2b 2  b 4 
4
30 = 2  xy  yz  zx 
3 1  3 a 2
 ab  b 2 a 2 – ab  b 2 

ir
15 = xy  yz  zx
 4x    5
4 3x  4

v.iSn
Ex.30 If x² + 2 = 2x, Find x4 – x3 + x2 + 5
12 = 4 × a 2  ab  b 2 
Sol. x² + 2 = 2x
1 15
3x   Squaring both sides,
4x 4
2 2
a 2
 ab  b 2  = 3 ...(i)
Squaring both side, x 2
 2   2x 

dnag
2 2
a 2
– ab  b 2  = 4 ...(ii)
 1   15  4
x  4  4x  4x 2 2
 3x     (i) – (ii)
 4 x   4  2ab = –1
x 4  –4

eYari
1 1 225 x² + 2 = 2x –1
9x 2  2
 2  3x  = ab =
16x 4x 16 x 2  2x – 2 2
1 225 3 x 2  2  x – 1 Ex.33If x = a² + b², y = 2ab , find
9x 2   –

9x 2 
16x
1
2

16x 2

16 2
201
16
x –1=
x2
2
x4 – x3 + x2 + 5
snhe a4  b4
a 2 – ab 2  b 2
(a) x + y (b) x – y
Put the value of of x4 = –4
kgei
1 1 –4 – x2(x – 1)+5 (c) xy (d) 2xy
Ex.28 If a  x  and b  x – , find 1 – x2(x – 1) Sol. x = a² + b²
x x
Put the value of x – 1 Squaring both side,
ERna

the value of a 4 – 2a 2b 2  b 4
x2  x2 x ²  a ²  b ² 
2
(a) 10 (b) 4 (c) 16 (d) 8 1–
2
2
a 4 – 2a 2b 2  b 4 = a – b 
2 2
Sol. x ²  a 4  b 4  2a ²b ² ...(i)
x4 4
1– =1+
aBryn

2 2 2 y  2ab
=  a  b a – b   = 3
a + b = 2x .....(i) y ²  2a ²b ² ...(ii)
Ex .31 If a 4  a 2b 2  b 4  8 and
From (i) and (ii)
2 a 2  ab  b 2  4 find the value of
a –b  ....(ii) x2  a4  b4  y2
Les

x ab .
From (i) and (ii)
wa. th

Sol. a 4  a 2b 2  b 4  8 x2 – y2  a 4  b4
4 2 2 4
a – 2a b  b a 4  b 4  8 – a 2b 2 ....(i) Now,
2 2 2
a  b  4 – ab Put the value of
 2
=  2x   = 16 Squaring both sides,
x a4  b4
wwM


2 2
Alternate: a 2
b  = (4 – ab)2 a 2 – ab 2  b 2
Let x = 1 a4 + b4 + 2a2b2 = 16+a2b2– 8ab  x – y   a ²  b ² – 2 ab 
Then, From equation (i),  
a 2, b = 0 8–a²b²+2a2b2 = 16+a2b2– 8ab
8 = 8ab x 2 – y2
Put the value of a and b in equa- ab = 1 =
x –y
tion, Alternate:-
a 4 – 2a 2b 2  b 4 We use formula,  x  y  x – y 
a 4  a 2b 2  b 4  = x  y 
Then, x – y 
= 24 – 2  22  0  0 4 = 16 a 2
 ab  b 2
a 2
– ab  b 2
 Ex.34 If x + y =1, x4 + y4 = –1, Find
Option c is correct. 8 = 4 × a – ab  b  x² y²– 2xy
2 2

Rakesh Yadav Readers Publication Pvt. Ltd. 111

For More Visit : www.LearnEngineering.in


For More Visit : www.LearnEngineering.in

Sol. x+y=1 Sol. (x + y)³ = x³ + y³ + 3xy(x + y)  (m – 3)² + (m – 3) = 5


Squaring both side  x³ + y³ = 0 (As given)  m² + 9 – 6m + m – 3 = 5
(x + y)2 = (1)² (x + y)³ = 3xy(x+ y)  m² – 5m + 1 = 0
x2 + y2 + 2xy = 1 (x + y)² = 3xy  m² + 1 = 5m
x² + y.² = 1 – 2xy
x+y= 3xy 1
Again Squaring both sides, m+ =5

(x² + y²)2 = (1 – 2xy)2 Ex.38 If x4 + y4 = x²y² find x6 + y6. m
x4+y4+2x2y2 = 1+ 4x2y2–4xy Sol. x6 + y6 = (x²)³ + (y²)³ Now,
Put the value of = (x² + y²)(x4 + y4 –x²y²)
x4 + y4 = –1 1
= (x² + y²) (x²y² – x²y²) m³ + = 5³ – 3 × 5
–1 + 2x² y² = 1+ 4x2y2–4xy m³
=0

r
–1 = 1 + 2x² y² – 4xy
–2 = 2(x² y² – 2xy) Ex.39 If (x –a)(x – b) = 1 and a – b + 5 1

Sni
m³ + = 110
x² y² – 2xy = –1 1 m³
= 0 find (x – a)³ – x – a 3 = ?

gv.i
1 1 1
   2,   Put the value of m = x + 3
Ex.35If x  y  z  3 ,
x y z
(a) 125 (b) –125
x ²  y ²  z ²  6 , Find xyz = ? Then,
(c) 0 (d) 140

ridna
1 1 1 Sol. (x – a)(x – b) = 1 1
Sol.   2 (x + 3)³ +  x  3  ³ = 110
x y z 1
 (x – b) =  x – a 

eeYa
xy  yz  zx  2xyz Ex.41If x(x –3) = –1 find the value of
x³(x³ – 18)
2 (x – a) (x – b) = 1 ...(i)
x  y  z   Sol. x(x –3) = –1
 a – b + 5 = 0
x ²  y ²  z ²  2  xy  yz  zx  –b=–a–5 –1
geisnh x–3=
9 = 6 + 2 (2xyz) Put the value (– b) In equation (i) x
3 = 4xyz (x – a) (x – a – 5) = 1
1
let (x – a) = M x+ =3
3 x
xyz = M(M – 5) = 1
Enak

4
1 1
p q r M – 5= x³ + = 3³ – 3 ×3 = 18
Ex.36 If x  y  z  1 & M x3
R

1 –1
x y z M– =5 x³ – 18 = ......(i)
   0 Find M x3
aryn

p q r
Now, Now,
p2 q 2 r 2
Les B

1 x³(x³ – 18)
  ?
x2 y2 z 2 M³ – = 5³ + 3 × 5 From Equation (i)
M3
p q r –1
1 = x³× =–1
Sol. Let a = , b = y and c = M³ – = 140 x3
x z M3
wa. th

Then, a + b + c = 1 Put the value of M = x – a x³(x³ – 18) = – 1


So, Ex.42 If (a + b)² = 21 + c², (b + c)² = 32
1 1 1
&   0 + a² and (c + a)² = 28 + b², find a
a b c 1 +b+c=?
wwM

 ab + bc + ca = 0 (x – a)³ – x – a 3 = 140
  Sol. (a + b)² – c² = 21
2
 a  b  c    (a + b+ c)(a + b – c) = 21 ...(i)
a ²  b ²  c ²  2 ab  bc  ca  Ex.40 If x² + x = 5 find the value of (b + c)² – a² = 32
1 = a² + b² + c² + 2(0)  (b + c + a) (b + c – a) = 32 ..(ii)
1
a² + b² + c² = 1 (x + 3)³ + x  3 ³ (c + a)² – b² = 28
   (c +a + b)(c + a – b) = 28..(iii)
p2 q 2 r 2
Hence,   1 Sol. Let (x + 3) = m Adding all three equations:-
x2 y2 z 2
Ex.37 If x³ + y³ = 0 find x + y = ? x+3=m  (a +b + c) [(a + b + c) + (b + c + a
x=m–3 ) + (c + a – b)] = 81
(a) 3xy (b) 2xy
Put the value of 'x'  (a + b + c)² = 81
(c) 3xy (d) 4xy x² + x = 5  a+b+c=9

Rakesh Yadav Readers Publication Pvt. Ltd. 112

For More Visit : www.LearnEngineering.in


For More Visit : www.LearnEngineering.in

3 3 3
a=b=c=4 Ex.50 If a2 + b2 = c2, Find the value of
a + b + c - 3abc Then 12 is possible a + b + c
1 2 2 2 a 6  b6  c 6
 (a + b + c)[(a - b) ] + (b - c) + (c - a) Ex.46 If a 3 + b 3 + c 3 - 3abc = 0,
2 a + b + c  0 and a, b & c a 2b2c 2
2 2 2
= (a + b + c) (a + b + c - ab - bc - ca) are natural number find the Sol. a2 + b2 = c2
(i) If (a + b + c) = 0 possible value of a × b × c Cube both side
3 3 3
then a + b + c - 3abc = 0 (a) 4 (b) 8 (c) 5 (d) 12 (a2 + b2)3 = (c2)3
3 3
a + b + c = 3abc
3
Sol.(b) We know that in this condition a a6 + b6 + 3a2 b2 (a²+ b2) = c6
3 3 3
(ii) If a + b + c - 3abc = 0 = b = c and given a, b and c are a6 + b6 + 3a2 b2 c2 = c6
a, b and c are distinct no natural no. we take option (b) a6 + b6 - c6 = -3a2 b2 c2
then. a + b + c = 0 because 8 = 2 × 2 × 2 A.T.Q.
3 3 3
(iii) a + b + c - 3abc = 0 We can say that 8 is possible

ir
a, b and c all are +ve integer no value of a × b × c a 6  b6  c 6

v.iSn
then a= b = c . Ex.47 Find the value of a 2b2c 2
(iv) a2 + b2 +c2 - ab - bc - ca = 0
2 2 2
a + b + c = ab + bc + ca (x 2 - y 2 ) 3 + (y 2 - z 2 ) 3 + (z 2 - x 2 ) 3 -3a 2 b 2 c 2
3 3 3 =
then a= b = c x  y   y  z   z  x  a 2b2c 2

dnag
Ex.43 If a + b + c = 0, then the value Sol. Let, a = x2 – y2 = –3
of a³ + b³ + c³ is: b = y2 – z2 Ex.51 If a1/3 + b1/3 = c1/3
(a) 0 (b) abc c = z2 – x2 Which statement is true
(c) 3abc a+b+c=0 (a) a3 + b3 - c3 = 3abc

eYari
(d) None of these Then, a3 + b3 + c3 = 3abc (b) a3 + b3 - c3 + 3abc = 0
Sol.  a³ + b³ ++ c³ – 3abc = (a + b c) Thus, p = x – y (c) (a + b - c)3 - 27abc = 0
(a² + b² + c² – ab – bc – ac) q=y–z (d) (a + b - c)3 + 27abc = 0
or a³ + b³ + c³ – 3abc = 0 r=z–x Sol.(d) (a)1/3 + (b)1/3 + (-c)1/3 = 0
 a³ + b³ + c³ = 3abc
Hence (c) is the correct option.
Ex.44 If a3 + b3 + c3 - 3abc = 0 and a
p+q+r=0 snhe
Then, p3 + q3 + r3 = 3pqr
A.T.Q.
If (a + b + c) = 0
Then a3 + b3 + c3 = 3abc
(a) + (b) + (-c) = 3(a)1/3 (b)1/3 (-c)1/3
kgei
+b+c  0
(x 2 - y 2 ) 3 + (y 2 - z 2 ) 3 + (z 2 - x 2 ) 3 a + b -c = -3a1/3 b1/3 c1/3
Which statement is true 3 3 3
Cube both side
(a) a>b>c (b) a = b = c x  y   y  z   z  x  (a + b - c)3 = (-3a1/3 b1/3 c1/3 )3
ERna

(c) a > b < c (d) a < b < c


Sol.(b) We know that, 3(x 2 - y 2 ) (y 2 - z 2 ) (z 2 - x 2 ) (a + b - c)3 = - 27abc
= 3 x  y y  z z  x   (a + b – c)3 + 27abc = 0
a3 + b 3 + c 3 - 3abc
= (a + b + c)[(a - b)2] + (b - c)2+ (c  (a2 - b2) = (a + b) (a - b) Ex.52 If x + y + z = 2s
- a)2 Find the value of (s - x)3 + (s -
aBryn

 a3 + b 3 + c 3 - 3abc = 0 ( As (x + y) (x - y) (y + z) (y - z) (z + x) (z - x) y) 3 + 3 (s - x) (s - y) z
given) x  yy  zz  x (a) y3 (b) x3 (c) z3 (d) 0
When a + b + c  0 Sol.(c) x + y + z = 2s
= (x + y) (y + z) (z + x)
Then (a - b)2 + (b - c)2 + (c - a)2 x+y+z=s+s
Ex.48 If a + b + c = 0 Find the value
Les

=0 s-x+s-y-z=0
of a 3 + b 3 + c 3 + 3 a b c
(a - b)2 = 0
wa. th

(a) 0 (b) 1 (s - x)+ (s - y) + (-z) = 0


(a - b) = 0
(c) abc (d) 6abc (s - x)3 + (s - y)3 + (-z)3 = 3(s - x) (s - y) (-z)
Then a = b
Sol.(d) (a + b + c) = 0 (s - x)3 + (s - y)3 - z3 = -3(s - x) (s - y) z
Same b = c
c=a Then a 3 + b 3 + c 3 = 3 a b c (s - x)3 + (s - y)3 + 3 (s - x) (s - y) z
wwM

So, a 3 + b 3 + c 3 + 3 a b c = z3
So. (a = b = c)
= 3abc + 3abc Ex.53 Find the value of
Ex.45 If a 3 + b 3 + c 3 - 3abc = 0 , a
= 6abc
+ b + c  0 and a, b & c are 3
2  3333  3343  3  3332  334
natural numb er f ind the Ex.49 If x = 1.235
possible value of a + b + c y = 3.422 Sol. 3
2  3333  3343  3  3332  334
(a) 4 (b) 8 (c) 5 (d) 12 z = 4.377 After describing
Sol.(d) We know that in this condition a Find x3 + y3 - z3 + 3xyz
= b = c and given a, b and c are = 3
3333  3333  3343  3  333  333  334
Sol. x + y = z
natural no. we take option (d)
because 12 is divide 3 equal (x) + (y) + (-z) = 0
(x)3 + (y)3 + (–z)3 = 3(x)(y)(–z) a3 + b3 + c3 - 3abc
natural part 1 2 2 2
x3 + y3 - z3 = –3xyz  (a + b + c)[(a - b) ] + (b - c) + (c - a)
12 2
4  x3 + y3 -z3 + 3xyz = 0
3

Rakesh Yadav Readers Publication Pvt. Ltd. 113

For More Visit : www.LearnEngineering.in


For More Visit : www.LearnEngineering.in

Ex.57 Find (a - b)3 + (b - c)3 + (c - a)3 =? Sol. a2 + b2 + c2 - ab - bc - ca


1
(333  333  334 )[(333 – 333)² Sol.  (a - b) + (b - c) + (c - a) = 0 1 
2 =
2 2 2
a  b  b  c  c  a  
=  (a - b)3 + (b - c)3 + (c - a)3 2 
 
(333 – 334 )2  (334 – 333 )2 ] = 3 (a - b) (b - c) (c - a)
1 2 2 2
Ex.58 The value of a³ + b³ + c³ – 3abc, 99  97  97  96  96  99 


1 2 
= 3
10000  1  1 Where a = 87, b = –126 and c =
2 39 is: 1 2 2 2

(a) 0 (b) 1259  2  1  3 


1 2  
= 1000  2 = 10
3
(c) –48
2 1
(d) None of these 4  1  9 7
2
Ex.54 If a = 20, b = 25, c = 15, Find 

=
Sol.  a + b + c = 0
a 3  b3  c 3  3abc Ex.62 If a = 556, b = 558 and c =561

r
 a³ + b³ + c³ – 3abc = 0
a  b2  c 2  ab  bc  ca
2 Find the value of

Sni
Thus (a) is the correct option.
Ex.59 Find the value of a³ + b³ + c³ – a2 + b2 + c2 - ab - bc - ca
Sol.
a3 + b3 + c 3 - 3abc = (a + b + c) Sol. a2 + b2 + c2 - ab - bc - ca

gv.i
2 2 2
3abc If a + b + c = 12 and ab +
(a + b + c - ab - bc - ca) bc + ac =47. 1  2 2 2
3 3 3 Sol. a + b + c = 12 = a  b  b  c  c  a  
a  b  c  3abc 2  
 (a + b + c)² = a² + b² + c² +

ridna
a 2  b2  c 2  ab  bc  ca 2(ab +bc + ac) = 144 1 2 2 2
A.T.Q.  a² + b² + c² + 2 × 47 = 144  556 558 558 561 561 556 
2  
 a² + b² + c² = 50
(a  b  c) a 2  b2  c 2  ab  bc  ca

eeYa
=   Now, since a³ + b³ + c³ – 3ab 1 1
4  9  25   38 = 19
a 2  b2  c 2  ab  bc  ca = (a + b + c) (a² + b² + c² – ab –bc 2

2
=a+b+c – ac) Ex.63If a2 = b+c, b2 = c+a
Put the value Then, c2 = a+b
= 20 + 25 + 15 = 60
geisnh
a³ + b³ + c³ – 3abc
Ex.55 If a + b + c = 3 and a2 + b2 + c2 = 6 , = 12(50 – 47) = 12×3 = 36 1 1 1
+ +
Ex.60 I f a = 9 9 7 , b = 9 9 9 a n d c 1 a 1 b 1 c
1 1 1 = 9 9 6 f in d th e v alu e of a 3
   1 , Find abc He re a, b and c non ze ro
a b c + b3 + c3 - 3abc number
Enak

Sol. Sol. a2 = b+c


1 1 1
Sol.   1 Divide by 3 both sides
a b c
a3 + b3 + c3 - 3abc
R

1 a2 b  c
ab  bc  ca 2 2
 (a + b + c)[(a - b) ] + (b - c) + (c - a)
2
=
1 a a
abc 2
aryn

ab + bc + ca = abc Then,
 999  997 2  bc
Les B

2 2
(a + b + c) = a + b + c + 2(ab+bc + ca)
2 2
   a=
1  2  a
 999  997  996 997  996 
(a + b + c)2 = a2 + b2 + c2 + 2abc 2   ca
 996  999 2  Thus, b =
(3)2 = 6 + 2abc 
   b
9 - 6 = 2abc
wa. th

1 ab
3   2992 14 c=
abc = 2 c
2
= 2992 × 7 = 20944 Now put the value of a, b and c
Ex.56 If a + b + c = 15
wwM

TYPE IV 1 1 1
and a2 + b2 + c2 = 83
bc + ca + ab
find a3 + b3 + c3 - 3abc a2 + b2 + c2 - ab - bc - ca
 1 1 1
Sol.
* a b c
1  2 2 2 a b c
= a  b  b  c  c  a   + +
2 
2 2 2 2 
(a + b + c) = a + b + c + 2(ab+bc + ca)  a  b c b c a a  b c
(15)2 = 83 + 2 (ab+bc+ca) When, a  b c
225 - 83 = 2 (ab + bc + ca) a² + b² + c² – ab – bc – ca = 0  =1
a  b c
ab + bc + ca = 71 a² + b² + c² = ab + bc + ca
Alternate:
a3 + b3 + c3 - 3abc = (a + b + c) Then a = b = c
We put the value
[a2 + b2 + c2 -(ab + bc + ca)] Ex.61If a = 99, b = 97, and c = 96 a=b=c=2
Find the value of
a3 + b3 + c3 - 3abc = 15(83-71) a2 = b+c = (2)2 = 2+2
a2 + b2 + c2 - ab - bc - ca
= 15 × 12 = 180 4=4

Rakesh Yadav Readers Publication Pvt. Ltd. 114

For More Visit : www.LearnEngineering.in


For More Visit : www.LearnEngineering.in

1 1 1 Thus, c+a = –b Now,


Then + + c2+a2–b2 = –2ac .....(III)
1 2 1 2 1 2 (1²)  (1²)  (1²)  (–2²)  (–2²)  (1²)
Put the value (1)4  (1)4  (–2)4
1 1 1
+ + =1 1 1 1
3 3 3 2
2 2 + 2 2 2 + 2 1 4  4 9 1
a b –c b c –a c  a 2 – b2 = =
x ²  y²  z ² 1  1  16 18 2
Ex.64 If xy  yz  zx  1 , then find    Ex.68If pq+qr+rp = 0, Find the value
 + +
–2ab –2bc –2ab
x y y z z  x p2 q2 r2
the value of z  x  y + +
a  b c O p2 – qr q 2 – pr r 2 – pq
= =0
–2abc –2abc Sol. pq + pr + rp = 0

ir
x ²  y²  z ² Alternate:- pq + rp = – qr
1

v.iSn
Sol. xy  yz  zx Same,pq + qr = – pr
We choose the value which we
follow the condition. qr + rp = – pq
x² + y² + z² = xy + yz + zx
If a+b+c = 0
In this condition x = y = z p2 q2 r2
a = 1, b = 1, c = –2 + +

dnag
Put the value of x = y = z = 1 p2 – qr q 2  pr r 2 – pq
Put the value a,b,c
x y y z z x 1 1 p2 q2
Then,   1
z x y = 2 + 2 2 2+ 2  2 + 2
a  b – c b c –a c a2 –b2
2 2
p  pq  rp q  pq  qr

eYari
1 1 1  1 1 1 1 1 r2
=   = +
1 1 1 2 2 2 2 2 2 + 2
1 1 – –2 1  –2 – 1 r  qr  rp
= 2+2+2=6
E x. 65I f
find
x ²  y ²  z ²  xy  yz  zx ,
the v alue of
+ 2 snhe
1
2
 –2  1 – 1
2 =
p2
+
q2
p p  q  r q p  q  r

r2
x y z 1 1 1
kgei
  = + + +
yz z x yx 1 1 – 4 1 4 – 1 4  1 – 1 r p  q  r

Sol. x ²  y ²  z ²  xy  yz  zx 1 1 1 p q r
ERna

= –
+ + =0 = + +
Here, 2 4 4 pq  r pq  r pq  r
x=y=z Ex.67 If x + y + z = 0 then the value
p q r
Let put the value of x = y = z =1 x ²y²  y² z ²  z ² x ² = =1
p q r
of is:
aBryn

Now, x 4  y4  z 4 Alternate:
x y z (a) 0 (b) 1/2 Put the value p = –1, q = 2
  (c) 1 (d) 2 r=2
yz z x yx
Sol. (x + y + z) =0 So,
 x² + y² + z² + 2(xy + yz + zx) = 0 pq  qr  rp
Les

1 1 1
=  
1 1 1  1 1  1  x² + y² + z² = –2(xy + yz + zx) = –1×2+2×2+2×–1 = 0
wa. th

 (x² + y² + z²)² = 4(xy + yz + zx)²


1 1 1 3 p2 q2 r2
=   =  x4 + y4 + z4 + 2(x²y² + y²z² + z²x²) = + 2
2 + 2
2 2 2 2 p – pq a – pr r – pq
= 4[x²y² + y²z² + z²x² + 2xyz (x + y
Ex.66 If a + b + c = 0, Find the value of + z)] (–1)2 (2)2
wwM

 x4 + y4 + z4 = 2(x²y² + y²z² + z²x²) = 2


 2 
1 1 1 (–1) – (2  2) (2) – (–1  2)
+ + [ (x + y + z) = 0]
a 2  b2 – c 2 b2  c 2 – a 2 c 2  a 2 – b2
x 2 y 2  y 2z 2  z 2 x 2 1 (2)2
Sol. a+b+c = 0  = 2
x4  y4  z4 2 (2) – (–1  2)
a+b = –c
Squaring Both side Hence (b) is the correct option. 1 4 4
=  
(a+b)2 = (–c)2 Alternate: 1– 4 4 2 4 2
a2+b2+2ab = c2 x+y+z=0 –1 4 4
Then, Let x = 1, y = 1, z = – 2 =  
a2+b2–c2 = –2ab ....(I) 3 6 6
Same, b+c = –a x 2 y 2  y 2z 2  z 2 x 2 –2  4  4 6
= = =1
b2+c2–a2 = –2bc ...(II) x4  y4  z4 6 6

Rakesh Yadav Readers Publication Pvt. Ltd. 115

For More Visit : www.LearnEngineering.in


For More Visit : www.LearnEngineering.in

 2 a + b + c + 2 b +a + c + 2 c + a + b
1 1
Ex.69If p×q×r = 1 Find  4 a+ 4 b + 4 c = 4 ( a+ b +c ) = 1–y
1  p  q –1 L.H.S = R.H.S z

1 1 So, x = (a+b+c)2 1
+ + Alternate: z= ....(ii)
1  q  r –1 1  r  p –1 1– y
Let, a = b = c =1
Then From (i) & (ii)
1 1 1
Sol. + + x –1 x –1 x –1 1 y
1 1  q  r 1 1 1 1
1 p  1  + + = 4×3 (i) z+ = +
q pq p 2 2 2 x 1 – y y –1

1 x – 1  3 =4×3 1 y 1– y
1
(Pqr = 1, r = pq ) = – 
2

r
1– y 1– y 1– y
x–1= 8

Sni
q 1 pq x=9 1 1 1
+ + (ii) x y  z 
q  pq  1 1 q  pq pq  1 q Option (B) y z x

gv.i
x = (a+b+c)2
pq  1  q = (1+1+1)2 = 1+1+1=3
q  pq  1
=1 = (3)2 = 9 1 1 1

ridna
So, It is proofed So, x  y  z    =3
Alternate x y z
p=1=q=r  x = (a+b+c)2
Alternate:-
Then x  a2 x  b2 x  c2

eeYa
Ex.71 If 2 2 + 2 2 + 1
1 1 1 b c c a a 2  b2 (i) x+ =1
= + + = 3 Find the value of x y
1 1 1 1 1 1 1 1 1
(a) a2+b2+c2
1 1 1 (b) a2+b2+c2–ab–bc–ca 1
= + + =1 Let, x =
3 3 3
geisnh
(c) (a+b+c)2 2
x  a 2 x  b2 x  c 2 (d) a2+b2+c2+ab+bc+ca Then y = 2
Ex.70 If + + Sol. We take option (A) a2+b2+c2
bc ca ab 1 1
= 4(a+b+c) a2  b2  c2 – a2 a 2  b2  c 2 – b2 So, + =1
+ 2 2
Enak

Find the value of x b2  c2 c2  a 2


(a) (a2+b2+c2) 1
(b) (a+b+c)2 a 2  b2  c 2 – c 2 Now, y + =
+ z
a 2  b2
R

(c) (a2+b2+c2–ab–bc–ca)
(d) (ab+bc+ca) If y = 2
b2  c 2 c2  a 2 a 2  b2
Sol. We take option (B)  2 + + Then z = – 1
aryn

b  c 2 c2  a 2 a 2  b2
Then x = (a+b+c)2 1
1+1+1 = 3
2 2 2+ = 2–1 = 1
Les B

2 2
a  b  c – a + a  b  c – b L.H.S = R.H.S –1
b  c a  c Then, (x = a2+b2+c2)
1
2 1 1 Now, z 
 a  b  c – c 2 Ex.72 If x + y = 1, y + =1 x
+ z
wa. th

a  b 2 2 Put the value z and x


a –b = (a+b) (a–b) 1
Find the value of (i) z + 1
 a  b  c – a a  b  c  a x –1+ = -1+2 = 1
 12
b  c (ii) x  y  z 
1 1 1
 
wwM

x y z a b c
a  b  c – ba  b  c  b Ex.73 If + + =1
+ bc ca a b
a  c 1
Sol. x + y = 1 a2 b2 c2
a  b  c – ca  b  c  c Find + +
+
a  b 1
bc ca a b
b  c2a  b  c x = 1– Sol. Divide and Multiply a + b + c
 + y
 b  c a b c
y –1 Now, + +
a  c2b  a  c x= ....(i) bc ca a b
+ y
a  c 1  a  b  c
a  b2c  a  b 1
y + =1
=
a  b  c
a  b z

Rakesh Yadav Readers Publication Pvt. Ltd. 116

For More Visit : www.LearnEngineering.in


For More Visit : www.LearnEngineering.in

 a b c  1 1 1 1
 
   
 (a+b+c) (c) = + (d) N.O.T Sol. a  2
 b  c c  a a  b c a b a
= a+b+c bc ac a –b Then, a = 1
Sol. + + =1 Put the value of a,
a a  b  c  b a  b  c a b c
 + We adding + 1 in this equation 1
bc ca  1100 
1100
b–c ac a–b
c a  b  c –1+ –1+ + 1 = 1–1
+ = a+b+c a b c 
ab  1  1+1=2
1
b –c – a ac –b a – bc
a 2  a b  c b2  b a  c + + =0
 + + a b c 1

ir
bc ca Ex.76 If a   2 , find the value of
a

v.iSn
– a – b  c a – b – c + a – b  c
c 2  c a  b +
= a+b+c a b c 50 1 49 1
ab (i) a  (ii) a –
=0 a 50 a 49
a2 b2 c2 –1 1 1

dnag
 +a+ +b+ +c + + =0 (iii) a 3  a 2  a  1
bc ca ab a b c
=a+b+c 1 1 1
Sol. a  2
c 1
2 2 2 = –
a b c b a a

eYari
Then, + + =0
bc ca a b Alternate: a=1
Alternate: Let, b = 1
1 
c=1 (i) a 50  50 =
150  50
a b c a 1
Then,
+
bc ca a b
+ =1

We break equation small step


Let, a = 0, b = 1
snhe
Put the value of b and c
1 –1 a 1 a –1
+ + =1
= 1

1
=2

a 1 1
kgei
Put the value of a and b 1 
a+1 + a–1 = 1 (ii) a 49 – 49 =
149 – 49
Then, a 1
0 1 c 1
a= 
ERna

+ + =1 2 = 1– =0
1 c c  0 0 1 1
1 1 1 1 (iii)
+ c = 1, Option (A) = – a3  a2  a  1
c b a c
1³ + 1² + 1 + 1 = 4
aBryn

1+ c² = c 1 1
c² = c – 1 = – =2–1=1 
12 1 Ex.77 If p   5 , f ind the
Again put the value a, b p–3
So, L.H.S = R.H.S value of :-
a2 b2 c2
+ + 1 1 1
Les

100
bc ca a b = – 100   
b a c (i)  p – 3  
wa. th

1 c2  p – 3
 0+ + TYPE – V
c  0 0 1 p² – 3p  4
(ii)
1 2 1
When a + = 2 then a = 1 
 +c a p 5
c Sol.(i)
wwM

p–3
Put the value of c² 1
Then, Proof a  2 Subtract 3 both sides,
a
1
1 a 2  1  2a  p – 3  5–3
So, +c –1 = 1–1 = 0  p – 3
c a² – 2a + 1 = 0
b – c a c a – b ( a –1)² = 0 1
Ex.74 If + + = 1 And So, a = 1  p – 3  2
a b c  p – 3
a-b+c  0 which statement is 1 Let a = (P – 3)
Ex.75 If a   2 , find the value of
true a Then,
1 1 1 1 1 1 1
a100  100 1
(a) = – (b) = – a a+ =2
b a c a b c a

Rakesh Yadav Readers Publication Pvt. Ltd. 117

For More Visit : www.LearnEngineering.in


For More Visit : www.LearnEngineering.in

 a = 1 = (P – 3) (iii) a4  a3  a2  a  1 1
Now, Proof a + =1
(–1)4 + (–1)3 + (–1)² + (–1) + (1) a
p–3=1
=1 Squaring both sides
p=4
100 1 1
100    Ex.80 If p + p–3 = 1, Find the value a² + = 1² – 2 = – 1
 p – 3   a²
 p – 3 of 1
1 a² + = –1
1100  100 = 2 1 a²
1 (i) (P – 3)42 + 42
Multiply a both sides
 p–3
(ii) p² – 3p  4  1
= a  a ²  a ²  –1× a

r
1
4² – 3×4 + 4 (ii) (P – 3)101 – p–3 101  
 

Sni
=8
1
(iii) P³ + 4P² + 5P + 1 a³ + =–a

gv.i
1 a
 When a   –2 , then a = –1
a 1
Sol. p + =1 1
p–3 a³ + +a=0
1 a

ridna
Ex.78 If a   –2 , Find the value Subtract 3 both sides
a a³ + 1 = 0
1 or
1 (P – 3) + p–3 = 1 – 3
of a 200    a³ = –1

eeYa
a 200
1
1 1 Ex.81 If a + = 1, Find the value
Sol. a   –2 a
(P – 3) +  p–3 = – 2
a 1
of a15 +
Now, a = –1 So,
geisnh a15
1 (P – 3) = – 1
a 200  1
P=–1+3=2 Sol. a+ =1
a 200 a
P=2
Then,
1
Enak
200
 –1  1 a³ = – 1
= 200
 –1 (i) (P – 3)42 + p–3 42
  1
1
= 1+1=2 a15 + 15 = (a³)
5
+ a³ 5
a  
R

1
1 (– 1)42 + 42 1
Ex.79 If a   –2 , find the value of  –1
a (– 1)5 + –1 5 = (– 1) + (– 1)
aryn

=1+1=2
 
99 1 32 1 =–2
(i) a  (ii) a 
Les B

a 99 a 31 1 Ex.82 If a² – a + 1 = 0, Find the value


(ii) (P – 3)101 – p–3 101 of
(iii) a 4  a 3  a 2  a  1
 
1 1
1 (i) a36 + (ii) a37 +
1
(–1) 101
– a 36 a 37
wa. th

Sol. a   –2  –1
101
a 1
So, a = –1 = (–1) – (–1) (iii) a38 +
a 38
= –1 + 1 = 0 Sol. a² – a + 1 = 0
1
(i) a 99  (iii) P³ + 4P² + 5P + 1 or
wwM

a 99 Put the value of P = 2


1
99 1 2³ + 4 × 2² + 5 × 2 + 1 a+ =1
 –1  a
99 = 8 + 16 + 10 + 1 = 35
 –1 Then,
TYPE – VI a³ = – 1
(–1) + (–1) = –2
1 1
1 (A). When a + = 1, (i) a36 + 36
(ii) a 32  a a
a 31 1
or
(a³)12 + a 3 12
 –1
32

1 a² – a + 1 = 0  
31
 –1 Then a³ = – 1
1
or = (–1)12 + –1 12 = 1 + 1 = 2
1–1=0 a³ + 1 = 0  

Rakesh Yadav Readers Publication Pvt. Ltd. 118

For More Visit : www.LearnEngineering.in


For More Visit : www.LearnEngineering.in

1 Then,
1
(ii) a37 + 1 = a² +
a 37 a²
a+ =1
Break the power multiple of 3 a
We know that a³ = –1 1
1 =a+ =–1
36
a × a + 36 Put the value a a
a a
3 Then,
1 x 
  =–1 1
= (–1)12 × a + –1 12  a 3 = a² + = (–1)² – 2
  a²
x³ =–1
1 =–1
=a+ = 1 (As Given) 27
a 1

ir
x³ = – 27  a50 + =–1
So, a 50

v.iSn
1
1 (B). When a + = –1
37
a + 37 = 1 a 1
a (iii) a51 +
or a 51
1 a² + a + 1 = 0

dnag
(iii) a38 + 38 1
a Then, (a3)17 + a ³ 17
Break the power multiple of 3 a³ = 1
1 1 1

eYari
a36 × a² + Proof :- a + =–1 = 117 + = 1+1=2
36
a  a² a 117
1 1 Ex.86 If a² + a + 1 = 0 find the value
= (–1)12 × a² + –1 12  a ² a² + = (–1)² – 2 = – 1 of (a² – a + 1) (a + 1– a² )
  a²

= a² +
1
a2

a a ² 

1 
snhe
Multiply a both sides.

a²  = – 1 × a

Sol. a² + a + 1 = 0
or

a+
1
=–1
a
kgei
1 We know that
=a+ = 1 (As Given) 1
a a³ + =–a a³ = 1
Then, a a² + 1 = – a ........(i)
ERna

a + 1 = – a²..........(ii)
1 1 (a² – a + 1) (a + 1– a² )
a² + = 1² –2 = – 1 a³ + a + =0
a² a From Equation (i) and (ii)
a³ – 1 = 0 (– a – a) (–a² – a²)
1 = – 2a × – 2a²
aBryn

a38 + =–1 a³ = 1
a 38 = 4a³ = 4 × 1 = 4
1 Ex.87 If a² + a + 1 = 0, find the value
1 Ex.85 If a + = – 1 Find the value of of a5 + a4 + 1
Ex.83 If a + = 1find the value of a
a Sol. a² + a + 1 = 0
a100 + a99 + a98 + a97 + a96 + a95 1 1 or
Les

(i) a48 + (ii) a50 + 1


1 a 48 a 50
a+ =–1
wa. th

Sol. a+ =1 a
a 1 Then,
Then, (iii) a51 +
a 51 a³ = 1
a³ + 1 = 0 a5 + a4 + 1
a100 + a99 + a98 + a97 + a96 + a95 1 a³ × a² + a³ × a + 1
wwM

= a100 +a97 + a99 + a96+ a98+ a95 Sol. (i) a48 + Put the value of a³ = 1
a 48
= a97 (a³ + 1) + a96 (a³ + 1) + a95 a² + a + 1 = 0 (As Given)
(a³ + 1) 1 1
116 + =1+1=2 Ex.88 If x1/4 + = 1 find the value
= a97 × 0 + a96 × 0 + a95 = 0 116 x 1/4
Note: When difference of the power is
3 then the value of both terms is 1 1
(ii) a50 + of x252 +
=0 a 50 x 252
x 3 1 1
Ex.84 If + = 1, Find the value a48 × a² + Sol. x1/4 + =1
3 x a 48  a ² x 1/4
of x³ Squaring both sides
x 1 1
Sol. Let a = = 116 × a² + x1/2 + 1/2 = 1² – 2 = – 1
3 116  a ² x

Rakesh Yadav Readers Publication Pvt. Ltd. 119

For More Visit : www.LearnEngineering.in


For More Visit : www.LearnEngineering.in

Again Squaring both side Then,


96
1
1 (ii) x + 96
x 1 3
x+
x
= (–1)² – 2 = – 1 x3 +

=  3 – 3× 3
Then, 1
(x6)16 + 6 16 1
x+
1
=–1
x  x³ + =3 3 –3 3 =0
x x³
x³ = 1 1 So,
Now, =( – 1)16 + –1 16
  1
1 = 1+1=2 x93 + =0
x 252
+ x 93
x 252
1 1

r
1 Ex.90 If x + = 3 find the value Ex.91 If x + = 3 find the value of
84 x x
(x³) + x³

Sni
84
  of (i) x6 + 5

gv.i
1 1 1 (ii) x102 + x96 + x101 + x95 + x100 ,
(1)84 + 1 84 (i) x92 + (ii) x93 + + x94
  x 92 x 93
=1+1=2 1
1

ridna
Sol. (i) x92 + Sol. x + = 3
TYPE – VII x 92 x
Break the power multiple of 6 Then,
1 x6 + 1 = 0
When x + =

eeYa
x 3 1
x90 × x² + 90
(i) x6 + 5
Then x  x²
x6+ 1 + 4 = 4
x6 + 1 = 0
1 0
or = (–1)15 × x² + –1 15  x ²
geisnh
  (ii) x102 + x96 + x101 + x95 + x100 ,
x6 = – 1
+ x94
1 1
Proof x + = 3 = –( x² + ) x (x + 1) + x95(x6 + 1) +
96 6
x x² x94(x6 + 1)
Cube both sides
Enak

1 = x96 × 0 + x95× 0 + x94 × 0 = 0


1 3 x = 3 (As Given) Note: When difference of the power is
x³ +

=   3 –3× 3 x
6 then the value of sum of both of
Then,
R

terms value is 0.
1 1
x³ + =0 2
TYPE – VIII
x³ x² + =  3 –2
aryn

x² If x+y = 0 Then x = –y
Multiply x³ both sides
1 or
x6 + 1 = 0
Les B

x² + =3–2=1 x = 0, y = 0
x6 = – 1 x²
If x2+ y2 = 0
1 So,
Then x2 = 0, x = 0
Ex.89 If x + = 3 , Find the value
x 1 And y2 = 0, y = 0
wa. th

x92 + =–1
of x 92 If (x–1)2 + (y–2)2 = 0 then we
1 1 can say x = 1 and y = 2
1
(i) x90 + (ii) x96 + (ii) x93 + Ex.92 If (x + 3)2 + (y - 5)2 + (z + 2)2 = 0
x 90 x 96 x 93
Break the power multiple of 6 find the value x + y + z
wwM

1
Sol. x + = 3 1 Sol. (x + 3)2 + (y - 5)2 + (z + 2)2 = 0
x
x90 × x³ +
Then x6 = – 1 x  x3
90

1 1 0 0 0
(i) x90 + = (–1)15 × x3 + –1 15  x 3
x 90   x = -3 y=5 z = -2
1 x+y+z = -3 +5-2
1
(x6)15 + 6 15 = –( x3 + )
x  x³ =0
Ex.93 If(a - 4)2 + (b - 5)2 + (c - 3)2 = 0
1
=( – 1)15 + –1 15 1 a+b
  x = 3 (As Given) find the value
x c
= – 1– 1 = – 2

Rakesh Yadav Readers Publication Pvt. Ltd. 120

For More Visit : www.LearnEngineering.in


For More Visit : www.LearnEngineering.in

Sol. (a - 4)2 + (b - 5)2 + (c - 3)2 = 0 TYPE – IX


1 1 x 1 x
Rationalising factor of the =  = 1
x 1 1  x 1 x
0 0 0 s urd x = a b and
Ex.100. If x = 2  3 , find the value
a=4 b=5 c=3
1
 a b. x6  x4  x2 1
a+b 4+5 x of
= =3 x3
c 3
1 1
Ex.94 If x2 + y2 + z2 + 4x + 2y + 5 = 0 Ex.98 If x = ,y= Sol. x = 2 3
find the value x2 + y3 + z4 52 6 5–2 6
Sol. x2 + y2 + z2 + 4x + 2y + 5 = 0 1 1
1 1  = 2– 3
x2 + 4x + 4 + y2 + 2y + 1 + z2 = 0 Find the value of  x 2 3
x 1 y 1

ir
(x + 2)2 + (y + 1)2 + z2 = 0
1

v.iSn
1 x 4
Sol. x = x
0 0 0 52 6
Then,
x = -2 y = -1 z = 0 1 1
 y 1

dnag
x2 + y3 + z4 = (-2)2 + (-1)3 + (0)4 Now, x3 
3
  4 – 3  4
x 5–2 6 3
=4-1+0=3 x
Ex.95 If a2 + b2 + c2 = 2 (a -2b -2c) - 9 1
y 3 1
find the value of a3 + b4 - c2 x = x   52
x3

eYari
Sol. a2 + b2 + c2 = 2 (a -2b -2c) - 9 Put the value of y in equation
a2 + b2 + c2 - 2a + 4b - 4c + 9 = 0 Now,
a - 2a + 1 + b2 + 4b + 4 + c2 - 4c + 4 = 0
2 1 1
=  x6  x4  x2 1
(a-1)2 + (b + 2)2 + (c - 2)2 = 0 x 1 y 1 =
x3

0
a=1
0
b = -2
0
c=2
=
1

1
x 1 1 1
x
snhe = x³ + x +
1
x
1
+ 3
x
kgei
a3 + b4 - c2 = (1)3 + (-2)4 - (2)2
1 1
= 1 + 16 - 4 = 13 1 1 x 1 x = x³ + 3 +x +
 = 1 x x
Ex.96 If 5x2 + 4xy + y2 + 1 + 2x = 0 x 1 1  x 1 x
ERna

find the value of x & y Note: Put the value


52 + 4 = 56
Sol. 5x2 + 4xy + y2 + 1 + 2x = 0
1 1
4x2 + 4xy + y2 + x2 + 2x + 1 = 0 If xy = 1 then  =1 Ex101. If x  5  2 6 Find the value
1 x 1 y
(2x + y)2 + (x + 1)2 = 0 1
aBryn

–3 x4 
Ex.99 x=  3 2  , of x ²
0 0 x ² – 3x  1
–3
2x = -y x = –1 y=  3– 2  Sol. x 52 6
x = –1,
Les

–1 –1
Find the value of (x + 1) + (y + 1) 1
2x = –y 5–2 6
wa. th

2×–1= –y –3 x
y=2
Sol: x =  3 2 
1
Ex.97 If (x + y - z - 1)2 + (y + z - x - 5)2 Then x  10
x
+ (z + x - y - 3)2 = 0 1 –3
 
wwM

 3– 2 =y 3 1
find x+ y + z x Then x  = (10)3 – 3×10
x3
Sol. (x+y - z- 1)2 +(y +z -x- 5)2 + (z + x - y - 3)2= 0 1
y 1
x x3  = 970
x3
Now,
0 0 0 Now,
(x + 1)–1 + (y + 1)–1
x+y-z=1 y+z-x=5 z+x-y=3 x divide or nominator and
1 1
Adding all three eqations = x 1  y 1 denominator
x + y - z + y + z - x + z + x - y= 1 + 5 + 3
1
x+y+z=9 1 1 x4 
=  x²
Then x+ y + z = x 1 1 1 x ² – 3x  1
9 =3
x

Rakesh Yadav Readers Publication Pvt. Ltd. 121

For More Visit : www.LearnEngineering.in


For More Visit : www.LearnEngineering.in

x4 1 Sol. x = 3 – 2 2 1
 2 x²   98
x x x x²
1
= x ² 3x 1  32 2 And,
–  x
x x x
1 3
1 x³   10  – 3  10
1 Then, y x³
x3  x
x3 1
= 1 x² y² x³   970
x –3  x³
x y x
Put the value in equation,
970 970 1
= = Put the value of y = 98  2 100

r
10 – 3 7 x =
970  3 973

Sni
Ex102. If x  7  4 3 , find the value 2
x²  1  1
  1
1  x  x

gv.i
x 4  3x 3  5x 2  3x  1 Ex.105. If x   10 , Find the value
of x
x 4 1 x
7x
Sol. 1 of

ridna
x 74 3 x³  x ² – 5x  1

1
7–4 3 Now, 7x
x Sol.
x ² – 5x  1

eeYa
1
1 x 6 x divide or nominator and
x   14 x
x denominator
Then, 1 3
x3  3
  6 – 3  6 7x
x
1 2
geisnh x
x²   14  – 2 7
x² 3 1 x ² – 5x  1 = 1
x  3  198 x –5
x x x
1
x²   194
x² Ex.104. If x = 5  2 6 and xy = 1, 7
Enak

x² divide or nominator and = 1


x ²  y ²  2xy x  –5
denominator Find the value of x ³  y ³  3xy x
x 4  3x 3  5x 2  3x  1
R

Sol. x = 5  2 6 1
x² x  10 (As Given)
= x
x4 1 1
aryn

x² 5–2 6
x 7 7
=
Les B

xy = 1 (As given) 10 – 5 5
x 4 3x 3 5x 2 3x 1
    Then,
x² x² x² x² x² 2p 
= 1 Ex.106.  , Find the
x4 1 5–2 6 p² – 2p  1 4
 y=
x² x² x
wa. th

1
x ²  y ²  2xy value of p 
3 1 p
x ²  3x  5   x ³  y ³  3xy
x x²
= 1 2p 
x²  1 Sol. 
wwM

x² Put the value of y  p² – 2p  1 4


x
1  1 8p = p² – 2p + 1
x²   3 x    5 1
x²  2 10p = p² + 1
x²  x x²
= divide by p both sides,
1 1
x²  x³ 
x² x³  3 1
p  10
194  3  14  5 241 Now, p
= = 1
194 194 x  10 a
x Ex.107 If x   1 , find the value of
Ex.103 If x = 3 – 2 2 , y = 3  2 2 Then x

x² y² 1 2 x²  x  a
Find the value of  x²   10  – 2
y x x² x ³ – x²

Rakesh Yadav Readers Publication Pvt. Ltd. 122

For More Visit : www.LearnEngineering.in


For More Visit : www.LearnEngineering.in

a a b c 1 2 1
Sol. x 1 + + = 13  = A  5 3  B  5 3  C
x a –1 b – 2 c – 3 2 1

x² + a = x ....(i) 53 – 53 1
x² – x = –a ....(ii) 1 2 3
Ex.110 If + + ......  1 
x 1 x  2 x  3
x²  x  a   5 3  1 multiply and divided
 
x³ – x² 1007  
= 1249
From equation (i) x  1007
x x  1 
x x x 1   5 3  1
x³ – x² find + ......  
x 1 x  2 x  1007  
 2

ir
2  1 
   1 1 
= 1 2 3
 5 3  1  5 3  – 5 3  1  (1)2 

v.iSn
x² – x Sol. + + ......
x 1 x  2 x  3     
From equation (ii)     
1007
x²  x  a –2 = 1249
= x  1007 a³+b³ =a+b(a²–ab+b²)
x³ – x² a

dnag
Type – X 1 2 3 1
 –1+ –1+ –1.......
x 1 x 2 x 3 53  1
a2 – bc b2 – ac c2 – ab  3
Ex.108 If   x  1

eYari
a2  bc b2  ac c2  ab –1 = 1249 – 1007  5 3   13
x  1007  
= 1 Find  
2a 2 2b 2 2c 2 x x x
 2  2  + ...... 1 1
1
2 x 1 x  2 x  1007 5 3  1 = A 5 3  B  5 3   c

Sol.
a  bc b  ac c  ab

a2 – bc b2 – ac c2 – ab
 
a2  bc b2  ac c2  ab
=1
= –242 snhe
Ex.111 If x = 101 find the value of
3
 5 1 =
5 1 6
1
 
2

 
 


x x 2 – 3x  3 
kgei
Added 1 every terms 53 1 2  1
  = A 5  3  B  5 3   c
6 6  
a 2 – bc b 2 – ac c 2 – ab Sol:  2
x x – 3x  3   
 2
1 2 1+ 2 + Comparison of the terms
ERna

a  bc b  ac c  ab
x³ – 3x² + 3x
1 =1+3 1 1
We add and substract 1 then, A = 0, B = and C =
2 2 2 2 6 6
a – bc  a  bc b – ac  b  ac x³–3x²+3x–1+1
 + = (x–1)³+1
a 2  bc b2  ac 1 1
aBryn

Put the value x Then, A+ B – C = 0 + – =0


6 6
c 2 – ab  c 2  ab = (101–1)³ +1
+ 4
c 2  ab = 100³+1 = 1000001 1
Ex.114 : If
Ex.112 If x = 102 find the value of 3
16  3 4  1
2a 2 2b 2 2c 2
Les

  2  2 4 x(x²+3x+3)
2
a bc b ac c ab =A 3 16 + B3 4 + C
Sol: x(x²+3x+3)
wa. th

Note: In this type of question when x³ + 3x² + 3x Find the value of A + B + C


base d is s am e we add or We add and substract 1 then, 1
substract 1 x³+3x²+3x+1–1 Sol. = A 3 16 + B3 4 + C
3
16  3 4  1
1 2 3 = (x+1)³–1
wwM

Ex.109 If + + = 10 , Put the value x 1 2 1


a –1 b  2 c  3  2 1
= A  4 3  B  4 3  C
= (102 + 1)³ +1 = 103³+1
Find the v alue of 43 – 43 1
= 1092727+ 1 = 1092728
a b c  1 
+ + 1
a –1 b – 2 c – 3 Ex.113 : If 3   43 – 1 multiply and divided
25 – 3 5 1  
 
1 2 3 = A 3 25 + B3 5 + C
Sol. + + = 10  1 
a –1 b  2 c  3 Find the value of A + B – C 1   4 3 – 1
 
 
1 2 3 1 2
a –1
+1+
b 2
+1+
c 3
+1 Sol. 3
25 – 3 5 1  1   1  1 
 4 3 – 1  4 3   4 3  1  (1)2 
= 10+1+1+1     
= A 3 25 + B3 5 + C     

Rakesh Yadav Readers Publication Pvt. Ltd. 123

For More Visit : www.LearnEngineering.in


For More Visit : www.LearnEngineering.in

Put the value of x = 12 2


1
Then, D
43 –1 Note:-   always added the
3
= 11 – 12³ 2
 1 = 11 – 1728 = – 1717 prodcut of number we will get
3
4  – 13 E x. 119 f ind the v alue of
  perfect square integer number
 
1  7231  7233 H ere D = Difference o f
1 1 7232 numbers
43 – 1 = 43 – 1 Sol. Let 7232 = x Ex.123 What will be added in the
 prodcut of numbers (174 × 182)
4 –1 3 Then,
That resulted will be perfect
1   x – 1   x  1 number
2  1
x

r
= A  4 3  B  43   c Sol. D = 8
 
1  x² –1

Sni
  D 8
= =x = =4
1
x 2 2

gv.i
1
4 3 1 = A 4 3  B  4 3   c So,
2
 –     x = 7232 D
2
3 3     = 4² = 16
Ex. 120 find the v alue of 2
Comparison of the terms

ridna
4  673  677 So,
1 1 675
A = 0, B = and C = – 16 will be added the product of
3 3 Sol. Let 675 = x number

eeYa
1 1 4   x – 2  x  2  Maximum or minimum value of
Then, A+ B + C = 0 + – =0  quadratic equation
3 3 x
Quadratic equation in general form
TYPE – XI 4  x² – 4
 =x 1. When a>0 (In the equation
Ex.115 If x = 11, Find the value of x
geisnh ax²+bx+c.) The expression gives
x5 –12x4 + 13x³ – 12x² + 12x – 5 Then, minimum value
Sol. x5 –12x4 + 13x³ – 12x² + 12x – 5 x = 675
Ex.121 Find the v alue of 4ac – b ²
Expandable from the equation y
4a
16  9748  9756
Enak

Then,
Max = 
x5–12x4 + 12x³ – 12x² +12x – 5 x³ 9752
Sol. Let 9752 = x 2. When a<0 (In the equation
x5 –11x4 – x4 + 11x³ + x³ ax²+bx+c) The expression gives
=
R

–11x² – x² + 11x + x – 5 +x³ 16   x – 4  x  4  maximum value


Put the value of x = 11 =
x
= 11 – 5 + (11)³ = 11 – 5 + 1331 4ac – b ²
aryn

16  x ² – 16 =
= 1337 = =x 4a
Ex.116 x = 8 find x4 – 9x³ + 9x² – 9x x Min = – 
Les B

Sol. x4 – 9x³ + 9x² – 9x So, Ex.124 Expression 5x²–8x+14 what


Expandable from the equation x = 9752 will be minimum value ?
Then, Ex.122 What will be added in the Sol. 5x²–8x+14
4 3
x –8x – x + 8x² –8x–x
3 prodcut of numbers (30 × 36) a> 0
wa. th

Put the value of x = 8 That resulted will be perfect


square number 4ac – b ²
=–8 Then, min value =
Ex.117 If x = 12 find x4 – 13x³ + 13x² Sol. (30 × 36) 4a
– 12x + 11 Diffe rence of the number here,
wwM

Sol. x4 – 13x³ + 13x² – 12x + 11 between = 36 – 30 a = 5, b = –8, c = 14


Expandable from the equation =6
Then, 4  5 14 – 8 ²
4 6 Now, min =
x – 12x³ – x³ + 12x² + x² – 12x + 11 Divided by 2 in difference = 45
2
0 0 0
= 11 =3 280 – 64 216
Ex.118 If x = 12 find x4 – 14x³ + 13x² Square of 3 = 3² = 9 = =
20 20
÷ 12x + 11 (30 × 36) + 9 = 1080 + 9
Sol. x4 – 14x³ + 13x² ÷ 12x + 11 = 1089 54
Expandable form the equation 1089 is perfect square of 33 Min value =
5
Then, So,
4
x – 12x³ – x³ + 12x² + x² – 12x + 11 – x³ Ex.125 Find the minimum value of
9 will be added the product of (x–2) (x–9)
0 0 0 number

Rakesh Yadav Readers Publication Pvt. Ltd. 124

For More Visit : www.LearnEngineering.in


For More Visit : www.LearnEngineering.in

Sol. (x–2) (x–9) Relation between A.M & G.M 1


= x²–2x–9x+18 A.M > G.M Then, m  –5
m
= x²–11x+18 When a,b,c are +ve Real number
Coefficient of x² is 1 which 1>0 1
1 Minimum value of m  =2
Then min value m and are two number m
m
= 2–5 = –3
4 1 18 –  –11 ² 72 – 121
= = 1 So, minimum value of F(x) = –3
4 1 4 M 1 Ex.130 Find the minimum value of
A.M = m , G.M = M
–49 2 m
1
min value = x²+  5 , here x > 2
4 A.M > G.M x² – 2
Ex.126 Find the maximum value of

ir
1 1
12–7x–x² m Sol. x²+ 5
1

v.iSn
m > x² – 2
Sol. 12 – 7x – x² or –x² – 7x +12 2 m
2 m
Here a<0 1
Cofficient of x² is –1 which less 1 (x²–2)+ +5+2
m x2 – 2
Than 0 m > 1  

dnag
So, 2 Let x²–2 = m
4ac – b ² 1 1
Max value = m Now, m  7
4a m > 2 m

eYari
= a = –1, b = – 7 and c = 12 2
Minimum value = 7+2 = 9
When m +ve Real number.
4  –112 –  –7 ² Ex.131 Find the maximum value of
= 1
4   –1 Minimum value of m  =2 x4
m , where x is Real number

=
–48 – 49 –97 97
–4
=
–4
=
4
And snhe
We can say minimum value
1 Sol.
8
x 1

x4
x8 1
Ex.127 Find the maximum value of mn 
kgei
=2
5–12x–3x² mn componendo and dividendo
Sol. 5–12x–3x² When m is + ve Real number divide by x4
or Ex.128 If x >1, find the minimum
ERna

–3x²–12x5 1 x4
value of f (x) = x ²  –3 1
Here coefficient of x² is –3 which x² – 1 x4
=
less than 0, x8 1 1
1 4
x4  4
Now, Sol. x²  –3 x x
aBryn

a = –3, b = –12, c = 5 x² –1
Add and substract 1, 4 1
4ac – b ² Now, Min value = x  =2
So, max value = x4
4a
1
(x²–1) + –3+1
2 1
Les

=
4  –3  5 –  –12 ²  –1
x ² Then, max value =
4  –3 Let x² –1 = m
wa. th

(iii) If x+y will be given then xy


–60 – 144 1 will be maximum
= m –2
–12 m When x=y
1 Ex.132 Find maximum value xy if x+y
–204
wwM

= = 17 Minimum value of m  =2 = 20
–12 m
Then, Sol. x+y = 20
(ii) a and b are two numbers For maximum x = y
2–2 = 0
Then, Then x = y = 10
So, minimum value F(x) = 0
a b Ex.129 Find the minimum value of x² Max value x y = 10×10 = 100
A,M =
2 1 Ex.133 Find the maximum value xy,
and G.M = ab + –4 if x + y = 25
x² 1
Thus when a, b and c are three Sol. Add and substract Sol. x + y = 25
numbers For maximum x = y
Then, 1
Now, (x²+1) + –4–1 25
A.M = a+b+c x2 1
G.M = 3 abc
  Then x=y =
2
Let x²+1 = m

Rakesh Yadav Readers Publication Pvt. Ltd. 125

For More Visit : www.LearnEngineering.in


For More Visit : www.LearnEngineering.in

x=6 Put the value m


25 25
Max value = x y × x=y=z=6 Then
2 2 Max value = (4)³ = 64
Minimum value = x + y + z
625 = 6+6+6 = 18 Ex.137 If x+y+z = 24, maximum value
= of
4 Ex.136 If x+y +z = 18, Find the
Maximum value of (x–1) (y–2)(z–3) (x–1) (y–2)(z+3)
(iv) If x y will be given then
Sol. For maximum Sol. For max value (x–1) = (y–2) = (z–
x + y will be minimum when x 3) = m
= y here x & y (+ve) Real number (x–1)= (y–2)=(z–3) = m
x = m +1
Ex.134 Find the minimum value of x x = m+1
y = m +2
+ y if x y = 16 y = m+2 z = m –3
Sol. x y = 16 z = m+3  x+y+z = 24

r
For minimum value x = y = z  x+y+z = 18 (As given) Put the value x, y and z

Sni
x=y=4 Put the value x, y, and z m +1 + m+2 + m – 3 = 24
Min value = x + y = 4 + 4 = 8 m+1 + m+2 + m+3 = 18 3m = 24

gv.i
Ex.135 Find the minimum value of x 3m = 12 m=8
+ y + z if x y z = 216 m=4 Max value = (x+1) (y+2)(z –3)
Sol. xyz = 216 So, max value of = m×m×m = m³

ridna
Min value x = y = z (x–1) (y–2)(z–3) = m×m×m So, max value = (8)³ = 512
x×x×x = x³ = 216 = m³

eeYa
EXERCISE
geisnh
1. The value of
a b c a  b c 13. If x =3 + 2 2 , t h en t he v alue of
5. If   then is equal to
 1  1  1  1  3 4 7 c

1  
1  
1  
1  
 is:  1 
 x  x  1 x  2 x  3 
 x – 
 is :
(a) 0 (b) 1 (c) 2 (d) 3  x 
1 (a) 1 (b) 2
Enak

(a) 1  (b) x +4 144 14.4


x4 6. If  , then the value of x is (c) 2 2 (d) 3 3
0.144 x
14. If p = 9 99, t hen t he v alue of
1 x4 (a) 144 (b) 14.4
3 p p 2  3 p  3  1 is
R

(c) (d) (c )1.44 (d) 0.0144


x x 7. If 1 < x < 2 , t hen t he v alu e of  
(a) 1000 (b) 999
2. If x = 7 – 4 3 , t h en t he v alu e of 2 2 (c) 998 (d) 1002
aryn

x – 1  x – 3 is
 1 2
(a) 1 (b) –2 xy  y
x  

Les B

 is : (c) 3 (d) 2x – 4 15. If x : y = 7 : 3 then the value of 2 2


 x x –y
8. Given that 100.48 = x, 100.70 = y and xz =
y2 , then the value of z is close to is
(a) 3 3 (b) 8 3
(a) 1.45 (b) 1.88 (c) 2.9 (d) 3.7 3 4 3 7
(c) 14  8 3 (d) 14 (a) (b) (c) (d)
4 3 7 3
wa. th

5
9. If 47.2506 = 4A + 7B + 2C + + 6E ,
2a  b D x
3. If  3 , then find the value of
then the value of 5A + 3B + 6C + D +
16. If xx x
 
= x x , then x equals
a  4b
3E is
a b (a) 53.6003 (b) 53.603 4 2 9 3
wwM

(a) (b) (c) (d)


a  2b (c) 153.6003 (d) 213.0003 9 3 4 2
10. If 3x+3+7= 250, then x is equal to 17. If a = 7, b = 5 and c = 3, then the value
5 2 10 10 (a) 5 (b) 3 (c) 2 (d) 1 of a2 +b 2 +c 2 –ab –bc –ca is
(a) (b) (c) (d) (a) 12 (b) –12 (c) 0 (d) 8
9 7 9 7 1 2 3 4 5 31 1
11. If     .....   x t he 1 1 1 1 1 1 1 1 1 1 1 1
1 3 1 5 4 6 8 10 12 64 2 . .  . . –3 . .  . .
4. If A : B = : B:C= : and C : D value of x is 3 3 3 4 4 4 3 4 5 5 5 5
2 8 3 9 18. is
(a) 31 (b) 32 (c) 36 (d) 37 1 1 1 1 1 1 1 1 1 1 1 1 
.  .  . – .  .  . 

5 3 3 3 4 4 5 5 3 4 4 5 5 3 
= : then find the ratio of A : B : C 3 1 3 –1
6 4 12. If x  & y  , t hen equal to
:D 3 –1 3 1
2 3 47 49
(a) 6 : 4 : 8 : 10 (b) 6 : 8 : 9 : 10 value of x2+ y2 is : (a) (b) (c) (d)
(c) 8 : 6 : 10 : 9 (d) 4 : 6 : 8 : 10 (a) 14 (b) 13 (c) 15 (d) 10 3 4 60 60

Rakesh Yadav Readers Publication Pvt. Ltd. 126

For More Visit : www.LearnEngineering.in


For More Visit : www.LearnEngineering.in

1 6 1 8 1 2 1
19. If x = 7 – 4 3 , then x  is equal (a) x  (b) x  42. If 4b   2, then the value of
x 6 8 2
x x b
to:
8 1 6 1 3 1
(a) 1 (b) 2 (c) 3 (d) 4 (c) x –
8 (d) x –
6 8b 
3 is
x x b
5 1 5 –1 (a) 0 (b) 1 (c) 2 (d) 5
20. If a  & b  , then the 7–2
5 –1 5 1 31. If  a 7  b , then the value of
value of 72
2 2 a is
 3 3  3 –6  3 2x –1
a  ab  b 43. If 
      =   , then x is
2 2 is 11 4  5  5  5 
a  –ab  b (a) (b) –
3 3 equal to

ir
3 4 3 5 (a) –2 (b) 2 (c) – 1 (d) 1

v.iSn
(a) (b) (c) (d) 4 –4 7
4 3 5 3 (c) (d) 3 x  3 – x
21. If a = 4.36, b = 2.39 and c = 1.97, then
3 3 44.  2 then x is equal to
the value of a3 – b3 – c3 – 3abc is 3 x – 3 – x
1 1 1
(a) 3.94 (b) 2.39 (c) 0 (d) 1 32. If a   1 and b   1 then c 
b c a
5 12 5 7

dnag
3a  5b (a) (b) (c) (d)
 5 , then a : b is equal to is equal to 12 5 7 5
22. If
3a – 5b
1 3
(a) 2:1 (b) 2:3 (c) 1:3 (d) 5:2 (a) 0 (b) (c) 1 (d) 2 45. If x  , t he n t he v al ue of
23. If p : q = r : s = t : u = 2 : 3, then 2 2

eYari
(mp + nr +ot ) : (mq +ns +ou) equals :
33. If x  3  2 , then t he v alue of  1 x  1 – x 
(a) 3:2 (b) 2:3 (c) 1:3 (d) 1:2  
 is
24. If x : y = 3 : 4 , then (7x + 3y) : (7x –3y) is
 3 1   1 x – 1– x 

equal to : x  
(a) 5 : 2 (b) 4 : 3
 3  is

25.
(c) 11 : 3 (d) 37 : 19

For what value(s) of a is x 


1
4
x a
2

(a) 6 3

(c) 18 3
x 
snhe
(d) 24 3
(b) 12 3
46.
(a) – 3 (b) – 1

If
x 4 x –4
x 4 – x – 4
(c) 1 (d)

 2 then x is equal to
3
kgei
34. If x + y = 7, then the value of x3 + y3 (a) 2.4 (b) 3.2 (c) 4 (d) 5
a perfect square ?
+21xy is
1
1 1 1 1 (a) 243 (b) 143 (c) 343 (d) 443 3
47. If x  , the v alu e of
(a)  (b) (c) –
5
(d) 1 1 1  2  1
ERna

18 8 4 35. If x 3
3
y 3
 z , then {(x + y – z) +27
3
 3
26. If a  b,then which of the following 1 
xyz} equals : x
 – 
3  is
statements is true? (a) –1 (b) 1 (c) 0 (d) 27  x 

a b (a) 0 (b) – 2 (c) + 2 (d) 3 2


aBryn

a b a b 36. If  =1, a  0, b  0 the value of


(a) = ab (b) < ab b a 2
2 2 x – x 1 3
a3 +b3 is
48. If 2
 , then the value of
a b (a) 0 (b) 1 (c) – 1 (d) 2 x  x 1 2
(c) > ab (d) All of the above 37. If p = 99, then value of p(p2 +3p +3) is
2  1
(a) 999 (b) 9999 x  

Les

27. If x, y are two positive real number and  is


(c) 99999 (d) 999999  x
x1/3= y1/4, then which of the following
wa. th

relations is true? 38. If 5 x  12 x  13 x , then x is equal (a) 4 (b) – 5 (c) 6 (d) 8


(a) x3 = y 4 (b) x3 = y
to 2 1
(c) x = y4 (d) x20 = y15 49. If x  3  8 , then x  2 is equal to
25 x
3 1 x 1– x (a) (b) 4 (c) 9 (d) 16
28. If x  , then + is (a) 38 (b) 36 (c) 34 (d) 30
4
wwM

2 1 1 x 1– 1– x
equal to 39. If x, y and z are real number such that 50. If x  5  2 6 , t hen t he v alue of
(x – 3)2+ (y –4 )2+(z –5)2 = 0 then (x + y
(a) 1 (b) 2/ 3 + z) is equal to  1

 x  
 is.
(c) 2 – 3 (d) 2 (a) – 12 (b) 0 (c) 8 (d) 12  x
29. If for non-zero, x, x2 – 4x – 1 = 0, the
2 1 (a) 2 2 (b) 3 2
40. If x  3  8 , then x  2 is equal to
1 2 x
value of x  2 is (c) 2 3 (d) 3 3
x (a) 38 (b) 36 (c) 34 (d) 30
(a) 4 (b) 10 (c) 12 (d) 18 51. If x  3  2 , t hen t he v alue of
1  1
41. If x –  4 , then x  
  is equal to
 1 1  2 1  2 1  x  x  2 1 
30. x  
 x  
x  2 – 1
x  2  1
 is x 
 
2  is :
 x  x  x  x   x 
(a) 5 2 (b) 2 5 (c) 4 2 (d) 4 5
equal to (a) 4 (b) 6 (c) 9 (d) 10

Rakesh Yadav Readers Publication Pvt. Ltd. 127

For More Visit : www.LearnEngineering.in


For More Visit : www.LearnEngineering.in

9 a b c x 1
52. If x  6 , t he n t he v al ue of 63. If   = 1, then the value 73. If = , then value of
x 1–a 1–b 1–c 2
2x 5 x 2 6
 2 9  1 1 1  1
x   of + +

 2  is
x  1– a 1– b 1– c x  x 
  is:
 
(a) 1 (b) 2 (c) 3 (d) 4
(a) 8 (b) 9 (c) 10 (d) 12 1 1
2x – y 1 3x – y (a) 2 (b) (c) – (d) – 2
64. If = , then value of 2 2
1 3 1 x  2y 2 3 x y
53. If 2p+ =4, then value of p  3 is 74. If a, b, c are real and a2 +b2 +c2 = 2 (a
p 8p is : –b –c)–3 then the value of 2a –3b +4c
(a) 4 (b) 5 (c) 8 (d) 15 is
54. If a4 +b4 = a2b2, then (a6 +b6) equals
1 3 4 (a) – 1 (b) 0 (c) 1 (d) 2

r
(a) (b) (c) (b) 1
(a) 0 (b) 1 5 5 5 75. If (3a +1)2 +(b – 1)2 +(2c –3 )² = 0, then the

Sni
(c) a2 + b 2 (d) a2b4 + a4b2 value of (3a +b +2c) is equal to;
1 2x (a) 3 (b) – 1 (c) 2 (d) 5
If x   5 , then

gv.i
1 65. 2 is 76. The value of the expression
55. If x  3 , t he n t he v al ue of x 3x – 5 x 3
x 2 2
equal to a  b  b – c 
+ +
1
1 1
b  c c  a   bc – a 
a –

ridna
x³ 
x is : (a) 5 (b) (c) 3 (d) 2
5 3
x² – x  1 c – a
3 5 7 11 x
3
3 a – bb – c

eeYa
(a) (b) (c) (d) 66. If 1 –  , then x equals
2 2 2 2 100 5 1
(a) 2 (b) 4 (a) 0 (b) 3 (d) 2(c)
1 3
56. If a   1  0 a0 then the value of (c) 16 (d) (136)1/3
77. If (a – 3)2+(b – 4)2+(c – 9)2= 0, then the
a
(a4 –a) is: 4 3 5 2
geisnh value of a b c is :
67. If = a b 6 , then the values
(a) 0 (b) 1 (c) 2 (d) – 1 48  18 (a) –4 (b) 4 (c) 4 (d) 2
1 1 of a and b are respectively 78. If 1.5x = 0.04y, then the value of
57. If x  a  and y  a – , then the
a a 9 4 3 4 2 2
y –x
Enak

value of x4+y4–2x2y2 is (a) ,– (b) ,


15 15 11 33 2 2 is
(a) 24 (b) 18 (c) 16 (d) 12 y 2xy  x
58. If a = 11 and b = 9, then the value of
9 2 3 4
, , 730 73 73 74
R

a 2  b 2  ab  (c) (d)
10 5 5 15 (a) (b) (c) (d)
  77 77 770 77
 a3 – b3  is
  x 32 1
aryn

68. If 1  = , then the value of x is 79. If , then the value of a² –331a is


1
961 31 a 3 11
1
(a) (b) 2 (c) (d) 20
Les B

(a) 1331331 (b) 1331000


2 20 (a) 63 (b) 61 (c) 65 (d) 64 (c) 1334331 (d) 1330030
59. If p = 1 01, t hen t he v alue of
a c e 1 1
69. If  
3p p 2 b d f =3 80. If x2+y2+ 2  2  4 , then the value of
 
– 3 p  3 – 1 is x y
wa. th

(a) 100 (b) 101 2a 2  3c 2  4e 2 x2+ y2 is


then =? (a) 2 (b) 4 (c) 8 (d) 16
(c) 102 (d) 1000 2b 2  3d 2+ 4 f 2
81. If x2 = y + z, y2 = z +x, z2 = x +y, then
60. If x = 19 and y = 18, then the value of (a) 2 (b) 3 (c) 4 (d) 9 the value of
2 2 1
x y  xy 1
If 2 x   5 . Find the value of 1 1
wwM

70.
3 3 is 3x + + is
x –y y 1
x 1 z 1
(a) 1 (b) 37 5x
2 . (a) – 1 (b) 1 (c) 2 (d) 4
(c) 324 (d) 361 6x  2 0 x 1 82. If a2 +b2 = 2 and c2 +d2 = 1 then the
61. If 50% of (p –q) = 30% of (p +q), then p value of (ad –bc)2 +(ac+bd)2 is
: q is equal to 1 1 1 1
(a) (b) (c) (d)
(a) 5 : 3 (b) 4 : 1 4 6 5 7 4 1
(c) 3 : 5 (d) 1 : 4 71. If x varies inversely as (y2 – 1) and x is (a) (b) (c) 1 (d) 2
9 2
equal to 24 when y = 10, then the value
a b 2a 3b 4ab
62. If  , then value of is of x when y =5 is 83. If x = a  b , the value of
3 2 3a – 2b (a) 99 (b) 12 (c) 24 (d) 100 a b
72. If x2 + y2 +2x + 1 =0, then the value of x  2a x  2b
x31 +y35 is + is
12 5 12 x – 2a x – 2b
(a) (b) (c) 1 (d) (a) – 1 (b) 0 (c) 1 (d) 2
5 12 7 (a) a (b) b (c) 2 ab (d) 2

Rakesh Yadav Readers Publication Pvt. Ltd. 128

For More Visit : www.LearnEngineering.in


For More Visit : www.LearnEngineering.in

106. If a = 2.361, b = 3.263 and c =5.624, then


1 1 3 the value of a3 +b3 – c3 +3abc is
84. If m  = 4, 96. If x  , f in d t he v al ue of
m –2 4x 2 (a) (p – q) (q – r)³ + (r – p)³
(b) 3 (p – q) (q–r) (r–p)
2 1 3 1
find the value of m – 2    8x  (c) 0 (d) 1
2 3 .
 m–2  8x
2
(a) – 2 (b) 0 (c) 2 (d) 4 (a) 18 (b) 36 (c) 24 (d) 16 107. If p = 124, 3 p p  3 p  3  1  ?
 
85. If a2 +b2 +2b +4a +5 =0, then the value
1 1 1 (a) 5 (b) 7 (c) 123 (d) 125
a –b 97. If =  ( x0, y0, xy )
of is x y x y 1
a b 108. If x  2 and x is real , then the
then the value of x3–y3 is x
1 1 (a) 0 (b) 1 (c) – 1 (d) 2

ir
(a) 3 (b) – 3 (c) (b) – 98. If x =a(b–c), y = b(c –a) and z =c(a–b), 17 1
3 3 value of x 
19 is

v.iSn
3
 x  y   z 
3 3 x
x y xy  
then   
 
 
  =?
a  b   c  (a) 1 (b) 0 (c) 2 (d) –2
86. If x–y = = , the numerical 109. If x : y = 3 : 4, then the value of
7 4
value of xy is xyz
(a) (b) 3 xyzabc 5 x – 2y

dnag
3abc =?
4 3 1 1 7 x  2y
(a) (b) (c) (d)
3 4 4 3 3xyz xyz
(c) (d) 7 7 7 7
2 2 2 abc abc (a) (b) (d) (c)
x y z

eYari
  25 29 23 17
87. If x + y + z = 0, then =? 99. If xy(x +y) = 1, then the value of
yz zx xy 110. If x +y = 2z t h en t he v al ue of
(a) (xyz)2 (b) x2 +y2 +z2 1 3 3 z
– x – y is: x
(c) 9 (d) 3 3 3 + is
x y y –z
88. If a +b+c = 0, then the value of x –z

a b b
1
c
     
 
1
+ a c b a
(a) 0

1 4
snhe
(b) 1

100. If x  4 = 119 and x >1, then the


x
(c) 3 (d) –2
(a) 1 (b) 3
2
(d) 2 (c)

111. If a3b = abc = 180, a, b, c are positive


1
kgei
1 integers, then the value of b is
+ c a c b
   3 1 (a) 110 (b) 180 (c) 4 (d) 25
value of x  is
(a) 1 (b) 0 (c) – 1 (d) – 2 3 112. If a, b ar e r ational number and
x
ERna

89. If a + b +c = 0, then the value of


2 2 2
(a) 6 13 (b) 8 13 a – 1 2  3  b 2  a , the value of
a b c
2
a – bc
is (c) 13 13 (d) 10 13 a  b  is
(a) 0 (b) 1 (c) 2 (d) 3 (a) – 5 (b) 3 (c) – 3 (d) 5
aBryn

1
90. If x2 +y2 –4x –4y +8 = 0, then the value 101. If 3x  = 5, t h en t he v al ue of 113. If ax 2 +bx +c =a(x –p)2, then the rela-
of x –y is 2x tion among a, b, c would be
(a) 4 (b) – 4 (c) 0 (d) 8 (a) abc = 1 (b) b2 = ac
3 1 (c) b² = 4ac (d) 2b =a +c
91. If x = b +c –2a, y = c +a –2b, z =a +b –2c, 8x 
then the value of x2+y2–z2+2xy is 3 is: 114. If a +b + c + d =1, then the maximum
27 x
(a) 0 (b) a +b +c value of
Les

(c) a – b + c (d) a + b – c 1 10 (1 +a) (1 +b) (1 +c )(1 + d) is


wa. th

92. For real a, b, c if a² +b² +c² = ab +bc (a) 118 (b) 30 3


2 27 1 
3
3 5  4

a c (c) 0 (d) 1 (a) 1 (b) 


  (c) 
 
 (d)  
+ca, then value of is: 2  4 4 
b 102. If x + y = z, then the expression x³+y3 –
115. If a2 +b2 +c2 +3 = 2(a+ b +c) then the
(a) 1 (b) 2 (c) 3 (d) 0 z³ +3xyz wil be equal to :
value of
wwM

(a) 0 (b) 3xyz


1 ( a +b+ c) is
(c) –3xyz (d) z3
93. If x   3 then the value of x18+x12+x6 (a) 2 (b) 3 (c) 4 (d) 5
x
a
+1 is 103. If the sum of and its reciprocal is 1 1 2 1
(a) 0 (b) 1 (c) 2 (d) 3 b 116. If x – = 5, then x  2 is :
x x
94. If for two real constants a and b the and a  0 , b  0 , then the value of
expression ax3 +3x2 –8x +b is exactly (a) 5 (b) 25 (c) 27 (d) 23
a3 +b3 is
divisible by (x + 2) and (x –2) 117. If x = 3  2 2 , then the value of
(a) 2 (b) –1 (c) 0 (d) 1
(a) a = 2, b =12 (b) a = 12, b = 2
(c) a = 2, b = –12 (d) a = –2, b = 12 104. If x =2 –21/3 + 22/3 then the value of x3–
 1 
6x2+18x + 18 is
95. If x2 –3x +1 = 0, then the value of  x –
 
 is:
(a) 22 (b) 33 (c) 40 (d) 45  x
3 1 105. If a3 –b3 –c3 –3abc = 0, then
x  (a) 1 (b) 2
3 is (a) a = b = c (b) a +b+ c =0
x
(a) 9 (b) 18 (c) 27 (d) 1 (c) a + c = b (d) a = b + c (c) 2 2 (d) 3 3

Rakesh Yadav Readers Publication Pvt. Ltd. 129

For More Visit : www.LearnEngineering.in


For More Visit : www.LearnEngineering.in

118. If a +b +c = 0, then the value of 2 140. If a +b = 1 and a3 + b3 +3ab = k, then


 1 the value of k is
a 2  b2  c 2 x  
129. If   =3 then the value of (x72 +x66
is  x (a) 1 (b) 3 (c) 5 (d) 7
a 2 – bc +x54 + x 24+x6 +1) 141. If a = 34, b = c = 33, then the value of
(a) 0 (b) 1 (c) 2 (d) 3 (a) 0 (b) 1 (c) 84 (d) 206 a3 +b3 +c3 – 3abc is
1 (a) 0 (b) 111 (c) 50 (d) 100
119. If n = 7+ 4 3 , then the value of
130. If a   3 , then the value of 142. If (2x )(2y) =8 and (9x )(3y) = 81, then (x,
a y) is:
 1 
 n 
 
 is: 1 (a) (1,2) (b) (2,1) (c) (1,1) (d) (2,2)
 n  a6 –  2 will be
a6 143. The expression x4 –2x2+ k will be a per-
(a) 2 3 (b) 4 fect square when the value of k is
(a) 1 (b) 2 (c) 3 3 (d) 5 (a) 2 (b) 1 (c) – 1 (d) – 2
(c) – 4 (d) – 2 3 131. If x3 +y3 = 35 and x + y = 5, then the
1 1 1 1 1 1 1 1 1 1 1 1 1 1

r
120. If x = 3  2 , t h en t he v alue of . .  . . –3 . .  . .
value of 
x y will be : 3 3 3 4 4 4 3 4 5 5 5 5

Sni
 1 144.
1 1 1 1 1 1 1 1 1 1 1 1 
x  
  is .  .  . –
 x 1 5 2  .  .  . 
3 3 4 4 5 5 3 4 4 5 5 3 

gv.i
(a) (b)
(c) 6 (d)
3 6 3
(a) 2 2 (b) 2 3
132. If a3 – b3 = 56 and a – b =2 then value
(c) 2 (d) 3 2 3 47 49
of a2 +b2 will be : (a) (b) (c) (d)
121. If p + q =10 and pq = 5, then the (a) 48 (b) 20 (c) 22 (d) 5 3 4 60 60

ridna
p q
numerical value of q  p will be a b a b c
133. If (a2 +b2)3 = (a3 +b3)2 then  =? 145. If + + = 1, t hen the
b a 1–a 1–b 1–c
(a) 16 (b) 20 (c) 22 (d) 18

eeYa
1 2 1 2
122. If x=3+ 2 2 and xy = 1, then the value (a) (b) (c) – (d) – 1 1 1
3 3 3 3 value of is  
2 2
1–a 1–b 1–c
x  3xy  y 1
of is 134. If x  5 , t he n t he v al ue of (a) 1 (b) 2 (c) 3 (d) 4
x 2 – 3xy  y 2 x 146. If a, b, c are real numbers and a2 +b2 +
geisnh
x 4  3x 3  5x 2  3x  1 c2 = 2 (a –b –c) –3 then the value of 2a
30 70 35 37 –3b +4c is
(a) (b) (c) (d) x4 1
31 31 31 31 (a) – 1 (b) 9 (c) 1 (d) 2
x y z 43 47 41 45 147. The v alue of t he expr e ssion
123. If   , then (a) (b) (c) (d)
b c c a a b 23 21 23 21 2 2
a – b + b – c 
Enak

x y z +
; fn   gS
, rks% 1 b – c c – a a – bc – a
b c c a a b 135. If x is r eal, x  0 a nd
x
2
R

x –y y –z z –x c – a
(a)   3 1
b –a c –b a –c x 
3 = 0, t hen t he v alu e of a – bb – a
x
aryn

x y z
(b)   4 1
a b c  1
x  (a) 0 (b) 3 (d) 2 (c)
   is 3
Les B

x –y y –z z –x  x
(c)   148. If (x –3)2 +(y –5)2 +(z – 4)2 =0 then the
c b c (a) 4 (b) 9 (c) 16 (d) 25
value of
(d) None of the above is true
124. If x –y = 2, xy = 24, then the value of 1 2 2 2
136. If x   3 , t he n t he v al ue of x y z
(x2 +y2) is x   is
wa. th

(a) 25 (b) 36 (c) 63 (d) 52 9 25 16


 5 1  (a) 12 (b) 9 (c) 3 (d) 1
x2 y2 x
  
5  is
125. If the expression 2 + tx + is a per-  x  149. x varies inversely as square of y. Given
y 4 that y = 2 for x = 1, the value of x for y
fect square, then the values of t is (a) 322 (b) 126 (c) 123 (d) 113 = 6 will be equal to
wwM

(a) 1 (b) 2 (c) 0 (d) 3 4 1 1 1 1


137. If m  4 = 119, then m – =? (a) 3 (b) 9(d) (c)
126. If a = x +y, b = x – y, c = x +2y, then a2 m m 3 9
+b2 +c2 – ab – bc – ca is 150. If x2 –y2 = 80 and x –y = 8, then the
(a) 4y2 (b) 5y2 (c) 6y2 (d) 7y2 (a) 3 (b) 4 (c) 2 (d) 1 average of x and y is
1 138. If x + y + z =6, then the value of (a) 2 (b) 3 (c) 4 (d) 5
127. If x   2, x  0 t hen v alue of (x –1)3+(y – 2)3 +(z – 3)3 is
x x y 
1 (a) 3(x – 1) (y +2) (z –3) 151. The third proportional to 
y  x 
 and
2
x  3 is equal to (b) 3(x + 1) (y – 2) (z –3)  
x
(a) 1 (b) 2 (c) 3 (d) 4 (c) 3(x – 1) (y – 2) (z +3) 2 2 is
x y
a b (d) 3(x – 1) (y – 2) (z –3)
128. If  =1, a  0, b  0 the value 139. If a +b +c =6, a2 +b2 +c2 = 14 and a3 +b3 (a) xy (b) xy
b a +c3 = 36, then the value of abc is
of a3 +b3 is
(a) 0 (b) 1 (c) – 1 (d) 2 (a) 3 (b) 6 (c) 9 (d) 12 (c) 3 xy (d) 4 xy

Rakesh Yadav Readers Publication Pvt. Ltd. 130

For More Visit : www.LearnEngineering.in


For More Visit : www.LearnEngineering.in

160. If x and y are positive real numbers


43 3 and xy =8, then the minimum value of  2 3
152. The value of is 170. If x 
3 – 
 , then the value of
74 3 2x + y is  x x
(a) 9 (b) 17 (c) 10 (d) 8
(a) 5 3 – 8 (b) 5 3  8 161. If the expression x² +x +1 is written in 1
2
x 
2 is
(c) 8 3  5 (d) 8 3 – 5
 1 2 2 x
the form x  
   q , then the pos- 1 4 1 4
4 15  2
153. If x  , t he v alue of (a) 2 (b) 2 (c) 3 (d) 3
5 3 sible values of q are 9 9 9 9
171. If x2 – 3x + 1= 0, then the value of
x  20 x  12 1 3
+ is (a)  (b)  1 1
x – 20 x – 12 3 2 x2 + x   is
2

ir
x x

v.iSn
(a) 1 (b) 2 (c) 3 (d) 5 2 1 (a) 10 (b) 2 (c) 6 (d) 8
(c)  (d) 
154. If x = 5 – 21 , then the value of 3 2 a 2 b
2
162. If a 2 – 4a – 1 = 0, t h en v alu e of   
172. If a2 +b2 = 5ab, the value of b2 2
x  a 
is 1

dnag
32 – 2x – 21 3
a2+ 2 +3a– is is :
a a (a) 32 (b) 16 (c) 23 (d) – 23
1 1
(a)  3 – 7  (b)  7 – 3  (a) 25 (b) 30 (c) 35 (d) 40 173. If ; fn xy + yz + zx =0 , t h en r ks
2 2

eYari
 1 1 1 
1 1 163. One of the factors of the expression    
(x,y,z  0)
(b)
2
 7  3 (d)
2
7  3 2
4 3x +5x – 2 3 is :
x2 – yz y2 – zx z2 – xy 
 
a –b b –c (a) 3 (b) 1
a b b c
155. The v al ue of x  x  (a) 4x + 3 (b) 4x + 3 (c) x + y +z (d) 0

x 
(a) 1
c a c –a

(b) 2 (c) – 1 (d) 0


(c) 4x – 3

164. If
snhe (d) 4 x –

2
x  3 – 5 , then the value of x –
3
174. If a +b +c = 9 (where a, b, c are real
numbers ), then the minimum value
of a2 +b2 +c2 is
(a) 100 (b) 9 (c) 27 (d) 81
kgei
x 1 1 16x +6 is 175. If a2 +b 2 + 4c 2 = 2(a +b – 2c) – 3 and a,
156. If  – , t hen t he v alue of x b, c are real, then the value of (a2 +b 2
a a x (a) 0 (b) – 2 (c) 2 (d) 4
+c 2) is
– x 2 is :
2 2 2
ERna

x y z 1 1
1 1 165. If x + y + z = 0, then   =? (a) 3 (b) 3 (c) 2 (d) 2
(a) –a (b) (c) a (d)  yz zx xy 4 4
a a
(a) (xyz)2 (b) x2 + y2 z2 176. Number of solutions of the two equa-
1 tions 4x – y = 2 and 2x –8y +4 = 0 is
157. If x  99 , f in d t he v al ue of (c) 9 (d) 3
aBryn

x (a) zero (b) one


166. If ; fn a +b +c = 0, then the value of
(c) two
100 x
a b b c c a  a b c  (d) infinitely many
2
2x  102x  2  
  
    

c a b  b c c  a a  b a  b 15
1 1 1 1 177. If  and  , then
Les

(a) (b) (c) (d) (a) 8 (b) – 3 (c) 9 (d) 0 b 5 c 16


6 2 3 4
wa. th

1 2 2
18c – 7a
167. If a, b, c are non - zero a   1 and
4x – 3 4y – 3 4z – 3 b 2 2 is equal to
45c  20a
158. If + + =0 t hen
x y z
1
b  1 , then the value of abc is : 1 2 3 1
wwM

1 1 1 c (a) (b) (c) (d)


the value of   is 3 5 4 4
x y z (a) – 1 (b) 3 (c) – 3 (d) 1
168. If a + b +c = 2s, then 178. If x  0, y  0 and z  0 and
(a) 9 (b) 3 (c) 4 (d) 6
2 2 2 2 1 1 1 1 1 1
xy xz yz
159. If = a, = b and =c,
s – a  s – b  s – c  s
x
2

y
2

z
  
2 xy yz zx then the
xy xz y z 2 2 2
a b c
where a, b, c are all non -zero num- relation among x, y, z is
(a) a2 +b2 +c2 (b) 0 (a) x + y + z = 0
bers, then x equals to
(c) 1 (d) 2 (b) x + y = z
2abc 2abc
(a) (b)
ab  bc – ac ab  ac – bc 2 1 1 1 1
169. If x = 3 + 2 2 , the value of x  2 is (c)   =0
2abc 2abc x x y z
(c) (d)
ac  bc – ab ab  bc – ac (a) 36 (b) 30 (c) 32 (d) 34 (d) x = y = z

Rakesh Yadav Readers Publication Pvt. Ltd. 131

For More Visit : www.LearnEngineering.in


For More Visit : www.LearnEngineering.in

189. If x + y + z = 6 and x2 +y2 +z2 = 20 then


1 1
the value of x3 +y3 +z3 –3xyz is
179. If x = 3t, y =
2
t  1 , then the value (a) 64 (b) 70 (c) 72 (d) 76
203. If 3 = 3 3 +c and a,
4  32  1 a 4  b 2
of t for which x =2y is 3 b, c are rational numbers then a + b +
 1 c is equal to
1 2 190. If x = 1– 2 , the value of x – 
  is:
(a) 1 (b) (c) – 1 (d)  x
2 3 (a) 0 (b) 1 (c) 2 (d) 3
1 (a) – 8 (b) 8 (c) 2 2 (b) 1
2 2 3
180. If x  x a is a perfect square, 191. If x = a – b, y = b – c, z =c – a, then the 204. If x  2  3 , t hen the v alue of
5 numerical value of the algebraic ex-
then a is 3 1
pression x3 +y3 +z3 –3xyz will be x 
(a) a + b +c (b) 0 3 is
1 1 1 1 x
(a) (b)  (c) (d) – (c) 4(a +b+c) (d) 3 abc
100 10 10 10 (a) 8 (b) 9 (c) 2 (d) 4

r
181. Find the value of x for which the ex- 3
3– 2 3 2 205. If x  5  2 , then the value of x3 –6x2
pression 2–3x –4x2 has the greatest

Sni
192. If x = and y = , then
value. 3 2 3– 2 +12x– 13
the value of x3 +y3 is : (a) – 1 (b) 1 (c) 2 (d) 0

gv.i
41 3 3 41
(a) – (b) (c)  (d) (a) 950 (b) 730 (c) 650 (d) 970 206. The simplest form of the expression
16 8 8 16
2 2 2 2
182. The expression x4 –2x 2 +k will be a p
– p p –1 p
perfect square if the value of k is 193. If 2x+  3 , t h en t he v alue of  
3 2 2

ridna
x 2p  p p  3p p  1

1 1 1
x3+  2 is 1
(a) 1 (b) 0 (c) (d) 3
4 2 x (a) 2p2 (b) 2
2p

eeYa
5x 1 9 25 7
183. If = , then the value of (a) – (b) – (c)
(d) 11 1
2x ²  5 x  1 3 8 8 8 (c) p +3 (d)
p3
 1 194. If a + b + c = 15 and a2 +b2 +c2 = 83
then the value of a3 +b3 +c3 –3abc
x
  

geisnh 1
 2x  (a) 200 (b) 180 (c) 190 (d) 210 207. If x  2 , t he n t he v al ue of
(a) 15 (b) 10 (c) 20 (d) 5 x
1
1
195. If x  =1, then (x+1)5 + 5  2 1  3 1 
184. If xy (x +y) = 1, then the value of x x  3   is
x 1 x  1 
 2 
x 

x 
Enak

1 3 3 equals
– x – y is: (a) 20 (b) 4 (c) 8 (d) 16
3 3 (a) 1 (b) 2 (c) 4 (d) 8
x y 208. If a,b,c be all positive integers then the
1 1 1 least positive value of a3 +b3 +c3 – 3abc
(a) 0 (b) 1 (c) 3 (d) – 2
R

196. If   , then the value of is.


1 a b a b (a) 0 (b) 2 (c) 4 (d) 3
2
185. If x >1 a nd x   83 t h en a3 – b3 is 209. When f(x) = 12x3 – 13x2 –5x+7 is di-
2
x
aryn

(a) 0 (b) – 1 (c) 1 (d) 2 vided by (3x +2), then the remainder
197. If a +b +c = 0, then a3 +b3 +c3 is equal is
3 1 (a) 2 (b) 0 (c) – 1 (d) 1
x – to
Les B

3 is 210. If the equation 2x2 – 7x +12 =0 has two


x (a) a +b + c (d) abc
(a) 764 (b) 750 (c) 2 abc (d) 3 abc  
(c) 756 (d) 760 198. If x =y = 333 and z =334, then the value roots and , then the value of 
 
of x3 +y3 + z³ – 3xyz is
 1 2 1 is
wa. th

3 (a) 0 (b) 667


186. If a  
  = 3, then a 
3 =? (c) 1000 (d) 2334
 a a 7 1 7 97
199. Out of the given responses one of the (a) (b) (c) (d)
(a) 2 3 (b) 2 factors of 2 24 24 24
(a2 –b2)3+(b2–c2)3+(c2–a2)3 is
(c) 3 3 (d) 0 3 3 
wwM

(a) (a + b)(a–b) (b) (a + b)(a+b) 211. If x   4 (a3 + b3) and 3x  4


x 3
(c) (a – b)(a–b) (d) (b – c)(b–c) x
x 1
187. If = , then the value of (a3 + b3), then a2 – b2 is equal to
2 2
x – 2x  1 3 b
200. If a  then the value of a3 +b3 is (a) 4 (b) 0 (c) 1 (d) 2
1
3 b –a
x  3 is : 212. The term to be added to 121a2+64b2 to
x (a) 6 ab (b) 0 (c) 1 (d) 2 make a perfect square is
201. If p – 2q = 4, then the value of p 3– (a) 176 ab (b) 276 a2b
(a) 64 (b) 110 (c) 81 (d) 124 8q 3 – 24pq – 64 is (c) 178 ab (d) 188 b2a
 1 (a) 2 (b) 0 (c) 3 (d) – 1
188. If 213. If a = 2+ 3 , t h en t he v alue of
x
   4 ,
 t hen t he v alue of 202. If x = – 1, t hen t he v alu e of
 x
1 1 1 1 1 1 1  2 1 
4 1       –1 a
  
x  99 98 97 96 95 94 2
4 is : x x x x x x x  a 
x
(a) 64 (b) 194 (c) 81 (d) 124 (a) 1 (b) 0 (c) – 2 (d) – 1 (a) 12 (b) 14 (c) 16 (d) 10

Rakesh Yadav Readers Publication Pvt. Ltd. 132

For More Visit : www.LearnEngineering.in


For More Visit : www.LearnEngineering.in

214. For what value(s) of k the expression 224. If a + b + c +d =4, then find the
1 6 1
1 2 233. If x   5 , then x 
6 is
p  p k is perfect square ? 1 x x
4 value of +
1 – a1 – b1 – c (a) 12098 (b) 12048
1 1 1 (c) 14062 (d) 12092
(a) 0 (b)  (c)  (d)  1 234. If x² –3x+1 =0, then t he value of
4 8 2 +
 1–b  1–c 1 – d 6 4 2
1 x x x 1
215. The reciprocal of x  is 3 will be
x 1 x
+
x x 1 – c 1 – d1 – a (a) 18 (b) 15 (c) 21 (d) 30
(a) 2 (b) 235. If x is a r at iona l number and
x 1 x 1 1

ir
3 3
1 1 1 – d1 – a1 – b x  1 – x – 1

v.iSn
(c) x – (d) x 2 2 = 2, then the sum of
x x (a) 0 (b) 5 (c) 1 (d) 4 x  1 – x – 1
216. If a, b, c are positive and a+b+c =1, 1 numerator and denominator of x is:
225. If x – 1 , t h en t he v alue of
1 1 1 x (a) 3 (b) 4 (c) 5 (d) 7

dnag
then the least value of   is
a b c 236. If x  5 2, t h en t he v alue
4 1
(a) 9 (b) 5 (c) 3 (d) 1 x –
2 2
x 2x – 3x – 2
2

217. If a 2  3 = b 2 –   
3 = 1, then the 3x  5x – 3 2 is equal to

eYari
3x – 4x – 3
value of
1 1 3 (a) 0.1785 (b) 0.525
1 1 (a) (b) (c) (d) 0 (c) 0.625 (d) 0.785
2 + 2 4 2 4
a 1 b 1 237. If a = 2.234, b= 3.121 and c = –5.355,
226. If x +y =15, then (x–10)3+(y –5)3 is

(a) – 1

218. If
(b) 1 (c) 4

2  3 a  2 – 3 b  1 then the
(d) 9
(a) 25
(c) 625

227. If x
2
snhe
(b) 125
(d) 0
1
then the value of a3 +b3+c3–3abc is
(a)– 1 (b) 0 (c) 1 (d) 2
238. If x2 +y2 +1 =2x, then the value of x3 +y5
is
2 =66, then the value of

kgei
x (a) 2 (b) 0 (c) – 1 (d) 1
1 1 239. If 3(a2+b2+c2)= (a +b +c)2 then the rela-
value of  is 2 tion between a, b and c is
a b x – 1  2x
=?
x (a) a = b = c (b) a = b c
ERna

(a) 1 (b) 2 (c) 2 3 (d) 4 (c) a < b < c (d) a > b > c
(a) 8 (b) 10, – 6
240. If x(x–3) = – 1, then the value of x 3(x 3 –
1 1 1 (c) 6, – 10 (d) 4
219. If a  =b  =c  (a  b  c), 18) is
b c a 228. If a2 +a + 1 =0, then the value of a9 is (a) – 1 (b) 2 (c) 1 (d) 0
aBryn

(a) 2 (b) 3 (c) 1 (d) 0 241. The factors of (a2+4b 2+4b–4ab–2a– 8)


then the value of abc is
2 are
1 229. If x  1 , t he n t he v al ue of (a) (a –2b–4) (a –2b+2)
(a) 1 (b)  2 (c) 0 (d)  x
2 (b) (a –b–2) (a + 2b +2)
220. If (x –2) is a factor of x2 +3Qx –2Q, then 2 (c) (a +2b–4) (a +2b+2)
x x2
the value of Q is (d) (a +2b–4) (a –2b+2)
Les

2 is
(a) 2 (b) – 2 (c) 1 (d) – 1 x 1 – x  242. The value of
wa. th

221. If a +b = 12, ab = 22, then (a2 +b2) is (a) 1 (b) – 1 (d) 2 (d) – 2
equal to 1 1 2ax
230. If x = –2k and y =1 –3k, then for what – 
value of k, will be x = y ? 2 2 2 4 2 2 4
a²  ax  x a – ax  x a a x  x
(a) 188 (b) 144 (c) 34 (d) 100 (a) 0 (b) 1 (c) – 1 (d) 2
is
231. Find the value of
wwM

1 1 (a) 2 (b) 1 (c) – 1 (d) 0


222. If x  3 – an d y  3 ,
3 3 x ²  y²  zx  y – 3z + 3 xy 3 z 2 243. If x =11, then the value of x5–12x4+12x3–
12x2+12x –1 is
when x = +1, y = – 3, z = – 1,
2 2 (a) 5 (b) 10 (c) 15 (d) 20
x y
then the value of  is 1 244. I f p = 9 9 , t h e n t h e v a l u e o f p
y x (a) 1 (b) 0 (c) – 1 (d) (p 2 +3p +3) is
2
(a) 10000000 (b) 999000
(a) 3 (b) 3 3 1
x  2 , t he n t he v al ue of (c) 99999 (d) 990000
232. If
(c) 16 3 (d) 2 3 x 245. Which one is not an example of an equal-
ity relation of two expressions in x:
223. If x² +ax +b is a perfect square, then  2 1  3 1  (a) (x +3)2 = x2 +6x +9
which one of the following relations x
  x  3 
2   is (b) (x + 2y)3 = x3 +8y3 +6xy (x +2y)
between a and b is true  x  x 
(a) a2 = b (b) a2 =4b (c) (x +2)2 =x2 +2x + 4
(c) b = 4a (d) b2 =a (a) 20 (b) 4 (c) 8 (d) 16 (d) (x +3 ) (x – 3) = x 2 – 9

Rakesh Yadav Readers Publication Pvt. Ltd. 133

For More Visit : www.LearnEngineering.in


For More Visit : www.LearnEngineering.in

3
 1 2 4x –x
268. If 4a –
4
+3=0 ] then the value of
246. If a  
  =3, then the v alue of 256. The value of when a
 a 2x  16x – 3
x = 9999 is 1
3 1 a³ – +3=?
a  (a) 1111 (b) 2222 a³
3 is (c) 3333 (d) 6666
a
257. If a3+b3 = 9 and a+b =3, then the value 7 3 21 21
(a) 0 (b) 1 (c) 2 (d) 6 (a) (b) (c) (d)
1 1 16 16 64 16
1 of  is 269. If x = z = 225 and y = 226 then the
247. If a   3 , then the value of a18 +a12 a b
a value of: x² + y³ + z³ – 3xyz
+a6 +1 is (a) 765 (b) 676
1 3 5
(a) (b) (c) (d) – 1 (c) 674 (d) 576
(a) 0 (b) 1 (c) 2 (d) 6

r
2 2 2 270. If x²+x=5 then the value of:
248. If x = 997, y = 998 and z =999 then
258. If t 2 –4t+1 =0, then the value of

Sni
the value of x2 +y2 +z2 –xy –yz –zx is 1
(a) 0 (b) 1 (c) – 1 (d) 3 3 1 (x + 3)³ + 3
x  3

gv.i
t 
3 is:
1 t
249. If x  3 , t he n t he v al ue of (a) 140 (b) 110 (c) 130 (d) 120
x (a) 44 (b) 48 (c) 52 (d) 64 271. If m = – 4, n = –2, then the value of m³
– 3m² +3m + 3n + 3n² + n³ is
259. If 3 a  3 b  3 c , then the simplest

ridna
2 (a) 124 (b) –124 (c) 126 (d) – 126
3x – 4x  3
is value of (a+b–c)3 +27abc is 272. 2x – ky + 7 = 0 and 6x – 12y + 15 =0
2
x – x 1 (a) – 1 (b) 3 (c) – 3 (d) 0 has no solution for:
260. If 4x+5y =83 and 3x : 2y =21 : 22, (a) k = – 4 (b) k =4

eeYa
4 3 5 5 then (y –x) equals (c) k = 1 (d) k = –1
(a) (b) (c) (d) (a) 3 (b) 4 (c) 7 (d) 11 273. If x = 332, y = 333, z = 335, then the
3 2 2 3
value of x³ + y³ + z³ – 3xyz is
261. If 3 2 3 3 2 3 (a) 7000 (b) 8000
250. If x  3  2 2 , x  a  a b + a – a  b ,
(c) 9000 (d) 10000
geisnh
then x3 +3bx is equal to
6 4 2
x x x 1 (a) 0 (b) a (c) 2a (d) 1 1
then 3 is equal to 274. If 2+ x 3 = , then the simplest
x 24 2 3
x 1
(a) 216 (b) 192 262. If value of x is:
12 =7 t h en t he v al ue of (a) 1 (b) –2 (c) 2 (d) –1
x
Enak
(c) 198 (d) 204
275. If m – 5n = 2, then the value of (m 3 –
251. If a + b + c = 0, then the value of
72 125 n³ – 30mn) is :
(a + b – c)2 + (b + c – a)2 + (c + a – b)2 is x 1
(a) 6 (b) 7 (c) 8 (d) 9
(a) 0 (b) 8abc 36
x
R

(c) 4(a2 +b2+c2) (d) 4(ab+bc+ca) 276. If x = a 3 b a 3 b........ , t he n t he


252. If p3 +3p2 +3p = 7, then the value of (a)343 (b) 433 (c) 432 (d) 322
263. If P = 99 then the value of P(P²+3P+3) value of x is:
p2+2p is
aryn

(a) 989898 (b) 998889


(a) 4 (b) 3 (c) 5 (d) 6 (a) 5 ab ³ (b) 3 a 5b
(c) 988899 (d) 999999
253. If x = 2015, y = 2014 and z =2013, then
Les B

value of x2+y2+z2 –xy–yz–zx is 264. If x = 2 then the value of x3+ 27x² +


243x + 631 (c) 3 a 3b (d) 5 a 3b
(a) 3 (b) 4 (c) 6 (d) 2
(a) 1321 (b) 1233
254. If 3a 2=b 2  0, t hen t he v alue of (c) 1231 (d) 1211 1 12 1
277. If x  =2, then the value of x –
3 3 265. If x² + y² + z² = 2(x + z –1), then the x x12
wa. th

a  b – a – b value of; x³ +y³+z³ = ? is:


2 2 is (a) – 1 (b) 2 (c) 0 (d) 1 (a) – 4 (b) 4 (c) 2 (d 0
a  b  a – b
1 1
266. If x =1, t h en t he v al ue of 278. If x =1, t h en t he v al ue of
3b b 2b x x
wwM

(a) (b) b (c) (d) 2


2 2 3 x  3x  1
2 is:
=? x ²  7x  1
1 1 x² – x  2
255. If x >1 and x  =2 , then the (a) 1/2 (b) 3/7 (c) 2 (d) 3
x 12 (a) 2/3 (b) 2 (c) 1 (d) 4 279. If x +(1/x) =2, then the value of x7 +
(1/x5) is:
4 1 5 – 3 5 3 (a) 25 (b) 212 (c) 2 (d) 27
value of x –
4 is 267. If x = and y= , then
280. The term, that should be added to(4x²
x 5 3 5 3
+8x) so that resulting expression be a
58975 59825 x ²  xy  y ² perfect square, is:
(a) (b) the value of =? (a) 2x (b) 2 (c) 1 (d) 4
20736 20736 x ² – xy  y ² 281. If 999x + 888y = 1332 and 888x + 999y
= 555
57985 57895 65 67 69 63 Then the value of x + y is?
(c) (d) (a) (b) (c) (d)
20736 20736 63 65 67 61 (a) 888 (b) 1 (c) 555 (d) 999

Rakesh Yadav Readers Publication Pvt. Ltd. 134

For More Visit : www.LearnEngineering.in


For More Visit : www.LearnEngineering.in

293. If p + m = 6 and p³ + m³ = 72, then the


1 1 value of pm is xyz
282. If x = , y= , then t he (a) (b) 0
2 3 2– 3 (a) 6 (b) 9 (c) 12 (d) 8 abc
value of 8xy(x² + y²) is 294. When xm is multiplied by xn, product
(a) 112 (b) 194 (c) 290 (d) 196 is 1. The relation between m and n is 3 x yz 2 xyz
(a) mn = 1 (b) m + n = 1 (c) (d)
abc abc
x2 x –2 (c) m = n (d) m = – n
283. If a = , then the value
x 2 – x –2 2p 1 x a 2 x ² –y²
295. If = , then the value of 306. If = , then the value of
of a² –ax is P² – 2P +1 4 y a2 x ²+y²
(a) 2 (b) 1 (c) 0 (d) – 1
284. If a + b = 1, find the value of a³ + b³ –  1 2a 4a
ab – (a² – b²)²  p+ p  is (a) (b)
(a)0 (b)1 (c) – 1 (d)2
  a²+2 a²+4

ir
1 2

v.iSn
285. If a – = 5, then the value of (a) 7 (b) 1 (c) (d) 10 2a 4a
a–3 5 (c) (d)
296. If x = 2, y = 1 and z = – 3, then x3 + y3 + a²+4 a²+2
1
(a–3)³ – z3 –3xyz is equal to
(a–3)³ (a) 6 (b) 0 (c) 2 (d) 8 a b
307 If   2, then the value of a – b is:
b a

dnag
(a) 7 (b) 14 (c) 2 (d) 5 297. (x3 +y6) (x3 – y6) is equal to
286. (3x –2y) : (2x +3y) = 5 : 6, then one of (a) x6 – y12 (b) x9 – y16
(a) 2 (b) –1 (c) 0 (d) 1
(c) x6 + y12 (d) x9+ y36
2 308. If x( x+y+z)=20, y(x+y+z)= 30, &
3x + 3y  1 1
z(x+y+z)=50, then the value of 2(x+y+z)
the value of 3  is 298. The sum of and is

eYari
– 3y x y x –y is:
 x 
2y 2x (a) 20 (b) 10 (c) 15 (d) 18
1 1 (a) 2 2 (b) 2 2 309. If x+y=4, x²+y²=14 and x > y. Then the
(a) (b) 5 (c) (d) 25 x –y x –y
25 5 correct value of x and y is:

287. If x – 3 – 2=0

and y– 3 + 2 = 0 , then value of


(c)2
–2y
x –y
2 (d) 2
2x
y –x
snhe
2
299. If x + y = 2a, then t he value of
(a) 2– 2 , 3

(c) 2+ 3 , 2– 3
(b) 3, 1

(d) 2+ 3,2 2
310. If for non-zero x, x² – 4x – 1= 0 the
kgei
a a
 x ³ – 20 2  –  y³ + 2 2  + is 1
x –a y–a
(a) 2 (b) 3 (c) 1 (d) 0 value of is x²  :
(a) 0 (b) – 1 (c) 1 (d) 2 x²
288. 3(a²+b²+c²) = (a+b+c)² , then the rela- 300. For real a, b, c if a² + b² + c² = ab + bc
ERna

tion between a, b and c is (a) 4 (b) 10 (c) 12 (d) 18


a+c
(a) a = b  c (b) a  b  c + ca, the value of is :
b 3– 2
(c) a  b = c (d) a = b = c (a) 2 (b) 1 (c) 0 (d) 3 311. If a 
3+ 2
301. If p³ – q³ = (p – q) {(p – q)² – xpq} , then
aBryn

1 1 1 1
– – find the value of x is:
289. x = a 2 +a 2 , y = a 2 –a 2 , t h en (a) – 1 (b) 3 (c) 1 (d) – 3 3+ 2 2 2
a b
302. If x + y + z = 6 and xy + yz + zx = 10, then and b = , then + value
4 2 2 4 2 2 3 3 3
the value of x + y + z – 3xyz is: 3– 2 b a
value of (x –x y –1) + (y –x y +1)
(a) 36 (b) 40 (c) 42 (d) 48 of :
(a) 13 (b) 12 (c) 14 (d) 16
x 1 a 1–y b (a) 1030 (b) 970
Les

303. If = and  , then the


290. If m = 5+ 5+ 5....... 1 y a (c) 1025 (d) 930
x –1 b
wa. th

312. If (2a – 1)² + (4b – 3)² + (4c + 5)² = 0 then


n= 5– 5– 5–...... x –y
value of is: a³ + b³ + c³ – 3abc
1  xy the value of is:
then among the following the relation
a² + b² + c²
between m & n holds is 2 2 2 2
a –b a b
wwM

(a) m–n+1=0 (b) m+n+1=0 (a) (b) 3 3 3


(c) m+n–1=0 (d) m–n–1=0 ab 2ab (a) 3 (b) 2 (c) 0 (d) 1
8 8 8
3–5x 3–5y 3–5z 2 22ab
a –b
291. If + + =0, t hen t he (c) (d) 2 2 2
2x 2y 2z 2ab a b a  1   3 , then find the value
2 2
304. If a + b + c²– ab – bc – ca = 0 then a :
313. If  
2 2 2  a
value of + + is b : c is:
x y x of a 3 0 + a 2 4 + a 1 8 + a 1 2 + a 6 + 1
(a) 1 : 2 : 1 (b) 2 : 1 : 1
(a) 20 (b) 10 (c) 5 (d) 15 (a) 0 (b) 27 (c) 1 (d) –1
(c) 1 : 1 : 2 (d) 1 : 1 : 1
292. If 2s = a + b + c, then the value of s(s
305. If x = a(b – c), y = b(c – a), z = c(a–b) 1 1 1
– c) + (s –a) (s – b) is then the value of   , then the value of
314. If
(a) ab (b) 0 a b a b
3 3 3
a+b+c x y z a3–b3 is:
(c) abc (d) a +  +  is:
2   b  c  (a) 3 (b) 2 (c) 1 (d) 0

Rakesh Yadav Readers Publication Pvt. Ltd. 135

For More Visit : www.LearnEngineering.in


For More Visit : www.LearnEngineering.in

ANSWER KEY
1. (d) 33. (c) 65. (b) 97. (a) 129.(a) 161.(b) 193.(c) 225.(b) 257.(b) 286.(d)
2. (d) 34. (c) 66. (b) 98. (c) 130.(b) 162.(b) 194.(b) 226.(d) 258.(c) 287.(d)
3. (c) 35. (c) 67. (d) 99. (c) 131.(b) 163.(d) 195.(b) 227.(b) 259.(d) 288.(d)
4. (c) 36. (a) 68. (a) 100.(d) 132.(b) 164.(c) 196.(a) 228.(c) 260.(b) 289.(d)
5. (c) 37. (d) 69. (d) 101.(b) 133.(b) 165.(d) 197.(d) 229.(a) 261.(c) 290.(d)
6. (d) 38. (b) 70. (d) 102.(a) 134.(a) 166.(c) 198.(c) 230.(b) 262.(d) 291.(b)
7. (d) 39. (d) 71. (a) 103.(c) 135.(b) 167.(a) 199.(a) 231.(b) 263.(d) 292.(a)
8. (a) 40. (c) 72. (a) 104.(c) 136.(c) 168.(c) 200.(b) 232.(b) 264.(b) 293.(d)
9. (c) 41. (b) 73. (b) 105.(d) 137.(a) 169.(d) 201.(a) 233.(a) 265.(b) 294.(d)
10. (c) 42. (a) 74. (c) 106.(c) 138.(d) 170.(b) 202.(c) 234.(c) 266.(b) 295.(d)
11. (c) 43. (c) 75. (a) 107.(d) 139.(b) 171.(a) 203.(a) 235.(b) 267.(d) 296.(b)

r
12. (a) 44. (b) 76. (b) 108.(c) 140.(a) 172.(c) 204.(d) 236.(c) 268.(c) 297.(a)

Sni
13. (b) 45. (d) 77. (c) 109.(c) 141.(d) 173.(d) 205.(d) 237.(b) 269.(b) 298.(b)
14. (a) 46. (d) 78. (b) 110.(a) 142.(a) 174.(c) 206.(b) 238.(d) 270.(b) 299.(a)

gv.i
15. (a) 47. (c) 79. (b) 111.(b) 143.(b) 175.(d) 207.(b) 239.(a) 271.(d) 300.(a)
16. (c) 48. (b) 80. (a) 112.(d) 144.(c) 176.(b) 208.(a) 240.(a) 272.(b) 301.(d)
17. (a) 49. (c) 81. (b) 113.(c) 145.(d) 177.(d) 209.(d) 241.(a) 273.(a) 302.(a)

ridna
18. (c) 50. (c) 82. (d) 114.(d) 146.(c) 178.(d) 210.(b) 242.(d) 274.(d) 303.(d)
19. (d) 51. (d) 83. (d) 115.(b) 147.(b) 179.(b) 211.(c) 243.(b) 275.(c) 304.(d)
20. (b) 52. (c) 84. (c) 116.(c) 148.(c) 180.(c) 212.(a) 244.(c) 276.(d) 305.(c)
21. (c) 53. (b) 85. (c) 117.(b) 149.(d) 181.(d) 213.(b) 245.(c) 277.(d) 306.(b)

eeYa
22. (d) 54. (a) 86. (a) 118.(c) 150.(d) 182.(a) 214.(a) 246.(a) 278.(a) 307.(c)
23. (b) 55. (c) 87. (d) 119.(b) 151.(a) 183.(d) 215.(a) 247.(a) 279.(c) 308.(a)
24. (c) 56. (a) 88. (b) 120.(b) 152.(a) 184.(c) 216.(a) 248.(d) 280.(d) 309.(c)
25. (b) 57. (c) 89. (c) 121.(d) 153.(b) 185.(c) 217.(b) 249.(c) 281.(b) 310.(d)
26. (c) 58. (a) 90. (c) 122.(d) 154.(b) 186.(d) 218.(d) 250.(d) 282.(a) 311.(b)
geisnh
27. (d) 59. (a) 91. (a) 123.(a) 155.(a) 187.(b) 219.(a) 251.(c) 283.(d) 312.(c)
28. (b) 60. (a) 92. (b) 124.(a) 156.(c) 188.(b) 220.(d) 252.(b) 284.(a) 313.(a)
29. (d) 61. (b) 93. (a) 125.(a) 157.(c) 189.(c) 221.(d) 253.(a) 285.(b) 314.(d)
30. (d) 62. (a) 94. (c) 126.(d) 158.(c) 190.(d) 222.(b) 254.(a)
Enak

31. (b) 63. (d) 95. (b) 127.(b) 159.(c) 191.(b) 223.(b) 255.(a)
32. (c) 64. (b) 96. (a) 128.(a) 160.(d) 192.(d) 224.(a) 256.(c)
R

SOLUTION
aryn
Les B

1   1   1  1 1 74 3 1 3
4. (c) A : B = :
1. (d)   1
  1  x  1   1  x  2  = × 2 8
x      x 7–4 3 74 3
 8 : 6
1  1  74 3
wa. th

= = 7 + 4 3  4 : 3
 x 

3 49 – 48
 1 5
Taking L.C.M of each term. 1  B : C  3 : 9
 x+ =7–4 3 +7+4 3 = 14
 x 1  x  1  1  x  2 1 x  9 : 15  3:5
 x   x 1   x 2 
wwM


      2a+b 5 3
3. (c) = 3 (giv en)  C : D  6 : 4
a+4b
 x  3  1 2a + b = 3 (a+4b)
 20 : 18  10 : 9
 x 3  2a + b = 3a + 12b
  A : B : C : D
 –a = 11b 4 : 3
1 x 4 a = –11b 3 : 5
 × (x +4)  10 : 9
x x a+b
 a+2b 8 : 6 : 10 : 9
2. (d) x = 7 – 4 3
–11b+b a b c
5. (c) = = = k
1 1  –11b+2b 3 4 7
  a = 3k
x 7 – 4 3 10
–10b b = 4k
By rationalisation = =
–9b 9 c = 7k

Rakesh Yadav Readers Publication Pvt. Ltd. 136

For More Visit : www.LearnEngineering.in


For More Visit : www.LearnEngineering.in

a+b+c 3k + 4k + 7k
 = = 2 3 1 3 –1 1 1 2 –1
c 7k 12. (a) x = and y =  = ×
3 –1 3 1 x 2 1 2 –1
144 14.4
6. (d)
0.144
=
x ( gj d k i fj es; d j . k d j usi j )
1
144  1000 144   x =
y 2 –1
  = = 2 –1
144 x  10 1
3 1 3 1
144 x = ×
 1000  3 –1 3 1 1
10 x  x –
x
144 2
 x 
1000  10 =
 3 1  =  2 –1 
2 1–

ir
3 –1

v.iSn
144
x = = 0.0144 = 2 1– 2 1 = 2
10000 3 1  2 3 42 3
7. (d) 1 < x < 2 = = 14. (a) P = 999
2 2
2 2
x – 1 + x – 3
2  3  
3 p p2  3 p  3  1 

dnag
=
(square root cancel with square)
 x – 1 + x – 3 = 2x – 4 2  3 p 3  3 p2  3 p  1
8. (c) 10 0.48 = x x2 = 2  3  = 4 + 3 + 4 3
10 0.70 = y
3  p  1 3

eYari
and x z = y 2 = 7 + 4 3 
0.48 z 0.70 2
 10  = 10   y2 =
1
×
7 – 4 3
3
7 4 3 7 – 4 3 = 3  999  1 
 10 0.48z
= 10 (If a = a , if 1.40 x y

ba se e qua l pow er a re eq ua l:
(x = y)
 0.48z = 1.40
z =
140
48
=
12
35
= 2.9
y2 =

= 7 – 4 3
snhe
7 – 4 3
49 – 48
=
7 – 4 3
1
15.
= 3 10 00 

(a)
3

x :y
7: 3
= 1000

 x2 + y2 = 7 + 4 3 + 7 – 4
kgei
3
5 = 14 2
9. (c) 47.2506=4A+7B+2C+ +6E xy  y 21  9 30 3
D
47.2506 = 4×10+7×1+2× 0.1000 +
Alternate:-  2 2 = = =
x –y 49 – 9 40 4
5×0.0100 + 0 + 6×0.0001 1
ERna

 A = 10 B = 1 C = 0.1000 x² + y² = x² + 2 x
1
x 16. (c) x x x 
= x x 
D = = 100 , E = 0.0001 2
1  1  1 x
=  x+  –2  x=   3 
100  x  y
aBryn

x x = x 2 
x  
 5A + 3B + 6C + D + 3E 2
 
= 5 × 10 + 3 × 1 + 6 × 0.1 + 100 +  3 1 3 – 1
3 × 0.0001 =    –2 3
= 50 + 3 + 0.6 + 100 + 0.0003  3 –1 3  1 xx x = x 2x
= 153.6003 (If bases are s ame then their
10. (c) 3 x+3 + 7 = 250
Les

 3 1  3 –1 2 2 2 power is also same)


3 x+3 = 250 – 7
   –2   
wa. th

3 x+ 3 = 243 3 3
=    x x = x or
3 x+ 3 = 3 5 3 –1 2 x = 2
x + 3 = 5  
x = 2 2
 (a+ b)² + (a–b)² = 2(a² + b²)  3 9
1 2 3 4 5 31 x =   =
2  2 4
wwM

11.(c) × × × × ×.......×
4
1
6 8 10 12 64 2


 3 
2
 (1)
2
  –2
17. (a) a² + b² + c² – ab – bc– ca
1
= x = 2 = a – b 2  b – c 2  c – a 2 
2   2
 
5
 1 30  1  = (3 + 1)² –2 = 16 – 2 = 14
=
1
 7 – 5 2   5 – 3 2   3 – 7 2 
   × 2  2
2    13. (b) x = 3 + 2 2
2 24
 1 306

=  
  =
1 x = 2 + 1 + 2 2 =  2 1  =
2
 4  4  16  =
2
= 12
2  2x 18. (c)
x = 2 + 1
1 1 1 1 1 1 1 1 1 1 1 1 1 1
     – 3     
or 36 = x 1 1 3 3 3 4 4 4 3 4 5 5 5 5
2 2 = 1 1 1 1 1 1 1 1 1 1 1 1
x 2 1      –      
 x = 36 3 3 4 4 5 5 3 4 4 5 5 3

Rakesh Yadav Readers Publication Pvt. Ltd. 137

For More Visit : www.LearnEngineering.in


For More Visit : www.LearnEngineering.in

A 3 + B 3 + C 3 – 3ABC = (A + B + C)
1 Here, A = x and B = a
(A 2 + B 2 + C 2 – AB – BC – CA)  a + = 3
a
3 3 3 1 1
1 1 1 1 1 1 B =  a =
 3   4 – 3 3  4  5   5  1 8 8
     
2 2 2 a2 + 2 = 9 – 2 = 7 (ab = 1) 26. (c) Given that a  b
 1 1 1 1 1 1 1 1 1 a
3  4  5 – 3  4 – 4  5 – 5  3 Let a = 16, b = 4
     
21  by options
 1 1 1   1 
2
1
2
1
2
1 1 1 1 1 1 a  ab

        4    5  – 3  4 – 4  5 – 5  3  2 7 1 8 4 a+b 16+4
 3 5 4   3      
a So, = = 10
=  1
= = =
7 –1 6 3 2 2
 1 2  1 2  1 2 1 1 1 1 1 1  2
        –  –  –   a  – ab
 3  4 5 3 4 4 5 5 3  2 and
a ab = 16  4 = 8
20  15  12 47 21. (c) a = 4.36 b = 2.39 c = 1.97 a+b >

r
=
60
=
60
 ab
a – b – c 2

Sni
= 4.36 – 2.39 – 1.97  option (c) is correct.
19. (d) x = 7 – 4 3
= 0 27. (d) x 1/3 = y 1/4

gv.i
= 4 + 3 – 4 3 a³ –b³ – c³ – 3abc
 LCM of 3, 4 = 12
2
1
= (2) + 2
  3 – 2 × 2 3
= (a– b– c)[(a– b) 2+ (b– c) 2+(c– a) 2 ]   x1/3 
12
 y1/4 
12

ridna
2

= 2– 3  = 0 x4 = y3
take power '5' on both sides
 [(a 2 + b 2 – 2ab = (a–b) 2] 3a  5 b 5 5
22. (d) = 5
 2 – 3 
2
3a – 5 b   x4  = y
3
 

eeYa
x =

x 2– 3  3a + 5b = 15 a – 25b  x 20 = y 15
1 1 2 3  12a = 30b 3
=  28. (b) x =
x 2– 3 2 3  2a = 5b 2
a : b
geisnh
= 2 3 3 2 3
5 : 2 or 1 + x = 1 + =
23. (b) p : q = r : s = t : u = 2 : 3 2 2
1
 x 
x mp  nr  ot =

2 2 3 
Enak

 mq  ns  ou 2 2
= 2 – 3 + 2 + 3 = 4
(divides and multiply by 2)

5 1 5 –1 m  2x  n  2 x  o  2x 42 3
  1 + x =
R

20. (b) a = b = m  3x  n  3x  o  3x 4
5 –1 5 1
1 3  2 3
2 x m  n  o  2 =
aryn

1 4
 a =  3 x m  n  o  =
b 3 2
1 2   3   2 1 3
Les B

=
 mp + nr + ot : mq + ns + ou 4
1
a + b = a + 2
a 2:3
1 + x  1  3 
24. (c) x : y = 3 : 4 4
5 1 5 –1
wa. th

 + 1 3
5 –1 5 1
 x   1 x =
2
y 7 y  3 7  3  3
7x  3y   4
Similarly,
=y

5 1 2 5  5 1– 2 5 7x – 3y  7 x – 3  = 3 3 –1
2 2  y  7 4 – 3 1– x  2
 
wwM

5 – 1 
1 x 1– x
21 2112
 
3 1 1 x 1– 1– x
62 5 6–2 5 12
 = = 3 4 4 11 1 3 3 –1
5 –1 4 = 21 = 21 – 12 =
–3 3 2 2
= +
2 2 4 4 1 3 3 –1
a  ab  b 1 1–
2 2
 2 2 1
a – ab  b
25. (b) x + x + a2 1 3 3 –1
4 = 
1 2 3 3 3– 3
a   ab 2
2 1 2
= 2
a
1
=  x  2
8
 x a

1 3

3 1
a  – ab 3 3 1 3 3 1
a
2 2
[(A + 2BA + B ) = (A + B)²] 2
   
Rakesh Yadav Readers Publication Pvt. Ltd. 138

For More Visit : www.LearnEngineering.in


For More Visit : www.LearnEngineering.in

Alternate:  x + y + 3x 1/3.y 1/3.(x 1/3 +y 1/3) = z


1 1 2
=  =
3 3 3 1  x + y – z + 3x 1/3.y 1/3.z 1/3 = 0
 a +
b
= 1 ..... (i)
29. (d) x 2 – 4x – 1 = 0  x + y – z = –3x 1/3 y 1/3 z 1/3
x 2 – 1 = 4x (cubing again both sides)
1 b –1
(divide x both sides)  a = 1 –
b
=
b (x + y – z) 3 = – 27 xyz
1 (x + y –z) 3 + 27xyz = 0
x – = 4 1 b 1
x
a
=
b –1
 b+ c =1 a b
 1
36. (a)
1 b a
x2 + – 2 = 16 1
x2 1
= 1 – b, c 
a  0, b 0
c 1–b a 2 + b 2 = ab
1

ir
x2 + = 18 a 2 + b 2 – ab = 0
x2 1 1 b
 (a + b) (a 2 + b 2 – ab) = (a + b) ×0

v.iSn
 c +
a
=
1 – b b –1
[(multiply both sides by (a + b)]
 1  1  2 1 
30. ( d)  x  x  x – x   x  2 –1 1 b 1–b a3 + b3 = 0
     x  = – = = 1
1–b 1–b 1–b 37. (d) p = 99
 2 1  p(p 2 + 3p + 3)

dnag
 x  2 1 33. (c) x = 3 2
 x  = p 3 + 3p 2 + 3p + 1 – 1
1 = (p + 1) 3 – 1
 1  2 1   1  2 1   x = 3– 2
=  x  x  x  2 –1 x – x   x  2  1 = (100) 3 – 1 = 1000000 – 1
  x     x  = 999999

eYari
 1
x3 +
x3 38. (b) 5 x
 12 x  13 x
 A + B   A 2 – AB + B2  = A 3 +B3
3 By option put x = 4
 A – B   A 2 + AB + B2  = A 3 –B3  x3 =  3 2 
 5 4
 12 4
 13 4

 3 1  3 1  6 1
= x  3  x – 3  = x – 6
 x  

7 –2
x  x
=    
3

= 3 3 2 2 3 6
3
snhe
 2
3
 3 3  2



3 2
3 2


 

5 2 + 12 2 = 13 2
169 = 169
kgei
31. (b) = a 7 b hence, x = 4
7 2 = 3 3 2 2 9 2 6 3
39. (d) (x – 3) 2 + (y – 4) 2 + (z – 5) 2 = 0
x 3 = 9 3  11 2  (x – 3) 2 = 0 x = 3
7 –2 7 –2
L.H.S = × (y – 4) 2 = 0 y = 4
ERna

7 2 7 –2 1
(Rationalisation) 3 = 9 3 – 11 2 (z – 5) 2 = 0 z = 5
x
2
(x + y + z)  4 + 3 +5  12
 7 –2  74–4 7 x + 3
1
= 9 3  11 2 + 9 3 – 11 2 40. (c) x = 3 + 8
= 2 = x3
aBryn

 7 –  4 7–4
= 18 3 1 1
Alternate: =
11 – 4 7 x 3 8
=
3 x = 3 2
11 4 4 11 1 1 1 3 – 8
Les

– 7 = – 7 + 3 = 3– 2 = ×
3 3 3 x x 3 8 3 – 8
wa. th

= a 7+b = R.H.S 1
and x + = 3 2 + 3– 2
x 3 – 8
Compare the cofficients of 7 and = = 3 – 8
constant term = 2 3 9 –8
wwM

3
4 1  1  1 1
a = – x3  = x   – 3 x  
3 x3 x x x+ = 3+ 8 + 3– 8 = 6
  x
11 3
b =
3 = 2 3  
– 3 2 3  1
x + = 6
1 1 1 = 24 3 – 6 3 = 18 3 x
32. (c) a+ =1, b+ =1, c+ = ?
b c a 34. (c) x + y = 7 (cubing both sides) squaring both sides
Put values, (x + y) 3 = (7) 3
x 3 + y 3 + 3(x + y)xy = 343 1
1 x2 + 2 + 2 = 36
a = b = 2 c = –1 x 3 + y 3 + 21xy = 343 x
2
35. (c) x 1/3 + y 1/3 = z 1/3
1 (cubing both sides) 1
1 x2 +
c+ = –1 +  1  = –1 + 2 = 1 1/3 3 1/3 3 2 = 34
a 2  x1/3  y  = z   x

Rakesh Yadav Readers Publication Pvt. Ltd. 139

For More Visit : www.LearnEngineering.in


For More Visit : www.LearnEngineering.in

1 3x  3–x 2 x 9 1
41. (b) x – = 4 44. (b) =  =
x 3x – 3–x 1 4 9 –1
(by c–d rule )
1 x 10
x2 + 2 – 2 = 16 3 x 2 1 3  =
x 4 8
 = =
3– x 2 –1 1
(On Squaring) 10
 x = × 4 = 5
1  A C  8
 = 
 x2 + 2 = 18 B D 1
x  
A +B C +D 47. (c) x =  2 1 3
 = 
1  A –B C –D  Take cube on both sides
x2 + 2 + 2 – 2 = 18

r
x 3x  x3 = 2 + 1
= 3

Sni
1 3–x 1 1 2 –1
x2 + 2 + 2 = 20 Squaring both sides  x
3 =
2 1
×

gv.i
x 2 –1
3x
2 = 9 2 –1
x 1 3–x =
   = 20 3 + x = 27 – 9x 1
x

ridna
  10x = 24
1
24 12 3 = 2 – 1
1 x = = x
x+ = 20 10 5
x

eeYa
1
3
= 4 5 =2 5 45. (d) x =  x3 – 3 = 2 +1– 2 +1=2
2 x
1 2
42. (a) 4b 2 + = 2 1 x  1– x 1 x  1– x x – x 1 3
2 × 48. (b) = Given
b 2
1 x – 1– x
geisnh 1 x  1– x x  x 1 2
2
1 2
  
(2b) 2 + b
 
+ 4 – 4 = 2  1 x  1 – x  x  x 
1
 – 1
= 2 2  x
  3
 1 x  – 1– x   
2  1  2
Enak

 2b+ 1  x  x    1
 
b
– 4 = 2  x 
 1 x 1– x  2 1– x
2
=
2 1 x –1 x x 1 
R

 2b+ 1  Let    y
 
b
= 6  x 
 2 2
22 1– x 1 1– x
= =
aryn

1 2x x x  1  – 1
2b + = 6  
x 
b 
Les B

3
Take cube both sides 3  x  1  1 =
1 1– 2
3 = 4 × 2  
x 
 2b+ 1 
  = ( 6 )
3 3
 b
y–1 3
wa. th

=
1 1  2b+ 1 1  1 3 y+1 2
8b3+ 3 +3×2b×  = 6 6  
2 × 2 = 2
2
b b  b =  = 3  2(y – 1) = 3(y + 1)
3 3 2y – 2= 3y + 3
1
y = – 2 – 3 = – 5
8b 3 + 3 + 6 6 = 6 6
wwM

b x  4  x – 4 2 1
46. (d) =  x = – 5
1 x  4 – x – 4 1 x
8b 3 + 3 = 0
b by C – D rule 49. (c) x = 3 + 8
3 –6 2x –1
3 3 3 x 4 2 1 3 2
x = 9 + 8 + 2 × 3 8
43. (c) 5 5 = 5  = =
      x –4 2 –1 1
2
x = 17 + 6 8
3–6 2x –1 2
3 3 2
 5 = 5  x 4  3 1
       = 1 2 = 17 – 6 8
 x –4   x
 – 3= 2x – 1
1
 2x = – 2 x4
x2 + = 17 + 6 + 17 – 6
 = 9 again C & D rule 2 8 8 = 34
 x = –1 x –4 x

Rakesh Yadav Readers Publication Pvt. Ltd. 140

For More Visit : www.LearnEngineering.in


For More Visit : www.LearnEngineering.in

50. (c) x = 5 + 2 6 9 1
x = 56. (a) a + + 1 = 0
x = 3+ 2 + 2 3 × 2 x a
x2 = 9
2 2 1
x =    
3 + 2 +2 3 × 2 9 9
a +
a
= –1
Hence x 2+ 2 =9+ = 10
2 x 9 Squaring both sides
x =  3  2 
1 1
x = 3 + 2 53. (b) 2p + = 4  a2 + 2 + 2 = 1
p a
Similarly
Divide by 2
1
1  a2 +
2p 1 4 2 = – 1
 = 3 – 2 a

ir
x + =
2 2p 2

v.iSn
–1
1  a2 + 1 = .....(i)
1 2
x + = 3 + 2 + 3 – 2 a
x p + = 2
2p
= 2 1
3 Take cube on both sides  a + = – 1 (Given)

dnag
a
51. (d) x = 3 + 2 3 a2 + 1 = – a
 p+ 1  .....(ii)
x2 = 3 + 2 + 2 6
  
2p 
= (2) 3
–1

 – a = 2 fr om equat ion (i)
a

eYari
2
x = 5 + 2 6
1 1  p+ 1  and (ii)
1 p 3+ 3 +3×p× 2p  2p  = 8 a3 = 1
8p  
2 = 5 – 2 6  a³ – 1 = 0
x
1 3
1
x 2+ 2
x
p3 +
8p

1
snhe
3 +
2
× 2 = 8

57.
 a4 – a = 0 × a = 0
(Multiply a both sides)

(c) x = a +
1
=5+2 6 +5 – 2 6 p3 + a
kgei
3 = 8 – 3 = 5
8p
= 10
1
54. (a) a 6 + b 6 = (a 2) 3 + (b 2) 3 y = a –
9 = (a 2 + b 2) (a 4 – a 2b 2 + b 4) a
52. (c) x + = 6
ERna

x  a 6 + b 6 = (a 2 + b 2) × 0 = 0
1 1
Take values of x  (x + y) = a + + a – = 2a
Let x = 3 1 a a
55. (c) x + = 3 (Given)
x
9 1 1 2
3 + = 6  (x – y) = a + – a + =
aBryn

3 3 1 a a a
x 
Prove So, x = 3 x (Divide by x)  x 4 + y 4 – 2x 2y 2 = (x 2 – y 2) 2
2  ((x + y) (x – y)) 2
9 9 x – x 1
 x2 + 2 = 9 + = 10 2
x 9 3  2 
x 1 21   2a  
Les

Alternate:  x  a
x 2 x
2  
x
wa. th

9  2 =
1
2
= (4) = 16
x + = 6 x x 1 x –1
x –  58. (a) a = 11
x x x x
b = 9
On squaring
1 2 2
2 2 a  b  ab
 9 x  
wwM

2 3 3
x 
x
 = 36 x a –b
  = 1
x  –1  (a3 – b3) = ((a – b) (a2 + ab + b 2))
x
81 9 2 2
a  b  ab
x 2+ 2 +2 × ×x = 36 1
x x
 x +
x
= 3  a – b  2
a  ab  b
2

81
x2 + 2 – 18 = 0 1 1 1 1
x  x2 + 2 = 9 – 2 = 7 = = =
x a –b 11 – 9 2
2 59. (a) p = 101
x –
9 2 1
 x
 = 0 x 
2 = 3 p(p 2 –3p+3)–1
  x 7 7
 1
=
3 –1
=
2
x  –1 = 3 p 3 –3p 2 +3p–1
x

Rakesh Yadav Readers Publication Pvt. Ltd. 141

For More Visit : www.LearnEngineering.in


For More Visit : www.LearnEngineering.in

 [(p – 1) 3 = p3 – (1) 3 – 3p (p – 1)] 64. (b)


2x–y 1
(Cross Multiply)
4 3 5 2  4 3 –3 2 
= 3 (p–1)3 x+2y 2

48 – 18
= p – 1 = 101 – 1 = 100  4x – 2y = x + 2y
60. (a) x = 19 y = 18 3x = 4y 8 6  18 8 6 18
= = +
x : y = 4 : 3 30 30 30
2 2
x  y  xy
3x – y 34–3
3 3  = 4 3
x – y =
3x  y 343 6 +
15 5
2 2
x  y  xy 12 – 3 9 3 3 4
= 2 2 = = =
12  3 15 5 = + 6
( x – y )( x  y  x y) 5 15

r
1 1
1 3 4

Sni
65. (b) x + = 5
=
x –y
= = 1 x  + 6 = a + b 6
19 – 18 5 15

gv.i
61. (b) 50% (p – q) = 30% (p + q) 2x By comparing cofficients of ratio-
 2 nal and irrational parts.
p–q 3 3x – 5x  3
= (p + q) 3 4
2 10 (Divide by x)

ridna
 a= b=
5 15
 1 2x
2
50% 
2  
x  3, 4 
2 3  5 15 

eeYa
3x 5x 3 3x  –5  
 5(p –q) = 3 (p + q) –  x
x x x
 5p – 5q = 3p + 3q x 32
2 68. (a) 1 =
 2p = 8q 2 961 31
= =
 1 
 1p = 4q 3 x
geisnh
 –5 35 – 5 (Squaring both sides)
x

 p : q  
x 1024
 4 : 1
 1+ =
2 1 961 961
 
10 5
a b a 3
Enak
961  x 1024
62. (a)
3
=
2
 b =
2  =
3 3 961 961
x
66. (b) 1– =
100 5 x = 1024 – 961 = 63
2a  3b 23 32
R

=
3a – 2b 33 – 22 2 a c e 3
3 3 69. (d) = = =
x
 1– = 5 b d f 1
66 12  
aryn

100
= =
9–4 5 29  39  49
9 3 
Les B

x
a b c  1– = 2 1  3 1  4 1
63. (d) + + = 1 25 100
1–a 1–b 1–c 18  27  36 81
16 3 = = = 9
Add 3 both sides x
 = 234 9
25 100
wa. th

a b c
 + + +3=1 + 3 1
1–a 1–b 1–c 16  100 70. (d) 2x + = 5
 = x³ 3x
25
 a   b   6x 2 + 1 = 15x
 1 – a  1 + 1 – b  1 +  16 × 4 = x³
   
wwM

 4=x 5x
 2
 c  6x  20 x  1
 1 = 4 4 3 5 2
1 – c 67. (d) = a + b
  48  18
6
5x 5x 1
= = =
15 x  20 x 35 x 7
a 1 – a  b 1 – b  4 35 2
  1–a  +  1–b  + 
    16  3  9  2 1
71. (a) x  y2 – 1 (Given)
c 1–c  4 35 2
  = 4 
 1–c  4 33 2
1
4 3 5 2 4 3 –3 2 x = k × 2
1 1 1 y –1
 + + = 4  ×
1–a 1–b 1–c 4 33 2 4 3 –3 2 (k is constant)

Rakesh Yadav Readers Publication Pvt. Ltd. 142

For More Visit : www.LearnEngineering.in


For More Visit : www.LearnEngineering.in

Now x = 24 when y = 10 given  2a – 3b + 4c 78. (b) 1.5x = 0.04y


= 2 × 1 – 3 × (–1) + 4× (–1)
1 x 0.04 4 10 2
= 2 + 3 – 4 = 1
 24 = k × 2  y = = × =
(10) – 1 75. (a)(3a + 1) 2 + (b – 1) 2 + (2c – 3)² = 0 1.5 100 15 75
 (3a + 1) 2 = 0
k 2 2
 24 =  3a = –1 y –x y – xy  x
99  2 2 =
k = 24 × 99 –1
y  x  2xy  y  x 2
 a =
 x = ? 3 y –x 75 – 2 73
y = 5 (b – 1) 2 = 0  b –1 = 0  yx
= =
75  2 77
1  b = 1 79. (b) a 1/3 = 11, a = 11 3 = 1331
x = 24× 99× a 2 – 331a = a (a – 331)
(2c – 3) = 0

ir
25 – 1
= 1331 (1331 – 331)

v.iSn
1 3 = 1331× 1000 = 1331000
= 24× 99× x = 99  c =
24 2
1 1
72. (a) x 2 + y 2 + 2x + 1 = 0 –1 3 80. (a) x2 + y2 + 2 + y2 = 4
 3a +b+2c =3× + 1+ ×2 x
 x 2 + 2x + 1 + y 2 = 0 3 2

dnag
(x + 1) 2 + y 2 = 0 Take x = y = 1
= – 1 + 1 + 3 = 3
Hence both terms are squares and
1 1
there addition is zero so, it can be 2 2 1 + 1 + + = 4
(a–b) (b–c)
possible only when both t er ms 76. (b) + + 1 1
(b–c)(c–a) (a–b)(c–a)

eYari
are zeros. Hence
 x+1=0 2
x2 + y2 = 1 + 1 = 2
x=–1 y = 0 (c–a) 81. (b) x 2 = y + z ........(i)
(a–b)(b–c) x2 + x = y + z + x
 x 31 + y 35
add x on both sides

73.
=

(b)
(–1) 31 + (0) 35 = –1

2x
2
x
 5x  2
=
1
6
Now

 b –c c –a
 
snhe
a – b 

2
×
a – b 
a –b

x (x +
y2 = x + z
1) = x + y + z

y 2 + y = x + y +z
........(ii)
kgei
add y on both sides
x Multiply divide by (a – b) in Ist term y (y + 1) = (x + y +z)
x  (b – c) in IInd term  z2 = y + x ........(ii)
1
2
x 5x 2 =  (c – a) in IIIrd term z2 + z = x + z + y
ERna

2   6
x x x add z on both sides
a – b 2 a – b  z (z + 1) = x+ y + z
 +  x (x + 1) = x + y + z
1 b – c  c – a  a – b 
1
2 x 1
aBryn

 2x  5 = 6
x b – c 2 b – c  x yz
=
x 1
+
a – b  b – c  c – a  y 1
x 
1
 2  x
 +5 = 6 2 x y z
=
y 1
  c – a  c – a 
Les

x 1
a – b  b – c  c – a  z 1
wa. th

 6 – 5 = 2    =
 x  Let a – b = x x y z z 1
b – c = y By adding them
x 1 c – a = z x y
 1 = 2   
 x   x+y+z=0 =
x yz
+
x yz
+
wwM

 x 3 + y 3 + z 3 = 3xyz
1
1  (a – b) 3 + (b – c) 3 + (c – a) 3 z 1 1 1
 x + =
= 3 (a – b) (b – c) (c – a) x yz
= +
y 1
+
x 2 x 1 z 1
74. (c) a 2 + b 2 + c 2 = 2(a – b – c) – 3
3 a – b  b – c  c – a  x yz 1 1 1
 a2 + b2 + c2 = 2a – 2b – 2c – 3  3 = = + + =1
a – b  b – c  c – a  x yz x 1 y 1 z 1
 a 2 + b 2 + c 2 – 2a + 2b + 2c + 1
77. (c)(a – 3) 2 + (b – 4) 2 + (c – 9) 2 = 0 Alternate:-
+ 1 + 1 = 0
a – 3 = 0 a = 3 x = y = z = 2
 (a 2 – 2a + 1) + (b 2 + 2b + 1) +
(c 2 + 2c + 1) = 0 b – 4 = 0 b = 4 1 1 1 1
c – 9 = 0 c = 9  + + = +
 (a – 1)2 + (b + 1)2 + (c + 1)² = 0 2 1 2 1 2 1 3
a = 1  a b c = 349 1 1
b = – 1 + = 1
c = –1 = 16 = ±4 3 3

Rakesh Yadav Readers Publication Pvt. Ltd. 143

For More Visit : www.LearnEngineering.in


For More Visit : www.LearnEngineering.in

82. (d) a2 + b2 = 2 b 2 + c 2 = a 2 – 2bc


c2 + d2 = 1 x y xy
86. (a)x – y = = = k (let) a 2  b2  c 2
Put values of a, b, c, d 7 4

Take a = b = 1 x – y = k ..........(i) a 2 – bc
c = 1 x + y = 7k .........(ii)
d = 0 a 2  a ² – 2bc
xy = 4k .........(iii) 
 (ad – bc) 2 + (ac + bd) 2 a 2 – bc
 x – y = k .....(i)
 (0–1)² +(1+ 0)²
x + y = 7k ...(ii)

2a 2 – 2bc


2 a 2 – bc  = 2
 (–1) 2 + (1) 2 = 2
x = 4k a 2 – bc a 2 – bc
4ab y = 3k 90. (c)x 2 + y 2 – 4x – 4y + 8 = 0
83. (d) x = x 2 + 4 – 4x + y 2 + 4 – 4y = 0
a b
 xy  4k × 3k = 12k² (x – 2) 2 + (y – 2) 2 = 0

r
x 2b 12k 2 = 4k x – 2 = 0, y – 2 = 0

Sni
 = x = 2 , y = 2
2a a b
1 4  x–y=2–2=0
k =  xy = 4k =

gv.i
x  2a 2b  a  b 3b+a 3 3 91. (a)x = b + c – 2a
= = y = c + a – 2b
x – 2a 2b – a – b b–a 2 2
x 2 z z = a + b – 2c
(By C – D rule) y
87. (d) + + = 0  x + y + z = ( b + c – 2 a ) +

ridna
yz zx xy
 again
(c + a – 2b ) + (a + b – 2c)
x 2a x
2
x y
2
y z
2
z
 Now
= = × + + × =0 = x 2 + y 2 + 2xy – z 2
2b a b

eeYa
yz x zx  y xy z = (x+ y)2 – z2 (A2 – B2 = (A + B) (A –B)
x  2b 2a  a  b 3a  b (x + y – z) (x + y + z)
= = 3 3 3 = As we know (x + y + z) = 0
x – 2b 2a – a – b a –b x y z
= = 0  x2 + y2 – z2 + 2xy = 0 × (x + y – z) = 0
xyz
x  2a x  2b 3b  a 3a  b 92. (b)a 2 + b 2 + c 2 = ab + bc + ca
 + = +
geisnh
 If x + y + z = 0
x – 2a x – 2b b –a a –b take value a = b = c = 2
then, x 3 + y 3 + z 3
3b  a – 3a – b a c 22
 3xyz)  = = 2
 b 2
b –a
3xyz
Enak

2 b – a   xyz
= 3 1
2b – 2a 93. ( a) x + = 3 ( t a k e cu b e on b ot h
 =
b – a  = 2
88. (b) a + b + c = 0
x
b –a
sides)
R

1 1 1
+ + 3
84. (c) m + = 4 x 1  3
m –2 (a  b )(b  c ) (a  c )(b  a )



x


  3
aryn

1 1
 (m – 2) +
m – 2 = 2 (c  a )(c  b ) 1 1 x  1 
Les B

3
x   x
3 + 3x  = 3 3
 Squaring both sides x x  
(a  c )  (b  c )  (a  b )
1  (a  b )(a  c )(b  c ) 1
3
(m – 2) 2 +
m – 2 2 + 2 × (m – 2) ×  x 
x
3 +3  3 = 3 3
wa. th

2(a  b  c )
1  (a  b )(a  c )(b  c )
= 0
1
3
= 4 x  = 0
m – 2 (  a + b + c = 0) x
3
89. (c) a + b + c = 0 x6 = – 1
1 
wwM

Assume values a = 2 b = –2 c = 0
2  x 18 + x 12 + x 6 + 1
(m – 2) +
m – 22 = 2
a+ b + c = 2 – 2 + 0 = 0(satisfy)
= (–1) 3 + (–1) 2 + (–1) + 1
85. (c) a 2 + b 2 + 2b + 4a + 5 = 0 2 2 2 = –1 + 1 – 1 + 1= 0
a b c
a 2 + b 2 + 2b + 4a + 4 + 1 = 0 94. (c) ax 3 + 3x 2 – 8x + b is divisible
 2
a² + 4a+ 4 + b² + 2b + 1 = 0 a – bc by (x +2) and (x–2)
(a + 2) 2 + (b + 1) 2 = 0  (x + 2) and (x – 2) are factors
a + 2 = 0 a = –2 440 8
b + 1= 0 b = – 1   = 2  x + 2 = 0  x = –2
4–0 4
x – 2 = 0 x = 2
a –b –2  1 Alternate:-
 a + b + c = 0 Put x = –2
a b –2 – 1
b + c = – a  a (–2)3 + 3 (–2)2 – 8 (–2) + b = 0
–1 1 Squaring both sides = – 8a + 12 +16 + b = 0
 = (b + c ) 2 = a 2 – 8a + b + 28 = 0
–3 3 – 8a + b = – 28............. (I)
b 2 + c 2 + 2bc = a 2

Rakesh Yadav Readers Publication Pvt. Ltd. 144

For More Visit : www.LearnEngineering.in


For More Visit : www.LearnEngineering.in

and
Put x = 2 x y 1
= x²   2  11  2
 3
a(2) + 3(2) – 8 × 2 + b = 0 2 xy x²
 8a + 12 – 16 + b = 0 xy = (x + y)² 2
x 2 + y 2 + 2xy = xy
 1
8a + b – 4 = 0 x    13
8a + b = 4 ......(II) x 2 + y 2 + xy = 0  x 
From equation (I) & (II)  x 3 – y 3 = (x – y) (x 2 + y 2 + xy)
(x 3 – y 3) = (x – y) × 0 = 0 1
 – 8a + b = –28 98. (c)x = a (b – c) x  = 13
x
8a + b = 4 y = b (c – a)
2b = –24 z = c (a – b)  Taking cube both sides
b = –12
x x 3 1 3

ir
a=2 Let
a
= b – c
a
= A x 
x
3 + 3 13 =  13 

v.iSn
95. (b)x 2 – 3x + 1 = 0
x 2 + 1 = 3x y y 3 1
Divide by x = c – a = B x 
b b 3 + 3 13 = 13 13
x
2 1
x
z z

dnag
= + 1
x x = a – b = C 3
c c x 
3 = 10 13
x
3x  A +B+ C = b– c +c– a+a – b = 0
  A 3 + B 3 + C 3 = 3 ABC 1
x 101. (b) 3 x  = 5

eYari
3 3 3 2x
1 x y z
x  = 3  a  +  + 
x   b  c  2
Cubing both sides
 Multiply both sides by
3
x y z

 x
3

1
3 + 3x 
x
1
x

x


1
 = 27
x
= 3×

=
3 xyz
a snhe
×
b
×
c 2
 3x × 3 + 2 x × 3 = 5 ×

10
1 2 2
3

1 1
kgei
3 abc 2x  =
 x 
3
+3×3 = 27 3x 3
x 99. (c)xy (x + y) = 1
 Taking cube on both side
1 1
ERna

x3  = 18 x + y = 1 1  1 
xy 8x 3   2x  3x 
x3 27x 3
+ 3×2x ×
3x  
Cubing both sides
1 3 3
96. (a) x   1  10 
4x 2 =  
aBryn

 (x + y) 3 = 3 3  3 
Multiply by 2 both sides x y

1 1 10 1000
2x  1 8x ³   2 
 2x
= 3
x³ + y³ + 3xy (x + y) = 3 3 27x ³ 3 27
x y
Take cube both sides
Les

1 1000 20
3 1  1  8x 3  = –
 2x 1 27 x 3 27 3
wa. th

=    = (3) 3 x³ + y³ + 3 = 3 3  x  y  xy 
2x  x y  
 1000 – 180 820 10
= = = 30
1 27 27 27
3 1 1 2x  1 
= 8x  3 +3×2x×   3 3 – x³ – y³ = 3 102. (a) x + y = z
2x  x y

wwM

8x 2x
x + y – z = 0
= 27 1 If a+ b+ c = 0
4
100. (d) x 
4 = 119 x > 1 then a³ +b³ + c³ –3abc = 0
3 1 x
= 8x  + 3 × 3 = 27  x 3 + y 3 – z 3 = –3xyz
3
8x
4 1  x 3 + y 3 – z 3 + 3xyz = 0
 x 
3 1 4 +2 = 119 + 2 = 121 3xyz – 3xyz = 0
8x  x
3 = 27 – 9 = 18
8x a b
2 103. (c) + = 1
1 1  2 1  b a
1 x   = (11) 2
97. (a) = + 2 a 2 + b 2 = ab
x y x y  x  
 a 2 + b 2 – ab = 0
1 2 1  a 3 + b 3 = (a + b) (a 2 – ab + b 2)
 xy x 
2 = 11 = (a + b)× 0 = 0
x

Rakesh Yadav Readers Publication Pvt. Ltd. 145

For More Visit : www.LearnEngineering.in


For More Visit : www.LearnEngineering.in

118. (c) Put a = b = 1 and c = – 2


104. (c) x = 2 – 21/3 + 2 2/3 112. (d) (a – 1) 2 + 3 = b 2 + a we get a + b + c = 1 + 1 – 2 = 0
x – 2 = 2 2/3 – 2 1/3 ........ (I) 0 = 0 (satisfy)
Comparing Cofficient of 2 &
Take cube both sides
constant terms. a ²  b²  c ²
(x – 2) 3 = (2 2/3 – 2 1/3) 3 
 a=3 a ² – bc
x 3 – 8 – 6x(x – 2) = (2 2/3) 3 – (2 1/3) 3 –
3×2 2/3.2 1/3 (2 2/3 – 2 1/3)  a–1=b 12  12   –22 6
x3 – 8 – 6x2 + 12x = 22 – 2 – 3 – 1 = b = 
2
1 – 1  –2 3 = 2
b = 2
21
(x – 2) a + b = 3 + 2 = 5 119. (b) n = 7 + 4 3
32 3
113. (c) ax 2 + bx + c = a (x – p)²
From equation (I) ax 2 + bx + c = a(x 2 + p 2 – 2px) n = 4 + 3 + 4 3
 x 3 – 8 – 6x 2 + 12x = 4 – 2 – 3 × 2  ax 2
+ bx + c = ax + ap – 2apx 2 2
2
 3

r
(x – 2) n = (2) 2 + + 2 × 2 × 3
Comparing cofficients of x² and x
x³ – 8 – 6x² + 12x = 2 – 6x + 12

Sni
x³ + 18x – 6x² – 8 – 14 = 0  b = – 2ap
2
x ³ + 18x – 6x 2 – 22 = 0 –b
n = 2  3 

gv.i
 x3 – 6x² + 18x + 18 = 22 + 18 = 40  p = 2a .......(i)
105. (d) a 3 – b 3 – c 3 – 3abc = 0 n  2  3
and c = ap 2
 a–b–c=0
1

ridna
a = b + c b2 2– 3
 c = a × (From (i)) 
106. (c)a = 2.361 2 n
4a
b = 3.263
1
c = 5.624 4ac = b2
n + =2 3 2– 3 = 4

eeYa
a + b – c = 0 114. (d) a + b + c + d = 1 n
2.361 + 3.263 – 5.624 = 0 (1 + a) (1 + b) (1 + c) (1 + d)
120. (b) x = 3 2
 a3 + b 3 – c 3 + 3abc 0  For maximum value a, b, c, d
107. (d) p = 124
1 1 1 3– 2
geisnh
a = b = c = d = = × = 3 – 2
4

3 p p2  3 p  3  1  4
x 3 2 3– 2

 1   1  1  1  5 1
= 1 4  1 4 1 4 1  4  =  
= 3 p3  3 p2  3 p  1         4 x +
x
= 3  2  3 – 2 = 2 3
115. (b) a 2 + b 2 + c 2 + 3 = 2(a + b + c)
Enak

121. (d) p + q = 10 .... (i)


3 3 a 2 + b 2 + c 2 + 3 = 2a + 2b + 2c
= 3  p  1 = 3 125  = 125 and pq = 5
a 2 – 2a + 1 + b 2 – 2b + 1 + c 2 – 2c +
1 = 0 Squaring both sides of equation (i)
1 (a – 1) 2 + (b – 1) 2 + (c – 1)² = 0 (p + q) 2 = (10) 2
R

108. (c) x + = 2 a = 1 p 2 + q 2 + 2pq = 100


x
b = 1 p 2 + q 2 + 2 × 5 = 100
(assume x = 1, so,1 + 1 =2)
c = 1 p 2 + q 2 = 90
aryn

1 (a + b + c)  1 + 1 +1 = 3 Now,
1
19
Les B

x 17 + 17
19 = (1) + 1 2 2
x 1 116. (c) x – = 5
p q p q 90
x  q+p= pq
= =18
5
= 1 + 1 = 2
109. (c) x : y = 3 : 4 1
x2 + – 2 = 25 1
x2 122. (d) x = 3 + 2 2 , xy = 1, y² =
5 x – 2y
wa. th

53 – 24 x²
 7 x  2y = 1
73  2 4  x2 + = 27
x2 1 1
y = = = 3–2 2
15 – 8 7 117. (b) x = 3 + 2 2 x 32 2
= =
21  8 29
x = 2 + 1 + 2 2
wwM

1
110. (a) x + y = 2z 2  x+ = 3+ 2 2 +3– 2 2 = 6
x
x – z = z – y  2 + (1) 2 + 2 × 1 × 2
x – z  – (y – z) ........(i) 2 2 1
x =  2 1   x 
2 = 36 – 2 = 34
z x
x x z
+ = – 2
x –z y –z x –z x –z x =  2 1  1
3 x
2

2 2 2
x –z x  3 xy  y x
x = 2 1 = 2
= = 1 2 = 2 1
x –z x – 3 xy  y x  –3
1 1 2 –1 2
111. (b) a³b = abc = 180   = 2 –1 x
x 2 1 2 –1
or a =1, b = 180
then c = 1 1 34  3 37
 x – = 2  1 – 2 1 = 2 = =
 b = 180 x 34 – 3 31

Rakesh Yadav Readers Publication Pvt. Ltd. 146

For More Visit : www.LearnEngineering.in


For More Visit : www.LearnEngineering.in

x y z 1  ab = 8
123. (a) = = (given) 127. (b) x  = 2, x 0 (a – b) = 2
b c c a a b x (a – b) 2 = a 2 + b 2 – 2ab = 4
put x = 1 = a 2 + b 2 = 4 + 2ab
x y
 b c = 1 + 1 = 2 a 2 + b 2 = 4 + 2 × 8 = 20
c a 133. (b) (a 2 + b 2) 3 = (a 3 + b 3) 2
1
x x –y x –y  x2 + = 1 +1 = 2  a 6 + b 6 + 3a 2b 2(a 2 + b 2) = a 6 + b 6
x³ + 2a 3b 3
 b  c = b  c – c  a = b – a
a b  a 6 + b 6 + 3a 4b 2 + 3a 2b 4 = a 6 + b 6
Similarly 128. (a) + = 1 + 2a 3b 3
b a
y z  3a 4b 2 + 3a 2b 4 = 2a 3b 3
= 2 2
c a a b a +b  a 2b 2 (3a 2 + 3b 2) = 2a 3b 3
= 1 3a 2 + 3b 2 = 2ab

ir
y y –z y–z ab 3 (a 2 + b 2) = 2ab

v.iSn
 c  a = c  a – a b = c – b again a 2 + b 2 – ab = 0
a 3 + b 3 = (a + b) (a 2 – ab + b 2) a 2  b2 2
  =
z x = (a + b) × 0 = 0 ab 3
 a b =
b c
2 a b 2
 b+a =

dnag
z –x
x 
1 3
z z –x 129. (a)  x
 = 3
 a  b = a  b – bc = a – c   1 1
134. (a) x + = 5 x2 + +2 = 25
x x2
1
x –y y –z z –x x + = 3 1

eYari
 b –a
=
c –b
=
a –c
x x2 + = 23
x2
124. (d) x – y = 2, xy = 24 (giv en) 1
x 2 + y 2 – 2xy = 4  x3 + 3 + 3 3 = 3 3 x 4  3x 3  5x 2  3x  1
x 2 + y 2 – 2 × 24 = 4 x now,
x4 1
x 2 + y 2 = 4 + 48 = 52

125. (a)
x
y
2
2 + tx +
y
4
2
(giv en)


3
x 
x
1

x6 + 1 = 0
snhe
3 = 0
divided by x 2,

=
x 4 3x 3 5 x 2 3x
x2
 2  2  2  2
x x x
1
x
x4
kgei
 x6 = – 1 1
T o m a k e i t a p er f e c t s q u a r e i t 
should be in the form x2 x2
 x 72 + x 66 + x 54 + x 24 + x 6 + 1
A² + 2AB + B² = (A + B)² 3 1
12 11 9 4 x 2  3x  5  
x 6   6 x6   6
ERna

2 2  + x + + x x x2
x y =
1
= y + tx + 2 + x6 + 1 x2 
     (–1) 12 + (–1) 11 + (–1) 9 + (–1) 4 + – x2
= A² + 2AB + B² 1 + 1 1  1
x2   3x    5
aBryn

x y  1–1–1+1–1+1=0 x2  x
A = , B = & 2AB = tx =
y 2 1 2 1
130. (b) a + = x  2
a 3 x
x y 6
a = – 1
So, tx = 2 × × 23  3  5   5 43
y 2  
1 1 23 23
Les

tx = x  a 6 – a 6 + 2 = –1 –  –1 + 2 1
wa. th

t =1 = –1 + 1 + 2 = 2 135. (b) x 3 + = 0
126. (d) a = x + y x3
131. (b) x 3 + y 3 = 35
b = x – y 1
3
1  1
 x+y=5 
c = x + 2y  x  x  – 3x × x  x  x  = 0
Take cube on both sides,  
1 (x + y) 3 = (5) 3 3
wwM

a² + b² + c² – ab – bc – ca = ((a – x 3 + y 3 + 3xy(x + y) = 125  1  1


2 x  x  – 3 x  x  = 0
35 + 3xy(5) = 125    
b) 2 + (b – c) 2 + (c – a) 2)
15xy = 125 – 35 3
1 15xy = 90  1  1
= ((x + y – x + y)² + (x – y – x –2y)² xy = 6 x  x  = 3 x  
2    x
+ (x + 2y – x – y)²) 1 1 x y 5 2
 x y =  1
xy = 6 x  x  = 3
1  
= ((2y)² + (–3y)² + y²) 3
132. (b)a – b = 56 3
2 2 2
 a–b=2  1 
1   x    = (3) 2
= (4y² + 9y² + y²)  a3 – b 3 – 3ab (a – b) = (2) 3
 x 
2 (By cubing)
4
56 – 3ab × 2 = 8  1
1 – 6ab = 8 – 56 x  x  = 9
= (14y²) = 7y²  
2 6ab = 48

Rakesh Yadav Readers Publication Pvt. Ltd. 147

For More Visit : www.LearnEngineering.in


For More Visit : www.LearnEngineering.in

1 + 4 + 9 = 14
1 1
3
1
3
1  1 1 1
3
136. (c) x + = 3 1 + 8 + 27 = 36 3  4  5  – 3 3  4  5
x      
 abc = 1 × 2 × 3 = 6 = 2 2 2
(Squaring both sides) 1  1  1 – 1  1  1  1  1  1
Alternate: 3 4 5 3 4 4 5 5 3
       
1  (a + b + c) 2 = a 2 + b 2 + c 2 + 2(ab
x2 + = 7 + bc + ca)  A 3 + B 3 + C 3 – 3ABC = (A 2 + B2
x2
36 = 14 + 2(ab + bc + ca) + C2 – AB – BC – CA) (A + B + C)
On cubing both sides
(ab + bc + ca) = 11
1 1 1 1
x³   3x   x    27  a3 + b3 + c3 – 3abc = (a + b + c) (a 2 Let = A
x³ x x + b + c 2 – ab – bc – ca)
2 3

1  36 – 3abc = 6 (14 – 11)


1
 x³   3  3  27 = B
x³  36 – 3abc = 6 × 3 4

r
–3abc = 18 – 36
1

Sni
x³   18 3abc = 18 1
x³ = C
abc = 6 5

gv.i
140. (a) a + b = 1
 1  1 
  x ³  x ³   x ²  x ²   18  7 By cubing  A+B+C  (A 2 +B2 +C2 –AB–BC –CA)
a³+b³ +3ab (a + b) = 1³   A2 +B2 +C2 –AB–BC–CA 
a³ + b³ + 3ab = 1 (a + b=1)

ridna
 5 1   1
x  5    x    126 a³ +b³ + 3ab = k =A + B + C
 x   x From above both equations
k = 1 1 1 1
 5 1   A + B + C =
3
+
4
+
5
x  5   3  126 141. (d) a = 34, b = 33, c = 33

eeYa
 x   a 3 + b 3 + c 3 – 3abc = (a + b + c) ×
20  15  12 47
 5 1  1  = =
 x  5   123 a – b 2  b – c 2  c – a 2  60 60
 x  2 
geisnh 1 a b c
1 = (34 + 33 + 33) ×
137. (a) m 4 + = 119 2 145. (d) + + = 1
m4 1–a 1–b 1–c
 34 – 33 2   33 – 33 2   33 – 34 2  Adding 3 on both sides
1  
m4 + + 2 = 119 + 2
m4 b
Enak
a c
1 + + +3 = 1 + 3
1 
2 = 100 × (1 + 0 + 1) 1–a 1–b 1–c
 2 2
 m + 2   121
 m  = 100 × 1 = 100 a b c
R

(a) 2x.2 y +1+ +1+ +1 = 4


1 142. = 8  2 x+y = 2 3 1–a 1–b 1–c
m2 + = 11 x + y = 3 .....(i)
m2
aryn

9 x .3 y = 81 a 1–a b+1–b c+1–c


1 3 2x .3 y = 3 4  + + = 4
m² + – 2 =11 – 2 1–a 1–b 1–c
Les B

m²  2x + y = 4
2 .....(ii) 1 1 1
 1 + + = 4
m – m   9 from equation (i) and (ii), 1–a 1–b 1–c
 
x+ y = 3 146. (c)a 2 + b 2 + c 2 = 2 (a – b – c) – 3
wa. th

1 2x + y = 4
m–  3 – – –  a 2 + b 2 + c 2 – 2a + 2b + 2c + 1 + 1 +
m –x = –1 1 = 0
138. (d) x + y + z = 6
(x – 1) 3 + (y – 2) 3 + (z – 3) 3
x = 1  a 2 – 2a + 1 + b 2 + 2b + 1 + c 2 + 2c +
y = 2 1 = 0
 as x + y + z = 6
wwM

Take values
 (x, y) = (1, 2)
143. (b) x 4 – 2x 2 + k  (a – 1) 2 + (b + 1) 2 + (c + 1) 2 = 0
x = 1, y = 2, z = 3
 (A + B)² = A 2 + 2AB + B 2 a = 1, b = – 1, c = –1
(1 + 2 + 3) = 6
 (1 – 1) 3 + (2 – 2) 3 + (3 – 3) 2 = 0 (A – B) 2 = A 2 – 2AB + B 2  2a – 3b + 4c = 2 × 1 – 3 × –1 + 4
 (x 2) 2 – 2 × x 2 + k ×– 1
Now assume values in options.
option 'd' satisfies the given (A) 2 – 2 × AB + B 2 = 2+ 3 – 4 = 1
relation.  A = x 2, B = – 1
2 2
Hence 'd' is correct. B2 = K a – b  b – c 
139. (b) a + b + c = 6 (–1) 2 = K 147. (b) + +
= a 2 + b 2 + c 2 = 14 K = 1
b – c  c – a  a – b  c – a 
a 3 + b 3 + c 3 = 36 1 1 1 1 1 1 1 1 1 1 1 1
Put values as      – 3      2
144. (c) 3 3 3 4 4 4 3 4 5 5 5 5
1 1 1 1 1 1 1 1 1 1 1 1
c – a 
a = 1, b = 2, c = 3      –      
1 + 2 + 3 = 6
3 3 4 4 5 5 3 4 4 5 5 3  – b  b – c 
a

Rakesh Yadav Readers Publication Pvt. Ltd. 148

For More Visit : www.LearnEngineering.in


For More Visit : www.LearnEngineering.in

2 b² x  12
a – b  a – b  151. (a) Third proportional of a and b =  x – 12 +
x  20
 ×
a – b  + a x – 20
b – c  c – a 
x 
y
3 5 3 3 3 5
2
b – c  Third proportion of y  and = +
b – c   x 5– 3 3– 5
× +
 – b  c – a 
a b – c 
x²  y² 3 5  3 –3 3 – 5
=
2 5 – 3
c – a  c – a  2
×
a – b  b – a  c – a 
 x²  y²  x²  y²
= x²  y²
 xy
2 5 –2 3
x y = = 2
 xy 5– 3
3 y x
a – b 

ir
154. (b) x = 5 – 21
 +
b – c  c – a  a – b 

v.iSn
43 3 2x = 10 – 2 21 ......(i)
3 152. (a)
2
b – c  74 3
 2x =  7 +  3 
2
– 2  7  3 
+ (By Rationalization of denominator)
a – b  c – a  b – c  2

dnag
3 43 3 7–4 3  2x =  7– 3 
c – a  ×
74 3 7–4 3 1 2
a – b  b – a  c – a  x =  7– 3 
2
(4  3 3 ) 7 – 4 3  

eYari
3 3 3
a – b   b – c   c – a  = 1 2
=
 b – c  c – a  a – b 
49 – 48  x = 2  7– 3 
(If x + y + z = 0 then x³ + y³ + z³ = 3xy) 4+ 3 3 1
3 a – b  b – c  c – a 
=  7– 3 
=

148. (c)
b – c  c – a  a – b 
(x –
= 3

3) 2 + (y – 5) 2 + (z – 4) 2 = 0
3) 2 = 0
– 16 3– 12 × 3
28 + 21 3
snhe
×7–4 3


2

x
32 – 2x – 21
 (x – x = 3 28 – 36 + 5 3 = 5 3 –8
kgei
(y – 5) 2 = 0 y = 5
(z – 4) 2 = 0 z = 4
7– 3
=
x2 y2 c2 153. (b) x =
4 15
5 3
2  
32 – 10 – 2 21 – 21  
ERna

 9
+
25
+
16
x  20 x  12 7– 3
9 25 16 + = ? =
 9
+ +
25 16
= 3 x – 20 x – 12 2  22  2 21 – 21 
aBryn

1 4 5  3 2  5  12
149. (d)x  (Inversely proportional) x = 5 3
=
5 3 7– 3
y2 =
 2 
k 2  20  3
2

 21  1  – 21 

x = or
5 3
y2
Les

7– 3
(y = 2) for (x = 1) (Given) x 2 5 =
 
wa. th

= 2 21  1 – 21
k k 12 5 3
 1 =  1=
 22 4 by C – D rule
=
 7– 3 
k = 4 x  12 2 5 5 3 2
 For y = 6 =
wwM

x – 12 2 5– 5– 3 155. (a) (x b+c) b–c (x c+a ) c–a (x a+b) a–b (x  0)


4 1 1 2 2 2 2 2
–b2
x = 2 =
9
=
9 3 5 3 = xb –c .
xc –a .
xa
 6 =
5– 3 2
–c 2 c 2 –a 2 a 2 –b 2 = x 0 = 1
150. (d) x 2 – y 2 = 80 xb
 (x – y) (x + y) = 80 x 2 3
= x 1 1
x – y = 8 ......(I) 20 156. (c) = –
5 3 a a x
 (x + y) × 8 = 80
(x + y) = 10 ......(II) 2 3 5 3
x  20 x 1 1
Now average of x and y = – = –
x – 20 2 3– 5– 3 a a x
x y 10
= = 3 3 5  x – 1 1
2 2
=  a  = –
= 5 3– 5   x

Rakesh Yadav Readers Publication Pvt. Ltd. 149

For More Visit : www.LearnEngineering.in


For More Visit : www.LearnEngineering.in

160. (d) xy = 8 Given


1–x 1 164. (c) x = 3 – 5
= So, (x, y) = (1, 8)
a x We have to Questions the options x = 3 + 5 – 2 3 5
x (1 – x) = a and check them
x – x2 = a (8, 1) x = 8 – 2 5 × 3
(2, 4)
1 (4, 2) x – 8 = – 2 15
157. (c) x + = 99
x  2x + y = 2 × 1 + 8 = 10 (Squaring both sides)
 x 2 + 1 = 99x 2 × 8 + 1 = 11 x² + 64 – 16 x = 60
2(x 2 + 1) = 2 × 99x 2 × 2 + 4 = 8 minimum x² + 4 – 16x = 0
2x 2 + 2 = 198x 2 × 4 + 2 = 10 x² + 6 – 16x = 2
Hence in this question we have all 165. (d) x + y + z = 0
100 x the options. So, take all positive
= factor otherwise we should have x 2 y2 z2

r
2
2x  2  102 x
to take –ve values also. yz + zx + xy = ?

Sni
(x, y) = (1, 8)
100 x 100 x 1
= = = (8, 1)
x3 y3 z3

gv.i
198 x  102 x 300 x 3 (2, 4) + +
xyz yxz zxy
(4, 2)
4x – 3 4y–3 4z–3 (–1, –8)
158. (c)
x
+
y
+
z
= 0
(–8, –1) x3  y3  z 3

ridna
(–2, –4) zxy
4x 3 4y 3 4z 3 (–4, –2) x + y + z = 0
 – +
y

y
+ – = 0 161. (b) x 2 + x + 1 = 0 .....(i)
x x z z  x 3 + y 3 + z 3 – 3xyz = 0

eeYa
2 x + y 3 + z 3 = 3xyz
3
3 3 3  1
= 4 – + 4 – y + 4 – = 0 x  2  + q2 = 0 3xyz
x z  
 zxy = 3
1 1 1 1 1 166. (c) a + b + c= 0
12 – 3  x  y  z  = 0  x2 + + 2 × × x + q2
 
geisnh
4 2 Have values
a = 1, b = 2, c = – 3
1 1 1  2 1
– 3  x  y  z  = – 12 = x2 + x +  q 
4  = 0 ......(ii)  a+b b+c  c+a  a b c 
  
  c  a  b   b+c  c+a  a+b 
 
Comparing constant term of equa-
1 1 1
Enak

  tion (i) and (ii)


x y z = 4  1  2 2 – 3 –3  1   1  2  –3 
  –3  1  2   2–3 –3+1 1+2 
1    
 q2 + = 1
xy xz yz 4
 (– 1 – 1 – 1) (– 1 – 1 – 1)
R

159. (c)
x  y =a, x  z = b, y  z = c 1 3  – 3 × – 3 = 9
= q2 = 1 – =
Now 4 4
1 1
aryn

x y 1 xz 1 yz 167. (a) a + =1 , b +


= 1
3 3 b c
 xy = a , xz = b , yz q =  = 
4 2 Values of a, b, c assume
Les B

1 162. (b) a 2 – 4a – 1= 0 1
= a 2 – 1 = 4a
c a =
2
1 1 1 1 1 1 1 1 b = 2
a – = 4
 y + x = a, z + x = b , z a
wa. th

c = – 1
Squaring both sides
1
1 1  abc = × 2× – 1 = – 1
+ 1
y = c 2
a + – 2 = 16
2
a2 168. (c) a + b + c = 2s
Now we have to find the value of x
 let
wwM

1 1 a = 2
1 1 1 1 1 1 2
a + = 18
a + b – c = y + x + z +x a2 b = 1
c = 1
1 1 1  1 s = 2
–  a2 + + 3 a – a 
y – z a2   2 2 2 2
= 18 + 3 × 4 = 18 + 12 = 30
s–a  + s–b  +  s–c  +s
1 1 1 2  2 2 2
a +b +c
 + – = 163. (d) 4 3x 2  5x – 2 3 = 0
a b c x
2 2 2 2
bc  ac – ab 2
4 3 x 2 + 8x – 3x – 2 3 = 0  2–2 +  2–1 + 2–1 +2
=  2 2 2
abc x 4x  3x  2 – 3  
3x  2 = 0 2 +1 +1

2abc 0 11 4 6
x = bc  ac – ab  4x – 3   
3x  2 = 0 
4 11

6
1

Rakesh Yadav Readers Publication Pvt. Ltd. 150

For More Visit : www.LearnEngineering.in


For More Visit : www.LearnEngineering.in

Squaring both sides Note:-


169. (d) x = 3 + 2 2
2 2
For two linear equaltions
x 2 = (3 + 2 2 ) 2
a b a b a 1x + b 1y + c 1 = 0
b +   + 2 × b × a = 25
  a  a 2x + b 2y + c 2 = 0
(Squaring both sides) Where x and y are variable.
= 9 + 8 +12 2 a2 b2
a1 b
2 + = 25 – 2 = 23
 1 then there will be unique
b a2 (i)
a2 b2
= 17 + 12 2
173. (d) xy + yz + zx = 0
 xy + zx = –yz solution.
1 1 17 – 12 2
2 = ×  xy + yz = –zx a1 b c
x 17  12 2 17 – 12 2
(ii)  1  1 , t hen inf init e
 yz + zx = –xy a2 b2 c2
= 17 – 12 2

ir
1 1 1 soultion
 + +

v.iSn
2 2 2
1 x – yz y – zx z – xy a1 b1 c1
2  
 2 +x = 17+12 2 +17–12 2 = 34 Puting values of –yz, –zx , –xy from (iii) then no solution.
x a2 b2 c2
abov e
3 2  3 1 1 4 –1
 2  –8

dnag
170. (b)x  –  =  +
x 2 2
  x x   xy+zx y   xy + yx 
So, the equations have only one
3 1 solution
 3x – 2 = +
z 2   yz + zx  a 4 b
x 15

eYari
177. (d) = and =
b 5 c 16
3 1 1
 3x – = 2  x x + y + z  + y x + y + z  a b 4 15 3
x  × = × =
b c 5 16 4
1

 3x


1

x
= 2


+ z x +y+ z

1


snhe
 
 1 1 1

a
c
=
3
4
1 2  x + y + z   x y z 
kgei
 x – =  a2 
x 3 c2 18 – 7 2 
Squaring both sides 1  zy  xz  xy  18c 2 –7a 2  c 
  x + y + z     45c 2 +20a 2
=
xyz   a2 
c 2  45  20 2 
ERna

1 4
 x2 + 2 – 2 = 1  c 
x 9
 x y z × 0 = 0
2
1 a 9
4 4 174. (c) a + b + c = 9 18 – 7   18 – 7 
 x2 + 2 = + 2 = 2 For minimum value a = b = c c  16
aBryn

x 9 9
= 2 = 9
171. (a) x 2 – 3x + 1 = 0  3a = 9 a 
45  20   45  20 
c
  16
 2
x + 1 = 3x 9
a = = 3
3 63
1 For minimum value a = b = c = 3 18 – 225  4 1
x + = 3 16
Les

a 2 + b 2 +c 2 = 3² + 3² +3² = = =
x 45 16  225 4
45 
wa. th

Squaring both sides  9+9+9  27 4


175. (d) a 2 + b 2 + 4c 2 = 2(a + b – 2c) – 3
1 1 1 1 1 1 1
 x2 + 2 + 2 = 9
 a2+b2 + 4c2 – 2a – 2b + 4c + 3=0 178. (d)
x 2 + y 2 + 2 = xy + yz +
 a 2 – 2a + 1 + b² – 2b + 1 + 4c 2 + x z zx
4c + 1 = 0 Go through options 'd'
wwM

1
2 (a – 1) 2 + (b – 1) 2 + (2c + 1) 2 = 0 take x = y = z
x + 2 = 7
x  a–1=0 a = 1 1 1 1 1 1 1
b – 1 = 0 b = 1 2 + 2 + 2 = 2 + 2 + 2
1 1 x x x x x x
 x2 + 2 + x + –1
x x 2c + 1 = 0 c =  Option d is right
2 179. (b) x = 3t, (I)
= 7 + 3 = 10
172. (c) a 2 + b 2 = 5ab 1 1
 a2 + b2 + c2 = 1 + 1 + = 2 + y =
2
( t + 1)
2 2 4
a b
 ab
+
ab
= 5
1 9 1
x = 2y
= = 2 1
4 4 4  x = 2× (t + 1)
a b 2
 b
+
a
= 5 176. (b) 4x – y = 2
2x – 8y + 4 = 0 x = t + 1 (II)

Rakesh Yadav Readers Publication Pvt. Ltd. 151

For More Visit : www.LearnEngineering.in


For More Visit : www.LearnEngineering.in

 3t = t + 1(from equation (i) and (ii)


1 1
2t = 1 2x + = 10 = a³ + = 0
x a³
1 Divide by 2 both sides
t =
2 x 1
1 10 187. (b) =
x + = = 5 x ² – 2x  1 3
1 2x 2
180. (c) x2 + x + a2
5 184. (c) xy (x + y) = 1 1 1
A 2 + 2 × AB +B 2 = (A + B )² 1  x² 2x 1
=
3
x + y = – 
2 xy x x x
1  1 
x²+2× ×x+a²=  x   Cubing both sides
10  10 
1 1
1 

r
A = x 1 =
 (x + y) 3 = 3 3 3
x  – 2
x y

Sni
1 x
B =
10 1 1

gv.i
x³ + y³ + 3xy (x + y)= 3 3 x + – 2 = 3
1 x y x
B = a =
10 1
2 – 3x – 4x 2 = 0
1  1   x + = 3 + 2 = 5

ridna
181. (d) x
– 4x² –3x+2= 0
x³ + y³ + 3 = 3 3  x  y  xy 
x y   Taking cube on both sides
ax² + bx +c = 0
In quadratic equation 1 3
x 1

eeYa
(i) When a > 0 3 3 – x³ – y³ = 3     = (5)³
x y  x 
4ac–b²
Minimum value = 1
4a 1
185. (c) x2 + = 83  x³ + + 3 × 5 = 125
(ii) When a < 0 x² x³
geisnh
Subtracting 2 from both sides
4ac–b² 1
Maximum value = 1  x³ + = 110
4a x³
 x² + – 2 = 83 – 2
 In –4x²–3x + 2 x²
1
a < 0 2 188. (b) x + = 4
Enak

 1 x
 Maximum value
 x –
= Maximum value x
 = 81 Squaring both sides
 
2 1
4×–4×2–  –3  1 x² + + 2 = 16
R

 x – = 9 x²
4×–4 x
Take cube on both sides 1
aryn

–32 – 9 41 x² + = 14
= = x²
–16 16 1 x –
1 
x³ – – 3   = 729 Squaring again
Les B

182. (a) x 4 – 2x² + k x³  x 


1
2 x4 +
 (x²)² – 2 × x² × 1 +  k 1 4 = 196 – 2 =194
x³ – – 3 × 9 = 729 x
x³ 189. (c) x + y + z = 6
2
  x² + y² + z² = 20
wa. th

 A² – 2 × A × B + k 1
x³ – = 729 + 27 = 756  (x + y + z) 2 = (6) 2

 k = 1  x² + y² + z² + 2 (xy + yz + zx) = 36
2
k = 1  1
 20 + 2 (xy + yz + zx) = 36
186. (d) a  a
 = 3
 
wwM

5x 1  2(xy + yz + zx) = 16
183. (d) 2 =
2x  5x  1 3 1 xy + yz + zx = 8
a + = 3
a  x³ + y³ + z³ – 3xyz
5 = (x + y +z) (x² + y² + z² – xy – zx – yz)
1 Take cube on both sides
2x ² 5x
1 =
  3 3  x³ + y³ + z³ – 3xyz = 6 (20 – 8)
x x x a 1 3 = 6 × 12 = 72



a


=  3 190. (b) x = 1 –
5 2
1
1 1 1  1  1 1 1 2
2x  5 = 3 = a³ + + 3a × a   =3 3 = × k i fj es; d j . k)
x a³ a  a  x 1– 2 1 2
 2x 1  1 1 2 1 2



x
 5  = 15

= a³ +

+ 3 3 = 3 3 =
1–2
=
–1
=   2 1 

Rakesh Yadav Readers Publication Pvt. Ltd. 152

For More Visit : www.LearnEngineering.in


For More Visit : www.LearnEngineering.in

c² – a² = C
1 27 – 36 –9
= = A + B + C = a² – b² + b² – c²
 x– = 1 – 2 + 2 + 1 = 2 8 8 + c² – a² = 0
x
Take cube 1 –9  A³ + B³ + C³ – 3ABC
 x³ +

+ 2 =
8
+ 2
1
3 3 = (A + B + C) [(A – B)² + (B – C)² +
x 1  1 2
 –  = (2) 3
 x –  = 8 1 –9  16 7
 x  x = x³ + + 2 = = (C – A)²]
x³ 8 8 A³ + B³ + C³ – 3ABC = 0
191. (b) x = a – b 194. (b) a + b + c = 15 A³ + B³ + C³ = 3ABC
y = b – c a² + b² + c² = 83 Where A = a² – b² etc.
z = c – a  (a + b + c )² = (15)²  A³ + B³ + C³ = 3 × (a² – b²) (b²
x+y+z=a– b+b– c+c– a=0 – c²) (c² – a²)
 a² + b² + c² + 2ab + 2bc +2ca = 225 Hence (a + b) (a – b) is a factor
 x³ + y³ + z³ – 3xyz = (x + y + z) (x²

ir
+ y² + z² – xy – yz – zx) = 0 83 + 2 (ab + bc + ca) = 225 b²
200. (b) a =

v.iSn
 2 (ab + bc + ca) = 225 – 83 = 142 b–a
3 – 2
192. (d) x = ab + bc + ca = 71  a (b – a) = b²
3 2  a³ + b³ + c³ – 3abc = (a + b + c) (a² + ab – a² = b²
b² + c² – ab – bc – ca) a² + b² – ab = 0
3 2   a³ + b³ = (a + b) (a² + b² – ab)

dnag
a³ + b³ + c³ – 3abc =15 (83 – 71) =
y =
3 – 2 15 × 12 = 180  a³ + b³ = 0
201. (b) p – 2q = 4
1 1 Take cube on both sides
195. (b) x+ =1
 x = x 1 (p – 2q)³ = (4)³
y

eYari
adding (1) both sides p³ – 8q³ – 3p × 2q (p – 2q) = 64
p³ – 8q³ – 6pq × 4 = 64
1 1 p³ – 8q³ – 24pq = 64
y =  xy = 1  x+1+ = 1 + 1 p³ – 8q³ – 24pq – 64 = 0
x x 1
202. (c) x = –1

 x+y =
3 – 2
3 2

2
+
3 2
3 – 2

2
 (x+1)+ 

Put x + 1 = 1
snhe
 1 

 x  1
=2
1
x 99
1
–1
+
1
x 98
+
x 97
1
+
x 96
1
+
1
x 95
1
+ 94 +
x

   
kgei
3 – 2 3 2 1 x
 and = 1
1 x 1 1 1 1 1
= 99 + 98 + + +
32–2 6 322 6
1  –1  –1  –1 97
 –196
ERna

  10  (x + 1) 5 +  x  15 1 1 1
1
95 + 94 + –1
 (x + y) 3 = x 3 + y 3
+ 3xy (x + y)
= 1+1 = 2  –1  –1  1
1
 (10) 3 = x 3 + y 3 + 3 × 1(10)
1 1 1 = – 1+1–1+1–1+1+ –1 = – 2
aBryn

196. (a) + = –1
 x³ + y³ = 1000 – 30 = 970
a b a b 1
2 203. (a) 3 =a 3 4 +b 3 2 +c
193. (c) 2x + = 3 a b 1 4  32  1
x  =
ab a b 1 1
1 3  (a + b)² = ab  3 4  32  1 = 2 1
Les

 x +
= 1
  2
x 2  a² + b² + 2ab = ab 23   23  1
wa. th

Taking cube on both sides a² + b² + ab = 0


 
 
3 3 a³ – b³ = (a – b) (a² – ab + b²)
x 1 3 a³ – b³ = 0  (  A³– B³=(A–B) (A²+AB+B²)
=    = 2 197. (d) If a + b +c = 0 1
 x    Put, A = 3 , B = 1
then, a³ + b³ + c³ – 3abc = 0 2
wwM

a³ + b³ + c³ = 3abc
1 198. (c) x = y = 333, z = 334  1 
x  a
 x³ + y³ + z³ – 3xyz  2 3 – 1
x  
1  
x³   a ³ – 3a 1 = 2
x³ = (x+y+z)((x–y)² + (y – z)²) + (z – x)²  1   1  1
2

2  2 – 1   2   2 3  1 
3  3
     
1  1 27 1     
x³ + + 3x   =  (333 + 333 + 334)[ (333 + 333) 2 +
 x  2
x³ 8
(333 – 334)² + (334 – 333)²]
 1 
 2 3 – 1
1 3 27 1    1 
 x³ + + 3 × =  
x³ 2 8  (1000) (0 + 1 + 1) = 1000 
3   2 3 – 1
2
 1  
199. (a) (a² – b²)³ + (b² – c²)³ + (c² – a²)³ 3 3  
1 27 9  2  – 1
 x³ + = – Let a² – b² = A  
x³ 8 2 b² – c² = B  

Rakesh Yadav Readers Publication Pvt. Ltd. 153

For More Visit : www.LearnEngineering.in


For More Visit : www.LearnEngineering.in

209. (d) f(x) = 12x³ – 13x² – 5x + 7


 2 1 If we divide f(x) by
1
1
– 1 = a  23  + b  2 3 + c
x² +

= 2 x =1
23    (3x + 2) then
  4(a³ + b³) = 1 + 3
(Comparing the terms ) 3x + 2 = 0
a = 0
 a³ + b³ = 1
2
b = 1 x = – Let a = 1 b = 0
3
c = –1  a² – b² = 1 – 0 = 1
3 2 or you can directly put the value
 a+b+c=0+1–=0  –2   –2   –2 
 f  = 12   –13   of x also,
 3   3   3 
204. (d) x = 32  3 212. (a) 121a² + 64b²
 –2  = (11a)² + (8b)² + 2 × 11a × 8b
x3 = 2 + – 5  3  + 7 = (11a + 8b)²
3  
 So term added t o make perfect

r
1 1 2– 3 8 52 10 square = 176 ab
= × = – 12× – + +7

Sni
x 3
2 3 2– 3 27 9 3
213. (b) a = 2 + 3
gj d k i fj es; d j . k –96 – 156  90  189

gv.i
2
=
27  a² = 2  3 
2– 3
 4–3
= 2– 3
= 4 + 3 + 4
–252  279 27 3
= = = 1

ridna
1 27 27 = 7 + 4 3
 x³ + x ³ = 2 + 3 + 2 – 3 = 4

3
210. (b) 2x² – 7x + 12 = 0 1 1
205. (d) x = 5 + 2  =
roots are  , a² 74 3

eeYa
 x – 2 = 3
5
Take cube on both sides c –b 7–4 3
  = + ,  +  = =
3 a a 7  4 3  7 – 4 3 
 (x – 2)³ = 3 5 
7 12
 x³ – 8 – 3 × 2 × x [x – 2] = 5
geisnh
    = + 2 ,  = 2 = 6 =
7–4 3
1
 x³ – 8 – 6x² + 12x = 5
   ²  ² 1
 x³ – 6x² + 12x – 13 = 0  + =  a² + = 7+ 4 3 + 7– 4 3
   a²
p²–p p²–1 p²
= 14
Enak

206. (b)
2p³+p² + p²+3p + p+1     ² – 2
= 1
In such type of question a  214. (a) p + + p + k²
ssume values of p. 4
2
R

 Let p = 1 7  1
 2  – 26 49
p +  k²  4 
1–1 1–1 1   – 12  p +
= = 4  
 + + 6 6
aryn

2+1 1+3 1+1


2 1  1
1 49 – 48 1   p +2× × p + k²  4
1 = = 2  
Les B

= 0 + 0 + = 64 24
2 2
Now check options (b)
 A 2 + 2 × A × B + B²
3
211. (c) x³ + = 4 (a³ + b³)
1 1 x  1
 A = p B² =  k²  4 
2p² = 2  
wa. th

1
Hence option (b) is Answer. and 3x + = 4 (a³ + b³)
x³ 1
1 B =
3 1 2
207. (b) x + = 2
x  x³ + = 3x +
x x³ 2
 Put x = 1 1 1
wwM

1 1 3  k² + = 2
 x³ – = 3x – 4  
 1 + x³ x
1 = 2
2 = 2 (satisfy) 1  1 1 1
  k² + =
 x³ – x ³ = 3 x – x  4 4
 1   1     
x²  x²  x³  x³  k² = 0
   
 1  1  k = 0
= (1 + 1) (1 + 1)   x – x   x ²  x ²  1
= 2 × 2 = 4     1
208. (a) a, b, c, are +ve integers 215. (a) Reciprocal of  x  x 
 1  
So, minimum value is a = b = c = 1 = 3 x – x 
 Putting the value of x in equation   1
a³ + b³ + c³ – 3abc x
1 =  x  1  =

= 1 + 1 + 1 – 3 × 1× 1× 1 =0 x² + 1 + = 3 x x ² 1
x² 
Hence minimum value is 0.

Rakesh Yadav Readers Publication Pvt. Ltd. 154

For More Visit : www.LearnEngineering.in


For More Visit : www.LearnEngineering.in

216. (a) F or m in i m um v al ue of 220. (d) (x – 2) is a factor of x²+3Qx–2Q 224. (a) a + b + c + d = 4


for (x – 2) = 0 1 1
1 1 1
+ + x² + 3Qx – 2Q = 0 + 1–a 1–b 1–c
a b c        
1–a 1–b 1–c
a = b = c
 4 + 3 × Q × 2 – 2 × Q = 0
a + b + c = 1 (given)  4 + 6Q – 2Q = 0 1 1
+ 1–c  1–d  1–a  + 1–b 1–c 1–d 
4Q = – 4
1
 a = b = c = Q = – 1 Put a = 0, b = 0 and c
3 221. (d) a + b = 12 ....... (I) = 2 and d = 2
ab = 22 ...... (II) a + b + c + d
1 1 1 = 0 + 0 + 2 + 2 = 4 = 4 (satisify)
= = = 3 Squaring both sides of equation (I)
a b c
 a² + b² + 2ab = 144 1 1
1 1 1 a² + b² + 2 × 22 = 144 1–0  1–0  1–2 + 1–0  1–2 1–2
 Minimum value of a + b + c

ir
a² + b² = 144 – 44 = 100
1 1

v.iSn
= 3 + 3 +3 = 9 + +
1 1 1–2 1–2 1–0  1–2  1–0 1–0 
222. (b) x= 3 – & y= 3 +
217. (b) a 2  3  = b 2– 3   = 1 3 3
1  1  1 1
 + + +
–1  1  –1  –1 –1
1 1 x² y² x ³  y³

dnag
a = b =  y + x = xy  –1 + 1 + 1 – 1= 0
2 3 2– 3
1
1  x  y  x ² – xy  y²  225. (b) x – = 1
 a =
b = x
xy

eYari
1 1 1 1 4 1
 + x –
a²+1 b²+1  x+y= 3 – 3 + 3 + 3 =2 3 2
x
 2
1 1 3x  5x – 3
 +  1   1 

1
1

1

1
b²+1  xy =

= 3 –
snhe


1
3 –

=
8
 
3 
3 
3
divide and multiply by x

x
x
4

x
1
3
 + 3 3
kgei
1+b² b²+1  2
3x 5x 3
b²  x  y   x ²  y²+2xy–2xy – xy   –
 x x x
b² 1 xy

ERna

+
b²+1 b²+1
b²+1 

x  y  x  y 
2
– 3 xy  x
3

1

x
3

1

 b²+1
= 1 xy = 3   1
3x – 5 3x – 5
x  x 
aBryn

 2 8
218. (d) 2  3  a = 2 – 3  b = 1 2 3 2 3   – 3
3

 1
1  8  x –
= 1
x
 a
= 2  3 3 Take cube on both sides
By rationals
Les

2 3 12 – 8  3
2  3 3  4  1
1   =3 3 x –  = (1) 3
wa. th


b
= 2– 3 8 8  x 
3
1 1 223. (b) x² + ax + b 1  1
 + =2– 3 +2+ 3 = 4  x³ – – 3x –  = 1
a b 1 2 x³  x 
 x² + 2 ×
2
a × x +  b
wwM

1 1 1 1
219. (a) a+ = b+ = c+ x³ – – 3 = 1
b c a  A² + 2 × A × B + B² = (A + B) 2 x³
To save your time assume values of
1 1
a, b, c according to equation.  A = x, B = a, B 2
2  x³ – = 4

1 2
Let a = 2, b = – 1 & c =
2
=  b , B = b
1
3
x –
2 x³ 4 4 1
1 1 1 1  1  = = = =
2+ = – 1+ = +  x  2a be perfect square  1  3  1  5 8 2
–1 1/2 2 2   3x –   5
 x 
= 1 = 1 = 1 1 1
at b = 2 a b= 4 a² 226. (d) x + y = 15
1
 abc = 2 × – 1 × = – 1  a² = 4b  x – 10 = 5 – y
2

Rakesh Yadav Readers Publication Pvt. Ltd. 155

For More Visit : www.LearnEngineering.in


For More Visit : www.LearnEngineering.in

x – 10 = – (y – 5)
2 1
Take cube on both sides x  1 11 x + = 3
 x = = 1 x
 (x – 10)³ = (– (y – 5))³
2
x – x²
 (x – 10)³ + (y – 5)³ = 0 1
230. (b) y = 1 – 3k and x = –2k  x³ + + 3 × 3 = 27
(giv en) x³
1
227. (b) x²+ = 66  For x = y
x² 1
–2k = 1 – 3k x³ + = 18
1 k = 1 x³
 x²+ –2 = 66 – 2
x² 231. (b)  x ²  y²  z  x  y – 3z + 3 xy³ z ² 6 4
x x  x²  1
x = 1 y = – 3 z = –1 (given)

2 x³
x –
1
  = 64  1 9 –1 1– 3  3 + 3 1   –33 1 6

r
 x  x x4 x² 1
= 3 + (– 3)= 0  + +

+

Sni
x³ x³
2
x –
1
232. (b) x +
1
= 2 ........(I)
  = (8) 2 1 1

gv.i
x x
   x³ + + + x
Squaring both sides x³ x
1 1  18 + 3 =21
 x – = 8
 x² + + 2 = 4
x

ridna

 x  13 –  x – 13
x² 1 2x 1 235. (b) = 2
–   x² + = + 2  x  12 –  x – 12
x ² – 1  2x x x x x²

eeYa
 = x Cubing equation (I)  A³ – B³ = (A – B) (A² + AB + B²)
x
x A² – B² = (A – B) (A + B)
1  1
 x³ + + 3x   = 8  x  1 – x  1   x  12   x –1  x  1   x –12 
 1 x³  x  
x – 2  x  1 – x  1 x  1  x –1
  x 
geisnh
1 =2
1 x³ + + 6 = 8
x³ x ²  1  2x  x ² – 1  x ²  1 – 2x   2
1  2x 
When x – = + 8 1
x x³ + = 2
Enak
x³ 3x ²  1
 2x
= 2
x –
1
Then  x
 + 2 = 8 + 2 = 10  1  1  3x² + 1 = 4x
    x ²  x ²   x ³  x ³  = 2× 2 = 4
3x² – 4x + 1 = 0
R

1 3x² – 3x – x + 1 = 0
When x – = – 8 1 3x (x –1) –1 (x – 1) = 0
x 233. (a) x + = 5 (3x – 1) (x – 1) = 0
aryn

– 8 + 2 = – 6 x  3x – 1 = 0
 (10, – 6)  Take cube on both sides 1
Les B

228. (c) a² + a + 1 = 0 x =
3 3
 1
a ³  1³  a  1 a ² – a  1  x   = (5)³  x – 1 = 0
a ³ – 1  a – 1 a ²  1  a    x 
  1
For x = 1 =
1
wa. th

 (a³ – 1) = (a – 1) × 0 1
 x³+ +3×5 = 125 By adding numinator and denomi-
a³ – 1 = 0 x³ nat or
a³ = 1 1 +1 = 2
(a³)³ = 1³ 1 No option is satisfied
a9 = 1  x³+ = 110
x³ 1
wwM

 x =
3
2  Squaring both sides
229. (a) x+ =1 1 + 3 = 4
x 2
x² + 2 = x x³  1  236. (c) x = 5 + 2 gj d k i fj es; d j . k
x² – x = –2  
x³ 
= (110)²
 1 1 5 –2
x – x² = 2  = ×
x 5 2 5 –2
x²  x  2 1
 = divide & mulitply by x  x6 + 6 + 2 = 12100 5 –2
x ² 1 – x  x  = 5 –2
5–4
x² x 2 2
1 1
  x  1 x 6+ 6 = 12100–2 = 12098  x– x = 5 +2– 5 + 2 = 4
x x x x x

 x² x 1 – x  234. (c) x² – 3x + 1 = 0 2x ² – 3x – 2
x
1 – x  
3x ² – 4x – 3
 x² + 1 = 3x

Rakesh Yadav Readers Publication Pvt. Ltd. 156

For More Visit : www.LearnEngineering.in


For More Visit : www.LearnEngineering.in

1 1  a6 + 1 = 0
2x ² 2 3x 2 242. (d) –
– – 2x – –3 a ²  ax  x ² a ² – ax  x ²  a 6 = –1
 x x x x
3x ² 3 4x  3 = a 18 + a 12 + a 6 + 1
– – 3x – –4 2ax
+ = (–1) 3 + (–1) 2 – 1 + 1
x x x x a 4  a ²x ²  x 4 = –1 + 1 –1 + 1 = 0

 1 a² – ax  x² – a² – ax – x2 2ax 1
2x – = + 248. (d) x 2 + y 2 + z 2 – xy – xz – yz = [(x
–3
x (a²  x²  ax )(a²  x² – ax ) a 4  a 2x ²  x 4 2

 2 4 – 3 – y) 2 + (y – z) 2 + (z – x) 2]
 1  –2ax 2ax
3x –  – 4 3 4 – 4
 x = + 1
2 2 a4  x4  a²x²
a 2  x 2  – ax  =
2
[(997 – 998) 2 + (998 – 999) 2

+ (999 – 997) 2]

ir
8–3 5
= = =0.625 –2ax 2ax

v.iSn
12 – 4 8 = + 1
a4  x4  2x2a2 – a²x² a 4  x 4  a 2x ² = (1+1+4) = 3
237. (b) a = 2.234 2
b = 3.121 –2ax 2ax
c = –5.355 = 4 4 2 + = 0 1
a  x  x a² a  x 4  a2x ²
4
249. (c) x+ =3
 a + b + c= 0 x

dnag
243. (b) x = 11
a³ + b³ + c³ – 3abc = (a + b + c)
x 5 – 12x 4 + 12x 3 – 12x 2 + 12x – 1
(a² + b² + c² – ab – bc – ca) = 0 2  1
= x5 – 11x4 – x4 + 11x3 + x3 – 11x2 –x² + 3x 3
238. (d) x² + y² + 1 = 2x 2  –4 3x   – 4
x² – 2x + 1 + y² = 0
11x + x – 1 3x  3 – 4x

x x

 x
= (11) 5 – 11× (11) 4 – (11) 4 + 11 × 2 2

eYari
(x – 1)² + y² = 0 x  1– x x 1 x x  1  –1
(11) 3 + 11³– 11 × (11) 2 – (11 ×  –  x
If A² + B² = 0 x x x  
11) + (11 × 11) + (11) – 1
As powers are even = 0 – 0 + 0 + 0 + 11 – 1 = 10
it can possible only 244. (c) p = 99 33 – 4 9–4 5
when A = 0 & B = 0 = = =

 x – 1 = 0
x = 1
y = 0


snhe
p(p² + 3p +3)
p³ + 3p² + 3p + 1 – 1
(p + 1)³ – 1
(99 + 1)³ – 1
250. (d)
3 –1

x = 3 + 2 2

1
3 –1 2
kgei
 x³ + y 5 = 1 + 0 = 1  = 3 – 2 2
239. (a) 3(a² + b² + c²) = (a + b + c)²  (100)³ – 1 x
by options a = b = c  1000000–1 = 99999 1
3(a² + a² + a²) = 9a² 245. (c) From option (c) LHS (x + 2)² = x² x + = 3 + 2 2 + 3 – 2 2
ERna

x
 9a² = 9a² + 4x + 4
RHS = x² + 2x + 4 1
240. (a) x  x – 3   –1 x + = 6
 LHS  RHS x
–1
aBryn

 (x – 3) = 2 3
x  1 x 1
246. (a) a  a  = 3    = (6) 3
Taking cube on both sides    x 
3
 –1  1
 (x – 3) 3 =    a +
a
= 3 1
 x  x3 + 3 + 3 × 6 = 216
Cube on both sides x
Les

–1
 x³ – 27 – 9x (x – 3) =
x³ 3
wa. th

 1 3 1
–1 a  a 
 
=  3 x3 +
x
3 = 216 – 18 = 198
 x³ – 27 – 9 × – 1 =

1 6 4 2
–1  a³ + + 3 3 = 3 3 x x x 1
 x³ – 27 + 9 =
x³ a³  3
wwM

x
1
–1  a³ + = 0 1
 x³ – 18 = a³ 1
x³ = x3 + x + + 3
1 x x
 x³(x³ – 18) = –1 247. (a) a + = 3
a
241. (a) a² + 4b² + 4b – 4ab – 2a – 8 1 1
Take cube on both sides = x3 +
= a² – 4ab + 4b² – 2a + 4b – 8 3 + x +
x x
= (a –2b)² – 2 (a – 2b) – 8 1 1
1  3
Put t = a – 2b  a³+ a ³ +3a× a a  a  =  3 = 198 + 6 = 204
= t² – 2t – 8 251. (c) (a + b – c) 2 + (b + c – a) 2 + (c + a
= t² – 4t + 2t – 8 1 – b) 2 = ?
= t(t – 4) + 2(t – 4)  a³+

+3× 3 3 3  a + b + c = 0 (given)
= (t + 2) (t – 4)
= (a – 2b – 4) (a – 2b + 2) 1  a + b = –c
 a3 + = 0
(Put the value of assume t ) a³  b + c = –a

Rakesh Yadav Readers Publication Pvt. Ltd. 157

For More Visit : www.LearnEngineering.in


For More Visit : www.LearnEngineering.in

 a + c = –b 1 337 1 1 1
3
 (a + b – c) 2 + (b + c – a) 2 + x2 + 2 – 2 = –2  a + b + 3 a 3b 3 c = c
x 144
(c + a – b) 2
1 1 1
 (–c –c) 2 + (–a –a) 2 + (–b –b) 2 2 3
x –
1 337 – 288 49  a + b – c = –3 a 3 b 3 c
 (–2c) 2 + (–2a) 2 + (–2b) 2  x
 = = Again take cube both sides
  144 144
 4c 2 + 4a 2 + 4b 2  (a + b – c) 3 = –27 abc
1 7
 2
4(a + b + c ) 2 2
x – =  (a + b – c) 3 + 27abc = 0
252. (b) 3
p + 3p + 3p – 7 = 0 2 x 12
260. (b) 4x + 5y = 83
p 3 + 3p 2 + 3p + 1 = 7 + 1 3x : 2y = 21 : 22
= (p + 1) 3 = (2) 3  1   1 25 7 175 x: y = 7 : 11
 x  x  x – x  = × =
= p + 1 = 2    12 12 144 let x = 7 and y = 11

r
 p = 1 y –x = 11 – 7

Sni
 p 2 + 2p = 1 + 2 = 3  2 1  175 = 4
 x –
2  =
253. (a) x = 2015  x  144

gv.i
261. (c)x = 3 a  a ²  b ³ + 3
a – a³  b³
y = 2014
z = 2013 1  2 1  2 1  Take cube on both sides
x  x –
 x4 – 4 = 2  2 = x³ = (a + a²  b³ ) +
1 x  x  x 

ridna
= x 2 + y 2 + z 2 – xy – yz – zx = [(x
2 1
175 337 58975
– y) 2 + (y – z) 2 + (z – x) 2]
144

144
=
20736
(a – a ²  b ³ ) + 3  a  a²  b³  3

1 256. (c) x = 9999 1

 

eeYa
2 3
= [(2015 – 2014) + (2014 – a – a ²  b³
2
2013) 2 + (2013 – 2015) 2 ] 4x
3
–x  2
x 4x – 1 
1 2x  1 6x – 3 = 3 2x  1 2x – 1  3
a  a ²  b³  3 a – a ³  b³ 
= (1 + 1 + 4) = 3 2
2
geisnh x³ = 2a + 3  a ² – a ²  b ³   3
x 
254. (a) 3a = b 2 (given)
2 
x 4x 2 – 1  x
= =
3 3 3  4 x 2 – 1 3 2

a+b –  a–b  x³ = 2a + 3
 –b 3
3
 x 
a+b 2   a–b 2 
9999
= 3333
Enak
2
3
3 3
 3 3
257. (b) a + b = 9 2
3 3 3 3

a  b  3ab a  b – a – b – 3ab a – b  b  x
x³ = 2a + 3 
a + b = 3  
 2 2 2 2
a  b  2ab  a  b – 2ab Assume values, a = 2, b = 1 x³ = 2a – 3bx
R

 (2) 3 + 1 = 9 x³ + 3bx = 2a
3 2 3 2 2 + 1 = 3 262. (d) Given
2b +6a b b +3a b
 
aryn

2 2 2 2 1 1 1 3 x 24  1
2a +2b a +b  + = +1 =
a b 2 2 = 7
x12
Les B

b³  b ³ 2b ³ 258. (c) t 2 – 4t + 1 = 0
 t 2 + 1 = 4t x 24  1 x 24 1
 b² 1   
 b² b ²   1 2 x 12
x12 x12
3 3  t 1 4t
=
t t 12 1
 x 
wa. th

2b 3b x12 = 7
 
4 2 t+ = 4 
 t Cubing both sides
3 [take cube both sides] 3
 12 1 
 x   3
1 1 25 1   1  x12  = 7
wwM

255. (a) x + = 2 = t + + 3t t  t  t  = 64
x 12 12 t  
36 1 3  x12  1 x12  1 
1   x  36
  
x12 
675 t³ + = 64 – 12 = 52 x x12 
x2 + 2 + 2 = t³
x 144 = 343
1 1
1 625 t +3
= 52 36
t3  x 
x2 + 2 = – 2 x 36 + 3 × 7 = 343
x 144
3
259. (d) a  3b  3c 36 1
1 625 – 288 Take cube both sides  x 
x2 + 2 = x 36 = 343 – 21
x 144 3 3
 3 a  3 b   3 c  36 1 x 72  1
1 337  x 
2
x + 2 = 1 1 x 36 = x 36
x 144
 a+b+3 a 3 b 3 3a  3b  = c = 322

Rakesh Yadav Readers Publication Pvt. Ltd. 158

For More Visit : www.LearnEngineering.in


For More Visit : www.LearnEngineering.in

263. (d) Given P = 99


5 3 
2
5 3
2 1
find P(P 2 + 3P + 3) = ?
 x y 
    =
2
[225+225+226][(225226) 2 +
to put value in equation 2
5  3
2
2 2
 (+226–225) +(225–225) ]
99 ((99) 2 + (3 × 99) + 3)
2 2
 676
(100 – 1) [(100 – 1) 2 + [3×(100 –
1] +3)  x y 
2
 5  3
 
2
 1  1  0  = 676
2
 (100 – 1) [10000 + 1 – 200+ 300 1
 x + y = 8 ..................(i)
– 3 + 3] 270. (b) x2 + x = 5 then (x + 3)3 + 3

 5 3 5 3
x + 3
(100 – 1) (10000 + 100 + 1)
Again, x y 
    Let x + 3 = m
 (100 – 1) (10101) 5 3
 99 × 10101
   5  3 x =m – 3
then (m – 3)2 + (m –3) = 5
 m2 + 9 – 6m + m – 3 = 5

ir
99 99 99
4 5  3 m2 – 5m + 6 = 5

v.iSn
264. (b) Given, x = 2 m2 – 5m = –1
2
Find x 3 + 27x 2 + 243x + 631
 to put value x = 2  (x  y )  2 15 .........(ii) –1
m– 5=
 2 3 + 27×2 2 + (243 × 2)+631
m
 5 3  5 3  1

dnag
 8 + 108 + 486 + 631 And, xy  m+ = 5
 1233  5 3  5 3  m
265. (b) Given, x 2 + y 2 + z 2 = 2 (x + z – 1 ) xy = 1
1
then m3 + = 125 – 15
Find- x 3+ y 3+ z 3 = ? Substitutes values in the question. m3

eYari
 x 2 + y 2 + z 2 = 2 (x + z – 1 )
2 1
 x 2 + y 2 + z 2 = 2x + 2z – 2  x  y   xy m3 + = 110
 x 2 + y 2 + z 2 = 2x + 2z – 1– 1  2 m3
 x  y   xy Here m = x + 3 then
 (x 2+1–2x) + y 2 + (z2+1–2z) = 0
 (x–1) 2 + y 2 + (z–1) 2 = 0 1
82  1
 (x–1) 2 = 0
 x = 1
 y2 = 0
 y = 0
 2 15 2  1 
 snhe
63

61

4
(x+3)3 +
x + 3
271. (d) Given, m = – 4, n = – 2
Find m³–3m²+3m+3n+3n²+n³
3 = 110
kgei
 (z – 1) 2 = 0 268. (c) Given, 4a  30 Putting value of m and n
 z = 1 a
Value substituted in question,
 (– 4)3 – 3 (– 4)² + 3(– 4) + 3×(– 2)
1 3 + 3 (– 2)² + (– 2)³
 x 3+ y 3+ z 3 Find: a  3  3  ?
ERna

 13 + 0 + 13 a  – 64 – 48 – 12 – 6 + 12 – 8
 2 4  – 64 – 60 – 2
 4a   3
1 a  – 126
266. (b) Given, x   1
x 1 3 272. (b) 2 x – ky +
7 = 0 ......(i)
aBryn

2  a  6 x – 12y + 15 = 0 .....(ii)
Find: ? a 4
x2  x  2 There has no solution for
3
 1  –3 3 a1 b1 c1
1  a – 
 = 
  
x  1 
a = 
x  4 a2 b2 c2
x 2 +1 = x [Cubing both sides]
Les

(x 2 –x) = – 1 2 –k
1 1 1  27  =
Putting value in, a3   3a   a   
wa. th

 6 – 12
a3 a a  64
2
1 k
x 2
x 2 = 2 1  3  27 = 
 a3   3   K= 4
a3  4  64 3 12
267. (d) Given, 273. (a) Here, x = 332 ,
1 27 9
wwM

5 3 5 3  a3    y = 333 , z = 335
x , y a 3 64 4
5 3 5 3 Find x³ + y³ +z³ – 3 xyz
1 27 9
2 2  a3  3  3 1
Find:
x  y  xy
? a3 64 4 = a  b  c 
x 2  y 2  xy 2
192  171 3 1 21
  a  3  3  a – b  ²  b – c  ²  c – a 
64 a 64
x 2  y 2  2xy  xy
 269. (b) According to the question,
x 2  y 2  2xy  xy 332  333  335
 x = z = 225 
 
 333 – 332 ²  335 – 333
2
2 y = 226  2 
 x  y   xy ?  x 3+ y 3+ z 3– 3 x y z = ?
 2
1000
 x  y   xy As we know, 
 335 – 332 ²    1²  2²  3² 
1 2

Now, x  y 
 5 3  5 3  x 3+y 3+z 3–3xyz =  x  y  z 
2 1000
 5 3  5 3  [(x –y ) 2+(y –z) 2+(z–x) 2]
 14  = 7000
2

Rakesh Yadav Readers Publication Pvt. Ltd. 159

For More Visit : www.LearnEngineering.in


For More Visit : www.LearnEngineering.in

From equation (i)


1 22  22
274. (d) If 2  x 3  a
2 3 1 22  22
 x  =1
Find x = ? x
4
a 1
1  x² + 1 = x 4
 2 3 x 
2 3 a 2 – ax = 1 2 –1×2 =1 – 2 = –1
 x ²  1  3x 284. (a) Let a = 0
  x ²  1  7 x b = 1
2 – 3
 2 3 × x 
 a 3  b 3  ab  (a 2  b 2 )2
1 x  3x 4x 1
  =
 0  1  0  (0  1)2
 2x 3 2– 3 x  7x 8x 2
11 = 0

r
 x = -1 1
275. (c) Given m–5n = 2 279. (c) x  2
1

Sni
x
find m³ – 125 n³ – 30mn 285. (b) a  5
(a  3)
7 1

gv.i
m – 5n = 2
Find x 
5
2
x 1
 (m–5n)³ = 2³ (cubing both sides) a3 53
a3
 m³–125n³–3m×5n (m–5n) = 8 1
 x   2  Let x = 1

ridna
 m³ – 125n³ – 15mn×2 = 8 x 1
a  3  a  3  2
 m³ – 125n³ – 30mn = 8  
276. (d) Given Cubing both sides
1 1

eeYa
3
3 To, put value in question,  1 
x = a b a 3 b ..... ......(i)  a  3   23
a 3 
1 1 
on squaring both side  x 
7 7
 1  5
  
x5 1
3 1 1
 x² = a 3 b a 3 b ....  1+1 = 2 a  3   3. a  3 
geisnh
280. (d) Given expression, a  3
3
a  3
On cubing both sides  4x 2 + 8x
Let P should be added,  1 
 a3 
 x6 = a 3b 3
a b a 3 b .....  4x 2 + 8x + p   a  3   8
 (2x) 2 + 2 × (2x) × 2
  
 x6 = a3b x from equation (i)
Enak

[(a+b) 2 = a 2+b 2+2ab]


3 1
 On dividing above eq. by x we get Term that should be added = 2 2 = 4 a  3  3
3 [ 2 ] 8
6 3
P = 4 a  3
x a bx
R

 = 281. (b) 999x + 888y = 1332 3 1


x x
888x + 999y = 555
a  3  3
 8  6  14 Ans.
 x 5
= a³b a  3
aryn

1887 ( x  y ) = 1887
 x = 5 a ³b 3x  2y 5
286. (d) 
x+y = 1 2x  3y 6
Les B

1 282. (a) According to the question,


277. (d) Given: x  = 2 ...........(i) 18x  12y  10x  15y
x 1 1
 x  , y
2 3 2 3 8 x  27y
12 1
The value of x – =?
wa. th

12 x 27
x 1 2 3 
 x  , y 8
1 2 3 2 3
 if x = 1  x =2
x 1 2 3 3 x  3y 2
y   
1+1=2 3 x  3y 
wwM

2 3 2 3 
 
12 1
Then, x –
12  x  2 3 , y 2 3 2
x  3 27  3
8
8xy (x ²+y²)  
 3 27  3
8
12 1 2 2
 
 1 –
12 8 2– 3 2 3  2– 3
     2  3  
1 
 3  2 2 2
 1 – 1=0
 8  1 7  2 3  7  2 3  = 112  
 
  (5)  25
3  1
1 283. (d) According to the question,
278. (a) Given: x =1 ........(i) 287. (d) According to the Question
x
x 2  x 2 x =
a 3 + 2
x ²  3x  1 x 2 x 2
Find =?
x ²  7x  1 Put x = 2 y= 3 – 2

Rakesh Yadav Readers Publication Pvt. Ltd. 160

For More Visit : www.LearnEngineering.in


For More Visit : www.LearnEngineering.in

292. (a) According to the question  x + y = 2a


(x 3 – 20 2 ) – (y 3 + 2 2 )
If 2S = a + b + c
3 1 2
=[( 3 + 2 ) 3 – 20 2 – ( 3 – 2)
3
S=
a+b+c
2
 Let x = 3, y = 1, a = 2
– 2 2]
Let a = 10, b = 10, c = 10 a a
 x – a  +
y –a
=3 3 + 2 2 + 9 2 + 6 3 – 10+10+10
 S=
20 2 – 3 3 + 2
2 2 2
2 +9 2 –6 3
30  3 – 2 +
1 – 2 
– 2 2 S = = 15
2
= 9 3 –9 2 2
2 –9 3 +9 2 = 0  S(S – C) + (S – a) (S – b)   0
1 1

ir
288. (d) SHORTCUT METHOD 15(15 – 10) + (15 – 10) (15 –
Always do these types of question 10) = 75 + 25 = 100 300. (a) a² + b² + c² = ab + bc + ca

v.iSn
with the help of Now check from option. Let a = b = c = 1
Put a = b = c = 1 Option (a) ab = 10×10 = 100  a² + b² + c² = ab + bc + ca
3(1 2 + 1 2 + 1 2) = (a + b + c) 2 (Satisfied)
293. (d) p + m = 6 .....(i)
 1²+1²+1² = 1×1+1×1+1×1
3 = 3 satified

dnag
p³ + m³ = 72  3 = 3
So this is answer  a = b = c
(p + m) (p² + m² – pm) = 72
289. (d) According to the question, a+c
(p + m) [(p + m)² – 3pm] = 72  to find =?
1 1 [  p² + m² = (p + m)² – 2pm] b
x = a + & y= a – 6[(6)² – 3pm] = 72 from (i)

eYari
a a 1+1
36 – 3pm = 12  =2
 x 4 – x²y² – 1 + y 4 – x²y² + 1 pm = 8 1
= x 4 – 2x²y² + y 4 294. (d) xm × xn = 1 301. (d) p³ – q³ = (p – q) {(p – q)² – xpq}
= [x² – y²]² x m+n = xº (  xº = 1)  p³ – q³ = (p – q) [p² + q² – 2pq –




a+
1

a
2

–

a–
1

a
2


2

295. (d)
m + n = 0
m = –n snhe
2p
p²–2p+1
=
1

xpq]
p³ – q³ = (p – q) [p² + q² – 2pq –
(–3)pq]
4  p³ – q³ = (p – q) (p² + q² + pq)
kgei
 1 1 
2
2 1  So, x = –3
= a
  2 – a –  2 =
 a a  1 4  because a³ – b³ = (a – b) (a² +
p–2+
ERna

p b² + ab)
= [4] 2 = 16 302. (a) Given
(Divide p both in nu. & de.)
290. (d) Let m = 5 5 5  x + y+ z = 6
1
Factor = (a)×(a+1) p + – 2= 8  xy + yz + zx = 10
p
aBryn

Here m = a + 1 To find x³ + y³ + z³ – 3xyz = ?


or m – 1 = a ......(i) 1  Using formula.
p + = 10
p  (x + y + z)² = x² + y² + z² + 2 (xy
Let n = 5– 5– 5
296. (b) According to the question, + yz + zx)
Factor = (a)×(a+1) x = 2, y = 1, z = – 3  6² = x² + y² + z² + 2 × 10
Les

Here n = a ......(ii)  x³ + y³ + y³ – 3xyz = ?


From (i) & (ii) As we know that  36 = x² + y² + z² + 20
wa. th

m – 1 = n a + b + c = 0 then a³ + b³ + c³ –  x² + y² + z² = 16
or m – n– 1 = 0 3abc= 0
 2 + 1 – 3 = 0  x 3 + y 3 + z 3 – 3xyz =
3 – 5x 3 – 5y 3 – 5z
291. (b) + + = 0 x³ + y³ + 3³ – 3xyz = 0 (x + y + z ) x²  y²  z ² – xy – yz – zx 
2x 2y 2z 297. (a) According to the question
wwM

 (x 3 + y 6) (x 3 – y 6) = 6 16 –  xy  yz  zx  
3 5 3 5 3 5
or – + – + – =0  x 6 + x 3y 6 – x 3y 6 – y 12 = 6 [16 – 10]
2x 2 2y 2 2z 2

x 6 – y 12 = 6 × 6 = 36
3 298. (b) According to the question, 303. (d) Given:
3 3 35
or + + =
2x 2y 2z 2 1

1 x 1 a
  x –1 =
b
x y x –y
1 1 1 352
or + + = x –yx y 2x x a b
x y z 2 3  =
 2
x –y
2  2
x –y
2 1 a –b
2 2 2 3522 299. (a) Given, x + y = 2a to (using componendo & dividendo)
or + + =
x y z 2×3 a a a b
Find +  x= ........(i)
=10 x – a y – a  =? a –b

Rakesh Yadav Readers Publication Pvt. Ltd. 161

For More Visit : www.LearnEngineering.in


For More Visit : www.LearnEngineering.in

Again, so value of 2(x + y + z) 1


1– y b 2(10) = 20
  a + b + c  a – b  ²   b – c  ²   c – a  ² 
 2
 = a² + b² + c²
1 y a 309. (c) x² + y² = 14
x + y = 4 ...... (i)  then, 2a – 1 = 0
1 y a squaring both sides
 =
1– y b x² + y² + 2xy = 16 1
a = 2
14 + 2xy = 16
1 a b 2xy = 2
  4b – 3 = 0
y = a –b xy = 1
x² + y² = 14 3
a –b subtrance (2xy)from both sides.  b =
 y =
a b
........(i)
x² + y² – 2xy = 14 – 2 xy 4
(x – y)² = 14 – 2 × 1  4c + 5 = 0
x –y
 From question,

r
1  xy x – y = 12
–5
 c =

Sni
a + b a – b  x – y = 2 3 ...........(2) 4
– solve equation (1) and (2)
a – b a  b 

gv.i
 1
a  b a – b  y =2 – 3  a + b + c  a – b  ²   b – c  ²   c – a  ² 
1   2
 
a – b a  b  x = 2 + 3
a² + b² + c²

ab 2 – a – b 2 310. (d) x² – 4x – 1 = 0

ridna
2 2 2
1  1 3 5   1 3  3 5  –5 1  
 a ² – b ² 1 1 1   –   –        –  
2  2 4 4   2 4   4 4   4 2  
x – = 4  2 2 2
2ab x 1 3  –5 
4ab      
  2 2 4  4 

eeYa
2 a ² – b ²  a ² – b²  1
 x  = 4²–
 x 1  2  3 – 5   2 – 3 
2 2 2
 8   –5 – 2  
304. (d) Given (squaring both sides)         
2 4   4  4
   4  
a² + b² + c² – ab – bc – ca = 0
to find a : b : c = ? x
2

1  1 9 25
 2 = 18  
geisnh x 4 16 16
 According to the question,
311. (b) Given
a² + b² + c² – ab – bc – ca = 0 1
 a² + b² + c² = ab + bc + ca 3– 2 3 2  0 =0 (a + b + c = 0)
a = ,b= 2
3 2 3– 2
 Let a = b = c = 1
Enak
2
 1²+1²+1²=1×1+(1×1)+(1×1) a² b²  1
to find  =? 313. (a)  a   = 3
b a  a
 3=3
 So ratio of a : b : c = 1 : 1 : 1 a³ + b³
 1
R

=?
ab a+ = 3
x y z 3
a
305. (c) =b–c; =c–a; =a–b a + b – 3ab  a + b 
a b c  = ? cube both sides
aryn

3 3 3 ab
 x  y   z   x  y  z  1 1  1 3

a  
b 
    3
c   a 
    3– 2 3 2 a3 +  3a  a
a  a 
=  3
Les B

        b  c   a +b = + a3
3 2 3– 2
 x y z 
 = 3x yz
    0  2 2 1

 a b c  abc  3– 2   3 2  a3 +
a3
3 3
 2 2
x a+2 3 – 2
wa. th

306. (b) = = 3 3
y a–2
2 
2  2
3  2
2
 1
x2 a + 2  1 a3 + = 0
2
= a3
y a – 22  2 × (5)  a + b = 10
Again a6 + 1 = 0
wwM

Applying Componendo & Dividendo


 a + 2 ² – a – 2 ² 8a  3– 2  3 2   a 30 + a 24 + a 18 + a 12 +a 6 +1
x ² – y² 4a
= =   a × b = = a (a 6+ 1)+a 12 (a 6+1)+a 6+1
24
x ² + y² a + 2 ² +  a – 2 ² 2a² + 8 a² + 4  3 2  3– 2
= a 24 (0)+a 12 (0)+0 = 0
a
+
b
=2
 ab = 1
307. (c) 1 1 1
b a a + b 3 – 3ab a + b  314. (d) = +
a² + b² = 2ab  a b a b
(a – b)² = 0 ab
a – b = 0 10³ – 3  1  10 1 b a
308. (a) Put (x + y + z) = 10
 1 =
a b ab
x = 2  1000 – 30 = 970
y = 3 312. (c) (2a–1)²+(4b–3)²+(4c+5)² = 0 ab = a² + b² +2ab
z = 5 to find a² + b² + ab = 0
x(x + y + z) = 20 a³ – b³ = (a–b) (a² + b²+ ab)
2(10) = 20
a³ + b³ + c³ – 3abc
 = (a–b) (0) = 0
Similarly other will satisfied a² + b² + c²

Rakesh Yadav Readers Publication Pvt. Ltd. 162

For More Visit : www.LearnEngineering.in


For More Visit : www.LearnEngineering.in

CHAPTER

12
TRIGONOMETRY
1. Trigonometric Ratio: Relations between Trigonometric Side opposite to right angle is
Ratios :- BC, which is hypotenuse h.

ir
C

v.iSn
1 p 12 h 13
h (i) cosec   sin  = = , cosec  = p  12
p sin  h 13
q or cosec θ × sin θ =1
b 5 h 13
A b B cos  =  , sec  = 

dnag
1 h 13 b 5
To study different trigonometric ratio (ii) sec  
cos 
functions we will consider a right p 12 b 5
angled triangle. Suppose ABC is a or sec θ × cos θ =1 tan  =  , cot  = p  12
b 5

eYari
right angled triangle with  A = 90°. 1
(iii) cot   Ex.2 If 15 cot  = 8 then calculate the
We can obtain six different t an  remaining trigonometric ratio.
trigonometric ratio from the sides of
these triangle. They are respectively or cot θ ×tan θ =1
AC AB AC AB BC
BC ' BC ' AB ' AC ' AB '
and
BC
AB
 B =  the n thes e ratio ar e
. If (iv) t an   snhe
sin 
cos 
cos 
Sol.
q
15
kgei
(v) cot   8
respectively called sin  , cos  , tan  , sin 
cot  sec  and cosec  . Clearly for the 8 b
TYPE - 1 cot  = 15  p
given angle  , AC (p) is perpendicu-
ERna

lar, AB (b) is base and BC (h) is hy- Ex.1 Write all the six t-ratios value Let b = 8k p = 15 k
potenuse. Hence six different trigono- in the given figure: From pythagoras theorem, h2 =
metric ratios are follows (see the given A p2 + b2 = (15k)2 +
figure) (8k)2
aBryn

Trigonometric Ratios:- 5 90° 12 or, h = 225 k + 64k2 = 289 k 2


2 2

AC p Perpendicular or, h = 289k ²  17k


sin = = = q
BC h Hypotenuse B C
p 15k 15
Hence, sin  =  
AB b Base Sol. In  ABC is, a right angle
Les

cos = = = h 17k 17
BC h Hypotenuse triangle with  A = 90°,
wa. th

b 8k 8
cos  =  
AC p Perpendicular A h 17k 17
tan = = =
AB b Base 90° 12 p 15k 15
=p
5

tan  =  
b=

BC h Hypotenuse b 8k 8
cosec = = =
wwM

AC p Perpendicular q
B C h 17k 17
h = 13  
BC h Hypotenuse sec  =
b 8k 8
sec = = = Let AC = 12 = p and AB = 5 = b
AB b Base Then from Pythagoras theorem, h 17k 17
AB b Base cosec  = p = =
cot = = = BC = 2 2
AB  AC  5  12 2 2 15k 15
AC p Perpendicular
4
Clearly sin  and cosec  are recipro- = 25  144 = 169 = 13 Ex.3 If t an   , then cos   ?
3
cals to each other. Similarly cos  and Here side opposite to  is AC
sec  are reciprocals to each other which is p. (a) 4 5 (b) 3
5
while tan  and cot  are reciprocals Side adjacent to  is AB, which 1
is b. (c) 3 4 (d) 5
to each other.

Rakesh Yadav Readers Publication Pvt. Ltd. 163

For More Visit : www.LearnEngineering.in


For More Visit : www.LearnEngineering.in

BC 4 Let XOX' and YOY' be two mutually 1. sin( )   sin 


Sol.(b) t an    perpendicular lines. These lines
AB 3 divide the plane into four parts and 2. cos( )  cos 
 AC  (4)  (3)  5 2 2 e ac h o n e o f t h e m i s c al l e d a
3. tan( )   tan 
quadrant.
C
Complementary Angle. 4. cot( )   cot 

For a given angle  its complementary


5. sec( )  sec 

4 angle is (90°–  ). 6. cosec ( )  cosec 

A (B) T-ratios of ( 90 0   ) in terms of


those of  :-

r
B

90
A 1. sin(90 0   )  cos 

Sni
3

°–
AB 3


 cos    2. cos(90 0   )  sin 

gv.i
AC 5
4  3. t a n (9 0 0  ) c o t 
B C
Ex.4 If tan  = , the value of
3 4. cot (90 0   )  t an 

ridna
From definition,
1  sin 
is:- side opposite angle θ AB
5. sec(90 0   )  cosec
1  sin  sin  = =
hypotenuse AC 6. cos ec(90 0   )  sec 

eeYa
(a) 1 2 (b) 1 3 and cos (90° –  ) (C) T-ratios of ( 90 0   ) in terms of
(c) 1 9 (d) 1
13
side along with angle (90° – θ) AB those of  :-
= =
hypotenuse AC 1. sin(90 0   )  cos 
4 BC
geisnh
Sol.(c) t an     sin   cos(90 –  )
3 AB 2. cos(90 0   )   sin 
Similarly, we can prove that 3. t an(90 0   )   cot 
and AC  (3)2  (4)2  5
 cos   sin(90 – )
Enak

BC 4 4. cot (90 0   )   tan 


 sin   
AC 5  90,270.....(odd
  multiple 5. sec(90 0   )  cosec
C of 90°) will be changed
R

6. cosec (90 0   )  sec 


 0,180,360...........(
   multiple
of 180°) will not be changed (D) T-ratios of ( 180 0   ) in terms
aryn

4 Change will be in of those of  :-


following manner:
Les B

1. sin(180 0   )  sin 
sin  cos & cos   sin 
tan  cot & cot   tan 2. cos(180 0   )   cos 
A B sec  cosec  & cosec   sec 
3
3. t an(180 0   )   t an 
4 Signs of Trignometric Ratios:-
wa. th

1
1  sin  5 1 Y 4. cot (180 0   )   cot 
 
1  sin  1  4 9
II I 5. sec(180 0   )   sec 
5
Quadrants:- sin  6. cosec (180 0   )  cosec 
 ve All +ve
wwM

90°
Y cosec  (E) T-ratios of ( 180 o   ) in terms
Changing line X’ X of those of  :-
O
III IV
II Quadrant
1. sin(180 0   )   sin 
o
I Quadrant
t an  cos 
 ve
0 0
9 0  < 1 8 0 0   9 0
 ve 2. cos(180 0   )   cos 
Non-Changing line
cot  sec 
180° X' O X 0°, z360°
3. t an(180 0   )  tan 
III Quadrant IV Quadrant Y’
1 8 0 0   2 7 0 0 2 7 00   3 6 0 0 Trigonometric Ratios of Allied 4. cot (180 0   )  cot 
Angles
(A) T-ratios of (  ) in terms of those
5. sec(180 0   )   sec 
Y'
270°
of  :- 6. cosec (180 0   )  cosec 

Rakesh Yadav Readers Publication Pvt. Ltd. 164

For More Visit : www.LearnEngineering.in


For More Visit : www.LearnEngineering.in

T-ratios of ( 270 o   ) in terms = cot (2 × 360 + 60)°


(F) 5. sec(n  360 0   )  sec
of those of  :- 1
6. cosec (n  360 0   )  cosec = cot 60° =
3
1. sin(270 0   )   cos  Value of some specific angle of
Sol.(v) sin 960°
trigonometrical (t)-ratio function.
2. cos(270 0   )   sin  = sin(900 + 60)°
We must learn the following table
 900° multiple of 180°, so no change of
3. t an(270 0   )  cot  to solve the question bas ed on Trignometry function.
trigonometrical (t)-ratio angle 0°, = sin(2 × 360 + 180 + 60) = sin
4. cot (270 0   )  t an  30°, 45°, 60°, 90° (180 + 60) = –sin60°
5. sec(270 0   )   cos ec 0° 30° 45° 60° 90°
 – 3  sin 180    
cos ec(270 0   )  sec =   – sin  

ir
6. 2
1 1

v.iSn
3
(G) T-ratios of ( 270 o   ) in terms sin 0 2 2
1 Sol.(vi) cos(1020)°
2 = cos (1080– 60)°
of those of  :-
3 1 1 1080 multiple of 180°, so no change In
1. sin(270 0   )   cos  cos 1 2 0 Trignometry function.
2 2

dnag
= cos (3 × 360 – 60)°
2. cos(270 0   )  sin  1
tan 0 1 3  1
3. t an(270 0   )   cot  3 = cos 60° =
2
1

eYari
4. cot (270 0   )   t an  cot  3 1 3
0 Sol.(vii) sec (1500°)
0 = sec (1440 + 60)°
5. sec(270   )  cos ec 2 = sec (4 × 360 + 60)°
6. 0
cosec (270   )  sec
sec 1 3
2 2 
 sec n  360   
(H) T-ratios of ( 360 o   ) in terms
of those of  :-
cosec  2 snhe 2
2
3 1
= sec 60° = 2
  sec  
kgei
1. 0
sin(360   )   sin  TYPE - II Ex.6
cos(90 0  A ). sec(360 0  A ). tan(180 0  A )
2. cos(360 0   )  cos  Ex.5 find the value of following sec(A  720). sin(A  540 0 ).cot(A  90 0 )
=?
(i) sin 120° (ii) cos 210°
ERna

3. tan(3600   )   tan  (iii) Tan570° (a) 0 (b) 1 (c) -1


(iv) cot 780° (v) sin960° (d) None of these
4. cot (360 0   )   cot  Sol. (b)
(vi) cos1020°
5. sec(360 0   )  sec (vii) sec 1500° cos  90 0  A  . sec  360 0  A  . tan 180 0  A 
aBryn

Sol. (i) sin 120° sec  A  720 0  . sin  A  540 0  . cot  A  90 0 


6. cosec (360 0   )   cos ec = sin (90 + 30)°
T-ratios of ( 360 o   ) in terms si n A . sec A t an A 
(I) sin 90    cos  
sec 2360  A .sin 3180 0  A . cot 90 0  A 
    
of those of  :-   
   
Les

3
1. sin(360 0   )  sin  = cos 30° =  sec     sec 
2
wa. th

2. cos(360 0   )  cos  Sol.(ii) cos210° and cot      cot  


= cos (180 + 30)° sin A. sec A. tan A
3. t an(360 0   )  t an  
cos 180    – cos  sec A   sin A    t an A 
4. cot (360 0   )  cot 
wwM

sin A. sec A. t an A
– 3  1
5. sec(360 0   )  sec = – cos30° = sin A. sec A. t an A
2 Ex.7 sin720° – cot 270° – sin 150°
6. cosec (360 0   )  cosec  Sol.(iii) Tan 570° cos120° is equal to:–
= Tan (540 + 30)°
(J) o
T-ratios of ( n  360   ) in
(540° multiple of 180°, Then no change (a) 1 2 (b) 1 3
terms of those of  :- Tan (540 +  ) = Tan  ) (c) 1 5 (d) 1 4
1. sin(n  360 0   )  sin  1 Sol.(d). sin720 º – cot270º –
0
= Tan 30° = sin150º.cos120º
2. cos(n  360   )  cos  3
Sol.(iv) cot 780° = sin (2×360º+0º) –cot(360º–90º)
3. t an(n  360 0   )  t an  = cot (720 + 60)°
– sin
4. cot (n  360 0   )  cot   cot (n × 360 +  ) = cot  (90 0  60 0 ). cos(90 0  30 0 )

Rakesh Yadav Readers Publication Pvt. Ltd. 165

For More Visit : www.LearnEngineering.in


For More Visit : www.LearnEngineering.in

 s i n 0 0  co t 9 0 0  c o s 6 0 0 . s i n 3 0 0 13 AC Sol.(d) sec 17° – sin 73°


Sol.(b) sec    = sec 17° – sin (90° – 17°)
5 AB
1 1 1 = sec17°- cos17º
0 0    and BC  13 2  5 2  12
2 2 4 1
= - cos17º
BC 12 cos 17
sin 37 0  sin   
Ex.8. Find the value of :- AC 13 x2
cos 53 0 2 2
C 1 – cos 17 sin 17 y2
(a) 1 (b) -1 (c) 0 (d) 0   
Sol. (a) cos 17 cos 17 x2
1–
sin 37 0 sin 37 0 sin 37 0 y2
0
  1
cos 53 cos(90  37 ) sin 37 0
0 0

r

Ex.9. Evaluate :- sin2 60º+cos2 30° + x²

Sni
= y² ² – x² = y y² – x ²
y
cot245º y²

gv.i
+ sec2 60 º - cosec230º + cos2 0º:- 
A B Ex. 14. If cosec 39° = x, the value of
(a) 3 2 (b) 5 2 since  lies in the fourth 1
+ sin 239 ° +

ridna
(c) 7 2 (d) 2 quadrant cosec 2 51
12 1
3  sin   – tan251° – is:
Sol. (c) We know that sin 60  0 13 sin 2 51 sec 2 39

eeYa
2
Ex .12. If 3 tan   4  0 , whe re (a) (b)
3 0
x2 –1 1– x2
cos 30  , cot 45 0  1 
2     , then the value of
2 (c) x 2 – 1 (d) 1 – x2
sec 60 = 2
geisnh
2 cot   5 cos   sin  is :- 1
 sin 2 60 0  cos 2 30 0  cot 2 45 0  sec 2 60 0 Sol.(c) + sin239° + tan251°
2 0 2 0
53 7 cosec 2 51 
 cos ec 30  cos 0 (a)  (b)
10 10
1
 3 2  3 2 –
Enak
 7
    
2 2 2 2
  1  2  2  1   23 37 2
sin 51 . sec 2 39
 2   2   2 (c) (d)
  10 10 = s in 2 5 1° + sin 2 39 °
+ tan2(90°– 39°)
R

x cos ec 2 30 0 . sec 2 45 0 4
Ex.10 If Sol. (c) 3 tan   4  0  t an   
3 1
8 cos 2 45 0. sin 2 60 0 –
sin 90 – 39.sec 2 39
2
aryn

= tan2 60º - tan2 30º, then the


value of x is :- = cos2 39° + sin2 39° + cot239°
Les B

(a) -1 (b) 0 (c) 1 (d) 2 5


4 1
Sol. (c) –
cos 39.sec 2 39
2

2 2
[  sin (90°– θ )= c os θ ),
x  2    2 2  1 
2
wa. th

2 2
   3 
 3

4
3
3 3
tan(90°– θ )= cot θ ]
 1   3  sin   , cos    , cot    = 1 + cot2 39° – 1 = cosec2 39°– 1
8   
 2  2  5 5 4 = x2 – 1
[ sin  is positive and cos  is Ex. 15 Find the value of cos (180° +
8 1 3
wwM

 8x    8     x  1 A) + cos (180° + B) + cos (180°


3 2 4 negative in II quadrant].
+ C) + cos (180° + D) Where A,
x B, C and D are the vertices of a
13 Ex.13 If sin 17° = , then the cyclic quadrilateral ?
Ex.11. If sec   and  lies in the y
5 (a) 0 (b) 1
value of sec 17°– sin 73° is: (c) 2 (d) 2 cos A
fourth quadrant, then the
value of sin  is :- y2 – x2 x2 Sol.(a)
(a) (b) cos 180 0  A   cos 180 0  B   cos
12 12 xy y2 – x2
(a) (b) 
13 13 0
 C   cos 180 0  D 
x2 x2 180
5 5 (c) y y 2  x 2 (d)
y y2 – x 2
(c)  (d)   cos A  cos B  cos C  cos D
13 13

Rakesh Yadav Readers Publication Pvt. Ltd. 166

For More Visit : www.LearnEngineering.in


For More Visit : www.LearnEngineering.in

Proof 5  = 90°
  cos 180 0  C   cos 180 0  D 
(i) sin A. secB = 1
 = 18°
 cos C  cos D A + B = 90° (given)
Then, B = 90 – A  If A + B = 90° 
[  A  C  B  D  180 0 c y c l i c  tanA. tanB = 1
Now, sinA.sec.(90 – A)
quadrilateral] sinA.cosec.A

 cos C  cos D  cos C  cos D  0 Ex.22 If cot2  cot3  = 1
1 find the value of
 sinA × =1
Some Useful formula sin A
5 5
Same as we can proof all sin . cos
(i) 2
sin  + cos  = 12 2 2
remaining results same this
or sin2  = 1– cos2  process Sol. 2  + 3  = 90°

ir
or cos2  = 1 – sin2  And their vice-versa are 5  = 90°

v.iSn
also true.
(ii) 1 + tan2 = sec2   If A + B = 90° 
when sinA. secB = 1,  tanA. tanB = 1
or sec2  – 1 = tan2 
then we can say A + B = 90°
or sec2  – tan2  = 1

dnag
Ex. 16 The value of (sin25°. sec65°)
5 5
(iii) 1 + cot2  = cosec2  is equal to:- Now, sin . cos
2 2
or cosec2  – 1 = cot2  Sol. 25° + 65° = 90°
or cosec2  – cot2  = 1 put the value of 5 
 If A + B = 90° 

eYari
Proof we know,  sinA. secB = 1 90 90
A = sin . cos
2 2
So, sin25°. sec65° = 1
= sin45°. cos45°
h Ex.17 The value of
p

B b C Sol.
to :-
snhe
0

23° + 67° = 90°


0
(tan 23  t an 67 ) is equal =
1
2

1

2 2
1

Ex.23 If sin (x + 4)° sec (x – 4)° = 1


kgei
p b
sin  = cos  =  If A + B = 90°  2x
h h find the value of tan
Now,  tanA. tanB = 1 3
sin2  + cos2  Sol. sin(x + 4)° sec (x – 4)° = 1
ERna

2 2
So, t an 23 0. t an 67 0 = 1 x + 4 + x – 4 = 90°
 p b  p2 b2
=   +   = 2 + 2 = Ex.18 The value of 2x = 90°
h h h h tan10°. tan25°. tan 65°. tan 80° is x = 45
aBryn

p2  b 2 Now,
h2 Sol.
2x
tan10° tan25° tan65° tan80° tan
 In right angle  ABC 3
p2 + b2 = h2
=1 put of value of x
Les

then sin2  + cos2 


Ex.19 If sin(3x– 6) = cos (6x – 3) find 2  45 90
wa. th

h2 the value of x. = tan = tan


= =1 3 3
h2 Sol. 3x– 6 + 6x – 3 = 90°
Same as we can proof all 9x = 99° 1
remaining results same this x = 11 = tan 30° =
3
wwM

process  If A+ B = 90, 
TYPE - III  then sinA = cosB  Ex.24 If cos(90 –  ) = sin (3  – 5)
If A + B = 90°, Ex.20 The value of cos40°.cosec50° find the value of 
Results Sol. 40° + 50° = 90° Sol. cos(90 –  ) = sin(3  – 50)
(i) sin A. secB = 1
or sinA = cosB  If A + B = 90°  cos(90 –  ) cosec (3  – 50) = 1
(ii) cos A. cosecB = 1  cosA. cosecB = 1
or secA = cosecB 90 –  + 3  – 50 = 90°
(iii) tanA. tanB = 1 So, cos40°. cosec50° = 1  if cosA.cosecB = 1
or tanA = cotB  A + B = 90° 
(iv)cotA. cotB = 1 Ex.21 If tan 2  tan3  = 1
2 2
(v)sin A +sin B = 1 find the value of 
2 2
2  = 50°
(vi)cos A +cos B = 1 Sol. 2  + 3  = 90°  = 25°

Rakesh Yadav Readers Publication Pvt. Ltd. 167

For More Visit : www.LearnEngineering.in


For More Visit : www.LearnEngineering.in

Ex.25 If cot (x – 50) = tan (80 – 2x) 1 – tan  5 1 


find the value of tanx + sinx (vi) tan(45 –) =
(iii) cos 36 0   
1+ tan
Sol. cot (x – 50) = tan (80 – 2x)  4 
cot (x – 50) cot (80 – 2x) = 1 cos–sin
= 10 – 2 5
 if cotA.cotB = 1  cos+sin (iv) si n 3 6 0 
 then A + B = 90°  Ex.26 Find the value of the following 4
x – 50 + 80 – 2x = 90 (i) sin75° (ii) cos75° (iii)
10 2 2
– x + 30 = 90 tan15° (iv) tan75° (vii) sin 22 
x = – 60° Sol.(i) sin75° 2 2
Now, tanx + sinx sin(45° + 30°)
= tan(–60)° + sin (–60)° 10 2 2
= sin45° cos30° + cos45°sin30° (viii) cos 22 
= – tan60° – sin60° 2 2

r
 tan(– )  – tan   1 3 1 1 3 1 Ex.27 The value of

Sni
 sin(– )  – sin   = × + × =
2 2 2 2 2 2 cos15 – sin15
is

gv.i
= – (tan60° + sin60°) cos15  sin15
3 1
 3 sin75° = = cos15°
–3 3 2 2 Sol. cos15 – sin15
=–  3 2  =
  2 (ii) cos75° cos15  sin15

ridna
cos(45° + 30°) = tan (45 – 15)
TYPE–IV
= cos45°cos30° –sin45°sin30°
1
Sum and Difference Formula = tan 30° =

eeYa
1 3 1 1 3 –1 3
(i) sin (A+B) = sin A. cos B+cos = × – × =
A sin B 2 2 2 2 2 2 Ex.28 The value of
(ii) sin (A – B) = sin A. cos B – tan40° + 2tan10° is equal to
cos A sin B 3 –1
cos75° = = sin15° (a) tan40° (b) cot40°
(iii) cos (A+B) = cos A. cos B – sin
geisnh 2 2 (c) sin40° (d) cos40°
A sin B (iii) tan15° Sol. We know,
(iv) cos (A – B) = cos A. cos B+sin tan(45 – 30) 40° + 10° = 50°
A sin B tan 45 – tan30 both sides take tan
(v) 2 sin A. cos B = sin (A+B)+sin =
Enak

1  tan45.tan30 tan(40° + 10°) = tan50°


(A-B)
(vi) 2 cos A. sin B = sin (A+B)- 1 tan40  tan10
1–  = tan50°
sin (A-B) 3 3 –1 1 – tan40.tan10
R

(vii) 2 sin A. sin B = cos (A-B)-cos = 1 = tan40° + tan10°


1 1  3 1 
(A+B) 3 = tan50° –
aryn

(viii) 2 cos A. cos B = cos


(A+B)+cos (A-B) 3 –1 tan50°.tan40°.tan10
tan15° = = cot75°
Les B

(ix) sin2 A-sin2 B = sin (A+B) . sin 3 1 û


(A-B) (iv) tan75° 1 \ tanA.tanB = 1
(x) cos2 A-cos2 B = cos (A+B) . tan(45 + 30) if A + B = 90
cos (A-B) tan 45  tan30  tan40° + tan10°
wa. th

= = tan50° – tan10°
Tangent Formulae 1 – tan45 tan30
 tan40° + 2tan10° = tan50°
tan A  t an B 1 Now, tan50° = tan.(90° – 40°)
1
(i) t an(A  B)  3
1  t an A. tan B 3 1 = cot40°
= 1 =
wwM

tan A  t an B 1 –1  3 –1 Ex.29 The value of


(ii) t an(A  B )  3
1  t an A. tan B tan57  cot 37
cotA·cotB – 1 is equal to
3 1 tan33  cot 53
(iii) cot(A + B) = tan75° = = cot15°
cotA + cotB 3 –1 (a) tan33°.cot53°
cotB·cotA +1 Trignometric Ratios (b) tan53°.cot37°
(iv) cot(A – B) =
cotB – cotA of Specific Angles (c) tan33°.cot57°
 5 1  (d) tan57°.cot37°
1+ tan (i) sin 18 0    tan57  cot 37
tan(45 +) =
(v)
1 – tan  4  Sol.
tan33  cot 53
cos+sin tan 57  cot 37
= 10  2 5
cos–sin (ii) cos1 8 0   tan(90 – 57)  cot 53
4

Rakesh Yadav Readers Publication Pvt. Ltd. 168

For More Visit : www.LearnEngineering.in


For More Visit : www.LearnEngineering.in

1 9 1 2
9  1
tan57  = tan2   1
tan37 9 –1 Now, =
 2
tan   1 92 – 1
cot 57  cot(90 – 37)
2sin  10
 = 82 41
2cos  8 = =
1 80 40
tan 57 
 tan 37 5
1 tan = 4
 tan 37 4 Ex. 33 If tan  = , then the value of
tan 57 3
Perpendicular
Now, tan  = 3sin   2cos 
tan57 tan37  1 Base is
3sin  – 2cos 
tan37 2 2

ir
Hypotenuse = 5   4
 tan57 tan37  1 4

v.iSn
tan57 = Sol. tan  =
41 3
A
1 sin  4
 × tan57° 
tan37 cos  3

dnag
41
 tan57°. cot37° 5
sin  2  2

TYPE-V cos  3
B 4 C

eYari
Use of componendo and b 4 3sin  2

dividendo- Then, cos  = = 2cos  1
h 41
Apply C and D
x a x a sec   tan  5
If = =
y b, Then Ex.31 If = , then find 3sin   2cos  2  1

x+y a+b
=
x–y a–b
y b
snhe
sec  – tan 
The value of sin 
sec   tan  5
3

3sin   2cos 

3sin  – 2cos  2 – 1

3
kgei
Sol. = 3sin  – 2cos 
x a sec  – tan  3
Proof  Apply C & D Alternate:-
y b
ERna

Add 1 in both side. sec   tan   sec  – tan  3sin   2cos 


 sec   tan  – sec  – tan  3sin  – 2cos 
x a
1
y +1 = b 53
divide all terms by cos 
=
3 tan   2
aBryn

x  y a b 5–3
= ..........(i) 3 tan  – 2
y b 2sec  8
 =
subtract 1 in both side. 2tan  2 4
 tan  = (given)
x a 3
–1 = –1
Les

1
y b
cos   4 4
wa. th

1 32
x –y a –b  sin   =4 3 42 6
sin 
y = .........(ii) cos  4 = = =3
b 3 –2 4 – 2 2
3
(i) /(ii)
1
Ex.34 If 2cot  = 3, Then find the
wwM

x y a b So, sin  =
4
x –y = a –b 2cos  – sin 
sin   cos  5 value of
Ex. 32 If = , find the 2cos   sin 
sin   cos  sin  – cos  4
Ex.30 If = 9 find the Sol. 2cot  = 3
sin  – cos 
2
value tan  and cos  tan   1 3
value of cot  =
tan2   1 2
sin   cos  9
Sol. =
sin  – cos  1 sin   cos  5 cos  3
Sol. = , 
Apply C & D sin  – cos  4 sin  2
sin   cos   sin  – cos  Apply C and D 2cos  3
 sin   cos  – sin  – cos  tan  = 9 
sin  1

Rakesh Yadav Readers Publication Pvt. Ltd. 169

For More Visit : www.LearnEngineering.in


For More Visit : www.LearnEngineering.in

Apply C and D 289 – 169 Ex. 37 In a  xyz,  y = 90°


 2sin  cos  =
2cos   sin  3 1 169 xy = 2 6 and xz – yz = 2
=
2cos  – sin  3 –1 120 Find the value of secx + tanx = ?
 2sin  cos  = Sol.
2cos   sin  4 169 x
= =2
2cos  – sin  2 60 x
 sin  cos  =
So,
169 a  2
2 6
2cos  – sin  1 Alternate:-

2cos   sin  2
y z
a
Alternate:-
5 13 Let yz = a,
or

r
2cos  – sin  Then xz = 2 + yz = 2 + a
12
Apply pythagorean theorem

Sni
2cos   sin 
12 or 5
Divide all terms (in numerator 2
2 6  2
+ a  = (a + 2)2

gv.i

and in denominator) by sin.  sinq + cosq = 17 p b
13 h 2
2cot  – 1  2 6  = (a + 2)2 – (a)2

ridna
2cot   1 p b
 24 = (a + 2 – a) (a + 2 + a)
3 h h
 24 = 2 (2a + 2)
 cot  = (given) App ly p ythagore an here
2  24 = 4(a + 1)

eeYa
hypotenuse is 13, Then other sides of
3 right angle triangle will be 5 and 12.  6=a+1
2 –1 Now, C h e c k  a=5
2 3 –1 2 1
3 = = = 5 12 17 Hence, xz = (5 + 2) = 7
2  1 3 1 4 2  
2 13 13 13
geisnh yz = 5
But we cannot find exact value
TYPE - VI 7 5
of base and perpendicular, here no  secx + tanx = +
affect of value of sin  and cos  . This 2 6 2 6
Som e p y thagore an natural
question because both are product.
Enak

number will help in solving the 12 6


5 12 60 = = = 6
pr oblem on trigonom etr ic ratio Hence, sin  cos  =   2 6 6
angle. 13 13 169
Ex. 38 In a  ABC,  B = 90°
R

pythagorean theorem 3
(Bas e) 2 + (P erp end icular ) 2 = Ex.36 If sin  + cos  = , find the AB – BC = 2, and AC = 2 5
17
(Hypotenuse)2 find the value of
aryn

value of sin  .cos 


32 + 42 = 52, 62 + 82 = 102, cos2A – cos2C = ?
Sol. Sol.
5 + 12 = 13 , 102 + 242 =262,
2 2 2
A
Les B

82 + 15² = 172, 72 + 242 = 252, 15 17


or A
202 + 212= 292,92 + 402 = 412, etc. 2 5
8 a  2
17
Ex.35 If sin  + cos  = find the 8 or 15
wa. th

13 C
B C
value of sin  .cos  3 a
sin  + cos  = Let the triangle of side BC = a
17 17
Then AB = BC + 2 = a + 2
Sol. sin  + cos  =
13   Now,
wwM

squaring of both side 2

2
p
+
b
=
p b  (a + 2)2 + (a)2 = 2 5  
 17  h h h
 (sin  + cos  )2 =    a2 + 4 + 4a + a2 = 20
13 so, h  17
 2a2 + 4a = 16
 sin2  + cos2  + 2sin  cos  Apply pythagorean sides here
 a2 + 2a = 8
289 hypotenuse is 17, then other
= sides 8 and 15  a2 + 2a – 8 = 0
169  a2 + 4a – 2a – 8 = 0
8 15 23 (a + 4) (a – 2) = 0
289 Now, Check   
 1 + 2sin  cos  = 17 17 17 a = –4, a = 2
169
8 15 120 side of  is always positive
289 Hence, sin  .cos  =  
 2sin  cos  = –1 hence,
169 17 17 289
We take a = 2

Rakesh Yadav Readers Publication Pvt. Ltd. 170

For More Visit : www.LearnEngineering.in


For More Visit : www.LearnEngineering.in

Now, TYPE -VII


8 27 35
 AB = 4, BC = 2 = + = =7
5 5 5 Functio n and Invers e
 cos2A – cos2C function 
L.H.S = R.H.S
2 2
 4   2  3 (a) If sin θ + cosec θ = 2
=  – 
  2 5   2 5  So, cot  =
4 then sin θ = cosec θ =1

16 4 12 3 Ex. 40 If 2 – cos2  = 3sin  cos  ,  sinn θ + cosecn θ =2


 – =  n  natural no.
20 20 20 5 find the value of tan 
Ex.41 If sin θ + cosec θ = 2 find the
Ex. 39 If 2sin  + 15cos2  = 7,
1 2 1 value of sin100 θ +cosec100 θ
(0°<  < 90°) find the value of (a) (b) 0 (c) (d)
cot  2 3 3 Sol. sin θ + cosec θ =2

ir
Sol. 2–cos2  = 3sin  cos  Then, sin θ = cosec θ = 1
3 5 1 1

v.iSn
(a) (b) (c) (d) divide by cos2  both sides so, sin100 θ + cosec100 θ = 1
4 4 2 4
= (1)100 + (1)100 = 2
Sol. 2sin  + 15cos2  = 7, 2 – cos 2  3sin  cos 
 = (b) If cos θ + sec θ = 2
 2sin  + 15 (1 – sin2  ) = 7 cos 2
cos 2 
then cos θ = sec θ = 1

dnag
 2sin  + 15 – 15 sin2  = 7  2
2sec  – 1 = 3tan 
15sin2  – 2sin  – 8 = 0  cosn θ + secn θ = 2
  2(1 + tan2  ) – 1 = 3tan 
15sin2  – 12 sin  + 10sin  – Ex.42 If cos θ +sec θ = 2, find the
  2 + 2tan2  – 1 = 3tan 
8=0 value of cos10 θ + sec10 θ = ?

eYari
 2tan2  – 3tan  + 1 = 0
 (3sin  + 2) (5sin  – 4) = 0 Sol. cos θ +sec θ = 2
 (2tan  – 1) (tan  – 1) = 0
 3sin  + 2 = 0 or 5sin  – 4 = 0 cos θ = sec θ = 1
 2tan  –1= 0, tan  – 1 = 0
–2 4 Then, cos10 θ + sec10 θ
sin  = sin  = 1 = (1)10 + (1)10 = 1+1 = 2

3 5
Value of  between 0° and 90°
so sin  is positive. Then we
 tan  =
2 snhe
or 1
So, option (a) is correct
(c) If tan θ + cot θ = 2
so tan θ = cot θ =1
tann θ + cotn θ = 2
4
kgei
1
take sin  = tan  = Ex.43 If tan θ + cot θ = 2 find the
5 2
Alternate:- value of tan50 θ + cot60 θ
4 p
sin  = = Sol. tan θ + cot θ = 2
ERna

5 h Take options (a)


tan θ = cot θ =1
2 2 A
b= 5 – 4  =3 tan θ + cot60 θ = (1)50+(1)60=1+1=2
50

(d) If sinA + cosB = 2


b 3
aBryn

Hence, cot  = p = 5 Then A = 90º


4 1 B = 0º
Alternate:- Ex.44 If sinA + cosB = 2, then find the
 A+B
B C value of tan
2 2
Les

4 5
wa. th

1 p Sol. SinA + cosB=2


So, tan  = =
3 2 b
3 2 2
1 1
We take option then, h = 2  1 A = 90º
4
wwM

h= B = 0º
b 5
3
cot  = = p Now,
4 Now, tan
A+B
2–cos2  = 3sin  cos  2
2 2
h= 3   4 = 25 = 5
 2 
2
1 2
2–  =3× × 90+0
Now, check  5  5 5 = tan
2
= tan45º = 1
2sin  + 15cos2  = 7
2 6 6 (e) If sinA + cosB =0
4 3 = then A = 0°, B = 90º
2× + 15 ×   5 5
5 5 Ex.45 If sinA + cosB + sinC = 3, then
L.H.S = R.H.S
8 9
= + 15 × 1 find the value of cot
A+B+C
5 25  tan  = 3
2

Rakesh Yadav Readers Publication Pvt. Ltd. 171

For More Visit : www.LearnEngineering.in


For More Visit : www.LearnEngineering.in

Sol. sinA + cosB + sinC=3 Ex. 49 If sin  + cos β = 2(0°  (a) 1 (b) -1 (c) 2 (d) 0
β    90°), then sin Sol.  cos 90 0  0
1 1 1  cos10 . cos 2 0................. cos179 0  0
 2  2β 
 A = 90º   =
 3  Ex. 53 The value of tan1° tan2°tan3°
B = 0º .......tan 89° is :
C = 90º  
(a) sin (b) cos
2 3 1
90+0+90 = cot 60º = 1 (a) 1 (b) 0 (c) 3 (d)
cot  2 3
3 3 (c) sin (d) cos
3 3 Sol. tan 1°.tan 2°. tan3°......tan 45°
Ex .46 If  is acute and Sol. sin  + cos β = 2 sin   1; .....tan88°tan89°
= (tan 1°.tan 89°)

r
tan  +cot  =2, cos β  1 (tan2°. tan88°) .... tan 45°

Sni
then tan7  +cot9  is equal to:   = 90°; β = 0° = (tan 1°.cot 1°). (tan 2°. cot2°)
.... tan 45° = 1

gv.i
(a) 3 (b) 3 (c) 2 (d) 4
2  β   180  [  tan (90° – θ ) = cot θ ,tan
 sin  sin 
Sol. t an cot  2 3  3  θ .cot θ = 1]

ridna
tan  = cot  = 1 Ex.54 The value of : cos20º + cos40º
3 + cosº + ...... + cos160º +
So,  tan7  + cot9  = 1 + 1 = 2 = sin 60°  sin 60 
2 cos180º is:-
Ex .47 If tan(x  y )  3 and (a) 1 (b) -1

eeYa
 3
cot (x  y )  3 , then what
cos  cos 30  (c) 0 (d) 1 2
3 2
are the smallest positive value Ex.50 If sin 1  sin 2  sin 3  3 , Sol. cos 1 80 0      cos 
of x and y respectively?
then
geisnh
 cos160 0  cos 180 0  20 0    cos 20 0
(a) 45 0 , 30 0
cos 1  cos 2  cos 3  ? similarly
(b) 150 , 60 0 (a) 0 (b) 1 (c) 2 (d) 3 cos140° = – cos40°,

(c) 45 0 , 150 Sol. sin 1  sin 2  sin 3  3 cos120° = – cos60°
Enak

cos100° = – cos80°
(d) 30 0 , 45 0  sin 1  sin 2  sin 3 Now,
  cos 20 0
 cos160 0  
R

Sol. tan (x + y) = 3 
 1  2  3 
2
then x + y = 60° .....(i) 0 0
 cos 1  cos 2  cos 3  0 cos 40  cos140 
aryn

cot (x – y) = 3
Ex.51 If cos2  + cos2  = 2, then the 0 0
 cos 60  cos120 
Les B

Then x – y = 30° .....(ii)


from (i) and (ii) value of tan3  + sin5  is :
  cos 80 0  cos100 0   cos180 0
x = 45° & y = 15°
1 = cos 20 0  cos 2 0 0  cos 40 0  cos 4 0 0 
Ex.48 If 2 cos 31  1 and (a) – 1 (b) 0 (c) 1 (d)   
wa. th

3 0 0 0
cos 60  cos 60    cos 80  cos 80 0 
2 sin 22  3 , then what Sol. cos2  + cos2  = 2
 cos180 0
will be the value of 1 and 2  1 – sin2  + 1 – sin2  = 2
 cos180 0  1
wwM

(a) 300, 200 (b) 600, 400  sin2  + sin2  = 0 Ex. 55 sin25° + sin26° + ...........sin284°
(c) 200, 300 (d) 450, 450 + sin285° = ?
 sin  = sin  = 0
1 1 1
Sol. 2cos3 θ 1 = 1  cos3 θ 1 = =   =  =0 (a) 39 (b) 40
2 2 2
 tan3  + sin5  = 0
cos60°
TYPE - VIII 1
 3 θ 1 = 60°  θ 1 = 20° (c) 40 (d) 39
2
Series Base
3 Sol. Let the number of terms be n,
2 sin 2 2  3  sin 22   sin 60 0
2 then By tn = a + (n – 1)d
Ex. 52 The value of cos1 º, Here,
 2 2  60 0   2  30 0 cos2º,cos3º...... cos179 0 is:-  a = 5, d = 1

Rakesh Yadav Readers Publication Pvt. Ltd. 172

For More Visit : www.LearnEngineering.in


For More Visit : www.LearnEngineering.in

 85 = 5 + (n – 1) 1 Here, Sol.(d) sin21000° + cos21000° = 1


 n – 1 = 85 – 5 = 80  19 20  for every value of  in sin2 
+ = = + cos2  will be 1
 n = 81 40 40 40 2
 sin25°+ sin26° + .... + sin245° 
Ex. 61 If sin260° + cos2(3x – 9°) = 1
+ .... + sin284° + sin2 85° So, sum of 2 terms = sin² =1 Then value of x is
= (sin25° + sin285°) + (sin26° + sin2 2 Sol. sin260° + cos2(3x – 9)° = 1
84°) + .....+ to 40 terms + sin2 45° n = 19
This is similar to sin2  + cos2 
= (sin25° + cos2 5°) + (sin26° + 19 20
cos²6)+.... + to 40 terms + sum = + sin²  So,
2 2 60° = 3x – 9
sin245°
1 1 19 69° = 3x
= 40 + = 40 = + sin² 90º x = 23°

ir
2 2 2
Ex.62If 3sin2  + 7 cos2  = 4, then the

v.iSn
sin 90 – θ  cos θ 19 21 value of tan  is (where 0
   +1=
sin2 θ  cos 2θ= 1  2 2 <  <90°) :
 
Ex.56 The value of sin10º + sin20º + Ex.59 If 1+sinx + sin 2 x + sin³x
(a) 2 (b) 5 (c)
... + sin340º + sin350º +.....  = 4+2 3 , find the

dnag
Sol. sin10º+sin20º+...+ value of x 3 (d) 6
sin340º+sin350º Sol. It is a G.P Series Sol.(a) 3 sin2  + 7(1– sin2  ) = 4
 sin(360º – 350º) + sin(360º–
340º) +...sin180º ....sin340º +
a  3 sin2  + 7 – 7 sin2  = 4

eYari
then, S  =
sin350º 1– r  7– 4 sin2  = 4
 – sin350º–sin340º....+sin180º a = 1, r = sinx
3
+...sin340º+ sin350º = 0 1  4 sin2  = 3  sin  =
[sin{360º– θ }= – sin θ ,sin180º  = 4 +2 3 × 2
= 0])
Ex.57 The value of
cos²1º + cos² 3º + ...... + cos²
1– sin x

 4–2 3
4–2 3
snhe

So,  = 60°
tan  = tan60° = 3
89º + cos² 90º 2
kgei
Sol. cos²1º + cos² 3º +cos² 5º ...... + Ex.63 If cos4  – sin4  = , then the
1 4 3
cos² 89º + cos² 90º  =
1– sin x 4–2 3 value of 2 cos2  – 1 is :
89  1
ERna

n= +1 = 45 4 2 3
2 (a) 0 (b) 1 (c) (d)
45 4 3 2
1
sum =  = 4 2 3 2
2 1– sin x –
4 4 Sol.(c) cos4  – sin4  =
aBryn

45 cos² 90º 45 3
 + = (divide by 4 all terms)  (cos 2  + sin 2  ) (cos 2  –
2 2
0
Ex.58 The value of 1 2
1 sin2  ) =
 2 3  = 3 3
sin² + sin² + sin² + 1– sin x 1 – 2
Les

40 40 40 2
20 comparing both equation  cos2  – sin2  =
wa. th

3
..... sin²
40 3 2
 2 3 sin x =  cos2  – (1– cos2  ) =
Sol. sin² +sin² +sin² +..... 2 3
40 40 40 sin x = sin60º 2
wwM

19 20 x = 60º  2 cos2  – 1 =


+ sin² + sin² 3
40 40 TYPE- IX Ex.64 sin  + sin2  = 1
Find the value of cos2  + cos4 
 
...... Sin²
   (A) sin 2 θ + c o s 2 θ = 1
+ sin² +sin² Sol. sin  + sin2  = 1
sin² 40 +sin² 40 40 40 40
or
sin  = 1 – sin2 
2 2
 19 sin θ = 1 – c o s θ sin  = cos2 
 (sin² + sin² ) +
40 40 or Now, cos2  + cos4 
2 18 c o s 2 θ = 1 – s in 2 θ
(sin² + sin² ). .... + Put the value cos2 
40 40 Ex. 60 What is the value of sin 1000°2
cos2  + (cos2  )2
20 + cos2 1000°?
sin² sin  + sin2  = 1 (given)
40 (a) 1000 (b) 100 (c) 10 (d) 1

Rakesh Yadav Readers Publication Pvt. Ltd. 173

For More Visit : www.LearnEngineering.in


For More Visit : www.LearnEngineering.in

Ex.65 If sinx + sin²x =1, then the Sol. x = (1 + sin  ) (1 + sin  ) (1 sin  cos 
value of Ex.71 =
+ sin  ) .....(i) x y , then sin  –
cos12x + 3cos 10x + 3cos 8x +
cos6x –1 is equal to : x = (1 – sin  ) (1 – sin  ) (1 cos  is equal to
(a) 2 (b) 1 (c) 0 (d) –1 – sin  ) .....(ii) (a) x – y
(b) x + y
Sol.(c) sinx + sin²x =1 (i) × (ii)
x –y
 sinx =1– sin²x = cos² x x2 = cos2  .cos2  .cos2  (c)
 sin² x =cos4 x x ²  y²
x = + cos  cos  cos 
Now, cos12x +3cos10 x + 3cos8x y–x
+ cos6x)–1 cos  cos  (d)
Ex. 69 If cos  = m and sin  = n, x ²  y²
= (cos4 x+cos²x)³–1 = (sin²x +

r
sinx)³–1 sin  cos 
Find the value of Sol. = y = k (Let)
= 1³–1 = 0

Sni
(m2 + n2) cos2  x
Ex.66 If sin  + sin2  + sin3  = 1, sin  = kx ........ (i)

gv.i
Find the value of cos 6  – cos  cos  = ky ........(ii)
Sol. cos  = m
4cos4  + 8cos2  = ? (i)² + (ii)²
Sol. sin  + sin2  + sin3  = 1 cos  = m cos  ......(i) sin²  + cos²  = (kx)² + (ky)²

ridna
1 = k² (x² + y²)
 sin  + sin3  = 1 – sin2 
cos 
 sin  (1 + sin2  ) = cos2  1
sin  = n k=
 sin  (1 + 1– cos2  ) = cos2  x ²  y²

eeYa
cos  = n sin  ......(ii)
 sin  (2 – cos2  ) = cos2  Now, sin  – cos  = kx – ky = k (x – y)
from (i) and (ii) squaring both Put the value of k
Squaring. (for making sin 
sides
into cos  ) x –y
 m2cos2  = n2sin2 
geisnh  sin  – cos  =
 sin2  (2 – cos2  )2 = cos4  x ²  y²
 m2cos2  = n2(1 – cos2  )
 (1 – cos 2  ) [4 + cos 4  – 4 Ex.72 If 3sin  + 4cos  = 5 find the
 m²cos²  = n² – n²cos² 
cos2  ] = cos4 
value of tan  .
 4 + cos4  – 4cos2  – 4cos2  –  m²cos²  + n² cos²  = n²
Enak

Sol. 3sin  + 4cos  = 5


cos6  + 4cos4  = cos4   (m² + n²) cos²  = n²
3 4
 –cos6  + 4cos4  – 8cos2  = –4 Ex. 70 If r sin  = 1, and rcos  = 3 sin  + cos  = 1 ....(i)
5 5
R

Hence,
Then the value of
cos6  – 4cos4  + 8cos2  = 4 We know, sin²  + cos²  = 1
 
3 tan   1 And r or
aryn

Ex.67 If cos  + cos2  = 1, Find the


sin  .sin  + cos  .cos  = 1
value of sin8  + 2sin6  + sin4  Sol. rsin  = 1 ...........(i)
.... (ii)
Les B

Sol. cos  = 1 – cos2  rcos  = ..........(ii) comparison of (i) and (ii)


3
cos  = sin2  (i)2 + (ii)2 3
squaring both side Then, sin  =
2 5
cos2  = sin4  ......(i) (rsin  )2 + (rcos  )2 = (1) +  3
wa. th

8 6
Now, sin  + 2sin  + sin 4 4
r2sin2  + r2cos2  = 1 + 3 cos  =
sin q + 2sin q + sin q
8 6 4 5
r2 (sin2  + cos2  ) = 4
r2 = 4 3
a
2
2ab b
2
sin  5 3
wwM

r=2 tan  = = 4 =
(sin4  + sin2  )2 (i)/(ii) cos  5 4
From equation (i),
r sin  1 Alternative:-
So, (cos2  + sin2  )2 = 1 =
r cos  3 3 sin  + 4 cos  = 5
Ex.68 If x = (1 + sin  ) (1 + sin  ) (1 +
sin  ) = (1 – sin  ) (1 – sin  ) 1  
tan  =
(1 – sin  ) Then, Find the value 3 p b
of x
(a) + cos  .cos  .cos 
Now,  3 tan   1  tan  =
p
=
3
Put the value of tan  b 4
(b) + cos2  .cos2  .cos2 
1 Ex.73 If (a² – b²) sin  + 2ab cos 
(c) + sec  .sec  .sec  = 3× +1 = 2 = a² + b² then find the value of
3
(d) + sin  .sin  .sin  tan 

Rakesh Yadav Readers Publication Pvt. Ltd. 174

For More Visit : www.LearnEngineering.in


For More Visit : www.LearnEngineering.in

Sol. (a² – b²) sin  + 2ab cos  = a² Ex.75 If 29 tan θ =31 , find the value Sol. x sin θ = y cos θ .....(i) (given)
+ b² 1  2sinθ cos θ  x sin³ θ + y cos³ θ = sin θ .
of
  1 – 2sinθ cos θ cos θ .
p b Sol. (sin θ + cos θ )² = sin² θ + cos² θ  x sin θ . sin² θ + y cos3 θ
p a²–b² + 2 sin θ cos θ = sin θ .cos θ
tan  = =
b 2ab = 1+2 sin θ cos θ and (sin θ –  y cos θ . sin² θ + y cos³ θ
Ex.74 If xcos  – ysin  = x ²+y² , cos θ )² = 1– 2 sin θ cos θ . = sin θ .cos θ
1  2sinθ cos θ  y cos θ (sin² θ +cos² θ )
cos ² sin ² 1 Now, =
and + = x²  y² 1 – 2sinθ cos θ = sin θ .cos θ
a² b²

ir
2  y cos θ = sin θ .cos θ
which statement of true sinθ + cos θ

v.iSn
= 2  y = sin θ
(i)



=1
sinθ– cos θ Thus x = cos θ
a² b² Divide cos θ all terms Now, x² + y² = cos² θ + sin² θ = 1

dnag
x² y² 2
 tan θ  1 
Ex.78 10sin4 +15 cos4 = 6 , find
(ii) + =1   the value of 27 cosec6 + 8
a² b²  
 tanθ –1  sec6 .
x² y² Sol. 10 sin4 +15 cos4 = 6 ×1
(iii) + =1 31

eYari
b² a²  tan θ = (given)  10 sin 4 +15 cos 4 = 6×
29
x² y² (sin² θ +cos² θ )²
(iv) – =1 Put the value of tan θ
b² a²  10 sin4 + 15 cos4 = 6 (sin4 +
2
cos4 + 2 sin² cos²  )
Sol. xcos  – ysin  =

x cos 
+
(–y )sin 
x²  y²
 31

 29
=  31

 29
snhe
– 1

 1   60 
   29 
  2

  29 


2


divide by cos4 both sides
10 tan4 +15 = 6 (tan4 +1+2
tan² )
x ²  y ² = 1...(i)

x ²  y²
kgei
 10 tan4 +15 = 6 tan4 +6 +12
We know, = (30)² = 900
tan²
cos²  + sin²  = 1 .....(ii) Ex.76 If tan = n tan β and sin =  4tan4 – 12 tan² + 9 = 0
ERna

comparison of (i) and (ii) m sin β  (2 tan² –3) = 0


x Find the value of cos²  2 tan2 = 3
cos  = x ²  y² Sol. tan = n tan β 3
 tan² =
aBryn

2
–y sin nsinβ
sin  = x ²  y ²  Now, 27 cosec6 + 8 sec6 .
cos cosβ
= 27 (cosec² )³ + 8 (sec² )³
cos ² sin ² 1 put (sin = m sin β ) = 27 (1+cot² )³ + 8(1+ tan² )³
+ = x ²  y ² ....(iii)
Les

a² b² m cos β = n cos .....(i) 3 2


Put tan² = then cot² =
wa. th

Put the value of cos  & sin  in (iii) 2 3


m sin β = sin ....(ii)
2
adding after squaring (i) and (ii)  2 3  3 3
 x  1 = 27 1   + 8 1  
  = 27
   a2 + m² cos² β + m² sin² β = n²  3  2
 x ²  y² 
wwM

cos² + sin² 5 3 5 3
2  
  +8 
 
 –y  1 1 m²(cos² β + sin² β ) = n² cos² 3  2 
   b2 = x ²  y ² + (1– cos² )
 x ²  y ²  125 125
m² = n² cos² +1– cos² = 27 × +8× = 125
x² y²
27 8
m² – 1 = cos² (n²–1)
2 + +125 = 250
x ²  y ²  a x ²  y ²  b 2 m² –1 Alternate:-
cos² = 10 sin4 +15 cos4 =6
1 n² –1
= x²  y² 10 15
Ex.77 If x sin³ θ + y cos³ θ sin² .sin² + cos² .
= sin θ .cos θ . and x sin θ – y 6 6
x² y² cos² = 1 .....(i)
Hence, + =1 cos θ = 0 find the value of x² + y² sin² + cos² = 1 ....(ii)
a² b²

Rakesh Yadav Readers Publication Pvt. Ltd. 175

For More Visit : www.LearnEngineering.in


For More Visit : www.LearnEngineering.in

comparison (i) and (ii) Ex.80 If u n = cos n  +sin n  , then Sol. a cos θ +b sin θ = c ....(i)
10 15 the value of 2u6 – 3u4 + 1 is : a sin θ – b cos θ = x (let)
sin² = 1 and cos² = 1 (a) 1 (b) 4 (c) 6 (d) 0 ...(ii)
6 6
Sol. un = cosn  + sinn  (i)² + (ii)²
3 2  u6 = cos6  + sin6  a² + b² = c² +x²
sin² = and cos² = x² = a² + b² – c²
5 5 (cos2  )3 + (sin2  )3
Now, 27 (cosec² )³ + 8(sec² )³  (cos 2  + s in 2  ) 3 – 3 So, (a sin θ – b cos θ )² = a² + b² – c²
3 3 cos2  .sin2  (sin2  + cos2  ) 7
5  5  Ex.83 If sin θ + cos θ = , find the
= 27   + 8   = 27 × [ a3 + b3 = (a + b)3 – 3ab (a + b)] 5
3  2  1 – 3 cos2  .sin2 
 value of cos θ – sin θ =?
125 125 u6 = 1 – 3cos2  .sin2 

r
+8 × 7
27 8 u4 = cos4  + sin4  Sol. Sin θ + cos θ =
5

Sni
= 125 + 125 = 250  (cos 2  + s in 2  ) 2 –
2 2 cos θ – sin θ = x (let)
2cos  .sin 

gv.i
Ex.79 If Tn = sinn θ + cosn θ , then
 1– 2 cos2  .sin2   7 2
T3 – T5  (1)² + (1)² =   + (x)²
?  2u6 – 3 u4 + 1 5 
T1

ridna
 2 (1 – 3 sin2  cos2  ) – 3 (1– 2 49
(a) sin θ .cos θ sin2  cos2  )+1  2– = x²
25
 2–3+1=0
(b) sin2 θ .cos θ 49 1

eeYa
Alternate:-  x² = 2 – =
(c) sin2 θ .cos2 θ un = cosn  + sinn  25 25
(d) sin θ .cos2 θ u6 = cos6  + sin6  1
u4 = cos4  + sin4  So, x = 
T3 –T5 5
Sol.(c)
geisnh
Let  = 0°
T1 Ex.84 If sin θ + cos θ = 2 cos θ ,
Then, u6 = 1 & u4 = 1
3 3 5 5 Now, 2u6 – 3u4 + 1 then find the value of sin θ –
= sin  cos  – sin  cos   2×1–3×1+1=0
cos θ .
sin cos Sol. sin θ + cos θ = 2 cos θ
Enak

(B). If a sin θ + b cos θ = m.....(i)


sin θ – cos θ = x (let)
b sin θ – a cos θ = n .......(ii)
sin 3– sin 5  cos3– cos5   Then a²+b² = m²+n²  (1)² + (1)² = ( 2 cos θ )² + (x)²
R


sin cos Proof (i)² + (ii)²  2 = 2cos² θ +x²
3
 2
sin  1 – sin   cos  1 – cos  3
 2
  (a sin θ +b cos θ )² + (b sin θ – a  x² = 2 – 2 cos² θ
aryn


sin   cos  cos θ )² = m² + n²  x² = 2– 2 cos² θ
 (a² sin² θ +b²cos² θ + 2ab sin θ  x² = 2 (1– cos² θ )
Les B

sin3 . cos 2   cos 3 . sin2 


  x² = 2 sin² θ .
sin   cos  cos θ ) + (b² sin² θ + a² cos² θ –
sin2 . cos 2 sin   cos   2ab sin θ cos θ ) = m² + n² x=  2 sin θ

sin   cos   a² (sin² θ + cos² θ ) + b² (cos² θ + x y
wa. th

sin² θ ) = m² + n². Ex.85 If sin θ + cos θ = 1, then.


 sin . cos  2 2 a b
Alternative:- a² + b² = m² + n² y x
Let  = 45°, then Ex.81 If sin θ + cos θ = x, find the Find the value of sin θ –
b a
value of sin θ – cos θ
wwM

n n n  2 –n  cos θ =?
 1   1   1  
2 

Tn        2   2 Sol. 1 × sin θ + 1 × cos θ = x x y
 2  2  2 Sol. sin θ + cos θ = 1
Let 1×sin θ – 1× cos θ = n a b
1 1
– use property a²+b² = m²+n² y x
T3 – T5 2 –1/ 2 – 2 – 3 / 2 2 2 2 sin θ – cos θ = P (let)
   Here, a = b = 1 , m = x , n(let) b a
T1 21/ 2 2 (1)² + (1)² = (x)² + (n)²  x 2  y 2
1 2 – x² = n²   
    = (1)² + (P)²
 a  b 
1 n = ± 2 –x ²
 2 2  x ² y²
2 4 So, sin θ – cos θ = ± 2 –x ²   = 1 + P²
a ² b²
T3 – T5
i .e .  sin2 . cos 2  Ex.82 If a cos θ + b sin θ = c, Then x² y²
–1
T1 P=  
find (a sin θ – b cos θ )= ? a² b²

Rakesh Yadav Readers Publication Pvt. Ltd. 176

For More Visit : www.LearnEngineering.in


For More Visit : www.LearnEngineering.in

1 Now, (B). (s ec θ + tan θ ) ( s ec θ –


Ex.86 If sin θ + cos θ = , Then 6 tan θ )=1
2 5
find the value of sin θ – cos θ 1
=? 1 sec θ – tan θ =
secθ + tanθ
1 6 1
Sol. sin θ + cos θ = cosec θ = sec θ + tan θ =
2 5 secθ – tanθ
sin θ – cos θ = x (let) if sec θ + tan θ = x
5
1 2 Ex.90 If sec² θ + tan² θ = find the 1
3 then, sec θ – tan θ =
(1)² + (1)² =   + (x)² x
2  value of θ . while

ir
Ex.92 If sec θ + tan θ = 3, find the
1 (0º  θ  90º)

v.iSn
x² = 2 – value of cos θ
4 5 Sol. sec θ + tan θ = 3 ........(i)
Sol. sec² θ + tan² θ =
7 3
1
x² = then sec θ – tan θ =
4 5 3

dnag
1+ tan² θ + tan² θ = .......(ii)
7 3
x = adding (i) + (ii)
2 5
2 tan² θ = –1 1
Ex.87 If 3 sin θ + 4 cos θ = 5, find the 3 2 sec θ = 3+

eYari
3
value of 4 sin θ – 3 cos θ ?
2 10
Sol. 3 sin θ + 4 cos θ = 5 2 tan² θ = 2 sec θ =
3 3
4 sin θ – 3cos θ = x (let)


(3)² + (4)² = (5)² + x²
x=0
So, 4sin θ – 3cos θ = 0
tan θ = snhe
so, θ = 30º
1
3
sec θ =

3
5
3

Hence, cos θ =
kgei
TYPE - X Ex.91 If tan² = 1+ 2 tan² β , find 5
Ex.93 If sec θ + tan θ = x, find the
(A) 1+tan² θ = sec² θ the value of 2 cos – cos β value of tan θ
sec² θ –1 = tan² θ
ERna

=? Sol. sec θ + tan θ = x ....... (i)


sec² θ – tan² θ = 1
Sol. tan² = 1+2 tan² β (Using 1
Ex.88 If sec² θ + tan² θ = 9 find the sec θ – tan θ = ......(ii)
identity) x
value of sin θ (0º < θ <90º)
(i) and (ii)
aBryn

 sec² – 1=1 +2 (sec² β – 1)


Sol. sec² θ + tan² θ = 9
sec² –1=1 +2sec² β –2 1 x² –1
1 + tan² θ +tan² θ = 9  2 tan θ = x – =
x x
2 tan² θ = 8  sec² –1= 2sec² β –1
x ² –1
tan² θ = 4 sec² = 2 sec² β tan θ =
Les

 2x
p Ex.94 If sec θ + tan θ = 5, find the
wa. th

tan θ = 2, =  sec = 2 sec β


b value of sin θ .
1  1  Sol. sec θ + tan θ = 5 .......(i)
p 2   2 
 cos β 
Now, sin θ =  cos 1
5 h 5  
2 then sec θ – tan θ =
wwM

5
 cos β = 2 cos .......(ii)
1
Adding (i) and (ii)
Ex.89 If sec² θ + tan² θ = 11, find  2 cos – cos β = 0
1 26
the value of cosec θ Alternative:- 2 sec θ = 5+ =
5 5
(  oº< θ < 90º)  = 45º & β =0º satisfies
13 h
Sol. sec² θ + tan² θ = 11 tan² = 1 + 2 tan² β sec θ = =
5 b
1 + tan² θ + tan² θ = 11 Now,
put  = 45º & β = 0º in 2
2 tan² θ = 10
cos – cos β 13
tan² θ = 5 12 12
sin θ =
= 2 cos45º – cos0º = 1 – 1 = 0 13
tan θ = 5 5

Rakesh Yadav Readers Publication Pvt. Ltd. 177

For More Visit : www.LearnEngineering.in


For More Visit : www.LearnEngineering.in

Ex.95 If sec θ + tan θ = 5 , Then the Now, 2


sec θ + tanθ – sec θ – tan2 θ

tanθ  1 5 
value of is 3 tanθ – sec θ  1
tanθ –1
( sec² θ – tan² θ =1)
Sol. sec θ + tan θ = 5 .......(i) 4
sec θ + tan θ
secθ + tanθ – secθ  tanθsecθ –tanθ
1 
tanθ – secθ 1
sec θ – tan θ = .......(ii)
5 5 3 8
 = =2  sec θ + tanθ1 – sec θ  tanθ
(i) – (ii) 4 4 4
1 Take option (b) 2x = 2×1 = 2 tanθ – secθ  1
2 tan θ = 5 –  sec θ + tan θ
5 Ex.97 If tan2 θ = 1 – e2, then sec θ +

r
tan3 θ . cosec θ = ? 1 sinθ 1  sinθ 1 – sinθ
24

Sni
   
2 tan θ = 3/ 2 cos θ cosθ cos θ 1 – sinθ
5 (a)  2 – e 2 

gv.i
12 (b) (2 – e²)2/3
tan θ = 1 – sin ²θ cos2 θ
5 2 1/2  cos θ 1 – sinθ  cos θ 1 – sinθ

(c) 2 – e     

ridna
tan θ +1
Now, (d) None of these cos θ
tanθ – 1  So,
Sol. We have, sec  + tan3  .cosec  1 – sinθ
Put the value tan θ

eeYa
cosec  (C). a sec θ – b tan θ = c .....(i)
12 = sec  (1 + tan3  )
 1 17 sec  b sec θ – a tan θ = d
5 = 5  17
12 7 7 = sec  ( 1 + tan2  ) .....(ii)
–1 5 or
5 = sec  .sec2 
geisnh
1 a tan θ – b sec θ =d
Ex.96 If sec θ = x  (0º θ <90º), = sec3  = (sec2  )3/2 = (1 + Then, (a²–b² = c² – d²)
4x
tan2  )3/2 or a sec θ + b tan θ = c
then sec θ + tan θ is equal to
= (1 + 1 – e2)3/2 .....(i)
x 1  tan2  = 1 – e2 = (2 – e2)3/2
Enak

(a) (b) 2x (c) x (d) b sec θ + a tan θ = d


2 2x Alternate:- .....(ii)
1 Let  = 45º then, (a²–b² = c² – d²)
R

Sol. sec θ = x + (given) .....(i) Proof (i)² – (ii)²


4x Then, tan²  = 1– e²
tan² 45 = 1– e²  (a sec θ – b tan θ )² – (b sec θ –
Let sec θ + tan θ = a .....(ii)
aryn

1 = 1– e² a tan θ )² = c² – d².
1 e² = 0  (a² sec² θ +b² tan² θ – 2ab
Then sec θ – tan θ = ....(iii)
Les B

a Now, sec  + tan³  .cosec  sec θ tan θ )


(ii) + (iii) Put  = 45º –(b² sec² θ + a² tan² θ –2ab
1 = sec45º + tan²45º.cosec45º sec θ tan θ )= c² – d²
2 sec θ = a+ a² sec² θ + b² tan² θ – b²sec² θ –
= 2 + 1× 2 
wa. th

a
a²tan² θ = c² – d²
a 1
+ = 2 2 = (2) 3 2  a² (sec² θ – tan² θ ) – b²(sec² θ
sec θ = .....(iv)
2 2a Now, we take option (a) – tan² θ ) = c² – d²
Compare (i) and (iv) 3 a² – b² = c² – d²
wwM

(2–e²) 2
a Ex.99 If a sec θ – b tan θ = 10
Hence, x = , a = 2x Put e² = 0
2 and b sec θ – a tan θ = 5
3
Then = 22 find the value of a² – b²
so, sec θ + tan θ = 2x
Sol. a sec θ – b tan θ = 10
Alternate:- So, (a) option is correct
Ex.98 Find the value of b sec θ – a tan θ = 5
1 (use property)
sec θ = x + sec θ + tanθ – 1
4x a² – b² = (10)² – (5)²
Let x = 1 tanθ – sec θ  1 a² – b² = 100 – 25 = 75
Ex.100 If 5 sec θ – 3 tan θ = 7 find
1 5 sec θ + tanθ – 1
Then, sec θ = 1 + = Sol. the value of
4 1 4 tanθ – sec θ  1
3 sec θ – 5 tan θ =?

Rakesh Yadav Readers Publication Pvt. Ltd. 178

For More Visit : www.LearnEngineering.in


For More Visit : www.LearnEngineering.in

Sol. 5 sec θ – 3 tan θ = 7 (i) and (ii) 16 – 25 = 49 – m²


3 sec θ – 5 tan θ = m. (let) 1 m² = 49 + 9
(5)² – (3)² = (7)² – (m)² 2 cosec θ = 4+ m= 
4 58
25 – 9 = 49– m²
17 TYPE – XII
m² = 33 2 cosec θ =
4 (A) If A+B = 45º or 225º
m=  33
17 h then,
TYPE- XI cosec θ = = (i) (1+tanA) (1+tanB) = 2
8 p
and
(A) 1+ cot² θ = cosec² θ 2 2
b= 17 – 8 = 15 (ii) (1–cotA) (1– cotB) = 2
cosec² θ –1 = cot² θ Proof

ir
cosec² θ – cot² θ = 1 Now, (i) A+B = 45º

v.iSn
Ex.101 cosec² θ + 2 cot² θ = 10, then 17 Both side take tan.
8 tan (A+B)= tan45º
find the value of sin θ +
cos θ when o° < θ <90º tanA + tanB
15 =1

dnag
Sol. cosec² θ + 2cot² θ = 10 1– tanA tanB
1 + cot² θ + 2 cot² θ = 10 b 15
cos θ = =  tanA + tanB = 1– tanA tanB
3 cot² θ = 9 h 17
 tanA + tanB + tanA tanB = 1
cot θ = Ex.104 If cosec θ + cot θ = 5 +2,

eYari
3 Adding 1 both side.
So, θ = 30º then find the value of sin θ .  1+ tanA + tanB + tanA tanB =
Now, sin θ + cos θ Sol. cosec θ + cot θ = 5 +2 1+1
= sin30º + cos30º .....(i)  1(1+tanA) + tanB (1+tanA) = 2

=
1
2
+
3 1 3
2
=
2
Then,
snhe
cosec θ – cot θ =
1
52
= (ii)
Hence, (1+tanB) (1+tanA) = 2
A+B = 45
Both side take cot.
Ex.102 If cosec² θ + cot² θ = 3, find 5 –2 .....(ii) cot (A+B)= cot45º
kgei
the value of cos θ . when (i) + (ii) cot A . cot B –1 1
(0º< θ <90º)  
2 cosec θ = 2 5 cot A + cot B 1
ERna

Sol. cosec² θ + cot² θ = 3  cotA cotB –1 = cotA + cot B


cosec θ = 5
1+cot² θ + cot² θ =3  cotA [cotB –1] –1– cotB +1 –
2 cot² θ =2 1 1=0
So, sin θ =
cot² θ = 1 5  cotA [cotB –1] –1 [cotB –1] = 2
aBryn

cot θ = 1 (c) a cosec θ – b cot θ = c  (cotA–1)(cotB –1) = 2


So, b cosec θ – a cot θ = d Ex.106 Find the value of
θ = 45º
or (1+tan5º) (1+tan40º)
Now, cos θ
Sol. A+B = 5° + 40° = 45º
1 a cot θ – b cosec θ = d
then, (1+tan5º) (1+tan40º) = 2
Les

= cos45º = (i)² – (ii)²


2 Then, (a² – b² = c² – d²) Ex.107 Find the value of
wa. th

(B) cosec² θ – cot² θ = 1 or (1+tan1º) (1+tan2º) (1+tan3º)


(cosec θ – cot θ ) (cosec θ + (1+tan44º)(1+tan43º)
a cosec θ + b cot θ = c
(1+tan42º)
cot θ ) = 1 ....... (i)
Sol. (1+tan1º) (1+tan44º)
cosec θ – cot θ b cosec θ + a cot θ = d
wwM

(1+tan2º) (1+tan43º)
.......(ii)
1 (1+tan3º) (1+tan42º)
= (i)² – (ii)²
cosecθ + cotθ (a² – b² = c² – d²) 1º+44º = 2º+43º
If cosec θ – cot θ = x, = 3º+42º = 45º
Ex.105 If 4 cosec θ + 5 cot θ = 7,
so, 3 pair of such term
1 then find the value of 5
then , cosec θ + cot θ = = 2×2×2 = 8
x cosec θ + 4 cot θ =?
Ex.108 If A+B = 45º, then find
Ex.103 If cosec θ – cot θ = 4, find Sol. 4 cosec θ + 5 cot θ = 7
the value of cos θ (given) tanA tanB
. =?
Sol. cosec θ – cot θ = 4 .....(i) 5 cosec θ + 4 cot θ = m (let) 1– tanA 1– tanB

1 Using identity tanA tanB


then, cosec θ + cot θ = .....(ii) (4)² – (5)² = (7)² – (m)² Sol. .
4 1– tanA 1– tanB

Rakesh Yadav Readers Publication Pvt. Ltd. 179

For More Visit : www.LearnEngineering.in


For More Visit : www.LearnEngineering.in

(i) sin(60– θ ) sin θ .sin(60+ θ ) Ex.113 The value of


1 1 sin12º.sin48º.sin54º =?
cotA . cotB 1
= sin3 θ Sol. sin12º.sin48º.sin54º
= 1 1 4
1– 1– (sin72ºdivide and multiply)
cotA cotB
(ii) cos(60 – θ ) cos θ .cos(60+ θ )  sin12º. sin4 8º.s in72 º ×
1 1 1 1
= . = = cos3 θ .sin54º
cot A –1 cot B –1 4 sin72º
(iii) tan (60 – θ ) tan θ .tan(60+ θ ) here θ = 12°
1 1 1
. = = tan3 θ
cot A –1 cot B –1 2 1 1
Ex.110 The value of tan10º tan20º  sin3×12º × .sin(90º–36º)
(B) If A+B+C = 180º tan40º =? 4 sin 72º

r
then, Sol. Here θ = 10º
1 1

Sni
(i) tanA + tanB + tan C  tan(10º)tan(2×10º)tan(4×10º)  sin36º.cos36º ×
= tan(3×10º) 4 sin 72º
= tanA.tanB.tanC

gv.i
(ii) cotA.cotB + cotB.cotC + 1 1
cotC. cotA=1  tan30º =  .2 sin3 6º. cos3 6º
3 42
(iii) sin2A+sin2B+sin2C Ex.111 The value of

ridna
= 4sinA.sinB.sinC 1
sin20º.sin40º.sin80º =? ×
Proof, Sol. sin20º.sin40º.sin80º sin 72º
A+B+C = 180º    1 1

eeYa
A+B = 180º – C 60– θ 60+ θ  sin2×36º ×
θ 8 sin72
Both side take tan
Here, θ = 20º
tan (A+B) = tan (180º–C) 1 1 1
1  sin72º× =
tanA + tanB – tanC = sin3 θ 8 sin 72º 8
=  4
geisnh
1– tanA tanB 1 Ex.114 The value of 1–sin10º sin50º
1 sin70º = ?
 tanA + tanB = – tanC + tanA = sin(3×20º)
tanB TanC 4 Sol. 1–sin10ºsin50ºsin70º
 so, tanA+tanB+tanC 1 here θ = 10º
Enak

= tanA.tanB.tanC = sin60º
4 1 
(tanA.tanB.tanC) divided by  sin3 10º 
= 1–  
both side
1 3 3 4 
× =
R

then, 4 2 8
1 1 1 7
Ex.112 The value of = 1– ×sin30º = 1– × =
1 1 1 4 4 2 8
+
aryn

+ =1  5 7 3
tanB.tanC tanA.tanC tanA.tanB sin . sin . sin . sin . is TYPE - XIV
cotA cotB + cotB cotC + 9 9 9 9
Les B


cotC.cotA =1 equal to T-radius of Multiple Angles :-
Ex.109 The value of tan37º + tan73º  5 7 3 2 t an A
Sol. sin . sin . sin . sin . (i) sin 2 A  2 sin A. cos A 
+ tan70º is equal to 9 9 9 9 1  tan 2 A
Sol. Sum of angle 37º + 73º +70º =
(ii) cos 2 A cos 2 A si n ² A 2 cos 2 A – 1
wa. th

Put value of  = 180º


180º
 sin20º. sin100º. sin140º.
so, tan37º + tan73º + tan70º 2 1  tan 2 A
= tan37º.tan73º.tan70º
sin60º =1  2 sin A 
 sin20º. sin(180 º–80 º). 1  tan 2 A
TYPE - XIII
wwM

3 2 t an A
sin(180º–40º). (iii) t an 2 A 
Morri's law
2 1  t an 2 A
If 4 θ < 60º
3 (iv) sin 3 A  3 sin A  4 sin 3 A
1  sin20º. sin40º. sin80º.
(i) sin θ .sin2 θ .sin4 θ =
4
sin3 θ 2 (v) cos 3A  4 cos3 A  3 cos A
[  sin(180º – θ )= sin θ ]
(ii) cos θ .cos2 θ .cos4 θ 3 tan A  t an 3 A
(vi) t an 3 A 
1 
1
sin60º ×
3 1  3 t an 2 A
= cos3 θ 4 2
4 (vii) sin C + sin D = 2
(iii) tan θ .tan2 θ .tan4 θ = tan3 θ 1 3 3 3  C+D   C–D 
 × ×  sin 
 2   .cos 
 2  
For all value of θ 4 2 2 16    

Rakesh Yadav Readers Publication Pvt. Ltd. 180

For More Visit : www.LearnEngineering.in


For More Visit : www.LearnEngineering.in

(viii) sin C – sin D 1


1
 cos15º.sin15º 22 =A
 C+D   C–D  2 2
= 2 cos 
 2   .sin 
 2   45º = 2A
    Again multiply and divide by 2
(ix) cos C + cos D = 2 c os 1 1
 ×2 sin15.cos15º  cos45º =1–2sin²22
 C+D   C–D  2 2 2

 2   .cos 
 2  
    1 1 1 1  2 1 
 .sin30º  × = sin 22  x (let)
 
(x) cos C – cos D = 2 s in 4 4 2 8  2 
 C+D   C–D  Ex.117. The value of 1

 2   .sin 
 2    = 1 – 2x²

ir
    sin2x 2

v.iSn
x 1
1 sin
Ex.115. If sin 2 x  , the value of 4  2x² = 1 –
2
5
sin2x 2 –1
(sin x  cos x ) is :- Sol.  x² =

dnag
x 2 2
sin
7 4 4
2 –1 2
(a) (b)
5 5 First we solve sin2x x=
2 2

2

eYari
 sin2x = 2sinx.cosx
6 2 2– 2 2– 2
(c) (d) x  = =
5 5 2sin2 
4 2
   . cosx
2 Ex.120 The value of
1 3 sin20º – 4sin³20º
Sol. sin2x =
5
add 1 both side
= 4sin snhe
x
2
x
.cos .cosx
2
Sol. 3sin20º – 4sin³20º = sin3×20º
= sin60º
kgei
1 6  x  x 3
1 + sin2x = 1 + = = 4 sin 2 
  .cos . cosx
 =
5 5 2 2  2 2
Ex.121 Find the value of
6
ERna

 sin 2 x  cos 2 x  2 sin x . cos x  x x x


5 = 4×2 sin .cos .cos .cosx 1 3
4 4 2 –
now, sin10 cos10
[ sin 2 x  cos 2 x  1 and
x x x 1 3
sin 2 x  2 sin x . cos x ] 4  2sin .cos .cos .cos x
aBryn

4 4 2 Sol. –
 sin10 cos10
2 6 x
  sin x  cos x   sin
5 4 cos10 – 3 sin10
=
x x sin10 cos10
 8 cos .cos .cosx
6 4 2
Les

 sin x  cos x  1 3 
5 2  cos10 – sin10 
wa. th

2 1
Ex.116 The value of
1 – tan 22 = 2 2 
2
Ex.118. Find the value of 1 sin10 cos10
0 0 2
1 1 1  tan 22
cos15º. cos7 .sin7 . =? 2 1
2 2 We can write = sin30º &
wwM

1 – tan²A 2
0 0
1 1 Sol. cos2A =
Sol. cos15º. cos7 .sin7 1  tan2 A 3
2 2 = cos30º
2
Multiply and divide by 2
1
1  cos2 ×22 = cos45º 2 sin30 cos10 – cos 30º sin10
cos15º. 2 =

2 sin10 cos10
1 multiply and divide by 2
 0 0 =
2cos 7 1 .sin7 1  2
2.2sin 30 – 10
 2 2  =
 1 2sin10 cos10
Ex.119 Find the value sin 22º =?
1 15 2 4sin20
 cos15º×sin2.
2 2 Sol. [cos2A = 2 cos²A–1 = 1–2sin²A] = =4
sin2  10

Rakesh Yadav Readers Publication Pvt. Ltd. 181

For More Visit : www.LearnEngineering.in


For More Visit : www.LearnEngineering.in

(Using formula; 2sin A.sinB = cos(A 1


2cos y – 1 – B) – cos(A + B)
Ex.122. If cos x = then find n = tan  – sin  = 1 –
2 – cos y 2
1  Now, m² – n² = (m + n) (m –n)
the value of 1 – 2  – cos 80 
2 
=  1  1
x y 2sin(90 – 80) = (2) ×  2   = 4 ×
tan cot 2 2
2 2
1 –1  2cos 80
= take option (a) 4 mn
cos x 2cos y – 1 2cos 80
Sol. = 2 – cos y
1  cos 80  1  1 
= 1 = mn = 1   1 – 
 Applying Componendo-Divi- 2cos 80 2  2
dendo rule 2
TYPE- XV

r
 1  1 1
cos x  1 = (1)² –  = 1– =
 2 

Sni
 Trigonometry expression is inde- 2 2
cos x – 1 pendent of angle so we can put any

gv.i
value of θ except result should not 1
2cos y – 1  2 – cos y So, 4 mn = 4×
= 2cos y – 1 – 2  cos y indetermine 2
  0 1 So option (a) is correct.
, . , 

ridna

cos x  1 1  cos y   0 0

 = 3 cos y – 1 Ex.125 If si n   c os   p and
cos x – 1
1 1 1 sec   cos ec  q , then what
Note:- =0, = =0
  1 

eeYa
x is the value of 2p:
2cos2 –11  It is better to put θ = 0º if ex-
2 (a) p(q 2  1)
 x pression does not contain cosec θ
1 – 2sin2 – 1 (b) p(1  q 2 )
2 or cot θ otherwise θ = 45º
(i)
geisnh
If sin θ ,cos θ in equation, Try (c) q(1  p 2 )
y 2 to put θ =0º or 90º
1  2cos –1 (d) q(p 2  1)
2 (ii) If sin θ , cos θ , tan θ , sec θ ,
=  y  cosec θ , cot θ try to put θ = 45º Sol. Let   45 
3 1 – 2sin2 – 1
 2  Ex.124 If tan  +sin  =m and tan  -
Enak

then p = sin45°+cos45° = 2
sin  = n, then find the value
and
x y of m² –n² =?
2cos 2 2cos 2 q = sec 45   cosec45   2 2
2 (a) 4 mn
R

2 = (b) mn

2 x  2 y Now, 2p = 2 2
–2sin 3  –2sin  (c) m²n² (d) m³n³
2  2 Sol. m2-n2 = (tan  -sin)2 - (tan  - Take option (d)
aryn

sin  )2 2
x 1 y 
 q  p  1  2 2  2
2
  –1  2 2  2p
Les B

 cot2 = cot2
2 3 2 = 4 tan  . sin  
[ (a  b)2  (a  b)2  4ab] Hence, option (d) is correct.
x y
 3 = tan2 .cot² = 4 tan  .sin  Ex.126 Find the value of :
2 2
= 4 tan ².sin² 1  2 sec 2 A. tan 2 A  sec 4  tan 4 A
wa. th

x y (a) 0 (b) 1
Hence, tan
2
.cot =
2 3 = 4 tan ²  1 – cos ²  (c) sec²A.tan²A
= 4 tan ² – tan².cos ² (d) None of these
1
Ex.123 Find the value of cosec10° Sol. 1   sec 4 A  t an 4 A  2 sec 2 A. tan 2 A 
wwM

2 sin ²
= 4 tan ² – .cos ² 2
– 2sin70° cos ² = 1  sec 2 A  t an 2 A
 
1 = 4 tan² – sin ²
Sol. cos ec10 ° – 2sin70° Now, mn = (tan  + sin  ) = 1 1  0
2
(tan  –sin  ) Alternative:-
1 mn = tan²  – sin²  ....(i)
= –2sin70° Take A = 45°, then
2sin10 From equation (i) Given Exp.
1 – 2.2sin70 sin10 = 4 mn =1 + 2 sec2 45º. tan2 45º - sec4
= Alternative:- 45º-tan4 45º
2sin10
Let  = 45° 2 4
1 – 2 cos 60 – cos 80
=
m = tan  + sin  = 1 +
1 =1  2  2 1   2 1
2sin10
2 =0

Rakesh Yadav Readers Publication Pvt. Ltd. 182

For More Visit : www.LearnEngineering.in


For More Visit : www.LearnEngineering.in

1  cos  2x 1  sin 2  2–
1

1
Ex.127 If x  , then =? Sol. m = cosec  - sin  = l³=
1  cos  1  x2 sin  2 2
(a) sec  (b) t an  (c) cos2  2–
1

1
= and n = sec  – cos  m³ =
cot  (d) cos  sin  2 2
Let   60   l ³ = m³
Sol. 1 – cos 2  sin 2 
=  So, l = m
1 cos cos
 l 2 m2 (l 2 + m2 )
1 cos2  sin 2 
x 2  1 mn  .  sin .cos 
Put (l = m)
1 
 3 sin  cos   l ² ×l ² (l ² + l ²)
1
2 2 2 2
 cos2   3
 sin 2   3
 1 

ir
2 2
and m 3
n 3
   6
2l = 2 (l )² = 2 ×  3
=1
 sin  

 cos  
  2 

v.iSn
2x 2/ 3
   3
1x 2
1 4 4 Ex.131 If a cos³  + 3a cos  . sin²  =
1– cos 3
 sin 3

3  2
 2
m and a sin³  + 3a sin  . cos² 
sin 3
 cos 3
 = n, then the find value of
and option (B). tan  = tan 60°

dnag
2 2
= 3 cos 2   sin 2  1 m  n  3
 m  n  3
?
= 2
 2
& option (B) is correct.  sin  . cos   3
m n  3
(a) 2a
1
3 (b) 2a
2
3
Note:- at   0  cot   

eYari
2
=  mn  3
(c) a 2 3 (d) a 1 3
  can't be 0° & at  =45°
option (B) and (C) contradicts. Alternative:-
Sol. m + n = a cos3 θ + 3 a
Ex.128 If x = cosec  – sin  and y = Let   45 , then
cos θ .sin2 θ + a sin3 θ + 3a
sec  – cos  , then the value of
x2y2 (x2 + y2 + 3) is :
(a) 0 (b) 1 (c) 2 (d) 3
m=
1
2
and n =
1
2
snhe 2 2
sin θ . cos2 θ
= a (cos θ + sin θ )3
Similarly,
 1 3  1 3 m – n = a (cos θ – sin θ )3
kgei
Sol. take  = 45° 2 2

 m n 3 3
    m n 1/3

1 1 1 1  2  2  cos θ  sin θ    and


 x 2–  &y  2 –    a 
2 1
2 2 2 2  1  3
1  3 1/3
ERna

 2.    2.   m – n
cos θ – sin θ  
 x 2y 2 x 2  y 2  3 
1 1 1 1   2 2   a 

      3

2 2 2 2  1 1
= 2 1–
= 22/3 2/ 3 2 /3
 2.2 3 3 m  n m – n
1     

aBryn

 4  1 2  a   a 
4 2 1  3 2
&  mn  3   2 3 2
 cos θ  sinθ  cos θ – sinθ
2
Ex.129 If m  cos ec  sin  and n = 2
sec  - 2 2 2  2 (cos2 θ + sin2 θ ) = 2
 m 3
n 3
  mn  3
 (m + n)2/3 + (m – n)2/3 = 2a2/3
cos  , then find the value of
Les

2 2 Ex.130 If cosec  – sin  = l³ and sec  Alternate:-


m 3
n 3
?
wa. th

– cosec  = m³, then the value Let θ = 0°, then


2 2
(a) (m n ) 3 (b) (m n ) 3 of l 2 m2 (l 2 + m2 ) is : m = a and n = 0
1 1 (a) –1 (b) 0 (c) 1 (d) 2 (m + n)2/3 + (m – n)2/3 = a2/3 +
(c) (m n ) 3 (d) (m n ) 3
a2/3 = 2a2/3
Sol. take  = 45°
wwM

Rakesh Yadav Readers Publication Pvt. Ltd. 183

For More Visit : www.LearnEngineering.in


For More Visit : www.LearnEngineering.in

EXERCISE

tan  cot   1
1. = 2 , 0   90 , (a) sin (b) cos 18. If 2 cos θ – sinθ = ,
tan – cot 2 3 2
then the value of sinis  2
(c) sin (d) cos 0  θ < 90°  th e value of
2 3 1 3 3
(a) (b) (c) (d) 1 11. If sin sec(30° + ) = 1, 2 sin θ+cosθ is
3 2 2
(0< <60°), then the value of 1 3 1
2. The value of cot 10°. cot 20°.

r
sin + cos 2 is (a) (b) 2 (c) (d)
cot 60°. cot 70°.cot 80° is 2 2 3

Sni
1 2 3
(a) 1 (b) sin θ+cosθ

gv.i
(a) 1 (b) –1 (c) 3 (d) 2 3 19. If = 3 th en the
3 sin θ – cos θ
3. The value of cot18° (c) 0 (d) 2 value of sin4 θ – cos4 θ is

ridna
 1  12. If tan θ = 1, then the value of
cot 72cos2 22 
  is

 tan 72  sec 2
68   1 3 2 4
8 sin θ+5cosθ (a) (b) (c) (d)
is 5 5 5 5
1 sin3 θ – 2 cos 3 θ  7 cos θ

eeYa
(a) 1 (b) 2 (c) 3 (d) 2
3 20. If sec2 θ + tan θ = 7, then the
1 4
(a) 2 (b) 2 (c) 3 (d) value of θ when 0   θ  90°, is
4. If tan 15° = 2 – 3 , the value of 2 5
(a) 60° (b) 30° (c) 0° (d) 90°
tan 15° cot 75°+ tan 75°cot15°is
13. If ' θ ' be a positive acute angle sat-
(a) 14 (b) 12 (c) 10 (d) 8
geisnh 21. The simplified value of
4
5. If x, y are acute angles, 0 < x + isfying cos2θ + cos θ = 1, then the (sec x se cy +tan x tan y) ² –
y < 90° and sin (2x –20°) = cos value of tan2θ + tan4θ is 2
sec x tan y  tan x sec y  is :
(2y+20°), then the value of tan
3 1
Enak
(x + y) is : (a) –1 (b) 0 (c) sec2 x (d) 1
(a) (b) 1 (c) (d) 0
2 2
1 3 2 4
(a) (b) (c) 3 (d) 1 22. If A = sin θ + cos θ for any
3 2 4
R

14. If tan θ = , then the value of value of θ , then the value of A is


6. sin25°+sin210°+sin215° +.........+ 3
sin285° +sin290° is equal to 3 sin θ+2cosθ 3
aryn

is (a) 1  A  1 (b)  A 1
1 1 1 3 sin θ–2cosθ 4
(a) 7 (b) 8 (c) 9 (d) 9
Les B

2 2 2 (a) 0.5 (b) –0.5 (c) 3.0 (d) –3.0


13 3 13
sin 39
15. The simplified value of (secA – (c)  A  1 (d)  A 
cosA)² + (cosecA – sinA)² – 16 4 16
7. The value of + 2 tan
cos 51
11° tan 31° tan 45° tan 59° tan (cot A – tan A )2 is 23. If sin θ + cosec θ = 2, then the
wa. th

79° – 3(sin221°+sin269°) is ; 1 value of sin5 θ +cosec5 θ , when


(a) 2 (b) – 1 (c) 1 (d) 0 (a) 0 (b) (c) 1 (d) 2 0°  θ  90° is
2
2
cos θ (a) 0 (b) 1 (c) 10 (d) 2
8. If 3 and 16. If ' θ ' be an acute angle and
wwM

cot θ – cos2 θ
2
24. If tan2θ. tan4θ = 1, then the
tan θ + cot θ = 2, then the value
0° < θ < 90°, then value of θ is: of tan5θ + cot10 θ is value of tan 3θ is
(a) 30° (b) 45°
(a) 1 (b) 2 (c) 3 (d) 4 1
(c) 60° (d) None of these (a) 3 (b) 10 (c) 1 (d)
3
9. If A = tan 11° tan 29°, B = 2 cot 7
61° cot 79°, then ; 17. If sinθ – cos θ = and 0° < θ <
13 2 2
(a) A = 2B (b) A = – 2B 25. If cos   cos β = 2, then the
90°, then the value of sinθ +
(c) 2A = B (d) 2A= – B value of tan3  sin5 β is ;
cosθ is
10. If sin + cos =2;( 0  β < 
17 13 1 1 1
 2  β  (a) (b) (c) (d) (a) – 1 (b) 0 (c) 1 (d)
 90° ), then sin 
 =
 13 17 13 17 3
 3 

Rakesh Yadav Readers Publication Pvt. Ltd. 184

For More Visit : www.LearnEngineering.in


For More Visit : www.LearnEngineering.in

26. In a triangle ABC, ABC = 75° 35. If sec(7 θ +28º )= cosec (30º – 45. If tan 8  = cot2  , where 0 < 8 
c 3 θ ), then the value of θ is 
and ACB = , The circular < , then what is the value of
4 (a) 8° (b) 5° (c) 60° (d) 9° 2
measure of  BAC is    tan 5  ?
5  36. If tan 
 –   3 , the value of 1
(a) radian (b) radian 2 2
12 3 (a) (b) 1 (c) 3 (d) 0
cos θ is: 3
  46. If sin(A + B) = 1, where 0 < B <
(c) radian (d) radian 1 1
6 2 (a) 0 (b) (c)(d) 1 45°, then what is cos(A – B) equal
27. The angles of a triangle are (x + 2 2 to?
5 )°, (2x– 3)°and (3x+4)°. The 37. The value of cos25º – sin25º (a) sin2B (b) sinB

ir
value of x is (a) positive but less than 1 (c) cos 2B (d) cosB

v.iSn
(a) 30º (b) 31º (c) 29º (d) 28º (b) positive but greater than 1 47. If 5sin  + 12cos  = 13, then what
28. If A and B are complemen- (c) negative is 5cos  – 12sin  equal to?
tary to each other, then the (d) 0 (a) –2 (b) –1 (c) 0 (d) 1
valu e of S ec 2 A+S ec 2 B– 48. If 4 tan = 3, the n what is
38. In a right angled  ABC, right

dnag
2 2
Sec A.Sec B is 4sin  – cos 
(a) 1 (b) –1 (c) 2 (d) 0
4 equal to?
angle at B, if cos A = , then 4sin   9cos 
5
x what is sin C is equal to? 1 1 1 1

eYari
29. If sin 17°= y , then the value of (a) (b) (c) (d)
2 3 4 6
3 4 3 2
(sec17°–sin73°) is (a) (b) (c) (d) 49. If sin  – cos  = 0, then what is
5 5 4 5
sin4  + cos4  equal to?
y2 x2 39. I f  a n d  a r e c o m p l e m e n -
(a) 2
x y –x

x 2
2 (b)
y y –x

y
2

2
2
snhe
t a r y a n g l e s , then what is
cos cosecβ – cos sinβ equal to?
(a) 1
50. What is
(b)
3
4
(c)
1
2
(d)
1
4

(a) sec β (b) cos 


kgei
(c) (d) sin   cos tan   cot 
y x 2 – y2 x x 2 – y2
(c) sin  (d) –tan β sec   cosec
40. If 2 cot  = 3, then what is equal to
cot 30 – cot 75 
ERna

30. The value of is (a) 1 (b) 2


tan15 – tan 60 2cos  – sin  (c) sin  (d) cos 
equal to?
(a) 0 (b) 1 2cos   sin  51. What is s in 6  + cos 6  +
(c) 3 – 1 (d) –1 2 1 1 3 3sin2  cos2  equal to?
aBryn

(a) (b) (c) (d) (a) 0 (b) 1 (c) 2 (d) 4


31. The value of 3 3 2 4
52. What is
cot θ.tan 90 – θ – sec 90 – θ 1 1  sec  – tan  cos 
41. If sin  cos  = , then what is
2 2 2 1  sec   tan 1 – sin  equal
cose cθ+ sin 25  sin 65 
  sin6  + cos6  equal to? to?
Les

+ 3tan5tan15.tan30.tan75.tan85 1 (a) 1 (b) 2


wa. th

(a) 1 (b) 2 (c) 3 (d) (c) tan  (d) cot 


is 4
(a) 1 (b) –1 (c) 2 (d) 0 42. If sec  + tan  = 2, then what is 53. If sin  + cos  = 3 , then what
32. If sin(3x–20°) = cos(3y+20°) then the value of sec  ? is tan  + cot  equal to?
the value of (x +y ) is
(a) 1 (b)2 (c) 2 (d) 3
wwM

(a) 20° (b) 30° (c) 40° (d) 45° 3 5 5


(a) (b) 2 (c) (d) 54. If tan  + sec  = m, then what is
2 2 4
33. If cosθ cosec23°=1 , the value of
43. What is cosec (75°+  ) – sec sec  equal to?
θ is
(1 5°–  ) –tan(55 °+  ) + cot m2 – 1 m2  1
(a) 23° (b) 37° (c) 63° (d) 67° (a) (b)
(35°–  ) equal to? 2m 2m
2 2
34. If 2 cos θ – sin θ = 1, θ is a
  3 m 1 m2  1
positive acute angle, then the (a) –1 (b) 0 (c) 1 (d) (c) (d)
2 m m
value of θ is 44. If sin  + 2 cos  = 1, where 0 <  55. What is cosec (75° +  ) – sec(15°
(a) 60° (b) 30° –  ) equal to?
<  /2, what is 2sin  – cos  equal
1 to? (a) 0 (b) 1
(c) 45° (d) 22
2 (a) –1 (b) 1/2 (c) 2 (d) 1 (c) 2sin  (d) 2cos 

Rakesh Yadav Readers Publication Pvt. Ltd. 185

For More Visit : www.LearnEngineering.in


For More Visit : www.LearnEngineering.in

56. If  ABC is right angled at C, 1 2


II. There is only one values of x
then what is cos(A + B) + sin(A + (a) (b) in the first quadrant that sat-
10 10
B) equal to? isfies sinx – cosx = 0.
3 4 Which of the statements above
1 (c) (d)
(a) 0 (b) (c) 1 (d) 2 10 10 is/are correct?
2 (a) Only I
57. If  ,  and  are acute angles 64. If 0°<  < 90°, then all the trigo- (b) Only II
nometric ratios can be obtained (c) Both I and II
3 3 when (d) Neither I nor II
such that sin  = , cos  =
2 2 (a) Only sin  is given 72. If 7 sin2 θ+3 cos2 θ =4, 0  θ  90 ,
and tan  = 1, then what is  + (b) Only cos  is given then value of θ is

r
 +  equal to? (c) Only tan  is given
(a) 105° (b) 120° (c) 135°(d) 150°    

Sni
(d) any one of the six ratios is (a) (b) (c) (d)
58. If cosA + cos2 A = 1, then what is 2 3 6 4
given

gv.i
the value of 2(sin2A + sin4A)? 73. If tan 2θ  45 = cot 3θ ,where
65. What is the value of sin A cos A
(a) 4 (b) 2 (c) 1 (d) 1/2
tanA + cos A sin A cot A?
59. (1 – tanA)2 + (1 + tanA)2 + (1 – 2θ+45° and 3 q are acute
(a) sin2A + cosA

ridna
cotA)2 + (1 + cotA)2 is equal to
(b) sin2A + tan2A angles, then the value of θ is
(a) sin2A.cos2A
(c) sin2A +cot2A (a) 5° (b) 9° (c) 12° (d) 15°
(b) sec2A.cosec2A
(d) cosec2A – cot2A 74. If θ be acu te ang le and
(c) 2sec2A.cosec2A

eeYa
(d) None of these sin  15
66. What is the value of + cos θ= , then the value of
60. What is the v alue of 1  cos 
17
tan A – sin A 1  cos 
? ? cot 90 – θ is
sin3 A
geisnh
sin 
(a) 2cosec  (b) 2sec  2 8 8
sec A sec A (a) (b)
(a) (b) (c) sec  (d) cosec  15 15
1 – cos A 1  cos 2 A
67. If sin  cos  = 3 /4, the n
sec A 2 8 2
Enak

(c) (d) None of these sin4  + cos4  is equal to (c) (d)


1  cos A 17 17
(a) 7/8 (b) 5/8
x 2 – y2 2 2 7
(c) 3/8 (d) 1/8 75. If sec θ+tan θ  , then
R

61. If sin  = , the which one 12


x 2  y2 68. Consider the following
of the following is correct? (i) sin21° + cos21° = 1 sec 4 θ–tan4θ 
aryn

(ii) sec233° – cot257° =


2xy 7 1 5
(a) cos  = x 2 – y 2 cosec237°– tan253°
(a) (b) (c) (d) 1
Les B

Which of the above statement is/ 12 2 12


2xy are correct?

(b) cos  = x 2  y 2 (a) Only I (b) Only II 76. If 0  x  and secx= cosecy
(c) Both I and II 2
(d) Neither I nor II then the value of sin(x+y) is ;
wa. th

x –y
(c) cos  = x 2  y 2 69. If p = a sinx + b cosx and q = a
1 1
cosx – b sin x, then what is the (a) 0 (b) 1 (c)
(d)
value of p2 + q2? 2 3
xy x – y  (a) a + b (b) ab 77. If A, B and C be the angles of a
(d) cos  =
wwM

x 2  y2 (c) a2 + b2 (d) a2 – b2 triangle, the incorrect relation is;


70. The expression sin2x + cos2 x – 1
1  2sin  cos  A  B  C
= 0 is satisfied by how many (a) sin 
62. If a2 = , then what  2    cos 2
1 – 2sin  cos  values of x?  
(a) Only one value of x
a 1 A  B  C
is the value of ? (b) Two values of x (b) cos    sin

a –1 (c) Infinite values of x  2  2
(a) sec  (b) 1 (d) No value of x
A  B  C
(c) 0 (d) tan  71. Consider the following state- (c) tan 
   sec

ments  2  2
63. If the angle  is in the first quad-
I. There is only one value of x A  B 
rant and tan  = 3, then what is C
in the first quadrant that sat- (d) cot 
   tan

the value of (sin  + cos  )? isfies sinx + cosx = 2  2  2

Rakesh Yadav Readers Publication Pvt. Ltd. 186

For More Visit : www.LearnEngineering.in


For More Visit : www.LearnEngineering.in

78. If θ is a positive acute angle and 1 (c) a perfect square of an inte-


87. If sec θ=x + , 0  θ  90 ger
tan2θ.tan3θ  1 , then the value 4x (d) irrational
 5θ  then sec θ+tanθ is equal to 94. Evaluate : 3 cos 80° cosec 10° + 2
2cos 2
of  – 1
 is
 2  x 1 cos 59° cosec 31°
(a) (b) 2x (c) x (d) (a) 1 (b) 3 (c) 2 (d) 5
1 1 2 2x
(a) – (b) 1 (c) 0 (d) 88. The value of
2
95. sin θ – 3sinθ  2  0 wi ll be
2 2
2 true if
x  1 sin 25  sin2 65  is
79. If 2sin  2 (a) 0  θ<90 (b) 0  θ<90
    x + x 2 , then the
2
  3 2 (c) θ=0° (d) θ=90°
(a) (b) 1 (c) 0 (d)

ir
 1 2 3
value of x –   is 96. If tan  n tanβ and

v.iSn
 x
89.If sec θ  tan θ= 3 0  θ  90 , sin  m sin β , then cos 2 is
(a) –1 (b) 2 (c) 1 (d) 0
then the value of tan 3θ = ?
80. If cos θ+secθ=2 , the value of m2 m2
1 (a) (b)

dnag
cos θ+sec θ is
6 6
(a) undefined (b) n2 1 n2
(a) 4 (b) 8 (c) 1 (d) 2 3
81. The numerical value of m2 – 1 m2  1
1 (c) (d)
(c) (d) 3 n2 – 1 n2 1

eYari
5 2 2
+ + 2 is;
sec 2 θ 1  cot2 θ 3sin θ 7
90. If sin 60 – θ  cos  – 30 , 97. If cosec θ –cot θ = , then value
(a) 5 (b) 2 (c) 3 (d) 4 2
82. The numerical value of then the value of tan  – θ is
 1

 
1 
 cos θ cot θ 


 1

 –
1 


 cos θ cot θ 
snhe
(assume that θ and  are both
positive acute angles with
of cosec θ is;

(a)
47
28
(b)
51
28
(c)
53
28
(d)
49
28
is θ<60° and   30 ).
kgei
(a) 0 (b) – 1 (c) +1 (d) 2 98. If x sin45°= y cosec30°, then
1
sin θ  cos θ 5 (a) (b) 0 (c) 3 (d) 1 x4
83. If = , the value of 3 is equal to
ERna

sin θ – cos θ 4 y4
91. If a sin θ+bcosθ= c th en the
(a) 43 (b) 63 (c) 23 (d) 83
tan2 θ+1 value of a cosθ–b sinθ is ;
is
tan2 θ – 1 5 sin θ–3cosθ
(a)  –a 2  b 2  c 2 99. If 5 tanq= 4, then
aBryn

5 sin θ+2cosθ
25 41 41 40 (b)  a 2  b 2 – c 2
(a) (b) (c) (d) is equal to
16 9 40 41
84. If tan 7θ tan 2θ=1 , the n the (c)  a 2 – b 2 – c 2 2 1 1 1
(a) (b) (c) (d)
3 4 6 3
value of tan 3θ is (d)  a 2 – b 2  c 2
Les

100. 2 cosec223° cot267°– sin223°–


wa. th

1 1 sin267–cot267° is equal to
(a) 3 (b) – 92. If sin(A – B) = and cos(A + B)
3 2 (a) 1 (b) sec223°
2
1 (c) tan 23° (d) 0
1 = where A > B > 0 and A+ B
(c) (d) – 3 2 2
wwM

3
is an acute angle, then the value 101.The equation cos
2
θ=
x  y  is
85. The value of 4xy
B is
2 1  tan θ 1 – tan θ  only possible when
2 cos θ–1 
  

1 – tan θ 1  tan θ  (a)

(b)

(c)

(d)

(a) x = –y (b) x > y
6 12 4 2
is (c) x = y (d) x < y
93. The value of 152
(a) 4 (b) 1 (c) 3 (d) 2
( sin 30 2cos2 45  3sin 30 4cos2 45 + 102.If   β  90 , then the value of
86. If sec θ+tanθ=2 , then the value ......+ 17sin30°+18cos245°) is 2 2
of sec θ is (a) an integer but not perfect 1– sin 1– cos  ×
square 2 2
4 5 (b) a rational number but not 1  cot β1  tan β is
(a) (b) 5 (c) (d) 2
5 4 an integer (a) 1 (b) – 1 (c) 0 (d) 2

Rakesh Yadav Readers Publication Pvt. Ltd. 187

For More Visit : www.LearnEngineering.in


For More Visit : www.LearnEngineering.in

2sin 68 2 cot15 (a)  coscos β cos (a) –2 (b) 2


103. – –
cos 22 5 tan 75 (b)  sinsin β sin 7
3 tan 45.tan 20. tan 40.tan 50.tan 70
(c) (d) 2
is (c)  sincos β sec 2
5
equal to (d)  sinsinβ cos sin A
(a) –1 (b) 0 (c) 1 (d) 2 122.The value of
113.The numerical value of 1  cos A
104.If sin7x = cos11x, then the value
of tan9x+cot9x is 1 3 sin A
+ +2sin 2 θ + is (0°<A<90°)
(a) 1 (b) 2 (c) 3 (d) 4 1  cot2 θ 1  tan2 θ 1 – cos A
105. If ta n2= 1+2tan2β(here,,β will be (a) 2 cosec A (b) 2 sec A

r
(a) 2 (b) 5 (c) 6 (d) 3 (c) 2 sin A (d) 2 cos A
are positive acute angles), then

Sni
4 123.If tan θ – cot θ = 0, find the value
2 cos – cos β is equal to 114.The value of +
1  tan2 of sin θ +cos θ ,

gv.i
(a) 0 (b) 2 (c) 1 (d) – 1
1 (a) 0 (b) 1 (c) 2 (d) 2
+ 2 is
106. If tan θ +cot θ = 2, then the value 1  c ot 2 3 sin  124.If 3sin θ +5 cos θ = 5, th en

ridna
of tan100 θ +cot100 θ is (a) 4 (b) –1 (b) 2 (d) 3 5sin θ –3cos θ is equal to
(a) 2 (b) 0 (c) 1 (d) 3 115.The value of 3 (sinx + cosx) 4+6 (a) 3 (b)  5 (c) 1 (d) 0
(sinx+cosx)2+4 (sin6x+cos6x) is

eeYa
tan θ cot θ 125.If x sin 60°. tan30° = sec60°.
(a) 14 (b) 11 (c) 12 (d) 13
107.  is equal to cot45°, then the value of x is
1 – cot θ 1 – tan θ 116.The value of
(a) 2 (b) 2 3
(a) 1– tan θ – cot θ 1  sin θ cos θ 
sec θ 
  
 (c) 4 (d) 4 3
(b) 1+ tan θ – cot θ
geisnh
 cos θ 1  sin θ 
(c) 1– tan θ +cot θ – 2tan2 θ is 1
126.If θ = 60°, then 1  sin θ +
(d) 1+ tan θ +cot θ (a) 4 (b) 1 (c) 2 (d) 0 2
108.If sec θ +tan θ = 2+ 5 , then the 117.The elimination of θ from x
Enak
1
cos θ –y sin θ = 2 and x sin θ + y 1 – sin θ is equal to
value of sin θ +cos θ is ; 2
cos θ = 4 will give
3 7 1 θ θ
R

(a) (b) (a) x2+y2 = 20 (b) 3x2+y2 = 20 (a) cot (b) sec
5 5 (c) 5
(d)
5 2 2
(c) x2–y2 = 20 (d) 3x2–y2 = 10
aryn

118.The value of θ θ
109.The value of 1 cotθ – cosecθ (c) sin (d) cos
 cos2 A sin A  cos A sin2 A sin A – cos A   2 2
Les B

  
1 tanθ + secθ is equal to cos ec 2 A sin A – cos A sec 2 A sin A  cos A 
   
(a) 1 (b) 2 (d) 0 (d) – 1 2 tan2 30
× (sec2A– cosec2A) 127.If + sec 245°–sec 2
(a) 1 (b) 3 (c) 2 d) 4 1 – tan2 30
110.If x = a secθ cos , y =b
119.The value of 0 °= x sec 60°, then the value
wa. th

secθ sin ,z = c tan θ , then the of x is


1 1 (a) 2 (b) 1 (c) 0 (d) – 1
x 2 y2 z 2 – is
value of + – is ; cos ec θ –cot θ sin θ
a 2 b2 c 2 sin – cos
(a) 1 (b) 4 (c) 9 (d) 0 (a) 1 (b) cot θ 128.If tan θ = , then
sin  cos
wwM

sec θ+tanθ 5 (b) cosec θ (d) tan θ sin + cos is


111.If = , then sin θ
sec θ – tan θ 3 120. If cos θ +sin θ = 2 cos θ , then (a)  2 sin θ (b)  2 cos θ
is equal to ;
cos θ – sin θ is
1 1 2 3 1 1
(a) (b) (c) (d) (a) (b) – 2 cos θ (c)  sinθ (d)  cos θ
4 3 3 4 2 tan θ 2 2
112.If 1  sin 1  sinβ (c) – 2 sin θ (d) 2 sin θ 129.If 7 sin2 θ +3cos2 θ = 4,( 0°< θ <9
1  sin = 1 – sin 1 0°), then the value of tan θ is
121.If sin θ – cos θ = then value of
2 1 1
1 – sinβ 1 – sin , then (a) (b) (c) 1 (d) 3
third side is equal to sin θ + cos θ is; 3 2

Rakesh Yadav Readers Publication Pvt. Ltd. 188

For More Visit : www.LearnEngineering.in


For More Visit : www.LearnEngineering.in

p 137.In ABC ,  B  90 and AB : (a) sin4  –cos4 


130.If tan9° = , then the value of BC = 2 : 1, Then value of (SinA (b) 1– sin2  cos2 
q
+ CotC) (c) 1 + sin2  cos2 
sec2 81 (d) 1– 3sin2  cos2 
is 2 5
1  cot2 81 (a) 3  5 (b)
2 5 1 – cosB
2 2
148.If tan A = , then what is
q p q sinB
(a) (b) 1 (c) (d) 2
(c) 2  5 (d) 3 5
p q2 p 2tanA
x equal to?
131.If sec θ + tan θ = 5 then the value 138. If sin = x2–2x+2, then the 1 – tan2A
2
tan θ+1 value of x is tanB

ir
of is (a) (b) 2tanB
tan θ–1 (a) 0 (b) 1 2

v.iSn
(c) –1 (d) None of these (c) tanB (d) 4tanB
11 13 15 17 149. Assume the Earth to be a sphere
(a) (d) (c) (d) 139.The value of
7 7 7 7 of radius R. What is the radius
sin 43 cos19

dnag
 –8cos260° of the circle of latitude 40°S?
cos sin
132.If = a and = b, then cos 47 sin 71 (a) R cos40° (b) R sin80°
cos β sinβ
(a) 0 (b) 1 (c) 2 (d) – 1 (c) R sin 40° (d) R tan 40°
the value of sin2 β in terms of 140.The value of
1

eYari
150. If cos  > in the first quadrant,
a and b is  2 1 2 1 2
sin 7  sin 82 
  is then which one of the following
 2 2
a2  1 a2 – b2 is correct?
(a) (b) (a) 1 (b) 2 (c) 0 (d) 4
a2 – b2 a2  b2

(c)
a –12

a2 – b2
(d)
a –12

a 2  b2
(a) 0
snhe
141. If 3sinx + 5cosx = 5, then what
is the value of (3cosx – 5sinx)?
(b) 2 (c) 3 (d) 5
(a)  <

3

(b)  >

3

kgei
142. If  and  are complementary (c)  < (d)  >
133.The value of 6 6
angles, then what is
cos2 60 4sec2 30 – tan2 45 
151. If sin  + cos  =1, then what is
is  sin  cos  

the value of sin  .cos  ?
ERna

2
sin2 30 cos2 30 cosec .cosec    sin  cos   (a) 2 (b) 0
64 55 67 67
(a) (b) (c) (d) equal to? 1
3 12 12 10 (a) 0 (b) 1 (c) 1 (d)
2
aBryn

(c) 2 (d) None of these


2 5
134.If cosx  x – x  , the value 143. If A, B, C and D are the succes- 1  sin 
4 sive angles of a cyclic quadrilat- 152. What is equal to?
1 – sin 
of x will be eral, then what is
(a) 0 (b) 1 (d) –1 (a) sec  – tan 
cosA + cosB + cosC + cosD are
Les

(d) None of the above equal to? (b) sec  + tan 


wa. th

135.The numerical value of (a) 4 (b) 2 (c) 1 (d) 0 (c) cosec  + cot 
1 144. How many degrees are there in (d) cosec  – cot 
1 – sec 2 27 + an angle which equals two-third 153. Two sides of an acute angle tri-
cot2 63
of its complement? angle are 6 cm and 2 cm, respec-
wwM

1 (a) 36° (b) 45° (c) 48° (d) 60° tively. Which one of the follow-
2
– cos ec 2 27 is
sin 63 sin  cos  ing represents the correct range
(a) 1 (b) 2 (c) – 1 (d) 0 145. If + = 2 with 0 <  of the third side in cm?
cos  sin 

136.If x 
cos θ
, then
cos θ
is
< 90°, then what is  equal to? (a) (4, 8) (b) 4,2 10 
1 – sin θ 1  sin θ (a) 30° (b) 45° (c) 60° (d) 75°
equal to 146. If sin3  = cos(  –2°), where 3  (c) 4 2,8  (d) 4 2,2 10 
and (  –2°) are acute angles, 154. If cos1° = p and cos89° = q, then
1
(a) x –1 (b) what is the value of  ? which one of the following is cor-
x (a) 22° (b) 23° (c) 24° (d) 25° rect?
1 1 sin6  – cos 6  (a) p is close to 0 and q is close
(b) (d) 147. What is equal to? to 1
x 1 1– x sin2  – cos 2 

Rakesh Yadav Readers Publication Pvt. Ltd. 189

For More Visit : www.LearnEngineering.in


For More Visit : www.LearnEngineering.in

(b) p < q 162. If  is an acute angle and sin 


1
(c) p = q (a) 0 (b) 1 (c) 2 (d)
x –1 2
(d) p is close to 1 and q is close = , then what is tan 
to 0 2x
1
155. If 7 cos2  + 3 sin2  = 4 and 0 < equal to? 171. If cos2 θ – sin2 θ = , where 0 
3
 x –1 x 1
(a) (b) 
< , then what is the value of x 1 x –1
2 θ < , then the value of cos4 θ
2
tan  ? (c) x2 –1 (d) x2 1
– sin4 θ is
7 cos  cos 
(a) 7 (b) 3 (c) 3 (d) 3 163. – = 2 is satis-
1 – sin  1  sin 

r
fied by which one of the follow- 1 2 1 2
156. What is the value of [(1 – sin2  ) (a) (b) (c) (d)

Sni
ing values of  ? 3 3 9 9
sec2  + tan2  ] (cos2  +1) when
(a)  /2 (b)  /3

gv.i
0º<  < 90°? (c)  /4 (d)  /6 1 
172.If tan θ = and 0° < θ < ,
(a) 2 (b) >2 (c) >2 (d) <2 164. If 0º < x < 45° and 45° < y < 90°, 11 2
then which one of the following then the value of

ridna
157. If 0 <  < and p = sec2  , then is correct?
2 (a) sinx = sin y cos ec 2θ–sec2θ
which one of the following is cor- (b) sinx < sin y is
cos ec 2θ+sec2θ
rect? (c) sinx > sin y

eeYa
(a) p < 1 (b) p = 1 (d) sinx  sin y 3 4 5 6
(c) p > 1 (d) p > 1 (a) (b) (c) (d)
165. What is the value of sin 360° 4 5 6 7
158. In a  ABC,  ABC = 90°,  ACB cot30° – 2sec²45° + 3 cos 60° tan2
45° – tan2 60°? 3
= 30°, AB = 5 cm. What is the
geisnh 173.If sin θ = , then the value of
(a) 35/8 (b) –35/8 5
length of AC?
(c) –11/8 (d) 11/8
(a) 10 cm (b) 5 c m tanθ+cosθ
p is equal to
(c) 5 2 cm (d) 5 3 cm 166. If tan  = q , then what is cotθ+cosecθ
Enak

 p sec  – q cosec  29 31 34 37
159. If 0 <  < and cos  + 3 sin  (a) (b) (c) (d)
2 p sec   q cosec  equal to? 60 60 60 60
R

174.If (sin  +cosec )² + (cos  +


= 2, then what is the value of  ? p –q q2 – p2
(a) (b) sec )² =k + tan² + cot²  , then
    p q q 2  p2 the value of k is
aryn

(a) (b) (c) (d) (a) 1 (b) 7 (c) 3 (d) 5


3 4 6 2 p2 – q 2
Les B

160. If ABC is a right angled triangle (c) (d) 1 x


q 2  p2
at C and having u units, v units 175.If sin 21° = y , then sec21°–
167. The value of cosec2  – 2 + sin2 
and w units as the length of its
is always sin69° is equal to
sides opposite to be vertices A,B
(a) less then zero x2 y2
wa. th

and C respectively, then what is


(b) non-negative (a) (b)
tan A + tan B equal to? y y2 – x 2 x y2 – x2
(c) zero
u2 (d) 1 x2 y2
(a) (b) 1 168.Fi nd the valu e of 1 – 2 (c) (d)
vw y x 2 – y2 x x 2 – y2
wwM

sin2 θ + sin4 θ ,
w2 176.If sec  tan  2 , then the
(c) u + v (d) (a) sin4 θ (b) cos4 θ value of sin is (assume that
uv
(c) cos ec 4 θ (d) s ec 4θ 0º <  < 90°)
161. ABC is a right triangle with right
(a) 0.4 (b) 0.5
angle at A. If the value of tan B = 169.sin θ = 0.7, then cos θ ,
(c) 0.6 (d) 0.8
1 0º  θ <90° is 177.If 7sin = 24 cos  ; 0 <  <
, then for any real k the (a) 0.3 (b)
3 0.49 
length of the hypotenuse is of the (c) (d) 0.9 , then the value of 14tan –
0.51 2
form. 170.The value of sin265° +sin2 25° + 75cos –7sec is equal to
(a) 3k (b) 2k (c) 5k (d) 9k cos2 35° + cos2 55° is (a) 3 (b) 4 (c) 1 (d) 2

Rakesh Yadav Readers Publication Pvt. Ltd. 190

For More Visit : www.LearnEngineering.in


For More Visit : www.LearnEngineering.in

178.ABCD is a rectangle of which 187.If  and β are positive acute A  B , then A is


AC is a diagonal. The value of (a) 90° (b) 30° (c) 45° (d) 60°
2
angl es, si n(4 – β )= 1 and
tan  CAD  1 sin2  BAC is
 sinθ–2sin3θ
1 194.The value of is
1 cos(2 + β ) = ,then the value 2cos3 θ – cos θ
(a) 2 (b) (c) 1 (d) 0 2 equal to
4
of sin   2β is (a) sin θ (b) cos θ
sec θ–1 (c) tan θ (d) cotθ
179.For any real value of 1
sec θ  1 3
(a) 0 (b) 1 (c) (d) 2xy
=? 2 2 195.If cot  = x 2 – y 2 , then what is
(a) cotθ–cosecθ 188.If θ is a positive acute angle and

ir
cos  equal to?

v.iSn
(b) secθ–tanθ cosec θ = 3 , then the value of
x 2 – y2 x 2  y2
(c) cosecθ–cotθ cot θ–cosecθ is (a) (b)
x 2  y2 x 2 – y2
(d) tanθ–secθ
2 3 3 2xy
3 2– 3   2xy

dnag
  (a) (b) (c) x 2  y2 (d)
x2  y2
180.In a ABC , B  C = 3 3
3 4 196. For what value of  is (sin  +
and D divides BC internally in 3 2 3 cosec  ) = 2.5, where 0º <  < 90°?
(c)  2– 3  (d)

eYari
sinBAD 3 (a) 30° (b) 45° (c) 60° (d) 90°
the ratio 1 : 3 then, 197. If x cos60° + y cos0° = 3 and 4x
sinCAD 189.If (r cos θ – 3 )2+(r sin θ –1)2= 0,
sin30° – y cot 45° = 2, then what
is equal to then the value of is the value of x?

(a)
2
1
3
(b)
6
1
(c)
181.If sin 3A = cos(A – 26°), where
1
(d) 6
r tanθ+secθ
r sec θ  tan θ

4
snhe
is equal to

3
(a) –1 (b) 0 (c) 1 (d) 2
198. What is log (tan 1°) + log (tan 2°)
+ log (tan 3°) + ....+ log (tan 89°)
equal to?
3A is an acute angle then the 3 5
kgei
(a) (b) (c) (d) (a) 0 (b) 1 (c) 2 (d) –1
value of A is 5 5 4 4 199.If sin x cos x = 1/2, then what is
(a) 29° (b) 26° (c) 23° (d) 28° the value of sinx – cosx?
cos sin
(a) 2 (b) 1 (c) 0 (d) –1
ERna

182.The v a l u e o f sec2 θ – 190.If cos β = a, sinβ = b, then


200. If tan2y cosec2 x – 1 = tan2 y, then
which one of the following is cor-
sin2 θ–2sin4θ sin2 β is equal to
is rect?
2cos4 θ – cos2 θ (a) x – y = 0 (b) x = 2y
a2 – 1 a2  1
aBryn

(a) 1 (b) 2 (c) – 1 (d) 0 (a) (b) (c) y = 2x (d) x – y = 1


a 2  b2 a2 – b2
183.If x = a (sin θ + cos θ ) ,y = b cos x cos x
a2 – 1 a2 1 201. If 1  cos ec x + cos ec x – 1 = 2,
sinθ–cosθ then the value of (c) 2
a – b2
(d) 2
a  b2 then which one of the following
Les

x 2 y2 is one of the values of x?


 is 191.If 3 tan θ = 3 sin θ , then the value
(a)  /2 (b)  /3
wa. th

a 2 b2
of (sin2 θ –cos2 θ )is
(a) 0 (b) 1 (c) 2 (d) –2
(a) 1 (b) 3 (c)  / 4 (d)  /6
184.If sin 5θ = cos20° (0°< θ < 90°) 202. If x + y = 90° and sin x : sin y
1
then the value of θ is (c) (d) None of these = 3 : 1, then what is x : y equal
wwM

3 to?
(a) 4° (b) 22° (c) 10° (d) 14°
192.ABC is a right angle triangle and (a) 1 : 1 (b) 1 : 2
185.If 0°< θ < 90°and 2 sec θ = 3 right angle at B and A = 60° (c) 2 : 1 (d) 3 : 2
cosec2 θ , then θ is and AB = 20cm, then the ratio cos x sin x
    of sides BC and CA is 203. If cos y = n and sin y = m, then
(a) (b) (c) (d)
6 4 3 5 (a) 3 :1 (b) 1: 3 (m2 – n2) sin2 y is equal to
1  sin θ 1 – sin θ (c) (d) (a) 1 – n2 (b) 1 + n2
186. + is equal 3: 2 3 :2 (c) m2 (d) n2
1 – sin θ 1  sin θ
to 193.If tan(A + B) = 3 and tan 204. If p = tan2x + cot2 x, then which
one of the following is correct?
(a) 2 cos θ (b) 2 sin θ 1 (a) p < 2 (b) p > 2
(c) 2 cot θ (d) 2 sec θ (A – B) = , ( A  B )< 90°, (c) p < 2 (d) p > 2
3

Rakesh Yadav Readers Publication Pvt. Ltd. 191

For More Visit : www.LearnEngineering.in


For More Visit : www.LearnEngineering.in

205. What is the value of 223. If cosec  = p/q and  is acute,


(a) 3 (b) 2 (c) 0 (d) 1
5sin75 sin77  2cos13 cos15 then what is the v alue of
214.If sin (x + 54°) = cos x, where 0 <
cos15 sin77 x, x + 54° < 90°, then what is the
value of x?
 
p 2 – q 2 tan  ?
7sin81
– (a) 54° (b) 36° (c) 27° (d) 18° (a) p (b) q
cos 9
215.In the given figure, BC = 15 cm (c) pq (d) p2  q 2
(a) –1 (b) 0 (c) 1 (d) 2
and sin B = 4/5. What is the
206. If sinx + sin y = a and cosx + cos 224. If 2x2 cos60° – 4cot2 45° – 2tan
y = b, what is sin x.sin y + cosx. value of AB?
60° = 0, then what is the value of
cos y equal to? A
x?
(a) a + b – ab (b) a + b + ab (a) 2 (b) 3

r
 a 2  b2 – 2 
(c) a2 + b2 – 2 (d)  (c) 3 –1 (d) 3  1
2 

Sni
225. Which one of the following state-
207. If  is the angle of first quadrant ments is true in respect of the
B C

gv.i
such that cosec4  = 17 +
(a) 25 cm (b) 20 cm expression sin31° + sin32°?
cot4  , then what is the value of
sin  ? (c) 5 cm (d) 4 cm (a) Its value is 0
(a) 1/3 (b) 1/4 216. The smallest side of a right (b) Its value is 1

ridna
(c) 1/9 (d) 1/16 angled triangle has length 2 cm. (c) Its value is less than 1
The tangent of one acute angle (d) Its value is greater than 1
1
208. If x +   = 2 cos  , then what 3 226. Which one of the following is cor-
x

eeYa
is . What is the hypotenuse of rect?
4
1 the triangle? (a) sin 35° > cos 55°
is the value x2 +  2  ?
x (a) 5 cm (b) 2.5 cm 1
(a) 4cos2  (b) cos61°>
2
geisnh 10
(b) 4cos2  –1 (c) 1.25 cm (d) cm
3 1
(c) 2cos2  – 2 sin2 
217.If sin x – cos x = 0, then what is (c) sin 32° >
(d) cos2  – sin2  2
209. What is the value the value of sin4x + cos4x?
(d) tan 44° > 1
Enak

1 3 1 1 227. If sin  + cosec  = 2, then what


cot² θ – ? (a) 1 (b) (c) (d)
sin ²θ 4 2 4 is the value of sin4  + cos4  ?
(a) 1/2 (b) –1 tan x (a) 2 (b) 22 (c) 23 (d) 1
R

(c) –1/2 (d) 3/2 218. What is the expression –


1  sec x 228. If   R be such that sec  >0 and
210. If sin x = cos y and angle x and
tan x 2sec2  + sec  – 6 = 0
angle y are acute then what is
aryn

equal to?
the relation between x and y? 1 – sec x Then, what is the value of cosec  ?
(a) cosecx (b) 2 cosecx
Les B

(a) x – y =  /2 (c) 2 sinx (d) 2 cosx (a) 5 (b) 3 /2


(b) x + y = 3 /2 219. What is the expression (sin4x –
cos4 x + 1) cosec2 x equal to? (c) 3/ 5 (d) 2/ 3
(c) x + y =  /2 (a) 1 (b) 2 (c) 0 (d) –1 229. Under which one of the follow-
wa. th

(d) x + y =  / 4 220. If x + y = 90°, then what is ing conditions is the trigono-


metrical identity.
cos x.cosecy – cos x.sin y
m2 – n 2 sinx/(1 + cosx) = (1 – cosx)/sinx
211. If sin  = , then what is equal to? true?
m2  n 2
(a) cosx (b) sinx
wwM

(a) x is not a multiple of 360°


the value of tan  ?
(c) cos x (d) sin x (b) x is not an odd multiple of 180°
m2  n 2 2mn 221. If p = sin10 x, then which one of (c) x is not a multiple of 180°
(a) (b) the following correct for any
m2 – n 2 m2  n 2 (d) None of the above
value x?
230. If 3sin  + 4cos  = 5, then what
m2 – n 2 m2  n 2 (a) p > 1 (b) 0 < p < 1
(c) (d) (c) 1 < p < 2 (d) None of these is 3 cos  – 4sin  equal to?
2mn 2mn
222. What is the value of the expres- (a) 0 (b) 3 (c) 4 (d) 5
212. If sin(x – y) = 1/2 and cos (x + y) =
   231. If sec  = 13/5, then what is value
1/2, then what is the valueof x? sion cos2 + 4 cos2 – sec + 5
(a) 15° (b) 30° (c) 45° (d) 60° 8 4 3
2sin  – 3cos 
  of ?
213. If 1+ tan  = 2 , then what is the tan2 + sin2 ? 4sin  – 9cos 
3 8
value of cot  – 1? (a) 8 (b) 10 (c) 16 (d) 18 (a) 1 (b) 2 (c) 3 (d) 4

Rakesh Yadav Readers Publication Pvt. Ltd. 192

For More Visit : www.LearnEngineering.in


For More Visit : www.LearnEngineering.in

tan cot 249. If asec + btan +c = 0 and


7 7 3
232.If r sinθ = and r cos θ = 241. If = 2, (0°    psec + qtan +r = 0, then (br –
2 2 tan– cot
qc)² – (pc – ar)² = ?
then value of r is 90°), then the value of sin is (a) (aq – bp)² (b) (ap – bq)²
(a) 4 (b) 3 (c) 5 (d) 7 (c) (aq + bp)² (d) (aq – bp)³
 1 2 3 1 250. If P = acos³x + 3a cosx. sin²x and
233.If  = and sin θ= , then (a) (b) (c) (d) 1 Q = asin³x + 3acos²x.sinx, then
2 2 3 2 2
(P +Q)2/3 + (P – Q)2/3 = ?
242. If sin x + cos x = c, then sin6 x + (a) 2a2/3 (b) a1/3
the value of sin is
cos6 x is equal to. (c) 2a1/3 (d) a1/3
1 1 3 1  6c 2 – 3c 4 251. If 8cos² + 8sec² = 65 and
(a) 1 (b) (c) (d) (a)

ir
2 2 2 16 

v.iSn
234.If sec q + tan q = P, (P  0) the 0°<< , then 4cos2 is equal to
1  6c 2 – 3c 4 2
sec q is equal to: (b)
4 23 31
1 1 (a) – (b) –
8 8
P 
(a)   , P 0

dnag
3 P  1  6c2  3c 4
(c) 31 33
16 (c) – (d) –
1  1 32 32
(b) P  
 ,P 0
2 P   1  6c2  3c 4 252. If cos( –A) = a, cos( – B) = b,
(d)

eYari
4 then sin²(A–B)+ 2ab cos(A – B) is
 1
(c) 2 
P  
 ,P  0 1 – sin A cos A sin²A – cos²A
equal to
 P 243. cos A(sec A – cos ecA) . sin³A  cos³A = ? (a) a² – b² (b) a² + b²
 1 (a) sinA (b) cosA (c) b² – a² (d) 2ab
P – 
(d) 
 P
 ,P  0
235.The value of sin²22° + sin²68° +
cot² 30° is:
(c) tanA snhe
(d) cosecA
244. If cot +cos = m and cot –
cos = n, then find the value of
253. sin 4

8
+ s in 4
3
8
+ sin 4
5
8
+

7
kgei
(a) 5/4 (b) 3/4 m² – n². sin4 =?
8
(c) 3 (d) 4 (a) (b) 2 mn
mn 3 1
(a) (b) 1 (c) (d) 0
ERna

sec tan 51 2 2
236.If =2 then the (c) 3 mn (d) 4 mn
sec – tan 79 254. If sin + cos = a and sec +
245. If tan + sin = m and
value of sinq is cosec = b, then the value of b
tan – sin = n, then find the
91 39 65 35 (a² – 1) is equal to
aBryn

(a) (b) (c) (d) value of mn . (a) 2a (b) 3a (c) 0 (d) 2ab
144 72 144 72
237.If 1 + cos²q = 3 sinq cosq, then 1
255. cos15° cos7
1 1
°. cos82 ° = ?
(a) (m²–n²) (b) 2(m²–n²)
the integral value of cot q is 2 2 2
Les

  1 1 1 1 1 1
0  
  (c) (m²+n²) (d) (m²–n²) (a)
2
(b)
8
(c)
4
(d)
16
2
wa. th

 4 4
(a) 2 (b) 1 (c) 3 (d) 0 246. If cot +tan = x and sec– cos
238.The value of following is cos2 4 = y, then (x²y)2/3 –(xy²)2/3 =? cos(–)
° + cos55° + cos 125° + cos 204° (a) 4 (b) 3 (c) 2 (d) 1 256. 3tantan=1, then cos() =?
+ cos300°
wwM

(a) – 1/2 (b) 1/2 sin8– cos8 1 1


247. =? (a) (b) 2 (c) (d) 3
(c) 2 (d) 1 cos 2(1  cos ²2) 2 3
239. If sinA + cosecA = 3, then find the
1
sin4 A  1 (a) 1 (b) – (c) – 1 (d) 2 257. tan20° + tan40° + 3 tan20°.
value of . 2
sin2 A tan40° = ?
248. If (sec  +tan  ) (sec +tan )
(a) 1 (b) 0 (c) 7 (d) 0
(sec + tan) = (sec – tan ) 3 3
240. cos7° cos23° cos45° cosec83° (a) (b) (c) 3 (d) 1
cosec67° = ? 4 2
(sec – tan) (sec – tan), then
258. If x cosq – sinq = 1, then x² +
1 1 each of the side is equal to
(1+ x²) sinq equals
(a) 0 (b) 1 (c) (d) (a)  1 (b) – 1 (c) +1 (d) 4
2 2 (a) 1 (b) – 1 (c) 0 (d) 2

Rakesh Yadav Readers Publication Pvt. Ltd. 193

For More Visit : www.LearnEngineering.in


For More Visit : www.LearnEngineering.in

259.The simplified form of the given 276.The numerical value of


1 1
expression sinA cosA(tanA – (a) 3 (b) (c) 3 (d) 9 5
cotA) is (where 0° <A < 90°): 3 3 + 4 cos²  + :
cosec² 1+tan²
(a) 1 (b) 1 – cos²A 1
268. If secθ – tanθ = , the value (a) 7 (b) 9 (c) 4 (d) 5
(c) 1 – 2 sin²A (d) 2 sin² A – 1
3
p
Cos Cos 277.If cos  = , then the
260. Sin = n and Cos = m, then of sec  .tan  is: p²+q²
4 2 2 1 value of tan  is:
the value of cos²b is: (a) (b) (c) (d)
3 3 3 3
m²–1 m²–3 q p
(a) (b) 269.The value of (cosec a – sin a) (a) p (b) p²+q²
n²–1 n²–4

r
(sec a – cos a) (tan a + cot a)

Sni
m²+3 n²
(c) (d) (a) 4 (b) 6 (c) 2 (d) 1 q q
n²+3 m²+n² 270.If tan A = n tan B and sin A = m (c) (d)

gv.i
p²+q² p²–q²
261.Provided, sin(A – B) = sinA cosB sin B, then the value of cos² A
– cosA sin B, then sin15° will is 278.If sec  +tan  = 3,  being
be

ridna
m² + 1 m² + 1 acute, the value of 5 sin  is:
(a) (b)
3 –1 3 n² + 1 n² – 1
(a) (b) (a) 3 (b) 5
2 2 2 2 m² – 1 m² – 1 5 3
(c) (d)

eeYa
3 –1 3 1 n² – 1 n² + 1
5
(c) (d) tan²45°.sec60° (c) 4 (d)
2 2 2 271.If x cos²30°.sin 60°= 2
cosec 60°
262.If a is an acute angle and 2sina then the value of x x–xtan²30º
+ 15 cos²a = 7, then the value
geisnh 279.If =S in²30º + 4
of cota is 1 2 1+tan²30º
(a) 4/3 (b) 5/4 (a) (b) 2
3 3 Cot²45º–Sec²60º Then value of
(c) 4/5 (d) 3/4 x is:
263.If sin4q + cos4q = 2sin²qcos²q . q 1 1
Enak

(c) (d) 1 1 1 1
is an acute angle, then value of 2 2
(a) (b) (c) (d)
tanq is 4 3 2 5
3
(a) 2 (b) 1 (c) 3/5 (d) 0 272.If sin(θ+30°)= , then find
R

12 280.If cos θ +sin θ =m and Sec θ +


264.If A is an acute angle and cotA
cos²θ cosec θ =n then the value of
+cosec A = 3, then the value of
aryn

n(m²–1) is equal to:


sin A is
1 3 3 1 (a) mn (b) 4mn
(a) 1 (b) 4/5 (a) (b) (c) (d)
Les B

(c) 3/5 (d) 0 4 4 2 2 (c) 2n (d) 2m


265.The value of. co t41° 273.If 0  θ  90° and 281.If  + β = 90° then the expres-
.cot42°.cot43°.cot44°.cot45°.
c o t 4 6° . cot47°.cot48°.cot49°. 4cos² θ – 4 3cosθ + 3 = 0, tan
wa. th

then the value of θ is sion + sin²α + sin² β is


3 1 tanβ
(a) (b) 1 (c) 0 (d) (a) 30° (b) 90° (c) 45° (d) 60°
2 2 274.The value of sec4 A (1 – sin4A)– equal to :
266.If 5 cosθ +12 sinθ = 13, θ < θ 2tan2 A is (a) tan²  (b) tan² β
wwM

< 90°, then the value of sin is (a) 12 (b) 0 (c) 2 (d) 1 (c) sin² β (d) sec²
6 12 275.If A, B, C are the angles of a 282.The value of x in the equation
(a) (b) –  ABC then following is equal to:
13 13   
tan ² – cos²  x sin cos
 B+C  4 3 4
5 12 Sin 
(c) (d)   
13 13  2  tan is :
4 3
267.If tanθ – cotθ = 0 and θ is posi- B A
(a) sec (b) sec 3 3 2
tive acute angle, then the value 2 2 (a) (b)
4 3
tan (θ+15°) A A
of tan (θ–15°) is 1 3
(c) cosec (d) cos (c) (d)
2 2 3 2

Rakesh Yadav Readers Publication Pvt. Ltd. 194

For More Visit : www.LearnEngineering.in


For More Visit : www.LearnEngineering.in

3  2 3 1 
283.If sin2 then the value of 291. cos + cos + cos +.... 301. 3 cos 23 – sin 23 = ?
2 7 7 7 4 
sin 3 is equal to : (Take 0°   6 (a) cos43° (b) cos70°
+..cos =?
 ) 7 1
(c) cos53° (d)
cos53°
(a) 0 (b) 1 (c) –1 (d) 2 2
1 3 292. tan9° – tan27° – tan63° + tan81° = ? 302. 2cosx – cos3x – cos5x = ?
(a) (b) (c) 0 (d) 1
2 2 (a) 1 (b) 2 (c) 3 (d) 4 (a) 16cos³x sin²x
293. If 2 cosx + sinx = 1, then find 7 (b) sin³x cos²x
sin cos
284.If = 3 then the cosx + 6 sinx. (c) 4cos³x sin²x
sin – cos (a) 6 (b) 2 (c) 7 (d) 1 (d) 4sin³x cos²x

ir
4
value of sin is : 294. cos²48° – sin²12° = ? 303. 3 cosec20° – sec20° = ?

v.iSn
16 2 1 3
(a) (b) (c) (d) 5 –1 5 1 2sin20
25 5 5 5 (a) (b) (a) 2 (b)
4 8 sin 40
285.If cos 20° = m and cos 70° = n,
then the value of m² + n² is 3 1 4sin20

dnag
3 –1
(c) (d) (c) 4 (d)
1 3 1 4 2 2 sin40
(a) (b) 1 (c) (d)
2 2 2 cos12 – sin12 sin147
  3 304. + =?
295. sin   sin   =? cos12  sin12 cos147

eYari
3 –1 10 10 
286.If sinA – cosA = th en 1
2 1 1 1 (a) 1 (b) – 1 (c) 0 (d)
(a) (b) – (c) (d) 1 2
the value of sinA . cosA is 2 2 4

(a)
1
3
a
(b)
3
2
(c)
1
4
(d)
3
4
  snhe  
296. tan   – tan  – = ?
4 4
305. cos24° + cos55° + cos125° +
cos204° + cos300° = ?
–1
(a) 2 tan2 (b) 2 cot2 (a) 2 (b)
kgei
287.If sin   , then the value of sec– 2
b (c) tan2 (d) cot2
cos is (where 0º <  < 9 0º ) 1
(c) 0 (d)
1 2
ERna

a b2 297. If sin( – )= and cos (+) =


(a) 2 2 (b) 2 306. If m tan ( – 30°) = n tan ( +
b b a a b2  a 2
1 mn
a2 , where  ,are positive acute 120°), then =?
b2  a 2 2 m–n
(c) (d)
aBryn

b b2  a 2 b2  a 2 angles, then  &are_ (a) 2cos2 (b) cos2


288.The expression (c) 2sin2 (d) sin2
(a) 45° and 15° (b) 60° and 15°
1  sin  1  sin  (c) 15° and 45° (d) 45° and 60° x sin
 is equal to: 307. If tan = 1 – x cos & tan =
1  sin  1  sin  298. If  + –  = , then sin² +
Les

(a) 2 sin (b) 2 tan y sin x


wa. th

sin²– sin² = ?
(c) 2 sec (d) 2 cosec  1 – y cos
, then y = ?
(a) 2sin sincos
sec2 70ºcot2 20º 2 sin sin
289.If 2 2
 , (b) 2cos coscos (a) sin (b)
2(cosec 59ºtan 31º ) m sin
wwM

(c) 2sin sinsin sin sin


then m. is equal to: (c) (c) 1 – cos
(a) 2 (b) 3 (c) 4 (d) 1 (d) 2cos sinsin 1 – cos
4 299. 1+ cos2x + cos4x + cos6x = ? b ab
290. If cos (  +  )= and s in 308. If tan x = , then +
5 (a) 2cosx cos2x cos3x a a –b
5 (b) 4sinx cos2x cos3x a –b
( – ) =  ,lie between 0 (c) 4cosx cos2x cos3x =?
13 a b
(d) cosx cos2x cos3x 2sin x
 2cos x
and , then tan 2 = ? 300. sin12° sin48° sin54° = ? (a) (b)
4 sin 2x cos 4x
1 1 1 1 2cos x 2cos x
56 36 33 49 (a) (b) (c) (d) (c) (d)
(a) (b) (c) (d) 16 8 4 32 sin 2x cos 2x
33 33 56 36

Rakesh Yadav Readers Publication Pvt. Ltd. 195

For More Visit : www.LearnEngineering.in


For More Visit : www.LearnEngineering.in

309. tan +2tan2  +4 tan 4  + 8 sin – cos 1


cot8 = ? 318. If tan = , then (c) cos A cos B =
sin cos 5
(a) tan (b) tan2 (d) Both (b) and (c)
(c) cot (d) cot2 sin+ cos and sin– cos are
equal to cos 9  sin9
 n –1  326. =?
(a) 2 cos , 2 sin cos 9 – sin9
310. If sinA = n sin B, then  
n 1 (a) tan54° (b) tan36°
(b) 2 sin , 2 cos (c) tan18° (d) cot18°
 A B
tan   =? (c) 2 sin , 2 sin 1 1
2  327. If tan  = and tan  = , the
(d) 2 cos , 2 cos 7 3
 A – B  A – B cos2 = ?
(a) sin   (b) tan   319. If sin + sin = a and cos +

r
2  2  (a) sin2 (b) sin4
–

Sni
 A – B  A B cos = b, then tan   =? (c) sin3 (d) cos3
2 
(c) cot   (d) tan  
2 2 

gv.i
1 1
a²  b² 4 – a² – b² 328. If tan A = , tan B = , then
311. 2sinA cos³A – 2sin³A cosA = ? (a) (b) 2 3
4 – a² – b² a²  b²
1 cos2A = ?
(a) sin4A (b) sin4A a²  b² 4  a²  b² (a) sinB (b) sin2B

ridna
2 (c) (d)
4  a²  b² a²  b² (c) sin3B (d) cos3B
1 1 320. cos² +cos² ( +120°)+cos² ( – 329. 2sin²+ 4cos( +)sin sin +
(c) sin4A (d) sin4A 120°) = ?
4 8

eeYa
cos²( + )= ?
312. tanA + tan (180° + A)+ cot (90° + 3 1
A)+ cot (360° – A) = ? (a) (b) 1 (c) (d) 0 (a) sin2 (b) cos2
2 2
(a) 0 (b) 2 tan A (c) cos2 (d) sin2
1 – cos B
(c) 2 cot A (d) tan A – cot A 321. If tan A = , express tan
sinB 330. sin12° sin24° sin 48° sin 84° = ?
geisnh
2sin 2A in terms of tan B (a) cos20° cos40° cos60° cos80°
313. If = y, then
1  cos sin (a) tan2A = tanB (b) sin20° sin40° sin60° sin80°
1 – cos sin (b) tan2A = tan² B 3 1
=? (c) tan2A = tan² A + tan²B (c) (d)
1  sin 16 16
Enak

(d) tan2A = tan²A – tan²B


(a) y (b) 1/y 331. tan 5x tan 3x tan 2x = ?
322. If tan (A + B) = p and tan (A – B) =
(a) tan 5x – tan 3x – tan2x
1 q, then the value of tan 2A is
(c) 1+y (d) 1  y (b) 0
R

pq p–q
(a) p – q (b) 1  pq sin6x – sin3x – sin2x
sin70  cos 40 (c)
cos 5 x – cos 3x – cos 2x
314. =?
aryn

cos 70  sin 40 pq 1  pq (d) tan9x


(c) 1 – pq (d)
p–q 332. If cos + cos = 0 = sin + sin,
1
Les B

(a) 1 (b) sec 8A – 1


3 323. =? then cos2 + cos2 = ?
sec 4A –1
(c) 3 (d) – 3 (a) – 2 sin ( + )
tan2A tan 8A
(a) (b) (b) 2cos ( + )
sin4 A cos 4A tan 8A tan2A
wa. th

1
315. If + = , then (c) 2 sin ( + )
a b ab cot 8A cot 2A
(c) (d)
sin8 A cos 8 A cot 2A cot 8A (d) – 2 cos ( + )
+ =? 324. In a  ABC,  C = 90°, then the 333. If cosA = a cos B and sin A = b
a3 b3
wwM

equation whose roots are tan A sin B, then (b² – a²) sin²B = ?
1 a²b² & tan B is (a) 1 + a² (b) 2 + a²
(a) (a  b)3 (b) (a  b)2 (a) abx² + c² + ab = 0 (c) 1 – a² (d) 2 – a²
(b) abx² + c²x – ab = 0 334. If A + B + C = , then cos2A +
a 3b3 ab (c) abx² + c²x – ab = 0 cos2B + cos2C = ?
(c) (a  b)2 (d) (d) abx² – c²x + ab = 0
ab (a) 1 + 4 cosA cosB cos C
316. If 2ycos = x sin and 2x sec – 3 (b) – 1 + 4 sinA sinB cos C
325. If cos(A – B) = and tan A tan B (c) – 1 – 4 cosA cosB cosC
ycosec = 3, then x² + 4y² = ? 5
= 2, then (d) 1 + 4 sin A sin B sin C
(a) 4 (b) – 4 (c)  4 (d) 0 2 335. If A, B C are angles of a triangle,
317. If tan – cot = a and cos + (a) cosA cosB = then sin²A + sin² B + sin² C – 2
5
sin = b, then (b² – 1)² (a² + 4) =? cosA cosB cosC = ?
2
(b) sinA sinB = (a) 1 (b) 2 (c) 3 (d) 4
(a) 2 (b) – 4 (c)  4 (d) 4 5

Rakesh Yadav Readers Publication Pvt. Ltd. 196

For More Visit : www.LearnEngineering.in


For More Visit : www.LearnEngineering.in

336. If A + B + C = 180°, then sin2A 3


p 1 – q2 q q 2 –1
+ sin2B + sin2C = ? (c) (d) 346. If A + B + C = , then cos2A +
(a) 4 sinA sinB cosC q 1 – p² p 1 – p² 2
(b) 4 cosA cosB cosC 341. If y = a cos²x + 2bsinx cosx + c cos2B + cos2C = ?
(c) 4 sinA sinB sinC sin²x and z = a sin²x – 2b sinx (a) 1 – 4 sinA sinB sinC
(d) 8 sinA sin B sin C cosx + c cos²x, then (b) 1 – sinA sinB sinC
337. tan10° – tan50° + tan70° = ? (a) y + z = a + c (c) 1– 2sinA sinB sinC
(b) y + z = a + b (d) 1 – 3 sinA sinB sinC
(a) 0 (b) 1 (c) 3 (d) 3 (c) y + a = x + b sin(x  y) a  b tan x
338. For what value of  does the (d) None of these 347. sin(x – y) = , then tan y = ?
equation 4cos + 3cos 2 – 342. If A + B + C = , then a–b
cos A cos B (a) a (b) b
2sin3 + cos4 = 2 3 – 1 ?
+ + (c) a/b (d) b/a

ir
sinBsinC sinC sin A
    2 2

v.iSn
(a) (b) (c) (d) cos C 
6 3 4 2 =? 348. tan – tan – 3 tan 5 tan
sin A sin B 5 15
339. If (sinA+ sinB +sinC)² = sin²A + (a) 0 (b) 1 (c) 2 (d) 3
sin²B + sin²C, then which one is 
343. If sinA, cosA and tan A are in =?
true? 15

dnag
GP, then cos3A + cos²A = ?
(a) sinA+ sinB + sinC = 0 (a) 1 (b) 2 (c) 3 (d) 4 (a) 0 (b) 1 (c) 2 (d) 3
(b) cosA + cosB + cosC = 0 344. If A+ B = C and tan A = k tan B, 349. If A + B + C = 180°, then tan²
1 1 1 and A – B = , then sinC = ?
(c) + + =0 A B C

eYari
sin A sinB sinC (a) 0 (b) 1 + tan² +tan² =?
2 2 2
(d) None of these k 1 k 1 (a) 1 (b) 3 (c) 2 (d) 0
sin x cos x (c) (d) sin
k –1 k –1 cot ²15 – 1
340. If sin y = p and cos y = q, then 350. =?
345. If tan , tan are the roots of x² cot ²15  1
tan x =?
p
(a) q
q² – 2
1 – p²
p
(b) q
q² – 1
1 – p²
+ px + q = 0
snhe
(p  q) then tan ( + ) = ?
p
(a)
2
3

2
(b)
2
3
2
(a) p – 1 (b) q – 1
kgei
(c) – (d)
3 3
(c) 2q + p (d) None of these
ERna

ANSWER KEY
1. (b) 36. (c) 71. (b) 106. (a) 141. (c) 176. (c) 211. (c) 246. (d) 281. (d) 316. (a)
2. (d) 37. (a) 72. (c) 107. (d) 142. (b) 177. (d) 212. (c) 247. (b) 282. (d) 317. (d)
3. (a) 38. (b) 73. (b) 108. (a) 143. (d) 178. (c) 213. (b) 248. (a) 283. (d) 318. (a)
4. (a) 39. (c) 74. (b) 109. (b) 144. (a) 179. (c) 214. (d) 249. (a) 284. (a) 319. (b)
aBryn

5. (d) 40. (c) 75. (a) 110. (a) 145. (b) 180. (c) 215. (a) 250. (a) 285. (b) 320. (a)
6. (d) 41. (d) 76. (b) 111. (a) 146. (b) 181. (a) 216. (d) 251. (b) 286. (d) 321. (a)
7. (d) 42. (d) 77. (c) 112. (a) 147. (b) 182. (a) 217. (c) 252. (b) 287. (c) 322. (c)
8. (c) 43. (b) 78. (c) 113. (d) 148. (c) 183. (c) 218. (b) 253. (a) 288. (c) 323. (b)
9. (c) 44. (c) 79. (d) 114. (a) 149. (a) 184. (d) 219. (b) 254. (a) 289. (a) 324. (d)
10. (b) 45. (b) 80. (d) 115. (d) 150. (a) 185. (c) 220. (b) 255. (b) 290. (a) 325. (d)
11. (a) 46. (a) 81. (a) 116. (c) 151. (b) 186. (d) 221. (b) 256. (b) 291. (a) 326. (a)
Les

12. (a) 47. (c) 82. (c) 117. (a) 152. (b) 187. (d) 222. (c) 257. (c) 292. (d) 327. (b)
13. (b) 48. (d) 83. (c) 118. (c) 153. (b) 188. (c) 223. (b) 258. (a) 293. (a) 328. (b)
wa. th

14. (c) 49. (c) 84. (c) 119. (b) 154. (d) 189. (a) 224. (d) 259. (d) 294. (b) 329. (c)
15. (c) 50. (a) 85. (d) 120. (d) 155. (d) 190. (c) 225. (d) 260. (d) 295. (c) 330. (d)
16. (b) 51. (b) 86. (c) 121. (c) 156. (b) 191. (c) 226. (c) 261. (a) 296. (a) 331. (a)
17. (a) 52. (a) 87. (b) 122. (a) 157. (d) 192. (d) 227. (d) 262. (d) 297. (a) 332. (d)
18. (c) 53. (a) 88. (b) 123. (c) 158. (a) 193. (c) 228. (c) 263. (b) 298. (a) 333. (c)
wwM

19. (b) 54. (b) 89. (a) 124. (a) 159. (a) 194. (c) 229. (c) 264. (c) 299. (c) 334. (c)
20. (a) 55. (a) 90. (c) 125. (c) 160. (d) 195. (c) 230. (a) 265. (b) 300. (b) 335. (b)
21. (d) 56. (c) 91. (b) 126. (d) 161. (b) 196. (a) 231. (c) 266. (d) 301. (d) 336. (c)
22. (b) 57. (c) 92. (b) 127. (b) 162. (a) 197. (d) 232. (d) 267. (a) 302. (a) 337. (c)
23. (d) 58. (b) 93. (c) 128. (b) 163. (c) 198. (a) 233. (d) 268. (c) 303. (c) 338. (a)
24. (c) 59. (c) 94. (d) 129. (a) 164. (b) 199. (c) 234. (b) 269. (d) 304. (c) 339. (a)
25. (b) 60. (c) 95. (d) 130. (d) 165. (b) 200. (a) 235. (d) 270. (c) 305. (d) 340. (b)
26. (b) 61. (b) 96. (c) 131. (d) 166. (c) 201. (c) 236. (c) 271. (b) 306. (a) 341. (a)
27. (c) 62. (d) 97. (c) 132. (c) 167. (b) 202. (c) 237. (b) 272. (b) 307. (a) 342. (c)
28. (d) 63. (d) 98. (a) 133. (b) 168. (b) 203. (a) 238. (b) 273. (a) 308. (d) 343. (a)
29. (b) 64. (d) 99. (c) 134. (d) 169. (c) 204. (b) 239. (c) 274. (d) 309. (c) 344. (d)
30. (d) 65. (d) 100. (b) 135. (d) 170. (c) 205. (b) 240. (d) 275. (d) 310. (b) 345. (b)
31. (a) 66. (a) 101. (c) 136. (b) 171. (a) 206. (d) 241. (b) 276. (b) 311. (b) 346. (a)
32. (b) 67. (b) 102. (a) 137. (b) 172. (c) 207. (a) 242. (b) 277. (a) 312. (d) 347. (c)
33. (d) 68. (a) 103. (c) 138. (b) 173. (b) 208. (c) 243. (a) 278. (c) 313. (a) 348. (d)
34. (b) 69. (c) 104. (b) 139. (a) 174. (b) 209. (b) 244. (d) 279. (c) 314. (c) 349. (a)
35. (a) 70. (c) 105. (a) 140. (a) 175. (a) 210. (c) 245. (d) 280. (d) 315. (a) 350. (a)

Rakesh Yadav Readers Publication Pvt. Ltd. 197

For More Visit : www.LearnEngineering.in


For More Visit : www.LearnEngineering.in

SOLUTION

6. (d)
tan cot 2 0 cot180
1. (b) =2 = cos 22  (sin 50  sin2 10 0  ......  sin2 850 )  sin2 90 0
2
tan cot tan72.sec2 68
By componendo and dividendo
( tan72° = cot18°)) t  85  5  

rule, No. of terms  
   1 = 17

= cos²22º +cos²68º  5  

tan 3
 3 =cos²22º +sin²22º =1 17 1
cot 1 Sum of series= +1= 9
(  sin²q + cos²q = 1) 2 2

r
sin sin 4. (a) tan15º cot75º + tan75º cot15º use this approach 
 =3

Sni
cos cos = tan15º.cot(90–15º)+ tan(90– in exam 
 
15).cot15º
sin²q = 3cos²q

gv.i
= tan15°.tan15º + cot15°. sin39º
7. (d) +2tan11ºtan31º
sin²q = 3(1–sin²q) cot15º cos 51º
4sin²q = 3 = tan²15° + cot²15° ....(i) tan45ºtan59º tan79º –

ridna
[Formula] 3(sin²21º + sin²69º)
3 3 cot(90° – q) = tan q
sin²q =  sinq = = cos(90
sin 390
 2 tan 110 . tan (900  110 ).tan 310
4 2 tan(90° – q) = cotq]
0
 390 )

eeYa
Alternate:- .tan(90 0  310 ).1 3(sin 2 210 sin 2 (90 0  210 ))
Put value of tan 150 ,
tan cot sin 390
=2  1  =  2 tan 110. cot 110. tan 310.
tan– cot cot150  
 tan150  sin 390
By C and D rule, 
geisnh 
 1 1 (2  3) 
0
cot15     cot 310  3(sin2 210  cos 2 210 )
tan 3 
 (2  3 ) (2  3 ) (2  3) 
= = 1 2 3  0
cot 1 0
cot 15 = 2 + 3
cos 2
Enak

tan² = 3
Now put value in eq (i) 8. (c) =3
tan = cot²– cos 2
3 = tan 60º
tan2 150  cot 2 150  cos2  =3(cot2  – cos2  )
 = 60°
R

= (2– 3 )² + (2+ 3 )²  cos2  =3cot 2  – 3cos2 


3  4cos2  = 3cot2 
 sin = sin60° = =4+3– 4 3 +4+3+ 4 3
aryn

2
= 14 cos ²
2. (d) cot10º. cot20º .cot60º  4cos²q = 3
Les B

5. (d) sin(2x–20º) = cos(2y+20º) sin ²


.cot70º . cot80· (2x–20º) + (2y + 20º) = 90º 2
 4sin  =3
[In cot A.cot B If A  B  90° if sinA  cos B 
then, cotA.cotB  1]   3
then  sin 60º
sin  =
wa. th


 0  2
c ot10 .cot 80.cot 20.cot 70 .cot 60  A  B  90    = 60º
1 2(x  y )  90 0 Alternate method:-
3 Put  = 60°
wwM

x  y  450
1 cos2 60
= 1 × 1× tan(x  y )  tan450  1 =3
3 cot 60 – cos2 60
2
Alternate:-
3. (a) sin(2x – 20º) = cos(2y + 20º) 1 1
 sin(2x – 20º) = sin [90º – 4 4 =3
 1   LHS = =
cot180 cot 720.cos2 220   (2y + 20º)] 1 1 1

 tan 72.sec2 680  –
 2x – 20º = 90º – (2y + 20º) 3 4 12
cot180  2x – 20º = 90º – 2y – 20º  LHS = RHS = 3
cot180.cot 720.cos 2 220 
tan 72.sec 2 680  2x + 2y = 90º Here,
1  2 (x + y) = 90º 3=3
 x + y = 45º
(If A + B = 90º, then cotA.cotB = 1) So,  = 60°
 tan (x + y) = tan45º = 1

Rakesh Yadav Readers Publication Pvt. Ltd. 198

For More Visit : www.LearnEngineering.in


For More Visit : www.LearnEngineering.in

A tan11tan 29   + 30º +  = 90° Alternate:-


9. (c)  2  = 60° (secA – cosA)2 + (cosecA – sinA)2
B 2cot 61cot 79
 = 30° – (cotA – tanA)2
sin  + cos2  put  = 45°,
A tan 11º tan 29º sin30º + cos2 × 30º = 1 = (sec45° – cos45°)2 + (cosec45°

B 2cot(90º–29º ) cot(90º–11º ) 12. (a) If, tan  =1 – sin45°)2 – (cot45° – tan45°)2
2 2
A tan 11º tan 29º It means,  =45º  1   1 
 =  2 –  +  2–

 – (1 – 1)2

B 2 tan 11º tan 29º  2  2
8 sin   5 cos 
A 1 sin 3  – 2 cos 3   7 cos  1 1
 = + –0=1

ir
B 2 2 2
8 sin 45º 5 cos 45º

v.iSn
2A=B = 16. (b) tan  +cot  =2
sin 45º–2 cos 3 45º 7 cos 45º
3
If we put  =45º,
10. (b) sin  + cos  = 2
1 1 tan45º+cot45º=2
put,  = 90°,  = 0° 8 5 1+1=2  2=2
2 2

dnag
sin90° + cos0°= 2 = 3 3
=2 So,  =45º

 1   1   1 
 1+1 =2   – 2   7  tan5  +cot10 
2 = 2 matched  2  2  2 = tan545º+cot1045º

eYari
So,  = 90°,  = 0° 13. (b) cos2  + cos4  = 1 = (1)5+(1)10= 1+1=2
cos4  = 1 – cos2  7
 2   
o 17. (a) sin  – cos  =
cos4  = sin2  13
 sin  
 3 

2  90  0 o
= sin 
 
 = sin
 180 
 
o
cos²  =
snhe
cos²  .cos²  = sin² 
sin ²
cos ²
sin  + cos  = 2 – x 2

7
2
 49 
3  3  = 2–  = 2– 
kgei
   169 
cos² = tan²  13 
3 cos² + cos4 = tan2  + tan4 
= sin60° = cos30° = 289 17
2  cos 2  + cos 4  = 1 = 
ERna

169 13
Take option 'b' Here, tan 2  + tan 4  = 1
1
 90 3 sin   2 cos  18. (c) 2cos  –sin  =
cos = cos = cos30° 14. (c) 2
3 3 3 sin  – 2 cos 
aBryn

divide numerator & de put value of  =45º,


So, this is answer.
nominator by cos  1
11. (a) sin  sec(30º+  )=1 =
sin  2 cos  2cos45º–sin45º=
put value of  between 0ºto 60º, 3
cos 

cos 
2
if  =30º 3 sin  2 cos 
Les

– 1 1 1
cos  cos   sin 
sin30°sec(30º+30°)=1   tan 2× – =
wa. th

3 tan   2 
 cos  2 2 2
sin30° sec60º=1
=
3 tan  – 2 1 1
1 put value of tan 
 ×2=1 =
2 2 2
4
wwM

 1=1 (satisfy)
3  2 So, it satisfies with question.
3 6
 3 Now,
So,  =30º = 4 2
3 – 2 2sin  +cos  = 2sin45º+cos45º
sin  + cos2  3
=sin30º+cos2×30º 15. (c) (secA – cosA) 2 + (cosecA – 1 1 3
1 1 sinA) 2 –(cotA – tanA)2 = 2× + =
= sin30º+cos60º = + =1 2 2 2
= (sec2A + cos2A – 2secA cosA) +
2 2
(cosec2A + sin2A sin   cos 
Alternate:- 19. (b) 3
– 2cosecA sinA) – (cot2A + tan2A sin  – cos 
If, sin  sec  = 1 – 2cotA tanA)
sin  +cos  =3sin  –3cos 
then,  +  = 90° = sec2A – tan2A + cos2A + sin2A
+ cosec2A – cot2A – 2 sin  2
sin  sec(30° +  ) = 1 2sin  =4cos   
=3–2=1 cos  1

Rakesh Yadav Readers Publication Pvt. Ltd. 199

For More Visit : www.LearnEngineering.in


For More Visit : www.LearnEngineering.in

23. (d) sin  +cosec  =2


 tan
2 P

1 B put  =90º
=  2 +  2 –  2 .  2
2 2 2 2

=4–4=0
 P 2 B 1 
 sin90º+cosec90º=2
 sin  and cos =   x P
H H  1+1=2 29. (b) Sin17° =
5 5  y  H


 2=2
It, satisfies the question y
5 73o
sin5  +cosec5  x
= sin590º+cosec590º
17o
= (1)5+(1)5 = 1+1=2
24. (c) tan2  . tan4  = 1 y2 – x2

r
4 4
 sin  –cos  [If tanA. tanB = 1, then A + B = 90o]  Sec17o – Sin73o

Sni
 (sin2  +cos2  )(sin2  –cos2  ) 2  + 4  = 90o

gv.i
 1(sin2  –cos2  )  6  = 90o y y2 – x 2
= –
 3  = 45o y2 – x 2 y
 2 2  1 2 4 1 3

   –
   5–55  tan3q = tan45o = 1

ridna
 5  5
25. (b) cos2  + cos2  = 2 y 2 – (y 2 – x 2 )
2 2
20. (a) sec  +tan  =7 =
put value of  =  = 0o (y )( y 2 – x 2 )
 1 + tan²+tan²= 7

eeYa
 2tan²= 6  cos20o + cos20o = 2
y2 – y2  x 2 x2
 tan²= 3  (1)2 + (1)2 = 2 = =
2=2 y y2 – x2 y y2 – x2
 tan= 3 = tan60º
geisnh
[If satifies the question]
  = 60° cot 30o – cot 75o
tan3  + sin5  30. (d)
Alternate:- tan 15o – tan 60o
take help from option = tan30o + sin50o = 0 + 0 = 0
put  =60º, 26. (b) A tan 60o – tan 15o
Enak

=
sec260º+tan260º=7  tan 15o – tan 60o
(2)² + ( 3 )² = 7 o 4 – tan15 – tan 60
B 75 C
R

7= 7(matched) = = –1
c o
tan15 – tan 60
So,  =60º  180
= = 45o Alternate:-
21. (d) (secx .secy+tanx .tany) 2 –
aryn

4 4
(secx . tany + tanx .secy)2 cot 30º – cot 75º cot 30º – cot 75º
 BAC = 180o – 75o – 45o = 60o =
put value of x=y=45º,
Les B

tan15º – tan60º cot 75º – cot 30º


= (sec45ºsec45º+tan45ºtan45º)² 180o  c
cot 30º – cot 75º
– (sec45ºtan45º+tan45ºsec45º)2 = – cot 30º – cot 75º = – 1
c
2

= ( 2  2  1  1) – 2  1  1  2 
2
1° 
180o
 
31. (a) cot  . tan (90o –  ) – sec
wa. th

2
= 2  1 – 2 2   2
= 9 – 8 =1 c c (90o –  ) cosec + (sin225o + sin265o)
60o  o × 60 =
o
radian
22. (b) According to question, 180 3 + 3 (tan5o . tan15o . tan30o .tan75o
A = sin²  + cos4  27. (c) (x + 5)o + (2x – 3)o + (3x + 4)o = 180° tan850)
wwM

Put  = 90º for maximum value (Sum of all angles in triangle =cot  .cot  – cosec  . cosec 
of A is 180°) + (sin 2 25° + cos 2 25°) +
A = sin² 90º + cos4 90º 6x + 6o = 180o
A= 1+0 (x + 1) = 30o 3 [(tan5°. tan85°). (tan15 o
A= 1  x = 29o .tan75o) . tan30o]
Put  = 45º 28. (d) A + B = 90o =(cot2  – cosec2  ) + 1 + 3 (1.
for minimum value of A (Complementary angle)
A = sin²45º + cos445º 1
We can put A = B = 45o 1. ) tanA.tanB = 1
1 1 3 If A+B =90
o

A= + Or A = 30o , B = 60o
2 4 = Sec2A + Sec2B – Sec2A.Sec2B 1
= (–1) + 1 + 3 ×
3 3 = Se c 2 45 o + Se c 2 45 o – 3
A=  A lies in  A 1 = –1 + 1 + 1 = 1
4 4 Sec245o.Sec245o

Rakesh Yadav Readers Publication Pvt. Ltd. 200

For More Visit : www.LearnEngineering.in


For More Visit : www.LearnEngineering.in

32. (b) sin (3x – 20o) = cos (3y + 20o)  (7  + 28o) + (30o – 3  ) = 90o 40. (c)  2cot  = 3
[If, sinA = cosB then, A + B = 90°]
 4  + 58o = 90o 3
 (3x – 20o) + (3y + 20o) = 90°  cot  =
 4  = 32o   = 8o 2
 3x + 3y = 90o
   divide numerator and denomina-
 x + y = 30o
36. (c) tan  –  = 3 tor by sin  ,
Alternate:- 2 2
sin(3x – 20º) = cos(3y + 20º) 2cos  – sin  2cot  – 1
   2cos   sin  = 2cot   1
sin (3x – 20º) = sin [90º – (3y +  tan 90 – 2  = 3 [  = 180 ] o

20º)]  
3x – 20º = 90º – 3y – 20º  3
2 – 1 3 –1 2 1
(  sin(90º –  ) = cos )
 cot
2
= 3 = cot30o =
2
= = =

ir
3 3  1 4 2
3x + 3y = 90º  2  1

v.iSn
 = 30o 2
 x + y = 30º 2
33. (d) cos  . cosec23o = 1 41. (d) sin6  + cos6 
  = 60o
= (sin2  )3 + (cos2  )3
1 1
cos . =1  cos60o = = (sin2  + cos2  ) (sin4  + cos4 

dnag
sin23º 2
– sin2  cos2  )
 cos = sin23º 37. (a) Since, value of cos  de-
creases, from 0° to 90° at 45° it = (sin2  + cos2  )2 – 2 sin2  cos2
(If sinA = cosB then A + B = 90o)
is equal to the value of sin  .  –sin2  cos2  )

eYari
1 ( sin²  +cos²  =1)
= cosB Similarly,
cosecA = (1 – 3sin2  cos2  )
Value of sin  increases from 0
sinA = cosB
to 90° and at 45° it is equal to 1 3 1
A + B = 90o
23o + B = 90o
B = 90o – 23o = 67o
cosB = cos 
snhe
the value of cos 
For 0° <  < 45°, cos  > sin 
So, value of cos 25° – sin25° is
=1–3×
4
=1 – =
4 4
42. (d) By trigonometric identity,
sec2  – tan2  = 1
kgei
alway positive but less than1. (sec  + tan  )(sec  –tan  ) = 1
 B =  = 67º A
38. (b) 1
  = 67º  sec  – tan  = ....(i)
2
ERna

Alternate:- 5
4 and given, sec  + tan  = 2
cos .cosec23º = 1
On adding Eqs. (i) and (ii) we get,
1 1
cos . =1 B C 2sec  = +2 ....(ii)
aBryn

sin23º 2
 ABC,
cos = sin23º 5
4 sec  =
cos = cos (90º – 23º) cos A = i.e., 4
5
Alternate:-
 = 90º – 23º AB = 4 and AC = 5
Les

sec + tan = 2
  = 67º AB 4
wa. th

sinC = = 1 sin
34. (b) 2 (cos2  – sin2  ) = 1 AC 5 + =2
cos cos
39. (c) Since,  and  are comple-
1 1  sin
 cos2  – sin2  = mentary angle.
2 =2
  = 90 –  cos
wwM

Squaring both sides,


1 Now,
 2sin2  = 1 – 1  sin
2
2 cos  c osec – cos  sin  =4
cos ²
1 cos  2
 sin²  = = – cos  sin  1  sin
4 sin  =4
1 – sin ²
1 cos  2
 sin  = = sin30o = cos(90 – ) – cos .cos 90 –   1  sin
2 =4
1 – sin1  sin
 = 30 o
cos 
= – cos .cos  1  sin
35. (a) sec(7  + 28)o = cosec(30o – 3  ) cos  =4
1 – sin
[If secA. sinB = 1, then A + B = 90°] = 1 – cos 2  = sin2  = sin 1 + sin = 4 – 4sin

Rakesh Yadav Readers Publication Pvt. Ltd. 201

For More Visit : www.LearnEngineering.in


For More Visit : www.LearnEngineering.in

5sin = 3 47. (c)  5sin  + 12cos  = 13


sin   cos 
3 On squaring both sides, we get
sin =
25sin2  +144cos 2  +120 sin = sin  cos   1
5 sin   cos 
cos = 1 – sin ²  cos  = 169 sin  cos 
 25 (1 – cos 2  ) + 144(1 – Alternate:-
 3 2
cos = 1– 
   sin2  ) + 120sin  cos  = 169
5 
 25 – 25cos2  + 144 – 144
sinθ + cosθtanθ  cot θ
2
sec θ  cosec θ
9 sin  + 120sin  cos  = 169
cos = 1– Put θ = 45º
25  25cos2  +144sin2 
4 – 120sin  cos  = 169 – 169

r
 1 1 
cos =
5 2
(5cos  – 12sin  ) = 0

  1  1

  2 2

Sni

1 5  5cos  – 12sin  = 0 2 2

gv.i
 sec = =
cos 4
4sin  – cos  2
43. (b) cosec(75°+  ) – sec(15°–  ) – 48. (d) 2 4 1
4sin   9 cos  2 
  =1
tan (55°+  ) + cot(35°–  ) 2 2 2

ridna
On dividing both numerator and 2 2
 cosec(75°+  ) – cosec[90° – (15° –  )] denominator by cos  , we get 51. (b) sin  6 + cos6  +3sin2  c os2 
–tan(55° +  ) + tan [90° – (35° –  )]
4sin  cos  = (sin2  )3 + (cos 2  ) 3 + 3sin2 

eeYa
 cosec(75° +  ) – cosec(75° +  ) –
cos  cos  4 tan  – 1 cos2  (sin2  + cos2  )
– tan(55°+  ) + tan(55° +  ) = 0 = 4sin  9cos  = [ (a + b)3 = a3 + b3 + 3ab (a + b)]
 4 tan   9
44. (c) sin  + 2cos  = 1 cos  cos  Here,
On squaring both sides, we get
a = sin2  and b = cos2 
(sin  + 2cos  )2 = 1
geisnh
3 –1 2 1
= =  = (sin2  + cos2  )3
 sin2  + 4cos2  + 4sin  cos  = 1 39 12 6
= (sin2  + cos2  )3
 (1 – cos2  ) + 4(1 – sin2  ) + 49. (c) sin  – cos  = 0 = (1)3 = 1 [ sin2  + cos2  =1]
4sin  cos  = 1  sin  = cos 
Enak

 –(cos2  + 4sin2  ) + 4sin  1  sec  – tan  cos 


Since, sin  and cos  are equal 52. (a) 1  sec  tan 1 – sin 
cos  = 1 – 5 for  = 45°
R

 cos2  + 4sin2  – 4sin  cos  = 4  sin4  + cos4  = (sin45°)4 +  1 sin  


 (2sin  – cos  )2 = 4 1  –  cos 
(cos45°)4 cos  cos  
=
aryn

 2sin  – cos  = 2  1 sin  


1    (1 – sin )
4 4
 1   1  cos  cos  
45. (b) tan8  = cot2    + 
Les B

2  2
 tan8  = tan(90 – 2  )  cos   1 – sin  
1 1 1 1 2 1   cos 
 8  = 90 – 2  =  = = = =  cos 
  = 9°
4 4 4 4 2 cos   1  sin 1 – sin 
wa. th

sin   cos tan   cot  cos 


 tan5  50. (a)
sec   cosec cos   1 – sin 
 tan45° = 1 = cos   1  sin  – sin  cos  – sin  – sin2 
sin  cos  
46. (a) sin(A + B) = 1 sin   cos     cos 
cos  sin  
wwM

A + B = sin–11 = 1 1 cos   1 – sin 


 =
 (A + B) = 90° cos sin cos   1 – sin2  – sin  cos 
( sin–1 1= 90º)  sin2   cos 2   cos 
sin   cos  
 B = 90° – A  A = 90° – B  sin cosθ  cos   1 – sin 
Now, = sin   cos  = cos   cos 2  – sin  cos 
cos (A – B) = cos A cosB + sin A sin  cos  cos 
sinB ( sin² θ +cos² θ =1)
[ 1–sin2  = cos2  )
= cos (90 – B) cosB + sin (90 – B) 1
sinB sin   cos   
 cos   1 – sin 
sin cosθ  = 1
= sinB cosB + cosB sinB = cos  cos   1 – sin 
sin   cos 
= 2sinBcosB = sin2B cos 
sin  cos 

Rakesh Yadav Readers Publication Pvt. Ltd. 202

For More Visit : www.LearnEngineering.in


For More Visit : www.LearnEngineering.in

53. (a) sin  + cos  = 3 55. (a) cosec(75° +  ) – sec(15°–  ) Option:- (c)
= cosec(75°+  ) – sec[90 – (75° +  )] 2sec²A cosec²A
On squaring both sides, we get, = 2×sec²45º cosec²45º
2
= cosec (75° +  ) – cosec (75° +  )
(sin  + cos  )2 =  3 =0 = 2 × 2 × 2 = 8(satisfy)

 sin2  + cos2  + 2sin  cos  = 3 56. (c) In  ABC, if  C is 90°, then sin A
tan A – sin A – sin A
 A +  B = 180° – 90° = 90° 60. (c) = cos A
 1 + 2sin  cos  = 3 sin3 A sin3 A
Now,
( sin² θ +cos² θ =1) cos(A + B) + sin(A + B)
 1 
3 –1 2 =0+1=1 SinA 
 – 1
 1 – cos A
 sin  cos  =  1 =  cos A  =
2 2 3 sin ³ A cos A sin ² A

ir
57. (c) sin  =
Now, 2 Rationalising above equation

v.iSn
sin  cos   3 1 – cos A  1  cos A 

 = 60°  sin60  2 
tan  + cot  =
cos  sin   = cos A.sin2 A  1  cos A 

sin2   cos 2  Now,

dnag
1
=
sin .cos 

sin .cos 
1 – cos A
2

3 =
cos  = cos A.sin2 A 1  cos A 
1 2
 tan  + cot  = =1 sin2 A

eYari
1  3 = cos A.sin2 A 1  cos A
54. (b) tan  + sec  = m   = 30°  sin30  2   
 sec  = m – tan  1 1 sec A
and tan  = 1 . 
On squaring both sides, we get =
(sec  )2 = (m – tan  )2
 sec2  = m2 + tan2  – 2m tan 
snhe
  = 45° ( tan45º = 1)
  +  +  = 60° + 30° + 45° = 135°
58. (b)Given that, cosA + cos2 A = 1
cos A 1  cos A 1  cos A
Alternate:-
tan A – sin A
kgei
 sec2  – tan2  = m2 – 2m tan   cosA = 1 – cos2A = sin2A . (i) sin3 A
 1 = m2 – 2m tan  Now, Put A = 45º
( sec2  – tan2  =1) 2(sin2A + sin4A)
1
ERna

= 2 (sin2A + cos2A) [from Eq. (i)] 1–


m –1 2 tan 45º – sin45º 2
 tan  = = 2. (1) [ sin2  + cos2  =1) = 2 = =
2m sin3 45º  1 3
59. (c) (1 – tanA)2 + (1 + tanA)2 + (1 – 
 

On putting the value of tan  in cotA)2 + (1 + cot A)2  2
aBryn

initial equation, we get = 1 + tan2 A – 2 tan A + 1 + tan2A


2 –1
+ 2 tan A + 1 + cot2 A – 2cot A + 1
m2 – 1 + cot2 A + 2 cot A 2 2 –1
+ sec  = m = = 2 2
2m = 4 + 2 (tan2A + cot2A) 1 2
= (2 + 2tan2A) + (2 + 2 cot2A) 2 2
m2 – 1
Les

 sec  = m – = 2sec2A + 2 cosec2A = 2


2m  2 –1 
wa. th

2
2m – m  1 2
m 1 2  1 1  Now check from options
= 2   
 sec  = = cos2 A sin2 A  sec A
2m 2m
Option:-
Alternate:- 1  cos A
 sin2 A  cos 2 A 
wwM

tan θ + sec θ = m =2  sin 2 A.cos2 A  sec 45º 2


= = 1
Put θ = 0º 1  cos 45º 1
 tan0º + sec0º = m 2. 1 2
 0+1 =m =
sin2 A.cos 2 A 2 2 2
 m =1 = 2sec² A. cosec2A = =
2 1 2 1
 sec θ = sec0º = 1 Alternate:- Rationalise above equation
Now check from option (1 – tanA)² + (1 +tanA)² +
2 2 –1
m2  1 11 (1 – cotA)² + (1 + cotA)² = 
Option:- (b) : = Put A = 45º 2 1 2 –1
2m 2 1
(1–1)² + (1+1)² + (1–1)² + (1+1)² 2 2 –1
2 4+4=8 =   = 2 2 –1  (Satisfy)
= = 1 (Satisfy) 2 –1
2 Now check from option

Rakesh Yadav Readers Publication Pvt. Ltd. 203

For More Visit : www.LearnEngineering.in


For More Visit : www.LearnEngineering.in

x 2 – y2  tan2  = 9  1 + tan2  = 10
  
61. (b) Given that, sin  = 2
x  y2  sec2  = 10  sec  = 10
2sin .cos 1  2cos 2 – 1
2 2 2
=  +
 
 cos2  = 1 – sin2  1 1  2cos 2 – 1 2sin .cos
 cos  = .......(i) 2 2 2
 x 2 – y2 
2 10
=1–  2
 x  y 2   sin2  = 1 – cos2  
sin cos

= 2  2
1 9  
x 2 2
 y 2  – x 2 – y 2 
2
=1–  cos sin
10 10 2 2
= 2
x 2
 y2 
3
 sin  = .......(ii)  2  2 

r
2x 2.2y 2 10 2  sin 2  cos 2 
= .

Sni
= x 2  y2 2 Now,
  2 
sin .cos

3 1 4 2 2

gv.i
4x 2.y 2 2 sin  + cos  = + =
 2xy  10 10 10
= x 2  y2 2 =  2 2
   x  y 2  (since,  lies in first quadrant) = = 2cosec 
sin

ridna
 cos  = 2xy/x2 + y2 64. (d) If 0°<  < 90°, then all the trigo-
Alternate:-
Alternate:- nometric ratios can be obtained
when any one of the six ratios is sin 1  cos
x 2 – y2 +

eeYa
sin θ = 2 given. 1  cos sin
x  y2 Since, We use any of the follow-
Put θ = 90º
Put θ = 30º ing identity to get any trigonomet-
ric ratios sin90º 1  cos 90º
x= 3 +
geisnh
sin2  + cos2  = 1, 1 + tan2  1  cos 90º sin90º
y= 1
= sec2  and 1 + cot2 
1 1
2 2
= cosec2   2
  3 – 1 1 1
sin30º = 2 2 65. (d) sinA.cosA.tanA + cosA.sinA . cotA Now check the option,
Enak

 3  1 Option: (a)


= sinA.cosA. sin A + cos A .
1 3 –1 cos A 2cosec θ
=
R

2 3 1 cos A = 2cosec90º
sinA. = sin2A + cos2A = 1 = 2 × 1 = 2 (Satisfy)
1 2 sin A
=
aryn

2 4 [ sin2  + cos2  =1) 3


1 1 = cosec A – cot2A
2 67. (b) Given that, sin  .cos  =
4
Les B

= (Satisfy)
2 2 [ cosec²A – cot²A=1)
62. (d) Given that,  sin 4  + c os 4  = (s in 2  +
Alternate:-
cos2  )2 –2sin2  cos2 
1  2sin .cos  sinA cosA tanA + cosA sinA cotA
 a2 = = (1)2 – 2(sin  .cos  )²
1 – 2sin .cos  Put A = 45º
wa. th

sin 2
  cos2   2sin .sin 
 3
2
1 1 1 1 3 3 5
a = 2
sin 2
  cos2  – 2sin .cos   × ×1 + × ×1 =1 – 2  4  = 1 – 2. =1– =
2 2 2 2   16 8 8
2
sin   cos 
 a2 = 1 1 Alternate:-
wwM

sin  – cos 2  + = 1


2 2 3
a sin   cos  sin θ cos θ =
Now check from option, 4
 = sin  – cos 
1 cosec² A – cot² A
(applying componendo dividendo Put θ = 30º
Put A = 45º
formula)
2
3
a  1 sin   cos   sin  – cos  sin30º cos30º =
 =
a –1 sin   cos  – sin  – cos 
 2 – 1 4

2–1 =1 (Satisfy) 1 3 3
a  1 2sin   × =
  = tan 66. (a) Let 2 2 4
a – 1 2cos 
63. (d) Given that,  lies is in first sin  1  cos  3 3
f(  ) =   = (Satisfy)
quadrant and tan  = 3 1  cos  sin  4 4

Rakesh Yadav Readers Publication Pvt. Ltd. 204

For More Visit : www.LearnEngineering.in


For More Visit : www.LearnEngineering.in

 sin4 θ + cos4 θ Also, there is only one value of x 77. (c) A  B  C    1800
in the first quadrant that satis-
 sin430º + cos430º
fies in sinx – cosx = 0 A  B 180 C
 3 4
4   
1   
72. (c) 7sin2  + 3cos2  = 4 2 2 2
 
   +  
2   2   7sin2  + 3(1 – sin2  ) = 4
AB
 7sin2  + 3 – 3sin2  = 4  sin 
1 9  2 
 +  4sin2  = 1
16 16  c  c
1 9 10 2
1 = sin    cos 2
2  2 
 sin  =  
 = 4
16 16 Similarly,

ir
1
5  sin  = = sin30° A B  c

v.iSn
 2 cos    sin
8 2 2
 
68. (a) We know that, sin 2  +  c  180
 = 30o = 6  
cos2  = 1 is true A B  c
cot   tan

dnag
I. sin21° + cos21º = 1 which is 73. (b) tan(2 450 )  cot 3  2  2
also true.
II. sec2 33º – cot2 57° = cosec237° [if tanA =cotB then A + B = 90º] A B  c
tan   cot
– tan²53° (2q+ 45°) + 3q =90°  2  2

eYari
Now,
sec2(90 – 57)º = cosec257º and 5 450  900 So, option (C) is incorrect
cot257º = cot2 (90–33)º = tan233° 78. (c) tan2 θ tan3 θ = 1
45
 sec233° – cot257° = cosec257º –   90 (2 3)  900
tan² 33º
Hence,
Statement II is not true.
5
snhe
15  Base
74. (b) cos  = 17  Hypo.
5 900
(If tanA . tanB = 1 then A + B = 90°)
69. (c) Given, 2 5
kgei
p = asinx + bcosx = 2 cos  1 = 2cos²45º – 1
2
q = acosx – bsinx
1
ERna

 p2 = a2 sin² x + b2 cos2 x + 2ab sin =2× –1 = 0


2
x cosx
and q2 = a2 cos2x + b2sin²x – 2 ab  πx  1
79. (d) 2sin   = x² + x ²
sinxcosx  2 
aBryn

 p2 + q2 = a2 (sin2x + cos2x) + b2 Let x = 1


perpendicular = 8
(cos2x + sin2x) = a2 + b2
= cot (90º – θ ) = tanq 1
70. (c) Given that, sin2x + cos2x – 1 = 0 2 sin 90. 1 = 1² +
sin2x + cos2x = 1 1²
8  P
which is an identity of trigono- =  tan  2×1= 1+1
15  B 
Les

 2 = 2 matched, so x = 1
metric ratio and always true for
wa. th

every real value of x. 75. (a) (sec 4  tan4 ) 1


So, the equation have an infinite so, x –
2 2
 (sec  tan ) (sec  tan ) 2 2 x
solution.
71. (b) I. Given that, sinx + cosx = 2 1
 1 (sec 2 tan2)  1– = 0
wwM

On Squaring both sides, we get 1


 (sinx + cosx)2 = 4 (  sec² – tan² = 1)
80. (d) cos sec 2
 (sin2x + cos2x) + 2sinx .cosx = 4 7 7
 1 = Put  00
 1 + sin2x = 4 12 12
 sin2x = 3  sin2x  3 76. (b) sec x  cosec y cos 00  sec 00  2
Hence, there is no value of x in  1 + 1 = 2  2= 2
if secA  cosecB
the first quadrant that satisfies  (matched, so  = 0°)
0
sinx + cosx = 2 then A  B  90  6 6
II. sinx – cosx = 0 = cos  sec 
x  y  900 6 0 6 0
  = cos 0  sec 0
 tanx = 1 = tan  x= sin(x + y) = sin90° = 1
4 4 = (1)6  (1)6  1  1  2

Rakesh Yadav Readers Publication Pvt. Ltd. 205

For More Visit : www.LearnEngineering.in


For More Visit : www.LearnEngineering.in

5 2  1  (1  3  2 3)  (1  3  2 3)  5 5
81. (a) 2
 2
 3sin2 =  2     2sec  sec
sec  1  cot    1 3  2 4

2 4x ²  1
= 5cos2 2
 3sin2  1  4  2 3  4 – 2 3  87. (b) sec θ =
cosec  
=  – 2  
 4x
  –2 
= 
5cos2 2sin2 3sin2 tan θ = sec ²θ – 1
= 2 2 1  8 1
5(c o s   s in ) =    =  4  2 2
2  2 2  4x ²  1 
(  sin² + cos² = 1) =   –1
Alternate (I):-  4x 
=5

r
1  tan 1 – tan
(2cos² q – 1) × 
2 2
 1 1   1 1    
 4x ²  1 – 4x 

Sni
82. (c)       1 – tan 1  tan = 2
cos cot
 cos cot

4x 
= (2cos² q – (sin² q + cos² q))

gv.i
= (sec tan) (sec– tan)
2 2

1  tan  1 – tan  16x 4  1  8x ² – 16x ²
= sec2 tan2 ×   = 2
4x 

ridna
1 – tan ² 
 
[  1  tan  sec ] = 1
2 2

= (cos² q – sin²q) 16x 4  1 – 8x ²


83. (c) sin cos 5 =

eeYa
sin cos
=
4




4x  ²
1  tan ² 2 tan 1  tan ² – 2 tan
 4 sin 4 cos 5 sin 5 cos ×  sin ²  2

 1– 
 4x ² – 1
sin 9cos  cos ²  =

geisnh 4x 
2

= (cos²q – sin²q)
sin
 9 4x ² – 1
cos  2  2 tan ²  =

  × cos²q
 4x
 cos ²– sin ²
Enak

 tan 9
 sec θ + tan θ
= 2 (1+ tan²q) × cos²q
tan ² 1 9²  1 82 41
= =  = 2 sec²q. cos²q = 2 =
4x ²  1
+
4x ² – 1
tan ² – 1 9² – 1 80 40
R

Alternate(II):- 4x 4x
84. (c) tan7. tan2 1
1  tan 1 – tan 4x ²  1  4x ² – 1
aryn

[If, tanA.tanB= 1 then, A + B = 90°] (2cos² q – 1) 


  
 =
4x
1 – tan 1  tan
 7 + 2 = 90°
Les B

 9= 90° put q = 0° 8x ²


= = 2x
4x
  100 1  tan0º 1 – tan0º 
=(2cos²0º – 1) × 
1 – tan0º  1  tan0º 
 Alternate:-
 
1
wa. th

0
 tan 3 = tan 30  1
3 1  0 1 – 0  sec θ = x +
= (2 × 1 – 1) 
1 – 0  1  0 
 4x
85. (d)   put x = 1
1  tan 1  tan = (2 – 1) (1 + 1) = 2
wwM

2
(2cos  1)  1 5 H
86. (c) secq+ tanq = 2 .......(i)
 
1  tan 1  tan
 sec θ = 1 + = =
4 4 B
 sec²q – tan²q = 1
Put 600  (secq – tanq) (secq + tanq) = 1 P 3
tan θ = =
1  tan 600 1  tan 60 0  B 4
 
 2 cos 2 60 0  1  0
 
1  tan 60 0   (secq – tanq) =
1
Now,
1  tan 60 sec tan
sec θ + tan θ
 12  1  3 1  3  1
= 2( 2 )  1
   (secq – tanq) = ......(ii) 5 3 53 8
  1  3 1  3 
  2 = + = = =2
adding equation (i) and (ii) 4 4 4 4
1  (1  3 )2  (1  3 )2  by option (b),
 
= 2  1  (1  3 )(1  3 )  1
   sec tan sec tan 2  2x = 2 × 1 = 2
 2

Rakesh Yadav Readers Publication Pvt. Ltd. 206

For More Visit : www.LearnEngineering.in


For More Visit : www.LearnEngineering.in

88. (b) sin2 250  sin2 650  A + B = 60º Alternate:-


Adding both side Put value of θ = 90º [take
= sin2 250  sin2 (90  250 ) help from options]
 (A  B )  (A  B )  300  600
= sin2 250  cos2 250 = 1 sin² θ – 3sin θ + 2 = 0
Alternate:-  2A  900 sin²90º – 3sin90° + 2 = 0
If A + B = 90°  A  450 1–3×1+2= 0
then sin²A + Sin²B = 1 [  sin90º = 1]
 A - B  300 0 = 0
So, sin2 250  sin2 650 = 1
[matched]
B  A  30 0  450  30 0  150
89. (a) secq+ tanq = 3 ....(i) So, this is answer.

ir
 sec²q – tan²q =1 
15º 
 radian 96. (c) sin = msin β

v.iSn
 (secq – tanq)(secq + tanq) =1 180º 12
Squaring both sides
93. (c)152 (sin 30º + 2cos²45º
1 + 3 sin30º...........+17sin30º + sin²= m²sin² β ...(i)
 secq – tanq = ......(ii)
3 18cos²45º) tan= ntan β

dnag
Subtract equation (i) from (ii)  152 Squaring both sides
1
 2tanq = 3 –
2
1
  2 1  1 1  1 2  tan² = n²tan² β
   3  .....17   18   
3 
2 
 2
 2 2   
 2 

eYari
  sin ² sin ²
3 –1 2  n²
 2tanq =   1 1 
 152   1  1  ............  8  9
1 cos ² cos ²
3 3
2 2 2  (value put in sin² β )
 tanq =


1
3
= tan30°

1 

1 1
snhe
This is in A.P. where,

a = , d = , n = 18
2 2
sin ²

n ² sin ²
cos ² m ² 1 – sin ²
 q = 30°  tan30 
0

kgei
(from equation (i))
 3  18  1 1 
 tan3q = tan 90° = undefined  S152 = 152  2 2  2  18 – 1 2 
  

1
= m ² 1 – sin ²
 
ERna

90. (c) sin(600 )  cos( 300 ) cos ²


18  17    m² 
0 0 0  152  2 1  2  
(60 – )  (  30 )  90    (m² – sin²) = n² cos²
[if sinA  cosB 19 m²– (1 – cos²) = n² cos²
aBryn

0  152 × 9 ×  12996  m² – 1 + cos² =n²cos²


then A  B  90 ] 2
m² – 1 = cos² (n² – 1)
()  900  300 12996 = 114
94. (d) 3cos80º cosec10º+ 2cos59º m² – 1
0 cos²=
()  60 cosec31º n² –1
Les

1 1 Alternate:-
tan( )  tan 600
wa. th

 3 cos 80º  2cos 59º


sin10º sin 31º According to question,
tan60°  3 1 tan = n tan β
 3cos 80º +2
91. (b) a sin θ + bcos θ = c....(i) sin 90º –80º
sin = m sin β
let
wwM

1
a cos θ – b sin θ = x ......(ii) cos59º sin 90º –59º tan sin
  n = , m=
Squaring and adding equation tan β sin β
 3+2= 5
(i)and (ii)
= a² + b² = c² + x² 95. (d)sin² θ – 3sin θ + 2 = 0 Put = 30º and β = 60º
= a² + b² – c² = x²  sin² θ – 2sin θ – sin θ + 2 = 0 tan30º 1/ 3 1
=x =  a² + b² – c² n= = =
 sin θ (sin θ – 2)–1 (sin θ – 2) = 0 tan 60º 3 3
1
92. (b) sin( A  B )  = sin30º  (sin θ – 1) (sin θ – 2) = 0 sin30º 1/2 1
2 m= = =
[  sin θ  2] sin 60º 3 /2 3
 A – B = 30º
1  sin θ = 1 = sin 90º 3
cos(A+B) = = cos60º  θ = 90º Then, cos² = cos²30° =
2 4

Rakesh Yadav Readers Publication Pvt. Ltd. 207

For More Visit : www.LearnEngineering.in


For More Visit : www.LearnEngineering.in

by option (c), =(cos² ) (sin² ) (cosec² β )


sin θ cosθ
m² – 1 5 –3
cos θ cos θ 5 tan θ – 3 (sec² β )
n² – 1 = =
sin θ cos θ 5 tan θ  2
5 2 = cos²(90 – β ). sin² .
Put the value of m and n, cos θ cos θ
 1 2  4 cosec² β . sec²(90 – )
1 5   – 3

  –1
 –1  5 1
 3 3 = = = sin ² β .cosec² β .sin ²  .
  4  6
 1 2 1
–1 5    2 cosec² = 1

   –1  5
3  9 2sin 68º 2 cot15º
100. (b) According to question, 103. (c) – –
3 2cosec²23º cot²67º – sin²23°– cos 22º 5 tan 75º

r
= (satisfied)
4 sin²67º – cot²67º 3 tan 45º .tan 20º.tan 40º.tan 50º .tan.70º

Sni
7 =2cosec²23º cot²(90º–23º) – 5
97. (c) cosec θ – cot θ = .....(i) sin²23º – sin²(90º–23º) – cot² 67º

gv.i
2 2sin68º 2cot15º
=2co se c² 23º tan ²23º – = – –
 cosec² θ – cot² θ = 1 (sin²23º + cos²23º) – cot²67º cos(90º –680 ) 5tan(90º –15º )

ridna
 [cosec θ – cot θ ](cosec θ + cot θ ) = 1 2 3  1.(tan 20º .tan 70º )(tan 40º.tan 50º )
= – 1 – cot ²67º 5
 (cosec θ + cot θ ) 2
cos 23º
1 =2sec²23º – 1 – cot² (90 – 23º) 2sin 68º 2 cot15º

eeYa
= = –
=2sec²23º – 1 – tan²23º sin 68º 5 cot15º
 cosecθ – cotθ 
=2sec²23º – (1 + tan²23º) 3 1 11
2 =2sec²23º– sec²23º = sec²23º –
 cosec θ + cot θ = .. (ii) 5
7
geisnh
2 (tanA.tanB = 1 if A + B = 90º)
Adding both equations, x  y 
101. (c)cos² θ = 2 3
4xy =2– – = 1
7 2
 2cosec θ =  5 5
2 7  max. value of cos² θ = 1
Enak

104. (b) sin7x = cos 11x


2
49  4 53 x  y   sin7x = sin(90º – 11x)
 =  1=
14 14 4xy  7x = 90º – 11x
R

53  7x +11x = 90º
cosec θ =  4xy = (x + y)2
  18x = 90º
28
aryn

 4xy = x² + y² + 2xy
98. (a) x sin45º = y cosec30º  x = 5º
 0 = x² + y² – 2xy = tan9x + cot9x
Les B

x cosec30º  0 = (x – y)² = tan45º + cot45º


 = =1 +1=2
y sin 45º  0 =x–y  x = y
102. (a)(1–sin ²  ) (1–cos²  ) × 105. (a)tan²= 1+2 tan² β
x 2
wa. th

2 2

y = 1/ 2 = 1 (1+cot² β ) (1+tan² β )  sec² –1 = 1+ 2 (sec² β – 1)

4
=(cos²) (sin²) (cosec² β )  sec²–1 = 2 sec² β – 1
x4 2 2 64
4 = 
(sec² β ) 2
  1  = 1 1
wwM

y    =
put = β = 45º, cos ² cos ²β
= 43
=cos²45º. sin²45º. cosec²45º. sec²45º  2cos² = cos²
99. (c)  5tan θ = 4
1 1  2 cos = cos β
4 = . . 2. 2 = 1
 tan θ = 2 2
5  2 cos – cos β = 0
Alternate:- Alternate:-
5 sin θ – 3cosθ
 + β = 90ºor  =90 – β tan²= 1+2 tan² β
5 sin θ  2 cos θ
(1–sin²)(1–cos²)× Put β = 45°
Di vi de n umerator and
denominator by cos θ (1+cot² β ) (1+tan² β )  tan² = 1+ 2. tan²45°

Rakesh Yadav Readers Publication Pvt. Ltd. 208

For More Visit : www.LearnEngineering.in


For More Visit : www.LearnEngineering.in

 tan² = 3  sec θ –tan θ = 5 –2 ....(ii) =sec² θ (cos²  +sin²  ) – tan² θ


 tan = 3   (  sin²+cos² = 1)
 1  5 – 2 = 5 – 2 = 5 – 2
  = 60° 
 5 2 5 –2 5–4 

= sec² θ –tan² θ = 1
Put= 60°, β = 45° add eq (i) + (ii)
sec θ+tanθ 5
2sec θ = 2 + 5 + 5 –2 111. (a) =
2 cos– cos β sec θ – tan θ 3
 2sec θ = 2 5
= 2 cos60°– cos45°  3sec θ + 3tan θ = 5sec θ – 5tan θ
 sec θ = 5
1 1 sec θ
= 2× – 1  8tan θ = 2sec θ  =4
2 2  cos θ = tan θ

ir
5

v.iSn
1 1  sin² θ + cos² θ = 1 1 cos θ 1
= – =0   = 4  sin θ =
2 2 sin² θ = 1–cos² θ cos θ sin θ 4
106. (a) tan θ + cot θ = 2 112. (a) Let,
 1 2

dnag
Put θ = 45º  sin² θ = 1 –  
 (1+sin) (1+sinβ ) (1+sin γ ) = (1–
 5
1 + 1 = 2 (matched) sin )(1–sinβ ) (1–sin γ ) = x
4
So, θ = 45º  sin² θ =
5  x.x=(1+sin )(1–sin )(1+sinβ)
 tan10045º + cot10045º

eYari
 1100 + 1100 = 2 2 (1–sinβ) (1 + sin γ ) (1– sin γ )
 sin θ =
5 x² = (1–sin²)(1–sin² β )(1–
tan θ cot θ
107. (d) +
1 – cot θ 1 – tan θ  sin θ + cos θ sin² γ )

tan θ
1
tan θ
=
2
5
snhe
+
1
5
=
3
5
x² = cos². cos² β . cos² γ
x =  cos.cos β .cos γ
= +
kgei
1 1 – tan θ 109. (b) (1+cot θ – cosec θ )
1– 113. (d)
tanθ (1 + tan θ + sec θ )
Put, θ = 45º 1 3
  2sin ²θ
ERna

tan ²θ 1 1  cot ²θ 1  tan ²θ


= tan θ – 1  tan θ 1 – tan θ  =(1 + cot45º – cosec45º)
(1 + tan45º + sec45º)
1 3
= (1 + 1 – 2 ) (1 + 1 + 2) =   2sin ²θ
tan ²θ 1 cosec²θ sec ²θ
= tan θ – 1 – tan θ tan θ–1
aBryn

  = (2 – 2 ) (2 + 2) =sin² θ + 3cos² θ + 2sin² θ


tan ³θ –1 =[2² – ( 2 )²] =3(sin² θ + cos² θ )
= tan θ(tanθ – 1) [  (a – b) (a + b) = a² – b²] =3 × (1) = 3
=4–2= 2 (  sin²+cos²=1)
Les

=
 tan θ –1 tan ²θ  tan θ  1 110. (a) x = a sec θ cos  Alternate:-
wa. th

tan θ  tan θ – 1
y = b sec θ sin  1 3
[  a³ – b³ = (a – b) (a² + b² +ab)] + + 2sin² θ
1  cot2 1  tan2
z = c tan θ
tan ²θ + tanθ  1 Put θ = 45º
=
wwM

tan θ x
 = secq.cos  1 3 1
= tan θ + cot θ + 1 a  + +2×
11 1 1 2
108. (a) y
 = secq.sin  1 3
sec θ + tan θ = 2 + 5 ..(i) b  + +1
2 2
 sec² θ – tan² θ = 1 z
 (sec θ – tan θ ) (sec θ + tan θ ) = 1  = tanq 1 3  2 6
c  = =3
1 2 2
 sec θ – tan θ = x ² y² z ² 114. (a)
2 5   –
a ² b² c ²
4 1
1   3 sin ² 
 sec θ – tan θ = sec² θ .cos²  + sec² θ . sin²  – tan² θ 1 ta n ²  1 co t ² 
5 2

Rakesh Yadav Readers Publication Pvt. Ltd. 209

For More Visit : www.LearnEngineering.in


For More Visit : www.LearnEngineering.in

2 2 2
4 1 sin A . cos A (sin A  cos A )
 1  1 
=  3sin ² 1
2(sin²+cos²) =   + m²
sec ² cosec² (sin A  cos A ) ×

 
2 
cos ² A sin ² A 

= 4cos² + sin²+ 3sin²
1
= 4(cos²+ sin²)  2=  m²
 (sinA + cosA)² + (sinA – cosA)²  4
= 4× (1) = 4 =  (sinA – cosA) (sinA + cosA) 
  1 7
(  sin² +cos² = 1)
 m² = 2 – =
Alternate:- × sin²A – cos²A  4 4
= 2(sin²A + cos²A) = 2
4 1 7
+ + 3sin² 1 1 m=
1  tan  1  cot2
2
– 2
119. (b)

r
Put  = 45º cosecθ – cotθ sin θ
7

Sni
4 1 1  sin+cos =
 + +3× cosec²θ – cot²θ 2

gv.i
1 1 11 2 = – cosecθ
cosecθ – cotθ sinA sinA
1 3 122. (a) +
 2+ + [  cosec² θ –cot² θ = 1 ] 1+ cosA 1– cosA
2 2

ridna
= cosec θ + cot θ – cosec θ = sinA 1– cosA +sinA 1+ cosA
4 1  3 8
 = =4 = cot θ 1+ cosA1– cosA
2 2
120. (d) cos θ +sin θ = 2 cos θ

eeYa
115. (d) 3(sinx + cosx) 4 + 6 = sinA–sinAcosA+sinA+sinAcosA
(sin x + cos x)² + 4 (sin6 x + cos6 x) Squaring both sides, 1 – cos ²A
Put, x = 90º  cos² θ + sin² θ + 2cos θ sin θ 2sin A
3(sin90º + cos 90º)4 + 6 = = 2 cosecA
= 2cos² θ
geisnh sin ²A
(sin 90º + cos 90º)²+4
(sin6 90º + cos 690º)  2cos² θ – cos² θ – sin² θ 123. (c) tan θ – cot θ = 0
= 3(1 + 0)4 + 6(1 + 0)2 + 4 (16 + 0) = 2 cos θ sin θ Put θ = 45º
= 3 + 6 + 4 = 13  cos² θ – sin² θ tan45º – cot45º = 0
Enak

116. (c) = 2sin θ .cos θ 1–1= 0


0 = 0 (matched)
 1  sin θ cos θ 
 (cos θ – sin θ )(cos θ +sin θ )
sec θ  cos θ  1  sin θ  – 2tan ²θ
  So, θ = 45°
R

= 2sin θ .cos θ
Take, θ = 0º sin θ + cos θ
 (cos θ –sin θ )( 2 cos θ ) = sin45º + cos45º
aryn

1  sin 0º cos 0º 
= sec0º×    – 2 tan ²0º
 cos 0º 1  sin 0º  = 2sin θ .cos θ 1 1
=  =
Les B

2
1  0 1  2sin θ.cosθ 2 2
= 1×   – 20 = 2
 1  1 0  cos θ – sin θ =
  2 cos θ 124. (a) 3sin θ + 5cos θ = 5
117. (a) x sin+ y cos = 4 = 2 sin θ 5sin θ – 3cos θ = x
wa. th

Squaring both sides, Alternate:- and squaring and adding


both sides,
x²sin ² + y²cos² + 2x y cos +sin = 2 cos 
5² (sin ² +cos² ) + 3²
sin.cos = 16 ....(i) cos – sin = x
2 (sin²+ cos²) = x² + 5²
2(sin²+cos²) = 2 cos² +x ²
wwM

x cos– y sin = 2
 5² + 3² = x² + 5²
again squaring both sides, 2 – 2cos² θ = x²  x² = 9
x² cos²+ y² sin² – 2xy 2(1– cos² θ ) = x²  x = 3
sin.cos = 4 .....(ii) 2sin² θ = x² 125. (c) x sin60º.tan30º
on adding eqn. (i) and (ii), x= = sec60º.cot45º
2 sin θ
(x² + y²) (sin²+ cos²) 3 1
1  x. . = 2. 1
= 16 + 4 121. (c) sin θ – cos θ = ....(i) 2 3
2
x² + y² = 20
sin θ + cos θ = m .....(ii) x
cos
2
A . sin
2
A(sin A  cos A )  = 2
on squaring and adding both 2
118. (c) (sin A  cos A ) + sides,  x= 4

Rakesh Yadav Readers Publication Pvt. Ltd. 210

For More Visit : www.LearnEngineering.in


For More Visit : www.LearnEngineering.in

126. (d) θ = 60º (  1 + tan²= sec²)


12
+1
1 1 2 2
2(sin  cos ) tan θ+1 5 12  5 17
= 1  sin θ + 1 – sin θ sec² θ =  = = =
2 2  (sin cos)2 tan θ –1 12 12 – 5 7
–1
5
1 1 1 2
= 1  sin 60º + 1 – sin 60º 
2 2  cos2 (sin cos)2 cos
132. (c) =a
cos β
1  3 1  3
= 1    + 1 –   1  2
2   = cos = a cos β
 2  2  2  cos sin cos
On squaring both sides,
1 sin + cos =  2 cos θ
 

ir

= 2 3  2– 3  cos² = a² cos² β

v.iSn
2 2 129. (a) 7sin² θ + 3cos² θ = 4
1 1  7sin² θ + 3(1–sin² θ ) = 4  1 – sin² = a² (1 – sin² β ) ....(i)
=
2 2

2  42 3  4 –2 3
  7sin² θ + 3 – 3sin² θ = 4 Again, sin = b sin β

dnag
1 2 2   4sin² θ = 4 – 3 Squaring both sides
= 
4
 3 1    3 –1 

 1  sin² = b² sin² β
 sin² θ =
4 put the value of sin²  in
1
 

eYari
= 3 1  3 –1 1 equation (i)
4  sin θ =
2  1 – b² sin² β = a² (1– sin² β )
θ
= 2 3 = 3 = cos30º = cos  θ = 30º  1 – b² sin² β = a² – a²sin² β
4 2 2

127.(b)
2tan ²30º
1 – tan²30º
sec ²45 – sec ²0º
snhe
 tan = tan30º =

p
1
3


a²sin² β – b²sin² β = a² –1
sin² β (a² – b²) = a² – 1
kgei
= xsec60º 130. (d) tan9º =
q a² – 1
2  sin² β =
 1  a ² – b²
2  sec ²81º sec ²81º
= 133. (b)
ERna

 3 2

 1 
2
+  2 –1 1  cot ²81º cosec²81º
(  1 + cot² = cosec²) cos ²60º 4 sec ²30º – tan ²45º
1– 
 3 sin ²30º  cos ²30º
1
= sin² 810
aBryn

= x×2 cos ²81º 1 


2
 2 2 2
1 =tan²81º = tan²(90º – 9º) 
    4 
  –1

2× =  2   3 
 3 + 2 – 1 = 2x 1 q² 1
1 =cot² 9º = =
1– tan² 9º p² (  sin²A + cos²A = 1)
Les

3
131. (d) If sec θ +tan θ = 5 .....(i)
1 4 4
wa. th

1 16
2 3  sec² θ –tan² θ = 1 =  –1=  –1
     2 – 1 = 2x 4 3 4 3
3 2  (sec θ +tan θ )(sec θ –tan θ ) = 1
 2=x×2 3  64 – 12 55
1 = =
 x= 1 (sec θ –tan θ ) = ....(ii) 12 12
wwM

5
s in  – c o s  5
128. (b)tan θ = s in   c o s  subtracting eq. (ii) from (i)
134. (d)cosx = x² – x +
 (sec θ + tan θ ) – (sec θ – tan θ ) 4
Squaring both sides and after
that adding '1' both sides, 1 1 1 1 5
=5– = x² – 2 × x × + – +
5 2 4 4 4
(sin cos)2
 1 + tan² θ = 1  2
(sin cos)2  1
25 – 1 24 =  x –  + 1 > 1
 2 tan θ = =  2
 sec² θ 5 5
2 2 = – 1  cosx  1
sin cos  sin – cos tan θ =
12
 so value of x is none of the
= 2
5
sin cos above

Rakesh Yadav Readers Publication Pvt. Ltd. 211

For More Visit : www.LearnEngineering.in


For More Visit : www.LearnEngineering.in

1 1 1 2 5 1
135. (d) 1 + – sec²27°    2 × =1
cot ²63 5 2 2 5 2
x 143. (d) We know that, in a cyclic
1 138.(b) sin = x² – 2x +2 quadrilateral sum of opposite
+ – cosec²27° 2
sin ²63 angle is 180°
put value of x from options
=1 + tan²63° – sec²27° + cosec²  A + C = 180° .......(i)
x=1
63° – cosec²27° and B + D = 180° .......(ii)
= 1 + cot²27° – sec²27° +   cosA + cosB + cosC + cosD
sin ×1 = 1² –2×1 + 2
sec²27°– cosec²27° 2 = cosA + cosB + cos(180° – A) +
=1 + cot²27° – cosec²27° sin90° = 1 – 2 + 2 cos(180° – B)

r
=1–1=0 1 = 1 (satisfied) From Eqs. (i) and (ii),

Sni
= cosA + cosB – cosA – cosB = 0
cos sin 43 cos19
136. (b) x  139. (a)  – 8 cos² 60° 144. (a) Given,  +  = 90° ....(i)

gv.i
1 – sin cos 47 sin 71
By given condition,
cos1  sin  1 
2
 8  
   2
1 – sin1  sin =1+1–      =

ridna
 2  
  3
cos1  sin (If A + B = 90°, then sinA = cosB)
2 2
=   = = (90° –  )
1 – sin ² SinA cos B

eeYa
3 3
= 1 or =1
CosB sin A {from Eq. (i)}
cos1  sin
= =2–2=0
cos ² 2
1 1   = 60° –    = 36°
3
1  sin
geisnh
140. (a) Sin² 7 + sin² 82
= 2 2
cos sin  cos 
0 145. (b)Given,  2
cos 1 1  1  cos  sin 
 = sin² 7 + sin²  90º –7
1  sin x 2  2 
 sin2  + cos2  = 2sin  cos 
Enak

Alternate:-
1 1  sin2  = 1 = sin90°
cos =sin² 7 + cos² 7
x = 2 2  2  = 90º   = 45°
R

1 – sin = 1 (  sin² q + cos² q = 1) 146. (b)Given, sin3  = cos(  – 2°)


put q = 0° 141. (c) Given that, 3sinx + 5cosx = 5
 sin3  = sin[90° – (  – 2°)]
aryn

cos 0 1 On squaring both sides, we get


x= = 9sin x + 25cos2 x + 30sinxcosx = 25
2  3  = 90° –  + 2°
1 – sin 0 1– 0
Les B

 9(1 – cos2x) + 25(1 – sin2x) + 92


 x=1 30sinxcosx = 25  4  = 92°   = = 23°
4
cos cos 0 1  9 + 25 – (9cos2x + 25sin2x – 30sinx
=  
cosx) = 25 sin6  – cos 6 
1  sin 1  sin 0 1  0
wa. th

147. (b)
 9 = (3cosx – 5sinx)2 sin2  – cos 2 
= 1
n o w c h e ck o p ti o n b y  3cosx – 5sinx = 3 3 3

putting x = 1 only option =


sin  – cos 
2 2

142. (a) If  +  = 90º


sin2  – cos2 
wwM

(b) satisfying.
Put  = 45º
137. (b) sin
2
 – cos2 sin4   cos 4   sin2  cos 2 
= 45º =
sin2  – cos 2 
5 –1
sin cos   2 = sin4  + cos4  + 2sin2  .cos2
coseccosec ×  sin  cos   – sin2  cos2 
= (sin2  + cos2  )2 – sin2  cos2 
–1
1 1  2
= 1 – sin2  cos2 
 2 2
AC = 2²  1²  5  2 2  
 1

1


sinA + cotC

 2

2 1 – cos B
148. (c) tanA =
sinB
BC BC –
1
  2 × (2) 2 Put A = 30º, B = 60º
AC AB

Rakesh Yadav Readers Publication Pvt. Ltd. 212

For More Visit : www.LearnEngineering.in


For More Visit : www.LearnEngineering.in

1 – cos 60º 1  sin  1 sin  ( sec2  – tan2  = 1)


tan 30º = = = + = sec2  (cos2  +1)
sin60º cos  cos  cos 
= sec  + tan  = sec2  .cos2  + sec2 
1 = 1 + sec2  >1 + 1 > 2
1– a2  b2 – C 2
1 2 153. (b) cos  =
= 2ab ( sec2  >1 for 0 <  <90°)
 3
3 (By cosine rule) 157. (d) We know in the interval  
2
62  22 – C 2 40 – c 2  
1 1 = = 0, 2  , sec2  is increasing from
 = (Satisfy) 262 24
3 3 For acute angle, 1 to   p > 1

ir
2tan A 2tan30º 40 – c 2 158. (a) In  ABC,

v.iSn
 2 = cos  > 0  > 0  C2 < 40
1 – tan A 1 – tan2 30º 24
A
1 2  0 < c < 2 10
2
3 3 (since, C cannot be negative) .......(i) 60°

dnag
= = 2 = 3
 1 2 Also, b + c > ac > 6 – 2  c > 4 5cm
1– 
 

 3 3
Now check the option,
From Eqs. (i) and (ii), c  4,2 10  90° 30°
B C
154. (d) We know that, the value of cos

eYari
Option (c):- tanB
 is decreasing from 0º to 90°. AB 1 5
tan60º = 3
 cos1° > cos89° cos60° =  =
AC 2 AC
149. (a)  p>q
 AC = 10cm
A

o
r
40°

R
B
snhe
Also, cos 1° is close to 1 and
cos 89° is close to 0.
Hence, option (d) is correct.
159. (a) Given, cos  + 3 sin  = 2

1 3
kgei
155. (d) C  cos  + sin  = 1
2 2
In  OAB, sin30°cos  + cos30°sin  = 1
2 
3
ERna

AB r  sin(30°+  ) = sin90°
cos40° =  cos40° =
OB R q  30°+  = 90°
A 1 B
 r = R cos40°

So, the radius of the circle of Given, 7 cos2  + 3sin2  = 4   = 60° =
aBryn

latitude 40° S is R cos 40°. 3


 7(1 – sin2  ) + 3(sin2  ) = 4
150. (a) We know that, If value of cos  160. (d) In  ABC,
 7 – 4 sin2  = 4
increases, then the value of A
 4sin2  = 3
 decreases.
Les

1 3
 sin  = +
 cos  > 2 v w
wa. th

 
For, 0 <  <
 cos  > cos60º   < 2
3 C u B
wwM

151. (b) Given, sin  + cos  = 1 3 BC u


sin  = ,  = 60º tanA = =
On squaring both sides, we get 2 AC v
(sin2  + cos2  ) + 2sin  cos  = 1
3 v
 1 + 2 sin  cos  = 1  tan  = = 3 andtanB =
1 u
 sin  .cos  = 0 156. (b) [(1 – sin2  ) sec2  + tan2  ] Also, u2 + v2 = w2 .....(i)
1  sin  1  sin 1  sin  (cos²  + 1) (by Pythagoras theorem)
152. (b) = 1 – sin 1  sin 
1 – sin  ( sin2  + cos2  = 1) u v u2  v 2
 tanA + tanB = + =
= [cos2  .sec2  + tan2  ](cos2  + 1) v u uv
1  sin 2 2
1  sin  ( cos²  .sec²  =1)
= = w2
1 – sin 
2
cos2  = (1 + tan2  ) (cos2 + 1)
=
uv
[from Eq. (i)]

Rakesh Yadav Readers Publication Pvt. Ltd. 213

For More Visit : www.LearnEngineering.in


For More Visit : www.LearnEngineering.in

161. (b) Given, 3 1


 3 2 = 1
C =  2  × 3– 2
   2 + 3 3
1
1  2 cos4q – sin4q =
1k
×   × 1 2–
2  3 3
(  (a² – b²)(a²+b²) = a4 – b4)
9 3 –35
= – 4 + –3 = 1
A B 8 2 8 172. (c) tanq =
3k 11
p
k 166. (c) tan θ = cos ec ² – sec ²
q
tan B =
3k cos ec ² sec ²

r
p sec θ – q cosecθ
=

Sni
In  ABC, p sec θ  q cosecθ 1 1 cos ² – sin ²

AB2 + AC2 = BC2 ......(i) sin ² cos ²  sin ².cos ²

gv.i
 p sec θ =
(by Pythagoras theorem)  1 1 cos ² sin ²
cosecθ 
 – q
 
2  cosecθ  sin ² cos ² sin ².cos ²
  3k  + (1k)2 = BC2 =  p sec θ
cosecθ  q

ridna
   sin ²
 BC2 = 4k2  BC = 2k  cosec  cos ²
1 – 

cos ² – sin ²  cos ²
162. (a) Given,  
p tan– q cos ² sin ²  sin ²
= p tan q cos ²
1  

eeYa
x –1  cos ²
sin  
2x
p 1
In ABC, using Pythagoras theo- p–q 1–
q 1 – tan ²
rem,
=   11  5
AC2 = AB2 +BC2 p 1  tan ² 1 6
geisnh
p  q 1
C q 11
p2 – q 2 3
2x
x–1
=
p2  q 2 173. (b) sinq=
5
Enak

 167. (b) cosec 2  – 2 + s in 2 


A B = (sin  –cosec  )2
2
 2x = AB + (x – 1) Hence, it is always non-negative.
R

 AB2 = x + 1  AB = x  1 168. (b) 1 – 2sin²q + sin4q


 (1 – sin²q)²
aryn

BC x –1 x –1  ( cos²q)²
 tan  = = =
Les B

AB x 1 x 1  cos4q 3 P
sinq= =
163. (c) Given, 169. (c) Sinq = 0.7 5 H
cos  cos   sin²q + cos²q = 1 So, B = 4
– 2 P=3
1 – sin  1  sin   (0.7)² + cos²q = 1
wa. th

H=5
cos   sin  cos  – cos   cos  sin   0.49 + cos²q = 1
 2 P B
1 – sin2   cos²q = 1 – 0.49 
tan cos B H
( 1–sin²  =cos²  )  
 cosq = 0.51 cot cos ec B H
wwM

2sin  cos  = 2cos2  


 170. (c) S i n ² 6 5° + si n ² 25° + P P
 sin  = cos  cos²35° + cos²55°
3 4 15  16 31
 tan  = 1 = sin²65°+ sin²(90° – 65°) +  
4 5  20 20
[cos²35° +cos²(90°– 35°)]  4 5 45 9
 
 = 4 = ( sin² 65° + co s²65°) 3 3 3 3
+ (cos²35° + sin²35°)
164. (b) As we know that, sinx is in- = 1+1=2 31
creasing from 0º to 90°. =
1 60
 siny > sinx. 171. (a)cos²q – sin²q =
3
(given) 174. (b) (sina + coseca )2 + (cosa +
165. (b) sin360°. cot 30° – 2sec2 45° + seca )² = k +tan²a + cot²a
3cos60°. tan²45°– tan²60° cos²q + sin²q = 1 (property)
(cos²q – sin²q) ( cos²q + sin²q) put a = 45°,

Rakesh Yadav Readers Publication Pvt. Ltd. 214

For More Visit : www.LearnEngineering.in


For More Visit : www.LearnEngineering.in

 (Sin45° +cosec45°)² + (cos45° In ABD ,


3
+ sec45°)² = K+ tan²45° + sina = = 0.6
5 BD AD
cot²45°   (By sin rule)
177. (d) 7 sina = 24 cosa sin BAD sin60
2 2
 1   1 
   2 
   2 sin 24 BD AD

 2   2

 =  sin BAD 
cos 7 3
=k+1+1 2
24  P
1  1  1  1   tana =
 2
2 2   2  2 
 2 2   7B
2  2   2
 3 BD
=k+2  AD = . ........(i)
2 sinBAD

ir
1 1 from ADC

v.iSn
 4 4 =k +2  k= 7
2 2
 CD AD
175. (a)   AD
sin DAC sin45

dnag
14 tana – 75cosa – 7seca 1 CD
= . ....(ii)
24 7 25 2 sinDAC
y² – x² = 14 × – 75× –7×
7 25 7 from equation (i) and (ii)

eYari
x = 48 – 21 – 25 = 2
In ABC , sin21°= 3 BD 1 CD
y .  .
178. (c) 2 sin BAD 2 sin DAC
AB = x
AC = y
BC  y ² – x ²
snhe 
sin BAD
sin DAC

3
2
 2
1
3
= (tan²a + 1)sin²b
kgei
 sec21° – sin69° 1 1
= 
=(tan²45°+ 1)sin²45° 3 2 6
AC BC
 – 2 181. (a) sin3A = cos(A – 26°)
BC AC  1 
 =2× 1 =1
ERna

= (1 + 1)    sin3A = sin[90º – (A–26º)]


2  2 2
2 2
 3A = 90º – A + 26º

AC  – BC 
=
y² –  y² – x²
 4A = 116°
BC AC  y y² – x ² sec– 1 1
aBryn

179. (c) = cos – 1  A = 29°


sec 1
y² – y ²  x ² x² 1
= 1 sin ²– 2sin4
 y y² – x² y y² – x² cos 182. (a) sec²q –
2cos 4– cos ²
176. (c) seca + tana = 2 ............(i)
Les

2 sin ²1 – 2sin ²


1 1 – cos 1 – cos = sec²q –
wa. th

seca – tana = ...........(ii) = = cos ²2cos ² – 1


2 1  cos sin ²
On adding equation (i) and (ii)
1 – cos 1 cos cos²– sin² 2cos²–1  1– 2sin²
=  – = sec²q – tan²q = 1
1 sin sin sin
wwM

2seca = 2  Alternate:-
2 = cosecq – cotq
180. (c) sin2– 2sin4
5 H A sec² θ –
seca = 2cos 4– cos 2
4 B
Put θ = 0º
sin2  – 2sin4 
sec² 0º –
2cos4  – cos2 
60° 45°
B 0–0
 1 D 3 C =1–
2 –1
 
B  = 60°, C  = 45° =1–0=1
3 4

Rakesh Yadav Readers Publication Pvt. Ltd. 215

For More Visit : www.LearnEngineering.in


For More Visit : www.LearnEngineering.in

183. (c) x =a(sinq + cosq) 189. (a) ( r co s q –


1  sin 1 – sin 3 )² +
y = b(sinq – cosq) =
cos ( rsinq – 1)² = 0
x
 = (sinq + cosq) 2 (r cosq – 3 )² = 0, ( rsinq – 1)² =
a = = 2sec
cos 0r cosq = ......(i)
3
x2 Alternate:-
 r sinq = 1 ......(ii)
a2 put q = 30°,
squaring and adding equation
= ( sinq + cosq)² ....(i) 1  sin 30 1 – sin 30 (i) and (ii)
= +
y 1 – sin 30 1  sin 30 r²cos²q + r² sin²q = 3 + 1
 = (sinq – cosq)
b r²(cos²q + sin²q )= 4

r
y2 r² = 4

Sni
= (sinq – cosq)² ....(ii) 1 1
 1  1 – r=2
b2 2 2

gv.i

On adding equation (i) and (ii) = 1 1 r sin 1
1 – 1  tanq =  and rcosq
x² y² 2 2 r cos 3
 

ridna
a ² b² = 3
3 1 4
= (sinq + cosq)2 + (sinq – cosq)² =  = r
1 3 3 3
cosq = secq =
x² y² Now check with option by r 3

eeYa
 
a ² b² putting q = 30°, r r

= sin²q + cos²q +2sinq cosq + r tan sec
2 2 4  3 3
sin²q + cos²q – 2sinq. cosq 2 sec30° =  r sec tan r ² 1
3 3 
x² y²
geisnh 3 3
 + = 2(sin²+ cos²) 187. (d) sin(4a – b) = 1= sin90°
a ² b²  2 
r 
=2×1=2 1  
 3   2r  2  2  4
cos(2a + b) = = cos60°
184. (d) sin5q = cos20° 2 = r 1
2
r ²  1 2²  1 5
Enak

 5q + 20° = 90° 4a – b = 90° 3


( If sinA = cosB then A + B = 90°) 2a + b = 60° Alternate:-
 5q = 70° adding 6a = 150° r=2
R

 q = 14° a = 25° r sin 1


185. (c)2 secq = 3 cosec²q  b = 10° tan = 
r cos
aryn

3
2 3 3  sin(a +2b)
   = 30°
Les B

cos sin ² 1 – cos ²  sin(25°+ 2× 10°)


2 tan 30 sec 30
 2 – 2 cos²q = 3 cosq 1 =
2sec 30 tan 30
 2cos²q + 3cosq – 2 = 0  sin45° =
2
 2cos² q + 4cosq – cosq – 2 = 0 1 2
wa. th

188. (c) 2 
 2cosq (cosq+ 2) – 1( cosq + 2) = 0 3 3 4
= 2 1 = 5
 (2cosq – 1) ( cosq + 2) = 0 2 
 2cosq – 1= 0 or cosq + 2  0 3 3
wwM

1  sin
 cosq = = cos60° or cos 190. (c) = b  sina = b sinb
2 3 sin
cosecq = 3
 cos cos ²
 q= 1 P
3 =a  = a²
sinq = cos cos²
3 H
1  sin 1 – sin
186. (d)   cotq – cosecq 1 – sin ²
1 – sin 1  sin   a²
1 – sin ²
2 3
2 2  –
 1 –sin²a = a² (1 – sin²b)
=  1  sin   1 – sin 1 1
 1 – b² sin²b = a² – a² sin²b
1 – sin ²  2– 3
[ value put in sina]

Rakesh Yadav Readers Publication Pvt. Ltd. 216

For More Visit : www.LearnEngineering.in


For More Visit : www.LearnEngineering.in

 1 –a² = b² sin²b– a² sin²b 193. (c) tan(A + B) = 3 = tan60° 196. (a) Given, (sin  + cosec  ) = 2.5
 1– a² = (b² – a²) sin²b  1 
1 5
1 – a² a² – 1 tan(A – B) = = tan30°   sin   sin   =
2
 sin²b =  3
b² – a ² a ² – b²  2sin2  – 5 sin  + 2 = 0
 (A + B) = 60° ......(i)
191. (c) 3 tanq = 3 sinq (A – B) = 30° .....(ii)  2sin2  – 4sin  – sin  + 2 = 0
Adding both equation  2sin  (sin  – 2) –1 (sin  – 2) = 0
sin
3 cos = 3sinq 2A = 90°  (2sin  – 1) (sin  –2) = 0
90  1
3 A=  sin  = (  sin   2)
=3 2 2

ir
cos
  = 30°

v.iSn
A = 45°
3 B = 15° Alternate:-.
cosq = sin θ + cosec θ = 2.5
3 sin – 2sin ³
194. (c) Put θ = 30º

dnag
then perpendicular = 6 2 cos ³ – cos
sin30º + cosec30º = 2.5
sin1 – 2sin ²
 cos 2 cos ²– 1 1
   + 2 = 2.5
2

eYari
6 sin 1 – 2 1 – cos ²

 
 2.5 = 2.5
 θ = 30º
(Satisfy)

cos 2 cos ² – 1
197. (d)Given, x cos60° + y cos0° = 3
3
(sin²q – cos²q)
 snhe
tan1  2cos ² – 2
2cos ²– 1 
x
2
+y=3

 x + 2y = 6 ......(i)
kgei
2
 P  B 
2
tan2cos ²– 1
   –     and 4x sin30° – ycot 45° = 2
H  H  2cos ²– 1
1
 tanq
ERna

 6  3
2 2
 4x × – y.1 = 2
    2
  3  – 3  Alternate:-
     2x – y = 2 ......(ii)
sin – 2sin ³ On solving Eqs. (i) and (ii),
6 3 1 2 cos ³ – cos
aBryn

 – = we get x = y = 2
9 9 3 198. (a) log(tan1°) + log (tan2°) + ....
sin1 – 2sin ²
192. (d)  cos 2 cos ²– 1 ...+ log (tan89°)
  = log (tan 1° tan2° ...tan 45°
(1 – 2sin²q = 2 cos²q – 1 = cos2q ..tan 88° tan 89°)
= cos²q – sin²q)
Les

[  tan89° = tan(90° – 1) = cot1°]


 tanq
wa. th

= log[tan1°cot1°) (tan2°cot2°)
2xy ....tan ....tan45°]
195. (c) Given, cot  = x 2 – y 2
= log (1°.1°...1°) = 0
AB = 20 cm C 199. (c) Now, (sinx – cosx)2 = (sin2x +
BC : CA = ?
wwM

cos2x) –2sinx cosx


BC
 = cosC 2
1 
CA x 2 –y = 1 – 2  
2
BC 
 = cos30°  1 
CA A 2xy B  sin x cos x  ,given  = 0
2
( C  180 – 90 – 60  30 ) In  ABC,
Alternate:-
AC2 = (x2 – y2) + (2xy)2
BC 3  AC = (x2 + y2)²  AC = x2 + y2
2
1
 = sinx cosx =
CA 2 2
AB 2xy
 3 :2  cos  = AC = x 2  y 2 Put x = 45º

Rakesh Yadav Readers Publication Pvt. Ltd. 217

For More Visit : www.LearnEngineering.in


For More Visit : www.LearnEngineering.in

Alternate:- 206. (d) Given, sinx + sin y = a and


1
sin45º cos45º = cosx + cos y = b
2 cos x sin x
 n and m on squaring both sides, we get
cos y sin y
1 1 1 sin2x + sin2y + 2sinx sin y = a2...... (i)
 × = Put x = y = 45º cos2x + cos2y + 2cos x cosy = b2...(ii)
2 2 2
cos 45º sin 45º On adding Eqs. (i) and (ii), we get
1 1  n and m
 = (Satisfy) cos 45º sin 45º (sin2x + cos2x) + (sin2y + cos2y) + 2(sinx
2 2 n = 1 m = 1 siny + cosx cosy) = a2 + b2
 sinx – cosx  (m²–n²)sin²y = (1²–1²)sin²45º  (sinx siny + cosx cosy)
sin45º – cos45º = 0
Now check from options a2  b2 – 2
1 1 =

r
– =0 Option (a):- 1 – n² 2
2 2

Sni
 1 – (1)² Alternate:-
200. (a) tan²y cosec²x –1 = tan²y
 0 (Satisfy) sinx + siny = a

gv.i
Put x = y = 45º
204. (b) Given, p = tan x + cot2x
2 cosx + cosy = b
tan²45º cosec245º–1 = tan²45º
= (tanx + cotx)2 – 2 Put x = y = 45º
2
sin45º + sin45º = a
1×  2 – 1 = (1)² 2

ridna
 sin2 x  cos ² x 
= –2
2– 1 = 1  sin x .cos x  a = 2
1 = 1 (Satisfy)
 2 
2
4 b = 2

eeYa
 x=y =   –2 = –2
sin2x  sin ²2x 1 1
x–y=0
Since, the maximum value of  sinx siny + cosx cosy =  =1
201. (c) Given, 2 2
sin 2x is 1.
cos x cos x Now check from option,
4
 2
geisnh
1  cosecx cosecx –1  pmin = –2=2
1 a 2  b2 – 2
Option: (d)
p > 2 2
2cos x cosecx
 2 Hence, p > 2
cosec 2x – 1 Alternate:- 2 2

 2    2 –2
Enak

cos x cosec x P = tan²x + cot²x 


 1 Put x = 45º 2
cot2 x
P = tan²45º + cot²45º
2 2 – 2
R

 P= 1+1 
 tanx = 1  x = P= 2 2
4
Put x = 30º
aryn

202. (c) Given, sinx : sin y = 3 : 1 2


P = tan²30º + cot²30º  = 1(Satisfy)
2
Les B

3 1  1 2 2 207. (a) Given, cosec4  – cot4  = 17


= : = sin60° : sin30°   3
2 2 P = 
 3
    (cosec2  – cot2  ) (cosec2  + cot2  ) = 17
 x : y = 60 : 30 P = 0.33 + 2.99
 1  cos 2  
x : y = 2 : 1 P = 3.32 1 = 17
wa. th

 sin2  
cos x  P2
203. (a) Given, cos y = n .....(i) 205. (b) ( cosec²  – cot²  =1)
2
5sin75ºsin77º 2cos13º cos15º  2 – sin  = 17sin2 
sin x
wwM

cos15ºsin77º 1
sin y = m ...... (ii)  18sin2  =2  sin2  =
7sin81 9
Now, (m2 – n2)sin2y –
cos 9 1
 sin2 x cos 2 x   sin  =
2
=  sin2 y – cos 2 y  sin y 5 cos15 sin 77  2sin77 cos15 3
=
cos15 sin77 (since,  lie in first quadrant)
1 – cos x  cos
2 2
y – cos2 x 1 – cos 2 y 

7cos9 1
= cos 9 208. (c) Given, x +   = 2cos 
cos2 y x
7cos15º.sin77º 7 cos 9º On squaring both sides, we get
cos2 y – cos2 x = –
=  1 – n 2 [from Eq. (i)] cos15º.sin77º cos 9º 1
cos2 y x2 + + 2 = 4cos2 
=7–7=0 x2

Rakesh Yadav Readers Publication Pvt. Ltd. 218

For More Visit : www.LearnEngineering.in


For More Visit : www.LearnEngineering.in

1 1 2 3
 x2 + = 2 (2cos2  – 1) 212. (c) Given, sin(x – y) = and  sin  = =
x2 2 x 5
= 2(2cos2  – sin2  – cos2  )
1 2  5 10
= 2cos2  – 2sin2  cos (x + y) =  = cm
2 3 3
1  sin(x – y) = sin30° 217. (c) Given, sinx – cosx = 0
209. (b) cot² θ –
sin ²θ and cos(x + y) = cos60°  sinx = cosx  tanx = 1
cos ²θ 1 cos ²θ –1  x – y = 30° and x + y = 60°  
= – =  tanx = tan  x=
sin²θ sin ²θ sin²θ  x = 45° and y = 15° 4 4
213. (b) Given, 1 + tan  = 2
– 1 – cos ²θ

ir
– sin ²θ  
= =  1  sin4x + cos4x = sin4 + cos4
 tan  = 2 –1

v.iSn
sin ²θ sin ²θ 4 4
( 1– cos² θ = sin² θ ) 1 4 4
 cot  –1 = –1  1   1  1 1 1
Alternate:- 2 –1 =  + = + =
 2   2  4 4 2

dnag
1 1 2 1
cot² θ – = × –1
s in 2 θ 2 –1 2 1 tan x tan x
218. (b) = –
1  sec x 1 – sec x
Put θ = 45º 2 1
= –1 = 2
tan x 1 – sec x – 1 – sec x 

eYari
1 2 –1
cot² 45º – =
2
sin 45º 214. (d) Given, sin (x + 54°) = cosx 1 – sec 2 x
 sin(x + 54°) = sin(90° – x) –2tan x sec x
1 ( 0° < x < 90°) =
 1– – tan2 x
 1 2


 2

 snhe
 x + 54° = 90° – x
 2x = 36°  x = 18°
215. ( a) B y us ing P ythagoras =
2
cos x = 2cosecx
 1 – 2 = –1 sin x
kgei
theorem
210. (c) Since, sinx = cosy cos x
A
As x and y are acute angles, Alternate:-
then
ERna

4 units 5 units tan x tan x


 –
x=y= 1  sec x 1 – sec x
4 Put x = 45º
C 15 cm B
aBryn

 AB²= AC²+BC² tan 45º tan 45º


 x+y= –
2 (5)²= (4)²+BC² 1  sec 45º 1 – sec 45º
BC = 3 units
3 units = 15 cm 1 1
m2 – n 2
 –
211. (c) Given, sin  = 2 1 2 1– 2
15
Les

m  n2 1 unit =
3 1 – 2 –1 – 2
wa. th

C
 2
15 1–
2 5 units =
3
×5 = 25 cm  2
n
m – n2

2 +
m  AB = 25 cm –2 2

2
wwM

–1
q 3 P
A B 216. (d) Since, tan  = =
4 B 2 2
In  ABC,
H= P  B = 9  16 = 25 = 5
2 2 Now check from option.
2 2
AB =  AC  – BC  Let the length of hypotenuse = x cm Option: (b)
C 2 cosec x = 2 × cosec45º
= m4 +n4 +2m2n2 – m4 +n4 –2m2n2 
= 2× 2 (Satisfy)
= 2
4m n  2mn 2 5 219. (b) (sin x – cos x + 1) cosec2x
4 4
3
= {(sin2x – cos2x) (sin2x + cos2x)
m2 – n 2
 tan  = q + 1} cosec2x
2mn A B
4 [ a² – b² = (a+b)(a–b)]

Rakesh Yadav Readers Publication Pvt. Ltd. 219

For More Visit : www.LearnEngineering.in


For More Visit : www.LearnEngineering.in

( 1 – cos2x = sin2x) p  sin 4  + cos 4  = sin 490° +


= {sin x – cos2x + 1) cosec2x
2
cosec30º = q cos490°
( 1 – cos2x =sin2x)  =1+0=1
p 228. (c) Given, 2sec2  + sec  – 6 = 0
1 2 = q
= 2sin2x. 2
sin2 x  2sec2  + 4sec  – 3sec  – 6 = 0
Alternate:- p = 2q  2sec  (sec  + 2) – 3(sec  + 2) = 0
(sin4x – cos4x + 1) cosec²x  p 2 – q 2 ×tan θ  (2sec  – 3) (sec  + 2) = 0
Put x = 90º (sec   – 2, sec  > 0)
(s in 49 0º – c os 4 9 0º + 1) = 4q 2 – q 2 × tan30º
cosec²90º 3
 sec  =

r
 (1 – 0 + 1) ×1 1 2
= 3q 2  =q
3

Sni
 2 2
224. (d) Given, 2x2cos 60° – 4cot245°  cos  =
3

gv.i
220. (b) cos x cosecy – cos x sin y – 2 tan60° = 0
( x + y = 90°, given) 1 4 5
 2x2 × – 4(1)2 –2× 3 = 0  sin  = 1 – cos 2  = 1 – =
2 9 3
= cos x cosec(90 – x ) – cos x.sin 90 – x 

ridna
=  x2 – 4 – 2 3 = 0 1 1 3
cos x .sec x – cos2 x  cosec  = = =
sin 5 5
 x2 = 4 + 2 3

eeYa
= 1 – cos 2 x = sin2 x = sinx 3
221. (b) We know that, 0  sin2x  1  x2 = 3 + 1 + 2 3
sin x 1 – cos x
 0  sin10x  1 2 229. (c) =
 x2 =  3 + (1)2 + 2 3 .1 1  cos x sin x
0  p  1
geisnh sin2x = (1 – cosx) (1 + cosx)
2 
( p = sin10x)  x2 =  3 1   (1 – cos2x) = (1 – cos2x)
 2.    which is possible for all values
222. (c)  cos  sin2  + 4cos2  x = 3 +1
8 8 4 of x except multiples of 180°.
Enak

1 Since for x = 180°, sinx = 0 and


  225. (d) We know that, sin 30° =
2 1 + cosx = 0
– sec + 5 tan2
3 3 Value of sin increases 0° to 90° 230. (a) 3sin  + 4cos  = 5
R

2  sin31° > sin30° and sin32° > sin30° 3cos  – 4sin  = x (Let)
 1  2
= 1+4 
 2 
–2+5  3 1 1 Using identity,
aryn

 sin31° > and sin 32° > 3² + 4² = 5² + x²


2 2
( sin2  + cos2  = 1) x=0
Les B

On adding both sides, we get


= 1 + 2 – 2 + 15 = 16 So,
1 1 3cos  – 4 sin  = 0
p sin31° + sin32°> +
223. (b)Given, cosec  = q 2 2
13
 sin31° + sin32° > 1 231. (c) Given, sec  =
wa. th

C 5
226. (c) We know that, sin  is increas-
ing in 0° to 90°. 169
p
 sec2  =
25
q 1
wwM

 sin30° = 169
2
  1 + tan2  =
25
A 2 B 1
p 2– q
 sin32°> 169
2  tan2  = –1
q 25
In  ABC, tan  = 227. (d) Given, sin  + cosec  = 2
p2 – q 2
144 12
1  tan2  =  tan  =
 2 2
p – q . tan  = q  sin  + 2 25 5
sin 
Alternate:-
 sin2  – 2 sin  + 1 = 0 sin 
p 2sin  – 3cos  2 –3

cosec θ =  (sin  – 1)2 = 0  sin  = 1  = cos
sin 
q 4sin  – 9cos  4 –9
 sin  = sin90°   = 90° cos 
θ = 30º

Rakesh Yadav Readers Publication Pvt. Ltd. 220

For More Visit : www.LearnEngineering.in


For More Visit : www.LearnEngineering.in

235. (d) Given 238. (b)Th e val u e o f co s 2 4 º


12  +cos55º +cos125º +cos204º
2  – 3  sin222° + sin268° + cot2 30°
2tan  – 3 5
= = + cos300º
4 tan  – 9  12   cos268° + sin268° + cot230°
4  – 9 We know that, cos ( 180º   )
5 
[From Eq. (i)]
sin(90 – )  cos 
cos(90 – )  sin 

 = – cos
co s24º + co s55º + co s
2
24 –15 9  1 + cot 30° (180º–55º) + cos(180º + 24º)
= = =3 2 + cos(360º – 60º)
48 – 45 3  1+  3 cos24º + cos55º – cos55º
7 – cos24º + cos60º
232. (d) r sinq = ......(i) [ cot30° = 3]
2

ir
 4 1
= cos 60º 

v.iSn
7 3 236. (c)Given, 2
r cosq = .....(ii) se c  ta n  51
2 2 239. (c) sin A + cosecA = 3
On squaring and adding both se c ta n  79
1
equation Sin A + =3

dnag
sec   tan  209 sin A
 
2 7 3 
2 sec   tan  79 Squaring both sides
7 
r² sin²q + r² cos²q =   + 
   2 

by componendo-dividendo
2    1
 a c a b c d  sin² A + +2=9

eYari
  ,   sin²A
49 147  b d a b c d 
r² (sin²q + cos²q) = 
4 4 sin4 A  1
sec 288 =9–2=7
  sin2 A
196 tan 130
r² =

r=
4
= 49

49 = 7
1 snhe
cos   288
 sin  130
240. (d) cos cosec = 1 if  + = 90°
cos7° cos23° cos45° cosec83°
cosec67°
kgei
 cos  = (cos7° cosec83°) (cos23°
233. (d) q +  = cosec67°) cos45°
2 1 288
  1 1
ERna

q +  = 90°......(i) sin  130 =1×1× =


2 2
1 130
sinq = Therefore, sin  
288 tan cot
2 241. (b) =2
65 tan– cot
aBryn

1  sin   tan + cot = 2tan – 2cot


sinq = sin30° = ......(ii) 144
2 237. (b)Given, 1+cos 2 = 3sin  3cot = tan
put q = 30° in equation(i) cos [O<</2]
3
30° + = 90° 1+cos2 = 3sincos = tan
Les

tan
Find cot = ?
= 60°
wa. th

By dividing sin2both sides tan² = 3


3  tan = 3 or  = 60°
sin = sin60° = 1cos2  3sin cos
2  
sin2  sin2  3
234. (b) Given sin = sin60° =
wwM

cosec2 + cot23cot 2
 Sec  + tan  = p.....(i)
1+cot 2  + cot 2   3co t 242. (b)sin x + cos x = c
Then,
[1+cot2 = cosec2] squaring both sides
1 1+2 cot2 = 3cot sin²x + cos²x + 2sin x cos x = c²
sec  – tan  = p .....(ii)
2cot2 = 3 cot– 1
c² – 1
From equation (i) + (ii) Let = 45º sin x cos x =
2
1  cot45º = 1 we know that,
 2 sec  = p+ p 2cot245º – 3cot 45º + 1 = 0 sin6 x + cos6 x = 1 – 3sin² x cos² x
2–3+1 =0
2
1 1 0=0  c² – 1 
sec  = 2  p  p  = 1 – 3  

  Therefore cot = cot45º=  2 

Rakesh Yadav Readers Publication Pvt. Ltd. 221

For More Visit : www.LearnEngineering.in


For More Visit : www.LearnEngineering.in

 c 4  1 – 2c²  1 2 1 2 1 – cos 2 sin2



=1–3 
 =1+ + – 1– + y = sec – cos = =
4  2 2 2 2 cos cos
 
4 2/3
1  6c² – 3c4 = = 2 2 
 1 sin2

= 2 (x² y) 2/3
=  2 2
. 
4 Now check from option  sin .cos  cos 
Alternate:-
Option:- (d) 4 mn 1
Put, x = 0 
cos ²
then c = 1  1  1 
put, c = 1 in all option 1 
  4  

2
1– 
2  1 sin4
2/3
  2/3  . 
1 (x y²) =  
 sin.cos cos ²

r
option (a) = , option (b) = 1
4 1
4 12 –

Sni
2
 sin ²
option (c) =
5
, option (d) =
5  2 
cos ²

gv.i
8 2
Hence, sin6x + cos6x = 1 = option (b) 1 1 sin2
 4 1– (x² y)2/3 – (xy²)2/3 = 2 –
243. (a) 2 cos  cos ²

ridna
1 – sin A.cos A sin2 A – cos2 A 1 4 cos2
.  4  2 = =1
cos A(secA– cosecA) sin3 A  cos3 A 2 2 cos ²
Alternate:-

eeYa
(1 – sin A cos A) = 2 2 (satisfy)
= Put = 45°
 sin A – cos A  . 245. (d) tan θ + sin θ = m
cos A 
 
 x = cot 45° + tan45° = 2
 sin A cos A  tan θ – sin θ = n
1
geisnh
Put θ = 45º y = sec45° – cos45° =
sinA cosAsinA –cosA tan45º + sin45º = m
2
sinA cosA sin2 A cos2A–sinAcosA
   4 2/3  2 2/3
1
1+ =m (x²y)2/3–(xy²)2/3=   – 
   
sin A(1 – sinA .cosA) 2  2 2
= = sinA
Enak

(1 – sinA .cosA) =2–1=1


1
244. (d) mn = cot² – cos² 1– =n sin8– cos8
2
247. (b)
cos ²(1 – sin²) cos 2(1  cos 2 2)
R

=  1  1 
sin ²  mn = 1 –
 1 
 
 4 4 2 2 2 2
 2 2 sin  cos sin – cos sin  cos 
mn = cot² . cos²
aryn

  2
= 2 2 2
 mn = cot .cos
= 1–
1
=
1 cos – sin  1 cos – sin² 
Les B

m² – n² = (m + n) (m – n) = 2 2
4 4
2cot . 2cos Now check from option,
=– 4
sin  cos 
4 2 2
= 4cot cos = 4 mn 1
m2 – n2
1  cos  sin – 2 sin .cos 
Option (d):-
4  
wa. th

Alternate:- We can say


cot θ + cos θ = m 2  2 
sin 4 + cos 4 = (sin² +
1 
1  1  – 1 – 1  
cot θ – cos θ = n = 4     cos² )² – 2sin² cos²

 2  2  
Put θ = 45º 2 2
1 – 2sin .cos 
wwM

Cot 45º + cos45º = m 1 1 2 1 2  =– 2 2 2 2


= 1  –1–   1  1 – 2sin .cos – 2 sin .cos 
1 4
 2 2 2 2 
 1+ =m 1
2 1 4  1 =–
=   = (Satisfy) 2
1 4  2
  2 Alternate:-
 m=1+ Answer is independent of angle
2 246. (d) x = cot + tan
 , so put  = 90°
1 cos sin
 n =1– = + sin8 90 – cos8 90
2 sin cos
cos180(1  cos ²180)
sin2 cos 2
2
 1 
2  1  1 1– 0 1
m² – n² = 1  1 –  = =
 –
  
2 sincos sincos =
–1(1  1)
=–
2
 2 

Rakesh Yadav Readers Publication Pvt. Ltd. 222

For More Visit : www.LearnEngineering.in


For More Visit : www.LearnEngineering.in

248. (a) k = (sec + tan ) (sec + ( cos2 = 2cos² –1) 3 1 4


  = =1 (Satisfy)
tan) (sec+ tan)....(i) 1 4 4 4
 cos2 = 2× –1
64 5 4 7
k = (sec – tan )(sec– tan)  3
2 – 64 –62 31 253. (a) sin4 +sin4 +sin4 sin
(sec– tan)....(ii) 8 8 8 8
= = =–
Multiplying equation (i)&(ii) k² 64 64 32
 3 3
= (sec²  – tan²  ) (sec² – 31 = sin4 +sin4 + sin4 ( – )
 4cos2 = – 8 8 8
8
tan²) (sec²– tan²) 
252. (b) cos( –A) = a, cos( –B) = b + sin4( – )
k² = 1  k =  1 8
Let  = 90°

ir
249. (a) asec + btan + c = 0
a = cos (90° – A) = sinA,  4 4 3
=2  sin 8  sin 8 

v.iSn
psec + qtan + r = 0 
b = cos(90° – B) = sinB  
Apply cross multiplication  4 
cosA = 1 – a² , cosB = 1 – b² 4   

method to solve these eqn = 2sin 8  sin   –  
 sin² (A – B)+ 2ab cos (A – B)  2 8 
sec tan 1

dnag
= = = (sinA cosB – cosA sinB)² + 2ab  4 4 
br – qc pc – ar aq – bp =2  sin 8  cos 8 
(cosA cosB + sinA sinB) 


br – qc pc – ar 2

= a 1 – b² – b 1 – a ²  sin 3  sin 
  
sec = aq– bp & tan = aq – bp    –    cos 

eYari
 8 8
 2 8  
sec² – tan2 = 1
2 2
+ 2ab  1 – a². 1 – b²  ab 

  2  

= 2 sin ² 8  cos ² 8  – 2sin ² 8 .cos ² 8 
 br – qc   pc – ar  = a² (1– b²)+ b² (1–a²)–2ab  
   
 aq – bp  –  aq – bp  = 1
   
Then, (br – qc)² – (pc – ar)² = (aq – bp)²
250. (a)P = acos³x + 3acosx sin²x
+ 2ab
snhe
1 – a². 1 – b²

1 – a². 1 – b² +2a²b²

1 – 1 
= 2  2 

sin 
2
 
 

 1
1 – 
4  = 2 
 4  =

3
2
= a² – a²b² +b² – a²b² +2a²b² 254. (a) sin + cos = a,
kgei
Q = a sin³ x + 3a cos²x sinx
Put x = 45° = a² + b² sec + cosec = b
Alternate:-
a 3a  sin cos
ERna

P= + , cos( θ –A) = a b (a² – 1) = 


 sincos  (sin²
2 2 2 2
cos( θ –B) = b  
a 3a + cos² +2sin cos –1)
Q= + Put θ = 90º
2 2 2 2 cos (90º – A) = a sin cos ×2sincos
aBryn

4a a = sinA =
sincos
P= = 2a , Q = 2a b = sinB
2 2 = 2a
(P + Q)2/3 + (P – Q)2/3 Put A = 60º
B = 30º 1  1 1 
2/3 255. (b) cos15.cos7 2.cos82 2
 
Les

2  
=  2a  2a +0 a=
3
×2
wa. th

2/3 2 0 0
= 2 2a = 2a 2/3
1 cos15º.2  cos 7 1 .sin7 1 
1  
b= 2  2 2 
251. (b)8cos² + 8sec² = 65 2
sin²(A–B) + 2ab cos(A–B) 1
wwM


  cos15°.sin15°
0° < < 3 1 2
2 sin²(60º–30º)+2×  cos(60º–30º)
2 2 Multiply and divide by 2
8 1 1
 8cos² + = 65 1 3 3   2 cos15º sin15º
cos ²    
2 2
4 2 2
 8cos4 + 8 = 65cos²
1 3 1 1
 8cos4 – 64cos² – cos² + 8 =0   =1    sin2 15º
4 4 2 2
 8cos² [cos² – 8]–1 (cos² –8)= 0 Now check the option 1 1
Option (b):- a² + b²    sin30º
1 2 2
 cos² = ,
8  3 2  1 2 1 1 1 1
    × × =
 cos² = 8 (not possible)  
2    
  2 2 2 2 8

Rakesh Yadav Readers Publication Pvt. Ltd. 223

For More Visit : www.LearnEngineering.in


For More Visit : www.LearnEngineering.in

256. (b)3tan tan = 1 6 3


1 3 1 1 sinA = =
    10 5
 =  = 30° 2 2 2 2
265. (b)cot41°. cot42°. cot43°.
cos(30 – 30) cos 0 3 1 cot44°. cot45°. cot46° cot47°.
cos(30 30)
= =2 sin15º 
cos 60 2 2 cot48°. cot49°
257. (c)tan 60°  (cot41°.cot49°). (cot42°.
262. (d)  is an acute angle
cot48°) (cot43°.cot47°)
tan20 tan 40 2sin+ 15cos2= 7
= (cot44°.cot46°) cot45° [cotA.
1 – tan 20tan 40 2sin+ 15 (1–sin2= 7
cotB = 1 [If A + B = 90°]= 1
2sin+ 15 – 15 sin2= 7
tan20° + tan40° = 3 – 266. (d) 5cos 12sin 13
15sin2–2sin– 8 = 0

r
3 tan20°.tan40° Let sin= x

Sni
5 12 13
15x2– 2x– 8 = 0 cos sin
tan20° + tan40°+ 3 tan20°. 13 13 13

gv.i
15x2– (12– 10)x– 8 = 0
tan40° = 3 [Divide whole question by
15x2– 12x+ 10x– 8 = 0 R.H.S. value]
258. (a)Given let x = 1 and θ = 0° 3x(5x – 4) + 2(5x – 4) = 0

ridna
......(i) (3x + 2) (5x – 4) = 0
 x cosθ – sinθ =1 5 Base 12 Perpendicular
2  1
 1×1 – (0) = 1  x  – (Rejected) 13 Hypo 13 Hypo
3

eeYa
1=1
putting value x = 1 and θ = 5
0° in equation (i) 4 4 13
x  a Hypo.
5 12
 x² + (1 + x²) sin θ
geisnh 3 Perpe.
 1² + (1 + 1²) × sin 0°= 1
4 Perp.
259. (d)Simplest form of  sin    Q
5 Hypo. Base
 sinA. cosA (tanA – cotA) 5
Base = 3
Enak

 sinA cosA  (Using triplets 3, 4, 5) P 12


 sinA. cosA  –  s in   =
 cosA sinA  H 13
Base
Therefore, cot   Perp.
R

 sin ²A – cos²A 267. (a) tan cot 0


 cos²A = 1 – sin²A Put = 45°
3
aryn

 2sin²A–1 cot   tan 45°– cot 45° = 0


4 1–1=0
260. (d) Given:-
Les B

263. (b)According to the question, 0 = 0 (Satisfied)


cos  cos  sin4 + cos4 = 2sin2 cos2 So,  = 45°
n= , m=
sinβ cosβ Put  = 45º
tan( 15)
 cos  = n sin β , and sin445º + cos445º = 2sin2 cos2 Now,
wa. th

tan( 15)
cos  = m cos β 1 1 1 1
  2  tan(45 15)
4 4 2 2 
cos²  = n²sin² β ....(i) tan(45  15)
1 1
wwM

cos²  = m²cos² β ....(ii)   Satisfy


equation (i) = (ii) 2 2 tan 60 3
   3 3  3
 tan45º = 1 tan 30 1
 n²sin² β = m²cos² β
264. (c)According to the question, 3
 n²(1– cos² β ) = m²cos² β
cosecA + cotA = 3 268. (c) SHORTCUT METHOD
n² 1 Put= 30°
 cos² β = cosecA  cotA =
m²+n² 3 1
261. (a)Given, sin(A–B) = sinA . 10 sec – tan =
cosB – cosA . sinB 2cosecA = 3
3
Let A = 45º, B = 30º
sin(45–30)º = sin45º. cos30º –cos45º 10 1
cosecA = sec30° – tan30° =
. sin30 6 3

Rakesh Yadav Readers Publication Pvt. Ltd. 224

For More Visit : www.LearnEngineering.in


For More Visit : www.LearnEngineering.in

271.. (b)x cos² 30º. sin60º Note:- Let A = 60º, B = 40º,


2 1 1
– = tan²45º.sec60º C = 80º
3 3 3 =
cosec60º B  C A
1 1  sin  2  = cos 2
= (satisfy) 2  
3 3  3  3 12  2
x  2   2  = 40º  80º
sec = 30°      2   sin
  2
 sec. tan  3
60º
 sec30°. tan30° 2 = cos
x = 2 2
2 1 3
2  120º 

ir
 × = 60º
3 3 3 3  sin  2  = cos 2

v.iSn
272.(b) sin(  +30º) =  
269. (d) (cosec a – sin a) 12  sin60º = cos30º
(sec a – cos a) (tan a + cot a)
3 3 3 3
Put a = 45° = =  =

dnag
2 3 2 2 2
(cosec 45° – sin 45°) (sec
45° – cos 45°) (tan 45º + cot 45°) sin (  + 30º) = sin60º  Therefore
  = 30º BC A
    sin  2   cos  2 
 2 – 1   2 – 1  ( 1 + 1)

eYari
 2    
2  2
 3 3
      276. (b)
cos²  = cos² 30º =  2  =
  4
1 9 5
273. (a)4 cos²  – 4 3 cos  + 3 = 0  + 4cos²θ +
×2=1 cosec²θ 1+ tan²θ
2
270. (c) According to the question
tanA = ntanB and sinA = msinB
snhe
Hit & trial method
Put  = 30º option (a)
4 cos² 30º – 4
 9 sin ²θ4cos ²θ
5
1tan ²θ
3 cos30º + 3 =0
kgei
tan A sin A  (1tan ²θsec ²θ)
n= m=  3
tan B sin B 3  9 sin ²θ4 cos ²θ5 cos ²θ
4  – 4 3  3 = 0
Put A = 30° and B = 60° 4
   2 
ERna

 9 (sin ²θcos ²θ) = 9


0 = 0 277. (a)
1
1 274. (d)According to the question Base p
2 , cosθ = =
n= 3 m= 3 Hyp p +q 2
2
aBryn

3  Sec4A (1– sin4A)– 2tan2A


2 Put A = 45° A
 Sec 4 45° (1 – sin 4 45°)–
1 1 p2 + q 2
n= m= 2tan245°
3 3 q
Les

 1
3  4 1 – 4  –2
wa. th

 
 cos2A = cos230° =
4 q
3 B p C
Now check from option to save 4× –2 (Hyp)² = Base² + perp².
your valuable time 4
Perpendicular = q
3–2=1
wwM

m2 –1 275. (d) Perp. q


Option (c) : tan θ = =
n2 –1 A Base p
2 278.(c) tanθ + secθ = 3
 1  1 –2 60º
  –1 –1 1
 3 3 3 secθ – tanθ =
3
=  1 2 = 1 = 8 40º 80º
  –1 –1 – B C
9 9 10
 3  As we know 2secθ =
3
 sin (90 –  ) = cos 
3 3 4
= (satisfy)   B+C   cosθ = ; sin θ
A 5 5
4  sin  2  = cos 2
  5 sin θ = 4

Rakesh Yadav Readers Publication Pvt. Ltd. 225

For More Visit : www.LearnEngineering.in


For More Visit : www.LearnEngineering.in

x – x tan²30°  tan2  + 1
3 –1
279. (c) (  sin2  + cos2  = 1) 286. (d)sinA – cosA =
1  tan²30° 2
= sin²30° + 4 cot² 45° – sec² 60°  sec2 
Shortcut Method:-
(  1+ tan2  = sec2  )
x 1–tan²30°  1 2 Put  = 60º
282. (d)
=   + 4 ×1 – (2)²
1  tan²30° 2  π π π π π 3 –1
tan2 – cos 2 = x sin cos tan  sin60º – cos60º =
4 3 4 4 3 2
  1 2 
x    tan²45º – cos²60º = x
1 –  3  
 3 1 3 –1

    sin45º cos45º tan60º  – =
1 2 2 2
 1 2 = +4–4

r
4 1 1 1
1  
  1– = x   3 3 –1 3 –1

Sni
 3 4 2 2  =
2 2

gv.i
 1 3 x 3 (Matched)
x 1 –   =
4 2 Hence, sinA . cosA
 3 
 1
1 =  sin60º . cos60º

ridna
1 4 3
3  x = 3 1 3
2   =
2 1 4 2 2 4

eeYa
x× = × 3 Alternate:-
3 4 3 283. (d) sin2 θ =
2
1 3 1 3 –1
x=× = sinA – cosA =
3 2 2 3 2
 sin2 θ =
geisnh 2 Squaring both side,
280.(d) According to question
Cosq + sinq = m  sin²A + cos²A – 2 sinA cosA
 sin2 θ = sin60º
Secq + cosecq = n 2
 2θ º = 60º  3 – 1
put q = 45° =  
Enak

cos45° + sin45° = m, 60  2 
sec45° + cosec45° = n  θº = = 30º
2  1 – 2sinA cosA
1 1
R

 So sin3 θ = sin 3 × 30º 3 1 – 2 3


+ = m, 2 + 2 =n =
2 2 = sin90º = 1
4
aryn

m = 2 ,n = 2 2 sin θ  cos θ 3
284. (a) =
sin θ – cos θ 1  2 sinA cosA = 1 – 2 2 – 3  
Les B

2
 n(m² – 1) = 2 2  2  –1
 sin θ  cos θ 3
4
to find sin θ = ? 4

=  by componendo dividendo  2–2 3


= 2 2 (2 – 1) = 2 2 sin θ – cos θ 1  2 sinA . cosA =
2
wa. th

Now check from option only sinθ 3 1


one option satisfy  = 3
cosθ 3 –1 sinA. cosA =
2m = 2 × 2= 2 2 4
 tan θ = 2
281. (d) Given:- a p
wwM

Perpendicular 2 287. (c) sin  = =


tan θ = = b h
 β = 90º Base 1 A
  + β = 90º 4
 2 
  = 90 – β  sin4 θ =   = 16 25
 5 a b
  , β are complementary 285. (b) cos20º = m
angles cos70º = n q
tan  So, m² + n² = cos²20º + cos²70º B C
  sin2   sin2  90 –  
tan  90 –   = 1 BC = b2 – a 2
2 2
tan  cos A + cos B = 1 [using pythagoras theorem]
  sin2   cos 2     sec  – cos 
cot   if A + B = 90º 

Rakesh Yadav Readers Publication Pvt. Ltd. 226

For More Visit : www.LearnEngineering.in


For More Visit : www.LearnEngineering.in

H B AC BC tan9° + cot 9° – (tan27° +


= – = – sec 2 70– cot 2 (90–70) cot27°)
B H BC AC
2 cos ec 2 59– tan2 90º –59
  sin9 cos9  sin 27  cos 27  
b b2 –a 2 = + – 
 cos 27   sin 27  

= – 2 cos9 sin9  
b2 – a 2 b =
m 1 1
2 = –
=
b2 –  b2 – a 2  2
sec 70 – tan 70 2
=
2
sin9cos 9 sin27cos 27
2 2
b b –a 2 2 2(cos ec 59 – cot 59) m 2 2 2(sin 54  – sin18 )
= – =
sin18 sin54 sin18  sin 54 
b2 – b2  a 2 a2 1 2 sec2  – tan2   1 
=   Using SinC and sinD formula

ir
= = 2 2
b b2 – a 2 b b2 – a 2 2 m cos ec  – cot   1
2(2sin18cos 36)

v.iSn
m=2×2=4 = =4
1  sin  1 – sin  sin18cos 36
288. (c) + 293. (a) sin x + 2cos x = 1 ...(i)
1 – sin  1  sin 
4 3 5 Put x = 90°

dnag
2 2
290. (a)cos ( +) =
 1  sin     1 – sin   5 sin90º + 2cos90º = 1
= 4 1+0=1
2
1 – sin 
3 1 = 1 (Satisfy)
1  sin   1 – sin   tan ( +) = Now,

eYari
4
= 7cosx + 6 sinx
cos 2 
= 7 cos90° + 6 sin90° = 6
2 5 5 13 294. (b) cos²48° – sin²12°
= = 2sec  sin ( –) =
cos 
Alternate:-
Put θ = 30º
snhe 13

5
12
[  cos²A – sin²B = cos (A + B).cos(A – B)]

cos60°.cos36° =



1  5  1
=
5 1
 tan( –) = 2  4  8
kgei
1  sin30º 1 – sin30º 12
 
1 – sin30º 1  sin30º  tan2 = tan ( ++ –) 5 1
[Using cos36º = ]
1 1 4
ERna

1 1– tan()  tan( –)


2 2 = 1 – tan()tan( –)  3 
1 +
 1  
1– 1 295. (c) sin 10  sin 10 
2 2    
3 5
aBryn


3 1 4 12 14 48 56 1    3
= 3 5 = × = 2sin   sin 
2  2 2   
10  10 

1 3
1– . 12 48 –15 33 
4 12
2 2 2sinA SinB = cos(A – B) – cos(A + B)
 2 3
1 1  2 4
Les

3 291. (a) cos + cos + cos +


 7 7 7 = cos – cos 

3 2  10 10 
wa. th

3 1 4 4 5 6
 = cos + cos + cos 1
3 3 7 7 7 = (cos36° – cos72°)
2
Now check from option to save  3
 2 3
wwM

your valuable time, option 'c' =cos +cos +cos +cos  – 7 


  1
7 7 7   = (cos36° – sin18°)
2 2
2sec θ = 2sec30º = 2 × (By putting the value of cos 36°
3  2  
 – 7 
+ cos   + cos – 7 
  and sin18°)
   
 5 – 1
4 1  5 1 –  1 1 1
= (Satisfied)  2 3 2 =  = × =
3 = cos + cos + cos – cos 2  4 4   2 2 4
7 7 7 7
sec 2 70 – cot 2 20    
 3
289. (c) 2 cos ec 2 59 – tan2 31 296. (a) tan 
 4   4 –
 – tan 
  – cos
7
– cos
7
=0   


2 292. (d) tan9º – tan27º – tan63º + 1  tan 1 – tan
= = –
m tan 81º 1 – tan 1  tan

Rakesh Yadav Readers Publication Pvt. Ltd. 227

For More Visit : www.LearnEngineering.in


For More Visit : www.LearnEngineering.in

1  tan ² 2 tan – 1 – tan ² 2 tan  sin² + sin² – sin² = sin(60º– θ ).sin θ .sin(60º+ θ )
=
1 – tan ² Here, θ = 12º
= 2sin sin
2.2tan 1 sin 36.sin54
= = 2tan2 (sin sin– cos cos) = .
1 – tan² 4 2sin36cos 36
Alternate:- = – 2sin sincos ( +)
1 sin54 1
  = – 2sin sincos(180° +) = . =
  8 sin54 8
tan   – tan  4 –
4    = 2sin sincos 1
Put θ = 15º Alternate:- 301. (d)  3 cos 23º – sin23º 
4
tan(60º) – tan(30º)  +– = 

r
 3 1 
1  cos 23 – sin23
 1   2 

Sni
 3–
  
  sin² + sin² – sin² 2  2 
 3
Put  = 90º

gv.i
2 1
  = 90º = (cos30°.cos23°– sin30° sin23°)
3 2
Now check from option,  = 0º [cosA cosB – sinAsinB = cos(A + B)]

ridna
Option (a):-  sin² 90º + sin² 90º – sin² 0º 1
2tan2 θ  1+1–0 = (cos53°)
2
 2tan2 ×15º  2 302. (a) 2cosx – cos3x –cos5x

eeYa
2 Now check from option, = 2cosx – (cos3x + cos5x)
 2tan30º = (Satisfy)
3 Option (a):- 2sin sin cos   3x  5x   5 x – 3x 
= 2 cosx – 2 co s 


 2
cos 



 2


 

297. (a) sin (–) = cos ( +) = sin  2sin90º sin90º cos0º
= 2cosx – 2cos4x cosx
geisnh
 2×1×1×1
(90° –  – ) = 2cosx (1 – cos4x)
 2 (Satisfy) = 2cosx (1 – 1 + 2 sin²2x)
 –  = 90 –  – 299. (c) 1 + cos2x + cos4x + cos6x = 4cosx sin²2x
 = 45°,  = 15° 1+ 2cos²x – 1+2c os = 4cosx (2sinx.cosx)²
Enak

Alternate:-  4x  6x   6x – 4x  = 16 cos³x.sin²x

 
 cos 
 
 Alternate:-
1  2   2 
sin( –) = Put x = 45° in question and all
2
R

= 2cos² x + 2cos5x.cos x option


 sin( –) = sin30º = 2cosx (cosx + cos 5x) 2cos45° – cos135° – cos225°
aryn

 – = 30º ...(i)  5x  x   5 x – x  2 1 1 4


= 2cosx 2 cos 
  cos 
  
 = + + =

  2   2   2 2 2 2
1
Les B

 cos( +) = = 2cosx. 2cos3x.cos2x When x = 45°, (sinx = cosx)


2
 cos( +) = cos60º = 4cosx cos2x cos3x  1 5
Alternate : Option (a) 16 cos5x = 16  

 + = 60º ...(ii)  2
We can put x = 0° in question
wa. th

Solve eq(i) and (ii) we get and all option 16 4


= =
 = 45º and  = 15º 1 + cos2x + cos4x + cos6x 4 2 2
298. (a)  +  = + =1 + cos0° + cos0° + cos0° = 4  1  1
5

Option Option (b) sin5x =   =



 2 4 2
wwM

 sin ( + ) = sin ( + ) (a) 2cosx cos2x cos3x = 2


 sin cos+ cos sin = – sin (b) 4sinx cos2x cos3x = 0  1 5 4
(c) 4 cosx cos2x cos3x = 4 Option (c) 4cos x = 4 
5  =

squaring both the sides  2  4 2
(d) cosx cos2x cos3x = 1
 sin²  (1– sin² )+ sin² (1 – 1
Hence, option (c) is correct =
2
sin²  )+2 sin  . c os . 300. (b)
divide and multiply by s i n 7 2 º ,  1 5 4
cos sin = sin² Option (d) 4cos x = 4 
5  =
2  4 2
sin12.sin 48.sin72.sin54  
 sin² – sin² sin² + sin² –
sin 72 1
sin²  sin² + 2sin  cos =
1 2
.cos sin = sin² Use sin3 θ Hence, Option (a) is correct.
4

Rakesh Yadav Readers Publication Pvt. Ltd. 228

For More Visit : www.LearnEngineering.in


For More Visit : www.LearnEngineering.in

3 1 4 1 – tan ² 1 1
303. (c) – = = 2cos2 +cot =
sin 20 cos 20 2 1  tan ² tan ysin

3 cos 20 – sin 20 Alternate:- 1


=  cot + cot =
y sin
...(ii)
sin 20.cos 20 n tan – 30º

m tan  120º from equation (i) and (ii)
 3 1 
2 cos 20 – sin20 1 1

=  2 2  Put θ = 90º

x sin
= y sin
sin20.cos 20 n tan 90º –30º
 
m tan 90º 120º x sin
2.2sin60cos 20 – cos 60sin20

ir
=  y = sin
2.sin20.cos 20

v.iSn
n tan60º
  b
4sin 40 m tan210º 308. (d) tan x =
= =4 a
sin 40
n tan60º
 

dnag
cos12º – sin12º sin147º m tan 180º 30 º b b
304.(c) + 1 1–
cos12º  sin12º cos147º a a
n tan60º = b + b
sin78 – sin12   1– 1
= m tan30º a a

eYari
sin78 sin12
sin147 n 3 1  tan x 1 – tan x
 
+ m 1 = +
cos147 1 – tan x 1  tan x
3
sin(180º– θ ) = sin θ
cos(180º– θ ) = – cos θ

2cos 45.sin33 sin33



n
m
snhe
 3
Rationalising above equation

=
1  tan x 1  tan x

1 – tan x 1  tan x
kgei
= – =0 m 1
2sin45.cos 33 cos 33 
 1 – tan x 1 – tan x
305. (d) cos24° + cos55° + cos125° + n 3 + 
1  tan x 1 – tan x
cos204° + cos300°
ERna

m+n 1 3 4
= cos24° + cos(90° – 35)+  = = = –2 2 2
cos(90° + 35°)+ cos(180° + 24°)+ m–n 1– 3 –2
= 1  tan x  + 1 – tan x 
cos(360° – 60°) Now check from option, 1 – tan ² x 1 – tan² x
=cos24° + sin35°– sin35º – Option (a):- 2cos2 θ 1  tan x 1 – tan x
aBryn

= +
1 Put θ = 90º 1 – tan²x 1 – tan²x
cos24°+ cos60°=
2 = 2 cos180º 2
306. (a) mtan(–30º) = ntan( +120°) = 2cos(90º + 90º)
=
1  tan x   (1 – tan x ) = sin ²x
= –2sin90º 1 – tan ² x 1–
m tan( –30°) = – n cot ( +30°)
Les

cos ²x
= –2 (Satisfy)
wa. th

–n 2
tan ( –30°).tan( + 30°) = x sin 2cos x
m cos ²x – sin² x =
307. (a) tan = =
1 – x cos cos 2x
cos ²x
3 tan– 1 3 tan 1 –n
. = 1 1 309. (c) tan +2tan2 + 4tan4 +
wwM

3  tan 3 – tan m  = – cot


tan x sin
8 1 – tan ²4
 tan A – tan B 
tan(A–B)=  1 1 2tan 4

 1  tan A tan B  +cot = x sin
tan  1 – tan²
3 tan ²– 1
=
–n
1  cot 2 2tan 
 
3 – tan ² m  
 cot + cot = x sin ....(i)
m 3 – tan ² =tan + 2tan2 +
=
n 1 – 3 tan ² y sin 8 tan ²4 8 – 8 tan ²4
Use Com pone ndo and tan = 2tan 4
1 – y cos
Dividendo
1 1 4
m n 4 – 4 tan ² = tan + 2tan2 +
=  tan = ysin – cot tan4
m–n 2  2tan ²

Rakesh Yadav Readers Publication Pvt. Ltd. 229

For More Visit : www.LearnEngineering.in


For More Visit : www.LearnEngineering.in

Alternate:- Put  = 45°


4 1 – tan ²2
= tan + 2tan2 + sinA = n sinB
2tan 2 2
Put, A = 90º & B = 30°
2
4 tan²2  4 – 4tan ²2a then, n = 2 = 1 1 =y
= tan  1 
2tan2  n – 1 1 2 2
 1
 n 1 tan60° = × 3 =
2   3 3 2
= tan + =y= =2–
tan2 Put these value in options 2 2 2

2 1 – tan ² A – B
= tan + 
Option (a) sin   = sin30° = 1
 =
1 – cos sin
2tan  2  2 1  sin

r
2tan ²  2 – 2 tan ² A – B 1 1 1

Sni
= 1–  2
2 tan Option (b) tan 
 2   = tan30° = = 2 2 =
  3 2 1
1

gv.i
1 1
= cot A – B 2
tan Option (c) cot 
 2   = cot30° = 3
Alternate:-  
2  2 – 1 = 2 –

ridna
.tan +2tan2+4tan4+8cot8 = 2 =y
 A  B 1
Put  = 30º Option (d) tan 
 2   = tan60° = 3
=tan30º+2tan60º+4tan120º+8cot240º   sin70 cos 40
314. (c)

eeYa
= tan30º+2tan60º–4cot30º+8cot60º Hence, option (b) is correct. cos 70 sin 40
1 8 311. (b)2sinA cos³ A – 2sin³ A cosA
cos 20 cos 40
= +2 3 –4 3 + 2sin Acos A(cos²A – sin² A) =
3 3 sin20 sin 40
1 8 1
geisnh 1 2cos 30.cos10
= –2 3 (2sin2A.cos2A) = sin4A =
3 3 2 2 2sin30.cos10
Alternate:- =cot30° =
9 3
= –2 3 .2sinA cos³A – 2sin³A cosA
Enak
3
Put A = 30º sin4 A cos 4 A 1
315. (a) + =
= 3 3 –2 3= 3 2sin30ºcos³30º–2sin³30º cos30º a b a b
Now check from options
R

3 3 1 3 a  b  a  b 
Option (c):- cot = cot30º – 2  
 a   sin4A + 
 b   cos4A = 1
8 8 2    
= 3 (Satisfy)
aryn

310. (b) sinA = n sinB 3 3 3 3 a  b 


– =  a sin²A 
  sin²A +
Les B

8 8 4  
sin A n
= Now check from options
sinB 1 a  b 
applying componendo dividendo rule 1 
 cos ²A 
 cos²A = 1
Option (b):- sin4A  b 
n 1 2
wa. th

sin A  sinB
 = Put A = 30º  ab ab 
sin A – sinB n –1 
 sin ²A  1& cos ²A  1

A  B A – B 1  a b 
2sin 
  cos 
  
  × sin120º
 2   2  2
a b
wwM

 A  B A – B
1  sin² A = & cos²A =
2cos 
 2  sin 
 
 ab ab
   2   × sin(90 + 30º)
2 Now,
n 1 1
=
n –1  × cos30º sin8 A cos8 A
2  +
A  B a3 b3
tan 
 
 1 3 3
  2  = n 1  × = (Satisfy) a4 b4
A – B n –1 2 2 4
= 4 + 4
tan 
 
 312. (d) tanA +tanA –tanA –cotA a  b a3 a  b b3
 2 
= tanA – cot A
 n – 1  A  B A – B ab 1

 
 tan 
 2   = tan 
 2   2sin
 n 1 313. (a) =y = ab 4 =
   
1  cos sin   (a  b)3

Rakesh Yadav Readers Publication Pvt. Ltd. 230

For More Visit : www.LearnEngineering.in


For More Visit : www.LearnEngineering.in

Alternate:- 317. (d) tan – cot = cos²0° + cos²120° + cos² (– 120°)


Put A = 45º and a = 1, b = 1 = a & cos + sin = b 1 1 3
=1 + + =
4
sin 45º cos 45º 4
1 Put,  = 45° 4 4 2
 + =
1 1 11 tan45° – cot45° = a, 1 – cos B
321.(a) tan A =
cos45° + sin45° = b, sinB
1 1 1
  =
4 4 2 1 1 B B
a = 0, + =b 1 – cos 2  1 – 1  2sin ²
2 2 2 2
1 1 = B = B B
 = (Satisfy) sin 2  2sin cos
2 2 11 2 2 2
= b b = 2
2
sin8 A cos 8 A

ir
B
  (b² – 1)² (a² + 4) sin

v.iSn
a3 b3 2 B
2 2
tanA = B = tan
sin8 45º cos 8 45º
 cos 2
 =    2 – 1 (0 + 4) 2
13 13 
 
B

dnag
= (2–1)² (4) = 4 A=  B = 2A
1 1 1 2
  sin – cos
16 16 8 318. (a) tan = tan2A = tan B
Now check from option sin cos 322. (c) tan2A = tan (A + B +A– B)

eYari
1 sin sin – cos tan(A B)  tan(A– B) pq
Option (a):- = = 1 – tan(A B).tan(A– B) = 1 – pq
3 cos sin cos
a +b
let, sin – cos = k sin
1
1 1 –1

1  1
3 
8
(Satisfy)

316. (a) 2 ycos = x sin


snhe
sin + cos = k cos
Adding after squaring,
sin² +cos²  –2sin cos  +
323. (b)
sec 8A – 1
sec 4A – 1
=
cos 8 A
1
–1
cos 4 A
sin² + cos² + 2sin cos
kgei
= 2ycosec = x sec 1 – cos 8A cos 4A
= k² (sin² +cos² ) = ×
Put value of x sec as following 1 – cos 4A cos 8A
2 = k²
2 x sec – ycosec = 3 2sin ²4A cos 4A
ERna

k= 2 = ×
2.2 y cosec – ycosec = 3 2sin ²2 A cos 8A
Hence, sin – cos = 2 sin 2sin 4A cos 4A.sin4A
4y cosec – ycosec = 3 =
sin + cos = 2 cos cos 8A  2sin²2A
3y cosec = 3
aBryn

319. (b) sin + sin= a, cos 2 A tan 8A


= y cosec = 1 =tan 8A. =
sin2A tan2A
or sin = y cos +cos = b Alternate:-
then, 2 ycos = xy Put,  = 90° &  = 30° sec 8A – 1
Les

 2cos = x sec 4A–1


1 3 3
wa. th

Hence, a=1+ = , b= Put A = 15º


2 2 2
x² + 4y² = 4cos² + 4sin² = 4 sec120º –1
Alternate:- – 1 
tan 
 2 
 = tan30° = sec 60º –1
Put  = 45°   3
wwM

1
2ycos45° = xsin45°  x = 2y –1
4 – a² – b² cos120º
2x.sec45° – y cosec45° = 3 option (b)   1
a²  b ² –1
2 (2x – y) = 3  2 (4y – y)= 3 cos60º
9 3
4– – 1
1 4 4 1 – –1
y= = 9 3 = sin30º –2 – 1
2  3 
1 = = –3
4 4 –1 2 –1
2 cos 60º
x= = 2 Hence, option (b) is correct
2 320. (a) Answer is independent of so Now check from options
1 put = 0° tan 8A
x² + 4y² = 2 + 4× =4 cos² +cos²( +120°)+ cos² ( –120°) Option (b):-
2 tan2A

Rakesh Yadav Readers Publication Pvt. Ltd. 231

For More Visit : www.LearnEngineering.in


For More Visit : www.LearnEngineering.in

Put A = 15º
1 tan3x  tan2x
1– 331. (a) tan 5x =
tan120º – cot 30º 49 48 24 1 – tan3x tan2x
= = 1 = =
tan30º tan30º 1 50 25  tan5x – tan 5x tan3x tan2x
49
= tan3x + tan2x
= – 3  3 = –3 (Satisfy)
2  tan5x tan3x tan2x = tan5x
a b 2tan 3 –tan3x – tan2x
324. (d)  + = TanA +TanB = + sin2 = 1  tan ² = 1
b a 1 332. (d) cos + cos β = sin +sin β
9
a²  b² Squaring both sides
= 2 9 3
ab = × =  cos ² +c os ² β +2 cos c os β
3 10 5

r
a b = sin² + sin² β +2sin .sin β
= tanA. tanB = . =1 4

Sni
b a cos2 =
5  (cos² – sin² ) +(cos² β –sin² β )
B

gv.i
sin4 = 2sin2cos2 +2(cos .cos β – sin sin β ) = 0
a c 3 4 24
=2× × =  cos2 +cos2 β + 2 cos ( + β ) =0
5 5 25

ridna
C A
b
hence,cos2 = sin4  cos2 +cos2 β = –2 cos( + β )
then equation
333. (c) cosA = a cosB  cos²A = a²
x² – ( +)x += 0 1 1 cos²B = a² (1– sin²B) .....(i)
328. (b) tanA = , tan B =

eeYa
2 3 sinA = b sinB  sin²A = b sin²B
 a²  b² 
x² – 
 x + 1 = 0
 1 ....(ii)
 ab  1–
1 – tan²A 4 3 Adding both equation
abx² – (a² + b² ) x + ab = 0 cos2A = = 1 =
geisnh
1  tan ²A 1 5 cos²A+sin²A = a² – a² sin²B + b²
abx² – c²x + ab = 0 4 sin²B
( a² + b² = c²)
2 1= a²+sin²B (b²–a²)
3 3 (b² – a²) sin² B = 1 – a²
325. (d)cos (A – B) = and 2tanB
5 sin2B = = 1 Alternate:-
Enak

1  tan ²B 1
9
sin A sinB cos A sin A
tanA tanB = =2 a= , b=
cos A cos B 2 9 3 cos B sinB
= × =
R

SinAsinB = 2cosAcosB.. (i) 3 10 5 Put A = 0º, B = 60º


3 Hence, cos2A = sin2B a = 2, b = 0
aryn

cosA cosB + sinA sinB =


5 329.(c) Put β = 0º such that no option  (b²–a²)sin²B =(0²–2²)×sin²60º
From eq (i) are same
Les B

3
3 2sin² β + 4c os( + β ). = –4  = –3
4
3cosA cosB =
5 sin .sin β + cos²( + β ) Now check from option
1 = 0 +0+cos² Option (c):- 1 – a²
wa. th

cosA cosB = Put a = 2


5 = cos² option (c) (We can also
2 put  = 0º) 1 – (2)² = 1 – 4 = – 3 (Satisfy)
sinA sinB = 330. (d) 334. (c) Put A = 90º, B = 60º, C = 30º
5
( A+B+C = )
wwM

326. (a) sin12º.sin48º.sin72º.sin24º.sin36º.sin84 then, cos 2A + cos2B + cos2C


cos 9 sin 9 sin81 sin9 sin72º.sin36º
= = cos 180º + cos120º + cos60º
cos 9 – sin 9 sin81 – sin9 Multiply and divide by sin36º &
1 1
2sin 45.cos 36 sin72º = –1 –  =–1
= = cot 36° 2 2
2cos 45.sin36 1 Put these value in all option
sin θ .sin(60– θ ).sin(60+ θ ) = sin3 θ
= tan54° 4 (a) = 1+4cos A. cos B. cos C
=1
1 1 1 1
327. (b)tan  = , tan  = = sin36º× sin72º× (b) = –1+ 4 sin A sin B. cosC
7 3 4 4
3 3
1 – tan ² = –1+4×1× .
1 1 2 2
cos2 = =
1  tan² sin72ºsin36º 16 = –1+3=2

Rakesh Yadav Readers Publication Pvt. Ltd. 232

For More Visit : www.LearnEngineering.in


For More Visit : www.LearnEngineering.in

(c) = –1– 4cos A cos B. cos C


9 tan10º –3 tan ³10º 1  1
= –1–0=–1  tan² x  p² – 1 = 1– q²
1 – 3 tan ²10º  
(d) =1+4 sin A sin B. sinC
3 3tan10º – tan ³10º q 2 –1
3 1 
= 1+4×1× . = 1+ 3 1 – 3 tan ²10º
2 2 q²
tan² x=
Hence, option (c) is correct  3 tanθ–tan³θ  1–p²
tan3θ   p²
335. (b) Let A = B = 45º and C = 90º 
 1 – 3tan ²θ 
sin²A + sin² B + sin²C –2cos A.
cos B. cos C = 3 tan30º = 3 p q² –1
tan x = q 1 – p²
1 1 338. (a)Solve it by option
1 1

ir
=  +1 – 2 ×  ×0 =2 Alternate:-
2 2 2 2 

v.iSn
put,  = sin x
336. (c) sin2A + sin2B + sin2C 6 p
= 2sin (A+B).cos (A–B) +2 sinC. sin y
  
cosC 4 cos +3 cos2 –2 sin3 + cos x
6 6 6 q 
= 2sin (18 0º–C).c os (A–

dnag
cos y
B)+2sinC. cosC 2
= 2sin C [cos(A–B) + cosC] cos4 Put x = 30º and y = 60º
6
= 2 sin C [cos(A–B)+cos(180º– sin30º 1
(A+B)] 3 3 1 p= 

eYari
= 4.  –2– sin60º 3
= 2 sin C [ cos(A – B) – cos 2 2 2
(A+B)] cos 30º
q=  3
= 2 sin C (2sin A. sin B) = 4 sin 4 3 cos 60º
= –1 = 2 3 –1
A sin B sin C 2
Alternate:-
A+B+C=180º
Put A = B= 45º and C = 90º in
snhe
hence, option (a) is correct.
339. (a) (sin A + sin B+ sinC)² = sin²A
+ sin² B + sin²C
 tanx = tan30º =
1
3
Now check from options
kgei
equation and option too. As we know that p q2 –1
sin2A + sin2B + sin 2C = sin (a+ b + c)² = a² + b²+ c² + Option (b):-
q 1 – p2
90º + sin 90º + sin 180º 2(ab + bc+ ca)
ERna

= 1+1+0=2  2(sinA sin B + sin B sin C + 1 2

as
Now check for option sin C sin A) =0  3  3 –1
2
 sin A sin B + sin B sin C + 3  1 
(a) =0 (b) =0 sin C sinA = 0 1– 
 

 3
aBryn

1 1 above condition will be true


(c) = 4× . .1=2 only when
2 2 1 3 –1
A = B = C =0 
3 3 –1
1 1  sin A + sin B + sin C =0 3
(d) = 8× . .1=4
2 2 sin x cos x
Les

340. (b) =p& q 1 2


Option (c) is correct. sin y cos y  3
wa. th

3 2
337. (c)tan 10º – tan50º + tan70º sin x 1
 tan10º + tan (60º+10º) – tan  sin y  (Satisfy)
p 3
(60º – 10º)
sin² x 341. (a) y = a cos² x + 2b sin x cos x +
wwM

tan60º  tan10º   sin ²y c sin² x


 tan 10º+ p²
1 – tan 60º.tan10º z = a sin² x –2b sin x cos x + c
cos x cos² x cos²x
tan60º – tan10º  cos y  =cos²y
– q q² y + z =a (sin² x + cos² x) + c
1  tan60º.tan10º
Adding both eqation (sin² x + cos²x)
3  tan10º y+z=a+c
 tan 10º + sin² x cos ² x
1 – 3 tan10º  = sin² y + cos² y =1 342.(c) A + B + C = 
p² q²
answer is independent of A, B
3 – tan10º divide by cos² x &C
– So put A = B = C = 60º
1  3 tan10º tan² x 1 1
  cosA cosB cosC
8tan10º p² q² cos ² x = + +
 tan10º + 1–3tan²10 sinBsinC sinCsinA sinAsinB
= sec² x = 1+tan²x

Rakesh Yadav Readers Publication Pvt. Ltd. 233

For More Visit : www.LearnEngineering.in


For More Visit : www.LearnEngineering.in

cos60º cos60º cos60º Alternate:-


= sin60º.sin60º + sin60º.sin60º + sin60º.cos60º 2sin x .cos y 2a
x² + px + q = 0 =
2cos x . sin y 2b
1 Put  = 45º
1
2 2  = 0º
tan x a
= 3× 3 3 =3. 3 =2 tan45º = 1 = =
. tan y b
2 2 4 tan0º = 0
343. (a) sinA. cosA & tanA are in G.P.  1² + p × 1 + q = 0
  2  
then, cos² A = sinA. tanA p+q=–1 ...(i) 348.(d) tan = tan 
 – 

3 5 15 
sin A.sin A 0² + p × 0 + q = 0
cos² A = q=0 ...(ii)
cosA 2 

r
From (i) and (ii) tan – tan
cos³A = sin² A = 1 – cos² A  5 15

Sni
cos³ A + cos² A =1 p = –1 and q = 0 tan = 2 
3 1  tan tan
344. (d) A + B = C, tan A = k tan B  tan ( +) = tan45º = 1 5 15

gv.i
and A – B =  Now check from options,   2 
 tan + tan tan tan
3 3 5 15
sinA k sinB –1

ridna
= p 2 
cosA cosB Option (b):- =
q –1 0 –1 = tan – tan
5 15
sinA cosB k
 =1 (Satisfy) 2  2

eeYa
cosA sinB 1  tan –tan – 3 tan
5 15 5
Using componendo & dividendo 3
rule 346.(a )If A + B + C = 
2 tan = 3
15
sinA cosB + cosA sinB k+1

geisnh 349. (a)Answer is independent of A,
cosB sinA – cosA sinB k – 1 
Put A = B = C = in question B, C and A + B + C = 180º
2 So put, A = B = C = 60º
sin A + B k+1
 and all options. A B C
sin A – B k –1
tan² +tan² tan² = tan² 30º+
Enak

cos2A + cos 2B + cos2C = cos + 2 2 2


sinC k+1 cos + cos = – 3 tan²30º + tan²30º

sin k –1
option (a) 1 1 1
=
R

  =1
k+1 3 3 3
sin C = sin   
k –1 = 1 – 4 sin . sin .sin = 1– 4 cot ²15º –1
2 2 2
aryn

350. (a)
345.(b) tan , tan β are roots of x² + cot ²15º 1
=–3
Les B

px +q = 0 cos ²15º – sin ²15º


sin x +y  a+b =
tan + tan β = – p cos ²15º  sin²15º
347. (c) sin x –y = a–b
  divide by cos²15º all terms
tan .tan β =q
1 – tan ²15º
wa. th

tan  tanβ sin x cos y + cos x sin y a+b = = cos 2×15º


= 1  tan ²15º
tan ( + β ) = 1 – tan.tanβ
sin x cos y – cos x sin y a–b
3
–p p = cos30º =
 Using componendo & dividendo 2
=
wwM

1– q q –1 rule

Rakesh Yadav Readers Publication Pvt. Ltd. 234

For More Visit : www.LearnEngineering.in


For More Visit : www.LearnEngineering.in

CHAPTER

MAXIMUM AND MINMUM VALUE


13
OF TRIGONOMETRIC FUNCTIONS
Limit of the values of TYPE-I Sol. option (d) is incorrect because
Trignometric Functions:
Ex.1 Choose the correct statements 2

ir
Min Max cosec θ = ,
among following. 3

v.iSn
sin θ & cos θ –1 +1 (a) cos40º> cos70º 3
(odd power) (b) sin35º> sin65º it means sin θ = , it is not
2
sin² θ & cos² θ 0 +1 (c) tan45º<tan46º possible
(even power) (d) cot40º< cot39º

dnag
the refore s in θ m axim um
tan θ & cot θ – + (e) sec20º>sec40º
value is 1
(odd power) (f) cosec20º<cosec30º
Ex.3Find minimum & maximum
tan² θ & cot² θ 0 Sol. When 0< θ <90º,
 value of 15+sin² θ

eYrai
(even power) (a) Value of cos θ decreases as
θ increases, hence, Sol. sin²
sec θ & cosec θ – +
(odd power) 40º <70º  cos40º >cos70º.
The statement is true.
sec² θ & cosec² θ 1  min. max.
(even power)

Note:- The value of sec θ & cosec θ


snhe
(b) As θ increases, value of
sin θ also increase, hence
65º> 35º
0 +1
min. value = 15+0 = 15
max. value = 15+1 = 16
can be anything –  to +  but value  sin65º>sin35º. Hence, the
kgei
of sec θ & cosec θ can't be between statement is false. Ex.4 Find minimum & maximum
– 1 & 1 but it can be – 1 & 1. (c) As θ incre ases value of value of 15–sin² θ
In creasing and Decreasin g Sol.
ERna

tan θ increases, hence sin²


functions : 45º<46º
(i) The value of sin  increases  tan45º <tan46º. Hence,
given statement is true. min. max.
–  0 +1
aBryn

from to . As  increases (d) As θ increases, value of


2 2 max. value = 15–0 = 15
cot θ de creases, hence
in this interval, then sin  min. value = 15–1 = 14
40º>39º Ex.5 Find m in & max value of
also increases.
 cot40º<cot39º. Hence, the
(ii) Ist quadrant : sin  increases 10+sec² θ
statement is true.
Les

from 0 to 1; cos decreases Sol. sec²


(e) As θ increases, value of
wa.th

fro m 1 to 0 and t an


sec θ increases hence 20º
increases from 0 to  . <40º min. max.
(iii) 2nd quadrant : sin  decreases  sec20º <sec40º. Hence, the +1 
min value = 10+1 =11
from 1 to 0; cos decreases statement is false.
wwM

max. value = 10+  = 


from 0 to -1 and t an (f) As θ increases, value of
* maximum value will not be
decreases, from  to 0. cosec θ decreases, Hence asked & equal to 
(iii) 3 rd quadran t : sin  20º<30º Ex.6 Find minimum & maximum
decreases from 0 to -1; cos  cosec20º>cosec30º. Hence, value of 10 sin θ –1
increases from -1 to 0; t an given statement is false. Sol.
Ex.2 Which statement is incorrect sin 
increases from 0 to  .
(iv) 4th qu adrant : sin  3 1
(a)sin θ = (b) cos θ =
increases from -1 to 0; cos 4 3 min. max.
increases from 0 to 1; t an –1 +
2 max. value = 10 ×(1) –1 = 9
decreases from  to 0. (c) sec θ = 4 (d) cosec θ =
3 min. value = 10×(–1) –1 = –11

Rakesh Yadav Readers Publication Pvt. Ltd. 235

For More Visit : www.LearnEngineering.in


For More Visit : www.LearnEngineering.in

Ex.7 Find minimum & maximum Sol. max = –14


13 3 3
value of min = – 21  cos θ – sin θ +5
(ii) when angle is different & 2 2
12sin² θ –3?
Independent then. max. value
Sol. 12 sin² θ –3
max. value = 12(1) –3 = 9 2
13   
2

min value = 12 × (0) – 3 = – 3   3 3 


=    +5
Ex.8 Find minimum & maximum 2  2  
value of 11+cos² θ Ex .13 Find the m axim um and
Sol. 169 27 14
cos² minimum value of 7sin² + 20 =  +5 = +5 = 12
4 4 2
cos² β

r
2 2
Sol. 7sin² + 20 cos² β 13   
3 3 
max. min.

i
min. value = –    

.iSn
+1 0
Now, 2  2  
max. = 11+1 = 12 max. = 7×1+20×1 = 27
min. = 7×0 + 20×0 = 0 +5 = –7+5 = –2
min. = 11+0 = 11
Ex .14Find the maximum and Ex.17 Find the m axim um and

agv
TYPE – II minimum value of
minimum value of 10tan² θ +
(i) In the expression of 15 sec² θ 16sin. 4sin. 2cos. 128cos

ridn
a sin² θ + b cos² θ . Sol. 10 tan² θ + 15 sec² θ Sol. 16sin. 4sin. 2cos. 128cos
= 10tan² θ + 15 (1+tan² θ )
if a > b if b > a 4 sinθ sinθ Cosθ

eeYa
Cosθ
= 10tan² θ +15 +15 tan² θ  2  . 22
  . 2 . 27 
max = a max = b
= 25tan² θ +15
min = b min = a
 24sinθ . 22sinθ . 2cosθ . 27cosθ
min. value = 25×0+15
Ex .9 find the m axim um and = 15 ( tan² θ min =0)  26sinθ+8cosθ
geisnh Max. Value 6sin θ +8cos θ .
minimum value of 15 sin² θ +10 * When a sec² θ + btan² θ is given
cos² θ . then coefficient of sec² θ is = 62  82 =10
value minimum is equal to a.
Sol. 15 sin² θ + 10 cos² θ Now,
(iii) If expression with a sin θ  b max. value = 210
= 15 sin² θ + 10 (1–sin² θ )
Enak

cos θ min. value = 2–10


= 15 sin² θ + 10 – 10sin² θ Ex.18 Find the m axim um and
max. value = a² + b²
= 5 sin² θ + 10
minimum v alue of 27sin x ×
min. value = – a² + b²
R

= 10 + 5sin² θ
Ex .15Find the maximum and 81cos x
(0 <sin² θ <1 )
minimum value of 5sin θ + Sol. 27sin x × 81cos x
aryn

max. = 10+5×1 = 15
min. = 10 + 5×0 = 10 12cos θ  (33)sinx× (34)cosx
Sol. 5 sin θ +12cos θ
Les B

Alternate:-  33sin x × 34cos x


2 2
15 sin² θ +10cos² θ maxi. value = =
5  12  33sin x 4cos x
max. = 15 (use above identity)
25  144 = 13 max. value = 3sinx + 4cosx
min. = 10
2 2
wa.th

Ex .10 f ind the maximum and 2 2 = 3  4 =5


mini. value = – 5  12 =–
minimum value of 4sin² θ + 7 so, max. value = 3 5
13
cos² θ +5 Ex .16Find the maximum and min. value = 3–5
Sol. 4 sin² θ + 7 cos² θ +5 minimum value of 5cos θ + 3 Ex.19 find maximum & minimum
wwM

min = 4+5 = 9 value of 5sin²x – 12sinx cosx


cos( θ +60º)+5 +10cos²x
max = 7+5 = 12 Sol. 5cos θ + 3 cos ( θ +60º) +5 Sol. 5sin²x – 12sinx cosx +10cos²x
Ex .11 Find the m axim um and  (4sin²x – 12sinx cosx + 9 cos²x)+
 5cos θ +3 (cos θ cos60º – sin θ
minimum value of sin60º)+5 cos²x + sin²x
12 sin² θ –17 cos² θ 1  (2sinx–3cosx)²+1
Sol. 12 sin² θ + (–17) cos² θ  5cos θ + 3 (c os θ × –sin θ for minimum value of 2 sinx –
2
max. = 12 3cosx = 0
min = –17 3 then,
)+5 minimum value = 1
Ex .12 Find the m axim um and 2
for maximum value
minimum value of 3 3 3
–14sin² θ – 21cos² θ  5cos θ + cos θ – sin θ +5 2sinx – 3cosx =
2 2
=
2 2 2  3 13

Rakesh Yadav Readers Publication Pvt. Ltd. 236

For More Visit : www.LearnEngineering.in


For More Visit : www.LearnEngineering.in

2
maximum value = 1 + 3 1 1
 13 = 1–
4
×4 sin² θ cos² θ maxi. =
24
×(1) = 4
2
= 1 + 13 = 14
Ex.20 Find the minimum & maxi- 3 1
= 1– (2sin θ cos θ )² mini. = ×0 = 0
mum value of 10 s in θ . 4 24
cos θ +1–2sin² θ 3 (sinn θ when n even max.=1,
Sol. 10 sin θ . cos θ +1–2sin² θ = 1– sin²2 θ . min.=0)
4
Ex.25 Find the minimum & maxi-
 5×2 sin θ cos θ +1–2sin² θ
3 mum value of
 5sin2 θ +1cos2 θ max. value = 1– ×(0) = 1
4 sin³ θ cos³ θ
max. = + 52 +12 = + 26
1 1

ir
3 1
min. value = 1– ×(1) = Sol. max = 
min. = – 26 4 4 23 8

v.iSn
Alternate:- –1 –1
TYPE-III min. =  (3 - odd power)
sin6 θ + cos6 θ . 23 8
In expres sion of sin² n θ + maxi. value = 1 Ex.26 Find the minimum & maxi-

dnag
cos²n θ . 6 6 mum value of sin110 θ . cos110 θ .
 1   1 
maximum value = 1 mini. value =  
 +
 
 1
 2  2 Sol. max =
minimum value = put θ = 45º 2110

eYrai
1 1 1 min = 0 (even power)
Ex.21 Find the minimum & maxi- = + =
8 8 4 TYPE-V
mum value of sin4 θ + cos4 θ º
Ex.23 Find the minimum & (A) a tan² θ + b cot² θ
Sol. sin4 θ +cos4 θ maximum value of sin² or
= (sin² θ + cos² θ )² – 2sin² θ cos² θ
= 1–2sin² θ cos² θ
multiply & divide by 2
θ +cos4 θ .
snhe
Sol. sin² θ +cos 4 θ
max. value = 1
a tan² θ +
b
tan²θ
min. value = 2 ab
kgei
2 2 min. value put θ = 45º
= 1– sin² θ cos² θ max. value = 
2  1 2  1 4
=  
 
 
 Proof
1  2  2
ERna

= 1– (2 sin θ cos θ )² a tan² θ + b cot² θ


2
1 1 3 = ( a tan θ –
=   b cot θ )² +2
1 2 4 4
= 1 – sin²2 θ . a b tan θ cot θ
2 TYPE-IV
aBryn

= ( a tan θ – b cot θ )²+2 ab


1 In the expression of sin n θ .
maxi. value = 1 – ×(0) = 1 But ( a tan θ – b cot θ )² is either
2 cosn θ .
0 or greater than zero.
1 1 1
mini. value = 1 – (1) = maxi. value =  a tan² θ + b cot² θ  0+2 ab
2 2 2n
Les

Alternate:- min. value = or, a tan² θ + bcot² θ  2 ab


wa.th

(i) When n is even Since value of a tan² θ + b cot² θ


sin4 θ +cos4 θ then min. value 0
maxi. value = 1 is greater than or equal to 2 ab ,
for minimum value = θ = 45º (ii) When n is odd its minimum value is 2 ab
mini. value = sin445º + cos445º  –1
wwM

Note- Do not write the given expres-


then min value   n
 1 4  1 4 1 1 1 2  sion as ( a tan θ + b cot θ )² –
=  
   
 = 44  2 Ex.24 Find the minimum & maxi-
 2  2 2 ab tan θ cot θ .
Ex.22 Find the minimun & maxi- mum value of sin4 θ .cos4 θ
In this situation m inim um
Sol. sin4 θ .cos4 θ .
mum value of sin6 θ + cos6 θ . value of, a tan θ + b cot θ
Sol. sin6 θ +cos6 θ 1 24 cannot be zero.
= sin4 θ .cos4 θ
= (sin² θ )³ + (cos2 θ )³ 24 Ex.27 Find the minimum value of
= (s in² θ +cos² θ )³ – 3 sin² θ 1 16 tan² θ + 9 cot2 θ .
= (2sin θ . cos θ )4
cos² θ (sin² θ +cos² θ ) 24 Sol. 16 tan² θ + 9 cot2 θ
= 1–3sin² θ cos² θ 1 min. value = 2 ab = 2 16×9
= sin42 θ
multiply & divide by 4 24 = 2×12 = 24

Rakesh Yadav Readers Publication Pvt. Ltd. 237

For More Visit : www.LearnEngineering.in


For More Visit : www.LearnEngineering.in

Ex.28 Find the minimum value of Sol. 16sin² θ + 9 cosec² θ Ex.36 Find the minimum value of
4tan² θ + 25 cot² θ cos² θ + sec² θ
min. value = 2 16×9
Sol. 4tan² θ + 25 cot² θ Sol. cos² θ + sec² θ
= 2×12 = 24
min. value = 2 4×25 min. value = 1+1 = 2
( a>b min value = 2 ab ) (D) In the expression of
= 2×10 = 20
Ex.29 Find the minimum value of Ex.33 Find the minimum value of a sec² θ + b cosec² θ
4 sec² θ + 25 cosec² θ 25 cosec² θ + 25 sin² θ . 2

Sol. 25 cosec² θ + 25 sin² θ min. value =  a+ b 


Sol. 4 sec² θ + 25 cosec² θ
= 4 (1+tan² θ ) + 25 (1+cot² θ ) mini. value = 2 ab maxi. value = 
= 4 + 4tan² θ + 25 +25 cot² θ Ex.37 Find the value of 4sec² θ + 9

r
2 25×25
= 29 + 4 tan² θ + 25 cot² θ cosec² θ .
2×25 = 50

i
.iSn
min. value = 4 tan² θ +25 cot² θ 2
or a b
= 2 4×25 = 20 mini. value = a+b = 25+25 = 50
Sol. mini. value =   =
2
min. value = 29+20 = 49 Ex.34 Find the minimum value of 4 9
 

agv
Ex.30 Find the minimum value of sin² θ + cosec² θ
tan² θ + cot² θ . = (2+3)² = 25
Sol. sin² θ + cosec² θ
Ex.38 Find the minimum value of

ridn
Sol. tan² θ + cot² θ min. value = 2 11 = 2 sin² θ +cosec² θ +cos² θ +sec² θ
min. value = 2 ab or +tan² θ +cot² θ .

eeYa
= 2 1×1 = 2 min. value 1+1 = 2
Sol. sin² θ +cosec² θ +cos² θ +sec² θ
(C) In expression of
(B) in expression of +tan² θ +cot² θ
a cos² θ +b sec² θ
a sin² θ + b cosec² θ = sin² θ +1 +cot² θ +cos² θ +
(i) If a  b
(i) if a  b
geisnh 1+tan² θ +tan² θ +cot² θ
minimum value = a+b
min. value = a+b  2+sin² θ +cos² θ + 2 tan² θ +2
(ii) If a  b
(ii) if a  b cot² θ
minimum value = 2 ab
min. value = 2 ab = 2+1+2 tan² θ +2 cot² θ
Enak

Ex.31 Find the minimum value of Ex.35 Find the minimum value of = 3 +2 tan² θ + 2 cot² θ
4sin² θ + 25 cosec² θ 10cos² θ + 15 sec² θ min. value of 2 tan² θ +2 cot² θ
Sol. minimum value = 4+25 = 29 Sol. 10 cos² θ +15 sec² θ = 2 ab
R

( a<b min value = a+b) minimum value = 10+15 = 25


Ex.32 Find the minimum value of = 2 2×2 =4
aryn

16 sin² θ + 9 cosec² θ ( a<b min. value = a+b) min. value = 3+4 =7


Les B

EXERCISE

1. Find the maximum value of (a) equal of 1/3 8. The gre ates t value of
sinx + cosx. (b) less than 1/3
wa.th

4 4
(c) greater than 1/3 sin   cos  is :
3 (d) not known
(a) 2 (b) (c) 2 (d) 3 1 3
2 5. The maximum value of 24 (a) 1 (b) (c) 0 (d)
sin  + 7cos  is 2 2

For 0 
wwM

2. the inequality (a) 7 (b) 17 (c)24 (d) 25 9. The least v alue of


2 6. Which of the following is 2
which holds is : 2 sin ²   3 cos  is:
correct?
(a)  sin t an  (a) sin1°<sin1 1
(b) sin t an (B)sin1°=sin1 (a) (b) 1 (c) 2 (d) 3
2
(c)  tan sin  (c) sin1°>sin1
10. What is the minimum value of
(d) t an sin 
(d) sin1°= sin 1 sin ²   cos 4  :
3. Which is smaller ? 180
(a) sin 76  (b) cos 76  7. The greatest value of 3 1
(c) both are equal (a) 0 (b) (c) 2 (d)
cos( x e x  7 x ²  3 x ) , x [ 1,  ) is 4 4
(d) None of these
(a) 0 (b) 1 (c)–1 11. Find the maximum value of
4. If the sine of an angle is 1/3,
then cosine of that angle is : (d) None of these sin¹¹³  . cos¹¹³  :

Rakesh Yadav Readers Publication Pvt. Ltd. 238

For More Visit : www.LearnEngineering.in


For More Visit : www.LearnEngineering.in

113 19. If P = sin220º +sin2 40º +sin2 50º 24. sin x  3 cos x is maxim um
3  + sin2 70º, then.
(a)   (b) 1 when
2  (a) 0 < P < 1 (b) 1 < P < 1.5
(a) x = 30° (b) x = 0°
113 113 (c) 1.5 < P < 2 (d) P = 2
1  1  (c) x = 45° (d) x = 60°
(c)   (d)   
4 2  20. If 0    , then which of the 25. The least value of tan²+cot²  is:
2
12. Find the minimum value of 1
following is true? (a) 2 2 (c) 3 (d) 3
(b)
16cosec²  + 25sec²  :
(a) (tan²  +cot²  )  2 26. What is the minimum value of
(a) 81 (b) 41 (c) 82 (d) 90
13. The le ast value of sin²  (b) (tan²  +cot²  )  2 sin 6   cos 6  ?
(c) (tan²  +cot²)  1 1 1

ir
+cosec²  + cos²  + sec²  is:
(a) (b)
(c) 1
(a) 3 (b) 4 (c) 5 (d) 6 (d) None of these 2 4

v.iSn
 (d) None of these

14. If 0    , then which of the 21. If 0 , which of the 27. The gre ates t value of
2 2 sin x cos x
following trigonometric ratios 81 . 27 is :
following is true?

dnag
can have the value 1.1? 1 (a) 3 5 (b) 3 4 (c) 3 (d) 33
(a) Sin  (b) Cos  (a) sin ²   2
sin ²  28. If 0   A  90  and cosA-sinA
(c) 2Sec  (d) 2tan  1
(b) sin ²   2 > 0 then

eYrai
15. Cos3  +sin3  is maximum sin ²  cosA + sinA can not be greater
when  is: 1 than :
(c) sin ²   2
(a) 15° (b) 30° (c) 45° (d) 60° sin ²  1 1 1
16. The value of x, for maximum (d) None of these (a)
3
(b)
2
(c)
2
(d) 2
value of (sinx+ cosx) is :
(a) 30° (b) 45° (c) 60° (d) 90°
17. The maximum value of sinx.
22. The least snhe v alue of
(4 sec ²   9cosec² ) is
29. In a ABC , if angle C is obtuse,
then
(a) tanA.tanB<1
cosx is: (a) 1 (b) 19 (c)25 (d) 7
kgei
(b) tanA.tanB  1
1 23. If     90  , then the maximum (c) tanA.tanB>1
(a) 2 2 (c) 2 (d) 1
(b)
(d) None of these
value of sin  . sin  is:
ERna

18. The maximum and minimum 30. The ratio of the greatest value
value of (1+ cos 2x) are: 1 3 of 2 - c o s x + s in ² x to its
(a) -1 and 1 (b) 1 and 2 (a) 1 (b) (c) least value is :
2 2
1 1 1 9 13 7
aBryn

and (d) None of these


(c)  (d) 0 and 2 (a) (b) (c) (d)
2 2 4 4 4 4

ANSWER KEY
Les

1.(a) 4.(c) 7.(b) 10.(b) 13.(c) 16.(b) 19.(d) 22.(c) 25.(a) 28.(d)
2.(b) 5.(d) 8.(a) 11.(d) 14.(d) 17.(c) 20.(a) 23.(b) 26.(b) 29.(a)
wa.th

3.(b) 6.(a) 9.(c) 12.(a) 15.(a) 18.(d) 21.(c) 24. (a) 27.(a) 30.(c)
wwM

Rakesh Yadav Readers Publication Pvt. Ltd. 239

For More Visit : www.LearnEngineering.in


For More Visit : www.LearnEngineering.in

SOLUTION

1.(a) Maximum Value of : = (sin ²   cos ² )2  2 sin ² . cos ²  12.(a) 16 cosec²   2 5 sec ² 
a sin x  b cos x  a²  b² = 1  2 sin ² . cos ²   1 = 16(1  cot ² )  25(1  tan ² )
here a = b = 1 Greatest value of sin 4 cos 4   1 = 41  16 cot ²   25 tan ² 
Maximum Value of : (i)
9.(c) 2 sin ²   3 cos ²  = Now,
sin x  cos x  1²  1²  2
2(sin ²   cos2 )  cos ²   M ini mum val ue of
Alternate
= 2  cos ²   2 b

r
 1 1
 ax ²   2 ab
sin x + cos x =  2 sin x  2 cos x  2 x²

i
.iSn
  Least value of 2 sin ² +
3cos²= 2 Minimum value of :
=(sinx.cos45°+cosx. sin45°) 2
16 cot ²   25 tan ²   2 16  25  40
10.(b) x  sin ²  cos4 

agv
= 2 sin (x + 45°) From (i)
= 1  cos ²   cos4  Minimum value of
Now, 1  sin(x  45 )  1
x = 1  cos ² (1  cos ² ) 16 cos ec²   25 sec ²   41  40  81

ridn
1  2  2 sin(x  45 )  1  2 Short Cut :
x = 1  cos ²  . sin ² 
Minimum value of
  2  (sin x  cos x )  2

eeYa
1 (acosec²   b sec ²  )
2.(b) In Ist quadran t, x =1  (2 sin . cos )²
4
sin t an true.
is = ( a  b )²
1
3.(b) In Ist quadran t as  x = 1  sin ² 2  0  sin ² 2   1  Minimum value of
geisnh 4
increases, the value of sin ( 16cosec²  25 sec ² )
increases. 1
when sin ² 2  0  x  1  (0)  1 = ( 16  25 )²  (9)²  81
Now, cos 76° = cos (90°–14°) = sin 4
14° and when 13.(c)
Enak

out of sin 76° and sin 14°, the (sin ²  cos ² )  (cos ec²   sec ² )
1 3
smaller one is sin14° sin ² 2   1 ,  x  1  (1) 
Hence, cos 76° is smaller. 4 4 = 1  (cos ec²   sec ² )
3  Minimum value of
R

4.(c)  x 1
i.e.
4
1 8 1 (a cos ec²   b sec ² )  ( a  b)²
cos   1  sin ²   1    i.e the least value of
aryn

9 9 9  Minimum value of
3
1 sin ²  cos4  
(cosec²   sec ² )  ( 1  1)²  4
Les B

So, cos   4
3
11.(d) sin 113
 . cos 113
  Minimum value of the given
5.(d) Max. value of a sin  + b cos  Exp. =1 + 4 = 5
1 113
= a²  b² = 113 (2 sin . cos ) 14.(d) Clearly, 0  s i n   1 and
2
wa.th

Max. value of 24 sin  +


113 113 0  c o s  1
7cos 1  113 1 
=   (sin 2  )   
2
  2 
= (24)² (7)² 25 for 0   
( 1  sin 2   1) 2
wwM

6.(a) Since 1 radian = 57°16'22''


Hence, the greatest value of
approx, Also sec θ  1 so 2sec θ  2
113
and sin 57°16'22'' >sin1°, 1 
sin 113  . cos113    
sin 1°<sin1 2  Now, tan  may have the
7.(b) since cos   1 for all real  . Short Cut : value 0.55
the given expression  1 for The maximum value of
n
 2 tan  may have the value
all x n 1 
n 1.1.
Hence, the greatest value=1 sin  . cos    
2 
8.(a) The maximum value of
15.(a) cos 3   sin 3 
sin 4  cos 4 = (sin ² )²  (cos ² )² + 1 
113  1 1 
sin 113 . cos113     = 2 cos 3   sin 3  
2  2 2 
2 sin ² .cos ²   2 sin ² .cos ² 

Rakesh Yadav Readers Publication Pvt. Ltd. 240

For More Visit : www.LearnEngineering.in


For More Visit : www.LearnEngineering.in

2
= 2 (sin 45  cos 3   1   (si n ²   cos ²  ) 
2

=  sin    2 2 =
   2 2
 cos 45 . sin 3 )  sin    si n . cos  
= 2 sin(45   3 )    Least value of tan² +cot² = 2
The maximum value occurs  0    2  sin   1 Alternate:-
when 22.(c) least value of 1
t an ²   cot ²   tan ²  
sin (45   3 )  1 (a sec ²   b cosec² )  ( a  b )²
tan ² 

i.e. 3   45     15   least value of b


It is in the form of a x ² 

ir
16.(b) sin x + cos x (4 sec ²   9 cos ec² ) x²

v.iSn
 1 1  Where a = b = 1
= 2 sin x  cos x  = ( 4  9 )² = (5)²=25
 2 2   minimum value
= 2  sin x. cos 45   cos x sin 45   23.(b) Let y = sin . sin  = 2 ab  2 1  1  2
1

dnag
6 6
= 2 sin  x  45   = (2 sin  . sin ) 26.(b) x = s i n   c os 
2
= 2 , wh en x  45   x = (sin ²  )³  (cos ²  )³
1
 max. = [2 sin  . sin (90    )]  x = (sin ²   cos ² )

eYrai
2
4 4
(sin x  cos x )  2 1 (sin   cos   sin ² . cos ² )
= (2 sin  . cos  ) x = 1  [(sin ²   cos ² )2  3 sin ² . cos ² ]
1 2
17.(c)sin x . cos x= 2 sin x. cos x 

=
1
2

sin 2 x 
1
y
snhe
=
1
OR

(sin 2  )
x = 1  3 sin ² . cos ² 

x =1
3
(2 s i n . c os  )²
2 2 2 4
kgei
[ 1  sin 2 x  1] Si nce, Max imu m value of 3
x = 1 (sin 2 )²
1 sin 2 is 1. 4
ERna

Hence max. value of sinx. cosx is .


2 1 3
18.(d) We know that  Maximum value of y  = 1 sin ²2 
2 4
1  cos   1
 0  sin ²2   1
aBryn

24. (a) Let y = sin x  3 cos x


 1  cos 2x  1 OR
 at sin ²2   0
1  1  1  cos 2x  1  1 1 3 
 0  1  cos 2 x  2 = 2 sin x  cos x  3
2 2  x 1 (0 )  1
4
19.(d)P = (sin ²20   sin ²70 ) 
Les

= 2 sin (x + 60º)
and at si n ²2   1
wa.th

(sin ²40   sin ²50 ) y will be maximum, when sin


(x + 60°) maximum 3 1
= (sin ²20   cos ²20 ) + x 1 (1) 
i.e. sin (x + 60°) = 1 = sin
4 4
(sin ²40   cos ²40 ) 90°  x + 60° = 90°
wwM

= 1+1 = 2  x = 30º 1
i.e.  x 1
20.(a) tan ²   cot ²  = 25.(a) t an ²   cot ²  4
tan ²   cot ²   si n ²  c os ² 
i.e least value of
2 tan . cot   2 tan .cot  =
cos ² 

sin ²  6 6 1
sin   cos  
= (tan   cot )²  2  2 4
4 4
1 sin   cos  27.(a)
21.(c) sin ²   =
sin ² . cos ²  sin x
sin ²  81 . 27 cos x  3 4 sin x . 3 3 cos x
1 (sin ²   cos ² )²  2 sin ²  cos ²  = 34 sin x 3 cos x
= sin ²   2 2 =
sin ²  sin ² .cos ²  For maximum value,

Rakesh Yadav Readers Publication Pvt. Ltd. 241

For More Visit : www.LearnEngineering.in


For More Visit : www.LearnEngineering.in

4 sinx + 3cos x must be maximum tanA + tanB


 2
and maximum value of :  1 - tanAtanB
= -tanC  1
  cos x    3
4 sin x  3 cos x = 2 4
 angle C is obtuse  
= 4²  3² = 5  tan C<0
 Greatest value of tanA + tanB 2
>0 (i) 13  1
sin x cos x 5    cos x 
81 . 27 3 1 - tanA + tanB =
4 

2
28.(d) cosA - sinA > 0 But since C is obtuse angle,
so A and B will  Max. value occurs at
sinA < cosA
tanA < 1  A < 45° both be less than  1

r
2 cos x  
 sin A + cos A < sin 45° + 2

i
.iSn
cos45°  Both tanA and tanB are
positive. 13
1 1 2 and it is
= + = = 2 Hence, from 4

agv
2 2 2 (i) 1–tan A . tan B > 0
and Min. value occurs at
i.e. sin A + cosA = 2  tan A .tan B <1
cos x  1 and it is 1

ridn
29.(a) since A+B+C=  30.(c) 2 – cos x + sin² x
= 2–cosx +1– cos² x 13
 A+B =  – C  The required ratio is .

eeYa
 tan(A+B)=tan(  –C) = -(cos² x + cos x ) + 3 4
geisnh
Enak
R
aryn
Les B
wa.th
wwM

Rakesh Yadav Readers Publication Pvt. Ltd. 242

For More Visit : www.LearnEngineering.in


For More Visit : www.LearnEngineering.in

CHAPTER

14
HEIGHT & DISTANCE
INTRODUCTION T the angle of elevation of the
One of the important application of source of light is :-

ir
trigonometry is in finding the (a) 400 (b) 300 (c) 600 (d) 900

v.iSn
height and distance of the point BC 1
which are not directly measurable. θ P Sol. t an     t an 30 0
AB 3
This is done with the help of
trigonometric ratios.    30 0

dnag
Angle of elevation O
Let O and P be two point where P is Some useful point C
at a higher level than O. Let O be at In this chapter we solve all the
the position of the observer and P questions with the help of ratio.

eYrai
be the position of the object. Draw a So me i mportant ratios are as len gt h
horizontal line OM through the point following :-
O. OP is calle d the l ine of TYPE -I 
B
A sh adow
observation or line of sight, Then
POM   is called of elevation of P
as observed from O.
(A)
snhe B Ex.2 The angle of elevation of the
top of a tower at a distance of
P 2 500 m from its foot is 300.
kgei
(Object) 1
The height of the tower is :-
on
vati 300
O
bs
er A 500  3 –1 m
ERna

3 (a)
ofo 3
e
LinAngle of elevation Angle Ratio of 30º =
(b) 500 m

O M Base : Height : Hypotenuse
aBryn

(Observer) Horizontal line :


Angle of Depression 3 : 1 2
500 3
Proof: (c) m
In the above figure, if P be at a 3
lower level than O, then MOP   In  OBA
is called the angle of depression.
500  3 1 
Les

OB (d)
(Observer) Tan30º= 3
Horizontal line OA
wa.th

O M
 1 OB BC h
Lin Angle of depression = Sol. t an 30 0  
eo 3 OA AB 500
fs OB : OA
igh
wwM

t 1 500 3
1: 3  h  500  
3 3
2 2
(Object) AB² = (OB)² + (OA)² =  3  1 Alternative
P In 30° :- height : Base
Angle subtended by a line at a point AB = 3  1  2.
In the adjacent figure suppose OT 1 : 3
is a tower, where O is the foot and T OB:OA :AB but Base = 500 (given)
is the top of a tower. Suppose P is a  
poi nt any wh ere in th e space 1: 3 : 2  3  500
(including ground). Join O – P and T
Ex.1 If the ratio of the length of a 500 500. 3
– P, then  OPT = θ is the angle  1 
subtened by tower OT at point P. pen to its shadow is 1 : 3 , 3
3

Rakesh Yadav Readers Publication Pvt. Ltd. 243

For More Visit : www.LearnEngineering.in


For More Visit : www.LearnEngineering.in

C i.e. Required distance (BC Ex.6 The angle of elevation of a


ratio value = moon when the length of the
3)
shadow of a pole is equal to
h = x = 10 3 m its height is :-
(a) 300 (b) 450 (c) 600 (d) 900
(B) B
Sol.
B
0
30
A B
500 m
Ex.3 The foot of a ladder leaning 2 1
against a wall of length 5
metre rest on a level ground x
A O

r
5 3 metre from the base of 1

i
the wal l. The angle of Angle Ratio of 45º

.iSn

inclination of the ladder with C A
= Base : Height : Hypotenuse x
the ground is :- 1 : 1 : 2
(a) 600 (b) 500 (c) 400 (d) 300 Let AB  x , then AC=x

agv
Proof:
Sol. In right angled ABC, AB x
In  OBA  t an    1
AC x

ridn
AB 5 1
t an      tan 30 0 OB
BC 5 3 3 tan45º=    45 0
OA
Ex.7 The banks of a river are

eeYa
   30 0 parallel. A swimmer starts
OB
1= from a point on one of the
A OA
banks and swims in a
 OB : OA straight line inclined to the
wall
geisnh 1 : 1 bank at 450 and reaches the
la

opposite bank at a point 20


5m
dd

 AB² = (1)² + (1)² m from the point opposite to


er

th e starti ng poin t. The


 AB = 2 breadth of the river is :-
Enak

 (a) 20 m (b) 28.28m (c)


B C
5 3 OB:OA :AB 14.14 m (d) 40 m
 1 : 1 : 2 
R

Sol. A
Ex.4 A ladder is resting against a
wall at a height of 10m. If the Ex.5 The angle of elevation of the
aryn

0
45
ladder is inclined at an angle top of a tower at a distance of 20
of 30º with the ground the 30 m from its foot is 450. The m
Les B

distance of the ladder from height of the tower is:-


the wall is: (a) 20 m (b) 30 m (c)
10 20 15
m m C B
(a) (b) 15 2 m (d)
2
m
wa.th

3 3
B Let A be the starting point
(c) 10 3 m (d) 20 3 m Sol. and B, the end point of the
swimmer. Then AB=20 m &
Sol.
BAC  45 0
wwM

A A 0 BC
Now, sin 45 
45
0

A
AB
C 30 m
10 1 1 BC
Base : height =1:1    BC  10 2  14.14 m
2 20
30º 30º  h eight  Base  30 m B
xm Alternate:- (C)
C B C 3 B
( let )
(original value) ( ratio value) AB
In ABC, t an 45 0  2
3
Ratio value AC
Original value
AB
AB  1  10 m 1   AB  30 m
30 A 1 O
 BC  3  10 3 m

Rakesh Yadav Readers Publication Pvt. Ltd. 244

For More Visit : www.LearnEngineering.in


For More Visit : www.LearnEngineering.in

Angle Ratio of 60º = Sol. C (a) 50 3 m

Base : Height : Hypotenuse (b) 25 3 m


1 : 3 : 2 (c) 25 m
h
Proof: 25
(d) 3 m
In  OBA 60
0
Sol. C
OB A B
tan60º= 10 3 m
OA
h
OB tan60º =
10 3

ir
3=
OA

v.iSn
h
 OB : OA 3 = 10 3 60
0

A B
3 : 1 h = 30 m. Let C be the position of the
 AB² = (OB)² + (OA)² Ex.10 One flies a kite with a thread kite and AC be the string.

dnag
150 metre long. If the thread
2 2  AC  50 m and
=  3  1 of the kite makes an angle
of 60° with the horizontol BAC  60 0
 AB = 3  1  2. line, then the height of the

eYrai
ki te from the grou nd BC BC 3
  sin 60 0  
OB:OA :AB (assuming the thread to be AC 50 2
in a straight line) is:-
 3 : 1 : 2  (a) 50 metre  BC  25 3 m

Ex.8 A tower is 50 3 meters


hi gh. Find th e an gle of
(b) 75 3 m snhe
(c) 25 3 metre
Hence, Height of the kite
= 25 3 m
Alternate :-
elevation of its top from a
kgei
(d) 80 metre In 60° AC : BC
point 50 meters away from its Sol.
foot:-
A A =2 : 3
(a)   60 0 (b)   45 0
ERna

 25  25
0 m 2
(c)   30 0 (d)   22 1 0 5025 3
2 15 h 3
Ex.12 The angle of the sun at any
60° 60°
aBryn

Sol. B distance is 600. The height


C B C 1 B
of the vertical pole that will
(original value) ( ratio value) cast a shadow of 40 m is :-
50 3 AC = length of thread, let
height = h m 40
(a) 20 m (b) m
 3
Les

C A Ratio value Original value


50m
wa.th

(c) 40 3 m (d) 20 3 m
Height of tower  AB   50 3 AC 2 150
Sol. B
AB 50 3  1 150 = 75
 tan     3  tan 60 0 2
wwM

AC 50
 75 3
3 Pole
   60 0
Ex.9 If the angle of elevation of i.e. the height of the kite
the top of a building from a = AB = h = 75 3 m 0
60
point 10 3 m away from its C A
Ex .1 1 The len gth of a stri ng 40 m
base is 600, the height of the between a kite and a point on
building is:- the ground is 50 m. The string Let height of pole AB = h m
(a) 10 m (b)20 m makes an angle of 600 with h
 tan 60 0 
10 the level ground. If there is no 40
(c) m (d)30 m slack in the string, the height
3
of the kite is:-  h  40 3 m

Rakesh Yadav Readers Publication Pvt. Ltd. 245

For More Visit : www.LearnEngineering.in


For More Visit : www.LearnEngineering.in

Alternative : Ex.13 The shadow of an electric pole Sol.


In 60° = Base : height standing on a ground is 40 m
less when the angle of A A
1 : 3 elevation changes from 300 to
450. The length of the pole is:-
 40  40 1
40 40 3 (a) 20  
3 1 m
45° 30°
45° 30°
(b) 20  3  1 m B D 20m C B 1 D 3 –1C
 height  40 3
(c) 20 m (original value) (ratio value)
TYPE -II (d) 20 3 m Let AB be a pillar of height

r
(A)
Sol. (By ratio) h metre.

i
.iSn
B A Ratio value original value
CD  3 –1 20

agv
20
 1 ×
1 3 1
h m

ridn
3 1
3 1

eeYa
A D 1 O 0 0
 3 1  30 45 height of the pillar =10
D B
40 m C
 
3  1 metre
(original valu e)
Proof: In  AOB Ex.15. The length of the shadow of a
geisnh
OB A vertical tower on level ground
tan30º= inc reas es b y 10 metres
OA
when the altitude of the sun
1 OB changes from 45° to 30°.
=
Then the height of the tower is
Enak

3 OA
 OB : OA 1
(a) 5 3  1 metres
1 : 3
R

.......... (i) (b) 5  3 – 1 metres


0 0
OBD 30 45
(c) 5 3 metres
D C 1 B
aryn

OB  3  1 5
tan45º= (d) metres
OD (r at io valu e) 3
Les B

Sol.
OB
1=
OD
  3 1   40 m
OB : OD
40  3 1   20
1 : 1   3 1 
wa.th

1 
........ (ii) 3 1  3  1
From (i) and (ii)
To make equal ratio  length of pole = 20( 3  1) AB = height of tower
OB : OA Ex.14 If the angle of elevation of In ΔABC
wwM

OB : OD
the Sun changes from 30° to AB
1 : 3 1 : 1 45°, the length of the shadow tan45° =
BC
OA : OB : OD of a pillar decreases by 20
AB
3 : 1 metres. The height of the  1= = AB : BC = 1 : 1
BC
1 : 1 pillar is:
.....(i)
3 : 1 : 1 (a) 20  3 – 1 m In ΔABD
OB : OA : OD AB
(b) 20  3  1 m tan30° =
BD
1 : 3: 1
then, (c) 10  3 – 1 m 
1

AB
AD = OA – OD 3 BD
AD = 3 – 1 (d) 10  3  1 m  AB : BD = 1 : 3 ....(ii)

Rakesh Yadav Readers Publication Pvt. Ltd. 246

For More Visit : www.LearnEngineering.in


For More Visit : www.LearnEngineering.in

Now, Sol. (d) Let the height of tower be AB height of tower


BC : AB : BD h m and BD = xm In ΔABC
1 : 1 A
AB
1 : 3 tan60° =
BC
1 : 1 : 3 ΑB
CD = BD – BC 3=  AB : BC = 3 : 1 ....(i)
h BC
=  3 1  units = 10m In ΔABD

10 AB
= 1 unit = 45º 60º tan45° =
3 –1 BD
A 1 B
3–1 D
AB
AB = 1unit = 5  3 1 

ir
metres 1 =
Ratio Figure BD
(B)

v.iSn
A  AB : BD = 1 : 1 ....(ii)
B Now
BD : AB : BC
1 : 1

dnag
3 3 : 1

h 3 : 3 : 1

A CD = BD – BC =  3 –1 

eYrai
 3 1  D 1 O 45º 60º
C D B 3 –1
 OAB 20m x   = 60 metre
Both of the figures are in
OB 1 comparison ratio 60
tan45º= = 1 unit =

OB : OA

1 : 1
OA 1

........(i)
 snhe
3 – 1 R = 20

20
AB = 3 units =
3 –1
60
3 –1
× 3

R=
kgei
3 –1 = 30 3  3 m
 
 OBD
20 Ex.18 The angle of elevation of the
OB
tan60º= x=1 R= tip of a tower from a point on
OD 3 –1
ERna

the ground is 45º. Moving 21


OB 3 1 m directly towards the base
3= 20
= 3 – 1
  = 20  3  1 of the tower, the angle of el-
OD 3 1
OB : OD .....(i)   2 evation changes to 60º. What
aBryn

= 10 ( 3 +1) m. is the height of the tower, to


3 :1 the nearest meter?
From (i) and (ii) Height of tower
(a) 48 m (b)49 m
To make equal ratio = AB = 3 10 3 1 (c) 50 m (d)51m
OB : OA OB : OD
  Sol. In  PBC,
1 : 1 3: 1
Les

= 10 3  3 m.
 
OB : OA : OD P
wa.th

Ex.17 The elevation of the top of a


3: 3: 1 tower from a point on the
AD = OA – OD = 3 – 1 ground is 45°. On travelling
Ex.16 The angle of elevation of the 60 m from the point towards h
wwM

top of a tower at a point on the tower the elevation of the


level ground is 45º. When top becomes 60°. The height
move d 20 m towards the of the tower (in metres) is
tower, the angle of elevation 45º 60º
bec omes 60º. What is the (a) 30 (b) 30 3 – 3
  A B C
height of the tower? 21m x
(c) 30 3  3
  (d) 30 3
3 –1 m Sol. h h
(a) 10   A tan60º =
x

x
 3

(b) 10  3  1 m h
 x ....(i)
3
(c) 10 3 – 3  m
h
In  PAC, tan45º = 1
(d) 10 3  3  m 21  x

Rakesh Yadav Readers Publication Pvt. Ltd. 247

For More Visit : www.LearnEngineering.in


For More Visit : www.LearnEngineering.in

 h = 21 + x depression becomes 600. The Ratio value original value


distance (in m) travelled by CD 3 50
h
 h = 21 + [from Eq. (i)] the car during this time is:-
3
50
50 3 1
 1  (a) 50 3 (b) 3
 h
1 – 3 
  21 3
  100
100 3 2
(c) (d) 100 3 3
21 3  3  1 3
 h  .
Sol. In  ADC 100 100 3
 3 – 1  3  1  AB  r at io valu e 
3

3
DC

r
 tan30º =
21 3  3  1 AC Ex.20 The angle of elevation of top

i
.iSn
= of a tree on the bank of a
2 1 50
 = river from its other bank is
= 49.68  50m 3 AC 600 and from a point 20 m
(C)

agv
B AC = 50 3 further away from this is
.......(i) 300. The width of the river is:-

ridn
In  DBC (a) 10 3 m (b) 10 m
50 (c) 20 m (d) 20 3 m
3  tan60º =

eeYa
BC Sol. (By ratio)
50 BC = width of river
60º 30º BC = A
3
O 1 D 2 A
.......(ii)
Proof
geisnh
(i) – (ii)
 OAB
AB  AC  BC
OB h
tan30º =  1 
OA  50  3  
Enak

 3
1 OB 0
30 60
0

= 100
3 OA  100 3 D
20 m C B

3
R

OB : OA .....(i) 3 (ori ginal valu e)


1: 3 D A
aryn

0
 OBD 30
60
0

OB
Les B

tan60º = 3
OD
50 m
OB 0
30 60
0

3 = OD
D 2 C 1 B
wa.th

OB : OD ....(ii) 0
60
0 (Ratio value)
30
3:1 A B C Ratio value original value
From (i) & (ii) Alternate :- ( by ratio ) 2 20 m
to make equal ratio D
1 10 m
wwM

OB:OA OB : OD He nce, width of the


river=BC=10 m
1: 3 3 :1
Ex.21 A man on the top of a rock
OB : OA : OD
3 rising on a sea-sho re
3:3 :1 observe s a boat coming
AD = OA – OD = 3 –1 = 2 towards it. If it takes 20
Ex.19 A man from the top a 50 m 0
mi nute for th e an gle of
B
0
30 60
hig h to wer, see s a car depression to change from
A 2 1 C 300 to 600 , how soon will the
moving towards the tower at
CD=50 m ( given ) boat reach the shore ?
an angle of depression of 300.
After some time, the angle of but CD = 3 ( according to ratio ) (a) 20 minute

Rakesh Yadav Readers Publication Pvt. Ltd. 248

For More Visit : www.LearnEngineering.in


For More Visit : www.LearnEngineering.in

(b) 30 minute Ratio value original value OB


(c) 10 minute 2 20 minute (2– 3 ) = OA
(d) 15 minute 1 10 minute
Sol. D (man)  For distance y (or ratio value
= 1) time taken =10minute  1 
t an 15º 2  3 
 

Ex.22 From the top of a 60 m high  2 3 
tower the angle of depression
h 1 OB
of the top and bottom of a =
0 0 building are observed to be 2 3 OA
30 60 OB : OA .....(i)
300 and 600 respectively. The
A x B y C height of the building is:- 1 : 2+ 3

ir
0 h
From figure, t an 60  y (a) 60 3 m (b) 40 3 m In  BOD

v.iSn
(c) 40 m (d)20 m
OB
 h  y 3............(i ) Sol. Let AB be the tower and PQ tan30º =
the building. OD
0 h
and t an 30  From right angled ABQ, 1

dnag
OB
x+y =
AB 60 3 OD
1 y 3 t an 60 0   3  OB : OD ....(ii)
  BQ x
3 x+y 1: 3

eYrai
 x  20 3
[ using (i)] .......(i) From (i)& (ii)
3y  x + y A to make same ratio
x OB: OA OB : OD
 2y  x  y 
2
 Time taken to cover a
distance from A to B
P
30
snhe
0

C 60 m
1 : 2+ 3
OB : OA : OD
1 : 3
kgei
= 20 minute h 1 : 2+ 3 : 3
 time taken to cover a unit 0 then,
60
distance Q B AD = OA – OD
x
ERna

20 and From right angled APC, = (2+ 3 ) – ( 3 ) = 2


 minute
x AC 1 AC Ex.23 The angles of elevation of the
t an 30 0    top of a tower from two points
 For distance 'y' time taken PC 3 x
A and B ly ing on the
aBryn

20 x
 y horizontal through the foot of
x AC =
3 the tower are respectively 15°
20 x put the value of x in equation and 30°. If A and B are on the
   10 minute same side of the tower and
x 2 20 3
AB = 48 metre, then the
Les

(i) AC = = 20
Alternative (By ratio) 3 height of the tower is:
wa.th

D
 PQ  BC  AB  AC  60  20  40 (a) 24 3 metre
 height of the building  h  40 m (b) 24 metre
h (D)
B (c) 24 2 metre
wwM

0 0
30 60 (d) 96 metre
A x B y C
Sol.
1
20 minute (original value) P P
D

15º 30º
A O 1
2 D 3
3 30° 15° 30° 15°
0 0
In  OAB Q B 48m A Q 3 B 2 A
30 60
B (original value) (ratio value)
A 2 C OB
tan 15º = PQ = Tower = h metre
(Real value) OA

Rakesh Yadav Readers Publication Pvt. Ltd. 249

For More Visit : www.LearnEngineering.in


For More Visit : www.LearnEngineering.in

Type - III P
Ratio Original
value value * If the angles of a tower from
AB 2 48 two points distant a and b from its
foot and in the same straight line h
48
 1 2 from it is complementary each
= 24 metre other then height of tower is. 
B B A a Q
i.e. height of the building
b
= PQ (ratio value = 1) = h
Ex.26 The distance between two
= 24 metre.
h pillars of length 16 metres
Note:- you also can make

r
and 9 metres is x metres. If
oth er ratio for differe nt two angles of elevation of

i
.iSn
angle. 90 –  their respective top from the
A
Alternate:- D a O bottom of the other are
We can use formula for this b complementary to each other
type the n the value of x ( in

agv
h = ab metres) is
* The angle of elevation of the
top of a tower from a point on Ex.25 The angle of elevation of the (a) 15 (b) 16 (c) 12 (d) 9

ridn
the horizontal is θ2 and top of a tower at two points Sol.
moving 'a' distance towards which are at a distance a and

eeYa
b from the foot in the same
the tower it becomes θ1 , the
horizontal line and on the
height of the tower is- same side of the tower, are
complementary. The height
B
geisnh
of the tower is :- If 1 2 = 90° then x  h1  h 2
(h= height of towers)
(a) ab (b) ab
x = 16  9 = 144 = 12 mtr
h
(c) a (d) b
b a
Enak

TYPE - IV
Sol. Let PQ be the given tower of * At a particular time for all object ratio
2 1
A height h. If A, B be given of height shadow are same
D O
R

a x points then suppose. Pole


a
h = cot – cot PAQ   and PBQ   Wall
aryn

2 1
0
     90
Ex.24 The angle of elevation of the
Les B

top of a tower from a point on Now in PAQ, h2


h1
the gound is 30º and moving
70 metres towards the tower h
t an   ..............(i ) x1 x2
it becomes 60º. The height a
wa.th

of the tower is: h1 h2


in PBQ, tan= =
x1 x 2
B
Sol. Ex.27 A vertical stick 12 cm long
h h casts a shadow 8 cm long on
t an    t an  90 0    
wwM

b b the ground. At the same time,


h a tower casts a shadow 40 m
h b long on the g round. The
 cot    tan   height of the tower is
30º 60º b h
A (a) 72 m (b) 60 m
D O
70 h b (c) 65 m (d) 70 m
 
70 a h 12 h2
70 Sol. 
h= = 8 40
cot 30º – cot 60º 1 h
3–
3 [from (i)- t an   ] 40 12
a h2 =
70 3 8
= = 35 3  h 2  ab  h  ab h2 = 60 m.
2

Rakesh Yadav Readers Publication Pvt. Ltd. 250

For More Visit : www.LearnEngineering.in


For More Visit : www.LearnEngineering.in

TYPE - V TYPE – VI P

A round balloon of radius r subtends At the foot of the mountain the


an angle  at the eye of an observer elevation of its summit is 450; after
while the angle of elevation of its ascending AB km towards the
mou ntai n up a slope of 30 0
centre is  . Then the height of its B
inclination, the elevation is found 0
60
N
centre from horizontal is to be 600, Then the height of the
0
15
mountain.
h = r sin . cos ec  2 45
0
30
0

A M Q

P Here, BN  PQ and BM  AQ,

ir
AB  4km, MAB  30 0 ,

v.iSn
A
r
MAP  45 0
O NBP  60 0 BAP  15 0

dnag
r
h 60°
and APQ  45 0
N B
2 B and BPN  30 0
2 APB  15 0

eYrai
0  ABP is isosceles and
C 45 0
30
D
Q M A AB  BP  4 k m
In PBN,
Proof....
Let O be the centre of balloon of
radius r. The observer's eye is at C,
snhe
 3  1
h= AB  
PN  BP sin 60 0
In ABM,
 BM  AB sin 30 0
kgei
2
ACB   and OCD   clearly,  
CA and CB are tangents to the circle.  PQ  PN  NQ  PN  BM
Ex.28 At the foot of the mountain  BP sin 60 0  AB sin 30 0
so ACO  BCO   2
ERna

the elevation of its summit


is 45 0 ; after asce ndin g 3 1  3 1 
In right angled OBC, 4  4  4  
4km towards the mountain 2 2  2 
 OB OB
sin   OC   r cos ec  u p a s l o pe o f 30 0
2
2 OC sin  2 
 2 3  1 km 
aBryn

inclination, the elevation


In right angled OCD, is found to be 60 0 , Then  Height of the mountain is

sin  
OD
 OD  OC sin   r cos ec  2 . sin  fi n d th e he i g h t o f th e = 2  3  1 km
OC mountain:- Alternae:-
 Height of the centre of the
 3 1   3 1 
Les

balloon is Sol. Suppose P be the summit of


PQ  AB   4  
the mountain and Q be the  2   2 
wa.th

r sin . cos ec 
2 foot.
=2  
3  1 km
wwM

Rakesh Yadav Readers Publication Pvt. Ltd. 251

For More Visit : www.LearnEngineering.in


For More Visit : www.LearnEngineering.in

EXERCISE

1. At a point on a horizontal line 6. There are two vertical posts, 10. An aeroplane when flying at a
through the base of a monu- one on each side of a road, just he ight of 3125 m fr om the
ment the angle of elevation of opposite to each other. One post ground passes vertically below
the top of the monument is is 108 metre high. From the top another plane at an instant
found to be such that its tan- of this post the angle of depres- when the angle of elevation of
1 sion of the top and foot of the the two planes from the same
gent is . On walking 138 other post are 30° and 60° re- point on the ground an 30° and
5
metres towards the monument spectively. The height of the 60° respectively. The distance

r
the secant of the angle of eleva- other post (in metre) is between the two planes at that
instant is

i
(a) 36 (b) 72 (c) 108 (d) 110

.iSn
193
tion is found to be . The 7. Two posts are x metres apart (a) 6520 m (b) 6000 m
12
and the height of one is double (c) 5000 m (d) 6250 m
height of the monument (in
that of the other. If from the 11. The shadow of the tower be-
metre) is

agv
mid-point of the line joining comes 60 meters longer when
(a) 42 (b) 49 (c) 35 (d) 56
their feet an observer finds the the altitude of the sun changes
2. The angle of elevation of the top
from 45° to 30°. Then the height

ridn
of a building from the top and angular elevations of their tops
to be complementary, then the of the tower is
bottom of a tree are x and y re-
spectively. If the height of the height (in metres) of the shorter (a) 20  3  1 m (b) 24  3 1m

eeYa
tree is h metre, then (in metre) post is
the height of the building is x x (c) 30  3  1 m (d) 30  3 –1 m
(a) 2 2 (b) 4
h cot x h cot y 12. A vertical post 15 ft. high is bro-
(a) cot x  cot y (b) cot x  cot y
ken at a certain height and its
geisnh x
(c) x 2 (d) 2 upper part, not completely sepa-
h cot x h cot y
(c) (d) rated meets the ground at an
cot x – cot y cot x – cot y 8. An aeroplane when flying at a
angle of 30°. Find the height at
3. The angle of elevation of the top height of 5000m from the ground
which the post is broken
passes vertically above another
Enak

of a tower from a point A on the (a) 10ft (b) 5ft


ground is 30°. On moving a dis- aeroplane at an instant, when the
(c) 15 3 2 – 3 ft
tance of 20 metres towards the angles of elevation of the two  
foot of the tower to a point B, ae r op lane s f r om the s am e
R

(d) 5 3 ft
the ang le of elevation in- p o in t o n the g r oun d ar e
60°and 45° respectively. The 13. The shadow of a tower is 3
creases to 60°. The height of
aryn

the tower is vertical distance between the times its height . Then the
aeroplanes at that instant is angle of elevation of the top of
(a) 3 m (b) 5 3 m
the tower is
Les B

(c) 10 3 m (d) 20 3 m (a) 5000  


3 –1 m (a) 45° (b) 30° (c) 60° (d) 90°
4. Two poles of equal height are 14. A man 6ft tall casts a shadow
standing opposite to each other 
(b) 5000 3 – 3  m 4ft long. At the same time when a
on either side of a road which flag pole casts a shadow 50 ft long.
wa.th

is 100m wide. From a point be-    The height of the flag pole is
tween them on road, angle of (c) 5000 1 – m (a) 80ft (b) 75ft (c) 60ft(d) 70ft
 3
elevation of their tops are 30° 15. The angle of elevation of an
and 60°. The height of each pole (d) 4500 m aeroplane from a point on the
wwM

( in meter) is 9. A man standing at a point P is ground is 60°. After 15 seconds


(a) 25 3 (b) 20 3 watching the top of a tower, flight,the elevation changes to
which makes an angle of eleva- 30°, If the aeroplane is flying at a
(c) 28 3 (d) 30 3 m tion of 30°. The man walks height of 1500 3 m, find the speed
5. The angle of elevation of the top som e distance towar ds the
of a chimney and roof of the of the plane.
tower and then his angle of el-
building from a point on the (a) 300 m/sec (b) 200m/sec
evation of the top of the tower
ground are 45° and xº respec- (c) 100m/sec (d) 150m/sec
is 60°. If the height of tower is
tively. The height of building is 16. There are two temples, one on
30m , then the d istance he
h metre. Then the height of the eac h bank of a rive r just
moves is
chimney,(in metre) is opposite to each other. One
(a) 22 m (b) 22 3 m temple is 54m high. From the
(a) h cot x+ h (b) h cot x – h
(c) h tan x – h (d) h tan x +h (c) 20 m (d) 20 3 m top of this temple, the angles of

Rakesh Yadav Readers Publication Pvt. Ltd. 252

For More Visit : www.LearnEngineering.in


For More Visit : www.LearnEngineering.in

depression of the top and the from its foot is 30°. The height 28. The angle of elevation of the top
foot of the other temple are 30° of the tower is of a vertical tower situated per-
and 60° res pectively. The 50 pendicularly on a plane is ob-
length of the temple is; (a) 50 3 m (b) 3
m served as 60° from a point P on
(a) 18 m (b) 36 m the same plane. From another
75
(c) 36 3 m (d) 18 3 m (c) 75 3 m (d) 3
m point Q, 10m vertically above
17. The angle of elevation of the top 23. From two points on the ground the point P, the angle of depres-
of a tower from the point P and lying on a straight line through sion of the foot of the tower is
Q at distance of ‘a’ and ‘b’ re- the foot of a pillar, the two 30°. The height of the tower is
spectively from the base of the angles of elevation of the top (a) 15 m (b) 30 m
tower and in the same straight of the pillar are complementary (c) 20 m (d) 25 m

ir
line with it are complemen- to each other. If the distance of
29. From a point 20 m away from

v.iSn
tary. The height of the tower is the two points from the foot of
the pillar are 9 metres and 16 the foot of a tower, the angle of
a elevation of the top of the tower
(a) ab (b) metres and the two points lie
b
on the same side of the pillar. is 30°. The height of the tower is
(c) ab (d) a2b2 Then the height of the pillar is

dnag
18. The ang le of ele vation of a (a) 10 3 m (b) 20 3 m
(a) 5m (b) 10m
tower from a distance 100 m (c) 9m (d) 12m 10 20
from its foot is 30°. Height of the 24. The top of two poles of height (c) 3
m (d) 3
m
tower is 24m and 36 m are connected by

eYrai
30. The angle of elevation of lad-
100 a wire. If the wire makes an
(a) m (b) 50 3 m der leaning against a house
3 angle of 60° with the horizontal,
then the length of the wire is is 60° and the foot of the lad-
200 der is 6.5 me tres from the
(a) 6m (b) 8 3 m
(c)
3
m (d) 100 3 m
19. A pole stands vertically inside
a scalene triangular park ABC.
(c) 8 m snhe
(d) 6 3 m
25. From the top of a hill 200 m
high the angle of depression of
house. The length of the lad-
der is
13
If the angle of elevation of the (a) (b) 13 meters
kgei
the top and the bottom of a 3
top of the pole from each cor-
ner of the park is same, then tower are observed to be 30° and (c) 15 meters (d) 3.25 metres
in ΔABC , the foot of the pole is 60°. The height of the tower is
31. The angle of elevation of sun
ERna

(in m);
at the changes form 30°to 45°, the
(a) centroid 400 3 2 length of the shadow of a pole
(a) m (b) 166 3 m
(b) circumcentre 3 decreases by 4 metres, the
(c) incentre 1 height of the pole is (Assume
aBryn

(c) 133 3 m (d) 200 3 m


(d) orthocentre 3  1.732 )
20. If the angle of elevation of a bal- 26. From a tower 125 metres high
the angle of depression of two (a) 1.464m (b) 9.464 m
loon fr om two c onse cutive
kilometre-stones along a road objects, which are in horizontal (c) 3.648 cm (d) 5.464 m
are 30° and 60° respectively, line through the base of the 32. A vertical pole and a vertical
Les

then the height of the balloon tower are 45° and 30° and they tower are standing on the same
wa.th

above the ground will be are on the same side of the level ground. Height of the pole
3 1
tower. The distance (in metres) is 10 metres. From the top of
(a) km (b) 2 km between the objects is
2 the pole the angle of elevation
2 (a) 125 3 m (b) 125
 3 – 1 m of the top of the tower and
(c) km (d) 3 3 km
wwM

3 angle of depression of the foot


21. A tower standing on a (c) 125 /  3 – 1 m(d) 125  3  1 m of the tower are 60° and 30° re-
horizontal plane subtends a 27. From a point P on the ground spectively. The height of the
certain angle at a point 160 m the angle of elevation of the top tower is
apart from the foot of the tower. of a 10m tall building is 30°. A (a) 20 m (b) 30 m
On advancing 100 m flag is hoisted at the top of the
towards it, the tower is found to building and the angle of eleva- (c) 40 m (d) 50 m
subtend and angle twice as be- tion of the top of the flagstaff 33. If a pole of 12 m height caste a
fore. The height of the tower is from P is 45°. Find the length shadow of 4 3 m long on the
(a) 80 m (b) 100 m of the flagstaff (Take 3  1.732 )
ground then the sun’s angle of
(c) 160 m (d) 200 m
(a) 10  30  2  m(b) 10  m
30  1 elevation at that instant is
22. The ang le of ele vation of a
tower from a distance 50 m (c) 10 3 m (d) 7.32 m (a) 30° (b) 60° (c) 45° (d) 90°

Rakesh Yadav Readers Publication Pvt. Ltd. 253

For More Visit : www.LearnEngineering.in


For More Visit : www.LearnEngineering.in

34. The angle of elevation of the top foot of another tower in the respectively. If the point is are
of a tower from a point on the same plane is 60º and the angle separated at a distance of 100
ground is 30° and moving 70 of elevation of the top of the m, then the height of the tower
meters towards the tower it second tower from the foot of is close to
becomes 60°. The height of the the first tower is 30º. The dis- (a) 86.6 m (b) 84.6 m
tower is tance between the two towers is (c) 82.6 m (d) 80.6 m
m times the height of the shorter 46. Two poles of heights 6m and
10
(a) 10 meter (b) metre tower. What is m equal to? 11m stand on a plane ground.
3
If the distance between their
(c) 10 3 metre (d) 35 3 metre 1 1
(a) 2 (b) 3 (c) 2 (d) 3 feet is 12 m, what is the dis-
35. From the top of a tower of height tance their tops?
180m the angles of depression 40. A spherical balloon of radius r (a) 13 m (b) 17 m

r
of two objects on both sides of subtends angle 60º at the eye (c) 18 m (d) 23 m

i
.iSn
the tower are 30° and 45°. Then of an observer. If the angle of 47. From the top of a cliff 200 m
the distance between the ob- elevation of its centre is 60º high, the angles of depression
jects are and h is height of the centre of of the top and bottom of a tower
the balloon, then which one of of observed to be 30º and 45º,

agv
(a) 180 3  3 m
  the following is correct? respectively. What is the height
of the tower?
(b) 180 3 – 3  m (a) h = r (b) h = 2r

ridn
(a) 400 m (b) 400 3
(c) 180  3 – 1 m (c) h = 3r (d) h = 2r
(c) 400 3 m

eeYa
41. What is the angle of elevation
(d) 180  3  1 m of the Sun, when the shadow of (d) None of these
48. The angle of elevation the tip
36. From the peak of a hill which is x of a tower from a point on the
300m high , the angle of depres- a pole of height x m is m?
3 ground is 45º. Moving 21 m di-
sion of two sides of a bridge lying
geisnh rectly towards the base of the
(a) 30º (b) 45º (c)60º (d) 75º
on a ground are 45° and 30°(both tower, the angle of elevation
42. On walking 120 m towards a
ends of the bridge are on the chimney in a horizontal line changes to 60º. What is the
same side of the hill). Then the through its base the angle of el- height of the tower, to the near-
length of the bridge is est meter?
Enak

evation of tip of the chimney


changes from 30º to 45º. The (a) 48 m (b) 49 m
(a) 300   m(b) 300 
3 –1 3 1
 (c) 50 m (d) 51m
height of the chimney is
300
49. What is the angle of elevation
R

(c) 300 3 m (d) m (a) 120 m (b) 60( 3 –1)m of the Sun when the shadow of
3
37. From an aeroplane just over a (c) 60( 3 +1)m a pole is 3 times the length
aryn

river, trees on the opposite (d) None of these of the pole?


bank of the river are found to 43. A ladder 20 m long is placed (a) 30º (b) 45º (c) 60º
Les B

be at an angle of 60° and 30° against a wall, so that the foot (d) None of these
respectively. If the breadth of of the ladder is 10 m from the 50. The angles of elevation of the
the river is 400 metres, then wall. The angle of inclination top of a tower from two points
the height of the aeroplane of the ladder to the horizontal situated at distances 36 m and
64 m from its base and in the
wa.th

above the river at that instant will be


is (Assume 3  1.732 ) (a) 30º (b) 45º (c)60º (d) 75º same straight line with it are
44. The angles of elevation of the complementary. What is the
(a) 173.2 metres
top of a tower from two points height of the tower?
(b) 346.4 metres
which are at distance of 10 m (a) 50 m (b) 48 m
wwM

(c) 519.6 metres


and 5 m from the base of the (c) 25 m (d) 24 m
(d) 692.8 metres
tower and in the same straight 51. The angle of elevation of the top
38. The shadow of a tower standing
on a level plane is found to be 50 line with it are complemen- of an incomplete vertical pillar
m longer when the Sun's eleva- tary. The height of the tower is at a horizontal distance of 100
tion is 30º, then when it is 60º. (a) 5 m (b) 15m m from its base is 45º. If the
What is the height of the tower? angle of elevation of the top of
(c) 50 m (d) 75 m
complete pillar at the same
(a) 25 m (b) 25 3 m 45. The angles of elevation of the point is to be 60º, then the
25 top of an inaccessible tower height of the incomplete pillar
(c) m (d) 30 m from two points on the same
3 is to be increased by
39. The angle of elevation of the top straight line from the base of
the tower are 30º and 60º, (a) 50 2 m (b) 100 m
of a tower 30 m high from the

Rakesh Yadav Readers Publication Pvt. Ltd. 254

For More Visit : www.LearnEngineering.in


For More Visit : www.LearnEngineering.in

bottom of tower. After 5 s, the (a) 59.4 m (b) 61.4 m


(c) 100  3 –1 m
 angle of depression becomes (c) 62.4 m (d) 63.4 m
30º. What is the approximate
62. A round balloon of unit radius
(d) 100  3  1 m speed of the boat assuming that
subtends an angle of 90º at the
it is running in still water?
52. The length of shadow of a tree eye of an observer standing at
(a) 31.5 km/h (b) 36.5 km/h
is 16 m when the angle of el- a point, say A. What is the
(c) 38.5 km/h (d) 40.5 km/h
evation of the Sun is 60º. What distance of the centre of the
58. Suppose the angle of elevation balloon from the point A?
is the height of the tree?
of the top of a tree at a point E
(a) 8 m (b) 16 m due East of the tree is 60º and
1 1
16 that at a point F due West of the (a) (b) 2 (c) 2 (d)
(c) 16 3 m (d) m 2 2

ir
3
tree is 30º. I f the dis tance
between the points E and F is

v.iSn
53. From a lighthouse the angles 160 ft, then what is the height 63. Person standing at the end of
of depression of two ships on of the tree? 1
oppsite sides of the lighthouse the shadow is times the
are observed to 30º and 45º. If (a) 40 3 ft (b) 60 ft 3

dnag
the height of lighthouse is h, length of the pole. At what angle
40
what is the distance between (c) ft (d) 23 ft of elevation will the man see
3 the Sun?
the ships?
59. The length of the shadow of a
(a) 60º (b) 30º (c) 45º (d) 15º

eYrai
(a)  3 1 h (b)  3 –1 h
 person s cm tall when the angle
of elevation of the Sun is  is 64. The angle of depression of ver-
 1  p cm. It is q cm when the angle
(c) 3 h 1 
(d)  
 of elevation of the Sun is
tex of a regular hexagon lying
 3 in a horizontal plane, from the
54. A telegraph post gets broken at
a point against a storm and its
top touches the ground at a dis-
tance 20 m from the base of
snhe
β .Which one of the following is
correct when β = 3  ?
top of tower of height 75 m located
at the centre of the regular hexa-
gon is 60º. What is the length
 tan – tan3 of each side of the hexagon?
kgei
the post making an angle 30º (a) p – q = s 
 

 tan3tan 
with the ground. What is the (a) 50 3 m (b) 75 m
height of the post?  tan 3 – tan
ERna

(b) p – q = s 
 
 (c) 25 3 m (d) 25 m
40  3 tantan 
(a) m (b) 20 3 m
3
 tan 3 – tan 65. Two houses are collinear with
(c) 40 3 m (d) 30 m (c) p – q = s 
 
 the base of a tower and are at
 tan3tan 
aBryn

distance 3 m and 12 m from the


55. The angle of elevation of the top
of a tower from the bottom of a  tan2  base of the tower. The angles

(d) p – q = s  
 of elevation from these two
building is twice that from its,  tan3tan
top. What is the height of the houses of the top of the tower
60. A radio transmitter antenna of are complementary.
building, if the height of the
Les

height 100 m stands at the top


tower is 75 m and the angle of of a tall building. At a point on What is the height of the tower?
wa.th

elevation of the top of the tower the ground, the angle of eleva-
from the bottom of the building tion of bottom of the antenna (a) 4 m (b) 6 m
is 60º? is 45º and that of top of antenna
(a) 25 m (b) 37.5 m (c) 7.5 m (d) 36 m
is 60º. What is the height of the
wwM

(c) 50 m (d) 60 m building? 66. The angle of elevation from the


56. The shadow of a tower is 15 m (a) 100 m (b) 50 m bank of a river of the top of a
when the Sun's altitude is 30º. tree standing on the opposite
What is the le ngth of the (c) 50  3 1 m
 bank is 60º. The angle of eleva-
shadow when the Sun's alti- tion becomes 30º when observed
tude is 60º? (d) 50  3 – 1 m
from a point 40 m backwards in
(a) 3m (b) 4m (c) 5m (d) 6m 61. The angle of elevation of the top a direction perpendicular to the
57. A man is watching from the top of an unfinished pillar at a point length of the river. What is the
to tower a boat speeding away 150 m from its base is 30º. If width of the river?
from the tower. The boat makes the angle of elevation at the
an angle of depression of 45º same point is to be 45º, then (a) 10 m (b) 20 m
with the man's eye when at the pillar has to be raised to a
(c) 30 m (d) 40 m
dis tanc e of 60 m fr om the height of how many metres?

Rakesh Yadav Readers Publication Pvt. Ltd. 255

For More Visit : www.LearnEngineering.in


For More Visit : www.LearnEngineering.in

ANSWER KEY
1. (a) 8. (c) 15. (b) 22. (b) 29. (d) 36. (a) 43. (c) 49. (a) 55. (c) 61. (d)
2. (c) 9. (d) 16. (b) 23. (d) 30. (b) 37. (a) 44. (c) 50. (b) 56. (c) 62. (b)
3. (c) 10. (d) 17. (a) 24. (b) 31. (d) 38. (b) 45. (a) 51. (c) 57. (a) 63. (a)
4. (a) 11. (c) 18. (a) 25. (c) 32. (c) 39. (b) 46. (a) 52. (c) 58. (a) 64. (c)
5. (b) 12. (b) 19. (b) 26. (b) 33. (b) 40. (c) 47. (d) 53. (a) 59. (c) 65. (b)
6. (b) 13. (b) 20. (a) 27. (d) 34. (d) 41. (c) 48. (c) 54. (b) 60. (c) 66. (b)
7. (a) 14. (b) 21. (a) 28. (b) 35. (d) 42. (c)

r
SOLUTION

i
.iSn
Put the value of d in equation (i)
1. (a) L–h
 d= h
tan x

agv
3h = 3
+ 20
 d = (L – h) cot x .....(i)
In ΔMDB , h  20 3

ridn
3h = 3
L MD
tany= =
Shortcut approach d DB 3h = h + 20 3

eeYa
Ist Case: d = L coty .....(ii)
from equation (i) and (ii) 2h = 20 3
AB Perpendicular 1
tanθ = BC
=
Base
=
5 (L – h) cotx = L coty
20 3
L cotx – h cotx = L coty h =
IIst Case: 2
h cotx = L (cotx – coty)
geisnh
AD Hypotenuse 193 h cot x h  10 3 metre
Sec = BD = Base
=
12 L = cot x – cot y Alternate:-
In ΔABD As we know the ratio of
3. (c) A
Hypotenuse = 193 A
Enak

Base = 12
A
Then perpendicular = 7
( By pythagoras theorem
3
R

In Case I Perpendicular is 1. unit


So equal this
30º D 60º C
aryn

1 7 B
tan = 1 unit 2 units
5 7
2 unit = CD = 20 metres
AB ='h' metre
Les B

1 unit = 10 metres
In ΔABC
3 units = AB = 3 ×10
AB
tan30°=
BC = 10 3 metres
wa.th

h
 4. (a)
d  20
1 h
AB = 7 × 6 = 42m 
3 d  20
wwM

2. (c)
3h = d + 20 ......(i) (100–x)
In ΔABD
BD = 100
L
AB h AB = CD = 'h' metre (Height of pole)
tan60° = =
BD d In ΔABP ,

h h
AB = tree 'h' 3=  tan 30 
d x
MD = Building 'L'
DB = CA = 'd' h  3d 1 h
 
In ΔMCA , 3 x
h
MC L – h d = ......(ii)
tan x = = 3  3h  x .......(i)
AC d

Rakesh Yadav Readers Publication Pvt. Ltd. 256

For More Visit : www.LearnEngineering.in


For More Visit : www.LearnEngineering.in

In ΔCDP In ΔACE 2h 4h
tan.cot = x  x
h AC
tan60° =  x² = 8h²
tan60° = CE
100 – x 
x² x
3 AC  h² =  h  metre
 3 (100 – x ) = h = = AC : CE = 3 : 1 ......(i) 8 2 2
1 CE
3 100 – 3h 8. (c)
   = h In ΔABD

(Put the value of x from equation (i) AB


tan30° =
 100 3 – 3h = h  4h = 100 3 BD

1 AB 60º 45º
h = 25 3 metre   AB : BD  1 : 3 ..(ii)

ir
3 BD
Alternate:-

v.iSn
Since BD = CE AC = 5000 m (given)
 AC : CE : AB In ΔACD ,

h h AC
tan60° =
CD

dnag
2 1 AC
3  AC:CD  3 :1
a CD
In ΔBCD ,

eYrai
 DE = AC – AB BC
= 108 – 36 = 72 metre tan45° =
h CD
Alternate:
In such question when angles are BC
30º 60º 1 = = BC : CD = 1 : 1 ........(ii)
30º and 60º CD

h =
a
cot2  cot1
100
snhe
30º
Now,
BC : CD
1 : 1
:
:
AC

1 : 3
h
kgei
100 1 : 1 : 3
1 x
h = 3 AB = AC – BC
3 60º
ERna

=  3 –1  units
 25 3 metre
2h 2 108 3 units = 5000 m
5. (b) AB = Building = h metre x = = = 72cm
3 3 5000
aBryn

1 unit = m
7. (a) 3
5000
xº AB =  3 – 1 units 
  3 –1

3

AD = chimney = 'y' meter  1 


= 5000 1–  m
Les

In ΔDCB 
 3 
wa.th

DB h y Alternate:
tan45° =
BC
 1=
BC A
From figure,
 BC = h + y .......(i)
x
In ΔACB OB = OD =
2
wwM

AB h
tanx° =
BC
 tanx =
BC
 BC = In ΔOCD , 3 unit B
60
h cotx....(ii) h 1unit º
from equation (i) and (ii)
tan = x  2h ........(i)
 h + y = h cot x x 45º
2
C D
 y = (hcot x –h) metre 1 unit
In ΔAOB
AC = 3 unit
6. (b) AB
tan(90 –)  BC = 1 unit
OB
2h 4h AB = ( 3 –1) unit
 cot  ........(ii)
x x According to question,
2
AC = 3 unit = 5000 m
Multiplying both equations,

Rakesh Yadav Readers Publication Pvt. Ltd. 257

For More Visit : www.LearnEngineering.in


For More Visit : www.LearnEngineering.in

So, PC = 2 units CD = BD – BC
5000
1 unit = m CD = 3– 1
3  2 × 10 3 = 20 3 metre
( 3 – 1) units = 60
So, t he diff er ence bet ween t he
10. (d) BC = 3125 m 60
aeroplane = AB = ( 3 –1) unit 1 unit =
3 –1
5000 60 3 1
( 3 –1)unit = ( 3 –1)m h = 
3 3 –1 3 1

 h = 30  3 1 m

1 
AB = 5000 1 –
 m In ΔACD Alternate:
 3
AC A

r
9. (d) A tan60° =
DC

i
.iSn
3 AC
=
1 DC 1 unit
AC : DC = 3 :1 .......(i)

agv
45º 30º
In DCB B D
1unit C
BC 3 unit
tan30° =

ridn
DC AB = 1 unit
h = 30 m
BC = 1 unit
PC = ? 1 BC
= CD = ( 3 –1) unit

eeYa
In ΔABP 3 DC
AB BC : DC = 1 : ......(ii) CD = ( 3 –1) unit = 60 m
tan30° = 3
BP Now, 60 3 1
1 AB AC : DC : BC Height (AB) = ×
3 –1 3 1
=  AB : BP = 1 : 3 .....(i)
geisnh : 1
3 BP 3
= 30 ( 3 +1)m
In ΔABC 3 : 1
AB 12. (b)
tan60° = 3 : 3 : 1
BC
Enak
BC = 1 unit = 3125 m
3 AB AB = AC – BC
  AB : BC = 3 : 1 ......(ii)  3 – 1 = 2 units
1 BC
BP : AB : BC AB = 2 × 3125 = 6250 m
R

MAB was straight earlier


3 : 1 11. (c) A AB + AM = 15 ft
3 : 1 In ΔABM
aryn

3 : 3 : 1 AB
tan30° =
Now BM
Les B

AB = 1 AB
3 units = 30 metre =
3 BM
30 3 h = height If AB = 1
1 unit = 
3 3 In ΔABC and BM = 3
wa.th

then AM = 2( By pythagoras theorem)


= 10 3 metre AB AB + AM = 2 +1  3 units = 15 ft
tan45° = AB = 1 unit = 5ft
PC = 3 – 1 = 2 units BC
13. (b)
= 10 3  2 = 20 3 metre 1 AB
= = AB : BC =1 : 1 .......(i)
1 BC
wwM

Alternate:
As we proof in Type (iii) In ΔABD
A
AB
tan30° = In ΔABC
BD
1 AB 1
3 unit = tan = 3
3 BD
tan = tan30°   = 30º
60º 30º  AB : BD = 1 : 3 ......(ii)
B
1unit C 2 unit D 14. (b) Height Shadow
Now, 6ft 4ft
AB = 30m = 3 units BD : AB : BC 3 : 2
1 : 1
So height of pole will be in same ratio.
30 3 3 : 1
3
1 unit =  = 10 3 metre = 50  = 75ft
3 3 3 : 1 : 1 2

Rakesh Yadav Readers Publication Pvt. Ltd. 258

For More Visit : www.LearnEngineering.in


For More Visit : www.LearnEngineering.in

CD = AB – AE
15. (b) h
= 3 – 1 = 2 units  Cot ....(ii)
a
A B = 3 units × 18 = 54 m
CD =2 units × 18 = 36 m By multiplying both equation
Alternate: h h
30º 60º tan.cot 
A b a

AB = CD = 1500 3 m h2 = ab  h = ab
(height of aeroplane) 30º 18. (a)
E C
In PDC h
CD x
tan60° =
PD

ir
60º
CD B D

v.iSn
3 = PD  CD:PD= 3 : 1 ...(i) AB = h = 54m
In ΔABC
CD = x
In ΔPBA AB
2h 2  54 tan30° =
AB x = = = 36m BC
tan30° =
PB
3 3

dnag
1 AB
Alternate:- =
1 AB 3 BC
=  AB : PB = 1 : 3 .....(ii) hcot
3 PB H = cot–cot
AC = BD and AB = CD

eYrai
PD : AB : PB Where,  = smaller angle
1 : 3  = greater angle m
1 : 3 h cot 30º
54 = 100
1 :3
DB = PB – PD
:

= 3 – 1 = 2 units
3

54 =
h 3
1
snhe
cot 30º – cot 60º  Height of tower =
19. (b)
20. (a) A
3
m

It should be on circumcentre.

AB = 3 units = 1500 3 m 3–
kgei
 1 unit = 1500 m
3
CA = DB  2 units = 3000 metre
h 3
ERna

Distance 3000 54 =
Speed = = =200 m/s 3 –1
Time 15
3
16. (b)
3h AB = height of balloon
aBryn

54 = In ΔABC
2
AB
54  2 tan60° =
BC
h = = 36 m
3
AB and CD are temples 17. (a) AB
A 3=  AB : BC = 3 :1
BD = width of river BC
Les

AB = 54 m In ΔABD
wa.th

In AEC
AB
AE 1 tan30° =  AB : BD = 1 : 3
BD
tan30º = =
EC 3 Now,
 AE : EC = 1 : 3 BC : AB : BD
wwM

In ABD AB is tower 1 : 3
 AQB  & APB  90 –
AB 1 :
tan60° = PB = a, BQ = b 3
BD
In ΔAQB
1 : 3 : 3
AB
3=  AB : BD = 3 : 1 ......(ii) AB CD = BD – BC
BD tanθ =
BQ = 3 – 1 = 2 units
EB = CD and EC = BD
Now, 2 units = 1 km
h
AB : BD : AE tan = b .....(i) 1
: 1 1 unit =
3 2
In ΔAPB
3 : 1
1 3
h AB = 3 unit =  3 = km
3 : 3 : 1 tan 90º –  2 2
PB

Rakesh Yadav Readers Publication Pvt. Ltd. 259

For More Visit : www.LearnEngineering.in


For More Visit : www.LearnEngineering.in

21. (a)  h² = 144 AB = h = 200z


A
h =
144 2h 2  200 1
h = 12 metre CD = x = = = 133 m
3 3 3
Alternate:-
h = 16  9 = 12 m. 26. (b) A
45° 30°
24. (b)
160m 125m
BD = 160m (given)
In ΔACD
exterior ACB  CAD  ADC
2 CAD + θ AC = wire

r
AB = Tower
AB and CD are two poles
CAD = θ

i
.iSn
In ΔAEC AB
 AC = CD In ΔABC tan45° =
AE BC
AC = 100 m
sin60° =
In ΔABC AC AB
1=  AB : BC = 1 : 1 ....(i)

agv
AC = 100 m 3 12 BC
BC = 160 – 100 = 60 m  
2 AC In ΔABD ,
Then AB = 80 m (By pythagoras theo-
(AE = AB – CD = 36 – 24 = 12 m)

ridn
rem) AB
24 tan30° = = AB : BD = 1 : 3 ....(ii)
AC = = 8 3 m BD
22. (b)
3
Now,

eeYa
25. (c)
BC : AB : BD
1 : 1
1 : 3
AB = Tower
BC = 50 m (Given)
geisnh 1 : 1 : 3
In ΔABC AB = hill =200 metre
CD = tower CD = BD – BC =  3 –1  units
AB
tan In ΔAPC
BC AB = 1unit = 125 metre
Enak
AP
AB 3 – 1 units = 125 3 – 1 metre
tan 30 
50
tan30° =
PC
CD =    
1 AP 27. (d)
50 = = AP : PC = 1 : 3 .......(i)
R

AB = m 3 PC
3
In ΔABD
23. (d) A
AB
aryn

tan60° = 45º
BD 30º

h AB
Les B

3 = BD = AB : BD = 3 : 1 ......(ii) AB – building = 10 m
In ΔABP
PB = CD and PC = BD
Now AB
tan30° =
AB : BD : AP BP
AB = Pillar
3 : 1
wa.th

1 AB
BC = 9 metre = = AB : B P
BD = 16 metre 3 : 1 3 BP
ADB  3 : 3 : 1 = 1: 3 ....(i)
In ΔABC CD = PB  AB – AP In ΔFBP
wwM

AB CD = 3 – 1 = 2 units FB
tan 90 – = AB = 3 units = 200 metre tan45° =
BC BP
200 FB
AB h CD = 2 units = 2
3 1 = = FB : BP
cot= BC = ...(i) BP
9 = 1 : 1 .....(ii)
1
In ΔABD = 133 metre Now,
3 AB : BP : FB
h Alternate:-
tan = 16 ...(ii) 1 : 3
A 1 : 1
By multiplying equation (i) and (ii)
h h
30º C
P
tan. cot = 9 × 16 h
x
h2 60º

144
= 1
D 10 3 m
B

Rakesh Yadav Readers Publication Pvt. Ltd. 260

For More Visit : www.LearnEngineering.in


For More Visit : www.LearnEngineering.in

FB =17.32 m 31. (d) In ΔABC


FA = FB – AB
= 17.32 – 10 AB 12
tanθ= =
= 7.32 metre BC 4 3
28. (b)
3
tan = 3
AB = pole
10m In ΔABC tan = 3 = tan60°

AB θ = 60°
tan45° = 34 (d)
AB = Tower BC
QP = 10 metre AB
1 = = AB : BC = 1 : 1 ......(i)

ir
QP BC
In ΔQBP , tan30° = In ΔABD

v.iSn
PB
AB
1 QP tan30°= = AB : BD = 1 : 3 ....(ii)
=  QP : PB = 1 : 3 .......(i) BD
3 PB BC : AB : BD
1 : 1

dnag
In ΔABP In ΔACD
1 : 3
AB ACB  CAD  ADC
tan60° = 1 : 1 : 3
BP 60° = CAD + 30°
CD = BD – BC
AB CAD = 30°

eYrai
= 3 –1 = 3 – 1 units = 4 m
3 = BP  AB : BP = 3 : 1 .....(ii) So,
4 AC = CD
CB = QP and CQ = BP AB = 1unit =
3 –1 AC = 70m
Now,
AB : BP : CB AC
= 2  3 1  = 5.464 m
3 : 1

3 : 1
32. (c) snhe cosec 60° =

2

3 AB
70
AB
kgei
3
AB = 35 3 m
Alternate:-
AB = pole CE = tower A
ERna

AB = 10 metre
29. (d) A
In ΔABE
h AB 3 units
tan30° =
BE
30°
aBryn

60° 30°
B C B 1 unit D
20m 1 AB C 2 units
= = AB : BE = 1 : 3 ......(i)
3 BE CD = 2 unit = 70 m
In  ABC
In ACD  1 unit = 35 m
AB h 1 AB = 35 3m
= tan30°   CD
BC 20 3 tan60° = 35. (d)
Les

AD A
20 3 CD
wa.th

 h = m = = CD : AD = 3 : 1....(ii)
3 1 AD
AD = BE and AB = DE 180 m
30. (b) Now, 300
450
AB : BE : CD D C
1 :
B
3 AB = 180m
wwM

1 : 3 In ABC
AB
tan 450 = BC
AC = Ladder
BC = 6.5 metres AB
1  AB : BC = 1:1 .....(1)
In ΔABC BC
CE = CD + DE = 30 + 10 = 40 metre
BC 33. (b)
In ABD
Cos60° =
AC AB
tan 300 =
1 6.5 BD
= m 1 AB
2 AC

AC = 13 m 3 BD

Rakesh Yadav Readers Publication Pvt. Ltd. 261

For More Visit : www.LearnEngineering.in


For More Visit : www.LearnEngineering.in

BC = 400 metres 2 units = 50m


AB : BD = 1 : 3 .....(ii)
In ABD 1 unit = 25m
Now,
AD CD = 3 = 25 3 m.
BC : AB : BD tan60° =
BD
1 :1 39. (b) Let h be the height of
1 : 3 AD shorte r tower , then the
3 =  AD : BD = 3 : 1....(i)
1 BD distance b etwe en the two
to make equal ratio towers is mh m.
In ΔADC
BC : AB : BD C
1 :1 : AD
3 tan30° =
CD = BD + BC
DC D
1 AD 30 m

r
CD =  
3  1 units =
3 DC
 AD : DC = 1 : 3 .....(ii) h

i
.iSn
AB = 1 unit = 180 m Now, 60º 30º
BD : AD : DC A B
CD =  
3  1 units
1 : 3
mh
h

agv
In  ABD, tan30º =
= 180  3 1 m  1 : 3 mh
1 : 3 : 3 1 1

ridn
36. (a) BC = BD + DC  
3 m
= 1 + 3 = 4 units.
4 units = 400m

eeYa
 m= 3
1 unit = 100m
AD = 3 unit
40. (c) In  ABO,
= 100 3 = 100 × 1.732 = 173.2m O
38. (b) Let h be the height of the r
geisnh
AB = height of peak = 300 m tower and BC be x m. C
CD = length of Bridge
h h
In ΔABC In  BCD, tan60º =
x 60º
AB
Enak

tan45° = h
BC
3 60º

x A B
AB
1 = = AB : BC = 1 : 1 OB
R

BC  h= x 3 ...(i) sin60º =
AO
In ΔABD
D OB
aryn

AB  AO = ....(i)
tan30° =
BD
sin60º
h Now, In  AOC,
Les B

1 AB
3
=
BD
 AB : BD = 1 : 3 60º OC
30º 60º sin 
Now, 2 AO
BC : AB : BD
A B C
50m x OC
1 : 1  AO = ....(ii)
wa.th

Now in  ACD, tan30º sin30º


1 : 3 From Eqs. (i) and (ii),
h
= OB OC
1 : 1 : 3 50  x 
CD = BD – BC sin60º sin30º
1 x 3
wwM

  h r
CD = ( 3 – 1) unit 3 50  x 
AB = 1 unit = 300 metre 3 1
 50 + x = 3x
 x = 25m 2 2
 3 – 1 units = 300
  3 – 1 metre
  h = 25 3 [from eq. (i)]  h = 3r
37. (a) Alternate:- 41. (c) Here , θ is the angle of
D elevation,

h
3 unit x
60° 30°
B C ?
60º 30º
C 1 unit B 2 unit A
400 m x/ 3

Rakesh Yadav Readers Publication Pvt. Ltd. 262

For More Visit : www.LearnEngineering.in


For More Visit : www.LearnEngineering.in

Perpendicular x 3 1 m h
tan θ =
Base
= x
= 60   tan60º =
x
= 3
So, the height of PC
3
h
= 60( 3 +1) metres  x= .....(i)
3
3x 43. (c) Let θ be the inclination of
=  3
x the ladder to the horizontal. A
Here, tan θ = 3
) B
 θ = 60º (  tan60º= 3 )
der h (Tower)

42. (c) Let h be the height of the ad


(L

ir
30º 60º
chimney. m B C D
20 

v.iSn
100 xm
In  BPC,
A 10 m O and in  ABD
h Now, In  AOB,
tan45º = 1 h = x ...(i)
x h 1

dnag
AO 10 1 tan30º = =
cos θ = = = 100  x 3
C(Chimney) AB 20 2
 cos θ = cos60º h
 3 h = 100 + x = 100 + 3
 θ = 60º

eYrai
h
44. (c) Giv en that, ang les are [From Equation (i)]
30º 45º complementary.
A  1 
120m B P Let  PBC = θ  3– 3
   h = 100
xm

Now, In  APC,
h 1
snhe
  PAC = 90 – θ
Let h be the height of the tower

2
3

h = 100  h = 50 3
tan30º = 120  x 
kgei
3  h = 50×1.732 = 86.6m
h 1 So, the require d he ight is
 120  h  [from Eq. (i)] 86.6m
3
ERna

Alternate:-
 3 h = 120 + h A
 3 h – h = 120
Now, in  PBC
aBryn

3 – 1 = 120 3 units
h   tan θ =
h
...... (i)
5
60º 30º
 h=
120

3 1
=
120  3  1 and in  PAC D C
B
2 units
3 –1 3 1 2 1 unit
Les

h
 R equired height of the tan(90º– θ ) = BC = 2 units = 100 m
wa.th

10 1unit = 50m
chimney(h) = 60  3 1  m
h Height = AD = 3 ×50
Alternate:-  cot θ = ...... (ii)
10
C = 50 3 = 50 × 1.732 = 86.6 m
On multiplying Eqs. (i) and (ii),
wwM

we get 46. (a) Given,


AB = 6m and EC = 11m
1 unit h h
tan θ .cot θ = . E
5 10
45º 30º h² 5m
P A =1  h=
1 unit B 3 –1

50 50 m A
unit 12m D 11m
Alternate:-
AB = 120 m
h= 5 10 = 50 m 6m
=  3 – 1 unit = 120 m
 45. (a) Let h b e the he ight of
120 3 1 inaccessible tower. B C
1 unit = × 12m
3 –1 3 1 Now, in  ACD  BC = 12m

Rakesh Yadav Readers Publication Pvt. Ltd. 263

For More Visit : www.LearnEngineering.in


For More Visit : www.LearnEngineering.in

 BC = AD = 12m 1
h
and ED = EC – CD = EC – AB  x ....(i)  tan θ =
3 3
(  AB = CD)
= 11 – 6 = 5m h  1 
θ = 30º  tan30º  3 
In  PAC, tan45º = 1   
In  AED, 21  x  
(AE)² = (AD)² + (ED)²  h = 21 + x 50. (b) Let CD = h be height of the
(by Pythagoras theorem) tower.
= (12)² + (5)² = 144 + 25 h D
 h = 21 + [from Eq. (i)]
= 169 = (13)² 3
 AE = 13m

r
 1 
So, the distance between their  h 1 –   21
 3

i
.iSn
tops is 13 m.  
β 
47. (d) Let AE = 200 m be the height A
B 36 m C
3 1
. 
of the cliff and BD = h m be the 21 3 64 m
 h
height of the tower.

agv
3 – 1 3  1 h
  In  BCD, tan =
36
A

ridn
30º 21 3 and In  ACD,
 3  1 = 49.68  50m
200–h

45º = h
2 tan β =

eeYa
Alternate:- 64
200m

B 30º
x C 
P But  + β =
2
h
geisnh 
45º  tan ( + β ) = tan
2
D x E 3 unit
tan  tanβ 1
In  ABC,  =
1 – tantanβ 0
Enak

200 – h 1 200 – h 60º 45º


C A
tan30º =  = 1 unit B 3 –1  1 – tan tan β = 0
x 3 x unit
 tan tan β = 1
R

 x = (200 – h) 3
AB = ( 3 –1) unit = 21m h h
200  × =1
and In  ADE, tan 45º = 36 64
aryn

x
1 unit =
 3  1
21
×
 h² = 36 × 64  h = 6 × 8
200  h = 48 m
 3 – 1  3  1
Les B

 1=  x = 200 m Alternate:-
x
From Eq. (i), h = 36  64
21 3  1
200 = (200 – h) 3 1 unit = m = 6×8 = 48 m.
2 51. (c) Let the height of the
wa.th

 3 – 1 incomplete pillar be x m and


21
 3 1  the increaseing height be PC
 h = 200 
 3 
m
PC = 3×
  2 =hm
= 49.68 = 50m
wwM

48. (c) In  PBC, P


P AB h
49. (a)  tan θ =
BC C
A
h x
60º
A 45°
B
100 m
45º 60º 
A B C C B In  ABC,
21m x
BC = 3AB x
h h tan45° =  x = 100 m
tan60º =   3 AB 1 100
x x  
and In  APB,
BC 3

Rakesh Yadav Readers Publication Pvt. Ltd. 264

For More Visit : www.LearnEngineering.in


For More Visit : www.LearnEngineering.in

x h
Alternate:- x = 75 3 – h
 3
tan 60° =
100 and In  BCE,
x + h = 100 3
h CE
tan60°=
 h = 100 3 – x = 100 3 – 100 BC
30º 45º
x y 75
 h = 100  3 –1 m   3 = x
x + y = h (cot 30º + cot 45º)
Alternate:-
 h ( 3 +1)  x 3 = 75 (from eq.(i))
A
x + y  ( 3 +1)h

ir
3 –1 54. (b) Let the height of post be x m.
 75 3 –h 3  3 = 75

v.iSn
unit  75 × 3 – 3h = 75
3 B A  3h = 75 × 3 – 75
60
1unit º 75×2
h–x  h=

dnag
45º x 3
C D
1 unit  h = 50 m
30° Alternate:-
CD = 1 unit = 100 m C B
20m

eYrai
AB = 100 ( 3 –1)m E
x 1
52. (c) Let the height of the tree be In  ABC, tan30º = =
h m. 20 3
30º A
C  x=
20
3
m snhe 20
....(i) 3 unit
D

2 unit
kgei
h and cos30º = 60º
h –x C B
60° 3 units
A B 3 20
=
ERna

16m 
2 h –x
2EC 2×75
h 40 AB = = = 50 m.
In  ABC, tan 60° = 3 3
16  h–x=
3
56. (c) In  ACD,
aBryn

 h = 16 3 m From equation (i),


CD
53. (a) In  PBT 40 20 60 3 tan 30º =
h= + = × AC
3 3 3 3
h 1 h
tan45° = =1  =
y = 20 3 m
Les

3 15
 y=h 55. (c) Let height of the building be
wa.th

h m and distance b etwe en 15


 h=
P building and tower be x m. 3
30° 45°
E D
wwM

30° 45°
x y A
30º
D 75 m
h
A T B

and in  PTA 30º 60º


h A
60º B x C
h 15m
B x C
tan 30° =  x = 3h ...(ii)
x
and in  BCD,
 Required distance, In  ADE,
CD
3+1 m ED 1 75 – h tan 60º =
x+y= 3h + h = h   tan30º =
AD

3
=
x
BC

Rakesh Yadav Readers Publication Pvt. Ltd. 265

For More Visit : www.LearnEngineering.in


For More Visit : www.LearnEngineering.in

58. (a) Let AC = h (Height of a tower) B


h
 3 = x x = Distance between A and F
 AE = 160 – x
h In  AFC,
 =x s
3
h
tan 30º =
15 x
 x= = 5m ( from Eq. (i))
3  
1 h A
Alternate:-  =  x= 3h ...(i) D q C
3 x
p
C In  BDC,

r
3 unit s s

i
.iSn
tan β = q  q =
h tan3
60º 30º
1 unit 2 unit 60º ( β = 3 , given) .....(i)
30º
F A (160– x ) E

agv
3 units = 15 m x On subtracting Eq. (ii) from Eq.
160ft (i), we get
15 N

ridn
1 unit = = 5m s s
3 p–q= –
tan tan3
So, the shadow will be 5m when

eeYa
angle becomes 60º  tan 3 – tan 
W E = s  tan tan 3 
57. (a) Let AB be a height of tower,  
60. (c) Let BC be a building of height
B CD = x m h m and CD = 100 m be a
30° 45°
geisnh S height of antenna.
and In  AEC, x = Distance between A and B
D
h
x tan 60º =
160 – x 100 m
Enak

 3 (160 –x) =h C
45º 30º
A 60 m C x D  3 (160 – 3 h) =h [from Eq. (i)] 60º h
R

 160 3 –3h) =h
A 45º
In  ACB, B
 4h = 160 3 x
aryn

AB  h = 40 3 ft In  ABC,
tan45º = Alternate:-
AC h
Les B

tan 45º =  x = h . . . .(i)


AB x
 1=
60 and in  ABD,
 AB = 60 m h
100  h
wa.th

Now, In  ADB, tan 60º =


x
30º 60º
60 x y 100  h
 tan30º = 160 
60  x 3 = h
[from Eq. (i)]
x + y = h (cotB + cotA)
wwM

1 60 160 = h (cot30º + cot60º) 


 = 3 h = 100 +h
3 60  x
 1   4 
160 = h  3   = h   
 3 – 1 h = 100

 60 +x = 60 3  3
 3

3 – 1 = 60 (1.73 – 1) 100
 x = 60   h=
160  3
= 40 3 ft.
h=
3 –1
= 60 ×0.73 = 43.8 m 4
43.8 18 59. (c) In  ABC, 100  3 1 
 Speed of boat =   h= ×
5 5 3 1
788.4
s
tan = p  p =
s
. . . (i)
 3 – 1  
= = 31.5 km/h tan
25 = 50 ( 3 +1)m

Rakesh Yadav Readers Publication Pvt. Ltd. 266

For More Visit : www.LearnEngineering.in


For More Visit : www.LearnEngineering.in

Alternate:- Alternate:- x = Distance between O and F.


D D G
3 –1 60º
unit h = 75m
3 C
3 units C 45º E D
60 1 unit
1unit º 30º
B A 60º
45º 3 units F O C
B A x
1 unit
CD = DB – BC AB = 3 units = 150 m 60º

ir
= ( 3 –1) unit 150 3
1 unit = × A B
3 3

v.iSn
( 3 – 1) unit = 100 m In  FOG,
= 50 3 metre 75
100 3 1 tan 60º =
1 unit = × So, CD = 50 3 ( 3 –1) x
3 –1 3 1

dnag
= 150 – 86.60 = 63.4 metres 75
= 50( 3 +1) metre 62. (b) Let O = Centre of the ballon  x= = 25 3 m
3
BC=1 unit = 50 ( 3 +1) metres OB = OC = Radii of the balloon
In regular hexagon  OEF,

eYrai
Eyes
61. (d) Let BC = x m height of un- A  OED,...are equilateral tri-
finished pillar and CD= h m = angles.
º
Raised height of pillar 45 5º
C 4  OF = OE = OA = OD = OC = OB

D 1
O
snhe
1
= OA = side of hexagon
 Length of hexagon = 25 3 m
h 65. (b) Let h be the height of the
kgei
B tower and  CBD = θ
C In  OAB,
 DAC = 90º – θ
ERna

OB
45º x sin 45º =
OA D
A 30º
B 1 1
150m  =  OA = 2
2 OA
aBryn

63. (a) Let h be the height of pole, h


In  ABC, then its s hadow = h 3
x 150 (given) θ (suppose) be the angle 90º– 
tan 30º =  x= ...(i) of elevation. A B 3m C
150 3
C 12m
Les

and in  ABD,
wa.th

hx hx  In  BCD,


tan 45º =  1=
150 150 h CD h
tan θ =  tan θ = . . (i)
BC 3
150
wwM

 150 = h + [from Eq. (i)] 


3 A and in  ACD
h/ 3 B
CD
150 3 –1 tan (90º – θ ) =
 h
  Then, tan θ =
h AC
3 h/ 3 h
 cot θ = . . . . . . .(i)
 tan θ = 3 = tan 60º 12
 h = 150×
1.732 – 1 On multiplying Eqs. (i) & (ii), we get
1.732  θ = 60º
h h
64. (c) Let OG be a height of the tan θ . cot θ = .
150  0.732 3 12
= = 63.39 tower,
1.732
Angle of elevation = Angle of h2
 = 1  h² = 36  h = 6m
= 63.4 m (approx) depression 36

Rakesh Yadav Readers Publication Pvt. Ltd. 267

For More Visit : www.LearnEngineering.in


For More Visit : www.LearnEngineering.in

66. (b) Let AB = h m be the height In  ABC, Alternate:-


of the tree and BC= x m be A
the width of the river h
tan 60º =  h= 3x . . . (i)
A x
and in  ADB, tan 30º
3unit
h 40  x
=  h= . . . (ii)
h 40  x 3
60º 30º
From Eqs. (i) & (ii), B 1 unit C 2 unit D
40  x CD = 40 m
30º 60º 3x =  2x = 40  x 2 units = 40m
3

r
D 40m C x B 1 unit = 20 m
=20 m So, the width of river = 20 m

i
agv
.iSn
ridn
eeYa
geisnh
Enak
R
aryn
Les B
wa.th
wwM

Rakesh Yadav Readers Publication Pvt. Ltd. 268

For More Visit : www.LearnEngineering.in


For More Visit : www.LearnEngineering.in

CHAPTER

TRIGONOMETRY CIRCULAR
15
MEASURE OF ANGLES

Angles: When two rays (initial and 4. Rel atio n between d egree
1

ir
terminal) meet at a point after rotation 1 second =1 =''
minute measure and radian measure:
60

v.iSn
in a plane then they are said to have
 π rad = 180°
described an angle. In other words:- 900 = 1 right
angle πx
 x   180 rad
Anti c n
rotati

de
0
60' = 1

dnag
si
n al
i 60'' = 1'
loc
o

rm
Te 180
k wis

Hence 1 right angle = 90° and x rad  x


positive π
e

= 90 × 60 = 5400

eYari
O
Initial side
= 5400 minutes 180
1 rad   5716'22"
side = 90 × 60 × 60 = 324000" 3.14
ia l
Init
rotation e

= 324000 seconds
clock w

O ne gative snhe
Again, 1  60'  60  60"  3600"
Thus to change degree into ra-
π
is

Ter m dian, multiply by and to


inal 180
side 2. Centesimal or French System:-
kgei
change radian into degree mul-
Unit of Angle: 1 right angle =100 grades ( =100g)
180
1. Degree (°) 2. Radian (c) 1 grade = 60 minutes ( = 60') tiply by .
π
ERna

3. Grade (g) 1 minute = 60 seconds ( = 60")


Systems of Measurement of 3. Circular System:- 22
If mentioned , take π = or 3.14.
Angles: 7
In this system , the unit of
1. Hexagonal System measurement is "radian".
aBryn

   22  
and 1  radian   rad
The angle between two perpen-  180  7  180  
Ang le (in r adian)
dicular lines is called a right o r 1  0 .0 1 7 4 6 r a d i a n
angle. A right angle is equal to a rc A C  AC
  For Ex:
90 degree, it is written as 90°. ra d iu s r
Les

c c
sin1 sin1° cos1 cos1°
Thus, if a right angle is divided
C
wa. th

into 90 equal parts then one part 1 radian or 1 is the angle » 57° » 0° » 57° » 0°
is called one degree. It is written subtended by an arc at the centre » sin 60° » sin0° » cos 60° » cos0°
then sin 60°> sin 0° then cos 60°< cos 0°
as1°. of a circle whose length is equal c c
to the radius of the circle. sin1 > sin 1° cos1 < cos 1°
If 1° is divided into 60 equal
wwM

parts, each part is called 1 C


minute. It is denote by 1' r Degree Radian Degree Radian
ar
c

1 B
th part of 1' is called one sec- O r A  /6 3 / 4
60 30° 135°
ond. It is written as 1". 45°  /4 150° 5 / 6
60°  /3 180° 
1 i.e. If arc=radius = r, then 90°  /2 270° 3 / 2
1 degree =10= right angle 120° 2 / 3 360° 2
90
r
  1, radian  1c
r Ex. 1 Convert the following degree
1 measure s in the rad ian
1 minute =1'= degree
60 when arc ACB = r measure.

Rakesh Yadav Readers Publication Pvt. Ltd. 269

For More Visit : www.LearnEngineering.in


For More Visit : www.LearnEngineering.in

(i) 42°30' (ii) –520° Ex. 2 Convert the following radian i.e.,  1 :  2 = l1 : l2
(iii) 72° 45' (iv) 115° 40' measure in degree measures
l
(v) 15° 12' 30" –5 c 5 c (ii) when = and  is
(i) 4c (ii) (iii) r
x 3 24
Sol. We know that x° = rad constant then l  r i.e.,
180 l1:l2= r1 : r2
c 5 C
1 85 (iv) (v) l
 (i) 42°30' = 42  16 7
2 2 (iii) when  = and l is con-
Sol.  radian = 180° r
 85 180 7 l
=  1radian = = 180° × stant then  
180 2 π 22 r

r
  17  5 17 180  7  4 l
=  (i) 4 radian = or r 

Sni
5  36  2 72 22 
i.e,  1 :  2 = r2 : r1 (reverse or-

gv.i
π –26 90  7  4 2520
(ii) –520° = –520 × = = = der)
180 9 11 11 Ex.3 If in two circles, arcs of the
(iii) 72° 45' same length subtend angle
1

ridna
1° = 60' = 229 degree 60° and 75° at the centre, find
11 the ratio of their radii.

 1 
1' =   5 5 B B
60  (ii) – = – ×180°

eeYa
3 3 r1 r2
60° l O 75°
l
3 = –5 × 60° = –300° O
45' =
4 5 5  180 A A
(iii) = 
Sol. Let the radii of two circles be
3 291 24 24
geisnh

So, = 72 = r1 and r2 respectively.
4 4 75 1 According to the question, arc
= = 37
2 2 AB = l (say) in the two circle.
291 
= × Given that  1 = 60°
4 180   180
Enak

(iv) = 
16 16   60
3  97   97  = 60 × = radian
= = 180 180
4  3  60 240 45 1
R

= =11 75 
4 4 And  2 = 75° = radian
2 180
(iv) 115° 40' = 115
3 5  180 5  180 900
aryn

(v) × = = l l
7  7 7   =  = r and 
r  1 1
2
Les B

 2 347 = 12857'14" l
 40'   = = r
3 3 5. Relation between length of 2
arc (l), radius (r) and angle or, l = r1  1 = r2  2
347  347 (  ):
wa. th

= × = 75
3 180 540 r1 2
P  180
(v) 15° 12' 30" B B or, r 1 = 60
l
 r
2
' ' r
 30   1  q l 180
30" =     O O
wwM

60   2  r 75 5
B A = =
60 4
 1 '  25 ' If an arc of length l of a circle Alternate: -
12' 30" = 12  =  
2 2  subtends  at its centre and since l is constant,
 radius of the circle is r then therefore r1 : r2
 25 1    5 
=    =   =  2 :  1 = 75° : 60° = 5 : 4
2 60  24 l
= r . Ex.4 If the arc of same length in two
 5 
 circles subtends angle 75° and
 15° 12' 30" = 15 24  Hence, 12 0° at their resp ective
l centres, then ratio of their
 (i) when  = and r is diameter is
 365  365  73 r
=   =  = (a) 8 : 5 (b) 5 : 8
24  24 180 864 constant then   l (c) 3 : 5 (d) 5 : 3

Rakesh Yadav Readers Publication Pvt. Ltd. 270

For More Visit : www.LearnEngineering.in


For More Visit : www.LearnEngineering.in

Sol. (a) Sinc e l is c onstant Ex.8 Find the angle in radian


through which a pendulum Sol. (a) B
therefore r1 : r2 =  2 :  1

cm
= 120° : 75° = 8 : 5 swings if its length is 75 cm

30
30

cm
 d 1 : d2 = 8 : 5 and the tip describes an arc
length 18 cm. O 30 cm C
Ex.5 The tip a pendulum swings. It
covers an arc of 50 cm and
subtends 60° at the fixed point. q 75 cm
The length of pendulum is BOC is an equilateral triangle
(a) 43.72cm (b) 45.72cm 18 cm since all sides are equal.
(c) 47.72cm (d) 45.27 cm Sol. Sup pose the p endulum 
Sol. (c) swings through an angle of  = 60° = 3 rad

ir
r q l 18
radian. then  = = rad l 
 = r  l =  r = 3 × 30

v.iSn
r 75
(see figure)
l = 50 cm 22 3
6 = 10 × = 31 cm
 = rad 7 7
 = 60° = 3 and l = 50 cm

dnag
25 Ex.11 In a circle of radius 50 cm the
l Ex.9 In a circle of diameter 50 cm,
length of a chord is 50 2 cm.
 using  = the length of a chord is 25 cm.
r The length of major arc of the
Find the length of minor arc
50cm 150 150

eYari
l and major arc of the chord. chord is
r= =  = cm 22
  (a) 245.5 cm (b) 235.5 cm
3 7 B (c) 255.5 cm
m
150  7 (d) None of these
25 c
25
= = 47.72 cm
22
Ex.6 The minutes hand of a watch
is 5 cm. How far does the tip
move in 20 minutes?
snhe
O 25 cm
cm

A
Sol. (b)
B' A
25 2

B
a
kgei
(a) 10 cm (b) 9.53 cm Sol. See the figure
Given that radius of the circle o 50 cm
(c) 11 cm (d) 10.47cm
Sol. (d) In 20 minute, hand covers 50cm
ERna

20 = = 25 cm and
× 2 2
60 chord AB of the circle = 25 cm 25 2 1
2 Clear ly  OAB is an sina = =   = 45°
= rad distance. 50 2
3 equilateral triangle, therefore
aBryn

1  2  = 90°
From  = , l= r  AOB = 60° Hence major arc of chord BB'
r'  subtends angle
= = (say)
=
2
5 =
10 
=
10 22
× 3  3
3 3 3 7 In minor arc AB = l then from = 360° – 90° = 270° = at centre.
Les

2
220 l
= = 10.47 cm  = r
wa. th

21 l
Ex. 7 When a pendulum of length 50  using  =
25  r
cm oscillates, it produces an l = r =
3 3
arc of 16 cm. The angle so
Here major angle = 360° – 60° major arc l =  × r = ×50
formed in degree measure is
wwM

2
(approx) :  5
= 300° = 300  = = 75  = 75 × 3.14 cm
(a) 18°25' (b)18°35' 180 3
(c) 18°20' (d) 18°08' = 235.50 cm
5 125 
Sol. (c) l = 16 cm Major arc = 25  = 6. If  is in radian and is very small
3 3
r = 50 cm then sin  =  = tan  (approxi-
Ex.10 The diameter of a circle is 60
l 16 8 cm. The length of minor arc mate)
 θ    radian
r 50 25 created by a chord of 30 cm is 7. Area of a sector (or Sectorial
8 180 8 180 1008 area) :-
    7  3
(a) 31 cm (b) 34 cm The are a of the sec tor
25  25 22 55 7
18  18  2 4 1 2
 18  18  60   1820' (c) 32 cm (d) 32 cm OAB  r
55  55  7 7 2

Rakesh Yadav Readers Publication Pvt. Ltd. 271

For More Visit : www.LearnEngineering.in


For More Visit : www.LearnEngineering.in

Here is in radian. length 11 cm.


 135 3
= ×  rad = rad
180 4
B B
Ex.14 Find the angle between the cm
2 00
r minute hand of a clock and the
O q 11 cm
hour hand when the time is
O 5:20 AM. A
r A
(a) 500 (b) 300 (c) 400 (d) 450
Note :- Radian is a constant angle. Sol. Given r = 200 cm,
Some Useful Points :- 11 l = Arc AB = 11cm
The ang le b etwe en two Sol. (c)  M  30 H Suppose angle subtended at
2
consecutive digits in a clock is the centre of circle be  ra-

r
Where  = angle dian
  

Sni
0 M = minute
30 
 radians  l 11
 6  H = hour then,  =  rad

gv.i
r 200
The hour hand r otates 11
through an angle of 300   20  30  5   rad = 180°
2
180

ridna
1 0  l rad = 
in one hour i.e.  = 110º  150º = 40º
  in one
2  Ex. 15 A wheel makes 180 revolu- 7

eeYa
minute. tions in one minute. Through or, l rad = ×180°
22
The minute hand rotates how many radians does it
through an angle of 60 in one turn in one second? Also find 11 11 7
 rad =  ×180°
minute. its degree measure. 200 200 22
Ex. 12 The angle between the hands of Sol.
geisnh
Wheel makes 180 revolutions
a clock at 4 hour 45 minute is in 1 minute. 7  180 7  45 7  9
= = =
200  2 100 20
1 1 180
(a) 112 (b) 122  Wheel makes 60 = 3 revo- 63 3
2 2 = 3
Enak
= = 3 degree
lutions in 1 second. 20 20
1
(c) 125° (d)127 Now,  One complete revolution
2 3
measure 2  radian. × 60 minutes = 3°9'
R

20
11  Three complete revolutions
Sol. (d) Using M = 30 H + A measure 2  × 3 = 6  radian Ex.18 The moon's distance from the
2
aryn

earth is 360000 km and its


Again  rad = 180°
here, H = 4, M = 45 diameter subtends an angle of
 6  rad = 6 × 180° = 1080°
Les B

30' at the eye of the observer.


11 Ex.16 A wheel mak es 2 40 Find the diameter of the moon.
 × 45 = 30 × 4 + A
re volutions p er m inute.
2 (a) 100  km
Through how many radians
247.5 = 120° + A (b) 1000  km
does it turn in 1 second ?
wa. th

 A = 247.5° – 120° = 127.5° (c) 1500  km


(a) 8  (b) 6  (c) 4  (d) 16 
Ex. 13 The angle between the hands (d) 2000  km
Sol. (a) Number of revolutions per
of a clock at half past one is Sol.(b) Diameter d = Arc AB
240 as the distance between moon
3 2 second = 4
wwM

60 and the earth is very large


(a) rad (b) rad
4 3 complete circles. c
 30 π 
5 5
A circle subtends an angle of θ = 30' =  60  180 
(c) rad (d) rad
12 6 2 c at its ce ntr e it 1 c
revolution  π 
= 
11
 Num ber of r adians in 4  360 
Sol. (a) Using M = 30 H + A
2
revolution = 4 × 2  = 8  c arc
θ 
11 Ex. 17 Find the degree and radian radius
× 30 = 30 × 1 + A measure of the angle sub-
2
tended at the centre of a circle  d
 A = 11 × 15 – 30 = 135° of radius 200 cm by an arc of
   d  1000 km
360 360000

Rakesh Yadav Readers Publication Pvt. Ltd. 272

For More Visit : www.LearnEngineering.in


For More Visit : www.LearnEngineering.in

(a) 252cm (b) 504cm


A
moon (c) 300cm (d) 500 cm  30  0 0
  30 '   1
Sol.(b) arc AB = diameter AB =4.4 cm    2
60
   
B
 1  c   c
moon 
    
 
 2 180   360 
30'
arc  4.4
B co
  
in radius 360 r
E (eye)
A

ir
Ex.19 If the angular diameter of the 4.4  360 4.4  360
moon be 30' , how far from the r cm  7

v.iSn
30'  22
eye a coin of diameter 4.4 cm
be kept to hide the moon ?  r  504 cm
E (eye)

dnag
EXERCISE

eYari
1. In radian measure 120° equals 7. Three interior angles of a quadri- 14. If one unit of an angle is
lateral are 60°, 120°, 90°. The re- 15°49'50" then measure of 100
 2 3 4 maining angle in circular mea- units of the angle equals
(a) (b) (c) (d)

2. 37
1
2
3
o
3 4 3

is equal to which of the fol- (a) (b)


snhe
sure is given by :

c c
(c)
c
(d)
3 c
(a) 1580°30'20" (b) 1582°3'20"
(c) 1583°3'20" (d) 1581°30'20"
15. A wheel make 90 revolutions in
3 2 4 4
kgei
lowing radian measure? half hour. Through how many
8. In  ABC,  A = 30°,  B = 60°.
5 7 5 7 deg ree does it turn in one
(a) (b) (c) (d) Find  C in circular measure : minute?
12 12 24 24
ERna

2c 3 c c c (a) 120° (b) 720°


1 (a) (b) (c) (d)
3. 11 is equivalent to the radian 3 4 4 2 (c) 1080° (d) 540°
4
measure 9. Radian value of 560°20' is 16. The angle in degree through
aBryn

which a pendulum of length 100


 3 1481 1481
(a) rad (b) rad (a) rad (b) rad cm savings and the tip describes
8 8 540 360
an arc length of 10 cm is
3  1681 1681 (a) 5°43'38" (b) 7°43'38°
(c) (d) rad (c) rad (d) rad
16 16 540 360
(c) 5°34'18" (d) 7°34'18"
Les

5 10. Radian measure of 72°40' is


4. right angle in radian equals 17.. Find in degrees the angle through
wa. th

6 109  109 
(a) rad (b) rad which a pendulum swings if its
5 5 7  270 180
(a) (b) (c) (d) length is 90cm and its tip describes
24 12 12 12 an arc of length 22cm .
5. 1 radian is equal to:- 219 219
(c) rad (d) rad (a) 140 (b) 13016'
wwM

0
540 360
   11. Measure of 6 rad is (c) 140 8' (d) 130
(a) 100° (b) 
 
180  (a) 343°18'11" (b) 341°18'11" 18. A rail road curve is to be laid out
(c) 341°38'11" (d) 343°38'11" on a circle. What radius should
0
180  12. If 1 rad = 57° 16' 21" then 10 rad
be used if the track is to change
(c) 
  (d) 900 equals
   direction by 25° in a distance of
(a) 570°16'21" (b) 573°43'10"
6. Find the degree measure 40 metres ?
(c) 571°43'40" (d) 572°43'30"
1 c 13. If one unit of an angle is 29° (a) 91.64 metres
corresponding to 
  :- 46'55" then five units of the angle (b) 90.46 metres
6  equals
(a) 9032' (b) 9032'43.6" (a) 148°54'35" (b) 146°54'35" (c) 89.64 metres
(c) 100 (d) None of these (c) 149°34'25" (d) 147°44'35" (d) 93.64 metres

Rakesh Yadav Readers Publication Pvt. Ltd. 273

For More Visit : www.LearnEngineering.in


For More Visit : www.LearnEngineering.in

19. By decreasing 15° of each angle 26. A pendulum of length 60 cm 33. Ananta's (A) and Shailvia's (S)
of a triangle, the ratios of their swings and creates an arc of 18 house are situated at a circular
angles are 2 : 3 : 5. The radian cm. The angle at the fixed point road and subtends 90° at a fixed
measure of greatest angle is : of the pendulum is point. If fixed point is at a distance
1 of 100 metre from each house, the
11    5 (a) 15° (b) 17 distance travelled between the
(a) (b) (c) (d) 2
24 12 24 24 both house on the road is
1 (a) 628 metres (b) 314 metres
20. In a triangle ABC,  ABC = 75° (c) 20° (d) 22
2 (c) 157 metres (d) 235.5 metres
27. Radius of a circle is 54 cm. If an 34. The angle covered by minute

and  ACB = . The circular arc of circle subtends an angle of hand of a watch during 1hour 15
4 20° at centre then length of the minutes noon to half past three

r
measure of  BAC is : arc is noon is
(a) 4.5  (b) 5 

Sni
5  1 4
radian (b) radian (a) 19 (b) 17 cm (c) 4.25  (d) None of these
(a) 7 7

gv.i
12 3 35. The angle covered by hour hand
of a clock from half past six in
6
 5 (c) 18 cm (d) None of these the morning to three O'clock in
(c) (d) 7
24 24 the noon is

ridna
28. An arc of length 40 cm subtends (a) 270° (b) 245°
21. The minute hand of a big wall-
1 (c) 255° (d) 265°
clock is 35 cm long. Taking
22 at the centre of the circle. 36. Assuming that the Moon's diam-
2

eeYa
22 
  , length of the arc, its ex- Radius of the circle is 1
7 eter subtends and angle   at
(a) 92 cm (b) 102 cm 2
tremity moves in 18 seconds is : (c) 96 cm (d) 108 cm the eye of an observer, find how
(a) 11 cm (b) 1.1 cm 29. The minute hand of a watch is
geisnh far from the eye of a coin of 10
(c) 6.6 cm (d) 6 cm 3cm long. How far does its tip cm diameter must be held so as
3 move in 50 minute ? just to hide Moon?
22. Two angles of a triangle are rad (a) 10.32 cm (b) 17.67 cm
2 5 6
(c) 15.71 cm (d) 18.23 cm (a) 112 cm (b) 110 cm
Enak

4 30. Find the angle between the hour 11 11


and rad. The triangle hand and the minute hand at half
3 5 6
past four . (c) 116 cm (d) 114 cm
(a) is an acute angled triangle 11 11
R

(b) is an obtuse angled triangle   37. The earth revolves in its axis in
(a) radian (b) radian
(c) is a right angled triangle 4 6 24 hours. How much angle does
(d) does not form
aryn

2  it move in 4 hours and 12 min-


23. If two angle of a triangle are 2 rad (c) radian (d) radian utes?
3 3
Les B

(a) 63° (b) 64°


1 31. In a circle of diameter 30cm , the
and rad then its third angle in (c) 65° (d) 70°
2 length of the chord is 15cm. Find 38. The angle formed by the hour-
degree is the length of the minor arc hand and the minute-hand of a
corresponding to the chord . clock at 2 : 15 p.m. is
3 3
wa. th

(a) 105 (b) 15


7 7 5 1

1

(a) cm (b) 5 cm (a) 22 (b) 30°(c) 27 (d) 45°


5 9 3 2 2
(c) 105 (d) 36
7 11 1
5
wwM

39. Two angles of triangle are and


24. A wheel revolves 24 times in 10 (c) cm (d) None of these 2
seconds. How many time does it 2
32. The angles of a triangle are in 1
take in revolving an angle of 110 radian. The measure of the
rad? Arithmetic Progression. The ra- 3
(a) 5 sec (b) 7.3 sec tio of the least angle in degrees third angle in degr ee
(c) 10 sec (d) None of these to the number of radians in the  22 
25. Radian measure of 40°20'50" is greatest angle is 60 :  . The  taking    is
7 
angles in degrees are:
481 681
(a)  rad (b)  rad (a) 30°, 60°, 90° 1 2
1196 1296 (a) 132 (b) 132
(b) 35°, 55°, 90° 11 11
581 581 (c) 40°, 50°, 90° 3
(c)  rad (d)  rad (d) 40°, 55°, 85° (c) 132 (d) 132°
2592 1296 11

Rakesh Yadav Readers Publication Pvt. Ltd. 274

For More Visit : www.LearnEngineering.in


For More Visit : www.LearnEngineering.in

40. A wheel rotates 3.5 times in one 45. If the sum and difference of two 49. Consider the following
second. What time (in second) I. The angular measure in ra-

does the wheel take to rotate 55 angles are 135° and respec- dian of a circular arc of fixed
12 length subtending at its cen-
radian of angle?
(a) 1.5 (b) 2.5 (c) 3.5 (d) 4.5 tively, then the value of the tre decreases, if the radius
41. The rad ian meas ure of angles in degree measure are of the arc is increases
63°14'51" is (a) 70°, 65° (b) 75°, 60° II. 1800° is equal to 5 radian.
(c) 45°, 90° (d) 80°, 55° Which of the above statements
c c
 2811   3811  46. The degree measure of 1 radian is/are correct?
(a)   (b)   (a) Only I (b) Only II
8000  8000   22  (c) Both I and II
 taking   
c c 7  (d) Neither I nor II
 4811   5811 

ir
(c)   (d)   (a) 50°16'22" (approx.) 50. How many degrees are there in
8000  8000 

v.iSn
(b) 57°16'22" (approx.) an angles which equals two-
42. When a pendulum of length 50 (c) 57°22 '16"(approx.) third of its complement?
cm oscillates, it produce an arc (d) 57°32'16" (approx.) (a) 36° (b) 45° (c) 48° (d) 60°
of 16 cm. The angle so formed 51. The earth takes 24 h to rotate
47. In a triangle ABC,  AB C

dnag
in degree measure is (approx) about its own axis. Through
c what angle will it turn in 4 h and
1 = 75°and  ACB = . The cir- 12 min?
(a) 22  (b) 18º35' 4
(a) 64° (b) 63° (c) 65° (d) 70°
2
cular measure of  BAC is

eYari
1 1
(c) 27  (d) 45° 5  52. If cos  > in the first quadrant,
2 (a) radian (b) radian 2
12 3 then which one of the following
43. A rail road curve is to be laid
is correct?
out on a circle. What radius
should be used if the track is to
change direction by 25° in a dis-
tance of 40 metres.
48.
(c)
5
6 snhe
radian (d)
5
2
radian

At what point of time after 3


(a)  <

3
(b)  >

3
O' clock hour hand and the
kgei
(a) 91.64 m (b) 81.23 m  
minute hand of a clock occur (c)  < (d)  >
(c) 95.67 m (d) 98.72 m at right angles for the first 6 6
44. An arc of a circle of radius 42 time? 53. What is the angle (in radian) in-
ERna

cm subtends an angle of 15° at (a) 9O'clock cluded between the hands of a


clock, when the time is 10 min
22 1
the centre. Taking  = ' the (b) 4th 37 min past 5?
7 6 (a) 17  /36 (b) 19  /36
aBryn

length of the arc is: (c) 5  /9 (d) 7  /12


8
(c) 3h 30 min 54. If clock started at noon, then
88 11
(a) cm (b) 11 cm what is the angle turned by hour
5 hands at 3 : 45 pm?
8
14 (d) 3h 32 min (a) 67.5° (b) 97.5°
11
Les

(c) 12 cm (d) cm (c) 112.5° (d) 142.5°


5
wa. th
wwM

ANSWER KEY
1. (b) 7. (b) 13. (a) 19. (a) 25. (c) 30. (a) 35. (c) 40. (b) 45. (b) 50. (a)
2. (c) 8. (d) 14. (c) 20. (b) 26. (b) 31. (b) 36. (d) 41. (a) 46. (b) 51. (b)
3. (d) 9. (c) 15. (c) 21. (b) 27. (c) 32. (a) 37. (a) 42. (b) 47. (b) 52. (a)
4. (b) 10. (a) 16. (a) 22. (a) 28. (b) 33. (c) 38. (a) 43. (a) 48. (d) 53. (b)
5. (c) 11. (d) 17. (a) 23. (d) 29. (c) 34. (a) 39. (c) 44. (b) 49. (a) 54. (c)
6. (b) 12. (d) 18. (a) 24. (b)

Rakesh Yadav Readers Publication Pvt. Ltd. 275

For More Visit : www.LearnEngineering.in


For More Visit : www.LearnEngineering.in

SOLUTION

 14. (c) 100 units


120  20 
1. (b) 120° =   rad 9. (c) 560°20' =  560  = (15° 49' 50") × 100
180 60  = 1500° 4900' 5000"
2  = 1500° + (81° + 40') + 83' + 20"
= rad  1
3 =  560  = 
 4900 40 5000 20 
3  1681 
    81 ,  83 

3 60 60 60 60 
1  75 
2. (c) 37 =   = 1500° + 81° + 40' + 1° + 23' + 20"
2 

r
2 1681  1681
= × =  rad = 1582°63'20" = 1583°3'20"

i
.iSn
3 180 540
75 5 5
=  = = rad  
90
2  180 2  12 24  40   2 15. (c) Wheel revolves =
10. (a) 72°40' =  72  =  72  30
 60 3
1  45  = 3 turn in one minute

agv
3. (d) 11 =    1 turn = 360°
4 4   218 

 218  
=        3 turn = 1080°
180 

ridn
3 3
 45  l 10
  16. (a) From  = ,  =
4    109  109 r 100
   rad  rad =   = rad

eeYa
180 16 3  90  270 
11. (d)  rad = 180° 1 180 18  7 
5 = rad = =  
4. (b) right angle 10   10 22 
6 180
 1 rad =  63 

 8

geisnh 
5  5 =   =  5 
=  rad = 11 11
6 2 12 6  180 6  180  7
6 rad = =
 22 8   480' 
5. (c)  rad = 1800 = 5   60' = 5° 
11 11 

21  180 3780
Enak

 = =
 180  11 11 '
7 
1 rad     7
   = 5°  43  = 5°43'   60 "
  11  11
 7  420 '
R

c 0 0  343  = 343°   = 5°43'38" (approximate)


1   1 180   1 180  11 11 
6. (b)           7  17. (a) r = 90 cm and
 6   6    6 22 
aryn

 2 ' arc (s) = 22 cm


105  0 = 343°  38 
11
 S  c  22   11  c
Les B


 
 11    
      
 
 120 "  r   90   45 
= 343°38'  
 ' ' 11 
 6  6   8
  9   9  60   9 32  0
 11 180   11 180 
0

 11   11   11  = 343°38'11" (approximate)  7   140


wa.th


    
 
Second Method,  45    45 22 
"
 8   1 rad = 57°16'22" 18. (a) θ = 25°
 932'   60   932'43.6"
 11  6 rad = 57° × 6 + 16' × 6 + 22" × 6
= 342° + 96' + 132'' 25  
wwM

7.(b) Fourth angle of quadrilateral = radians


= 360° – (60° + 120° + 90°) = 342° + (1° + 36') + (2' + 12") 180
= 90° = ( 1° = 60' and 1' = 60")
5
 180° =  c = 343° 38'12" (approximate)  radians
12. (d) 10 rad = (57°16'21") × 10 36
c = 570° + 160' + 210"
90° = s
2 = 570° + 2° + 40' + 3' + 30" θ
r
8. (d)  C = 180° – 30° – 60° = 90° = 572°43'30"
13. (a) 5 units = (29°46'55") × 5 s 40 40  36 40  36  7
 180° =  radian r     metre
 5 5 5  22
= 145° 230' 275"
  36
 90   90  radian = 145° + 3°+ 50' + 4' + 35"
180 2 = 148° 54' 35"  91.64 metre

Rakesh Yadav Readers Publication Pvt. Ltd. 276

For More Visit : www.LearnEngineering.in


For More Visit : www.LearnEngineering.in

19. (a) 2x + 3x + 5x = 180° – 45°= 135°


45  9 405 9  45 
 10x = 135° = = = 36 1   
11 11 11 2 
22 =   rad = rad
135 27 2 180 8
x  24
10 2 24. (b) The wheel makes
10 l l 40
 Largest angle rotation in one second  = r = =
r  
 
 27 24 8
 5x  15  5   15 It covers × 2  rad angle in
 2 10
320 320  7
1 second. = = = 101.8 cm
135  30 165 Hence in covering 110 rad,  22
 

ir
wheel takes 29. (c) The minute hand complete
2 2
one revolution in 60 minute.

v.iSn
 180   radian 110 110  10  7  In 50 minute it will cover
24 =
165   165 11  2 24  2  22
    radian 10 50 5
2 180 2 24  of the revolution.
60 6

dnag
= 7.3 second
20. (b) ABC  75
 180    radian  50  '  1 revolution = 2 radian.
25. (c) 40°20'50" = 40°  20 
60 
 5 5 5 5

eYrai
 75   75  radian   revolution  2 
180 12  125 '  125  6 6 3
= 40°   = 40°  
6  6  60  radian
 5
 BAC =  – 4 – 12  
 Dis tanc e moved by tip


12 – 3 – 5 4 
12

12
 radian
3

=  40

2905
25 


snhe
 2905 
 = 
72  72 

 3
5
3
cm  5 cm

=  rad
kgei
21. (b) Traced arc length by minute 22
72 180
hand in 60  60 seconds = 2  r  5 cm  15.71 cm
 Length of arc made in 18 sec- 581 581 7
= = rad 30.(a) Angle traced by the hour hand
ERna

2r 72  36 2592
onds   18 in 12 hours = 3600
60  60 l 18 3
26. (b)  = ,  =  rad  Angle traced by the hour
r 60 10 hand in 4 hrs 30 min.
22 35  18
 2   1.1 cm = 0.3 rad
aBryn

7 60  60  9  360 9
1 rad = 57°16'22" (approximate)
  hrs  
   1350
0.3 rad = 5.7° × 3 = more than  2  12 2
22. (a) 1 right angle = rad
2 17° and less than 18°
= 1.57 rad (approximate) Angle traced by the minute
1 hand in 60 min. =3600
3  From options = 17
Les

= 1.5 rad, which is an acute 2  Angle traced by the minute


2
wa.th

angle. 20 
hand in 30 min
27. (c) 20° =  = rad
4 180 9  360 0
= 1.33 rad, which is an acute
3
l 

  30   1800
angle From,  = l =  r = × 54  60 
wwM

Third angle = (  – 1.5 – 1.33) r 9


Thus, the angle between two
and = (3.14 – 1.5 – 1.33) 22 hands = 180º – 135º = 45º
=  54 cm
= 0.31 rad which is also an 79
acute angle. 
23. (d) Third angle 22  6 132 6  radian
= = = 18 cm 4
7 7 7
1 Alternate:-
=  rad – rad–2rad 28. (b) l = road
2
A S 11
 22 5  9  M  30 H
=  –  rad = rad 2
7 2 14
9 180 9 180  Where  = angle
=  =  ×7 M = minute
14  14 22

Rakesh Yadav Readers Publication Pvt. Ltd. 277

For More Visit : www.LearnEngineering.in


For More Visit : www.LearnEngineering.in

H = hour From 1 hour 15 minutes to half 11


past three, minute hand cov- Hence, × 15 = 30 × 2 + A
11 2
  30  30  4 ers 2 hours 15 minutes i.e.,
2 1 165 1
2 rotations. or, A = – 60 = 82 – 60
4 2 2
= 165º  120º
1 1

 It covers 2 × 2  = 22
 4 2
= 45º  radian = (4.5)  rad distance.
4 35. (c) From half past six in the morn- 1
Therefore, angle will be 22
31.(b) OA = OB =15 cm (radius) ing to 3 o'clock at noon, time 2
and chord AB =15 cm elapsed is 8 hours 30 minutes. 39. (c) Sum of two angle be

r
 OAB is an equilate ral Since hour hand covers 30° in 1 1 5
  rad

i
.iSn
5 minute therefore it covers =
triangle. 2 3 6
1 1
30° × 8 = 255° in 8 hours. 22  22 
2 2  rad = 180°    
O 7 7 

agv
36. (d)
5 180 5
A B  6 rad = 22 × 7× 6
moon

ridn
0
 c =
30  35 15  35
=
  AOB  60  
  22 11

eeYa
3 Remaining angle
B co
in
arc  15  35 525
 arc r  155 cm A = 180° – = 180° –
radius 3 11 11
30'
32.(a) Angles of triangle
geisnh 8 3
= 180° – 47 =132
(a – d)°, a°, (a + d)° 11 11
a – d + a + a + d = 180° E (eye) 40. (b) 1 rotation = 2  radian
arc AB = diameter AB = 10cm 3.5 rotation = 3.5 × 2  radian
 3a  180  a  60
Enak

o c
 1 1     22
a – d 60 60 1 = 3.5 × 2 × = 22 radian
     =  2  = 2 × 180 =  360  7
a d  180 3
 Wheel rotation in one
R

60 – d 1 l  10 second is 22 radian
   = r  
60  d 3 360 r  Wheel rotation in 55 radian
aryn

55
 180 – 3d = 60 + d 360  10  7 6 = 2.5 second.
r = = 114 cm 22
Les B

 4d = 120°  d = 30° 22 11
 Angles of triangle : 37. (a) Revolution in 24 hours  51  '
41. (a) 63°14'51" = 63 14 
a – d = 60° – 30° = 30° = 360° 60 
a = 60°  Revolution in 1 hours '
wa.th

a + d = 60 + 30 = 90°  17 
360 = 63 14 
= = 15° 20 
 24
33. (c) 90° =
2 Revolution in 4 hours  297  '  297 
= 63   =  63 
= 15° × 4 = 60° 20  20  60 
wwM

l
 = r or l =  r  Revolution in 60 minutes
 99   99  
= 15° =  63  =  63 
 Revolution in 12 minutes 20  20 400 
= ×100 metres
2 
15  12  25299 
3.14 =  3 =  
=  100 =157 metres 60 400 
2  Revolution in 4 hours 12
34. (a) 1.15 minutes = 60° + 3° = 63°  25299  
=    rad
q 38. (a) From Trick, 400 180 
3.30 11 
using, = M = 30H + A  2811   2811 
2 =   =  
400  20 8000 
Here, M = 15, H = 2

Rakesh Yadav Readers Publication Pvt. Ltd. 278

For More Visit : www.LearnEngineering.in


For More Visit : www.LearnEngineering.in

Trick, Value of 63°14'51" is 60°


l 11x
(approximate) From  = , l =  r  =180°
r 2
 Value of 63°14'51" should be
  22 1 360 8
more than =0.33  or, l = × 42 = × × 42  x= = 3h 32 min
3 12 7 12 11 11
From option (a), = 11 cm 49. (a) I. Arc length l = r 
45. (b) A + B = 135° ....(i)
 2811   2800   l
  =   (approx)  180 & for constant l,  = so if the
8000  8000  A–B= × = 15° ...(ii) r
12  radius increases,  decreases
 28  (i) + (ii) 2A = 150

ir
=    = 0.35  (approx) 
80  A = 75° II. 1800° = 1800 × = 10 

v.iSn
B = 135° – 75° = 60° 180
So, option (a) is correct.
180 50. (a) Given,  +  = 90°
42. (b) 46.(b) 1 radian = 1 × degrees
q  (complementary angles)
50

dnag
 2
180 630  3 &  = 
= ×7= =  57  3
22 11 11
16 2
3  or  = (90° –  )
= 57° +   60  '

eYrai
l 16 3
 = r   = 50 rad 11
2

 180  '  4 '   = 60° –
 16 180  = 57° +   = 57° + 16  3
=    11 11
50  

 16  18 7 
= 
5
 
22 
 snhe
4 "
= 57° + 16' +   60 
11

5
3
= 60°   = 36°

51. (b) In 24h, Earth rotate 360°


kgei
= 57°16'22" (approx.)
 16  9  7 

 1008 

360
  =   47. (b) Given,  ABC = 75°,  In 1 hr, Earth rotate = 15°
5  11  55  24

   180 
ERna


 ACB = rad=    = 45°  12 
 18  4 4   In 4h 12 min =  4   hr
18  = 18°35' (approx) 60 
55 
 BAC = 180° – (75° + 45°) = 60°
43. (a) 21
r    = hr,
aBryn

5
=  60   radian
25° 180 
l = 40 m. 21
Earth rotate = 15 × = 63°
 5
= radian
3 52. (a) If  decreases then
 5
Les

25° = 25 × rad = rad 48. (d) In 60 minutes, minute hand cosθ increases
180 36 rotate 360°
wa.th

In x minutes, minute hand 1


l 5 40 cos  >
From  = , = 360 2
r 36 r
rotate x = 6x
60  
40  36 cos  > cos = <
wwM

 r= In 60 minutes, hours hand ro- 3 3


5
tate 30° 53. (b) At 5 'o' clock, hour hand ro-
1008 In x minutes, hour hand rotate  5
= = 91.64 meter tate = 5 × =
11 30 x 6 6
x= In 60 min hour hand rotate
44. (b) 60 2
42 Initial angle of hour hand at 30' 
15° l clock is 90° and minutes hand = 30° =
6
is 0° 10 min hour hand rotate
Angle after x minutes
10  
15  = × =
15° = ×  =  x 60 6 36
180 12 = 6x –  90   = 90
2 Total angle of hour hand

Rakesh Yadav Readers Publication Pvt. Ltd. 279

For More Visit : www.LearnEngineering.in


For More Visit : www.LearnEngineering.in

Angle between two hands In 45 minute, hour hand rotate


5  30   31
= + = = 45
6 36 36 36 31  31 – 12 19 
= – = = =  30 = 22.5°
Angle rotate by minute hand in 36 3 36 36 60
10 minute 54. (c) In 12h, hour hand rotate = 360° Total angle turned by hour
In 1h, hour hand rotate = 30° hands at 3 : 45 pm = 90° + 22.5°

= 60° = In 3h, hour hand rotate = 90° = 112.5°
3

i r
agv
.iSn
ridn
eeYa
geisnh
Enak
R
aryn
Les B
wa.th
wwM

Rakesh Yadav Readers Publication Pvt. Ltd. 280

For More Visit : www.LearnEngineering.in


For More Visit : www.LearnEngineering.in

CHAPTER

MENSURATION
16
2-D (TWO DIMENSIONAL)

ir
v.iSn
2-D(FLATSHAPE) 3–D

dnag
eYari
Triangle Quadrilateral Polygon Circle

Parallelogram Trapezium
snhe
kgei
Rectangle Square Rhombus
ERna

Area:- The area of any plane figure Quadrilateral : A closed Length


is the amount of surface enclosed figure bounded by four sides.

Breadth
within its bounding lines. It is al- (i)  A +  B +  C +  D = 360°
ways expressed in square units e.g.
aBryn

c
D C
square metres, square inches etc.
Perimeter :The perimeter of a geo- h1 N (1) Its diagonals are equal &
d
metrical figure is the total length of b bisect each other.
the sides enclosing the figure. h2
(2) Area = Length × Breadth
Les

M
Note:- A B (3) Perimeter = 2 (Length +
a
wa. th

1 hectare = 10,000 m2 Breadth)


1
100 hectare = 1,000,000 m2 (ii) area of quadrilateral = × (4) Diagonal (d) = l 2  b2
2
Weight (mass) = Volume  density. (5) (i) Area of a path inside
one diagonal × (sum of per
What is Quadrilateral? a rectangular field:-
wwM

pendicular to it from opposite


A quadrilateral is a four-sided Area of path = 2x (l + b –2x)
vertices)
polygon with four angles. There (ii) Perimeter (P) = inner P
1 + Outer P
are many kinds of quadrilater- = (AC )(h1 ? h 2 )
als. The sum of the angle of 2 = 2(l + b) + 2( l + b - 4x)
quadrilateral is 360º. (iii) P = a + b + c + d = 4(l + b - 2x)
Types of quadrilaterals Rectangle Length
A rectangle is a four sided flat shape
(1) Rectangle (2)
where every angle is a right angle (90º)
Breadth

Square (3) also opposite sides are parallel and of


Parallelogram (4) equal length. A rectangle has two di-
Rhombus (5) agonals, they are equal in length and
Trapezium interesect in the middle.

Rakesh Yadav Readers Publication Pvt. Ltd. 281

For More Visit : www.LearnEngineering.in


For More Visit : www.LearnEngineering.in

(6) (i) Area of path outside a Length = 5x = 5×4 = 20,  230×210 = 48,300 m²
rectangular field:- Breadth = 7x = 7×4 = 28 Area of Path outside ABCD
Area of path outside = 2x = Area of PQRS – Area of ABCD
Diagonal = 202  282
(l + b + 2x)  48,300 – 36,000
(ii) Perimeter (P) = inner Pe- = 400  784 = 1184 m.  12,300 m²
rimeter + outer Perimeter 3. The ratio between the length Alternate:-
= 2(l + b) + 2(l + b + 4x) and width of the rectangular Area of Path outside
= 4 (l + b + 2x) field is 3 : 2. If only length is = 2x (l +b+2x)
increased by 5m. The new area  2×15 (200+180+30)
of the field is 2600m2. What is  30×410  12,300 m²
the width of the rectangular 6. A path of uniform width runs

r
B field? mid-way of the Rectangle field
x

Sni
(a) 60 (b) 50 (c) 40 (d) 65 having lenght 100m & Breadth
L 50m . If the p ath occup ies
Sol. Let length = 3x, then width = 2x

gv.i
x 1400m². then the width of the
 (3x + 5)2x = 2600 path is?
(7) (i) Area of path midway = x (l  (3x + 5)x = 1300 (a) 5 (b) 10 (c) 12 (d) 8
+ b – x) we, go through the option

ridna
Sol. Area of path midway = x (l + b – x)
(ii) Perimeter of Path (P) = 2(l option (c) 2x = 40  x = 20 1400 = x (100+50–x)
+ b) - 4x = 2( l + b – 2x) which satisfy the above equa- 1400 = 150x –x²
L tion x² – 150x + 1400 = 0

eeYa
 width = 2x = 40m  x² – (140+10)x +1400 = 0
Note : you can also solve the  x² – 140x – 10x +1400
above equation.  x (x –140) – 10(x –140)
x B 4. The length of rectangle, which = (x –140) (x –10)
geisnh
is 24cm is equal to the length  x  140 (is not possible be-
of a square and the area of the cause breadth is less than
rectangle is 176cm less than width)
(8) Room as a Rectangular figure:- the area of the square. What x =10
Enak

Area of four walls of a room is the breadth of the rectangle? Alternate:-


= Perimeter × Height = 2 × (L + Sol. Area of square = (side)2 = (24)2 In these question we take op-
B) × H = 576cm2 tion (b)
R

(9) Area of Roof and 4 walls Area of rectangle = length  = 10(100 + 50 – 10)
= 2H (L+B)+LB breadth = 576 –176 = 24 × x = 10 × 140 = 1400
So, (b) is correct.
aryn

(this formula can be use when  Breadth of rectangle


we have to paint a whole 7. Find the perimeter of a path in
400 50 2 the sixth question?
Les B

room.) = = = 16 cm
24 3 3 Sol.
EXAMPLES
5. A street of width 15 metres sur- 100m
1. Area of a rectangular field of rounds from outside a rectan- A A B B
breadth 15 cm is 180 sq. cm. gular gard en whose
wa. th

Find the length and perimeter measurment is 200 m × 180 m.


M N
of a rectangle The area of the path? P Q
Sol. Area = Length × Breadth 10 50m
180 = x × 15 S R
Sol. P Q P O
wwM

180 15 x
x= = 12 cm = length A B
15
Perimeter  2 (length + Breadth) D C C
 2 (15+12) = 54 cm L = 100m
2. Area of a rectangular field is 560 15
D C B = 50 m
sq. metre. Ratio of their length
& Breadth is 5:7. Find the x = width of path = 10m.
diagonal of a rectangle? S R Perimetor of path = permeter of
Sol. Area = Length × Breadth ABCD + Peremeter of PQRS –
Area of Rectangle ABCD = L × B Peremeter of MNOP
560 = 5x × 7x
 200×180=36,000 m² = 2 (B + x) + 2 (L + x) – 2 (x + x)
560 = 35x2
Area of Rectangle PQRS = 2L + 2B + 4x – 4x
x2 = 16
= (200+30)×(180+30) = 2(L + B) = 2(100 + 50) = 300
x =4

Rakesh Yadav Readers Publication Pvt. Ltd. 282

For More Visit : www.LearnEngineering.in


For More Visit : www.LearnEngineering.in

8. A rectangular park 60  40m2 11. A room 8m long, 6m broad and * Every Square is a Rhombus but
has two cross roads running 3m hig h h as two every Rhombus is not a square.
in the middle of the park and 1 a
the rest park has been lawn. If windows 1 m  1m and a door
the area of the lawn is 2109m2. 2
What is the width of the road? 1 d
Sol. x 2m× 1 m. Find the cost of pa- a a
2
pering the walls with paper 50
x 40m cm wide at 25 paise per meter:
Sol. Area of walls = 2(length +
a
60m breadth)  height = 2(8 + 1. Area = a² = (side)²
Total area of park = 60 × 40 6)  3 = 84m2

ir
1
= 2400m2 Area of two windows and a = × (diagonal)²

v.iSn
door 2
and area of lawn = 2109m2 2. Perimeter = 4a
(given)  1   1 2
= 2
1 ×1
+ 
2 ×1 
 = 6m 3. Diagonal (d) = 2 a
area of the cross roads = 2400-  2   2
4. Area of Path Inside Square

dnag
2109 = 291m2  Area to be covered = 84 – 6 = 4d (x–d)
 x(60  40  x )  291 = 78m2  d = length of Path
 Area of paper = Area to be
 x 100x  291  0
2
x = length of Square
covered = 78 m² 5. Area of Path outside Square

eYari
 (x  97)(x  3)  0  (length  breadth) of paper = = 4d (x+d)
 x  3 or 97 78 6. Area of Path midway Square
 len gth of paper = = d (2x–d)
 x3
Side

9.
[ x  97 is not possible]
A rectangular lawn 60  40m2
has two roads each 5m wide
78
50 snhe
100m = 156cm

156  25
7.

8.
In circle Radius =

Circumcircle Radius =
2
Side
 cost = = ` 39
kgei
running between the park. 2
100
One is parallel to length and 12. The dimensions of a room are Some-useful Results :
other is parallel to width. Cost 12.5 metres by 9 metres by 7 C D
ERna

of gravelling is 60 paise/m2. metres. There are 2 doors and


Find the total cost of gravel- 4 windows in the room, each
ling the path ? door m easure s (2 .5 ×1 .2 ) O
a/2
x = 5m me tr es and e ach wind ow
aBryn

(1.5×1) metres. Find the cost A B


of painting the whole room at a
(I)
x 1` 3.50 per square metre.
b = 40m R S
Sol. Area of 2 doors & 4 windows
= 2 (2.5×1.2) + 4 (1.5×1)
Les

l=60m  6+6 = 12 m²
wa. th

Sol. Area of path = x (l + b - x) Area of roof and 4 walls


= 5(60 + 40 - 5) = 2H (L + B)+ LB
= 5 × 95 = 475m2 = 2×7 (12.5+9) +12.5×9 P Q
a
60 = 14×21.5 + 112.5 (ii)
wwM

 Total cost = 475 × 100 = 413.5 m² 9. In figure (i) ABCD is a square


Area of painting = 413.5 –12 of side ‘a’
= ` 285 = 401.5m²
10. A room 8 meters long, 6 metres (a) O is the centre of the
Total Expense of painting incircle.
broad and 3 metres high. Find
the area of Room? = 401.5 ×3.5 In figure (ii) PQRS is a
Sol. As room consist of floor, roof = ` 1405.25 square of side ‘a’
& 4 walls, then the area of Square (b) P,Q,R and S are the cen-
room will be = Total surface tres of four quad
area of room A square is a four-Sided flat -rant of radius a/2 each.
 2 (l + b) × h + 2l b shape where every angle is 90º and all
In both case– Area of shaded
 2[(l + b)h + l b] the four sides are equal also the di-
 2[3(14)+48] agonals are equal and bisect each 3 2
region  a
 2[42+48] = 180 m² other at 90º. 14

Rakesh Yadav Readers Publication Pvt. Ltd. 283

For More Visit : www.LearnEngineering.in


For More Visit : www.LearnEngineering.in

10. If the additional of square in- BD = 5 cm square made by joining the


creases by x times, then the 1 mid-points of the sides of the
area of the square becomes x2 Area of Square = ×(diagonal)² larger square ABCD. There is
2
times. inscribed a circle, In  PQRS
11. If the area of the square is a 1
= × (5)² = 12.5 cm² and an equilateral  XYZ in-
cm2, then the area of the circle 2
formed by the same perimeter scribed in the circle.
4. A square plot is 200 m long. It
Find the ratio of the side of the
4a has a path 2.5 m wide all round
is = cm2 . it inside. Find the area of path? square ABCD to the side of the
 equilateral triangle XYZ.
Sol. P Q
EXAMPLES 2.5
A B

r
1. A square field has an arm of R
length 125 cm. Find the area D C

Sni
X
and Perimeter of a square?
Sol. Side (a) = 125

gv.i
195 200
Area = (side)² = (125)² S Q
= 15625 cm² Z
Y
Perimeter = 4×side

ridna
D 2.5 C
= 4×125 = 500 cm. A B
2. A square park has a side of 16 S R P
Area of Path Inside = Area of
cm. A person cross it across
Sol. Let side of ABCD = 2a

eeYa
Square PQRS – Area of Square 
diagonally. Find the distance he
ABCD  side of  PQRS =
covered?
 (200)² – (195)²
Sol. According to the question,
 40,000 – 38025  1975 m²
A B
AP 2  AS 2  a 2  a 2  2 a
Alternate:-
geisnh 
16 Area of path inside = 4d (x –d)
Square = 4×2.5 (200 –2.5) 2a a
= 1975 m²  radius of circle  
D C 2 2
16 5. The length of the side of a
Enak

Here  BCD is an right angle square is 14cm. Find out the Let side of  XYZ = b
triangle. ratio of the radii of the in-  radius of circumcircle of
B scribed and circumscribed
R

circle of the square. b


 XYZ =
? 3
16 Sol.
aryn

A B
b a a 2
 
Les B

D  
C 16
3 2 b 3
E
In Square diagonal is Angle O
Bisector. 2a 2 2
 
 D =  B = 45º D C
wa. th

b 3
P
sin D = = sin 45º 7. In the figure, ABCD is a square
H AB
Radius of incircle = OE = with side 10 cm. BFD is an arc
16 1 16 2
of a circle with centre C. BGD
wwM

= = = = 7cm
x 2 x is and arc of a circle with cen-
Radius of circum-circle = OB tre A. What is the area of the
 2 ×16 = 16 2 cm
Alterate:- Diagonal BD shaded region :
=
d = 2  a = 16 2 2
10
3. Find the area of a Square whose
2  14
diagonal is 5 cm? =  7 2cm
2 F
A B
Sol. 10
 Required ratio  7 : 7 2  1 : 2 G
5 6. In the given figure ABCD is a
D C square and PQRS is also a

Rakesh Yadav Readers Publication Pvt. Ltd. 284

For More Visit : www.LearnEngineering.in


For More Visit : www.LearnEngineering.in

Sol. Area of portion DFBGD = Area Sol.


of portion DFBC – area of 2 A B
 Let Base = x ; Altitude = x
 BCD 3
E 16 y
Area of Parallelogram = Base ×
1 2 1 10
r – ab sin θ Height
4 2
2
1 1 486 = x × x D C
= (10)2  10 10 3
4 2 Area of Parallelogram
486  3 ABCD = B×H
= 25  50 x² = = 729
2 160 = x ×10
= Area of portion DFBGD x = 27 x = 16 cm
 Area of portion DFBGD then Base = 27 cm Area of Parallelogram ABCD = B×H

ir
= (25  - 50) + (25  - 50)
2 160 = y × 16

v.iSn
= 50  - 100 Altitude = 27 × = 18 cm
3 y = 10 cm
Parallelogram 2. One side of a Parallelogram is Length of the parallelogram = 16 cm
19 cm. Its distance from the Breadth of the Paralleogram = 10 cm
If opposite sides of a quadri-

dnag
opposite side is 15 cm. Then 6. In a ||gm, the lengths of ad-
lateral are parallel, it is called
parallelogram. Its opposite area of the parallelogram will be: jacent sides are 11 and 13cm.
sides are also equal in length. 19
If the length of one diagonal is
Sol. 16cm, find the length of other

eYari
b diagonal.
D C
15 Sol. In a ||gm, d12 + d22= 2(a2 + b2)
 2
 16 + d2 = 11 + 13  × 2
2 2 2
a h a

A b B Height snhe
Area of Parallelogram = Base ×

 19×15  285 cm²


 d 22 = 2(290) – 256  d 2 = 324
 d = 18cm
(i) Area = base  height = bh 3. In a parallelogram, the lengths
kgei
7. Sides of a parallelogram are in
(ii) Perimeter = 2(a + b) of adjacent sides are 12 cm and
the ratio 5 : 4. Its area is
(iii) d12 + d22 = 2 (a2 + b2) (d1,d2 14 cm respectively. If the length
1000sq.units. Altitude on the
one diagonal is 16 cm, find the
greate r si de i s 20units,
ERna

= length of diagonals)
length of other diagonal?
(iv) Area = A Altittude on the smaller side is:
Sol. d1²+d2² = 2 (a²+b²)
Sol. Let the side of parallelogram
d1²+(16)² = 2(12²+14²)
= 2 s(s  a )(s  b)(s  d ) be 5x and 4x
d1²+256 = 2 (144+196)
Area o f paral lelo gram =
aBryn

where a & b are adjacent sides, d1² + 256 = 2×340 = 680


Base  height
d is the length of diagonal con- d1² = 680 – 256 = 424
1000 = 5x  20
necting the ends of the two d1 = 20.6 cm
sides and, 4. The two adjacent sides of a par- 1000
allelogram are 12 and 9 cm and  x= =10
5  20
Les

a bd the length of diagonal is 15 cm.


s Sides = 50 and 40units
wa. th

Find the area of Parallelogram? 


2
9  40  h = 1000
Sol.
b 1000
 h= = 25 units
12 15 40
wwM

d
a Rhombus
It is a Quadrilateral whose all
12  9  15 four sides are equal. Diagonal
S= = 18
EXAMPLES 2 bisect each other at 90º.

1. The area of a Parallelogram 486 Area = 2 18  6  9  3  2×54


cm². If its altitude is 66.66% of = 108 cm²
its base then find the base and 5. A parallelogram has an area of
altitude? 160 cm². If the distance between
its opposite sides are 10 cm and
2 2 16 cm. Find the sides of the
Sol. 66.66% = 66 %= ,
3 3 Parallelogram.

Rakesh Yadav Readers Publication Pvt. Ltd. 285

For More Visit : www.LearnEngineering.in


For More Visit : www.LearnEngineering.in

1. Diagonals bisect each 1


5. Area of Rhombus is 256 cm2.
other at 90°  × 20 = 10 cm One of the diagonal is half of the
2 other . Find the perimeter and
1 3. In a Rhombus, the length of two sum of the diagonals?
2. A ( d1  d 2 ) diagonal are 120 cm & 90 cm Sol. Let the one of the diagonals is
2 respectively, find its perimeter. (d1) = x
1 Sol. D1 = 120 cm other diagonal (d2) is = 2x
3. a  d12  d 2 2 D2 = 90 cm
2 1
1 1 Area of rhombus = ×d1×d 2
4. P = 4a Side (a) = D12 +D22 =
2
120 + 902
2
2 2
2 2
2
5. d1  d 2  4a 
1 256 =  x  2x

r
=  150 = 75 cm 2
2
EXAMPLES

Sni
Perimeter = 4 × 75 = 300 cm x2 = 256
1. The area of Rhombus is 24 cm² 4. One of the diagonals of a Rhom- x = 16

gv.i
has one of its diagonal is 6cm. bus of side 5 cm measures 8 cm. d1 = 16 cm, d2 = 32cm
Find the other diagonal? Find the area of the Rhombus. Sum of the diagonals = (16 + 32)
Sol. = 48 cm
A 5 cm B

ridna
1
Sol. Area =
2
×d1×d2 Perimeter = 2  d12 +d22 

4
1 O = 2  256+1024 

eeYa
24 = × 6×x
2
48

=2 1280 = 71.55 cm.


24  2 D C
=x 6. Perimeter of a Rhombus is 2p
6 unit and sum of length of
In Δ AOB
geisnh
x = 8cm diagonal is m unit, then area of
2. Find the side of a Rhombus A the Rhombus is
whose diagonals are12 cm and
2P P
16 cm. Sol. Side of Rhombus = =
5 4 2
Enak

Sol. As diagonals of Rhombus bisect ?


each other at 900 d1 + d2 = m (Given)
2 2 2
B  P   d1   d2 
O
R

A B 4 
   
   
  
2 2 2
8
By phythagoras theorem,
aryn

(AB)2 = (AO)2 + (OB)2 P 2 d12 d22


6  
O 25 = (AO)2 + 16 4 4 4
Les B

AO2 = 9 P² = d12 + d22


D C
AO = 3cm 2 2
 (d1 + d2)2 = d1 + d 2 +2d1 d2
AS AO = 3cm, then AC = 2 × AO
In Δ AOB,  m2 = p2 + 2 d1d2
= 2 × 3 = 6 cm
wa. th

D1 = 8 cm, D2 = 6 cm m2 –p2
A  d1d2 =
2
1
Area = ×D1×D2 1
2 d1 d2
Area of Rhombus =
wwM

8 ? 2
1
= ×8×6=24cm2
2 m 2 – p2
=
Alternate : d1² + d2² = 4a² 4
O B 7. The perimeter of a rhombus is
6 8² + d2² = 4 × 5²
(AB)2 = (AO)2 + (OB)2 d2² = 36 146 cm and one of its diago-
(AB)2 = 64 + 36 d2² = 6 nals is 55cm. The other diago-
AB = 10 cm
nal is :
1
Alternate:- area = d ×d Sol. Perimeter  4  Side
2 1 2
1 2 2 1 1 2 2
Side, a = d1 +d2 = 162 +122 1  4  . d1  d 2  2 d1 2  d 2 2
2 2 = × 8 × 6 = 24 cm² 2
2

Rakesh Yadav Readers Publication Pvt. Ltd. 286

For More Visit : www.LearnEngineering.in


For More Visit : www.LearnEngineering.in

(iv) If diagonals intersect at O, 4×6.25 25


 146  2 55 2  d 2 2 than  =
6.25 6.25
AO BO AB
 73  55 2  d 2 2 = = Area of Δ AOB = 25 cm2
OC OD DC
Area of Δ COD = 6.25 cm2
 732  552  d 2 2  ΔAOB  ΔCOD 4. A wall is the form of a trapezium
with height 8 m and parallel
 732  55 2  d 2 2 EXAMPLES sides being 6 m and 10 m. What
1. A trapezium has the perpen- is the cost of painting the wall,
 d 2 2  (73  55) (73 – 55) if the rate of painting is Rs. 25
dicular distance between the
 12818 two parallel sides 60m. If the per sq. m?

ir
 d 2  48cm lengths of the parallel sides be 1
a  b   h

v.iSn
40m and 130 m, than find the Sol. Area of trapezium =
8. The perimeter of a rhombus is 2
area of the trapezium.
40cm and the measure of an Sol. 1
angle is 60°, then the area of it is: 40 = (6+10)×8 = 64m2
2
Sol.

dnag
A Total cost of painting
60m = Rs 25 × 64 = Rs. 1600
5. Area of the trapezium formed
by x-axis; y-axis and the lines

eYari
130
3x + 4y =12 and 6x + 8y = 60 is:
B D 1
Area = a  b   h Sol. For 3x + 4y = 12
2 By putting x = 0, y = 3
C

Side 
40
4
=10cm

AB = AD = 10cm
1

2
= 5100 m2
snhe
130  40 60 = 85 × 60
By puttting, y = 0, x = 4
For 6x + 8y = 60,

By putting x = 0, y 
15
2. The cross section of a canal is a
kgei
2
 ABD =  ADB = 60° trapezium in shape. If the ca-
By putting y = 0, x = 10
 Area of the rhombus nal is l0 m wide at the top and
6 m wide at the bottom and the Y
ERna

3 area of cross section is 640m2. C (0,15


AB
2
 2 2
4 Find the length of canal?
Sol. Let the length of a canal = h m A
3 (0,3)
Area of trapezium canal
 2 1010  50 3cm2
aBryn

4 X’ X
1 O D (10,0)
= a+b ×h B (4,0)
Trapezium 2

It is a quadrilateral, whose any 1


640 = 10  6h
Les

two opposite sides are parallel. 2


1280 = 16 h
wa. th

Y’
h = 80m.
1
3. ABCD is a trapezium with  Area of Δ OCD = × OD× OC
AB||CD whose diagonal meet 2
at O. If AB = 2 CD and length of
wwM

1 15 75
CD = 2.5 cm. Find the area of  10 
2 2 2
(i) Perimeter = a + b + c +d Δ AOB and Δ COD ?
Sol.  Area of OAB
1 D C
(ii) Area= Sum of Parallel sides
2 1 1
Distance b/w them  OB OA  43  6
O 2 2
1
= (a + b) × h A B 75
2  Area of trapezium  6
AB = 5 cm 2
(iii) d12 + d22 = c2 +d2 +2 ab 
CD = 2.5 cm 75 12 63
(sum of squares of non-paral-  
2
Area of ΔAOB AB  2CD 4CD2
2 2 2
lel sides) + 2 ( product of par-
= = =  31.5sq.units
allel sides) Area of ΔCOD CD2 CD2 CD2

Rakesh Yadav Readers Publication Pvt. Ltd. 287

For More Visit : www.LearnEngineering.in


For More Visit : www.LearnEngineering.in

6. Find the distance between the 4


Sm(Sm - x )(Sm - y )(Sm - z ) BD AB 1
two parallel sides of a trape- A=
3 ? = =
zium if the area of the trape- DC AC 3
zium is 250sq.m and the two x y z (  AD is the angle bisector of  BAC)
parallel sides are equal to 15m where, Sm 
2  Area of Δ ADC
and 10m respectively. = 3 × 40 = 120cm2
1 (v) In circl e Rad ius A
Sol. Area= 2 (sum of parallel sides)
Area of Δ
=
× (distance between them) Semi–Perimeter
1 
 250  (15  10)  h

r
r=
2 s

Sni
 h = 20m (vi) Circum-circle Radius (R) B O D C

gv.i
Cyclic Quadrilateral: A quad- abc  Area of Δ ABC = 120 + 40
rilateral whose vertices lie on = 4×area of Δ = 160cm 2

the circumference of the circle. 4. What is the area of a triangle


(a, b, c = length of sides of

ridna
C Triangle) having perimeter 32cm, one
c side 11cm and difference of
D
b EXAMPLES other two sides 5cm?
Sol. Let the sides of triangle be a, b

eeYa
B 1. The area of a triangle whose
d
sides are 15m, 16m and 17m and c respectively,
a
is :  2s = a + b + c = 32
A
 11 + b + c = 32
(i) Area (A) a + b + c 15 +16 +17
Sol.
geisnh
= = 24m  b + c = 32 - 11 = 21 ....(i)
2 2
= s(s a)(s b)(s c )(s  d) and b - c = 5 ...(ii)
Aera (A) = s(s - a)(s - b)(s - c) By equations (i) and (ii)
abcd
where, s  2b = 26  b = 13
2 = 24(24 - 15)(24 -16)(24 -17)
Enak

(ii)  A +  B +  C +  D = 2  c = 13 - 5 = 8
= 24 × 9 × 8 × 7
(iii)  A +  C =  B +  D = 180°  2s = 32  s = 16
R

=  9 = 24 21m2 a = 11, b = 13, c = 8

Triangle 2. If the area of a triangle is  Area of triangle


aryn

250m 2 and base : height is 4 :  s s a (s b)(s  c )


A triangle is a polygon with 5, find its height :
Les B

three edges and three vertices. Sol. Let base = 4x, then height = 5x
It is one of the basic shapes in  16(16  11)(16  13)(16  8)
1
geometry. It denoted by Δ ABC.  2  4x  5x  250  16538 8 30 sq. cm
Type of triangles:-
 x2 = 25
wa. th

(1) Scalene Triangle:- 5. The area of the largest triangle


 x=5 that can be inscribed in a semi
A scalene triangle has all its
side s of d if fer ent le ng ths.  height = 5x = 5 × 5 = 25m circle of radius x in square unit
Equivalently, it has all angles of 3. In the figure given below, AD is:
different bisects  BAC. If the area of
wwM

Sol.
 ABD = 40cm2 and AC = 3AB, A
1
(i) Area = ×Base×height then the area of  ABC :
2
A
(ii) Area = s s–a s–b s–c 
(Hero's Formula) B
O x C
(iii) Semi-perimeter (S)
1
a+b+c B D C OA  BC  radius
= 2
2 1
×(BD)×(OA) Area of the largest triangle
(iv) If lengths of three medians Area of Δ ABD 2
=
of  ABC are x, y and z units, Sol. Area of Δ ADC 1 1 1
×(DC)×(OA)   BC  OA   2 x  x = x ²
then : 2 2 2

Rakesh Yadav Readers Publication Pvt. Ltd. 288

For More Visit : www.LearnEngineering.in


For More Visit : www.LearnEngineering.in

2. Isosceles Triangle Perimeter = 2a + b  x 2 + x 2 = 52


An lsosc eles triangle is a = 2×13+10 = 36 cm. 2x2 = 25
2. Find the altitude in the last
triangle with two equal sides 25
question? x2 =
also their opposite angles are Sol. 2
equal. A
Area of triangle
1 1 25
=  x2   = 6.25 cm2
13 13 2 2 2
. a = equal sides
..
b = unequal side 5. Two sides of a triangular field
a a
h = Altitude are 85 metres and 154 metres
h
respectively and its perimeter is
B C

ir
5 10 5 324 metres. Find the cost of lev-

v.iSn
b/2 b/2 4a 2 – b 2 eling the field at the rate of Rs 5
b h (altitude) = per sq. m.
2
Sol. P = Sum of all three sides
b 4  169 – 100 Third side of triangle
(i) Area = 4a 2 –b2 =

dnag
4 2 = 324 – (154 + 85)
(ii) Perimeter = 2a + b 24 = 85 metres
= = 12 cm.
2 2 Area of the field
4a 2 –b Alternate:-
(iii) h (Altitude)= = s s–a s–bs–c 

eYari
2 2 2
h= 13 – 5 = 12
If an isosceles triangle is right a +b +c p 324
angle triangle than 3. A plot of land is in the shape of S= = = = 162
a right angled lsosceles triangle. 2 2 2

A snhe
The length of hypotenuse is
50 2 m. The cost of fencing is
Rs. 3 per square meter. Find the
Area =
= 2772m2
162×8×77×77

Cost of leveling = 5×2772


a b total cost of fencing the plot ?
kgei
= Rs. 13,860
Sol. A 6. The perimeter of an isosceles,
B
a
C right-angled triangle is 2p unit.
ERna

The area of the same triangle is:


50 2
1 2
(iv) Area 
2
a (a) 3  2 2  p sq.unit
2

(v) h (hypotenuse) = a 2 B C (b) 2  2  p sq.unit


2
aBryn

 B = 90º
EXAMPLES  A =  C = 45º (c) 2  2  p sq.unit
2

1. The base and the same sides of In 45º triangle,


an isosceles triangle is 10 cm (d) 3  2  p sq.unit
2

2 a = 50 2
and 13 cm respectively. Find its Sol. A
Les

area and Perimeters ? a = 50


wa. th

A
AB = 50 cm
Sol. BC = 50 cm
1 2 1 2
13 13 Area = a =  50  = 1250 cm2
2 2
wwM

B C
Total cost of fencing the land
AB = BC = x
= 1250 × 3 = Rs 3750
B C
10 4. The hypotenuse of a right angle  AC  AB 2  BC 2  x2  x 2
a = 13 cm lsosceles triangle is 5 cm. Its
b = 10 area will be?  2 x units
b Sol. In triangle by phythagoras
Area = A  2x  2 x  2 p
4 4a 2 – b 2

10 2
4  13 – 10
2
5 cm

 x 2 2  2p 
= x
4
x
2p


2p 2 2 
=
10
4
 24  60 cm 2 B x
C 2 2 2  2 2  2 
Rakesh Yadav Readers Publication Pvt. Ltd. 289

For More Visit : www.LearnEngineering.in


For More Visit : www.LearnEngineering.in



2 2 2 p  
 2 2 p  (i) Area =
1
× Base × Height
Sol.
4 2 2
1 (ii) Perimeter = P + B + H R 25
 Area of triangle  x 2 15
2 P×B
(iii) Altitude (M) =
H
1
  2  2 p2
2
 2
 (iv) H² = P² + B² 20
Circumcircle Radius
4 24 2 2 P+B–H
 p (v) In radius (r) = or abc
2 2
= 4×area of triangle
 
 3  2 2 p 2 sq.units PB

r
7. A right angled isosceles tri- Area of triangle
P B H
angle is inscribed ina semi-

Sni
= s s–a s–b s–c 
circle of radius 7cm. The area H
(vi) Circum radius (R) =

gv.i
enclosed by the semi-circle but 2 15+20+25
exterior to the triangle is : S= = 30
EXAMPLES 2
Sol. Area of triangle
C

ridna
1. What is the radius of the incircle
90° of a triangle with sides 18, 24 = 30×15×10×5 = 150 cm2
and 30 cm? Circumcircle Radius
A O B Sol. As we know 18, 24 and 30 are

eeYa
abc
 ACB = 90° triplets, than the triangle will be =
right angle triangle . 4×area of triangle
AC = CB = X cm
AB = 14 cm 15×20×25
= = 12.5 cm
From  ABC
geisnh 4×150
AC2 + BC2 = AB2 Alternate:-
 x2 + x2 = 142 Hypotenus H
 2x2 = 14  14 R = 
2 2
Enak

 x2 = 14  7 R
25
 x= 14  7  7 2cm = = 12.5 cm
Incirecle Radius 2
3. The base and altitude of a right
R

1
 Area of ΔABC  2  AC BC =
Area of triangle angled triangle are 12cm and
Semi–perimeter 5cm respectively. the perpen-
aryn

1 dicular distance of its hypot-


 7 2 7 2  49sq.cm a+b+c 18  24  30 enuse from the opposite ver-
2 S= = = 36cm
Les B

2 2 tex is:
r 2 Sol.
area of semicircle = Area of Δ = s s–a s–b s–c  A
2
= 36  18  12  6 = 216
wa. th

22 7  7
= × = 77 cm² 5cm
7 2 h
Area of Δ
Required area Incirle Radius =
S B C
= 77 – 49 = 28 cm² 12cm
wwM

3. Right angle triangle:- 216


AC  12  5  13cm
2 2
= = 6cm.
It is a triangle with an angle of 36
90º π /2 radians. The sides a, Alternate:- 1
b and c of such a triangle sat- Area of Δ ABC = ×12 × 5
isfy the phythagoras theoram. P+B–H 2
A
r= = 30cm2
2
also area of Δ ABC
18 + 24 – 30
= = 6 cm
Perpendicular (H) 2 1 13
(P) (Hypotenuse) = × (AC)× h = h cm2
2. What is the radius of circle 2 2
u
M
tit

dr awn outs id e the


al

13 60 8
B C
triangle whose length of sides are  h  30  h  4 cm
(B) (Base) 15, 20 and 25 cm? 2 13 13

Rakesh Yadav Readers Publication Pvt. Ltd. 290

For More Visit : www.LearnEngineering.in


For More Visit : www.LearnEngineering.in

4. Equilateral triangle:- a = 40
It is a triangle whose all sides Perimeter = 3a Alternatively:-
and angle are equal. = 3 × 40 = 120 m.
A
a
2. ABC is an equilateral triangle. Circumradius(R) =
P and Q are two p oint on 3
a = Side
a a h = Altitude AB and AC respectively such a
h  10 =
that PQ  BC . If PQ = 5cm then 3

B
a/2 a/2
C the area of Δ APQ ?  a = 10 3
D A
a Sol.
3 2 a
600 Inradius =

ir
(i) Area = a 
4 2 3

v.iSn
(ii) Perimeter (2s) = 3a P Q

3 10 3
(iii) Altitude (h) = a 0
= = 5 cm
2 B
60
0
60 2 3
C
Alternate:-

dnag
a  PQ  BC
(iv) Incircle Radius (r) =
2 3 R 10
 APQ =  ABC = 60º r= = =5
a (corresponding anlge) 2 2
(v) Circumcircle Radius (R) =
3  AQP =  ACB = 60º 4. If the are a of squ are is

eYari
R (corresponding anlge)
We can say "r" = 3 3 times the area of an equi-
2 Δ APQ is an equilateral triangle
lateral triangle, then the ratio
(vi)  A =  B =  C = 60° 3 2 of the sides of the square to
a

A=
3 2 h2
4
a =
3 =
3
4
 25 =
snhe
Area of Δ APQ =

25 3
4
cm
2 the side of the equilateral
triangle is equal to :
Sol. Let the side of square = x and
the side of equilateral triangle
kgei
(vii) If P1, P2 and P3 are perpendicu-
3. If the circumradius of an equi- =y
lar lengths from any interior
lateral triangle be 10cm, then
point (O) of an equilateral  the measure of its in-radius is:  
 x 2  3 3   3 y 2   x 2  9 y 2
ERna

ABC to all its three sides re- Sol.  4


spectively, then:- A   4

x 3
 y  2  x :y  3:2
A
aBryn

F E

O 5. The in-radius of an equilateral


P N
p2 O p1 a triangle is of length 3 cm. Then
a
B D C the length of each medians is.
p3 Let AB = x cm Sol. A
Les

B M C X
 BD 
wa. th

a
2
3
P1 + P2 + P3 = a =h x2 3
2 AD  x 2   x cm. O
2 4 2
a  (P1  P2  P3 )
wwM

1 3 x
3  OD   x cm. B D C
3 2 2 3 AO : OD = 2:1
EXAMPLES AO = Circum radius
OB  BD 2  OD 2
1. The area of an e quilateral OD = Inradius = 3 cm
x2 x 2 4x 2 x AD = 2 + 1 = 3
triangle is 400 3 sq. m. Its     cm. AD = 3 × 3 = 9 cm
4 12 12 3
perimeter will be:- 6. The circum-circle radius of an
x equilateral triangle is 8 cm. The
3 2   10  x  10 3cm.
Sol. Area = a = 400 3 3 inradius of the triangle is-
4
400 3  4 x 10 3 a
a² = = 1600  OD    5cm. Sol. Circum circle radius =
3 2 3 2 3 3

Rakesh Yadav Readers Publication Pvt. Ltd. 291

For More Visit : www.LearnEngineering.in


For More Visit : www.LearnEngineering.in

a Circle
8
3 a 8 3 A circle is a set of points on a
plane which lie at a fixed
a 8 3 distance from a fixed-point.
In radius = = = 4cm.
2 3 2 3 The fixed point is known as
Alternate:- Sol. ‘centre’ and the fixed disatance
C
is called the ‘radius’.
R 8
r= = = 4 cm
2 2 cumference
Q cir

diameter
R
7. The area of an equilateral

10
triangle inscribed in a circle r

s
iu
r ad
20
is 9 3cm 2 . The area of the

R
.iSn
i
P O centre
circle is :
S
Sol. Area of equilateral  B
d
A
T

nagv
3
= (side)2 = 9 3 PQRS is a rectangle.
4 (i) Circumference or Perimeter of
 PQ = 10 + 20 + 10 = 40cm
 side = 6 cm circle

rid
 circum-radius of equilateral  for AP, (P) = 2 r  d (d  diameter )

eeYa
side 6
=  = 2 3 So, Area d 2 2
3 3 P (ii) Area = A = r 
4
 
2
of circle =   2 3  12 cm 2 S  Diameter of the circle = d
8. The area of an e quilateral
geisnh
triangle is 4 3 cm2. the length 60° 4A
=
of each side of triangle is A T 
A  60, PST  120 EXAMPLES
Enak

3 2 3 2
Sol. Area = a  4 3= a
4 4 120 1. The radii of two circle 7 cm and
 PSA  AST   60 24 cm. the area of third circle is
16 3 2
R

equal to the sum of the area of


a2 = = 16  a = 4 cm
3 and PAS  SAT  30 the two circles. The radius of the
Each side of triangle is 4 cm.  In Δ PSA , tan 30º third circle is.
aryn

9. From a point in the interior of Sol. Area of the C1 = π r12 = 49π cm2
PS 10
an equilateral triangle, the per-
Les B

= =
pendicular distance of the sides AP AP Area of the C2 = π r22 = 576π cm2
Area of C3 = Area of C1 + Area of
are 3m,2 3m and 5 3 m re- 10
 AP = C2
spectively. The perimeter of the tan 30
wa. th

triangle is : πr32 = 49π  576π = 625π


 AP = 10 3
Sol. P1  3 , r32 = 625
Similarly;
r3 = 25 cm.
P2  2 3, P3  5 3 QC = 10 3 2. The area of circle whose radius
wwM

 AC = PQ +AP + QC is 6 cm is trisected by two con-


3 centric circles. The radius of the
 P a = 40 + 10 3 + 10 3
2 smalest circle is
= P1  P2  P3 = 8 3 
= 20 2  3 cm  Sol.
 a = 16  AB = BC =AC
 Perimeter = 3a = 48m
10. An equilateral triangle cir- 
= 20 2  3 cm 
cumscribes all the cricles,
 Perimeter of  ABC
e a ch w i t h ra d i u s 1 0c m .
Area of C1 = πr 2
What is the perimeter of the
equilatral triangle?

= 60 2  3 cm  = π (6)2 = 36π

Rakesh Yadav Readers Publication Pvt. Ltd. 292

For More Visit : www.LearnEngineering.in


For More Visit : www.LearnEngineering.in

After Trisected, the area of 28  22   O = 90° and OP = OX = R


r2  7
1 22   OPTX, is a square of side R
Smallest circle =  36π = 12π
3 = 28× 7 = 7 ×7 ×4  OT = 2 R
Area of smallest circle,  r = 7  2 = 14m Similarly, AQTM is a square of
12π πr 2  d = 2r = 28m side r.
r2 = 12 6. In the given figure, when the
O R P
r = 2 3 cm outer circles all have radii ‘R’
3. The area of circle is increased then the radius of the inner
by 22 cm2 when its radius is in- circle will be :
creased by 1 cm. The original

ir
radius of the circle is. R

v.iSn
Sol. π (r + 1)2 – π r2 = 22 A
Q
π (r2 + 1 + 2r – r2) = 22
π (2r + 1) = 22 X M T

dnag
22 AT =
2r  1  22  2r
7
Sol.  OT = OA + AT
2r = 7– 1 A R R B
r = 3 cm R R  2R  (R  r )  2 r

eYari
P
4. The area of a circle is halved
when its radius is decreased R
D
Q R
C
  
2 1 R  1  2 r  
by n. Find its radius : R R
Sol. By the question, we have  
 r   2  1 R  3  2 2 R
r² – (r–n)² =
r 2
2
snhe
let radius of inner circle = r
A, B, C, D are the centres of
the four outer circles 8.


Th ree

 2  1
circles

of radi us

–r 2
kgei
 –(r–n)² =  ABCD is a square of side 2R
2 2  1, 2  1 and 1unit, touch
2  AC = 2 ( side)  2 (2 R )  2 2 R each other externally, then find
 r² –  2 r – n = 0

ERna

 PQ = AC - AP - QC the perimeter of the sur-


rounded part by three circles.
 r – 2(r – n) r  2 r – n   0

  
   = 2 2 R  R  R  2R 2 1  
Sol.
 r= 2 r – n or r = – 2 r – n  2r  2 R( 2  1)
aBryn

 r= 2 r– 2n  r  R( 2  1) A 2 +1 2 +1
B
2n 7. In the given figure, find the
 r= radius of smaller circle (r) :
2 –1
Les

1 1
5. The area of a circular field is
C
wa. th

equal to the area of a rectan-


gular field. The ratio of the
length and the breadth of the
rectangular field is 14 : 11
AB = 2  
2  1 and AC = BC
wwM

respectively and perimeter is =2+ 2


100 me ters. What is the r
diameter of circular field ?
Sol. Let length = 14x, then breadth
 AB2 = 4  2 1 
2

Sol.
= 11x and AC2 = BC2 = 2  2  
2

 2(14x + 11x) = 100


So, it is clear
 50x = 100  x = 2 O P AB2 = AC2 + BC2
 Area of rectangular field i.e. ABC is an isosceles right
= 28  22m2 angled triangle.
T
 Area of circular field X   ACB = 90° and  CAB
= 28  22m2
in  OPTX,  P =  X =  T = 90° =  ABC = 45°
  r2 = 28  22m2

Rakesh Yadav Readers Publication Pvt. Ltd. 293

For More Visit : www.LearnEngineering.in


For More Visit : www.LearnEngineering.in

E
 required perimeter 36 A
r   7 = 7 cm.
45
= 2  2 1  360 2.
36
The perimeter of a Semi-circle
D

is numerically equal to its area. C


45
B

+ 2  2 1  360
+
The lenth of the diameter is:-
1 2 Sol. Let AB = BC = x,
Sol. π r + 2r = πr
90 2 then AC  2 x
2 1
360 1 2
r (π + 2) = πr But AC = 128  8 2cm
1 1 2

= 2  2 2  1  2 

r
8 4 2π + 4 = π r 2x = 8 2  x = 8cm

Sni
4 Area of semicircls
2 r  2
=  2  1  1 π
1  x  1  x 

gv.i
2 2
4
      
2  2  2  2 
47 72 =
 r 2=
=
2
 22  22 22

ridna
1
Semi Circle Diameter = 2 
72 =  2 16 = 16  cm2
22 2
It is a figure enclosed by a 6. In the given figure ABCD is a

eeYa
diameter and the part of the 6
6 meters. square. Four equal semicircles
circumference cut off by it. 11
are drawn in such a way that
(i) Circumference (Perimeter) 3. A semi-circular shaped window they meet each other at ‘O’.
 r  2r  r  d has diameter 98 cm, its perim-
Sides AB, BC, CD and DA are
geisnh
eter equals:-
the respective diameters of the
r 2 Dia 98 four semicircles. Each of the
(ii) Area(A)= . Sol. Radius = = = 49 cm
2 2 2 side of the square is 8cm. Find
Perimeter = π r + 2r the area of the shaded region.
Enak

22 D C
= ×49+2×49 = 252 cm
s

r O
diu

7
Ra

4. The perimeter of semi-circular


d (diameter) area is 18cm, then the the ra-
(iii) The area of largest triangle A B
aryn

22
inscribed in a semi-circle of ra- dius is : (using  
) Sol.
dius r is r2. 7 D C
Les B

y
C Sol. Perimeter of semi-circular
x x
region = 18cm
 r  2r  18 y y

r (  2)  18
wa. th

 x
x
A B y
r  22 
Area of the  ACB  r 2 r   2  18 A B
  7 
Le t area o f e ach shaded
1  portion = x
wwM

  r  2r   36 
2
   r    18 and area of each unshaded
 7  portion = y
EXAMPLES total area of square = (8)2
18  7 7 1
1. If the perimeter of a Semi-cir-  r    3 cm = 64 cm2
36 2 2  4(x + y) = 64
cular field 36m. Find its radius?
5. The area of the square on AC  x + y = 16 .....(i)
Sol. Circumference = πr  2r as a side is 128cm. What is the Again in a semicircle,
πr + 2r = 36 sum of the areas of semicircles
r (π+2) = 36 1
drawn on AB and AC as diam- AOB = x + y + x =   42
eters, given ABC is an isoscles 2
 22 
r  2 = 36 right angled triangle and AC is
 7 
its hypotenuse.  2x + y = 8 ......(ii)

Rakesh Yadav Readers Publication Pvt. Ltd. 294

For More Visit : www.LearnEngineering.in


For More Visit : www.LearnEngineering.in

From (i) & (ii) we get. EXAMPLES Sol. Area of shaded region = Area
x = 8  - 16 = 8(  -2) of quadrant – Area of  AOD
1. At each corner of a triangles
 Total area of shaded region field of sides 26m, 28m, and  r2 1
= 32(  -2)cm2 30m, a cow is tethered by a rope =   42
4 2
of length 7m. The area ungrazed
Sector
by the the cows is.  44
A sector is a figure enclosed by Sol. =  4  4  4
4
two radii and an arc lying be- A
tween them. = 4(  1)cm 2

7m
7m
For sector AOB, = 4(3.14-1) = 4 × 2.14
?1
= 8.56 cm2

ir
3. Find the area of the shaded

v.iSn
7m
?3 region if the radius of each of

7m
O ?2
 the circle is 1cm.
r r B 7m 7m C

dnag
A B
Area grazed by all cows
l A B
 r 2 
Area of sector =
(i) l = Arc AB  ( 2r ) 360º

eYari
360 C
(ii) Area of sector ACBO  r 2 1  r 2 2  r 2 3
Now, =  
1 360º 360º 360º
= (arc AB)× radius
2

 (r 2 )

r 2 

snhe
=  360º 1 2 3 

Sol. ABC is an equilateral triangle


with sides = 2cm
Area of shaded region
360 Here,  1 +  2 +  3 = 180º = Area of  ABC - Area of 3
kgei
(iii)Perimeter (P) = Arc AB + 2r r=7 quadrants.
= l + 2r r 2
= 180º = 1r 2 3   
ERna

Segment of a circle 360º 2 (2) 2  3 r 2 ,


4  360 
A figure enclosed by a chord 1 22   60  ABC is an equilateral triangle
and an arc which it cuts off. =   72 = 77 cm
2 7
aBryn

D Now, Area of  ABC, 3  1


  4  3  1 
26  28  30 4  6
Semi-perimeter = = 42
2 
 3
O 2
Les

 r
Area of Δ ABC = s s–a s–b s–c 
r
Ring or Circular Path :
wa. th

B
= 42  16  14  12 = 336
R  outer radius
C
Ungrazed area = 336 – 77 r  inner radius
(i) Are a of segment A CB.
wwM

= 259 m2
(minor segment) = area of
sector ACBO – area of 2. In the adjoining figure, AOBCA
represents a quadrant of a
 OAB.
circle of radius 4cm with cen-
(ii) Area of segment ADB (ma- r
jor segment) = area of circle tre O. Calculate the area of the
– area of segment ACB. shaded portion. R
B
(iii)Perimeter (P) = arc AB +  .r
C

 πθ   
 2r   sin    D

 360  2  (i) Area (A) =  (R2 - r2)


2cm
(iv) Arc = Angle  Radius (ii) Perimeter = 2  (R + r)
A O

Rakesh Yadav Readers Publication Pvt. Ltd. 295

For More Visit : www.LearnEngineering.in


For More Visit : www.LearnEngineering.in

EXAMPLES 3. A circular Swimming pool with (vii) Perimeter (P) = n  a


a diameter of 42 ft has a deck of (where n = no. of sides, and, a =
1. The area of ring between two uniform width built around it.
concentric circles, whose cir-
length of each side)
If the area of deck is 43p sq. ft. (viii) Area (A)
cumferences are 44 cm and 66 Find its width.
cm. Sol. Area of swimming pool 1 1
Sol.
= pr2 = p(21)2
  p r  n a r
2 2
= 441p sq. ft.

r R

r
r

Sni
R 21 a
C1 (Smaller Circle); where, r = radius of inscribed

gv.i
C2 (Bigger Circle) circle
Circumference of C1 = 2πr1 Area of deck = 43p Sq.ft 2
 p(R2 – (21)2) = 43p 1 a

ridna
2πr1 = 44  A   n  a  R2   
43π 2 2
Circumt of C2 = 2πr2  R2 – 441 =
π
R = radius of circumscribed

eeYa
2πr2 = 66 R2 = 441 + 43 circle.
R2 = 484  R = 22
44
r1   7 = 7 cm Width of the deck = 22 – 21 na 2 
44 = 1 feet. or A  cot
4 n
geisnh
66 Polygon
r2   7 = 10.5 cm
44
It is a 2– Dimensional shapes.
EXAMPLES
Area of ring between circles They are made of three or more 1. An exterior angle of a regular
 π R 2 – R1  than three straight lines, and
2 2
polygon measures 36º. How
Enak

the plane is closed. many sides does the polygon


 2
 π 10.5 – 7 
2
 have?
Sol. No of sides of polygon = n
R

 π10.5  710.5 – 7


360º
Each exterior angle =
 π17.5  3.5 n
aryn

22 3600
Les B

 17.5  3.5 36º 


7 n
 192.5 cm² Polygon
2. A circular road runs ground a
n = n: of sides 360º
(i) Sum of Exterior angles n = 10
circular ground. If the difference 36º
wa. th

between the circumference of = 2π = 360º Sides of a polygon is 10.


the outer and inner circle is (ii) Sum of Interior angles 2. How many sides does a polygon
99m. the width of the road is:- = (n – 2) × 180º have if the sum of its interior
Sol. (iii) Interior angle + exterior angle angles is 1260º?
wwM

= 180º Sol. Sum of interior angles


(iv) E ach Interio r angl e = (n – 2) × 180
r 1260 = 180n – 360
n – 2  0
=    180 1260 + 360 = 180n
n 
1620
R 360 n =9
(v) Each Exterior angle = 180
Width of the Road = 2πR – 2πr n Sides of a polygon is 9.
 2π (R – r) = 66 (vi) No of diagonals in polygon 3. The ratio of inter nal and
66 7 21 external angle is 7:1. Find the
 = n n – 3
 R–r= = 10.5 cm = number of sides?
22 2 2 2
Width of the road = 10.5 cm. Sol. Ratio of I : E = 7:1

Rakesh Yadav Readers Publication Pvt. Ltd. 296

For More Visit : www.LearnEngineering.in


For More Visit : www.LearnEngineering.in

7x + x = 180º 360º ABCDEF is a regular hexagon.


8x = 180º 22.5  Joining the centre O with verti-
n
x = 22.5º ces A,C and E, we get,
So, exterior angles is = 22.2º 360
n =16  AFE =  AOE
360º 22.5
Each exterior angle = similarly,  OAC =  BAC
n No. of sides is 16.
also,  OEC =  DEC

There are several other parts of polygon such as 1


  ACE = the area of regu-
2
No. of Name Internal External
lar hexagon.
Sides angle angle
4. A 6 sided regular polygon is in-

ir
3 Triangle 60º 120º scribed in a circle of radius 10

v.iSn
4 Quadrilateral 90º 90º cm, find the length of one side
5 Pentagon 108º 72º of the hexagon?
6. Hexagon 120º 60º Sol.
A
7. Heptagon 128.57º 51.43

dnag
8. Octogon 135º 45º
9. Nonagon 140º 40º
O B
10. Decagon 144º 36º

eYari
Hexagon DE  BC 360º
 AOB = = 60º
6
A polygon with 6 sides is known  ADE =  AED = 60º Since OA = OB = 10 cm, triangle
as Hexagon.
snhe
 AD = DE = AE
 Side of regular hexagon
1
 6 = 2 cm
OAB is isosceles which gives
 OAB =  OBA
As all angle of the triangle are

kgei
3 equal, therefore it is equilateral
R triangle
3 3
Area =  4 = 6 3 cm2 Hence AB = OA = OB = 10 cm
2
ERna

5. The area of a square is 2304


60º 2. If area of a regular hexagon is cm 2. Calculate the area of a
r 216 3 sq. cm, then its perim- regular hexagon that has the
60º 60º
eter. same perimeter as this square.
a
Sol. Perimeter of square = 4a
aBryn

(i) Area = 6 × (Area of Equilateral 3 3 2 Perimeter of hexagon = 6a


Sol. Area = a
triangle of side a) 2 Area of square = a2 = 2304
3 2 3 3 2 3 3 2 a = 48
=6× a = a (a = side) 216 3 = a
4 2 2 4a = 192 cm
Les

(ii) Perimeter = 6a = Perimeter of square


432 3
wa. th

3  a2 6a = 192
(iii) Incircle radius (r) = a 3 3 a = 32
2
(iv) Circum Radius = a a2 = 144  a = 12
3 3a 2
Perimeter = 6 × 12 = 72 cm. Ar ea of he xagon =
EXAMPLES 3. Let ABCDEF be a regular hexa- 2
wwM

1. An equilateral triangle of side 6 gon. What is the ratio of the 3 3


area of the triangle ACE to that   1024 = 1536 3 cm2
cm has its corner cut off form a 2
regular hexagon. Area of this of the hexagon ABCDEF ?
Sol. 6. A regular hexagon with an area
regular hexagon.
Sol. A
of 150 3 cm2 is inscribed in a
circle. Find the area not covered
0
60600 O by hexagon?
D E 3 3 2
Sol. Area of hexagon  a
2
I F 3 3 2
B C a =150 3
H G 2

Rakesh Yadav Readers Publication Pvt. Ltd. 297

For More Visit : www.LearnEngineering.in


For More Visit : www.LearnEngineering.in

Let outer triangles are 'n' then hexagon etc.) is changed by


300 3
2 sum of outer angles = n × 180 – x% , its area chan ges by
a = = 100
3 3 two times sum of exterior angles
 x 
a = 10 = (n × 180 – 2 × 360) x 2  %
= 180 (n – 4)  100 
2
Covered = πr
If n = 5 then  A+  B +.....+  E x   ve if increases
As hexagon consist of 6 equilat-
=180 (5 – 4) = 180º x  ve if decreases
eral triangles
If n = 6 then  A+  B +.....+  F
A =180 (6 – 4) = 360º EXAMPLES
A If n = 8 then  A+  B +.....+  H
1. The length of a rectangle is in-
=180 (8 – 4) = 720º
creased by 50% . By what %

r
O B
Miscellaneous Problem should the width be decreased

Sni
to maintain the same area ?
Some Useful Results: Sol. % decrease in breadth

gv.i
O B
1. If each of the defining dimen-
Side of hexagon = Radius of circle sions or sides of any 2-D figures  100 x 
=  %
= 10 cm. are increased (or decreased) by  100  x 

ridna
x%, its Perimeter also increases
Area not couered = π (10)2 – 150 3 (or decreases) by x%. 10050 100 1
=   33 %
 100π – 150 3  54.35 cm2 2. If all the sides of a quadrilat- 150 3 3

eeYa
eral are increased (or de-
Alternatively :-
Octagon creased by x% , its diagonals
also increases (or decreasses) Let length = x and breadth = y
A polygon with 8 sides is known by x%.
as Octagon.  150  3 x
3. The number of revolutions made New length  x  
geisnh  100  2
(i) Perimeter = 8 × side by a circular wheel of radius r in
(ii) Area of regular octagon travelling distance ‘d’ is given by- As the area remains the
2
d same, the new breadth of the
= 2  2  1 side
 (no. of revolutions) n = rectangle - so,
Enak

2r
EXAMPLES 4. If the length and breadth of a 3x
 New breadth  xy
rectangle are increased by x% 2
1. The side of a regular octagon is
and y% respectively, then the
R

5 cm. Its area is ? 2y


area of rectangle will increases  New breadth 
Sol. Area of regular octagon 3
by:
aryn

2
= 2 2  1 side
  xy   de crease in breadth
x y %
Les B

2y y
= 2 2 1 5 = 50 
2
2 1  100   y
3 3
2. If the perimeter of a regular 5. If the length and breadth of a
rectangle are dicreased by x%  % decrease in breadth
octagon is 80 cm. Its area is ?
and y% respectively, then the y /3
Sol. Perimeter of regular octagon is
wa. th

100 1
area of rectangle will decreases = 100 = = 33 %
= 80 y 3 3
by :
8 × side = 80
xy  2. The length of a rectangle is in-
side = 10 cm 
Area of regular octagon x y % creased by 60% . By what
100 
wwM

2
 per-cent would the width be
= 2 2  1 side
 6. If the length of a rectangle is decreased so as to maintain
increased by x% , then its the same area ?
2
= 2 2 1 10 = 200  2 1  breadth will have to be de- Sol. Let length = width = 100m
Star:- Sum of outer angles of a star  100 x  If length = 160m, then let
Star forms by extending sides of a creased by  % in order width = x m
 100  x 
regular polygon. s.t. 160x = 10000
to maintain the same area of
B B
A B A A C the rectangle. 10000 1000 1
x    62
C 7. If each of the degining dimen- 160 16 2
H D sions or sides of any 2D figure
E
C D (triangle, rectangle, square, 1
G E  width is reduced to 37 %
E F circle, quadrilateral, pentagon, 2
D

Rakesh Yadav Readers Publication Pvt. Ltd. 298

For More Visit : www.LearnEngineering.in


For More Visit : www.LearnEngineering.in

3. A wire is in the form of an equi- In Rectangle


88  7
2 (l + b) = 660 r  14cm
lateral triangle with area 4 3 2 22
l + b = 330
m 2 . If it is change into a  Area =  r2
Length and breadth are in the
square, the side of a square
ratio of 6:5 22
will be.  1414  616sq.cm.
Sol. Area of an equilateral trianle  7
330
3 2 8. The radius of a circular wheel
= a is 1.75m. The number of revo-
4
lutions that it will make in
6 5
3 2 covering 11km is:
4 3 = a Sol. Total no. of revolution
4

ir
d d

v.iSn
16 3 L = =
a2 =  a=4m B Perimeter 2r
3 Area of rectangle = l × b
Perimeter of equilateral triangle = 180 × 150 1110007
= 1000
= 3a  3 × 4 = 12 cm = 27,000 cm2 2221.75

dnag
Perimeter of square = 4a 6. The diameter of a wheel is 0.14 9. The wheel of a motor car
4a = 12, a = 3m m. How many revolutions did makes 1000 revolutions in
Side of a square = 3m. it makes in moving 440 m. moving 110m. The diameter of
4. If the ratio of areas of two Sol. Circumference of a wheel the wheel is:-

eYari
squares is 225:256 than the = distance in 1 revolution Sol. As w h e e l m a k e s 1 0 0 0
ratio of their perimeter is: revolutions in moving 110 m
22 44 1000  110m
Sol. Area of first square, S1 = a2 2r = 2 × × 0.07 = m
7 100 110
= 225 1
a = 15
Area of second square,
S2 = a2 = 256
44
100
m
snhe
In 1 revolution wheel covers

2πr 
1000
110
1000
= Circumference
kgei
a = 16 In covering 440 m distacne, 110  7
Perimeter of square, S1 = 4a r 
wheel makes 44  1000
= 4 × 15 = 60, S2 = 4a = 4 × 64
= 64 440 7
ERna

 × 100 r 
Ratio of their perimeter 44 400
= 15 : 16 = 1000 revolution. 7
Diameter = 2r = 2 
Alternate:- Alternate:- 400
aBryn

d 7
A1  P 2 n= = = 0.035 m
 1 2πr 200
A2 =  
 P2  10. The cost of cultivating a square
(\ n = n : of revolution d
field at the rate of Rs. 190 per
2 440 hectare is Rs. 1710. The cost
S1  P1  P1 = distance) =
Les

×    P 2  22  0.07 of putting a fence around it at


S2  P2  the Rate of 50 paise per metre.
wa. th

2 = 1000 revolution.
7. A metal wire when bent in 1710
225 15 the form of a square encloses Sol. Area of square field =
= = 190
256 16 an area 484cm2. If the same
= 9 hectare
wire is bent in the form of a
wwM

5. A circular wire of diameter 210 1 hectare = 10,000 m2


cm is folded in the shape of a 22 9 hectare = 90,000 m2
rectangle whose sides are in circle, then (talking   ) Side of square
7
the ratio of 6:5. Find the area
enclosed by the rectangle. its area is: = a = 90,000 = 300
Sol. Perimeter of Circular wire Sol. Side of square
50
= Perimeter of Rectangle  484  22 cm Cost of fencing =
100
× 4a
2πr = 2 (l + b)  length of wire  22  4
1
22 = 88cm = × 300 × 4 = Rs. 600
2
 2πr = 2   105
7 22 11. A rectangular piece is 30m
 2  r  88  2   r  88
= 660 cm 7 long and 24 m wide. From its

Rakesh Yadav Readers Publication Pvt. Ltd. 299

For More Visit : www.LearnEngineering.in


For More Visit : www.LearnEngineering.in

four corrers, quadrants of Distance covered by Rakesh in Total lawn area = 625π –7π
radii 2.1 metres have been 15 Sec. = 618π m2
cut. The area of the remain- 16. In the following figure ABCD
15
ing part is. = 124 × = 31m
60 is a rectangle with AD and
Sol.
30 m DC equal to 1 and 2 units
C (Ending pt.)
respectively. Two quarter
circles are drawn with cen-
24 m M tres at B and A respectively.
A B Now a circle is drawn touch-
R
ing both the quarter circles
Area of Remaning part is (Starting pt.)
and done of the sides of the
Sum of length and Breadth = 31

r
= Area of Rectangle – 4 × area rectangle. Find the area of
of quarter circle metre.
the shaded region :

Sni
We know that hypotenuse in a
1 right angle triangle 25 m then D
× pr2 C

gv.i
=L×B–4×
4 length and breadth may be 24
and 7.
22 1
= 720 – × 2.1 × 2.1 (l + b) = 31 area = 24 × 7 = 168 m2.

ridna
7 14. A person observed that he re-
= 720 – 13.86 = 706.14 quired 40 seconds less time to
A 1 P 1 B
12. Area of circle is equal to the cross a circular ground along its

eeYa
area of a rectangle having diameter than to cover it once Sol. Let radius fo the circle is ‘r’
perimeter of 100cms and along a boundary. If his speed units
le ngth is more th an the 40 m/min, then the radius of OP = (1- r), OA = (1+ r) and
breadth by 6cms. What is the the circular ground is: AP = 1
Sol. Along bound ar y he c ove rs
diameter of the circle?
geisnh In  AOP; OA2 = AP2 + OP2
perimeter = 2pr
Sol. Let breadth = x,
Along diameter = 2r
then length = (x + 6) Time distance
 2(x + x + 6) = 100
2πr 2r
Enak

 2x + 6 = 50 = Speed – Speed
 x = 22cm
 breadth = x 40 2r
R

min = (p – 1)
= 22cm & length 60 40
 (1  r )2  12  (1  r )2
= 22 + 6 = 28cm
aryn

Area of circle 2 2r  22 
 =  –1 1
3 40 7   r= units
= Area of rectangle 4
Les B

  r 2  22  28 2  40  7
 1 
2
= 2r 
22  28
3  15  Area of smaller circle =   4 
 r2   7  7 4 7 r = 6.2 m (approx.)
22
wa. th

15. In a circular park with a radius 


 r  7  2  14cm of 25 m there are 7 lamps whose = square units
16
 Diameter = 2r = 28cm base are circles with a radius of
1 m. The entire area of the park Sum of the area of the quar-
13. In between the race of two
has grass with the exception of ter circles
wwM

friends Mohit and Rakesh in a


the base for the lamps. Calcu-   
rectangular field Mohit cheated
late the total lawn area? =   square units
Rakesh by taking diagonal path 4 4 2
Sol.
rather than a side path. Mohit
Area of shaded region
took 15 secs to reach ending
point at the rate of 100 m/min  
whereas Rakesh took same time = 2-   
16 2 
to. Reach the ending point at the
rate of 124 m/min. Find the 9
area of field. 2 = 2   0.23 square units
Area of the circular par =πr 16
Sol. Distance Covered by Mohit in 15
15  π (25)2 = 625π m2 Among the option choices,
sec = 100× = 25 m Area of 7 lamps = 7×π (1)2 = 7π m² option (b) is closest.
60

Rakesh Yadav Readers Publication Pvt. Ltd. 300

For More Visit : www.LearnEngineering.in


For More Visit : www.LearnEngineering.in

EXERCISE

1. If the length of the diagonal AC (a) 8 meters (b) 12 meters (a) 2 38 m (b) 4 38 m
of a square ABCD is 5.2 cm, then (c) 4 meters (d) 32 meters
the area of the square is : 9. The diagonal of a square is (c) 2 19 m (d) 154 m
(a) 15.12 sq. cm 4 2 cm. The diagonal of another 17. The difference of the areas of two
(b) 13.52 sq. cm square whose area is double that squares drawn on two line
(c) 12.62 sq. cm of the first square is: segments of different lengths is
(d) 10.00 sq . cm (a) 8 2 cm (b) 16 cm 32sq. cm, Find the length of the
2. The length of the diagonal of a greater line segment if one is

ir
(c) 32 cm (d) 8 cm longer than the other by 2 cm.
square is ‘a’ cm. Which of the

v.iSn
following represents the area of 10. The diagonal of a square A is (a (a) 7 cm (b) 9 cm
the square (in sq. cm ) ? +b). The diagoanal of a square (c) 11 cm (d) 16 cm
whose area is twice the area of
a square A is 18. A took 15 sec. to cross a rectan-
(a) 2a (b) gular field diagonally walking at
2 (a) 2(a +b) (b) 2(a+b)2

dnag
2 the rate of 52m/min and B took
(c) a /2 (d) a 2 / 4 (c) 2 a–b (d) 2 a  b the same time to cross the same
3. The breadth of a rectangular hall 11. The length of a rectangular gar- field along its sides walking at the
is three-fourth of its length. If the den is 12 metres and its breadth rate of 68 m/ min. The area of the

eYari
area of the floor is 768 sq. m. , is 5 metres. Find the length of the field is:
then the difference between the diagonal of a square garden hav- (a) 30 m2 (b) 40 m2
length and breadth of the hall is: ing the same area as that of the 2
rectangular garden: (c) 50 m (d) 60 m2
(a) 8 metres (b) 12 metres
19. The difference between the length
(c) 24 metres (d) 32 metres (a) 2 30 m (b) 13 m
4. Find the length of the largest rod
that can be placed in a room 16m
2
12.
(c) 13 m snhe (d) 8 15 m
The areas of a square and a rect-
angle are equal. The length of the
and breadth of a rectangle is 23
m. If its perimeter is 206 m, then its
area is
(a) 1520 m2 (b) 2420 m2
long, 12m broad and 10 m high,
kgei
3 rectangle is greater than the (c) 2480 m 2
(d) 2520 m2
(a) 123 m (b) 68 m length of any side of the square 20. The area (in m2) of the square
by 5 cm and the breadth is less which has the same perimeter as
2 1
ERna

by 3 cm. Find the perimeter of a rectangle whose length is 48 m


(c) 22 m (d) 22 m
3 3 the rectangle. and is 3 times its breadth is:
5. Between a square of perimeter 44 (a) 17 cm (b) 26 cm
(a) 1000 (b) 1024
cm and a circle of circumference (c) 30 cm (d) 34 cm
(c) 1600 (d) 1042
13. The perimeter of a rectangle is 160
aBryn

44 cm, which figure has larger 21. The perimeter of two squares are
area and by how much ? meter and the difference of two
sides is 48 meter. Find the side of 40 cm and 32 cm. The perimeter
(a) Square, 33cm2 of a third square whose area is
a square whose area is equal to
(b) Circle, 33 cm2 the difference of the area of the
the area of this rectangle.
(c) Both have equal area. two squares is
(a) 32m (b) 8m (c) 4m(d) 16m
Les

(d) square, 495 cm2 14. The perimeter of two squares are (a) 24 cm (b) 42 cm
wa. th

6. The perimeter of a square and a cir- 24 cm and 32 cm. The perimeter (c) 40 cm (d) 20 cm
cular field are the same. If the area (in cm) of a third square equal in 22. The perimeter of five squares are
of the circular field is 3850 sq meter. area to the sum of the areas of 24 cm, 32 cm,40 cm,76cm and
What is the area (in m2) of the square? these squares is : 80 cm respectively. The perimeter
(a) 4225 (b) 3025 (a) 45 (b) 40 (c) 32 (d) 48 of another square equal in area
wwM

(c) 2500 (d) 2025 15. A wire when bent in the form of a to sum of the areas of these
7. The perimeter of the top of a rect- square encloses an area of 484 squares is:
angular table is 28m., whereas sq. cm. What will be the enclosed (a) 31 cm (b) 62 cm
area when the same wire is bent
its area is 48m 2. What is the into the form of a circ le? (c) 124 cm (d) 961 cm
length of its diagonal? 23. There is a rectangular tank of
22 length 180 m and breadth 120
(a) 5 m (b) 10 m (Take  ) m in a circular field, If the area
(c) 12 m (d) 12.5 m
7
(a) 125 cm2 (b) 230 cm2 of the land portion of the field is
8. The breadth of a rectangular hall 40000 m2, what is the radius of
(c) 550 cm2 (d) 616 cm2
is three- fourth of its length. If the 22
16. Find the length of the longest rod the field ? ( Take  )
area of the floor is 192 sq. m., then that can be placed in a hall of 10 7
the difference between the length m length, 6 m breadth and 4 m (a) 130 m (b) 135 m
and breadth of the hall is: height, (c) 140 m (d) 145 m

Rakesh Yadav Readers Publication Pvt. Ltd. 301

For More Visit : www.LearnEngineering.in


For More Visit : www.LearnEngineering.in

24. The length of a rectangular hall 0.3m × 0.2m at 20 km/hour. Then (a) 25 metres (b) 50 metres
is 5m more than its breadth. The the time ( in hours ) for the water (c) 75 metres (d) 100 metres
area of the hall is 750m2. The level in the tank to reach 8 m is 42. The area of a rectangle is thrice
length of the hall is : (a) 50 (b) 120 (c) 150 (d) 200 that of a square. The length of the
(a) 15 m (b) 22.5 m rectangle is 20 cm and the
33. A street of width 10 metres sur-
(c) 25 m (d) 30 m rounds from outside a rectangu- 3
25. A cistern 6 m long and 4 m wide breadth of the rectangle is
lar garden whose measurement is 2
contains water up to a depth of 1 200 m × 180 m. The area of the times that of the side of the
m 25 cm. The total area of the path (in square metres ) is square. The side of the square,
wet surface is (in cm) is
(a) 8000 (b) 7000
(a) 55 m2 (b) 53.5 m2 (a) 10 (b) 20 (c) 30 (d) 60
2
(c) 7500 (d) 8200
(c) 50 m (d) 49 m2

r
34. The area of the square inscribed 43. The length and breadth of a
26. If the length and breadth of a rectangular field are in the ratio

Sni
in a circle of radius 8 cm is
rectangle are in the ratio 3 : 2 and 7 : 4. A path 4 m wide running all
its perimeter is 20 cm, then the area (a) 256 sq. cm (b) 250 sq. cm

gv.i
(c) 128 sq. cm (d) 125 sq. cm around outside has an area of 416
of the rectangle (in cm2) is m2. The breadth (in m) of the field is
(a) 24 cm2 (b) 36 cm2 35. Area of square with diagonal
(a) 28 (b) 14 (c) 15 (d) 16
(c) 48 cm2 (d) 12 cm2 8 2 cm is

ridna
44. How many tiles, each 4 decime-
27. The perimeter of a rectangle and (a) 64 cm² (b) 29 cm² ter square. will be required to
a square are 160 m each. The cover the floor of a room 8 m long
(c) 56 cm² (d) 128 cm²
area of the rectangle is less than
36. If the area of a rectangle be and 6 m broad?

eeYa
that of the square by 100 sq m.
(x2+7x+10) sq. cm, then one of the (a) 200 (b) 260 (c) 280 (d) 300
The length of the rectangle is
possible perimeter of it is 45. A godown is 15 m long and 12 m
(a) 30m (b) 60m (c) 40m (d) 50m broad. The sum of the area of the
28. A path of uniform width runs (a) (4x+14) cm (b) (2x+14) cm
(c) (x+14) cm (d) (2x+7) cm floor and the ceiling is equal to the
round the inside of a rectan-
geisnh sum of areas of the four walls. The
gular field 38 m long and 32 37. If the perimeter of a square and
a rectangle are the same. then the volume (in m3) of the godown is:
m wide, If the path occupies
600m 2 , then the width of the area P and Q enclosed by them (a) 900 (b) 1200
path is would satisfy the condition (c) 1800 (d) 720
Enak

(a) 30 m (b) 5 m (a) P < Q (b) P  Q 46. Length of a side of a square in-
(c) 18.75 m (d) 10 m (c) P > Q (d) P = Q scribed in a circle is a 2 units.
29. The perimeter of the floor of a 38. A cube of edge 6 cm is painted The circumference of the circle is
R

room is 18 m. What is the area of on all sides and then cut into unit
cubes. The number of unit cubes (a) 2a units (b) a units
the walls of the room, If the height
of the room is 3 m ? with no sides painted is
aryn

(a) 0 (b) 64 (c) 186 (d) 108


2a
(a) 21 m2 (b) 42 m2 (c) 4a units (d) units
39. The length of diagonal of a square 
Les B

2
(c) 54 m (d) 108 m2
30. A copper wire is bent in the shape is 15 2 cm. Its area is 47. The perimeter and length of a
(a) 112.5 cm2 (b) 450 cm2 rectangle are 40 m and 12 m
of a square of area 81 cm2. If the
respectively. Its breadth will be
same wire is bent in the form of
255 2 (a) 10m (b) 8m (c) 6m(d) 3m
a semicircle, the radius (in cm)
wa. th

(c) cm2 (d) 225 cm2


2 48. If each edge of a square be
22 40. A kite in the shape of a square doubled. then the increase per-
of the semicircle is ( take  )
7 with a diagonal 32 cm attached centage in its area is
(a) 126 (b) 14 (c) 10 (d) 7 to an equilateral triangle of the (a) 200% (b) 250%
wwM

31. A copper wire is bent in the form base 8 cm. Approximately how (c) 280% (d) 300%
of square with an area of 121 much paper has been used to 49. An elephant of length 4 m is at
cm2. If the same wire is bent in make it? (Use 3 = 1.732) one corner of a rectangular cage
the form of a circle, the radius (a) 539.712 cm2 of size (16 m × 30 m) and faces
(in cm) of the circle is (b) 538.721 cm2 towards the diagonally opposite
(c) 540.712 cm2 corner. If the elephant starts
22 (d) 539.217 cm2
( Take ) moving towards the diagonally
7 41. A lawn is in the form of a rect- opposite conrner it takes 15 sec-
angle having its breadth and
(a) 7 (b) 14 (c) 8 (d) 12 length in the ratio 3 : 4. The area onds to reach this corner. Find
32. Water flows into a tank which is 1 the speed of the elephant
200 m long and 150 m wide of the lawn is hectare. The (a) 1 m/sec (b) 2 m/sec
12
through a pipe of cross- section breadth of the lawn is (c) 1.87 m/sec (d) 1.5 m/sec

Rakesh Yadav Readers Publication Pvt. Ltd. 302

For More Visit : www.LearnEngineering.in


For More Visit : www.LearnEngineering.in

50. A circle is inscribed in a square 58. The radius of a wheel is 21 cm, 67. Three circles of radius 3.5 cm
of side 35 cm. The area of the re- How many revolutions will it each are placed in such a way
maining portion of the square make in travelling 924 metres? that each touches the other two.
which is not enclosed by the The area of the portion enclosed by
 22  the circles is
circle is use  =
 

(a) 962.5 cm² (b) 262.5 cm²  7  (a) 1.975 cm2 (b) 1.967 cm2
(c) 762.5 cm² (d) 562.4 cm² (a) 7 (b) 11 (c) 200 (d) 700 (c) 19.68 cm2 (d) 21.22 cm2
59. The area (in sq. cm) of the 68. The area of a circular garden is
1 2464 sq. m. how much distance
51. If the side of a square is (x  1) largest circle that can be drawn
2 inside a square of side 28 cm is : will have to be covered if you like
to cross the garden along its
3–x (a) 1724 (b) 784
units and its diagonal is (c) 8624 (d) 616 22

ir
2 60. The area of the ring between two diameter? ( use  )
7

v.iSn
units, then the length of the side concentric circ les, whose
of the square would be (a) 56m (b) 48m (c) 28m (d) 24m
circumference are 88 cm and 132 69. Four equal circles each of radius
4 cm, is
(a) units (b) 1 unit ‘a’ units touch one another. The
3 (a) 78 cm2 (b) 770 cm2 area enclosed between them

dnag
2
(c) 715 cm (d) 660 cm2
1 61. The diameter of a toy wheel is 14 22
(c) units (d) 2 units (  ). In square units, is
2 cm, What is the distance travelled 7
52. A rectangular carpet has an area by it in 15 revolutions?
6a 2

eYari
of 120 m2 and a perimeter of 46 (a) 880 cm (b) 660 cm (a) 3a2 (b)
metre. The length of its diagonal (c) 600 cm (d) 560 cm 7
is: 62. A can go round a circular path 8 41a 2 a 2
(a) 17 metres (b) 21 metres times in 40 minutes. If the (c) (d)
7 7
(c) 13 metres (d) 23 metres
53. If the length of a diagonal of a
square is 6 2 cm, then its area
snhe
diamete r of the cir cle is
increased to 10 times the origi-
nal diameter, the time required
by A to go round the new path
70. The area of the greatest circle
inscribed inside a square of side
22
kgei
will be once travelling at the same speed 21 cm is ( Take  )
as before is : 7
(a) 24 2 cm 2 (b) 24 cm2 (a) 25 min (b) 20 min 1 1
(c) 36 cm2 (d) 72 cm2 (c) 50 min (d) 100 min (a) 351 cm2 (b) 350 cm2
ERna

2 2
54. The length of a room is 3m more 63. The base of a triangle is 15 cm
than its breadth. If the area of a and height is 12 cm. The height 1 1
floor of the room is 70 m2, then of another triangle of double the (c) 346cm2 (d) 347 cm2
2 2
the perimeter of the floor will be- area having the base 20 cm is
71. The are a of an equilate ral
aBryn

(a) 14 metres (b) 28 metres (a) 9cm (b) 18 cm


(c) 34 metres (d) 17 metres (c) 8 cm (d) 12.5 cm triangle is 400 3 sq. m. Its
55. The length of a rectangle is twice 64. If a wire is bent into the shape of perimeter is :
the breadth. If area of the rect- a square, the area of the square (a) 120 m (b) 150 m
angle be 417.605 sq. m., then is 81 sq. cm, When the wire is (c) 90 m (d) 135 m
Les

length is- bent into a semicircular shape, 72. From a point in the interior of an
wa. th

(a) 29.08 metres the are a of the sem icir cle equilateral triangle, the
(b) 29.80 metres 22 perpendicular distance of the
(c) 29.09 metres (taking  ) is : sid es ar e
7 3 cm 2 3 cm
(d) 28.90 metres (a) 154 cm2 (b) 77 cm2 and 5 3 cm. The per imeter
wwM

56. The area of a sector of a circle of (c) 44 cm 2


(d) 22 cm2
radius 5 cm, formed by an arc of (in cm) of the triangle is
65. If the area of a triangle with base (a) 64 (b) 32 (c) 48 (d) 24
length 3.5 cm is : 12 cm is equal to the area of
(a) 8.5 cm2 (b) 8.75 cm2 73. The perimeter of a triangle is 30
square with side 12 cm. the
(c) 7.75 cm2 (d) 7.50 cm2 cm and its area is 30 cm². If the
altitude of the triangle will be
largest side measures 13 cm,
57. The radius of a circular wheel is (a) 12 cm (b) 24 cm
What is the length of the smallest
1.75 m. The number of revolu- (c) 18 cm (d) 36 cm side of the triangle?
tions it will make in travelling 11 66. The sides of a triangle are 3cm, 4
(a) 3cm (b) 4cm
cm and 5 cm. The area (in cm2)
 22  (c) 5cm (d) 6cm
use  =
km is  :
 of the traingle formed by joining
7 74. Diameter of a wheel is 3 m. The
  the mid points of this triangle is:
wheel revolves 28 times in a
(a) 800 (b) 900 3 3 minute. To cover 5.280 km
(c) 1000 (d) 1200 (a) 6 (b) 3 (c) (d)
2 4 distance, the wheel will take

Rakesh Yadav Readers Publication Pvt. Ltd. 303

For More Visit : www.LearnEngineering.in


For More Visit : www.LearnEngineering.in

22   87. A circle is inscribed in a square,


( Take  ): (a)  –

2
3


cm2 An equilateral triangle of side
7
  4 3 cm is inscribed in that circle.
(a) 10 minutes (b) 20 minutes (b)  3 – 2  cm2 The length of the diagonal of the
(c) 30 minutes (d) 40 minutes  
square (in cm) is
75. Find the diameter of a wheel that  
makes 113 revolutions to go 2 km (c) 2 3 – 



2
cm2 (a) 4 2 (b) 8 (c) 8 2 (d) 16
26 decameters. 88. The hypotenuse of a right angle
 
22 (d) 3 3 – 
  cm2 isosceles triangle is 5 cm. Its area
 2
( Take  ) will be
7 82. The area of the largest triangle (a) 5 sq. cm (b) 6.25 sq. cm
4 4 that can be ins crib ed in a (c) 6.50 sq. cm (d) 12.5 sq. cm
(a) 4 m (b) 6 m semicircle of radius r cm, is

r
13 11 89. From a point within an
(a) 2r cm2 (b) r2 cm2 equilateral triangle,

Sni
4 8 perpendiculars drawn to the
(c) 12 m (d) 12 m 1 2
r cm2

gv.i
11 11 (c) 2 cm2 (d) three sides are 6 cm, 7 cm and 8
2
76. The radius of a circular wheel is cm respectively, the length of the
1. 75m. The num ber of 83. The area of the greatest circle, side of the triangle is :
which can be inscribed in a (a) 7 cm (b) 10. 5 cm
revolutions that it will make in square whose perimeter is 120

ridna
travelling 11 km, is cm, is : 14 3
(a) 1000 (b) 10,000 22
(c) 14 3 cm (b) cm
2 3
(c) 100 (d) 10 (a) 15 cm2
7 90. In an isosceles triangle, the

eeYa
77. The circumference of a circle is 2 measure of each of equal sides is
100 cm. The side of a square 22  7 
(b)   cm2 10 cm and the angle between
inscribed in the circle is 7 2 
them is 45°, then area of the
2
100 2 50 2 22 15  triangle is
(a) cm (b) cm (c) 
 
geisnh
 cm2 25
7 2
  (a) 25 cm2 (b) 2 cm2
22  9 
2
2
100 (d) 
   cm2 (c) 25 2 cm2 (d) 2 3 cm2
(c) cm (b) 50 2 cm 7 2 
 84. The area of the incircle of an 91. The area of circle whose radius
Enak

78. A path of uniform wid th equilateral triangle of side 42 cm is 6 cm is trisected by two


surrounds a circular park, The concentric circles. The radius of
22
difference of internal and external is ( Take   ): the smallest circle is
7
R

circumference of this circular


(a) 2 3 cm (b) 2 6 cm
path is 132 metres. Its width is: (a) 231 cm2 (b) 462 cm2
(c) 2 cm (d) 3 cm
(c) 22 3 cm2 (d) 924 cm2
aryn

22 92. The area of an equilateral triangle


( Take  ) 85. The number of revolutions a
7
inscribed in a circle is 4 3 cm2. The
Les B

wheel of diameter 40 cm makes


(a) 22m (b) 20m (c) 21m (d) 4m in travelling a distance of 176 m, area of the circle is
79. Four equal sized maximum 22 16 22
circular plates are cut off from a is ( Take   ): (a)  cm2 (b)  cm2
square paper sheet of area 784 7 3 3
(a) 140 (b) 150
wa. th

sq. cm. The circumference of each 28 32


(c) 160 (d) 166 (c)  cm2 (d)  cm2
plate is 3 3
86. The length of the perpendiculars 93. If the difference between the
22 drawn from any point in the
( Take  ) circumference and diameter of a
7 interior of an equailateral triangle circle is 30 cm, then the radius
wwM

(a) 22 cm (b) 44 cm to the respective sides are p1,p2 of the circle must be
(c) 66 cm (d) 88 cm and p3. The length of each side of (a) 6cm (b) 7cm
80. The circum -radius of an the triangle is (c) 5cm (d) 8cm
equilateral triangle is 8 cm. The 2 94. The base and altitude of a right
in- radius of the triangle is (a)
3
p  p
1 2  p3
angled triangle are 12 cm and 5
(a) 3.25 cm (b) 3.50 cm cm res pectively. The
1
(c) 4 cm (d) 4.25 cm (b)  p1  p2  p3 perpendicular distance of its
3 hypotenuse from the opposite
81. Three coins of the same size
(radius 1 cm) are placed on a 1 vertex is
table such that each of them
(c)
3
 p1  p2  p3 4 8
touches the other two. The area (a) 4 cm (b) 4 cm
4 13 13
enclosed by the coins is (d)
3
p  p
1 2  p3 (c) 5 (d) 7 cm

Rakesh Yadav Readers Publication Pvt. Ltd. 304

For More Visit : www.LearnEngineering.in


For More Visit : www.LearnEngineering.in

95. From a point in the interior of an 102. A right triangle with sides 3 cm,4 108. The length of a sid e of an
equilateral triangle , the length cm and 5 cm is rotated about the equilateral triangle is 8 cm. the
of the perpendiculars to the three side 3 cm to from a cone. The area of the region lying between
sides are 6 cm, 8 cm and 10 cm volume of the cone so formed is the circumcircle and the incircle
respectively. The area of the (a) 16 cm3 (b) 12  cm3 22
triangle is (c) 15 cm3 (d) 20  cm3 of the triangle is ( use )
7
(a) 48 cm2 (c) 16 3 cm2 103. ABC is an equilateral triangle of
side 2 cm. With A, B, C as centre 1 2
(c) 192 3 cm2 (d) 192 cm2 and radius 1 cm three arcs are (a) 50 cm2 (b) 50 cm2
7 7
96. The area of the shaded region in drawn. The area of the region
the figure given below is within the triangle bounded by 1 2
the three arcs is (c) 75 cm2 (d) 75 cm2

ir
7 7

v.iSn
a   109. A wire, when bent in the form of
(a) 3 3 –  cm2 a square, encloses a region
c  2
having area 121 cm2 . If the same
 3  wire is bent into the form of a
(b)  3 – 
 cm2

dnag
 2  circle, then the area of the circle
a 2   22
(a)  – 1
  sq. units  
2 2 is ( Take  )
 (c)  3 –  cm2 7
 2

eYari
2
(b) a  – 1 sq. units (a) 144 cm2 (b) 180 cm2
  (c) 154 cm 2
(d) 176 cm2
(d)  – 3  cm2
2   2  110. If the perimeter of a semicircular
(c) a  – 1
 sq. units 104. The circumference of a circle is field is 36 m. Find its radius
2 

a2
(d) 2  – 1 sq. units
b
snhe
11 cm and the angle of a sector
of the circle is 60°. The area of
22
( use 
22
7
)

(a) 7 m (b) 8 m
kgei
the sector is ( use  )
97. The area of a circle is increased 7 (c) 14 m (d) 16 m
by 22 c m, if its radius is 29 29 111. The perimeter (in metres ) of a
increased by 1 cm. The original (a) 1 cm2 (b) 2 cm2 semicircle is numerically equal to
ERna

48 48
radius of the circle is its area ( in m²). The length of its
(a) 6 cm (b) 3.2 cm 27 27
(c) 1 cm2 (d) 2 cm2 22
(c) 3 cm (d) 3.5 cm 48 48 diameter is (Take  )
7
98. The area of the largest circle, that
aBryn

105. If the difference between areas of


can be drawn inside a rectangle the circumcircle and the incircle 3 6
with sides 148 cm. by 14 cm is (a) 3 metres (b) 5 metres
of an equilateral triangle is 44 11 11
(a) 49 cm2 (b) 154 cm2 cm2, then the area of the triangle
(c) 378 cm 2
(d) 1078 cm2 6 2
22 (c) 6 metres (d) 6 metres
Les

99. A circ le is inscr ibed in an is ( Take  ) 11 11


7
equilateral triangle of side 8 cm.
wa. th

112. One acute angle of a right angled


The area of the portion between (a) 28 cm2 (b) 7 3 cm2 triangle is double the other. If the
the triangle and the circle is length of its hypotenuse is 10 cm,
(a) 11 cm2 (b) 10.95 cm2 (c) 14 3 cm2 (d) 21 cm2 then its area is
(c) 10 cm 2
(d) 10.50 cm2 106. If the area of a circle inscribed in
wwM

a square is 9 cm2, then the area 25


100. In a triangular field having sides (a) 3 cm2 (b) 25 cm2
30m, 72m and 78m, the length of the square is 2
of the altitud e to the side (a) 24 cm2 (b) 30 cm2
2
75
measuring 72m is : (c) 36 cm (d) 81 cm2 (c) 25 3 cm2 cm2(d)
2
(a) 25 m (b) 28 m 107. The sides of a triangle are 6 cm,
8 cm and 10 cm. The area of the 113.If a triangle with base 8 cm has
(c) 30 m (d) 35 m
greatest square that can be the same area as a circle with
101. If the perimeter of a right-angled
inscribed in it, is rad ius 8 cm , then the
isosceles triangle is 4 2 4 cm, corresponding altitude (in cm) of
(a) 18 cm2 (b) 15 cm2
the length of the hypotenuse is; the triangle is
(a) 4 cm (b) 6 cm 2304 576 (a) 12 (b) 20
(c) cm2 (d) cm2
(c) 8 cm (d) 10 cm 49 49 (c) 16 (d) 32

Rakesh Yadav Readers Publication Pvt. Ltd. 305

For More Visit : www.LearnEngineering.in


For More Visit : www.LearnEngineering.in

114. The measures (in cm) of sides of 122. The length of each side of an 130. A straight line parallel to the base
a right angled triangle are given equilateral triangle is 14 3 cm. BC of the triangle ABC intersects
by consecutive integers. its area The area of the incircle (in cm2) is AB and AC at the points D and E
(in cm2) is (a) 450 (b) 308 respectively. If the area of the
(a) 9 (b) 8 (c) 5 (d) 6  ABE be 36 sq. cm. then the area
(c) 154 (d) 77
115. The are a of a r ight-ang led 123. Are a of the inc ircle of an of the ACD is
isosceles triangle having equilateral triangle with side 6 (a) 18 sq.cm (b) 36 sq. cm
hypotenuse 16 2 cm is cm is (c) 120 sq. cm (d) 54 sq. cm
(a) 144 cm2 (b) 128 cm2 131. The length of two sides of an
2
 isosceles triangle are 15 and 22
(c) 112 cm (d) 110 cm2 (a) sq. cm (b) 3 sq. cm
2 res pectively. What are the
116. The area of an equilateral triangle
(c) 6 sq. cm (d) 3 sq. cm possible values of perimeter ?

r
is 4 3 cm2. The length of each
124. A copper wire is bent in the form (a) 52 or 59 (b) 52 or 60
side of the triangle is:

Sni
of an equilateral triangle and has (c) 15 or 37 (d) 37 or 29
(a) 3 cm (b) 2 2 cm 132. The diameter of a wheel is 98 cm.

gv.i
area 121 3 cm2. If the same wire
The number of revolutions in
(c) 2 3 cm (d) 4 cm is bent into the form of a circle.
which it will have to cover a
117. An equilateral triangle of side 6 the area (in cm2) enclosed by the
distance of 1540 m is

ridna
cm has its corners cut off to form 22 (a) 500 (b) 600
a regular hexagon. Area (in cm2) wire is ( take  )
7 (c) 700 (d) 800
of this regular hexagon will be 133. The wheel of a motor car makes
(a) 364.5 (b) 693.5
1000 revolutions in moving 440

eeYa
(a) 3 3 (b) 3 6 (c) 346.5 (d) 639.5
125. At each corner of a triangular m. The diameter (in metre) of the
5 3 field of sides 26 m, 28 m and 30 wheel is
(c) 6 3 (d) (a) 0.44 (b) 0.14
2 m, a cow is tethered by a rope of
length 7m, the area (in m) (c) 0.24 (d) 0.34
118. A 7 m wide road runs outside
geisnh
around a circular park, whose ungrazed by the cows is 134. A bicycle wheel makes 5000
circumference is 176 m. the area (a) 336 (b) 259 revolutions in moving 11 km .
(c) 154 (d) 77 Then the radius of the wheel
22
of the road is : (use ) 126. In an equilateral triangle ABC, 22
7
Enak

P&Q are mid point of sides AB & (in cm) is ( take  )


7
(a) 1386 m2 (b) 1472 m2 AC respectively such that PQ 
(c) 1512 m 2
(d) 1760 m2 (a) 70 (b) 35 (c) 17.5 (d) 140
BC. If PQ = 5 cm then find the 135. Three circles of diameter 10 cm
119. The length (in cm) of a chord of a
R

length of BC. each are bound together by a


circle of radius 13 cm at a distance
(a) 5 cm (b) 10 cm rubber band as shown in the
of 12 cm from its centre is
(c) 15 cm (d) 12 cm figure.
aryn

(a) 5 (b) 8 (c) 10 (d) 12


127. ABC is an equilateral triangle, P
120. The four equal circles of radius 4
Les B

and Q are two points on AB and


cm drawn on the four corners of
a s quar e touch each other AC r espe ctiv ely suc h that
externally. Then the area of the PQ  BC . If PQ =5 cm, then area
portion between the square and the length of the rubber band (in
of APQ is : cm) if it is stretched is
wa. th

the four sectors is


(a) 9( – 4 )sq. cm 25 25 (a) 30 (b) 30 +10
(a) sq. cm (b) sq. cm (c) 10 (d) 60+20
4 3
(b)16 4 – sq. cm 136. If chord of length 16 cm is at a
distance of 15 cm from the centre
wwM

(c) 99  – 4 sq. cm 25 3
(c) sq.cm (d) 25 3 sq. cm of the circle, then the length of
4
(d) 169  – 4  sq. cm the chord of the same circle
128. The are a of a c ircle with
121. If the four equal circles of radius which is at a distance of 8 cm
circumference 22cm is
3 cm touch each other externally, from the centre is equal to
(a) 38.5 cm² (b) 39 cm²
then the area of the region (a) 10 cm (b) 20 cm
(c) 36.5 cm² (d) 40 cm²
bounded by the four circles is (c) 30 cm (d) 40 cm
129. In ABC , O is the centroid and 137. A semicircular shaped window
(a) 4 ( 9 – ) sq. cm
AD, BE, CF are three medians and has diameter of 63 cm, its
(b) 9 4 – sq. cm the area of AOE = 15 cm2 then 22
area of quadrilateral BDOF is perimeter equals ( )
(c) 5 6 – sq. cm 7
(a) 20 cm2 (b) 30 cm2 (a) 126 cm (b) 162 cm
2
(d) 6 5 – sq. cm (c) 40 cm (d) 25 cm2 (c) 198 cm (d) 251 cm

Rakesh Yadav Readers Publication Pvt. Ltd. 306

For More Visit : www.LearnEngineering.in


For More Visit : www.LearnEngineering.in

138. In an equilateral triangle ABC of 146. What is the area of the triangle 155. Two circles with centre A and B
side 10 cm, the side BC is trisected whose sides are 9cm,10cm and 11 and radius 2 units touch each
at D & E. Then the length (in cm) of cm ? other externally at ‘C’, A third
AD is (a) 30 cm2 (b) 60 cm2 circle with centre ‘C’ and radius
(a) 3 7 (b) 7 3 (c) 30 2 cm2 (d) 60 2 cm2 ‘2’ units meets other two at D and
147. The area of an isosceles triangle E. The n the area of the
10 7 7 10 is 4 square units, If the length of quadrilateral ABDE is
(c) (d)
3 3 the unequal side is 2 unit, the (a) 2 2 sq. units
139. The perimeter of a triangle is length of each equal side is
(b) 3 3 sq. units
40cm and its area is 60 cm2. If (a) 4 units (b) 2 3 units
the largest side measures 17 cm, (c) 3 2 sq. units
(c) 17 units (d) 3 2 units

ir
then the length ( in cm ) of the
148. What is the area of a triangle (d) 2 3 sq. units

v.iSn
smallest side of the triangle is
(a) 4 (b) 6 (c) 8 (d) 15 having perimeter 32 cm, one side 156. If the perimeter of a right angled
11 cm and difference of other two triangle is 56 cm and area of the
140. From four corners of a square sides 5 cm?
sheet of side 4 cm four pieces triangle is 84 sq. cm, then the
(a) 8 30 cm2 (b) 5 35 cm2

dnag
each in the shape of arc of a length of the hypotenuse is (in cm)
circle with radius 2 cm are cut (c) 6 30 cm2 (d) 8 2 cm2 (a) 25 (b) 50 (c) 7 (d) 24
out. The area of the remaining 157. If the length of each median of an
149. Area of equilateral triangle having
portion is : equilateral triangle is 6 3 cm, the
side 2cm is

eYari
(a) (8 –  ) sq. cm
(a) 4 cm² (b) perimeter of the triangle is
(b) (16 – 4 ) sq. cm 3 cm²
(a) 24 cm (b) 32 cm
(c) (16 – 8 ) sq. cm (c) 3 cm² (d) 6 cm² (c) 36 cm (d) 42 cm
(d) (4 – 2 ) sq. cm 150. The area of a circle is increased 158. The area of an equilateral triangle
141. If the numerical value of the
perim eter of an equilateral
triangle is 3 times the area of
snhe
by 22 cm2 when its radius is
increased by 1 cm. The original
radius of the circle is
is 4 3 sq. cm. Its perimeter is
(a) 12 cm (b) 6 cm
(a) 3 cm (b) 5 cm
kgei
it, then the length of each side of (c) 8 cm (d) 3 3 cm
the triangle is (c) 7cm (d) 9 cm
159. A gear 12 cm in diameter is
151. The radii of two circles are 5 cm
(a) 2 units (b) 3 units turning a gear 18 cm in diameter.
and 12 cm. The area of a third
ERna

(c) 4 units (d) 6 units When the smaller gear has 42


circle is equal to the sum of the
142. Each side of an equilateral area of the two circles. The radius revolutions. how many has the
triangle is 6 cm. Find its area of the third circle is : larger one made?
(a) 13 cm (b) 21 cm (a) 28 (b) 20 (c) 15 (d) 24
(a) 9 3 sq. cm (b) 6 3 sq. cm
160. The perimeter of a semicircle is
aBryn

(c) 30 cm (d) 17 cm
(c) 4 3 sq. cm (d) 8 3 sq. cm 152. The perimeter of a semicircular 18 cm, then the radius is:
143. The length of three medians of a path is 36 m. Find the area of this 22
triangle are 9 cm, 12 cm and 15 semicircular path. (using  )
7
cm. The area ( in sq. cm) of the (a) 42 sq. m (b) 54 sq. m
Les

triangle is (a) 63 sq. m (d) 77 sq. m (a) 5


1
cm (b) 3 1 cm
(a) 24 (b) 72
wa. th

153. The area of a circle inscribed in a 3 2


(c) 48 (d) 144 square of area 2m2 is (c) 6 cm (d) 4 cm
144. The area of the triangle formed 161. A circle and a rectangle have the
 
by the straight line 3x +2y = 6 (a) m2 (b) m2 same perimeter. The sides of the
and the co-ordinate axes is 4 2
rectangle are 18 cm and 26 cm.
wwM

(a) 3 square units (c)  m2 (d) 2 m2


The area of the circle is
(b) 6 square units 154. Three circles of radii 4 cm, 6 cm
(c) 4 square units and 8 cm touch each other pair 22
(d) 8 square units (Take  )
wise externally. The area of the 7
145.If the length of each side of an triangle formed by the line-
equilateral triangle is increased (a) 125 cm2 (b) 230 cm2
segments joining- the centres of the (c) 550 cm 2
(d) 616 cm2
by 2 unit, the area is found to be
three circles is 162. The area of a circle is 38.5 sq.
increased by 3 + 3 square unit.
(a) 144 13 sq. cm cm. Its circumference (in cm) is
The length of each side of the
triangle is (b) 12 105 sq. cm  22 
(a) 3 units (b) 3 units (c) 4 3 sq. cm use  =
 

 7 
(c) 3 3 units (d) 3 2 units (d) 24 6 sq. cm (a) 22 (b) 24 (c) 26 (d) 32

Rakesh Yadav Readers Publication Pvt. Ltd. 307

For More Visit : www.LearnEngineering.in


For More Visit : www.LearnEngineering.in

163. A circle is inscribed in a square 171. Three circles of radius a, b, c 178. From a point P which is at a
whose length of the diagonal is touch each other externally. The distance of 13 cm from centre O
area of the triangle formed by of a circle of radius 5 cm in the
12 2 cm. An equilateral triangle joining their centre is same plane, a pair of tangents PQ
is inscribed in that circle. The and PR are drawn to the circle
length of the side of the triangle is (a) a  b  c abc Area of quadrilateral PQOR is
(a) 4 3 cm (b) 8 3 cm (a) 65 cm2 (b) 60 cm2
(b) a  b  c  ab  bc  ca (c) 30 cm 2
(d) 90 cm2
(c) 6 3 cm (d) 11 3 cm (c) ab + bc + ca 179. A circular road runs around a
164. The area (in sq. unit) of the triangle (d) None of the above circular ground. If the difference
formed in the first quadrant by the 172. The radii of two circles are 10 cm between the circumference of the
line 3x +4y =12 is and 24 cm. The radius of a circle outer circle and the inner circle is

r
(a) 8 (b) 12 (c) 6 (d) 4 whose area is the sum of the area 66 meters, the width of the road is:

Sni
165. The height of an equilateral of these two circles is 22
triangle is 15 cm. the area of the (a) 36 cm (b) 17 cm ( Take   )

gv.i
7
triangle is (c) 34 cm (d) 26 cm
(a) 10.5 metres (b) 7 metres
(a) 50 3 sq. cm 173. A circ le is inscr ibed in an
equilateral triangle and a square (c) 5.25 metres (d) 21 metres

ridna
(b) 70 3 sq. cm is inscribed in that circle. The 180. A p erson ob serv ed that he
ratio of the areas of the triangle required 30 seconds less time to
(c) 75 3 sq. cm and the square is cross a circular ground along its
diameter than to cover it once

eeYa
(d) 150 3 sq. cm (a) 3:4 (b) 3 :8 along the boundary. If his speed
166. The area of an equilateral triangle was 30 m/ minutes. then the
(c) 3 3 : 2 (d) 3 3 :1
is 9 3 m 2 . The leng th radius of the circular ground is
174. If area of an equilateral triangle
(in m) of the median is is a and height b, then value of 22
geisnh ( Take   ):
(a) 2 3 (b) 3 3 b 2 7
is: (a) 5.5 m (b) 7.5 m
(c) 3 2 (d) 2 2 a
(c) 10.5 m (d) 3.5 m
167. The sides of a triangle are 16 cm, 1 1 181. The difference of perimeter and
Enak

12 cm and 20 cm. Find the area, (a) 3 (b) (c) 3 (d) diameter of a circle is X unit. The
3 3
(a) 64 cm2 (b) 112 cm2 diameter of the circle is
(c) 96 cm 2
(d) 81 cm2 175.If ABC is similar to DEF
X X
R

168. 360 sq. cm and 250 sq. cm are such that BC = 3 cm, EF = 4 cm (a) unit (b) unit
the area of two similar triangles.
 –1 1
and area of ABC = 54 cm 2,
If the length of one of the sides of X X 
aryn

then the area of DEF is : (c) unit  – 1


(d)   unit
the first triangle be 8 cm , then   
(a) 66 cm2 (b) 78 cm2
the length of the corresponding
Les B

(c) 96 cm 2
(d) 54 cm2 182. The area of the circumcircle of an
side of the second triangle is equilateral triangle is 3 sq. cm
176. The area of two similar triangles
1 1 ABC and DEF are 20cm2 and 45 .The perimeter of the triangle is
(a) 6 cm (b) 6 cm
5 3 cm2 respectively. If AB =5 cm, (a) 3 3 cm (b) 9 cm
wa. th

2 then DE is equal to (c) 18 cm (d) 3 cm


(c) 6 cm (d) 6 cm (a) 6.5 cm (b) 7.5 cm
3 183. A horse is tied to a post by a rope.
(c) 8.5 cm (d) 5.5 cm
169. The perimeter of an isosceles If the horse moves along a
triangle is 544 cm and each of 177.C 1 and C 2 are two concentric circular path always keeping the
circles with centre at O, Their
wwM

rope streched and describes 88


5 radii are 12 cm and 3 cm,
the equal sides is times the metres when it has traced out 72°
6 respectively. B and C are the
at the centre, the length of the
base . What is the area (in cm2) point of contact of two tangents
of the triangle ? drawn to C2 from a point A lying 22
on the circle C1. Then, the area rope is (Take   )
(a) 38172 (b) 18372 7
of the quadrilateral ABOC is
(c) 31872 (d) 13872 (a) 70 m (b) 75 m
170. The altitude drawn to the base of 9 15 (c) 80 m (d) 65 m
(a) sq . cm
an isosceles triangle is 8 cm and 2 184. Three circles of radii 3.5 cm, 4.5
its perimeter is 64 cm. The area (b) 12 15 sq. cm cm and 5.5 cm touch each other
(in cm2) of the triangle is externally. Then the perimeter of
(a) 240 (b) 180 (c) 9 15 sq. cm the triangle formed by joining the
(c) 360 (d) 120 (d) 6 15 sq. cm centres of the circles, in cm is

Rakesh Yadav Readers Publication Pvt. Ltd. 308

For More Visit : www.LearnEngineering.in


For More Visit : www.LearnEngineering.in

(a) 27 externally, then the area of the


1 circle with diamter AB is
(b)  [(3.5)2 +(4.5)2+ (5.5)2] (c) × area of ABC
(c) 27 4 (a) 36 (b) 64
(d) 13.5 1 (c) 144 (d) 256
185. Three sides of a triangular field (d) × area of ABC 198. If the numerical value of the
8
are of length 15 m, 20m and 25 height and the area of an
m long respectively. Find the cost 191. A wire of length 44 cm is first bent equilateral triangle be same. then
of sowing seeds in the field at the to form a circle and then rebent the length of each side of the
rate of 5 rupees per sq. m to form a square. The difference triangle is
of the two enclosed areas is (a) 2 units (b) 4 units
(a) Rs.300 (b) Rs.600
(a) 44 cm2 (b) 33 cm2 (c) 5 units (d) 8 units
(c) Rs.750 (d) Rs.150 2
(c) 55 cm (d) 66 cm2 199. If the length of a side of the

ir
186. A chord of length 30 cm is at a
distance of 8 cm from the centre of 192. ACB is an ang le in the square is equal to that of the

v.iSn
a circle. The radius of the circle is: semicircle of diameter AB = 5 cm diameter of a circle, then the ratio
(a) 17 cm (b) 23 cm and AC : BC = 3: 4. The area of of the area of the square and that
the triangle ABC is
(c) 21 cm (d) 19 cm 22
(a) 6 2 sq. cm (b) 4 sq. cm of the circle (  )

dnag
187. The radius of the incircle of a 7
triangle whose sides are 9 cm, 12 (c) 12 sq. cm (d) 6 sq. cm (a) 14 : 11 (b) 7 : 11
cm and 15 cm is 193. If the lengths of the sides AB, BC (c) 11 : 14 (d) 11 : 7
(a) 9 cm (b) 13 cm and CA of a triangle ABC are 10 200. The median of an equilateral

eYari
(c) 3 cm (d) 6 cm cm, 8 cm and 6 cm respectively
and If M is the mid-point of BC triangle is 6 3 cm. The area ( in
188. The ratio of inradius and
circumradius of a square is : and MN  AB to cut AC at N. cm2) of the triangle is
then area of the trapezium ABMN (a) 72 (b) 108
(a) 1 : 2 (b) 2 : 3
(c) 1 : 3 (d) 1 : 2
189. Three circles of equal radius ‘a’
cm touch each other. The area of
is equal to
snhe
(a) 18 sq. cm (b) 20 sq. cm
(c) 12 sq. cm (d) 16 sq. cm
194. In an equilateral triangle of side
(c) 72 3 (d) 36 3
201. If the numerical value of the
circumference and area of a circle
is same, then the area is
kgei
the shaded region is : 24 cm, a circle is inscribed touching (a) 6 sq. units
its sides. The area of the remaining (b) 4 sq. units
portion of the triangle is
(c) 8 sq. units
ERna

( 3 =1.732) (d) 12 sq. units


(a) 98.55 sq. cm (b) 100 sq. cm 202. The area of an equilateral triangle
(c) 101 sq. cm (d) 95 sq. cm is 48 sq. cm. The length of the
195. Two sides of a plot measuring 32 side is
aBryn

m and 24 m and the angle (a) 4 8 cm (b) 4 3 cm


 3   between them is a perfect right
  angle. The other two sides (c) 8 cm (d) 8 4 3 cm
(a)  2  a2sq. cm
  measure 25 m each and the other 203. The external fencing of a circular
three angles are not right angles. path around a circular plot of
Les

 6 3 –  The area of the plot in m2 is land is 33m more than its interior
  2
wa. th

(b)   a sq. cm (a) 768 (b) 534 fencing. The width of the path
 2  (c) 696.5 (d) 684 around the plot is
196. A and b are two sides adjacent to (a) 5.52 m (b) 5.25 m
(c)  3 – a2sq. cm
 the right angle of a right angled (c) 2.55 m (d) 2.25 m
triangle and p is the 204. The perimeter of a triangle is 54
wwM

 2 3 –  perpendicular drawn to the m and its sides are in the ratio


  2 hypotenuse from the opposite
(d)   a sq. cm 5 : 6 : 7. The area of the triangle is
 2  vertex. Then p2 is equal to
(a) 18 m2 (b) 54 6 m2
190. ABC is a right angled triangle. B 1 1
being the right angle Mid- points (a) a2 +b2 (b) 2  2 (c) 27 2 m2 (d) 25 m2
a b
of BC and AC are respectively 205. A circular wire of diameter 112
B ' and A ' . Area of  A ' B ' C is a 2b 2 cm is cut and bent in the form of
(c) (d) a2 – b2 a rectangle whose sides are in the
1 a 2  b2
(a) × area of ABC ratio of 9 : 7. The smaller side of
2 197. A is the centre of circle whose
the rectangle is
2 radius is 8 and B is the centre of
a circle whose diameter is 8. If (a) 77 cm (b) 97 cm
(b) × area of ABC
3 thes e tw o c i r c le s to uc h (c) 67 cm (d) 84 cm

Rakesh Yadav Readers Publication Pvt. Ltd. 309

For More Visit : www.LearnEngineering.in


For More Visit : www.LearnEngineering.in

206. If the perimeter of an equilateral 213. A bicycle wheel has a diameter unoccupied space inside the
triangle be 18 cm, then the length (including the tyre) of 56 cm. The triangle is
of each median is number of times the wheel will (a) 21 m² (b) 11 m²
rotate to cover a distance of 2.2 (c) 20 m² (d) 22 m²
(a) 3 2 cm (b) 2 3 cm
22 221. In the figure, OED and OBA are
(c) 3 3 cm (d) 2 2 cm km is (Assume   ) sectors of a circle with centre O.
7
207. Two equal maximum sized (a) 625 (b) 1250 The area of the shaded portion.
circular plates are cut off from a (c) 1875 (d) 2500 D
circular p ape r sheet of 214. If the altitude of an equilateral 4m A
circumference 352 cm. Then the triangle is12 3 cm, then its area
circumference of each circular O 45°
would be;

r
plate is
(a) 176 cm (b) 150 cm (a) 36 3 cm² (b) 144 3 cm²

Sni
3m B
(c) 165 cm (d) 180 cm (c) 72 cm² (d) 12 cm²
E

gv.i
208. The inradius of an equilateral 215. Let C1 and C2 be the inscribed and
triangle is 3 cm, then the circumscribed circles of a triangle
11 11
with sides 3 cm, 4 cm and 5 cm, (a) m² (b) m²
perimeter of that triangle is 16 8

ridna
(a) 18 cm (b) 15 cm area of C1
then is 11 11
(c) 12 cm (d) 6 cm area of C2
(c) m² (d) m²
209. The dif ference between the 2 4
9 16

eeYa
circumference and diameter of a (a) (b)
circle is 150 m. The radius of that 25 25 30
222. If the circumference of a circle is ,
22 9 4 
circle is ( Take   ) (c) (d) then the diameter of the circle is
7 16 25
geisnh
(a) 25 metre (b) 35 metre 216. A circular swimming pool is 15 30
(c) 30 metre (d) 40 metre surrounded by a concrete wall (a) 30 (b) (c) 60 (d)
4m wide. If the area of the  ²
210. The perimeters of a circle, a
square and an e quilater al concrete wall surrounding the 223. The outer and inner diameter of
Enak

triangle are same and their areas 11 a circular path be 728 cm and 700
are C, S and T respectively. pool is that of the pool, then cm respectively. The breadth of the
25
Which of the following statement path is
is true ? the radius(in m) of the pool :
R

(a) 8 (b) 16 (c) 30 (d) 20 (a) 7 cm (b) 14 cm


(a) C = S = T (b) C > S > T
217. If the area of a circle is A, radius (c) 28 cm (d) 20 cm
(c) C < S < T (d) S < C < T
of the circ le is r and 224. A piece of wire when bent to form
aryn

1 circumference of it is c, then a circle will have a radius of 84


211.A horse takes 2 seconds to
Les B

2 C r cm. If the wire is bent to form a


complete a round around a (a) rC = 2A (b) = square, the length of a side of the
A 2
circular field. If the speed of the square is
horse was 66 m/sec, then the r² A (a) 152 cm (b) 168 cm
radius of the field is, [Given (c) AC = (d) =C
wa. th

4 r (c) 132 cm (d) 225 cm


22 218. The sides of a triangle having area 225. The area of a circle is 324
 ] 7776 sq. cm are in the ratio 3 : 4
7  sq.cm. The length of its longest
: 5. The perimeter of the triangle chord (in cm.) is
(a) 25.62 m (b) 26.52 m is:
wwM

(c) 25.26 m (d) 26.25 m (a) 36 (b) 38 (c) 28 (d) 32


(a) 400 cm (b) 412 cm
212. The diameter of the front wheel (c) 424 cm (d) 432 cm 226. The circumference of a triangle
of an engine is 2x cm and that of is 24 cm and the circumference
219. The perimeter of a sheet of paper
rear wheel is 2y cm to cover the in the shape of a quadrant of a of its in-circle is 44 cm. Then the
same distance, find the number circle is 75 cm. Its area would ar ea of the triangle is
of times the rear wheel will
revolve when the front wheel 22 22
be ( = ) (taking   )
revolves ‘n’ times, 7 7
n yn (a) 512.25 cm² (b) 346.5 cm² (a) 56 square cm
(a) times (b) times (c) 100 cm² (d) 693 cm²
xy x (b) 48 square cm
220. A circ le is inscr ibed in an
nx xy (c) 84 square cm
equilateral triangle of side 8m.
(c) times (d) times (d) 68 square cm
y n The approximate area of the

Rakesh Yadav Readers Publication Pvt. Ltd. 310

For More Visit : www.LearnEngineering.in


For More Visit : www.LearnEngineering.in

227. If the length of each of two equal 233. The area of a circle whose radius 60° and the measure of one of its
sides of an isosceles triangle is 10 is the diagonal of a square whose sides is 10 cm. The length of its
cm. and the adjacent angle is 45°, area is 4 is: smaller diagonal is :
then the area of the triangle is (a) 4 (b) 8 (a) 10cm (b) 10 3 cm
(a) 20 2 square cm (c) 6 (d) 16
5
234. The diagonals of a rhombus are (c) 10 2 cm (d) 2 cm
(b) 25 2 square cm 2
32 cm and 24 cm respectively. The
perimeter of the rhombus is: 244. The perimeter of a rhombus is 100
(c) 12 2 square cm
(a) 80 cm (b)72 cm cm, If one of its diagonals is 14
(d) 15 2 square cm cm.Then the area of the rhombus is
(c) 68 cm (d) 64 cm
228. The inner-radius of a triangle is 235. The diagonals of a rhombus are (a) 144 cm2 (b) 225 cm2

ir
6 cm, and the sum of the lengths 24 cm and 10 cm. The perimeter (c) 336 cm2 (d) 400 cm2

v.iSn
of its sides is 50 cm. The area of of the rhombus (in cm) is : 245. The ratio of the length of the
the triangle (in sq. cm.) is (a) 68 (b) 65 (c) 54 (d) 52 parallel sides of a trapezium is
(a) 150 (b) 300 (c) 50 (d) 56 236. The perimeter of a rhombus is 40 3 : 2. The shortest distance
229. One of the angles of a right- cm, If one of the diagonals be 12 between them is 15 cm. If the

dnag
angled triangle is 15°, and the cm long, what is the length of the area of the trapezium is 450 cm2
hypotenuse is 1 m. The area of other diagonal ? the sum of the length of the
the triangle (in sq. cm.) is parallel sides is
(a) 12 cm (b) 136 cm,
(a) 1220 (b) 1250 (a) 15 cm (b) 36 cm

eYari
(c) 1200 (d) 1215 (c) 16 cm (d) 44 cm (c) 42 cm (d) 60 cm
230. If an isosceles triangle the length 237. The perimeter of a rhombus is 40 246. A parallelogram has sides 15 cm
of each equal side is 'a' units and m and its height is 5m its area is: and 7 cm long. The length of one
that of the third side is 'b' units, (a) 60 m2 (b) 50 m2 of the diagonals is 20 cm. The
then its area will be

(a)
a
4a²–a² sq. units
(c) 45 m2 snhe (d) 55 m2
238. The perimeter of a rhombus is 40
cm. If the length of one of its
area of the parallelogram is
(a) 42 cm2
(c) 84 cm2
(b) 60 cm2
(d) 96 cm2
4
kgei
diagonals be 16 cm, the length 247. Sides of a parallelogram are in
of the other diagonal is the ratio 5 : 4. Its area is 1000
b
(b) 4a²–b² sq. units (a) 14 cm (b) 15 cm sq. units. Altitude on the greater
4 (c) 16 cm (d) 12 cm
ERna

side is 20 units. Altitude on the


a 239. The area of a rhombus is 150 smaller side is
(c) 2a²–b² sq. units cm 2. The length of one of its (a) 20 units (b) 25 units
2
diagonals is 10 cm. The length of (c) 10 units (d) 15 units
the other diagonal is :
aBryn

b 248. The perimeter of a rhombus is 40


(d) a²–2b² sq. units (a) 25 cm (b) 30 cm
2 cm and the measure of an angle is
(c) 35 cm (d) 40 cm 60°, then the area of it is:
231. What is the position of the 240. The area of a regular hexagon of
circumcentre of an obtuse-angles (a) 100 3 cm2 (b) 50 3 cm2
side 2 3 cm is :
triangle?
Les

(a) It is the vertex opposite to the (a) 18 3 cm2 (b) 12 3 cm2 (c) 160 3 cm2 (d) 100 cm2
wa. th

largest side. (c) 36 3 cm2 (d) 27 3 cm2 249. Two ad jace nt side s of a
(b) It is the mid point of the parallelogram are of length 15 cm
241. Each side of a regular hexagon is
largest side. and 18 cm, If the distance between
1 cm. The area of the hexagon is
(c) It lies outside the triangles. two smaller sides is 12 cm, then
wwM

(d) It lies inside the triangles. 3 3 3 3 the distance between two bigger
(a) cm2 (b) cm2 sides is
232. The ratio of cirumference and 2 4
diameter of a circle is 22 : 7. If (a) 8 cm (b) 10 cm
(c) 4 3 cm2 (d) 3 2 cm2
(c) 12 cm (d) 15 cm
4 242. The length of one side of a
the circumference be 1 m, then 250. A parallelogram ABCD has sides
7 rhombus is 6.5 cm and its AB = 24 cm and AD = 16 cm. The
the radius of the circle is: altitude is 10 cm. If the length of distance between the sides AB
its diagonal be 26 cm, the length and DC is 10 cm. Find the
1 1 of the other diagonal will be:
(a) m (b) m distance between the sides AD
4 3 (a) 5 cm (b) 10 cm and BC.
(c) 6.5 cm (d) 26 cm
1 (a) 15 cm (b) 18 cm
243. The measure of each of two
(c) m (d) 1 m (c) 16 cm (d) 9 cm
2 opposite angles of a rhombus is

Rakesh Yadav Readers Publication Pvt. Ltd. 311

For More Visit : www.LearnEngineering.in


For More Visit : www.LearnEngineering.in

251. The ad jace nt side s of a diagonal is 40 2 cm. If the BE


paralleogram are 36 cm and 27 (a) AD (b)
cm in length, If the distance length of the diagonals of the BC EC
between the shorter sides is 12 rhombus are in the ratio 3 : 4,
cm, then the distance between then its area (in cm2) is AD  BE AD  BC
the longer sides is (a) 1550 (b) 1600 (c) (d)
AD  CE AD
(a) 10 cm (b) 12 cm (c) 1535 (d) 1536 266. perimeter of a rhombus is 2p unit
(c) 16 cm (d) 9 cm 259. ABCD is a parallelogram BC is and sum of length of diagonals
produced to Q such that BC is m unit, then area of the
252. If the diagonals of a rhombus are = CQ. Then rhombus is
8 cm and 6 cm, then the area of
square having same side as that (a) area ( ΔABC ) = area ( ΔDCQ ) 1 2
(a) m p sq unit
4

r
of rhombus is (b) area ( ΔABC ) > area ( ΔDCQ )
(a) 25 (b) 55 (c) 64 (d) 36 1

Sni
253. Two circles with centres A and B (c) area ( ΔABC ) < area ( ΔDCQ ) (b) mp 2 sq unit
4

gv.i
and radius 2 units touch each (d) area ( ΔABC )  area ( ΔDCQ ) 1
other externally at ‘C’ A third (c) m 2 – p 2 sq unit
 
260. ABCD is a parallelogram. P and 4
circle with centre ‘C’ and radius
Q are the mid- points of sides BC
‘2’ units meets other two at D and 1

ridna
E. The n the area of the
and CD respectively. If the area of (d) m 2  p 2 sq unit

4
quadrilateral ABDE is ABC is 12 cm2, then the area of 267. Area of regular hexagon with side
APQ is ‘a’ is
(a) 2 2 sq. units

eeYa
(a) 12 cm2 (b) 8 cm2 3 3 2
(b) 3 3 sq. units (c) 9 cm 2
(d) 10 cm2 (a) a sq. unit
4
(c) 3 2 sq. units 261. The area of a rhombus is 216 cm2
and the length of its one diagonal 12
geisnh (b) a2 sq. unit
(d) 2 3 sq. units is 24 cm. The perimeter (in cm) 2 3
254. The perimeter of a non-square of the rhombus is 9
rhombus is 20 cm. One of its (a) 52 (b) 60 (c) a2 sq. unit
2 3
diagonal is 8 cm. The area of the (c) 120 (d) 100
Enak

rhombus is 262. One of the four angles of a 6


rhombus is 60°. If the length of (d) a2 sq. unit
(a) 28 sq. cm (b) 20 sq. cm 2
(c) 22 sq. cm (d) 24 sq. cm each side of the rhombus is 8 cm, 268. In ABC , D and E are two
R

255. The perimeter of a rhombus is then the length of the longer points on the sides AB and AC
diagonal is
100 cm and one of its diagonals respectively so that DE  BC and
(a) 8 3 cm (b) 8 cm
aryn

is 40 cm. Its area (in cm2) is


AD 2
(a) 1200 (b) 1000 8 = . Then
BD 3
Les B

(c) 600 (d) 500 (c) 4 3 cm (d) cm


3 the area of trapezium DECB
256. In ABC , D and E are the points 263. The diagonals of a rhombus are is
of sides AB and BC respectively the area of ABC
12 cm and 16 cm respectively.
equal to
such that DE  AC and AD : The length of one side is
wa. th

(a) 8 cm (b) 6 cm 5 21
BD = 3 : 2. The ratio of area of (a) (b)
trap ezium ACED to that of (c) 10 cm (d) 12 cm 9 25
BED is 264. A parallelogram has sides 60 m 4 1
and 40 m and one of its diagonals (c) 1 (d) 5
(a) 4 : 15 (b) 15 : 4 5 4
wwM

is 80 m long. Its area is 269. The sides of a rhombus are 10


(c) 4 : 21 (d) 21 : 4
257. ABCD is a trapezium in which (a) 500 15 m2 (b) 600 15 m2 cm each and a d iagonal
measures 16 cm. Area of the
AB  DC and AB = 2 CD. The (c) 400 15 m2 (d) 450 15 m2 rhombus is
diagonals AC and BD meet at O. 265.ABCD is a trapezium with AD and (a) 96 sq. cm (b) 160 sq. cm
The ratio of area of triangles AOB BC parallel sides. The ratio of the (c) 100 sq. cm (d) 40 sq. cm
and COD is area of ABCD to that of  AED is 270. The lengths of two parallel sides
(a) 1 : 1 (b) 1 : 2 of a trapezium are 6 cm and 8 cm.
B E C If the height of the trapezium be
(c) 4 : 1 (d) 1 : 4 4 cm, then its area is
258. The length of each side of a (a) 28 cm² (b) 56 cm²
rhombus is equal to the length (c) 30 cm² (d) 36 cm²
of the side of a square whose A N D

Rakesh Yadav Readers Publication Pvt. Ltd. 312

For More Visit : www.LearnEngineering.in


For More Visit : www.LearnEngineering.in

271. If diagonals of a rhombus are 24 spent to make the ground usable (a) 2 : 5 (b) 2 : 5
cm and 32 cm, then perimeter of at the rate of 25 paise per sq. m.
The breadth of the ground is 50 (c) 4 : 25 (d) 4 : 5
that rhombus is
m. If the length of the ground is 288. The ratio of base of two triangles
(a) 80 cm (b) 84 cm
increased by 20 m. what will be the is x : y and that of their areas is
(c) 76 cm (d) 72 cm a : b. Then the ratio of their
expenditure (in rupees) at the same
272. The ar ea of an isosceles corresponding altitudes will be:
rate per sq. m.?
trapezium is 176 cm² and the
(a) 1,250 (b) 1,000 a b
height is 2/11th of the sum of its (a) : (b) 1 : 1
(c) 1,500 (d) 2,250 y x
parallel sides. If the ratio of the
280. A hall 25 metres long and 15
length of the parallel sides is 4 : x b
metres broad is surrounded by a
7, then the length of a diagonal
verandah of uniform width of 3.5 (c) ay : bx (d) a : y
(in cm) is

ir
metres. The cost of flooring the 289. The area of a field in the shape of

v.iSn
(a) 2 137 (b) 24 varandah, at Rs. 27.50 per a trapezium measures 1440m².
square metre is The perpendicular distance
(c) 137 (d) 28 (a) Rs. 9149.50 (b) Rs. 8146.50 between its parallel sides is 24m.
273. The perimeter of a rhombus is (c) Rs. 9047.50 (d) Rs. 4186.50 If the ratio of the parallel sides is
281. The outer circumference of a 5 : 3, the length of the longer

dnag
60 cm and one of its diagonal is
24 cm. The area of the rhombus circular race-track is 528 metre. parallel side is :
is The track is everywhere 14 metre (a) 75 m (b) 45 m
(a) 432 sq.cm (b) 216 sq.cm wide. Cost of levelling the track (c) 120 m (d) 60 m
at the rate of Rs.10 per sq. metre 290. If the ratio of areas of two squares

eYari
(c) 108 sq.cm (d) 206 sq.cm
is : is 225 : 256, then the ratio of
274. The area of the parallelogram their perimeter is :
(a) Rs. 77660 (b) Rs. 67760
whose length is 30 cm, width is 20 (a) 225 : 256 (b) 256 : 225
(c) Rs. 66760 (d) Rs. 76760
cm and one diagonal is 40 cm is (c) 15 : 16 (d) 16 : 15
282. The length and breadth of a
(a) 200 15 cm²

(b) 300 15 cm²


snhe
rectangular field are in the ratio
of 3 : 2. If the perimeter of the
field is 80m, its breadth (in
291. The area of a triangle is 216 cm2
and its sides are in the ratio 3 : 4 :
5. The perimeter of the triangle is:
(a) 6 cm (b) 12 cm
metres) is :
kgei
(c) 100 15 cm² (c) 36 cm (d) 72 cm
(a) 18 (b) 16 (c) 10 (d) 24
283. The sides of a rectangular plot are 292. A circular wire of radius 42 cm
(d) 150 15 cm²
in the ratio 5 : 4 and its area is is bent in the form of a rectangle
ERna

275. The area of a rhombus is 256 equal to 500 sq.m The perimeter whose sides are in the ratio of
sq.cm. and one of its diagonal is of the plot is : 6 : 5. The smaller side of the
twice the other in length.Then (a) 80 m (b) 100 m 22
length of its larger diagonal is (c) 90 m (d) 95 m rectangle is ( Take  ):
7
(a) 32 cm (b) 48 cm
aBryn

284. ABC is a triangle with base AB, (a) 60 cm (b) 30 cm


(c) 36 cm (d) 24 cm D is a point on AB such that AB (c) 25 cm (d) 36 cm
276. The length of two parallel sides of a = 5 and DB =3. What is the ratio 293. The ratio of the outer and the
trapezium are 15 cm and 20 cm. If of the area of  ADC to the area inner perimeter of a circular path
its area is 175 sq.cm, then its height is 23 : 22, If the path is 5 meters
of  ABC ?
Les

is: wide the diameter of the inner


(a) 25 cm (b) 10 cm (a) 2/5 (b) 2/3 circle is:
wa. th

(c) 20 cm (d) 15 cm (c) 9/25 (d) 4/25 (a) 110 m (b) 55 m


285. If the area of a triangle is 1176 (c) 220 m (d) 230 m
277. The cost of carpenting a room is
cm2 and the ratio of base and 294.The angles of a triangle are in the
Rs. 120 . If the width had been 4
corresponding altitude is 3 : 4, then ratio 3 : 4 : 5. The measure of the
metres less, the cost of the Carpet
the altitude of the triangle is: largest angle of the triangle is
wwM

would have been Rs. 20 less. The


(a) 42 cm (b) 52 cm (a) 60° (b) 75°
width of the room is :
(c) 54 cm (d) 56 cm (c) 120° (d) 150°
(a) 24 m (b) 20 m 295.The ratio of the area of a square
286. The sides of a triangle are in the
(c) 25 m (d) 18.4 m to that of the square drawn on
278. The floor of a corridor is 100 m 1 1 1 its diagonal is:
ratio : : . If the perimeter of
long and 3 m wide. Cost of 2 3 4 (a) 1 : 1 (b) 1 : 2
covering the floor with carpet 50 the triangle is 52 cm, the length (c) 1 : 3 (d) 1 : 4
cm wide at the ratio of Rs. 15 per of the smallest side is : 296.A square and an equilateral
m is (a) 24 cm (b) 10 cm triangle are drawn on the same
(a) Rs. 4500 (b) Rs. 9000 (c) 12 cm (d) 9 cm base. The ratio of their area is
(c) Rs. 7500 (d) Rs. 1900 287. If the diagonals of two squares (a) 2 :1 (b) 1 : 1
279. A playground is in the shape of a are in the ratio of 2 : 5. Their area (c) (d) 4 : 3
30 : 4
rectangle. A sum of Rs. 1,000 was will be in the ratio of

Rakesh Yadav Readers Publication Pvt. Ltd. 313

For More Visit : www.LearnEngineering.in


For More Visit : www.LearnEngineering.in

297. If the area of a circle and a square perimeter is 72 cm. The length of (c) 96 square units
are equal, then the ratio of their its greatest side (in cm) is (d) 60 3 square units
perimeter is (a) 24 (b) 27 (c) 30 (d) 36
(a) 1 : 1 (b) 2 :  315. The parallel sides of a trapezium
307. The ratio of the radii of two are in a ratio 2 : 3 and their
(c)  : 2 (d)  : 2 wheels is 3: 4. The ratio of their shortest distance is 12 cm.If the
298.The area of two equilateral circumference is area of the trapezium is 480 sq.
triangles are in the ratio 25 : 36. (a) 4 : 3 (b) 3 : 4 cm., the longer of the parallel
Their altitudes will be in the ratio: (c) 2 : 2 (d) 3 : 2 sides is of length :
(a) 36 : 25 (b) 25 : 36 308. The sides of a triangle are in the (a) 56 cm (b) 36 cm
ratio 2 : 3 : 4. the perimeter of (c) 42 cm (d) 48 cm
(c) 5 : 6 (d) 5 : 6
the triangle is 18 cm. The area 316. An equilateral triangle is drawn
299. If the length and the perimeter of

r
(in cm2) of the triangle is on the diagonal of a square . The
a rectangle are in the ratio 5 : 16. (a) 9 (b) 36

Sni
then its length and breadth will ratio of the area of the triangle to
be in the ratio (c) 42 (d) 3 15 that of the square is

gv.i
(a) 5 : 11 (b) 5 : 8 309. The ratio of the areas of the (a) 3 :2 (b) 1 : 3
(c) 5 : 4 (d) 5 :3 circumcircle and the incirle of an
300. Through each vertex of a triangle, equilateral triangle is (c) 2 : 3 (d) 4 : 3

ridna
a line parallel to the opposite side (a) 2 : 1 (b) 4 : 1 317. Two triangles ABC and DEF are
is drawn. the r atio of the (c) 8 : 1 (d) 3 :2 similar to each other in which
perimeter of the new triangle. AB = 10 cm, DE = 8 cm. Then the
thus formed, with that of the 310. In ABC , the medians CD and

eeYa
ratio of the area of triangles ABC
original triangle is BE intersect each other at O, then
and DEF is
(a) 3 : 2 (b) 1 : 2 the ratio of the areas of ODE
(a) 4 : 5 (b) 25 : 16
(c) 2 : 1 (d) 2 : 3 and ΔOBC is (c) 64 : 125 (d) 4 : 7
301. The ratio of the number giving the (a) 1 : 4 (b) 6 : 1
geisnh 318. The ratio between the area of two
measure of the circumference and (c) 1 : 12 (d) 12 : 1 circles is 4 : 7. What will be the
the area of a circle of radius 3 cm is
311. The ratio of the area of two ratio of their radii ?
(a) 1 : 3 (b) 2 : 3
isosceles triangles having the
(c) 2 : 9 (d) 3 : 2 (a) 2 : 7 (b) 4 : 7
same vertical angle (i.e. angle
Enak

302. The height of an equilateral between equal sides) is 1 : 4. The (c) 16 : 49 (d) 4 : 7
triangle is 4 3 cm. The ratio of ratio of their heights is
319. The area of a circle is proportional
the area of its circumcircle to that (a) 1 : 4 (b) 2 : 5
to the square of its radius. A
R

of its in-circle is (c) 1 : 2 (d) 3 : 4


small circle of radius 3 cm is
(a) 2 : 1 (b) 4 : 1 312. The ratio of length of each equal side
drawn within a larger circle of
and the third side of an isosceles
aryn

(c) 4 : 3 (d) 3 : 2 radius 5 cm. Find the ratio of the


triangle is 3 : 4. If the area is
303. The radius of circle A is twice that area of the annular zone to the
Les B

of circle B and the radius of circle 8 5 units². the third side is area of the larger circle (Area of
B is twice that of circle C. Their (a) 3 units the annular zone is the difference
area will be in the ratio (b) 2 5 square units units between the area of the larger
(a) 16 : 4 : 1 (b) 4 : 2 : 1 circle and that of the smaller
(c) 8 2 units circle)
wa. th

(c) 1 : 2 : 4 (d) 1 : 4 : 16
304. A circle and a square have equal (d) 12 units (a) 9 : 16 (b) 9 : 25
areas. the ratio of a side of the square 313. The ratio of sides of a triangle is (c) 16 : 25 (d) 16 : 27
and the radius of the circle is 3 : 4 : 5. If area of the triangle is 320. The diameter of two circles are
72 square unit then the length the side of a square and the
wwM

(a) 1 :  (b)  :1 of the smallest side is : diagonal of the square. The ratio
(c) 1 : (d)  :1 (a) 4 3 unit (b) 5 3 unit of the area of the smaller circle
305. The sides of a triangle are in the and the larger circle is
(c) 6 3 unit (d) 3 3 unit (a) 1 : 2 (b) 1 : 4
1 1 1 314. The ratio of sides of a triangle is
ratio : : and its perimeter (c) 2: 3 (d) 1 : 2
3 4 5 3 : 4 : 5 and area of the triangle
is 94cm . The length of the is 72 square units. Then the area 321. The ratio of the area of an
smallest side of the triangle is: of an equilateral triangle whose equilateral triangle and that of its
perimeter is same as that of the circumcircle is
(a) 18 cm (b) 22.5 cm
previous triangle is
(c) 24 cm (d) 27 m (a) 2 3 : 2 (b) 4 :
306. The sides of a quadrilateral are (a) 32 3 square units
in the ratio 3 : 4 : 5 : 6 and its (b) 48 3 square units (c) 3 3 : 4 (d) 7 2 : 2

Rakesh Yadav Readers Publication Pvt. Ltd. 314

For More Visit : www.LearnEngineering.in


For More Visit : www.LearnEngineering.in

322. If the perimeters of a rectangle 329. If the arcs of unit length in two breadth is increased by 10%, By
and a square are equal and the circles subtend angles of 60° and what percent is its area changed?
ratio of two adjacent sides of the 75° at their centres, the ratio of (a) 0% (b) 1%
rectangle is 1 : 2 then the ratio of their radii is (c) 5% (d) 100%
area of the rectangle and that of (a) 3 : 4 (b) 4 : 5 337. The percentage increase in the
the square is (c) 5 : 4 (d) 3 : 5 area of a rectangle. If each of
(a) 1 : 1 (b) 1 : 2 330. ABCD is a parallelogram in which its sides is increased by 20%, is:
(c) 2 : 3 (d) 8 : 9 diagonals AC and BD intersect at (a) 40% (b) 42%
323. The perimeter of a rectangle and O. If E, F, G and H are the mid- (c) 44% (d) 46%
an equilateral triangle are same. points of AO, DO, CO and BO 338. If the circumference of a circle is
Also, one of the sides of the respectively, then the ratio of the reduced by 50%, its area will be
perimeter of the quadrilateral reduced by

ir
rectangle is equal to the side of the
EFGH to the perimeter of (a) 12.5% (b) 25%
triangle. The ratio of the area of the

v.iSn
parallelogram ABCD is
rectangle and the triangle is (c) 50% (d) 75%
(a) 1 : 4 (b) 2 : 3
339. If the side of a square is increased
(a) 3 :1 (b) 1: 3 (c) 1 : 2 (d) 1 : 3
by 25%, then its area is increased
331. If the circumference of a circle by:

dnag
(c) 2 : 3 (d) 4 : 3
increases from 4 to 8 , what (a) 25% (b) 55%
324. The radius of a circle is a side of change occurs in its area ? (c) 40.5% (d) 56.25%
a square. The ratio of the area of (a) It doubles (b) It triples 340. If the rad ius of a cir cle is
the circle and the square is (c) It quadruples (d) It is halved

eYari
increased by 50% . its area is
(a) 1 :  (b)  : 1 332. If the length of a rectangle is increased by :
(c)  : 2 (d) 2 :  increased by 25% and the width (a) 125% (b) 100%
325. ABC is an isosceles right angled is decreased by 20%, then the (c) 75% (d) 50%
area of the rectangle :
triangle with B = 90°, On the
sides AC and AB, two equilateral
triangles ACD and ABE have
been constructed. The ratio of
snhe
(a) Increases by 5%
(b) decreases by 5%
(c) remains unchanged
341. If the length of a rectangle is
increased by 20% and its breadth
is decreased by 20%, then its
area
kgei
(d) increases by 10% (a) increases by 4%
area of ABE and ACD is
333. The area of a circle of radius 5 is (b) decreases by 4%
(a) 1 : 3 (b) 2 : 3 numerically what percent of its (c) decreases by 1%
ERna

(c) 1 : 2 (d) 1 : 2 circumfernce ?


(d) None of these
(a) 200% (b) 255%
326. Two triangles ABC and DEF are 342. If each side of a rectangle is
(c) 240% (d) 250%
similar to each other in which AB= increased by 50%, its area will
334. If the circumference and area of
10 cm, DE=8 cm. Then the ratio of be increased by
a circle are numerically equal,
aBryn

the area of triangles ABC and DEF (a) 50% (b) 125%
then the diameter is equal to :
is (c) 100% (d) 250%
(a) area of the circle
(a) 4 : 5 (b) 25 : 16 343. If the altitude of a triangle is
(c) 64 : 125 (d) 4 : 7  increased by 10% while its area
(b) (c) 2 (d) 4
327. ABC is a right angled triangle, B 2 remains same, its corresponding
Les

being the right angle. Mid-points 335. If D and E are the mid-points of base will have to be decreased by
wa. th

of BC and AC are respectively the side AB and AC respectively (a) 10% (b) 9%
B' and A' .The ratio of the area of of the ABC in the given figure 1 1
here, the shaded region of the (c) 9 % (d) 11 %
the quadrilateral AA'B'B to the 11 9
triangle is what per cent of the
344. If the circumference of a circle is
wwM

area of the triangle ABC is whole triangular region ?


(a) 1 : 2 (b) 2 : 3 A
increased by 50% then the area
(c) 3 : 4 will be increased by
(d) None of the above (a) 50% (b) 75%
328. The sides of a triangle are in the (c) 100% (d) 125%
D E 345. The length and breadth of a
1 1 1
ratio : : and its perimeter rectangle are increased by 12%
4 6 8 and 15% respectively. Its area
is 91 cm. The difference of the B C
will be increased by :
length of longest side and that of (a) 50% (b) 25%
shortest side is (c) 75% (d) 60% 1 4
(a) 27 % (b) 28 %
(a) 19 cm (b) 20 cm 336. The length of a rectangle is 5 5
(c) 28 cm (d) 21 cm de creased by 1 0% and its (c) 27% (d) 28%

Rakesh Yadav Readers Publication Pvt. Ltd. 315

For More Visit : www.LearnEngineering.in


For More Visit : www.LearnEngineering.in

346. If the sides of an equilateral 354. In measuring the sides of a


triangle are increased by 20%, rectangle, there is an excess of 9 3
(c) cm² (d) 3 3 cm²
30% and 50% respectively to 5% on one side and 2% deficeit 4
form a new triangle the increase on the other. Then the error 363. The area of a sector of a circle of
in the perimeter of the equilateral percent in the area is radius 36 cm is 72 cm². The
triangle is (a) 3.3% (b) 3.0 % length of the corresponding arc
 (c) 2.9% (d) 2.7% of the sector is
(a) 25% (b) 33 % 355. The length and breadth of a (a)  cm (b) 2 cm
3
(c) 75% (d) 100% rectangle are increased by 30% and (c) 3 cm (d) 4 cm
347. Each side of a rectangular field 20% respectively. The area of the
364. A square is inscribed in a circle
is diminished by 40%. By how rectangle so formed exceeds the of diameter 2a and another
area of the square by

r
much percent is the area of the square is circumscribing circle.
field diminished ? (a) 46% (b) 66% The difference between the areas

Sni
(a) 32% (b) 64% (c) 42% (d) 56% of outer and inner squares is

gv.i
(c) 25% (d) 16% 356. If side of a square is increased (a) a² (b) 2a² (c) 3a²(d) 4a²
348. The length of r ectangle is by 40%, the percentage increase 365. ABC is a triangle right angled at
increased by 60%. By what in its surface area is A. AB = 6 cm and AC = 8 cm.
percent would the breadth to (a) 40% (b) 60% Semi-circles drawn (outside the

ridna
be decreased to maintain the (c) 80% (d) 96% triangle) on AB, AC and BC as
same area? 357. If the diameter of a circle is diameters which enclose areas x,
1 increased by 8%, then its area is y and z square units,

eeYa
(a) 37 % (b) 60% increased by : respectively. What is x +y–z equal
2 to?
(a) 16.64% (b) 6.64%
(c) 75% (d) 120% (a) 48 cm² (b) 32 cm²
349. The length and breadth of (c) 165 (d) 16.46%
(c) 0 (d) None of these
358. One sid e of a r ectangle is
rectangle are increased by 20%
geisnh 366. Consider an equilateral triangle
and 25% res pectively. The increased by 30%. To maintain
of a side of one unit length. A new
increase in the area of the the same area, the other side will
equilateral triangle is formed by
resulting rectangle will be : have to be decreased by joining the mid-points of one,
(a) 60% (b) 50% 1 12 then a third equilateral triangle
Enak

(c) 40% (d) 30% (a) 23 % (b) 76 % is formed by joining the mid-
13 13
350. If each side of a square is increased points of second. The process is
by 10%. its area will be increased by (c) 30% (d) 15% continued. The perimeter of all
R

(a) 10% (b) 21% 359. The length and breadth of a triangles, thus formed is
(c) 44% (d) 100% rec tang le are d oubled. (a) 4 (b) 5
Percentage increase in area is
aryn

351. If the length of a rectangular plot (c) 6 (d) 7


of land is increased by 5% and (a) 150% (b) 200% 367. What is the area of the larger
Les B

the breadth is decreased by 10%. (c) 300% (d) 400% segment of a circle formed by a
How much will its area increase 360. The length of a rectangle is chord of length 5 cm subtending
or decrease? increased by 10% and breadth an angle of 90º at the centre?
(a) 6.5% increase decreased by 10%. The area of 25  
(b) 5.5% decrease the new rectangle is (a)   1
  cm²
wa. th

4 2 
(c) 5.5% increase (a) neither inc reas ed nor
(d) 6.5% decrease decreased 25  
352. The radius of circle is increased (b) 4  2 – 1 cm²
(b) increased by 1%  
by 1%. How much does the area (c) decreased by 2%
wwM

of the circle increase ? 25  3 


(d) decreased by 1% (c) 4  2  1 cm²
(a) 1% (b) 1.1%  
361. How many circular plates of
(c) 2% (d) 2.01% diameter d be taken out of a (d) None of these
353. The length of a room floor exceeds square plate of side 2d with 368. A rectangle of maximum area is
its breadth by 20m . The area of minimum loss of material? drawn inside a circle of diameter
the floor remains unaltered when 5 cm. What is the maximum area
(a) 8 (b) 6 (c) 4 (d) 2
the length is decreased by 10 m of such a rectangle?
362. What is the total area of three
but the breadth is increased by (a) 25 cm² (b) 12.5 cm²
equilateral triangles inscribed in
5 m. The area of the floor (in (c) 12 cm² (d) None of these
a semi-circle of radius 2 cm?
square meters) is: 369. If AB and CD are two diameters
(a) 280 (b) 325 3 3 of a cricle of radius r and they
(a) 12 cm² (b) cm² are mutually perpendicular, then
(c) 300 (d) 420 4

Rakesh Yadav Readers Publication Pvt. Ltd. 316

For More Visit : www.LearnEngineering.in


For More Visit : www.LearnEngineering.in

what is the ratio of the area of 377. The perimeter of a rectangle 385. The arc AB of the circle with centre
the circle to the area of the having area equal to 144 cm² and at O and radius 10 cm has length
ACD ? sides in the ratio 4:9 is 16 cm. What is the area of the
(a) 52 cm (b) 56 cm sector bounded by the radii OA,
  (c) 60 cm (d) 64 cm OB and the arc AB?
(a) (b)  (c) (d) 2
2 4 378. One side of a parallelogram is (a) 40  sq cm(b) 40 sq cm
370. What is the area of a circle whose 8.06 cm and its perpendicular (c) 80 sq cm (d) 20 sq cm
area is equal to that of a triangle distance from opposite side is
386.The length of a room floor
with sides 7 cm, 24 cm and 25 2.08 cm. What is the approximate
exceeds its breadth by 20m. The
cm? area of the parallelogram?
area of the floor remains
(a) 80 cm² (b) 84 cm² (a) 12.56 cm² (b) 14.56 cm²
unaltered when the length is
(c) 16.76 cm² (d) 22.56 cm²

ir
(c) 88 cm² (d) 90 cm² decreased by 10m bu t the
379. In the figure given below, the area

v.iSn
371. If the area of an equilateral breadth is increased by 5m. The
of rectangle ABCD is 100 sq cm,
triangle is x and its perimeter is O is any p oint on AB and area of the floor (in square
y, then which one of the following CD=20cm. Then, the area of metres) is:
is correct?  COD is (a) 280 (b) 325

dnag
(a) y4 = 432x² (b) y4 = 216x² A O B (c) 300 (d) 420
(c) y2 = 432x² (d) None of these 387.Find the perimeter of a square
372. A rectangular field is 22 m long whi ch i s sy mmetrically
and 10 m wide. Two hemis- inscribed in semicircle of radius

eYari
pherical pitholes of radius 2 m D Q C 10 cm.
are dug from two places and the (a) 40 sq cm (b) 45 sq cm (a) 80cm (b) 80 cm
mud is spread over the remaining (c) 50 sq cm (d) 80 sq cm
part of the field. The rise in the 380. If an isosceles right angled (c) 8 24 cm (d) 16 5 cm
level of the field is

(a)
8
93
m (b)
13
93
m (a) 3 units
snhe
triangle has area 1sq unit, then
what is its perimeter?
388. Consider the following statement
I. Area of a segment of a circle
is les s than area of its
kgei
corresponding sector.
(b) 2 2 +1 units
16 23 II. Distance trav elle d by a
(c) m (d) m circular wheel of diameter 2d
93 93 (c)  2  1 units
 cm in one revolution is greater
ERna

373. The area of an isosceles  ABC than 6d cm.


(d) 2  2  1 units

with AB = AC is 12 sq cm and Which of the above statements
altitude AD = 3 cm. What is its 381. A circular water fountain 6.6 m is/are correct?
perimeter? in diameter is surrounded by a (a) Only I
aBryn

(a) 18 cm (b) 16 cm path of width 1.5 m. The area of (b) Only II


(c) 14 cm (d) 12 cm this path (in sq m) is (c) Both I and II
374. A hospital room is to (a) 13.62 (b) 13.15 (d) Neither I nor II
accommodate 56 patients. It (c) 12.15 (d) None of these 389. The Perimeter of a rectangle is 82
m and its area is 400 sq m. What
Les

should be done in such a way 382. The area of a rectangular filed is


that every patient gets 2.2 m² of 4500 sq m. If its length and is the breadth of the rectangle?
wa. th

floor and 8.8 m³ of space. If the breadth are in the ratio 9:5, then (a) 18 m (b) 16 m
its perimeter is (c) 14 m (d) 12 m
length of the room is 14m, then
390. The area enclosed between the
breadth and the height of the (a) 90 m (b) 150 m
circumference of two concentric
room are respectively (c) 280 m (d) 360 m circles is 16 sq cm and their
wwM

(a) 8.8 m,4m (b) 8.4 m,4.2m 383. The area of a square inscribed in radii are in the ratio 5:3. What is
(c) 8 m, 4 m (d) 7.8 m,4.2m a circle of radius 8 cm is the area of the outer circle?
375. How many 200 mm lengths can (a) 32 sq cm (b) 64 sq cm (a) 9 sq cm (b) 16 sq cm
be cut from 10 m of ribbon? (c) 128 sq cm (d) 256 sq cm (c) 25 sq cm (d) 36 sq cm
(a) 50 (b) 40 (c) 30 (d) 20 384. The short and long hands of a 391. If the circumference of a circle is
376. What is the area between a clock are 4 cm and 6 cm long, equal to the perimeter of square,
square of side 10 cm and two respectively. Then, the ratio of then which one of the following
inverted semi-circular, cross- distances travelled by tips of is correct?
sections each of radius 5 cm short hand in 2 days and long (a) Area of circle = Area of square
inscribed in the square? hand in 3 days is (b) Area of circle  Area of square
(a) 17.5 cm² (b) 18.5 cm² (a) 4 : 9 (b) 2 : 9 (c) Area of circle > Area of square
(c) 20.5 cm² (d) 21.5 cm² (c) 2 : 3 (d) 1 : 27 (d) Area of circle< Area of square

Rakesh Yadav Readers Publication Pvt. Ltd. 317

For More Visit : www.LearnEngineering.in


For More Visit : www.LearnEngineering.in

392. If the circumference of two circle 399. The area of a rectangle, whose m. If a horse with 4.2 m long rope
are in the ratio 2:3, then what is one side is a is 2a². What is the tied at a vertex. The percentage
the ratio of their areas? area of a square having one of the of the total area of the field which
(a) 2:3 (b) 4:9 diagonal of the rectangle as side? is available for grazing is best
(c) 1:3 (d) 8:27 (a) 2a² (b) 3a² (c) 4a²(d) 5a² approximated by
393. If the area of a circle inscribed in 400. If the outer and inner diameters (a) 50% (b) 55%
an equilateral triangle is 154 sq of a stone parapet around a well (c) 59% (d) 62%
cm, then what is the perimeter ar e 11 2 cm and 70 cm
408. The areas of two circles are in the
of the triangle? respectively. Then, what is the
ratio 1:2. If the two circles are
area of the parapet?
(a) 21 cm (b) 42 3 cm bent in the form of squares, then
(a) 264 sq cm (b) 3003 sq cm
what is the ratio of their areas?
(c) 21 3 cm (d) 42 cm (c) 6006 sq cm (d) 24024 sq cm

r
(a) 1 : 2 (b) 1 : 3
394. In the  ABC, the base BC is 401. If the area of a  ABC is equal

Sni
to area of square of side length (c) 1 : 4 (d) 1 : 4
trisected at D and E. The line
6 cm,then what is the length of 409. If the four equal circles of radius

gv.i
through D, Parallel to AB, meets
the altitud e to AB , whe r e
AC at F and the line through E 3cm to uch each oth er
AB = 9 cm?
parallel to AC meets AB at G. If externally, then the area of the
(a) 18 cm (b) 14 cm
EG and DF intersect at H, then region bounded by the four

ridna
(c) 12 cm (d) 8 cm
what is the ratio of the sum of circles is:
402. What is the area of an equilateral
the area of parallelogram AGHF
triangle having altitude equal to (a) 49   sq.cm
and the area of the  DHE to the

eeYa
2 3 cm?
area of the  ABC? (b) 94   sq.cm
(a) 3 sq cm (b) 2 3 sq cm
1 1 1 1 (c) 56   sq.cm
(a) (b) (c) (d) (c) 3 3 sq cm (d) 4 3 sq cm
2 3 4 6
(d) 65   sq.cm
395. If the area of a circle is equal to
geisnh
403. If a lawn 30 m long and 16 m
the area of a square with side wide is surrounded by a path 2 410. If the diamete r of a circ le
m wide, then what is the area of circumscribing a square is 15 2
2  units, then what is the the path?
diameter of the circle? (a) 200 m² (b) 280 m² cm, then what is the length of the
Enak

(a) 1 unit (b) 2 units (c) 300 m² (d) 320 m² side of the square?
(c) 4 units (d) 8 units 404. If a c ircle circumscr ibes a (a) 15 cm (b) 12 cm
396. A s quar e, a cricle and an rectangle with side 16 cm and 12 (c) 10 cm (d) 7.5 cm
R

equilateral triangle have same cm, then what is the area of the 411. Three congruent circles each of
perimeter. circle? radius 4 cm touch one another.
(a) 48 sq cm (b) 50 sq cm
aryn

Conside r the following What is the area (in cm²) of the


statements (c) 100 sq cm (d) 200 sq cm portion included between them?
Les B

I. The area of square is greater 405. The lengths of two sides of a right
angled triangle which contain the (a) 8 (b) 16 3 –8
than the area of the triangle.
II. The area of circle is less than right angle ar e a and b, (c) 16 3 – 4 (d) 16 3 –2
the area of triangle. respectively. Three squares are
drawn on the three sides of the 412. The two diagonals of a rhombus
Which of the above statement is/
wa. th

triangle on the outer side.What are of lengths 55 cm and 48 cm.


are correct?
is the total area of the triangle If P is the perpendicular height
(a) Only I (b) Only II
and the three squares? of the rhombus, then which one
(c) Both I and II of the following is correct?
(a) 2(a²+b²)+ab
(d) Neither I nor II
wwM

(b) 2(a²+b²)+2.5ab (a) 36 cm < p < 37 cm


397. If the area of a rectangle whose (c) 2(a²+b²)+0.5ab
length is 5 more than twice its (b) 35 cm < p < 36 cm
(d) 2.5(a²+b²) (c) 34 cm < p < 35 cm
width is 75 sq units. What is the
406. A wall is of the form of a
perimeter of the rectangle? (d) 33 cm < p < 34 cm
trapezium with height 4 m and
(a) 40 units (b) 30 units parallel sides being 3 m and 5m. 413. The Perimeter of a triangular field
(c) 24 units (d) 20 units What is the cost of painting the is 240 m. If two of its sides are
398. If the altitude of an equilateral wall, if the rate of painting is 78 m and 50 m, then what is the
triangle is 3 cm, then what is Rs.25 per sq m? length of the perpendicular on
(a) Rs. 240 (b) Rs. 400 the side of length 50 m from the
its perimeter? opposite vertex?
(c) Rs. 480 (d) Rs. 800
(a) 3 cm (b) 3 3 cm (a) 43 m (b) 52.2 m
407. A grassy field has the shape of
(c) 6 cm (d) 6 3 cm an equilateral triangle of side 6 (c) 67.2 m (d) 70 m

Rakesh Yadav Readers Publication Pvt. Ltd. 318

For More Visit : www.LearnEngineering.in


For More Visit : www.LearnEngineering.in

414. A piece of wire 78 cm long is bent 421. If a wire of length 36 cm is bent (c) (128 –15 )/128 cm²
in the form of and isosceles in the form of a semi-circle, then (d) None of these
triangle. If the ratio of one of the what is the radius of the semi- 427. PQRS is a diameter of a circle of
equal sides to the base is 5:3, then circle? radius 6 cm as shown in the
what is the length of the base? (a) 9 cm (b) 8 cm figure above. The lengths PQ, QR
(a) 16 cm (b) 18 cm (c) 7 cm (d) 6 cm and RS are equal. Semi-circles
(c) 20 cm (d) 30 cm 422. In the given figure, the side of are drawn on PQ and QS as
415. The length of a minute hand of a square ABCD is 7 cm. What is diamete rs. What is the
wall clock is 9 cm. What is the the area of the shaded portion, perimeters of the shaded region?
area swept (in cm²) by the minute formed by the arcs BD or the
hand in 20 min? (take  = 3.14) circles with centre at C and A?
A D

ir
(a) 88.78 (b) 84.78
P S

v.iSn
(c) 67.74 (d) 57.78 Q R
416. In the figure given below, AB is a
line of length 2a, with M as mid-
point. Semi-circles are drawn on
(a) 12  cm (b) 14 cm

dnag
one side with AM, and AB as
(c) 16  cm (d) 18 cm
diameters. A circle with centre O and B C
radius r is drawn such that this (a) 7cm² (b) 28 cm² 428. A person rides a bicycle round a
circle touches all the three semi- circular path of radius 50 m. The
(c) 14 cm² (d) 21 cm²
radius of the wheel of the bicycle

eYari
circles. What is the value of r? 423. What is the maximum area of a
is 50 cm. The cycle comes to the
rectangle, the perimeter of which
starting point for the first time in
. is 18 cm?
O 1 h. What is the number of
(a) 20.25 cm² (b) 20.00 cm² revolutions of the wheel in 15

A
2a
M
a a
B
a
snhe
(c) 19.75 cm (d) 19.60 cm²
424. Three circular laminas of the
same radius are cut out from a
larger circular lamina. When the
min?
(a) 20 (b) 25 (c) 30 (d) 35
429. If a man walking at the rate 3
(a) (b) (c) (d) km/h crosses a square field
kgei
3 2 3 4 radius of each lamina cut out is diagonally in 1 min, then what is
417. A circle and a square have the the largest possible, then what is the area of the field?
same perimeter.Which one of the the ratio (approximate) of the (a) 1000 m² (b) 1250 m²
ERna

following is correct? area of the residual piece of the


(c) 2500 m² (d) 5000 m²
(a) The area of the circle is equal original lamina to its original
430. The difference between the area
to that of square total area?
of a square and that of an
(b) The area of the circle is larger (a) 0.30 (b) 0.35 equilateral triangle on the same
aBryn

than that of square (c) 0.40 (d) 0.45 base is 1/4 cm². What is the
(c) The area of the circle is less 425. A wire is in the form of a radius length of side of triangle?
than that of square 42 cm. If it is bent into a square, 1/2
then what is the side of the (a) (4 – 3 ) cm
(d) No conclusion can be drawn
418. What is the radius of the circle square? (b) (4 + 1/2
3 ) cm
Les

inscribed in a triangle having side (a) 66 cm (b) 42 cm


–1/2
(c) 36 cm (d) 33 cm (c) (4 – 3) cm
wa. th

lengths 35 cm, 44 cm and 75 cm?


(a) 3 cm (b) 4 cm 426. Seven semi- circluar areas are (d) (4 + 3 )–1/2 cm
(c) 5 cm (d) 6 cm removed from the rectangle
ABCD as shown in the figure 431. A horse is tied to a pole fixed at
419. A rectangle area of 6 sq m is to one corner of a 50 m × 50 m
below, in which AB = 2 cm and
wwM

be painted on a 3m × 4m board square field of grass by means of


leaving a border of uniform width AD = 0.5 cm. The radius of each
semi-circle, r,s,t, u and v is half a 20 m long rope. What is the
on all sides. What should be the area of that part of the field which
width of the border? of that of semi-circle p or q. What
is the area of the remaining the horse can graze?
(a) 0.25 m (b) 0.5 m (a) 1256 m² (b) 942 m²
portion?
(c) 1 m (d) 3 m (c) 628 m² (d) 314 m²
420. A wheel of a bicycle has inner A B 432. From a rectangular metal sheet
diameter 50 cm and thickness 10 r s t of sides 25 cm and 20 cm; a
cm. What is the speed of the p q
u v circular sheet as large as possible
bicycle, If it makes 10 revolutions is cut-off. What is the area of the
D C
is 5 s? remaining sheet?
(a) 5.5 m/s (b) 4.4 m/s (a) (128 –13 )/128 cm² (a) 186 cm² (b) 144 cm²
(c) 3.3 m/s (d) 2.2 m/s (b) (125 –13 )/125 cm² (c) 93 cm² (d) 72 cm²

Rakesh Yadav Readers Publication Pvt. Ltd. 319

For More Visit : www.LearnEngineering.in


For More Visit : www.LearnEngineering.in

433. What is the area of a right angled 439. The Perimeter of a square S1 is 445. A cycle wheel make s 10 00
is osce les triangle whose 12 m more than perimeter of the revolution is moving 440 m. What
hypotenuse is 6 2 cm? square S2. If the area of S1 equals is the diameter of the wheel?
three times, the area of S2 minus (a) 7 cm (b) 14 cm
(a) 12 cm² (b) 18 cm²
11, then what is the perimeter of S1? (c) 28 cm (d) 21 cm
(c) 24 cm² (d) 36 cm²
434. If A is the area of a triangle in (a) 24 m (b) 32 m 446. A c ircle is ins crib ed in a
cm², whose sides are 9 cm, 10 cm (c) 36 m (d) 40 m equilateral triangle of side a.
and 11 cm, then which one of the What is the area of any square
440. From a rectangular sheet of
following is correct? inscribed in this circle?
cardboard of size 5 cm×2 cm, the
(a) A < 40 cm² a² a² a² a²
greatest possible circle is cut-off.
(b) 40 cm² < A < 45 cm² (a) (b) (c) (d)
What is the area of the 3 4 6 8

r
(c) 45 cm² < A < 50 cm²
(d) A > 50 cm² remainting part? 447. Consider a circle C of radius 6

Sni
435. If x and y are respectively the (a) (25 – ) cm² cm with centre at O. What is the
areas of a square and a rhombus (b) (10 – ) cm² difference in the area of the circle

gv.i
of sides of same length, then what (c) (4 – ) cm² C and the area of the sector of C
is x : y? subtending an angle of 80º at O?
(d) (10 –2 ) cm²
(a) 1 : 1 (b) 2 : (a) 26 cm² (b) 16 cm²

ridna
3 441. A chord AB of a cricle of radius
20 cm makes a right angle at the (c) 28 cm² (d) 30 cm²
(c) 4 : 3 (d) 3 : 2
centre of the circle. What is the 448.The ratio of the areas of the in-
436. If the area of a circle, inscribed circle and the circum-circle of
area of the minor segment in

eeYa
in an equilateral triangle is 4 a square is:
cm², then what is the area of the cm²? (take = 3.14)
triangle? (a) 31.4 cm² (b) 57 cm² (a) 1 : 2 (b) 2 :1
(a) 12 3 cm² (b) 9 3 cm² (c) 62.8 cm² (d) 114 cm²
442. The minute hand of a clock is 14 (c) 1 : 2 (d) 2 : 1
geisnh
(c) 8 3 cm² (d) 18 cm² cm long. How much distance 449.The diagram represents the
437. In the given figure,  ABC is a does the end of the minute hand area swept by the wiper of a
right angled triangle, right angled
27 car. Wi th th e di men si on s
at A. Semi-circles are drawn on travel in 15 min? (take = ) given in the figure, calculate
Enak

the sides AB, BC and AC. Then 7


the shaded area swept by the
the area of shaded portion is (a) 11 cm (b) 22 cm wiper.
equal to which one of the (c) 33 cm (d) 44 cm
R

following? 443. A square of side x is taken. A O


A
rectangle is cut out from this 30° 7cm
aryn

square such that on side of the


A D
rectangle is half that of the
Les B

1 14cm
B C square and the other is rd of
(a) Area of  ABC 3
(b) 2 times the area of  ABC the first side of the rectangle. B C
What is the area of the remaining
wa. th

(c) Area of semi-circle ABC


(d) None of the above portion? (a) 102.67cm2 (b) 205.34cm2
438. In the given figure, ABC is a right (b) 51.33cm2 (d) 208.16cm2
3  7 
angled triangle, right angled at (a) 
 x² (b) 
 x² 450.If the length of a chord of a
4  8  circle at a distance of 12cm
wwM

B. BC = 21 cm and AB = 28 cm.
Width AC as diameter of a semi- from the centre is 10cm, then
circle and width BC as radius a 11  15  the diameter of the circle is :
(c) 
  x² (d) 
  x²
quarter circle are drawn. What 12  16  (a) 13 cm (b) 15 cm
is the area of the shaded portion? 444. A rectangle cardboard is 18 (c) 26 cm (d) 30 cm
A
cm×10 cm. From the four corners 451.Are a of the incircl e of an
of the rectangle, quarter circles equilateral triangle with side
of radius 4 cm are cut. What is 6cm is :
the perimeter (approximate) of

the remaining portion? (a) sq .cm (b) 3 sq.cm
B C
(a) 47.1 cm (b) 49.1 cm 2
(a) 425 cm² (b) 425.75 cm²
(c) 428 cm² (d) 428.75 cm²
(c) 51.0 cm (d) 53.0 cm (c) 6 sq.cm (d) 3 sq.cm

Rakesh Yadav Readers Publication Pvt. Ltd. 320

For More Visit : www.LearnEngineering.in


For More Visit : www.LearnEngineering.in

452.Th e adjace nt side s of a of a circle with perimter equal the square coincide with the
parellelogram are 36 cm and 27 to 24  : vertex of right angle of the
cm in length. If the distance (a) 144 (b) 144  triangle?
between the shorter sides is 576 2
(c) 154  (d) none of these (a) cm (b) 24cm2
12cm, then th e di stan ce 49
456.A circle is inscribed in an
between the longer sides is :
equilateral triangle of side 8cm. 24 2
(a) 10 cm (b) 12 cm (c) cm (d) None of these
The area of the portion between 7
(c) 16 cm (d) 9 cm 459. Area of the trapezium formed by
the triangle and the circle is :
453.A circle and a rectangle have x-axis; y-axis and the lines 3x +
(a) 11cm2 (b) 10.95cm2
the same perimeter. The sides 2 4y =12 and 6x + 8y = 60 is:
(c) 10cm (d) 10.50cm2
of the rectangle area 18cm (a) 37.5sq.unit (b) 31.5sq.unit
457.Find the ratio of the diameter

ir
and 26cm. The area of the circle (c) 48sq.unit (d) 36.5sq.unit
of the circles inscribed in and

v.iSn
is : 460.A square having area 200sq.m,
circumscribed an equilateral
(a) 125 cm2 (b) 230 cm2 is formed in such a way that the
2 triangle to its height.
(c) 550 cm (d) 616 cm2
(a) 1 : 2 : 3 (b) 2 : 4 : 3 length of its diagonal is 2 times
454.The perimeter of a semicircular
(c) 1 : 3 : 4 (d) 3 : 2 : 1

dnag
path is 36m. Find the area of of the diagonal of the given
this semicircular path. 458.Find the area of the largest (or square. Then the area of the new
(a) 42sq.m (b) 54sq.m maximum sized) square that square formed is:
(c) 63sq.m (d) 77sq.m can be made inside a right angle
(a) 200 2 sq.m (b) 400 2 sq.m

eYari
triangle having sides 6cm, 8cm
455.Find the area of a rectangle
whose area is equal to the area & 10cm when one of vertices of (c) 400sq.m (d) 800sq.m

ANSWER KEY
1.
2.
3.
4.
(b)
(c)
(a)
(c)
47.
48.
49.
50.
(b)
(d)
(b)
(b)
93. (b)
94. (b)
95. (c)
96. (c)
139. (c)
140. (b)
141. (c)
142. (a)
snhe
185. (c)
186. (a)
187. (c)
188. (a)
231. (c)
232. (a)
233. (b)
234. (a)
277. (a)
278. (b)
279. (a)
280. (d)
323. (c)
324. (b)
325. (c)
326. (b)
369. (b)
370. (b)
371. (a)
372. (c)
415. (b)
416. (c)
417. (b)
418. (d)
kgei
5. (b) 51. (b) 97. (c) 143. (b) 189. (d) 235. (d) 281. (b) 327. (c) 373. (a) 419. (b)
6. (b) 52. (a) 98. (b) 144. (a) 190. (c) 236. (c) 282. (b) 328. (d) 374. (a) 420. (b)
7. (b) 53. (c) 99. (b) 145. (a) 191. (b) 237. (b) 283. (c) 329. (c) 375. (a) 421. (c)
8. (c) 54. (c) 100. (c) 146. (c) 192. (d) 238. (d) 284. (d) 330. (c) 376. (d) 422. (c)
9. (d) 55. (d) 101. (a) 147. (c) 193. (a) 239. (b) 285. (d) 331. (c) 377. (a) 423. (a)
ERna

10. (d) 56. (b) 102. (a) 148. (a) 194. (a) 240. (a) 286. (c) 332. (c) 378. (c) 424. (b)
11. (a) 57. (c) 103. (c) 149. (b) 195. (d) 241. (a) 287. (c) 333. (d) 379. (c) 425. (a)
12. (d) 58. (d) 104. (a) 150. (a) 196. (c) 242. (a) 288. (c) 334. (d) 380. (d) 426. (a)
13. (a) 59. (d) 105. (c) 151. (a) 197. (a) 243. (a) 289. (a) 335. (c) 381. (c) 427. (a)
14. (b) 60. (b) 106. (c) 152. (d) 198. (a) 244. (c) 290. (c) 336. (b) 382. (c) 428. (b)
aBryn

15. (d) 61. (b) 107. (d) 153. (b) 199. (a) 245. (d) 291. (d) 337. (c) 383. (c) 429. (b)
16. (a) 62. (c) 108. (b) 154. (d) 200. (d) 246. (c) 292. (a) 338. (d) 384. (d) 430. (c)
17. (b) 63. (b) 109. (c) 155. (b) 201. (b) 247. (b) 293. (c) 339. (d) 385. (c) 431. (d)
18. (d) 64. (b) 110. (a) 156. (a) 202. (d) 248. (b) 294. (b) 340. (a) 386. (c) 432. (a)
19. (d) 65. (b) 111. (c) 157. (c) 203. (b) 249. (b) 295. (b) 341. (b) 387. (d) 433. (b)
20. (b) 66. (c) 112. (a) 158. (a) 204. (b) 250. (a) 296. (d) 342. (b) 388. (c) 434. (b)
21. (a) 67. (b) 113. (c) 159. (a) 205. (a) 251. (d) 297. (d) 343. (c) 389. (b) 435. (a)
Les

22. (c) 68. (a) 114. (d) 160. (b) 206. (c) 252. (a) 298. (c) 344. (d) 390. (c) 436. (a)
23. (c) 69. (b) 115. (b) 161. (d) 207. (a) 253. (b) 299. (d) 345. (b) 391. (c) 437. (a)
wa. th

24. (d) 70. (c) 116. (d) 162. (a) 208. (a) 254. (d) 300. (c) 346. (b) 392. (b) 438. (d)
25. (d) 71. (a) 117. (c) 163. (c) 209. (b) 255. (c) 301. (b) 347. (b) 393. (b) 439. (b)
26. (a) 72. (c) 118. (a) 164. (c) 210. (b) 256. (d) 302. (b) 348. (a) 394. (b) 440. (b)
27. (d) 73. (c) 119. (c) 165. (c) 211. (d) 257. (c) 303. (a) 349. (b) 395. (c) 441. (d)
28. (b) 74. (b) 120. (b) 166. (b) 212. (c) 258. (d) 304. (b) 350. (b) 396. (a) 442. (b)
29. (c) 75. (b) 121. (b) 167. (c) 213. (b) 259. (a) 305. (c) 351. (b) 397. (a) 443. (c)
wwM

30. (d) 76. (a) 122. (c) 168. (c) 214. (b) 260. (c) 306. (a) 352. (d) 398. (c) 444. (b)
31. (a) 77. (b) 123. (d) 169. (d) 215. (d) 261. (b) 307. (b) 353. (c) 399. (d) 445. (b)
32. (d) 78. (c) 124. (c) 170. (d) 216. (d) 262. (a) 308. (d) 354. (c) 400. (c) 446. (c)
33. (a) 79. (b) 125. (b) 171. (a) 217. (a) 263. (c) 309. (b) 355. (d) 401. (d) 447. (c)
34. (c) 80. (c) 126. (b) 172. (d) 218. (d) 264. (b) 310. (a) 356. (d) 402. (d) 448. (a)
35. (a) 81. (b) 127. (c) 173. (c) 219. (b) 265. (d) 311. (c) 357. (a) 403. (a) 449. (a)
36. (a) 82. (b) 128. (a) 174. (c) 220. (b) 266. (c) 312. (a) 358. (a) 404. (c) 450. (c)
37. (c) 83. (a) 129. (b) 175. (c) 221. (d) 267. (c) 313. (c) 359. (c) 405. (c) 451. (d)
38. (b) 84. (b) 130. (b) 176. (b) 222. (d) 268. (b) 314. (b) 360. (d) 406. (b) 452. (d)
39. (d) 85. (a) 131. (a) 177. (c) 223. (b) 269. (a) 315. (d) 361. (c) 407. (c) 453. (d)
40. (a) 86. (a) 132. (a) 178. (b) 224. (c) 270. (a) 316. (a) 362. (d) 408. (a) 454. (d)
41. (a) 87. (c) 133. (b) 179. (a) 225. (a) 271. (a) 317. (b) 363. (d) 409. (b) 455. (b)
42. (a) 88. (b) 134. (b) 180. (d) 226. (c) 272. (a) 318. (a) 364. (b) 410. (a) 456. (b)
43. (d) 89. (c) 135. (b) 181. (a) 227. (b) 273. (b) 319. (c) 365. (c) 411. (b) 457. (b)
44. (d) 90. (c) 136. (c) 182. (b) 228. (a) 274. (d) 320. (a) 366. (c) 412. (a) 458. (a)
45. (b) 91. (a) 137. (b) 183. (a) 229. (b) 275. (a) 321. (c) 367. (c) 413. (c) 459. (b)
46. (a) 92. (a) 138. (c) 184. (a) 230. (b) 276. (b) 322. (d) 368. (c) 414. (b) 460. (c)

Rakesh Yadav Readers Publication Pvt. Ltd. 321

For More Visit : www.LearnEngineering.in


For More Visit : www.LearnEngineering.in

SOLUTION

Diagonal 2 π (radius) = 44
1. (b) Side of a square = 16
2 44 7 == 4 cm
4
 Diagonal 2 radius = 2 22 = 7 cm
Area of square = 
 

9. (d) Side of the square
 2 
22 Diagonal 4 2
2 area of circle = ×7×7
=
5.2 =
5.2  5.2 7 =
2 2
=
=4
2 2 = 154 cm²
area of the square = 16
= 2.6 × 5.2 = 13.52 cm² Option (b) is the answer. (circle, area of new square= 32

r
2. (c) Area of square 33 cm²)
side of new square = 32

i
.iSn
6. (b) Let the side of square = a
Diagonal2 a2 = 4 2
  and the radius of circle = r
2 2 Diagonal of new square
perimeter of square
3. (a) Le t the le ng th of

agv
= circumference of circle = 4 2 × 2 = 8 cm
rectangular hall = x
4a = 2 π r
 Breadth of rectangular hall 10. (d) Diagonal of square A = (a + b)

ridn
4a side of square
3 r=
= x 2π Diagonal a+b
4 = =

eeYa
area of circle = 3850 m² 2 2
According to question, 2
Area = 768 m² 4a 4a a + b
π× × = 3850
2π 2π area of square A =  
3  2 
x  x = 768
4
geisnh 3850  22 22
3
16a²  a + b ²
7 =
x² = 768 2
4 a² = 3025 m²
area of square
768 4 7. (b) 2(l + b) = 28 B = 2 × area of square A
x² = = 256 × 4
Enak

3 l + b = 14
and l × b = 48 a + b  ²
x² = 256 4 = 32 m. = 2× = (a + b)²
(l + b)² = l² + b² + 2lb 2
Dif fe re nc e of length and (14)² = l² + b² + 48 × 2
R

196 - 96 = l2 + b2 side of square B =  a+b  ²


3 x 32
breadth = x– x = = =8m l² + b² = 100 = (a + b)
4 4 4
aryn

4. (c) Since the room is in cuboid = 1010 diagonal of square B = 


2 a + b
Les B

shape Diagonal = 10 m 11. (a)


Length of largest rod = Diagonal 8. (c) Le t the le ng th of
rectangular hall = x 5m
of cuboid
 Breadth of rectangular hall
32² 12 m
3
wa.th

= l ²b ²h ² = 16² 12²  area of the rectangular garden


3² = x
4 = 12 × 5 = 60 m²
1024 According to question,
= 256144  area of square = 60
9 Area = 192 m²
(side)² = 60
wwM

230412961024 3
= x  x = 192 side = 60
9 4
diagonal of the square
4624 68 2 3
= = = 22 m x ² = 192 = 2 side
9 3 3 4
5. (b) Perimeter of square = 44 cm = 2 × 60 = 120
1924
x² = = 64 × 4
4 × side = 44 3 = 2 30 m
side = 11 cm x = 644 = 16 cm 12. (d)
area of square = (side)² = (11)² d if fe re nc e of length and (a – 3)
= 121cm² 3 x
breadth = x – x = a (a + 5)
Circumference of circle = 44cm 4 4

Rakesh Yadav Readers Publication Pvt. Ltd. 322

For More Visit : www.LearnEngineering.in


For More Visit : www.LearnEngineering.in

According to question, 22
 14  14 264
a² = (a – 3) (a + 5) area of circle=π r² = side = = 32 m
7 4
a² = a² + 5a – 3a – 15
2a = 15 = 616 cm²
 Area of the square
16. (a) l = 10 m, b = 6 m, h = 4m = (side)² = (32)² =1024
15
a = length of diagonal (longest rod)
2 21. (a)
= 1003616 = 152 m
15 25
Length = a + 5 = +5=
2 2 = 2 38 a b
15 17. (b) Let the length of smaller line
breadth = a – 3 = –3 segment = x cm 4a = 40 4b = 32
2 a = 10 cm b = 8 cm

ir
15 – 6 9 The le ng th of larg er line
= = segment = (x + 2) cm area of third square

v.iSn
2 2 = a² – b² = 10² – 8²
According to question,
perimeter of the rectangle = 100 – 64 = 36cm²
(x + 2)² – x² = 32
= 2 (l + b)
x² + 4x + 4 – x² = 32 side of third square= 36 =6 cm

dnag
 25 9
28 perimeter of third square
= 2   = 34 cm x = = 7
 2 2 = 4 × 6 = 24 cm
4
13. (a) According to question, 22. (c) side of the square
The required length = x + 2

eYrai
2(l + b) = 160 = 7 + 2 = 9 cm perimeter
l+b = 80 ..... (i) =
18. (d) A D 4
l–b = 48 ....(ii)  Sides of all five squares are
on solving (i) and (ii) b
24 32 40 76 80
l = 64, b = 16
are a of s quar e = ar ea of
rectangle
(side)² = 64 × 16
B l snhe
= speed × time
C
BD = length of diagonal
= , ,
4 4 4 4 4
,

= 6, 8, 10, 19, 20
ATQ
,
kgei
side = 52 area of another square
64  16 = 32 m =  15 = 13 m
60 = 6² + 8²+ 10²+19²+20²
14. (b) Side of square , whos e
(side)² = 36 + 64 +100 +361 +400
ERna

perimeter is 24 cm BD = l²b²
side = 961 = 31
24  l² + b² = 13² = 169
= = 6cm perimeter of square
4 68
So, area of square = 6² Again, l+b =  15 = 17 = 31×4 = 124
60
aBryn

= 36 cm² 23. (c) Area of the tank


(l + b) = l² + b² + 2 l b = length × breadth
Again, side of square, whose
17² = 169 + 2 l b = 180 × 120 = 21600 m²
32
perimeter is 32 cm = = 8 cm 120 Total area of the circular plot
4 lb = = 60 m² = 40000 + 21600 = 61600 m²
So, area of this square 2
 area of circle = 61600
Les

= 8² = 64 cm² 19. (d) Let the breadth be = x m (radius)² = 61600


wa.th

According to question,  length = (23 + x) m 61600 7


Area of new square  2(x + 23 + x) = 206 (radius)² =
= 64 + 36 = 100 cm² 22
4x = 206 – 46
 side of the new square radius = 28 00  7
16 0
wwM

= 100 = 10 cm x = = 40 m = 7  7  400 = 7 × 20
4
Hence perimeter of new square = 140m
 length = 40 + 23 = 63 m 24. (d) Let the breadth of rectangle
= 10 × 4= 40 cm
 Required area =xm
15. (d) (side)² = 484 cm²
= 63 × 40 = 2520 m²  length = (x +5) m
side = 22 cm
20. (b) Length of rectangle = 48 m  Area of hall = length × breadth
perimeter of square = 4 × 22
Breadth of rectangle =16 m 750 = (x + 5)x
= 88 cm
According to question,
According to question, 750 = 30 × 25
Perimeter of square (clearly 750 = 30 × 25)
2 π r = 88 cm = Perimeter of rectangle
 x = 25, breadth = 25m
88 7 = 2 (48 + 16)
length = 25 + 5
r = 2 22 = 14 cm 4 × side = 2 × 64
= 30 m

Rakesh Yadav Readers Publication Pvt. Ltd. 323

For More Visit : www.LearnEngineering.in


For More Visit : www.LearnEngineering.in

25. (d) Required total area take help from options to save your  Two sides of rectangle
= Area of four walls + area of base valuable time take option(b) x = 5m = (x+ 2) (x +5)
= 2 × 1.25(6 + 4) + 6 × 4 (38 + 32 – 2 × 5) 2 × 5 = (70 – 10)  Perimeter = 2(x + 2 + x + 5)
= 49 m² × 10= 60 × 10 = 600 = 2(2x + 7) = 4x + 14
26. (a) 29. (c) Area of walls = Perimeter of 37. (c) Let the sides of rectangle be
2x base × height 6 cm and 2 cm (or any other
3x = 18 × 3 = 54 m² number)
Ratio of length and breadth 30. (d) a² = 81, a=9  Area of rectangle (Q) = 6 × 2
=3:2  Perimeter of square = 12 cm²
2(l + b) = 20 cm = 9 × 4 = 36 cm  Side of square = 4 cm
2(3x + 2x) = 20 cm  2r +r = 36  Area of square(P)

r
2 ×5x = 20 cm r(2 + ) = 36 = 4 × 4 = 16 cm²

i
.iSn
10x = 20
36
 P>Q
x=2
r = = 7 cm 38. (b) No. of cubes with no side is
 length = 3 × 2 = 6 cm, 22
2 painted = (n–2)3
breadth = 2 × 2 = 4 cm 7

agv
Where n is the side of the big-
area = length × breadth 31. (a) a² = 121,a = 11 ger cube
= 6 × 4 =24 cm² Required number
 Perimeter of square

ridn
27. (d) = 11 × 4= 44 cm = (6–2)³ = 64
b a
 Circum fere nce of circ le Diagonal

eeYa
l a = 44 39. (d) Side of square =
2
2(l +b) = 160m 4a = 160 2r = 44
l + b = 80m ......(i) 15 2
22 =
= 15 cm
a = 40 m 2× × r = 44 2
ATQ
geisnh 7
area of square = (side)²
a² – lb = 100  r = 7 cm
= (15)² = 225 cm²
(40)² –lb = 100 32. (d) Let the no. of hours be x 40. (a)
1600 – lb = 100  (0.3 × 0.2 × 20000) × x = 200
Enak

lb = 1500 ....(ii) × 150 × 8


Clearly, 50 + 30 = 80 2
and 50 × 30 = 1500 2001508
x = 32200
R

length = 50 m

28. (b) x = 200 hrs.


1
aryn

33. (a)
area of square = (Diagonal)²
32m 2
Les B

1 1
38 m = (32)² = × 32 ×32 = 16 × 32
2 2
area of path= 600 m² Area of path = 512 cm²
(l + b – 2x) 2x = 600 = 200 × 220 – 200 × 180 area of triangle
wa.th

(38 + 32 – 2x)2x = 600 = 44000 – 36000 = 8000 m²


(70 – 2x)2x = 600 34. (c) Diagonal of square = diam- 3 1.73288
eter of circle = 8 × 2 = 16 cm = 8 ²
4 4
600
(70 – 2x)x= = 300 16 = 1.732×2×8 =27.712 cm²
wwM

2  side of square = = 8 2 cm
2 Required are =(512+27.712)cm²
70x – 2x² = 300 2 = 539.712 cm²
2x² – 70x + 300 = 0  area of square = 8 2   41. (a) Area of the lawn
x² – 35x + 150 = 0 = 128 cm²
1
x² – 30x – 5x + 150 = 0 = hectare
x(x – 30) –5 (x – 30) = 0 8 2 12
35. (a) Side of square = = 8 cm
(x – 30) (x – 5) =0 2 1
x = 30 not possible  Area of square = 8 × 8 = 64 cm² length × breadth = × 10000m²
12
x = 5 (right) 36. (a) x² + 7x + 10 = x² + 5x + 2x + 10
= x(x + 5) + 2(x + 5) 10000
Alternate 4x × 3x = m²
(l + b –2x) 2x = area of path = 600 = (x+ 2) (x +5) 12

Rakesh Yadav Readers Publication Pvt. Ltd. 324

For More Visit : www.LearnEngineering.in


For More Visit : www.LearnEngineering.in

10000 2(15 × 12) = 2(15+12)× h 35


12x² = 2 × 180 = 2 × 27 × h 50. (b) A B
12
10000 180 20 35 35
x² = h=  m
12  12 27 3
100 Volume of the cuboid = l × b × h D C
x= 35
12 20
= 15 × 12 × According to the question,
100 3
Breadth = 3x = 3 × 35
12 = 60 × 20 = 1200 m³ Radius of circle =
2
100
=
4 = 25 m 46. (a) D Required area of shaded portion

ir
C
42. (a) Let the side of square = a cm .O 22 35 35
2

v.iSn
  35   
ATQ 7 2 2
l × b = 3a² A B = 1225 – 962.5 = 262.5 m2
3 side of a square = AB 51. (b) Diagonal of square = 2 side
20 × a = 3a²

dnag
2 of square
= 2 a units
a = 10 cm 3–x
1
 AC = Diagonal = 2 2 a Here a = (x + 1) and d =
43. (d) 2 2

eYrai
 Diameter = 2 a units
 d= 2a
Circumference= π × diameter
= π × 2a = 2 π a units. 3–x 1 
47. (b) Perimeter of rectangle = 40 m

2
= 2  2  x  1 


Area of path = (l + b + 2x)2x
where x = thickness of path
Let l = 7p , b = 4p
snhe
Length = 12 metre
 2(l + b) = 40
2 (12 + b) = 40
 x = 1 unit
52. (a) A B
kgei
{7p + 4p + 2(4)}2(4)= 416
(11p + 8)8 = 416 40 b
12 + b = = 20
11p + 8 = 52 2
ERna

D l C
11p = 44 b = 20 – 12 = 8 m
48. (d) Pe rc entage inc re as e in Let ABCD is a rectangular car-
44
p= = 4, p=4 pet having length l metre and
11  xy 
area = x  y  % breadth b metre and BD is a
aBryn

 breadth = 4 × 4 = 16 m  100  diagonal


44. (d) Area of the floor= 8 × 6 = 48 m² Here, x = 100%, y = 100%  As we know
= 4800 dm² (1m =10 dm)  Area = l × b = 120 ....(i)
 100  100 
Area of square tile = 100  100  %  Perimeter
 100 
= 4 × 4 = 16 dm² = 2(l + b) = 46
Les

= 300%
4800 Using formula
wa.th

No. of tiles = = 300 49. (b) D C


16  (l + b)² = l 2 + b² + 2l b
16 m  (23)² = l 2 + b² + 2 × 120
45. (b)
 529 = l 2 + b² + 240
A 30 m B
wwM

 l 2 + b² = 529 – 240
2 AC = AB² + BC²
 l 2 + b² = 289
= 30² + 16²
Shape of godown is cuboidcal  l 2 b 2 = 289
length = 15 m , breadth = 12 m, = 900256 diagonal = 17
height = h m diagonal of carpet is17 metres
= 1156 = 34 metre.
Area of four walls = 2(l + b)× h 53. (c) Diagonal of a square
area of floor = l × b Distance travelled by elephant
= 34 – 4 = 30 metre = 6 2 cm
area of ceiling = l × b
ATQ 30 6 2
speed of elephant = Side of a square = = 6 cm
l × b + l × b = 2(l + b)× h 15
2
2(l × b) = 2(l + b)× h = 2 m/s Area of a square = 6 × 6 = 36 cm2

Rakesh Yadav Readers Publication Pvt. Ltd. 325

For More Visit : www.LearnEngineering.in


For More Visit : www.LearnEngineering.in

54. (c) Let the breadth of floor = x m 58. (d) Circumference of wheel  Total distance travelled by
The n the le ng th of f loor = = 2πr wheel in 15 revolutions
= 15 × 44 cm = 660 cm
(x+3) m 22
= 2  21 cm = 132 cm 62. (c)
A.T.Q, 7
x × (x + 3) = 70
No. of revolutions . r
x2 + 3x – 70 = 0
Distance to be covered
x2 + 10x – 7x – 70 = 0 =
Circumference of circle Circumference = 2 π r
(x + 10) (x – 7) = 0
x = 7, x = – 10 = 924 100 = 700 Distance covered in 1 min
132
Breadth = 7m = 2 × 8 × πr

r
Length = 10m 59. (d) 40
New circumference

i
.iSn
Perimeter of floor = 2 (L + B)
= 2 (10 + 7) = 34 m . 28 cm = 2 ×  × r × 10
14 cm
55. (d) Let the breadth of rectangle 2r 10  40
Time taken =
=x m 2r  8

agv
then the length of rectangle Radius of the largest circle = 50 min
= 2x m 1 63. (b)

ridn
A.T.Q, =   side of square 
2
x × 2x = 417.605 12 cm
1

eeYa
2x2 = 417.605 =  28 = 14 cm
2
4170.605 area of the circle = π (radius)²
x2 = 15 cm
2
22 area of the triangle
=  14  14 = 616 cm²
83521
geisnh 7 1
x=  base × height
400 60. (b) =
2

x=
289
20
cm .. r
=
1
2
 15 × 12
Enak

289 = 90 cm²
Breadth = m area of another triangle = 2 × 90
20
R = 180 cm²
R

289  2π r= 88
Length =  2 = 28.90 m
20 1
88 7 =  base × height = 180
aryn

56. (b) Radius of circle = 5 cm r = 2 22 = 14cm 2


Length of arc = 3.5 cm
1
Les B

2 π R = 132 cm  20 × height = 180


1 =
 Area of sector = 2 lr 2
132 7
R = 2 22 = 21 cm
1 180  2
 3.5  5 = 8.75 cm² height = = 18 cm
2 The area between two circles 20
wa.th

57. (c) Radius of circular wheel = π (21)² – π (14)²


64. (b)
= 1.75 m = π { 21² – 14² }
Circ um fe re nc e of c ir cular = π (21+14)(21–14)
22
wwM

wheel = 2 π r = 2   1.75 m 22
7 = 35 7 = 770 cm² area of the square = 81 cm²
7
No. of revolutions side of the square = 81 =9 cm
61. (b)
perimeter of the square = 4 × 9
=
Distance to be covered
Circumference of circle
. 7 cm
= 36 cm
Now, According to question,
= 11000m π r + 2r = 36
22 circumference of wheel
2  1.75 m r ( π + 2) = 36
7 = 2π r
22 36 36 7
11000 = 2× ×7 r = =
= = 1000 7 22 2214
11 2
= 44cm 7

Rakesh Yadav Readers Publication Pvt. Ltd. 326

For More Visit : www.LearnEngineering.in


For More Visit : www.LearnEngineering.in

and AB = BC = AC = x
36  7 3 1 22 
= = 7 = 7 ² –  3.5 ²  Ar (ABC)
36 4 27 
= 1.967 cm² 1 1 1
area of the semi circle = 2 x  3 + 2 x 2 3 + 2  x 5 3
68. (a) π r² = 2464 cm²
r ² 22 72
= = × = 77 cm² 3 1 1 1
2 7 2 2464  7 x² =  x  3 +  x  2 3 +  x  5 3
 r = 4 2 2 2
65. (b) Area of square = (12)² 22
=144 cm² 3 1
= 784 = 28m x² =
×x × 8 3
Area of triangle 4 2
 diameter = 2r = 2 × 28 x = 16
1 = 56 cm
=  base × height  perimeter of triangle

ir
2 69. (b) Re quire d are a =Ar ea of = 3x = 3 × 16 = 48 cm
square – Area of circle

v.iSn
1 Alternative:-
=  12 × height 22
2 = (2a)² – π (a)² = 4a² – a² 2
7 side of equilateral  =
1 3
=  12 × height = 144

dnag
2 28a² – 22a² 6a² (sum of the altitudes draw from
= = inte rnal p oint)
7 7
144  2
height = = 24 cm 70. (c) Diameter of the circle 2
12 = Side of square side =
3
 3 2 35 3 

eYrai
66. (c) A 2 r = 21
21 2
r = m =  8 3 = 16 cm
2 3
D E
2 perimeter =3 × sides

B F C
snhe
22 21 21
 21 
Area = π r² = π  
2
693
= 3 × 16 = 48 cm
73. (c) Perimeter of  = 30 cm
Area = 30 cm²
 3² + 4² = 5² =   = cm²
kgei
7 2 2 2 Check the triplet
 ABC is a right angled
1
triangle = 346
2
cm² 5, 12, 13 ,  3, 4, 5
ERna

1 whose largest side is 13.


ar (ABC)=  AB × BC 71. (a) A
2 Also, 5² + 12² = 13²
And perimeter = 5 + 12 + 13
1 = 30 cm
=  3 × 4 = 6 cm²
aBryn

2 Smallest side = 5 cm
B C
 Required Area of  DEF 74. (b) Diameter of the wheel = 3 m
 Are a of an equilate ral
1 triangle = 400 3 Circumference = π × diameter
= ×6 22
4 66
3 3 =
=
Les

3 (side)² = 400 3 7 7
= cm² 4
Sinc e a whee l cove rs a
wa.th

2
400 3 4 distance e qual to its
(side)² = = 1600 c ir cumf er ence in one
67. (b) 3
3.5 3.5 revolution therefore distance
A C side = 40 m covered in 28 revolutions
wwM

3.5 3.5 perimeter = 3 × side


= 3 × 40 = 120 m 66
7 cm = 28 × = 264 m
3.5 3.5 72. (c) A 7
B 264 metres covered = 1 minutes
D 1
F 1 metre covered = minute
AB = BC = AC = 7 cm P 264
Area enclosed 5280
B E C 5280 metres covered =
= Area of equilateral 264
Let P be the point inside the
1 equilateral  ABC = 20 minutes
 ABC –  area of 1 circle  PD = 3 , PE = 2 3 ,PF = 5 3 75. (b) distance covered = 2 km 26
2 decameters

Rakesh Yadav Readers Publication Pvt. Ltd. 327

For More Visit : www.LearnEngineering.in


For More Visit : www.LearnEngineering.in

= (2 × 1000 + 26 ×10) side of the square


28
(1 decameter = 10 meter) = = 7 cm
4 Perimeter 120
= 2260 m = = = 30 cm
Distance covered in 1 revolution  c ir cumf er ence of each 4 4
circular plate = 2 π r radius of the circle
Total distance
= 22 side 30
Number of revolutions =2×  7 = 44 cm
7 = = = 15 cm
2 2
2260
= = 20 m Circum radius area of the circle
113 80. (c)
in radius
Now, π × diameter = 20 22 22
= × (radius)² = ×(15)²
7 7

r
20  7
diameter =
22

i
84. (b)

.iSn
70 4
= = 6 m Circum radius of equilateral tri-
11 11
76. (a) Distance c ov er ed in 1 side 

agv
revolution angle = =8
3
= circumference of wheel radius of in circle

ridn
22 side = 8 3
 1.75 m side 42 21
= 2× In radius of equilateral triangle = = = cm
7 2 3 2 3 3

eeYa
 Number of revolution area of incircle
side 8 3
11  1000 = = = 4 cm
=
22
= 1000 2 3 2 3 22  21 2 22 2121
2  1.75 81. (b) radius of each circle = 1 cm = 
   = 7  3
7
geisnh 7  3
with all the three centres an
77. (b) Radius of circle equilateral triangle of side 1 cm = 22 × 21 = 462 cm²
circumference 100 is formed .
= = area enclosed by coins = (area 85. (a)
2π 2π
Enak

of equilateral triangle) – 3× r =20 cm


When a square is inscribed in
the circ le , diag onal of the (area of sector of angle 60°)
square is equal to diameter of
3 60
R

the circle = (2)² – 3× × (1)² wheel of radius 20 cm


 Diagonal of square 4 360
no. of revolutions
aryn

100 100 3 1
=2× = = ×4–3× × distance to cover
2π π 4 6 =
circumference of wheel
Les B

 side of square
Diagonal 100 50 2   176007
=
2
=

=  3–
=  
 cm² = = 140
π  2 22220
78. (c) Let outer Radius = R
wa.th

and inner Radius = r 82. (b) 86. (a)


2 π R – 2 π r = 132
2 π (R – r) = 132
132  7
wwM

R–r= 2  22 = 21
M is the centre, BM = CM =r
He nc e, wid th of path = 2 1
metres. AM  BC,(AM = r)
In an equilateral triangle
79. (b) 1 2
. area of  ABC = r × 2r = r²
2 side = (P1 + P2 + P3)
28 cm 3
.
83. (a) 87. (c)
side of square papersheet
= 784 = 28 cm
radius of each circle

Rakesh Yadav Readers Publication Pvt. Ltd. 328

For More Visit : www.LearnEngineering.in


For More Visit : www.LearnEngineering.in

side of equilateral triangle =


100 1 50 2 169 = 13 cm
= 4 3 =   
2 2 2 2 length of perpendicular,
circumradius of triangle
AB×BC
= 25 2 cm² BD =
side 4 3 AC
= = =4
3 3 91. (a)  length of perpendicular to
see the figure hypotenuse to
side of square perpendicular×Base
= 2 × circum radius =
Hypotenuse
=2×4=8
Radius of circle = 6 cm 125 60 8

ir
Area of smallest circle = = = 4 cm
13 13 13

v.iSn
6²  95. (c) Side of equilateral triangle
= = 12
3
Diagonal of square = 8 2 cm 2
Radius of smallest circle =
3
P 1  P2 +P3 

dnag
88. (b) A
12 2
=  2 3 cm = (6 + 8 +10)
 3
x 5 cm A
2 48 3

eYrai
92. (a)
= × 24 = ×
3 3 3
B x C a
Side = 16 3 cm
isosceles right triangle B C area of triangle
 x² + x² = 5² = 25
2x² = 25
25
3
4
snhe
a² = 4 3 =
3
4
(side)² =
3
4 
 16 3
2

x² = a² = 4 × 4
kgei
2 a = 4 cm 3
= × 3× 16 × 16
Area of triangle a 4 4
Circum radius =  = 192 3 cm²
1 1 25
ERna

3 3
= × x² = × 96. (c)
2 2 2 area of circle = r ²
= 6.25 cm²  2 16 a
4 2
= 
 
  3 cm
89. (c) a
aBryn

 3 2a
93. (b)
area of shaded region = area of
r semicircle – area of triangle
2
a 1
Les

= – a2a
2 2
wa.th

2 Circumference – diameter = 30 cm
length of side = (P1 + P2 +P3) 2r – 2r = 30
3 a 2  
2r(– 1) = 30 = – a² = a² 
 – 1
 sq units
2 2 
2 2
= (6 + 7 + 8) = × 21 30 30  7 97. (c) According to question
wwM

3 3 r  =
 22  2  15 (R + 1)² – R² = 22
2
 – 1
 {(R+1)2 – R²} = 22
42 3
42 3 7 
=   = 14 3 cm
3 3 3 227
= 7 cm (R +1 +R)(R +1– R) = =7
90. (c) Area of isosceles triangle 22
A
94. (b) 2R + 1 = 7
1 R = 3 cm
= a² sin( is angle between 12 cm D 13 cm
2 98. (b)
equal sides)
B C
1 5 cm
= (10)² × sin45°
2 AC = 12²5² = 14425 radius of largest circle

Rakesh Yadav Readers Publication Pvt. Ltd. 329

For More Visit : www.LearnEngineering.in


For More Visit : www.LearnEngineering.in

breadth 14 4 a
= = = 7 cm = 2 = 4 cm and Inradius =
2 2 2 2 3
22 A
area =
7
× 7 × 7 = 154 cm² 102. (a)  a 2  a 2
99. (b)

  
 –  
 = 44
 3 2 3 
5
3
 a 2 a ² 2
 3 – 12  = 44
  
B 4 C  
cone is rotated about 3 cm 4a ² – a ² 44  7
 = = 14

r
side 12 22
in-radius of circle (r) = 3a ²

i
2 3

.iSn
 = 14
8 4 12
= =  a² = 56
2 3 3
 a = 2 14

agv
 4 2
area of circle = 
 
 3
 3 The cone so formed after rotat-  area= × 2 14 × 2 14
4

ridn
2216 ing about Side AB.
22 44 = 14 3 cm²
=  = =16.76 So, slant height of cone = 5 cm
7 3 21 106. (c) Side of square = diameter of

eeYa
1 the circle
Required area  Volume of cone = ×
3 r ²h area of circle = r² = 9
3 2 22  16 r = radius
= 8 – h = height  r = 3 cm
4 21  Volume of cone  Side of square = 3 × 2
geisnh
3 1 22 = 6 cm
= × 64 – 16.76 = × × 4× 4 ×3 = 16cm³
4 3 7  Area = 6 × 6
= 16 3 –16.76 103. (c) = 36 cm²
= 27.71 – 16.76 107. (d) The given triangle is a right
Enak

= 10.95 cm² angled triangle


100. (c)  side of the square
R

So, the triangle is right triangle Area of bounded region P×b 86 24
= P+b = 86 =
7
3 1
aryn

  22 – (1)²
30
78
4 2  24 2
 Area of square = 
7  
Les B

  2
 3– 
 cm
 
72  2
576
1 1 104. (a) 2r = 11 = cm²
× 30 × 72 = × altitude × 72 49
2 2 108. (b) Radius of circumcircle
wa.th

11  7 7
altitude = 30 m  r = 22  2 =
4 8
101. (a) = cm
Area of sector 3
2x Radius of incircle
wwM

x 
= × r² 8 4
360 = = cm
x
2 3 3
60 22 7 7  Required area = (R² –r²)
perimeter of triangle = 4 2 + 4 = × × ×
360 7 4 4     2 2 
x + x + 2x = 4 2 + 4 22  8   4  
77 29 =  3  –  3  
2x + 2 x = 4 2 + 4 = =1 cm² 7     
48 48
x 2 2  = 4  2  1 105. (c) Let the side of the triangle 22  64
 –
16 

4 be ‘a’ cm =  
x=
7 3 3 
2 a 22 2
Hypotenuse =  Circumradius = = × 16 = 50 cm²
2x 3 7 7

Rakesh Yadav Readers Publication Pvt. Ltd. 330

For More Visit : www.LearnEngineering.in


For More Visit : www.LearnEngineering.in

115. (b) Area of the shaded portion


109. (c) Side of square = 121 =11 cm
= 6 × 6 – (3)²
Perimeter of square = Circum- 16 2
ference of circle = 44 cm = 36 – 9
 2r = 44 = 9(4 – ) cm2
122. (c) Radius of incircle
44  7
16 2 14 3
 r = 22  2 = 7 cm Other sides = = 16 cm (as
2 = = 7 cm
2 3
 Area = r² the isosceles  ) 22
22  Area = r² = × 7× 7
= × 7 × 7 = 154 cm² 1 7
7  Area = 2 × 16 × 16 = 128 cm² = 154 cm ²

ir
110. (a) 2r + r = 36 123. (d) Radius of incircle
3

v.iSn
 r(2 +) = 36 116. (d) a² = 4 3 6
4 = = 3 cm
22 2 3
 r(2 + ) = 36  a² = 16 Area = r²
7
36  7
 a = 4 cm = 3 cm²

dnag
 r= =7m 117. (c) Side of hexagon 3
36 124. (c) a² = 121 3
1
Side of equilateral triangle 4
111. (c) 2r + r = =
r² 3  a = 22 cm
2
= 2 cm

eYrai
1  3a = 66 cm
 r(2 + ) = 2 r² Area of hexagon
Circumference of circle = 66 cm
 4 + 2 = r 3 3 3 3 2r = 66
= a² = ×4
4 2 2 66  7 21
 r =

+2

 Diameter= 2 
  2

4 
= 6 3 cm²
snhe
118. (a) The radius of park =
176
2
r = 2  22 =

Area = r² =
22
7
×
2
2
21 21
×
2
 
kgei
= 28 m = 346.5 cm²
6
= 6
m  Area of road = (28 + 7)² – 125. (b) Area grazed by the cow
11
112. (a) The angles of the given (28)² = (35 + 28)(35 – 28) 1 1 22
ERna

triangle are 90°,30° and 60° = r² = × ×7×7


22 2 2 7
= × 7 × 63 = 1386 m² = 77 m²
7
119. (c) 26 + 30 + 28
S= = 42
aBryn

A M B 2
12
. o
13 Area of field

= s s–a
 s–bs–c
10
In  OMB MB = 13² – 12² = 5
Les

P = =5 = 42  16  14  12 = 336 m²
2
 AB = 5 × 2 = 10 cm
wa.th

B = 5 3  Remaining area = 336 – 77


120. (b)
1 = 259 m²
 Area = 2 × 5 3 ×5 126. (b)
25 3
wwM

= cm²
2
113. (c) Let the altitude = x cm Area of shaded portion
= 8 × 8 – × 4²
1
 × x × 8 = × 8² = 64 – 16 As P and Q are mid-point and
2 = 16(4 – ) cm² PQ||BC
 64 121. (b)   APQ   ABC
 x=
4 AP PQ 1
 x =16  = =
AB BC 2
114. (d) The sides of the given triangle
are 3,4 and 5 cm BC
 PQ =
1 2
area = × 3 × 4 = 6 cm²  BC = 2PQ = 2 × 5 = 10 cm
2

Rakesh Yadav Readers Publication Pvt. Ltd. 331

For More Visit : www.LearnEngineering.in


For More Visit : www.LearnEngineering.in

127. (c)  by triplet we get sides 8 and 15


11  100  7
r = 5  2  22 = 35 cm  check the sides perimeter
= 8+15+ 17 = 40
135. (b) Length of rubber band
= 3d + 2r = 30 + 10 1
area = × 8 × 15  60
As PQ||BC 136. (c) 2
  APQ   ABC Hence sides are 15,8.
  APQ is also an equilat- smaller side = 8 cm.
140. (b)
eral 

3 25 3 In  OMB

r
  APQ = (5)² = cm²
4 4 OB = 15²8² = 17 cm

i
.iSn
128. (a) 2r = 22
OB = OD = radius
22  7 7 In  OND Area of shaded region
 r = 22  2 = = (4)² – (2)² = (16 – 4) cm²
2

agv
ND = 17² – 8² = 15 cm 141. (c) Let the side of the triangle
22 7 7 77
 r² = × × = CD = 15 × 2 = 30 cm be a
7 2 2 2

ridn
137. (b) Perimeter = 2r +r  Perimeter = 3a
= 38.5 cm²
129. (b) 22 63  3 
= 63 + ×  a ² 3

eeYa
7 2 3a = 4 
= 63 + 99 = 162 cm  
A
3
138. (c) 3= a
4
geisnh a = 4 units
ar (  AOE) = 15 cm² 3
142. (a) Area of  = a²
ar  BDOF = 2 × ar  AOE = 30 cm² B D F E C 4
130. (b) In  AFB 3
= × 36 = 9 3 cm2
Enak

AF  BC 4
AF² = AB² – FB² = 100 – 25 4
143. (b) Area of  = (Areaof 
AF = 5 3 3
R

In  ADF formed by median as side)


ar (  ABE) = ar(  ACD) = 36 cm²
AD² = AF² + DF² 4 1 
aryn

131. (a) The third side will be either =   9  12


 
15 or 22  10 2 3 2 
Les B

 Possible perimeter 5 – 
AD² = 75 +   ( 9,12,15 from triplet)
 3 
= 15 × 2 +22 = 52 4
and 22 × 2 + 15 = 59 = × 54 = 72 cm²
10 7 3
132. (a) No. of revolutions AD =
3 144. (a) 3x + 2y = 6
wa.th

Distance 139. (c) Let sides of triangle are a,b


= x y
Circumference and c respectively  =1
 largest side given =17 cm 2 3
1540100 Perimeter = a + b + c (Make R. H.S. equal to one)
= = 500
wwM

22 98 = 40 cm (given)  Coordinates of 
2 
7 2 area = 60 cm² (given) = (0,3),(2,0),(0,0)
In such questions take the
440 1
help of triplets which form right  Area of  = ×3×2
133. (b) 2r = 2
1000 angle triangle
= 3 square units
227 145. (a) Let each side of the triangle
 r = 50222 = .07
be a units
 Diameter = .14 m 3
 ((a+ 2)²– a²) = 3  3
11000  100 4
134. (b) 2r = So, here we have a side 17 cm 1
5000 (a² + 4 + 4a – a²) = 1 3
4

Rakesh Yadav Readers Publication Pvt. Ltd. 332

For More Visit : www.LearnEngineering.in


For More Visit : www.LearnEngineering.in

1  r² = 169 156. (a) Check tripletes


(4 + 4a) = 1 3 r² = 169 3,4,5
4
r = 13 cm 6,8,10
1 + a = 1 3  Radius of third circle = 13 cm 7,24,25
152. (d) Let the radius of the semi-  7, 24, 25 fulfill the given conditions
a = 3 units circle be = r
 2r + r = 36 1
9  10  11 r(2 +) = 36 area = × 24 × 7 = 84 cm²
146. (c) S = = 15 2
2  22  Perimeter = 7 + 24 + 25 = 56 cm
using hero’s formula 2 
r 
 = 36  Hypotenuse = 25 cm
 7 
Area = s s–a
 s–bs – c  157.(c) Length of median

ir
 36 

r  = 36
 3

v.iSn
= 15  6 5 4 7 = a= 6 3
2
r=7m
= 1800 = 30 2 cm² a = 12 cm
147. (c) Let the length of each equal  72  Perimeter = 12 × 3 = 36 cm
 Area =

dnag
side be a unit 2 158. (a) Area of equilateral 
2 22  7  7
 4a ² – 4 = 4 3
4 = 72 = 77 m² a² = 4 3
4

eYrai
4a ² – 4 = 8 153. (b) side of square= area a² = 16
4a² – 4 = 64 a=4
a² – 1 = 16 = 2 m = Diameter of circle  Perimeter = 4 × 3 = 12 cm
a² = 17  Radius of circle 159. (a) Distance covered by small gear
a = 17 units
148. (a) Sum of other two sides
=
2
2
=
1
2
snhe
m
= 2r × 42

= 84×
12
2
= 504
(a + b) = 32 – 11 = 21
kgei
2
and a – b = 5 22  1   No. of revolution by big gear
 Area = × 
  = m² 504
215 7  2 2
 a = = 13 cm 2 9 = 28
ERna

2
154. (d) 4 6 160. (b) Perimeter of semi-circle
21 – 5 = 2r + r = r(2 +)
b= = 8 cm 4 6
2  r(2 +) = 18
aBryn

Sides of the  = 11,8,13 cm 8 8 18 187


r = =
13  8 11 22 36
S= = 16 2
2 7
Side of  ABC = 10, 14, 12
 area 7 1
= cm = 3 cm
Les

10  14  12 2 2
= 16 16 – 13 S  18
 16 – 816 – 11 2
wa.th

161. (d) Perimeter of circle = 2r


= 16  3  8  5 = 8 30 cm² = 2(18 + 26) = 88 cm
area = s s  a  . s  bs  c  r = 44 cm
3 r = 14 cm
149. (b) Area of  = a²  18  8  4 6  24 6cm 2
4 22
wwM

 Area of circle = × 14 × 14
155. (b) 7
3 = 616 cm²
= × (2)² = 3 cm²
4 162. (a) Area of a circle = 38.5 cm²
150. (a) Let the original radius = r cm π r² = 38.5
 ((r+ 1)² – r²) = 22 38.5  7
r² =
22  7 Area (  ABDE) = 3 × ar(  AEC) 22
r² + 1 + 2r – r² = =7 7
22
3 r= cm
2r + 1 = 7 =3× (2)² (ADC is equilateral 2
r = 3 cm 4 Circumference of a circle = 2 π r
151. (a) Area of two circles triangle) 22 7
= (5² + 12²) = 169 cm² = 3 3 square units = 2  = 22cm
7 2

Rakesh Yadav Readers Publication Pvt. Ltd. 333

For More Visit : www.LearnEngineering.in


For More Visit : www.LearnEngineering.in

163. (c) Diameter of circle Note: In an equilateral triangle, =51 × 73984


Diagonal 12 2 length of median, angle bisec-
= = = 12cm = 51 × 272
2 2 3 = 13872 cm²
tor, altitude is equal to 170. (d)
12 2
Radius of circle = = 6 cm
2 sides
Rad ius of c ircumcir cle of 167. (c) clearly, 12 cm , 16 cm and
a 20 cm from a triplet
equilateral  =
3
 a = Radius × 3 = 6 3 cm Let AB = AC = a cm
164. (c) BD = DC = b cm

r
 Altitude of isosceles triangle
is also median

i
.iSn
They from a right triangle,
1 In right ADC
area of triangle = × 16 × 12 AD² = a² – b²
2
= 96 cm² 64 = a² – b² .........(i)

agv
Perimeter = 64
8 ² 360 36 a + a + 2b = 64
2a+ 2b = 64

ridn
168. (c) = =
x ² 250 25 a + b = 32 .........(ii)
3x + 4y = 12 a ² – b ² 64
8 36 6

eeYa
On dividing  =2
3x 4y   a b 32
 =1 x 25 5
12 12 [a² – b² = (a + b) (a–b)]
40 20 2 a–b =2
 Divide by 12 on both sides x=   6 cm  a + b = 32
make R.H.S = 1 6 3 3
On solving a = 17 , b = 15
geisnh
Note: The ratio of areas of two simi-
x y lar triangles is equal to the ra- 1
 =1 area of  ABC = × AD × BC
4 3 tio of square of their corre- 2
 Coordinates of point A = (0,3) sponding sides 1
169. (d) = × 8 × 30 = 120 cm²
Enak

point B = (4,0) 2
1 171. (a)
area of  OAB = ×4×3
2
R

= 6 sq units
165. (c) Height of equilateral  = 15 cm 5
aryn

AB = AC = BC
3 6
(side) = 15 AB + BC + AC = 544 x = AB = b + c
Les B

2
y = BC = a + b
152 5 5
side = BC + BC + BC = 544 z = AC = a + c
3 6 6
 semi-perimeter(s)
3 5BC 6BC 5BC AB BC  AC 2a 2b 2c
wa.th

area = (side)² = 544 = 


4 6 2 2
2 16BC = a+b+c
3 15  2  3 225  4 = 544
= 
 
   6 area of
4  3  4 3
5446  ABC  s s – x s – y s – z 
wwM

= 75 3 cm² BC = = 204
16 =a + b + c abc
3 5
166. (b) side ²  9 3  AB = AC = 204 = 170 cm 172. (d)  R ²  10 ² 24 ²
4 6
b R² = 10² + 24² = 100 + 576
(side)² = 9 × 4 = 36 Area of  ABC = 4a ² – b ²
4 R= 676 = 26 cm
side = 36 = 6 cm  where a = equal side 173. (c)
3 b = base
length of median =
2
side 204
=
4
  ² – 204 ²
4 170
3
= × 6 = 3 3 cm = 51 11560 – 41616
2

Rakesh Yadav Readers Publication Pvt. Ltd. 334

For More Visit : www.LearnEngineering.in


For More Visit : www.LearnEngineering.in

Let the side of an equilateral ar ABC 


= 12 × 5 = 60 cm²
AB ²
triangle = 'a' 176. (b) 
ar DEF  DE ² 179. (a)
and the side of square = 'b'
In-circle radius of equilateral 20 25

a 45 DE ²
 = 2 3
4525 225
 Diagonal of square DE² = = Let radius of outer circle = R
20 4
a a and radius of inner circle = r
= 2= 225
2 3 3 DE = ATQ 2R – 2r = 66
Now, 4
2(R – r) = 66

ir
Diagonal a a
b=   15

v.iSn
2 3 6 = = 7.5 cm 66 66  7 21
2 R–r= = =
2 2 2  22 2
177. (c)
Required ratio width = 10.5 m
3 180. (d) Distance covered in 30 sec-

dnag

4 3 6 onds
= 2
 a² 
a  4 a²
  30
  = 30 m/min× = 15 m
 6 60

eYrai
3 3 This is the difference of dis-
=  3 3 :2
2 tance of the boundary and the
AB = AC tangents drawn from
174. (c) Let the side of equilateral diameter
the same point equal
triangle = s Let 'R' be the radius

area of equilateral  =
3
4

OA = 12 cm snhe
OB = OC = 3 cm

ABO  ACO = 90°


kgei
In Right ABO

AB  12² – 3² = 135
3
2R – 2R = 15
ERna

2 =15  9 = 3 15
ar (ABOC) = 2× ar(ABO) 2R  – 1 = 15
height of equilateral triangle =
 
1
= 2× × AB × OB 15
3 2
aBryn

s 15 22 15  7
2 2R = = – 1 = 15 = 7
= 3 15 × 3 –1 7
2
 3  = 9 15 cm² 7
 s 3 R= = 3.5 m
s² 178. (b) 2
b²  2  3
   4
Les

  = 3 181. (a) Perimeter of the circle = cir-


a 3 3 3
s² s² cumference of circle
wa.th

4 4 Let 'R' be the radius


ATQ 2R – 2R = X
175. (c)
2R  – 1 = X
wwM

X
OQP  ORP = 90° 2R = –1
 ABC   DEF (radius is  tangent)
and PQ = PR (tangent drawn X
ar ABC  3² Diameter= π – 1
 from same point are equal)
ar DEF  4²  2R = diameter of the circle
PQ = OP ² – OQ ²  13² – 5²
54 9 = 12 182. (b)
 a
ar DEF  16 ar (PQOR) = 2 × ar(PQO) 3

1654 1
ar DEF  = =96 cm² =2× × PQ × OQ
9 2

Rakesh Yadav Readers Publication Pvt. Ltd. 335

For More Visit : www.LearnEngineering.in


For More Visit : www.LearnEngineering.in

Let the side of an equilateral AB = 30 cm Incircle radius a


triangle = 'a' OM  AB and OM = 8  = 2
Circumcircle radius
a  AM = BM = 15 cm a 2
 Circumcircle radius = 2
3 In Right OMA
 a 2 OA² = OM² + AM² 1
area of circumcircle =  
 OA² = 15² + 8² = = 1: 2
2
 3
OA² = 289 189. (d)
a ²
= 3 OA = 28 9
3 OA = 17 cm
a² = 9, a=3
Radius of circle = 17 cm
perimeter = 3 × a = 3 × 3 = 9 cm
187. (c)

r
183. (a)

i
.iSn
r 72° r
Hence
ABC is an equilateral triangle
AB = BC = AC = '2a' cm

agv
88
3
 Since, 9,12,15 forms a triplet area of  ABC  2a  ²
4

ridn
Length of arc = 2r
360 1
area of  A B C = × 9 × 12 3
72 22 2 =  4a ² = 3 a²

eeYa
2 r = 88 4
360 7 = 54 cm²
area of 3 sectors ( = 60°)
In-circle radius of triangle
887360
r= = 70 m area of triangle 60 a ²
72222 = =3× × a ² =
360 2
geisnhsemiperimeter of triangle
184. (a) area of shaded region = area of
54 54  2
=  = 3 cm  AB C – area of 3 sector
9  15  12 36
2 a ²
= 3a ² –
Enak

Alternate: 2
In a right triangle, with, P, B  2 3 – 
 
In ABC and H incircle radius =  2  a² cm²
 
R

perimeter of  ABC = (AB + BC


P B – H 190. (c) A
+ AC) = 2(3.5 + 4.5 + 5.5) =
2
= (13.5)×2 = 27
aryn

185. (c) 912 – 15 A'


Hence, r =
2
Les B

6
= = 3 cm B B' C
2
In  ABC and  A'B'C
Also Circumcircle radius
 15,20,25 form a triplet  =  C (common)
C
wa.th

H 15
Clearly, 25² = 15² + 20² =  = 7.5 cm  B' =  B ( AB||A'B')
ABC is a right triangle 2 2
  ABC   A'B'C
Area of Right ABC
188. (a) 2
area of A'B'C  B'C 
wwM

1
 AB  BC
 area of ABC =  BC 
=
2
2
1 1 1
=   =
= 1520 = 150 2 4
2
Cost of sowing seeds Let the side of square = a 1
= 150 × ` 5 = ` 750  ar  A'B'C = (area  ABC)
a 4
In circle radius of square = 191. (b) Perimeter of square = 44 cm
2
186. (a) 2
o
Circumcircle radius of square  44 
r Area of square =  
8 Diagonal a 2  4 
A B
15 M15 = = = 121 cm²
2 2

Rakesh Yadav Readers Publication Pvt. Ltd. 336

For More Visit : www.LearnEngineering.in


For More Visit : www.LearnEngineering.in

Circumference of circle 194. (a) Now, In  ADC,


= 2  r = 44 cm
24 cm 24 cm 252540
22  7 . s = = 45 m
r= = 7 cm 2
22
22 area of  ADC
 area of circle =  r² =
× (7)² 24 cm
= s(s–a)(s–b)(s–c)
7 Area of equilateral triangle
= 154 cm²
= 45(4 5 – 25)(4 5 – 25 )(45 – 40 )
Required difference 3 3
= 154 – 121= 33 cm² =  side  ² =  24  24
4 4 = 45×20×20×5
192. (d) C = = 20 × 3 × 5
3 × 6 × 24

ir
= 300 m²
= 144 3 cm² Area of the plot

v.iSn
A B = 384 + 300 = 684 m²
= 144 × 1.732
196. (c)
= 249.408 cm² A
Inradius of an equilateral

dnag
 AC B = 90° (angle in H
semi-circle)  side  a M
triangle =   P
AC : BC = 3 : 4 2 3 

eYrai
AB² = B C
AC 2 +BC 2 = 3²4² 24 b
= = 4 3 cm Length of perpendicular drawn
= 5 units 2 3 f rom the right angle
5 units = 5 cm Now, Area of incircle a×b
hypotenuse, P =
H
 ar  ABC = 2 × 3× 4

= 6 cm²
1 =
22
7
22
snhe
  Inradius  ²
P² =
a² b²

a² b²
= 4 3 4 3
kgei
193. (a) 7 P² =
A a² + b²
22  16  3 1056 ( H² = a² + b²)
= = 197. (a)
7 7
ERna

6
N = 150.86 cm²
Area of remaining part = area
.
A 8 4
.B
M B of Δ – area of incircle
C
aBryn

8 = 249.408 – 150.86
Diameter of the circle AB
1 = 98.548 cm² = 8 + 4 = 12 units
Ar(  ABC)= × 6 × 8 = 24 cm² 195. (d) A
2 25
12
D Radius = = 6 units
 ABC   MCN 2
Les

 C = C 32 25  Area of circle = π r² = π ×(6)²


wa.th

M = B ( MN||AB) = 36 π sq. units


3
ar (CMN)  CM 
2 B 24 C 198. (a) side  = 3 side  ²
 ar ( ABC) =  BC  2 4
 ABC = 90º
side = 2 units
wwM

AC = AB² + BC² 199. (a) Let the length of side of


2
1 1 = 32² + 24² square = a
=   = then the diameter of circle = d
 2 4
= 1024576 According to question,
ar (  MNAB) a= d
= 1 600 = 40 m
= ar(  ABC) – ar(  CMN)
Now, area of  ABC area of square a²
=4–1=3  =
area of circle  d² 
1 π 
= × AB × BC  4
24 2
 ar (  MNAB) = ×3
4 a² × 4 a² × 4
1
= 18 cm² = × 32 × 24 = 384cm² = =
2 π d² π a²

Rakesh Yadav Readers Publication Pvt. Ltd. 337

For More Visit : www.LearnEngineering.in


For More Visit : www.LearnEngineering.in

4 4×7 14 7 side
 54 = 21 metres
== = 18 3 = 2 3
π 22 11
15 18  21 side = 6 cm
 14 : 11 S= = 27
200. (d) Length of median of an 2 perimeter of equilateral triangle
 Area of  = 3 × 6 = 18 cm
3
equilateral triangle = side  = s (s – a)(s – b)(s – c) 209. (b) Circumference of circle = π d
2
Length of median, altitude,  π d – d = 150
= 27  12  9  6 d( π – 1) = 150
and angle bisector is
= 54 6 m²  22 
3 d – 1 = 150
= side  205. (a) circumference of circle  7 
2

r
= π × diameter 15
3 d× = 150

i
.iSn
a = 6 3 22 7
 =  112 = 352 cm
2 7
150 7
6 3 2  perimeter of rectangle = 352 d= = 70
15
a= = 12 cm 2 (l + b) = 352

agv
3 d 70
352 Radius = =
3 l+b= = 176 2 2
 area of  ABC = a² 2

ridn
4 = 35m
7
210. (b) Let radius of circle = r
3  176 Side of square = a
= ×12×12  smaller side =

eeYa
4 16
Side of equilateral Δ = b
= 3  3  12 = 36 3 cm² = 77 cm
According to question,
206. (c) p er im eter of equilate ral
201. (b) π r² = 2 π r 2 π R = 4a = 3b
triangle = 18 cm
r = 2 units
geisnh 3 × side = 18 cm πR 2
 Area of circle = π (2)²  a= 2
b= πR
18 3
= 4 π sq. units side = = 6cm Ratio of their areas:
202. (d) Area of equilateral triangle 3
3
3 3 π R² : a² : b²
Enak

4
= side  ² length of median = side
4 2 2 2
π R  3 2 
3 π R² :  2  :  π R
 side  ² = 48 3   4 3 
=  6 = 3 3 cm
R

4 2
48  4 π 3
207. (a) 1 : : π
(side)² = = 64 3 4 9
3
aryn

1
C : S : T
 1 2
 1 . r. . .r . Here, we can see that C > S > T
Les B

side =  64 (3)2  = 8 (3)4 Quic ke r Ap pr oach : Whe n


  p er im eter of two or m or e
203. (b) figure s ar e same the n the
figure who has more vertex is
.r . Circumference of paper sheet
greater in the area. Since,
wa.th

= 352
2 π R = 352
circle has infinite vertex.
. R
. 352 352  7
Therefore, C>S>T
R= = 211. (d) Distance c ov er ed in 1
2πR – 2πr = 33 2 π 2  22
revolution = Circumference of
wwM

33 33 7 = 56 cm circular field = 2 π r
(R – r) = = 2 22
2π R 56 Distance = speed × time
r= = = 28 cm
3 7 21 2 2 5
= 2 2 =  circumference of circular plate = 66 m/s × s = 165 m
4 2
= 2πr
thickness = 5.25 m  2 π r = 165
204. (b) Ratio = 5 : 6 : 7 22
= 2  28 22
sum of sides = perimeter 7 2  r = 165
= 176 cm 7
= 18
sides, 208. (a) Inradius of equilate ral 165  7
side r= 2  22
5 6
54 = 15 54 = 18 triangle =
18 18 2 3 = 26.25 m.

Rakesh Yadav Readers Publication Pvt. Ltd. 338

For More Visit : www.LearnEngineering.in


For More Visit : www.LearnEngineering.in

212. (c ) Circ um fe re nc e of f ront r = 1cm From (i) ÷ (ii)


wheel × no. of its revolutions = A
 r² A
circumference of rear wheel × =
no. of its revolutions 2 r C
3
2 π x × n = 2 π y × m (let ‘m’ is
the revolution of rear wheel)
. 5

C1
r
=
A
C2 2 C
B C
nx 4
rc = 2A
m=  Circum-radius of circle C2
y 218. (d)
A
213. (b) Distance to be covered in Hypotenuse
R=
one revolution = 2
Circumference of wheel 3x 5x

ir
= π × diameter In a right angled

v.iSn
triangle half of
22 hypotenuse is circum B 4x C
=  56 = 176 cm Area of right angled triangle =
7 radius
7776
Total distance = 2.2 km
5

dnag
= (2.2 ×1000 × 100) cm R= = 2.5 cm 1
2  × 4x  3x  7776
= 22,0000 cm 2
 Number of revolutions Area of C1 πr2
 =  6 x² = 7776
220000 Area of C2 πR2

eYrai
 x² = 1296
= = 1250
176 12 4  x = 36
A = 2 =
214. (b) 5 25
2  Perimeter of triangle
 
= 3x + 4x + 5x = 12 x
a

. G
a Pool = R snhe
216. (d) Let the radius of Swimming
= 12 × 36 = 432 cm
219.(b)
l
kgei
r

B a C R
90º
We know that in an equilateral R+4 O r
ERna

triangle a median also be a al- According to the figure,


titude  Perimeter = r + r + l
Altitude of an equilateral tri- Outer radius of Pool with con- 75 cm = 2r + length of arc

crete wall = (R + 4)
2r
3 According to question 75 cm = 2r +
aBryn

angle = a 4
2 11
R 2 =  (R + 4)2 –  R2 22  r
25 75 cm = 2r + 7  2
3
 a = 12 3 (given) 11
2 R2 × = R2 + 16 + 8R – R2 r = 21 cm.
25  Its area
Les

 a = 24 cm 11
 Then area of an equilateral tri- 1  22 
wa.th

2
25 R = 16 + 8R =   21  21
3 2 4 7 
angle = a 11R2 – 200R – 400 = 0 = 346.5 cm2
4 By option (d), (In such type of
220. (b) A
3 equation go through the option
=  24  24
wwM

4 to save your valuable time)


8 8
= 144 3 cm² R = 20
O
215. (d) Let a triangle ABC has sides 11 × (20)2 – 200 × 20 – 400 = 0
of measurement 3 cm, 4cm and 4400 – 4000 – 400 = 0 B C
D
5 cm 0 = 0 (satisfy) 8
using triplets (3, 4, 5) Radius of pool R = 20 cm According to the question,
217. (a) Area of circle = A Here OD = radius,
  ABC will be a right angled tri-
angle Radius of circle = r a 8
Circumference of circle = c r = =
 Inner radius of circle C1 2 3 2 3
 r² = A (i) 4
AB + BC – CA r 
= = 43–5 3
2 2 2 r = c (ii)

Rakesh Yadav Readers Publication Pvt. Ltd. 339

For More Visit : www.LearnEngineering.in


For More Visit : www.LearnEngineering.in

Required area of shaded portion 224. (c) According to the questions, 229. (b) According to the question,
3 2  4 2 A
 8 
 

4  3
75º
r = 84cm 1
3 16
  64  
4 3 15º
B C
22 16
 3  16  
7 3 P AB
Sin15º = =
= 10.95 m2 = 11 m2 H 1
221. (d) AB = sin15º

r
D
4 B BC
=

i
.iSn
A
Cos15º = =
H 1
O 45° a=? BC = Cos15º
B 1
Let the length of side of the
3 Area of ABC = × AB × BC

agv
E square be a cm 2
According to the question, (circumference of circle = perim- 1
eter square) = × sin15º cos15º
2

ridn

Area of sector OED = r2 × 2  r = 4a 1
360
= × sin2 × 15
22 4

eeYa
45 2×  84 = 4a
= ×4×4× = 2m2 7 [ sin2  = 2sin  cos  ]
360
Area of the sector OAB 132 cm = a 1
= × sin30º
225. (a) Area of circle = 324  cm2 4
  r2 = 324  1 1 1
= r2×
geisnh
360 r = 18cm = × = m²
4 2 8
45 Longest chord = diameter = 2r 1
= × 3 × 3 × = 2×18 = 36 cm = × 100 × 100
360 8
226. (c) Circumference of a  = 24 cm = 1250 cm²
Enak

9
= m2 a + b + c = 24 cm 230. (b) According to the question,
8
So, Area of shaded portion = Area a+b+c A
or S = = 12 cm
2
R

of OED – Area of OAB


9 16 – 9 Circumference of incircle a a
= 2– = 2  r (inner) = 44 cm
aryn

8 8
r (inner) = 7 cm
B C
7 7 22 11 2 Area of  = S × r (inner) b
Les B

=  =  = m
8 8 7 4 = 12 × 7 = 84 cm² Let AB = AC = a
222. (d) According to the question, 1 BC = b.
Circumference of a circle = 2r 227. (b) Area of  = ab sin θ
2 a a b
30  S=
wa.th

2r = 1 2
 = × 10 × 10 × sin45º
2 b
15 S = a+
r 2 2
 = 25 2 cm²
wwM

30 228. (a) According to the question Area = S(S–a)(S–b)(S–c)


D  2r  2
  Area =
223. (b) According to the question r=
S  b  b  b  b 
 a+ 2   a+ 2 –a   a+ 2 –a   a+ 2 –b 
    
50
semiperimeter =  25
2  b  b  b  b
Area =  a+ 2   2   2   a – 2 
R = 364 cm inner radius     
r = 350cm
Area
= Semi-perimeter b b²
Area = a²–
2 4
Area
6=
The breadth of the path = (R – r) 25 b
Area = 150 cm² Area = 4a²–b² sq. units.
= (364– 350)cm = 14 cm 4

Rakesh Yadav Readers Publication Pvt. Ltd. 340

For More Visit : www.LearnEngineering.in


For More Visit : www.LearnEngineering.in

231. (c) As we know circumcentre Then,


always made by the intersec- 20 = 256d 2 2
2 2
tion of half altitude Perimeter = 2 d d2 1
256 + d22 = 400
 In obtuse angle it will always be = 2 24²10² d22 = 400 – 256 = 144
out.
d2 = 144 = 12 cm
A
= 2 676
239. (b) Diagonal (d1) = 10 cm
= 2  26 = 52 cm
C area of Rhombus = 150 cm²
B 236. (c) 4 × side = 40 cm(given)
1
40 ×d1 × d2 = 150
2
 side = = 10 cm
D  Circum center
4 1
232. (a) According to the question,

ir
10 cm × 10 × d2 = 150
D C 2
2r  circumference

v.iSn
cm 150 2
2r  Diameter 6
10 cm 10 cm d2 =
10
2r 22 cm O
6 = 30 cm
 2r =
7 A B

dnag
10 cm 240. (a)
4 In  AOB,
1 22
 7 =
7 OB = 10² – 6²
2r

eYrai
11 = 100 – 36
22
 = A regular hexagon consists of 6
7  2r 7 = 64 = 8cm equilateral triangle ar ea of
1 2 Diagonal BD = 8 × 2 = 16 cm. regular hexagon
 = 237. (b)

 R =
2r
1
4
m
1 A
snhe D

5m
= 6
3
4
3
(side)²

= 6 a²
kgei
233. (b) Given: B C 4
 Area of square = 4 4 × side of rhombus = 40 m
3 2
side of rhombus = 10 m = 6 × 2 3
side² = 4
Sinc e rhom bus is als o a 4  
ERna

side = 2
parallelog ram ther ef or e its 3
 Diagonal of square = radius of = 6  12 = 18 3 cm²
area = base × height 4
circle
= 10 × 5 = 50 m² 241. (a) area of hexagon
2 side = r 238. (d) D C
aBryn

8
3
=6× (side)²
 r= 2 2 10 4
8 O
 Area of circle =  r² 3
A 10 B =6× (1)²
2 4
  ×  2 2  = 8  cm² Perimeter of Rhombus = 40 cm
3 3 3
Les

234. (a) We know that rhombus is 4 × side = 40 =6× = cm²


side = 10 cm 4 2
wa.th

parallelogram whose all four sides 242. (a)


are equal and its diagonals bisect We k now that d iagonals of A D
each other at 90º. rhombus bisect each other at
right angle,
A B
12 16 Therefore In right  OAB 10
wwM

OB² = AB² – OA²


16 12
=10² – 8² = 100 – 64 = 36 B 6.5 C
D C (  Rhombus is a ||gm
OB = 36 = 6 cm  area of Rhombus = base ×
 AB = 16²  12² Diagonal BD= 2 × OB = 2 × 6 height )
= 12 cm area of Rhombus
= 400 = 20cm Alternative = base × height
= side of rhombus = 6.5 × 10 = 65 cm²
1 2
 perimeter of the rhombus = 20 Side of rhombus = d1  d22 Also area of Rhombus
2
× 4 = 80cm
1 1
235. (d) If d1 and d2 are the lengths of 10 = 162 d 2 = ×d1 × d2
2 2
diagonals of rhombus.

Rakesh Yadav Readers Publication Pvt. Ltd. 341

For More Visit : www.LearnEngineering.in


For More Visit : www.LearnEngineering.in

1
Area of parallelogram =AB×AE = AE × DC = CF × AD
× 26 ×d2 = 65 5x × 20= 1000 AE × 36 = 12 × 27
2
x = 10 = AE = 9 cm
13 × d2 = 65 Area of parallelogram =AD × AF  Distance between bigger sides
d2 = 5cm 1000 = 4x ×AF = 9 cm
243. (a) A B
1000 = 4 × 10 × AF 252. (a) In a rhombus
AF = 25 cm
4a ²  d12 +d2 2
(smaller side altitude)
248. (b) 4a² = 8² + 6²
60°
D C 100
a² = = 25
In the above figure  ADC is an 4

r
equilateral triangle (as AC is a =5

i
.iSn
angle bisector) as  ABCD is a rhombus  Side of square = 5 cm
 AC = 10 cm (smaller diagonal)   ABC is an equilateral   Area of square = 25 cm²
244. (c) Side of rhombus
3 253. (b)

agv
100  ar (  ABC) = × 10 × 10
= = 25 cm 4
4
= 25 3 cm²

ridn
we know that in a rhombus 4a²
= d1² + d2²  ar (  ABCD) = 25 3 × 2 Area (  ABDE)= 3 × ar(  ADC)
 d2² = 4×(25)² – (14)² = 2500 –

eeYa
= 50 3 cm² (ADC is an equilateral triangle)
196 = 2304
249. (b)
3 2
 d2 = 2304 = 48 cm = 3 2 = 3 3 unit2
4
1
 Area = × d1 × d2
geisnh 20
2 254. (d) side of rhombus = = 5 cm
Area of parallelogram 4
1
= × 14 × 48 = 336 cm² = AD × FC = 15 × 12 A
2 D
245. (d) Let the parallel sides be 3x = 180 cm²
o
Enak

and 2x Area of parallelogram 4


AC = 8 cm
= DC × AE = 180
1 18 × AE = 180 B C
 (3x + 2x)× 15 = 450 5
R

2 AE = 10 cm
OC = 4 cm
 5x = 60  Distance between bigger sides
= 10 cm In Right  OBC
aryn

x = 12 OB² = BC² – OC²


250. (a)
 Sum of length of parallel sides = 5² – 4² = 9
Les B

= (3 + 2)×12 = 60 cm
246. (c) OB = 9 = 3 cm
BD = 2 × OB= 2 × 3 = 6 cm
AB = 24 cm area of Rhombus
AD = 16 cm
wa.th

1 1
Using Hero’s formula AE = 10 cm (Given) = × AC × BD = × 8 × 6 = 24 cm²
2 2
Area of Parallelogram
15720 Note: In the question do not get con-
S= = 21 cm = AE × DC = 10 × 24
2 fused with the wor ds
= 240 cm²
wwM

non–square its simply to clear that


Area of  ABC also, area of Parallelogram it is Rhombus
= FC × AD = 240 255. (c) A
= 21 21 – 20
 21 – 721 – 15 FC × 16 = 240 D
o
FC = 15
= 21114  6 = 42 cm² Distance between AD and BC

 Area of  ABCD = 15 cm B C
100
= 42 × 2 = 84 cm² 251. (d) side of Rhombus = = 25 cm
4
247. (b) 5
BD= 40 cm
OB = 20 cm
20
4 In right  OBC
Area of parallelogram OC² = BC² – OB²

Rakesh Yadav Readers Publication Pvt. Ltd. 342

For More Visit : www.LearnEngineering.in


For More Visit : www.LearnEngineering.in

OC = 25x² = 6400 Alternate:-


25² – 20² = 15 cm
 AC = 2 × OC 6400
x² =
= 2 × 15 = 30 cm 25
1 6400 80
Area = × BD × AC x= = = 16
2 25 5 In this question
1  AC = 4x = 4 × 16 = 64
= × 40 × 30 3
2 BD = 3x = 3 × 16 = 48 ar ( APQ ) =
8
A BC D 
= 600 cm²
256. (d) 1
area = × AC × BD ar (APQ ) 3
2 
ar (ABC ) 8

ir
1  ar ( ABC ) = 12

v.iSn
= × 64 × 48
2  ABCD = 2 × 12 = 24
= 1536 cm² 3
ar (  APQ) =  24 = 9 cm²
 DE || AC 259. (a) A D 8

dnag
261. (b)
  BDE   BAC

ar (BDE ) 2² 4 B C Q
 
ar (BAC ) 5² 25

eYrai
In ΔABC & ΔDCQ
ar(trap.ACED) = ar(BAC) –
ABC =DCQ
ar(  BDE) = 25 – 4 = 21
ACB =DQC
d1 = 24 cm

257. (c)
ar ACED
  21
ar BDE
=
4
= 21 : 4
BC = CQ
ABC  DCQ
snhe
ar (ABC )  ar (DCQ )
area of Rhombus = 216
1
2
× d1×d2 = 216
kgei
O 1
260. (c) 24d2 = 216
2
ERna

 AB = 2 CD 216 2
d2 = = 18 cm
AB 2 24
 area of ABCD = 24
CD 1 1 1
Draw QM and PN and intersect OA =  d1   24 = 12 cm
aBryn

2
them at O 2 2
ar AOB
   2 4
 = 4:1  Diagonals of Rhombus bisect
ar COD 1  1 1 each other at right angle
ar( POQC ) = × 24 = 6
4
(  AOB   COD ) 1 1
1 OD =  d2   18 = 9 cm
2 2
Les

258. (d) D 40
C  area (  PQC) = × 6 = 3
2
Now,
wa.th

o 40 2
3x 40 ar (  PQC)= 3
40 2 40 In Right AOD
40 1 AD² = AO² + OD²
A 40 B ar (QMAD) = × 24 = 12
2 = 12² + 9² = 144 + 81 = 225
Let AC = 4x and BD = 3x
wwM

1 AD = 225 = 15 cm
3x ar (  QAD) = × 12 = 6
 OA = 2x and OB = 2  Perimeter of Rhombus
2
ar (  ABP) = 6 = 4 × AD = 4 × 15 = 60 cm
In Right  OAB 262. (a)
ar(  PQC) + ar(  QAD)
2 +ar(  ABP) = 15
 3x 
2x ²   2  = 40 ar(  APQ) = 24 – 15 = 9 cm²
also

9x ² ar APQ 9 3
4x² + = 40² = 1600 ar ABCD = 24 = 8
4 Let ABC = 60°
16x² + 9x² = 1600 × 4  always it will be 3 : 8 OBC = 30°

Rakesh Yadav Readers Publication Pvt. Ltd. 343

For More Visit : www.LearnEngineering.in


For More Visit : www.LearnEngineering.in

 Diagonals of Rhombus are the 1


268. (b)
ar ABCD  2 AD  BC   EN
angle bisectors A
 2
In right BOC ar AED  1
 EN  AD 5 D
OB 2 E
= Cos 30° 3
BC AD BC
= B C
OB 3 AD DE  BC
 
8 2 266. (c)
A D   ADE =  ABC and
OB = 4 3 O
 AED =  ACB
 BD = 2 × OB b a
  ADE   ABC

r
= 2 × 4 3 = 8 3 cm B
P
C
2 ar  ADE   2 ² 4

i
263. (c)

.iSn
side of Rhombus ar  ABC  =  5 ² = 25
perimeter 2P P  area (  DECB) = area (  ABC)
= = =
4 4 2 – area (  ADE)

agv
Let, AC = 2a = 25 – 4 = 21
AC = 16, BD = 12 cm  OA = OC = a (ar DECB) 21

ridn
BD = 2b  =
 OA = 8 cm, OB = 6 cm (ar ABC) 25
OB = OD = b
 Diagonals of rhombus bisect 269. (a)

eeYa
A B
each other at 90° In Right  OBC,
In Right OAB P²
a² + b² = O
AB² = OA² + OB² 4
= 8² + 6² = 100 4a² + 4b² = P² .....(i)
geisnh D C
AB = 100 = 10 cm Also, 2a + 2b = m
AB = BC = CD = DA = 10 cm
on squaring,
60
BD = 16 cm
264. (b) A B 4a² + 4b² + 8ab = m²
4a² + 4b² = m² – 8ab ......(ii) In  ODC,
Enak

40 80 40 from (i) and (ii) OD = 8, CD = 10,  DOC = 90º


m² – 8ab = P²  OC
D C 8ab = m² – P² =
60 CD² – OD² = 10² – 8²
R

4× (2ab) = m² – P²
= 6 cm
608040 1
S(  ABD) = = 90 2ab = (m² – P²)  AC = 2 × OC = 2 × 6 = 12 cm
aryn

2 4 Now, Area of Rhombus ABCD


area of Rhombus
ar(  ABD) 1
Les B

1 1  d1 × d2
= 9 0 9 0 – 8 0  9 0 – 6 0   9 0 – 4 0  =  d1 × d 2 =  2a  2b =
2 2 2
= 90  10  30  50 1 1
= 2ab= (m²–P²)  16 × 12 = 96 cm²
4 =
= 300 15 m² 267. (c) 2
wa.th

270. (a) Area of trapezium


ar (  ABCD) = 2 × ar(  ABD)
60º 60º 1
= 600 15 m² =  sum of parallel sides   height
60º 60º 2
B E C
265. (d)
wwM

60º 60º
1 1
= 6  8   4 =  14  4
2 2
Re gular he xagon has 6 = 28 cm²
equilateral triangle A D
A N D 271. (a)
 Ar ea of Re gular hexag on = 12
Let EN  AD
6×area of equilateral triangle O 16
1 12
area of AED  × EN × AD 3 3 3
a² 16
2 =6× a² =
4 2 B C
area of trapezium ABCD
9 AC = 24, BD = 32
1 = a²
= AD  BC   EN 2 3  OB = OD = 16 and
2 OA = OC = 12

Rakesh Yadav Readers Publication Pvt. Ltd. 344

For More Visit : www.LearnEngineering.in


For More Visit : www.LearnEngineering.in

(Diagonals of Rhombus bisect 1


= Rs 100 ...(ii)
each other at 90º   18  24 = 216 sq cm
2 breadth 120 6
In  OBC, 
274. (d) Let ABCD is a || gm breadth – 4 100 = 5
BC² = OB² + OC² = 16² + 12² = 400
area of  ABCD = 2 ×area of  breadth = 24m
BC = 400 = 20 cm ADC 278. (b) Area of corridor = 100 × 3
perimeter = 20 × 4 = 80 cm For area of (  ADC) = 300 m²
272. (a) D 4x C
A 30cm B Carpet length
2x 40cm 300100
A B 20cm 20cm = = 600 cm
N 4x M 50
D 30cm C

ir
7x Cost of Carpet = ` 15 × 600
a = 20cm, b = 30cm,
1 = 9000

v.iSn
c = 40cm
area = (s um of parallel 279. (a) Old expenditure = `1000
2 a b c 20  30  40
S= = increase in area = 50 × 20 m²
sides)× distance between them 2 2 Increase in expenditure
1 = 45 cm = 50 × 20 × .25 = `250

dnag
(7 x 4 x )2x 176
2 area (  ADC)  New expenditure
= S(S – a)(S – b)(S – c) = 1000 + 250 = `1250
11x 2  176  x 2  16
280. (d) Area of verandah

eYrai
 x 4 = 45(45 – 20 )(45 – 30)(45 – 40 ) = (25+3.5)×(15+3.5)–25×15
AB = 7  4  28 cm = 527.25–375 = 152.25 m²
= 45  25  15  5
CD = 4  4  16 cm cost of flooring= 152.25×27.5
= 75 15 cm2 = Rs. 4186.50 (app.)
CM = 2  4  8 cm
AM=AN+NM
= AN+16
= 150 15 cm2
snhe
ar (  ABCD) = 2 × 75 15 281. (b) 2π R1 = 528
22
 2  7  R1 = 528
12
kgei
= 6+16=22 (AN=BM= =6) 275. (a) Let the diagonal of rhombus
2  R1 = 84 cm
d1 = x & d2 = 2x
AC 2  CM2  AM 2  New Radius = R1 – 14 = R2
1
ERna

Area of rhombus = 2 d1 d2  R2 = 84 – 14
AC 2  8 2  222
 R2 = 70
AC= 64  484  548  2 137 1
256 = (x)(2x) New Radius R2 = 84 – 14 = 70
273. (b) A 2
Area of Road = π (R12 – R22)
16 = x
aBryn

Longer diagonal = 2x = 2 (16)  = π × 14× 154


B
P
D = 32 cm  Total expenditure
276. (b) A 15cm B 22
=  14  154  10
C 7
Les

ABCD is a rhombus = Rs. 67760


h
wa.th

60 282. (b) Since the ratio of length and


AB 15cm D C breadth = 3 : 2
4 20cm
Let length of rectangular field
(Perimeter = 60 cm) As we know = 3x
AC = 24, AP = 12  Area of trapezium Breadth of rectangular field = 2x
wwM

[Diagonals of rhombus 1 Perimeter of the field = 80 m


=  sum of parallel sides   height
bisect perpendicularly] 2 2 (l + b) = 80
2 (2x + 3x) = 80
In  APB 1
AB = 15, AP = 12  175 = 2  20  15   h 2 × 5x = 80
80
 BP = 9  height = 10 cm x = 10 = 8
(B y p yth agoras th eo rem ) 277. (a) let the rate of carpenting then breadth = 2x
= Rs x/m²
BD  9  2  18 = 2× 8 = 16 cm
Area of rhombus  length × breadth × x 283. (c) The sides of a rectangular
1 = Rs 120 ...(i) plot are in the ratio = 5 : 4
=  diagonal1  diagonal2 length × (breadth – 4) × x
2 Let the length of rectangular

Rakesh Yadav Readers Publication Pvt. Ltd. 345

For More Visit : www.LearnEngineering.in


For More Visit : www.LearnEngineering.in

field = 5x 287. (c) Let diagonals be 2x and 5x 1


and the breadth of rectangular 1 × 3x × 4x = 216
2
fieled = 4x A1
 (2x )2
2 4 216  2
According to question,
A2 = 1 2
=
25 x² = 3  4 = 36
Area = 500 m²  5x 
5x × 4x = 500 m² 2 x=
36 = 6
20x² = 500 m²  4 : 25
a b  Perimeter =12 × 6 = 72 cm
500 288. (c) 292. (a)
x² = 20 = 25
h1 h2 r 42cm
5x
x=5
x y 6x
Length = 5x = 5 × 5 = 25 m

r
Breadth = 4x = 4 × 5 = 20 m 1 p e r im e t e r o f r e c t a n g l e =
 h1  x

i
.iSn
Perimeter of the rectangle 2 c ir c um f e r e nc e of c ir c ular
a
1 = wire
= 2(25 + 20)  h2  y b
= 2 × 45 = 90m 2 22
2(6x + 5x) = 2 ×  42
284. (d)

agv
C h1 x a h1 ay 7
  ,  22x = 2 × 22 × 6
h2 y b h 2 bx
x = 12

ridn
ay : bx
289. (a) Ratio of parallel sides clearly,
smaller side of rectangle
.D =5:3

eeYa
A B Let sides are 5x and 3x = 5 × 12 = 60 cm
2 3
1 293. (c)
5
AB = 5 cm (sum of parallel sides)× per-
2
DB = 3 cm
geisnh
pendicular distance= 1440 m²
 AD = 2 cm
2
1
ar   ADC   AD  2
5x  3x × 24 = 1440
ar   ABC 
=   4x × 24= 1440 2R 23
 AB 
Enak

2 1440 2r 22
2 4 x = 4  24 = 15m R 23
=    
5 25  length of longer side = 5x r 22
R

285. (d) Base : Corresponding = 5 × 15 Let R = 23x , r = 22x


altitude = 3 : 4 = 75 m  R –r =5
aryn

Let the base = 3x 290. (c)


23x – 22x =5
altitude = 4x
x =5
Les B

 area of triangle = 1176


 r = 22 × 5 = 110
1
 3x  4x = 1176 a1 ² 225 diameter of inner circle = 2r
2 ATQ, 
a2 ² 256 = 2 × 110 = 220 m
1176  2
wa.th

x² = 3  4 = 196 294. (b) Ratio of angles = 3 : 4 : 5


a1 225 15
3 + 4+ 5 =180°
x = 14 a2 = 256
=
16
 altitude = 4 × 14 = 56 cm 12 = 180°
Ratio of their perimeters
286. (c) According to question,
wwM

180
Ratio of sides of triangle are 4a1 a1 1= = 15°
15 12
=   3 : 4 : 5
1 1 1 4a 2 a2 16
= : : ×15 ×15 ×15
2 3 4
(Take L.C.M of 2, 3, and 4
 15 : 16 45 60 75 largest angle
which is 12) 291. (d) Clearly, 3,4 and 5 form a
295. (b) a

= 6:4:3 triplet therefore, consider the


triangle, a right triangle a a 2 a 2
Now, 6x + 4x + 3x = 52 a
Let the sides are 3x,4x,and 5x
13x = 52 a

x = 4 perimeter = 3x + 4x + 5x = 12x
a 2

 length of smallest side = 3x 1 a 2


= 3 × 4 = 12 cm area of triangle = ×3x ×4x
2

Rakesh Yadav Readers Publication Pvt. Ltd. 346

For More Visit : www.LearnEngineering.in


For More Visit : www.LearnEngineering.in

Let the side of square = a 3 306. (a) Let the sides be 3x,4x,5x and 6x
a1  18x  72, x 4
 Diagonal = a 2 2
Ratio of altitudes =
3  Greatest side = 6 × 4
 a²  a²  a 2  2
a2 = 24 cm
a1 5
307. (b) Ratio of circumference = Ra-
Area of square
= a 6 = 5 : 6 tio of radius = 3 : 4
Area of square on diagonal 2 308. (d) Let the sides be 2x, 3x and 4x
a² a² 1 299. (d) Let length = 5x
   9x = 18  x = 2
= a 2 2
a²2 2 = 1 : 2 l 5
   Sides are 4,6 and 8 cm re-
2l  b = 16 spectively
8l = 5l + 5b

ir
296. (d) Using hero’s formula
3l = 5b
468

v.iSn
l 5 S= = 9 cm
 =5:3 2
b 3
300. (c)  area = s s– a
 s– bs– c

dnag
= 9  5  3 1

area of square = 3 15 cm²


= 309. (b) Ratio of area = (Ratio of
area of equilateral triangle When we draw such figures as men-
radius)²

eYrai
tioned in the question the vertex of
a² 4 the old triangle are the mid points
=
3
=
3
= 4: 3 of the sides of new triangle and the  a a 2
a² = 
 : 
 =4:1
4 sides of the old triangle are half of  3 2 3
297. (d)
r
301. (b)
snhe
the opposite side.
 required ratio = 2 : 1
Circumference
=
2r
310. (a)

Area r ²
kgei
2 2
r² = a² = =
r 3 As D and E are mid-points of AB
a² 302. (b) Ratio of area= (R atio of
ERna

r² = 2
and AC
  a   DE||BC
a
 
radius)² =  3    ODE   BOC
r=  a 
 DE 1
 
aBryn

2r   and also =


Ratio of perimeter = 2 3  BC 2
4a = 4:1 (as D and E are mid-points)
r 303. (a) Ratio of area= (Ratio of ra-
= dius)² ar (ODE)  1 2 1
2a  ar (OBC) =   =
A B C 2
  4
Les

a Radius 4 : 2 : 1
 
wa.th

=  = =  :2 Area 16 : 4 : 1 311. (c)


2a 2 304. (b) r² = a²
298. (c) a² 
=
r² 1
wwM

h1 h2 a The given angle is same


= :1
r let vertical angle =
a1 a2
1 1 1 (   ABC and  DEF are
305. (c) Ratio of sides =: :
3 2 3 4 5 isoceles triangles)
4
a  1
= 20 : 15 : 12  when two angles are equal then
25
= 20 + 15 + 12 = 47 third angle is also equal
3 2
a2
  36  47  94   ABC   DEF
4 1  2  ABC is similar to  DEF
a1 25 5  Smallest side = 12 × 2
 = area of  ABC
a2 36 6 = 24 cm 
area of  DEF

Rakesh Yadav Readers Publication Pvt. Ltd. 347

For More Visit : www.LearnEngineering.in


For More Visit : www.LearnEngineering.in

316. (a) Let the side of square = a 322. (d) 2(l + b) = 4a


 side of  ABC 2 (a = side of square)
= 
 

 side of  DEF   Side of equilateral  = 2a 2(2 + 1) = 4a
1 side of  ABC 2 2 × 3 = 4a
3
=

4 side of DEF
Required ratio = 4   2a 3
a=
side of  ABC 1 2 2
= = a Required ratio
side of DEF 2
3
312. (a) Let the sides be 3x ,3x and 4x = ×2 = 3:2 l  b 1 2 2  4 8
4 =    = 8: 9
2 317. (b) Ratio of area= (Ratio of a²  3 2 9 9
 Area = 4x  4 3x
2
  – 4x
2
side)² 
  
2

r
4
2
ar  ABC 10 
323. (c) 2(l + b) = 3a

i
.iSn
= 4x ² 36x ² – 16x ² = 4x 2
20 x 2 = 
   = 25 : 16
ar  DEF
8  (a = side of equilateral triangle)
= 8x 3 5 = 8 5 = x³ = 1 r12 4 Let (b = a)
318. (a) 
= x=1 r22 7  2(l + a) = 3a

agv
 3rd side = 3 × 1 = 3 units 2(l + a) = 3a
313. (c) 3, 4 and 5 from triplet r12 4 2l + 2a = 3a

ridn
2
Let the sides be 3x,4x and 5x r2 7 2l = a
1 r1 4 2 Required Ratio
 2 × 3x × 4x = 72  

eeYa
= 2: 7 a
r2 7 7 a
l b a² 4
 6x² = 72 319. (c) Required ratio = =  2  
3 3 2 3a ²
5 ² –3 ² 5 ² – 3 ²  16 a² a²
 x² = 12  4 4
5 ² 5 ² 25
2
 x= 2 3
geisnh
 16 : 25 = = 2: 3
3
 Smallest side = 3 × 2 3 320.(a) Let side of square = a
r ² 
= 6 3 a 324. (b) Required ratio= 
radius of smaller circle = r² 1
2
Enak

314. (b) Let the sides be 3x, 4x and 5x = π :1


1 2a 325. (c) Let AB = 1, BC = 1
 area = × 3x × 4x = 72 radius of larger circle =
2 2
R

2
 6x² = 72 a 
 
x² = 12 2 
aryn

Required ratio =
x = 2 3  2a 2
 
Les B

 Perimeter of equilateral  =12  2 


   AC = 1²1² = 2
× 2 3 = 24 3 units
a² (using pythagoras)
24 3 4 1
Side of  = = 8 3 units = 3
2a ² = 2  1 : 2 1 ² 1
wa.th

3 ar ABE 
4 =  4 
3 2 ar ACD  3 2
321. (c) A 2 ²
 
area of  =
4
× 8 3  4
a =1:2
wwM

3 a/3
= × 64 × 3 = 48 3 unit² B C 2 2
arΔABC  AB  10  25
4 326. (b) =
  =
  =

315. (d) Let the parallel sides be 2x side a arΔDEF  DE   8  16
Circumradius = 
and 3x 3 3 327. (c)
1 Equilateral 
 area = (2x + 3x)× 12 = 480
2 3

5x= 80 4 3 3
Required ratio = 2

x = 16 a  4

 

 Longer parallel side  3
= 16 × 3 = 48 cm = 3 3 : 4π A'B' ||AB

Rakesh Yadav Readers Publication Pvt. Ltd. 348

For More Visit : www.LearnEngineering.in


For More Visit : www.LearnEngineering.in

 A' and B' are the mid-point. By mid perimeter of EFGH.


ar DECB
  3
point theorem
1  =
  A ' B ' C   ABC =  perimeter of ABCD ar ABC
  4
2
Let BB' = B'C = 1 Percentage of ar (DECB)
 BC = 2 perimeter of EFGH 1
= 3
( B' is the mid-point of BC) perimeter of ABCD 2 = × 100 = 75%
4
ar A ' B 'C   B ' C 2 1 2 1 331. (c) Old circumference = 4 π

  
   336. (b) Increment in breadth = 10%
ar ABC 
 2πr = 4π
 BC   2  4
10
ar(AA'B'B) = ar(  ABC) – 4π = =
r= = 2cm 100
ar(  A'B'C) 2π
Decrement in length = 10%

ir
Old area = π (2)² = 4 π cm²
10

v.iSn
ar AA ' B ' B
  3 New circumference = 8 π
= =3:4 = =
ar ABC  4 2πR = 8π 100

1 1 1 8π
328. (d) Ratio of sides = : : R= = 4cm

dnag
4 6 8
New area = 16 π cm²
1 1 1 –1
=  24 :  24 :  24 Option (c) is the answer % change = = 1%
4 6 8 ( area is quadruples) 100

eYrai
=6:4:3 332. (c) Length 4 5 Alternate:-
(Take L.C.M = 24) Breadth 5 4 using x = 10% (breadth),
ATQ perimeter = 91 area 20 20 y = –10% (length)
6 + 4 + 3 = 91 area remains unchanged
333. (d) Area of circle = π (5)² = 25 π xy
13 units = 91

1 unit =
91
13
=7
25π
snhe
Circumference of circle
= 2 π (5) = 10 π
% change = x + y

= 10 – 10 +
10  –10
 
100

= –1%
 100 = 250%
kgei
Diff. between long er and = 100
10π
shorter side = 6 - 3 = 3 units xy
334. (d) According to question, 337. (c) % increase = x + y +
 3 units = 7 × 3 = 21 cm Circumference of a circle 100
ERna

329. (c) By using result, = area of circle 20  20


= 20 + 20 + = 44%
R11 = R22 2 π r = π r² 100
r =2 338. (d) If circumference of circle is
R 
1  2  75  5  diameter of circle = 2r reduced by 50% then radius is
60 4 = 5 : 4
aBryn

R  =2×2= 4 reduced by 50%


2 1
330. (c) 335. (c)
A D
E O F

H G
Les

B C
( is constant)
wa.th

In  OBC,
H and G are the midpoints of  D and E are the mid points of Reduction in area
OB and OC sides AB and AC 3
DE||BC (By mid point theorem) = × 100 = 75%
1  4
 HG = BC
wwM

2 1 339. (d) Increase in area


also DE =BC
1 2 25  25
similarly, FG = CD = 25 + 25 +
2 ADE ABC 100
1 ADE  ABC
  xy
and EF = AD,   use formula : (x + y + )
2 100
AED  ACB
 = 50 + 6.25
1
HE = AB 2 = 56.25%
2 ar ADE
   DE 
=  
  340. (a) Increase in area
on adding, ar ABC
   BC 
HE + HG + FG + EF 5050
1
 1 2 1 = 50 + 50 +
100
=  AB + BC + CD + AD  = 
   = 4
2 2  = 100 + 25 = 125%

Rakesh Yadav Readers Publication Pvt. Ltd. 349

For More Visit : www.LearnEngineering.in


For More Visit : www.LearnEngineering.in

xy 25 – 9 4 2
341. (b) using x + y + % decrease = × 100= 64% 356. (d) 40% = 
100 25 10 5
Side Surface area
20  –20
  348. (a)  5 (5)² = 25
= 20 – 20 +  40% 24
100 7 (7)² = 49

= – 4% 24
(decrease by 4%) 8–5 % increase =  100 = 96%
 % Decrease = × 100 25
342. (b) Increase in area 8 Alternate
50  50 1 Per centage incr ease in surf ace
= 50 +50 +
100 = 37 % area

r
2
= 100 + 25 = 125%

i
.iSn
40  40
343. (c) Increase in altitude = 10% 349. (b)  = 40 + 40 + %
 100
=  = 80 + 16 = 96%

agv
xy
30 – 20 [% effect using x + y + ]
% Increase = × 100= 50% 100
20

ridn
357. (a) per centage incr ease in
350. (b) Side 10  11 area
Area no change Area 100  121  88

eeYa
decrease in base = 8  8 
 

121 – 100  100 
1 1 % Increase = × 100
= × 100 = 9 % 100 = 16 + 0.64 = 16.64%
11 11 = 21% 358. (a) Side of square is increased
344. (d) Increase in circumference by 30%
geisnh
351. (b) 
= Increase in radius  30 3
 = =
 Increment 10 0 10
 50%  1 200 – 189
2  Original
% Decrease = ×100=5.5%
200
Enak

352. (d) Radius 100  101


Area 10000  10201 Other s ide will hav e to be
5
R

Increase % = ×100 = 125% decreased by


4 201
%Increase = ×100 = 2.01% 3 1
10000 =  100 = 23 %
aryn

xy 13 13
345. (b) use x + y + 353. (c) Let the breadth = x cm
100 359. (c) Percentage increase in area
 length = (x + 20) cm
Les B

percentage change 100 100


According to the question, = 100 +100 + 100
= 300%
12  15 9
= 12 + 15 + = 27 + x(x + 20) = (x + 10)(x + 5) Alternate
100 5 L
 x² + 20x = x² +15x + 50
4 4
wa.th

= 27 + 1 = 28 %  5x = 50 2
5 5 Percentage increase
 x = 10
346. (b) 3
 Area = 10(10 + 20) = 300 m² =  100 = 300%
100 100 120 150 1
354. (c) 
wwM

 xy
360. (d) x  y +
100 130  100
Pe rime ter of e quilater al 10   –10 
triangle = 100 +100 +100 = 300 1029 – 1000 = 10 – 10 + = –1%
%error = ×100 = 2.9% 100
Perimeter of New triangle 1000 (Negative sign shows decrease)
= 120 + 150 +130 = 400 355. (d)  361. (c)
100 1 
% increase = ×100 = 33 % 
.d/2 .d/2
300 3
% increase in area
347. (b) 
.d/2 .d/2
 156 – 100
 = × 100 = 56%
100 2d

Rakesh Yadav Readers Publication Pvt. Ltd. 350

For More Visit : www.LearnEngineering.in


For More Visit : www.LearnEngineering.in

From the figure it is clear that,  Area of circumscribed square How, area of sector AOB
4 circular plates of diameter d = (2a)² = 4a² θ
can be made of a. Square plate  Difference between areas of = × r²
of side 2d with minimum loss 360º
outer and inner squares
of material. = 4a² – 2a² = 2a² 90º 25 25
= × × = cm²
362. (d)  AOB,  BOC and  COD 365. (c) In  ABC, by Pythagoras 360º 2 8
are equilateral  . C Now, area of minor segment
= Area of sector – Area of tri-
 Side = 2 cm z angle
3 y 8 10 25 r² 25 25
Now, total area = 3 × (Side)² = – = –
4 8 2 8 4

ir
B 2 C  25 – 50 
A 6 B =  

v.iSn

x  8 
2 2
2 2 Area of major segment = Area of
BC² = AB² + AC² = 36 + 64 = 10 cm
A 2 O 2 D Now, area of semi-circle = x circle – Area of minor segment
3  25 – 50 

dnag
=3× ×4 = 3 3 cm² 3 ² 9
4 =  cm² =  r² –  8


2 2  
363. (d) Area of sector = 72 cm²
Area of semi-circle = y
100 – 25  50 75  50
16 = =

eYrai
= cm² 8 8
cm 2
36
 Area of semi-circle = z 25 25  3 
= (3 +2) = 
  1
 cm²
25 8 4 2 
= cm²


r ²θ
360º
= 72
2
snhe
Now, value of x+y–z
 9 16  25
368. (c) ABCD be the rectangle in-
scribed in the circle of diam-
eter 5 cm.
=  2  2  – =0
kgei
  2
A B
72  360
 θ = = 20º 366. (c) Perimeters of triangles,
36  36
ERna

rθ D C
Now, length of arc =
180º  Diameter = Diagonal of rect-
×36×20 angle
= = 4 cm
180 Now, let x and y be the length
aBryn

= (3×1) + (3×0.5) + (3×.025) + and breadth of rectangle are re-


364. (b)
D C
(3×0.125)+ ... spectively,
3 + 1.5 + 0.75 + 0.375 Now In  ABD, AB² + AD² = (5)²
O
.
2a

 x² + y² = 25
3
Les

a 1 = 3  2 =6 units Since, they form Pythagoras


A B Sn = =
1  n 1  2 2 1 triplet,
wa.th

For inscribed circle,  x = 4 and y = 3


Diameter of circle = Diagonal 367. (c) In  AOB,
So, area of rectangle = 3×4
of square AO = OB = r (radius of circle)
Sinc e, s id es of square are = 12cm²
wwM

equal. 369. (b) Required ratio


Now, In  ABC by Pythagoras O Area of circle
theorem =
AB² + BC² = AC² 90º Area of ΔACD
5
2 AB² = 4a²
 A
. B C
AB² = 2a²  AB = 2 a C Minor r
Area of inner square = AB² = Segment

Using Pythagoras theorem, A r r B
 2a ² = 2a² AB² = OA² + OB²  (5)² = r² + r² O
r
For circumscribed square,
Diameter of circle = Side of 25
 r² = cm D
square 2

Rakesh Yadav Readers Publication Pvt. Ltd. 351

For More Visit : www.LearnEngineering.in


For More Visit : www.LearnEngineering.in

Then, 14×8.8 ×h = 8.8 × 56


r ² 1364
= 1  = m²
7 8.8  56
 2r r  h= =4m
2 Now, let the rise in level by h 14  8.8
m, then 375. (a) 1 m = 1000 mm
370. (b) Semi-perimeter of triangle
Area of remaining field × h  10 m = 10000 mm
a+b+c = Volume of mud dugged out Number of 200 mm lengths that
=
2 1364 704 can be cut from 10 m of ribbon
 ×h =
7+24+25 56 7 21 10000
= = = 28 cm = = 50
2 2 704  7 16 200
Area of circle = Area of triangle  h= = m
1364  21 93 376. (d) Area between square and

r
= s s – a s – b s – c  373. (a) Area of the  ABC semi-circles

i
.iSn
= Area of square – 2 Area of
= 28 28 – 728 – 2428 – 25 1 semi-circle
= × b×h
= 28  21 4  3 2

agv
= 7056 = 84 cm² A
5 cm 10 cm
371. (a) Area of equilateral triangle

ridn
3a² a a 10
= =x .....(i) 3 cm cm
4 (h)

eeYa
22
and perimeter = 3a = y = (10)² – 2× × (5)²
7
y B D C = 100 – 78.5 = 21.5 cm²
 a= ......(ii) b/2 b/2
3 377. (a) Let l = 4x and b = 9x
b
Now, puting the value of a from
geisnh  Area of rectangle = l×b
1
Eq. (ii) in Eq. (i), we get  12 = ×b×3 144 = 4x ×9x
2
 y 2 144
3   3  y2 12  2  x² 
3  = x  x =  b= = 8 cm 36
9 4 3
Enak

4  x² = 4  x = 2
y2 y2 b 8
x=  x= Here, BD = CD = = Now, l = 4×2 = 8 cm and

3 3 4 12 3 2 2
b = 9×2 =18 cm
= 4cm
R

 12 3 x = y²  Perimeter of rectangle
In r ig ht ang le d  AB D, b y
On squaring both sides, we get = 2 (l+b) = 2(8+18)
pythagoras theorem,
aryn

y4 = 432 x² = 2×26 = 52 cm
372. (c) Volume of mud dug out in AB = BD2 +AD2 378. (c) Area of parallelogram
Les B

two hemispherical pitholes = Base × Height


22 m  a= 42  32 = 16  9 = 8.06 × 2.08 = 16.76 cm²
= 25 = 5 cm 379. (c) Given that, CD = 20 cm
2m 2m 10 m Now, perimeter of an isosceles and area of rectangle ABCD
wa.th

triangle = 100 cm²


A O B
= 2a+b=2×5+8=10+8 = 18 cm
2 2 22 374. (a) Let the breadth and height
= 2×  r³ = 2 × × ×2³
3 3 7 of r oom be b and h m ,
wwM

respectively.
2  2  22  8 704 D Q C
= = m³ Then, according to the question,
21 21  AD × CD = 100 cm²
 1×b = n Area occupied by one
Area on which the muds is  AD ×20 = 100
patient
spread over  AD = 5 cm [in rectangle AB =
 14×b = 56×2.2
= Ar ea of fieled – Are a of CD = 20 cm and AD = BC = OQ
pitholes 562.2 = 5 cm]
= 1×b – 2 ×  r²  b= = 8.8 m
14 1
22  Area of  ODC = ×PQ × CD =
Now, total volume of the room 2
= 22 × 10 – 2× ×2²
7 is equal to total patients mul- 1
tiplied by volume occupied by ×5×20
176 1540 –176 2
= 220 – = each patient.
7 7 = 5×10 = 50 cm²

Rakesh Yadav Readers Publication Pvt. Ltd. 352

For More Visit : www.LearnEngineering.in


For More Visit : www.LearnEngineering.in

380. (d) Let AB = AC = a So, the length and breadth of a


θ
 BC² = AB² + AC² rectangular field are 90 m and 385. (c) Arc of length = 2 r.
50 m. 360º
(by Pythagoras theorem)
A
 Perimeter of rectangular field
90º = 2 (Length + Breadth) A
= 2 (90+50)=2×140 = 280 m O
 16
383. (c) Given that, radius of a circle
 
10
B D C = 8 cm
and diameter of a circle = 16 cm B
In  ABC,
θ
a²+a² = 2a²  BC = a 2 A a D  16 = 2 r.
360º

ir
90º + θ + θ = 180º
θ 16

v.iSn
a a
(s ince , sum of all inter ior  =
angles of any triangle is 180º) 360º 2r
B a C
 2 θ =90º Now, area of sector OAB
 θ =45º  Le ng th of a diam eter of a θ

dnag
square = Diameter of a cricle =  r².
Now, In  ABD, 360º
AD a  a 2 = 16 16
sin 45º =  AD = =  r². = 8r = 8×10
a 2 2r

eYrai
 a = 8 2 cm
1 = 80 sq cm
 Area of  ABC = ×AD×BC  Area of square ABCD = a²
2 386.(c) Let the breadth of floor be x
2
1 a = 8 2 metre.
= ×a 2 ×   = 64×2
 Length = (x + 20)metre


2


=1
2
= 1sq unit (given) = 128 sq cm
snhe
384. (d) Given that, length of hour
hand = 4 cm
 Area of the floor = (x + 20)x
sq.metre
In case II,
2
kgei
and length of minute hand (x + 10)(x + 5) = x(x + 20)
 a= 2 = 6 cm
 x2 + 15x + 50 = x2 + 20x
 Hour hand rotating in 1 day =
 Perimeter of  ABC  20x = 15x + 50
ERna

2 ×360º = 720º
= 2a + 2 a=2 2 + 2 . 2  Hour hand rotating in 2 days =  5x = 50
 x = 10 metre
= 2(1+ 2 ) units 
2 ×720º = 1440 × radius Area of the floor
381. (c) Area of path = Area of (foun- 180
aBryn

= x (x + 20)
tain + path) – Area of fountain Similarly,
= 10(10 + 20)
Minute hand rotating in 1 day
5m = 300
1. = 24 ×360º 387.(d)
.
3.3
m  Minute hand rotating in 3 days
A B

r=
Les

 x
5m = 72 ×360º × radius
1. 180
wa.th

= r (3.3 + 1.5)² –  (3.3)² D O C


 Distanc e trave lle d by hour Here ABCD is a square of side x.
= [(4.8)² – (3.3)²]
= (23.04 –10.89) = 12.15 m²  x
hand = 4×1440º × = 32  OC = 2 , BC = x, and
382. (c) Let length and breadth 180º
wwM

of a rectangular field are 9x and d is tanc e tr av e lle d b y OB = radius of circle = 10cm


and 5x minute hand In  OCB,
A D OB2 = OC2 + BC2

= 6×72º×360º × 2
5x 180 x
   + x2 = (10)2
= 6×144 2
B 9x D Required ratio 5x2
 Area of a rectangular field 2

= 4500 m² 32  4 = 100  x = 80


=
 9x × 5x = 4500 6 144  x = 4 5cm
 x² = 100 = (10)² 1 Hence, perimeter of the square
 x = 10 =
27 ABCD = 4x = 16 5

Rakesh Yadav Readers Publication Pvt. Ltd. 353

For More Visit : www.LearnEngineering.in


For More Visit : www.LearnEngineering.in

388. (c) 1. We know that, Area of  (5x)² – (3x)² = 16 394. (b)  ABC forms an equilateral
segment (PRQP)  25x²–9x² = 16 triangle.
= Area of sector (OPRQO) – Area  16x² = 16 A
r ²θ 1  x² = 1  x =1 x/3 x/3
of  OPQ = – r² sin θ  r1 = 5 and r2 = 3
360 2 x/3

.S  Area of the outer circle


=  r12 =  (5)² = 25 cm² x/3
G F
x/3

391. (c) Let the radius of a circle is


x/3
.
H x/3

.O r and a be the length of the side


of a square.
x/3
x/3 x/3 x/3

 Give n, c ir cumf er ence of a B x/3 D E x/3 C

r
x/3
P . Q
circle = Perimeter of a square

i
.iSn
R  2 r = 4a
So, the area of a segment of a where, AGHF form a rhombus and

circle is always less than area of  a= r = 1.57r  HDE is als o an
its corresponding sector. 2

agv
equilateral triangle.
II. Distance travelled by a circu- Now, area of the circle (AC)
 Area of rhombus = (Ar ea of
lar wheel of diameter 2 d cm =  r² = 3.14r²
 AGF + Area of  GFH )

ridn
in one revolution and area of the square (As)
= a² = 2.4649r² 3  x 2 3  x 2
= 2
2d = 2×3.14×d = 6.28d  Area of circle > Area of square = 
   + 
  

eeYa
2 4 3  4 3 
392. (b) Let the radii of two circles 2
which is greater than 6d cm. are r1 and r2. respectively. 3  x
= 2.  
 
389. (b) Given that, perimeter of a Given, 4 3 
rectangle = 82 m 3 x 
2
Circumference of 1st circle 2
 2 (Length + Breadth) = 82 m
geisnh = Now, area of  HDE = 
 
Circumference of IInd circle 3 4 3 
 Length + Breadth = 41 m
2r1 2 r1 2 3
 l+b = 41 m. . . .(i)  and area of  ABC = x²
Also, its area = 400 m²
 2r2 = 3  r2 3 4
By given condition,
Enak

 l.b = 400m²  r 2
 1 = 4 ...(i) Area of rhombus AGHF + Area of ΔHDE
Now, (l –b)² = (l+b)² – 4lb  r 
 2 9 Area of ΔABC
=(41)² – 4 (400)
Area of 1st circle r12
R

= 1681 – 1600 = 81 3 x 
2
 = 2 3  
 l–b=9 .....(iii) Area of IInd circle r2 4  
3  1
From Eqs. (i) and (iii),  r 2 4 = =
aryn

1
3 2 3
2l = 50  l = 25 m and =   = x
r
 2 9 4
b = 16 m
Les B

393. (b) We know that the radius of 395. (c) Given that,
 Required breadth (b) = 16 m Area of the circle = Area of
390. (c) Given that, ratio of their ra- a circ le ins cr ib ed in a
the square = (Side)²
dii = 5:3 a
i.e., r1 : r2 = 5 : 3 equilateral triangle = r² = (2  )²  r² = 4 
2 3
wa.th

where, a be the length of the 4


A r² = =4
r2 side of an equilateral triangle. 
.O r1
B
Given that, area of a circle
inscribed in an equilateral tri-  r= 4 = 2 units
wwM

angle = 154 cm²  Diameter of circle (d)


= 2.r = 2.2 = 4 units
 a 2
   
 = 154 396. (a) Let the radius of circle is r
r1 5 2 3  and the side of a square is a,
 = .....(i) then by given condition,
r2 3  a 2 154  7
 
 
 = = (7)² r
Let r1 = 5x and r2 = 3x 2 3  22 2 r =4a  a =
Also, given that, area enclosed 2
 a = 14 3 cm r  2
between the circumferences of
two concentric circles  Perimeter of an equilateral tri-  Area of square =  
2
= 16 cm² angle = 3a = 3(14 3 )
²r² 9.86r ²
 (r12 – r22) = 16 = 42 3 cm = = = 2.46r²
4 4

Rakesh Yadav Readers Publication Pvt. Ltd. 354

For More Visit : www.LearnEngineering.in


For More Visit : www.LearnEngineering.in

and area of circle = r² = 3.14r²  Perimeter of an equilateral tri-


angle = 3a = 3×2 = 6 cm 3
and let the side of equilateral  2 3 = a
399. (d) Given that, 2
triangle is x.
Then, by given condition,
E  a = 4 cm
 Area of equilateral triangle =
2r
3x = 2 r  x = 3
3 2a (4)² = 4 3 cm²
F 4
D C
3 403. (a) Required area of the path
 Area of equilateral triangle = a
4 EF = 30+4 = 34m, GF = 16+4
=20 m
3 4²r ² A B

ir
x² = × Area of rectangle = 2a² = l×b H G
4 9
l×b = 2a² = l×a  l=2a

v.iSn
 D 2m C
² Now, In  ACD,
= r ² = 1.89r²
3 3 AC² = AD² + CD² 2m 2m
16 m
Hence, Area of circle a² +4a² = 5a²

dnag
> Area of square  Side of square, AC
A 30m B
> Area of equilateral triangle = a 5 unit 2m
397. (a) Let the wid th of the
Hence, area of square E F
rectangle = x unit

eYrai
 Area of path = Area of EFGH -
 Length = (2 x + 5) unit = (a 5 )² = 5a² sq units
Area of ABCD
According to the question, 400. (c) Outer diameter = 112 cm = 34 × 20 – 30 × 16
Area = x (2x+5) and inner diameter = 70 cm = 680 – 480 = 200 m²
 75 = 2x² +5x
 2x² + 5x – 75 = 0
 2x² + 15x – 10x – 75 = 0
 x (2x +15) – 5 (2x +15) = 0
snhe
70
cm
. 112
cm
404. (c) In right  ABC,

D C

.
kgei
 (2x +15) (x–5)=0
12 cm
–15 O
 x =5 and Required area
2 
ERna

A 16 cm B
Since, width cannot be negative. 1
=  (112² – 70)²
 Width = 5 units and 4
AC² = AB² +BC²
length = 2x +5 1
= (12544– 4900) AC² = (16)² +(12)²
aBryn

= 2×5+5 = 15 units 4 = 256 + 144 = 400


 Pe rime te r of the r ec tang le 1 22  AC = 20 cm
= 2 (15+5) = 40 units = ×7644×
4 7  AO = 10 cm (radius)
398. (c) Height of equilater al tri-
angle, 1 and area of circumcircle = r²
= ×24024 = 6006 cm²
4 =  × (10)² = 100 cm²
Les

3
(AD) = × Side 401. (d) Let the length of altitude AB 405. (c) In  ABC,
wa.th

2 =l
3×side By given condition, BC = AB²+AC² = a²+b²
 3=
2 Area of  ABC = Area of square
 2 3 = 3 ×side
wwM

1 C
 ×Base×Altitude = (Side)²
2 3 2
 Side = = 2 cm a²+b²
3 1 36  2 b
 ×9×l=36  l=
A 2 9
 l = 8 cm A a B
402. (d) Area of equilateral triangle
a a
3  Required total area = a² + b²
= a²
4 2 1
+  a²  b²  + 2 ab
B D C 3
a But altitude = a
2 = 2(a²+b²)+0.5ab

Rakesh Yadav Readers Publication Pvt. Ltd. 355

For More Visit : www.LearnEngineering.in


For More Visit : www.LearnEngineering.in

406. (b) Area of trapezium Area of the shaded region  p× OD²+OC² = 1320
= Area of square of side 6cm - 4  a
1  55 2  48 2
= (3+5) × 4 = 16 m² right angled sector 
2  p×    
   =1320
2  2 
 Total cost of painting Rs. 25 32
364  1 1 
per sq m = 16×25 = Rs. 400 4  OD  BD and OC  AC 
 
407. (c) Suppose a horse is tied at  2 2 
 36  9  94   sq. cm.
vertex A. Then, area available 5329
grazing field is ADE. 410. (a) Let the sides of a square be  p× = 1320
4
A x cm,
1320
In  ABC, AC² = AB² +BC² p= =36.16
m

60º 36.5

r
2

36 cm < p < 37 cm
4.

D C

i
.iSn
D E 413. (c) Perimeter = a + b + c
cm x 240 = a + 78 + 50
2
a = 112
60º 60º 15

agv
A x B 1
B 6m C  Area of triangle = × Base ×
Now, area of curve ADE AC² = x² +x² 2

ridn
r²θ Height
=  (15 2 )² = 2x²  2x² = 225 ×2 A
360º
 x² = 225  x = 15 cm

eeYa
m
22  (4.2)²  60º Hence, length of the side of the

78
= = 9.24 m² square be 15 cm. c =50 m
7  360º

b=
h
411. (b) Since, all sides of a  ABC
and area of equilateral  ABC
are equal, so their all angles
geisnh
3 3 are equal to 60º
= (side)² = × (6)² = 15.57 B a =112 m C
4 4
A 4 4 B and also, Δ= s s–a s–bs–c
 Required percentage 60º 60º
4 4  Area of = 120 120 – 112120 – 78120 – 50
Enak

9.24 4 60º 4 = 120  8  42  70


= ×100
15.57 = 1680 m²
C
= 59.34% = 59% (approx)
R

1
408. (a) By given condition, Ar ea of Portion includ e  Area of triangle =
2
× Base ×
between circles Height
A1 r 21
aryn

1
= = = Area of triangle – Area of 3
A 2 r 22 2 1
sectors  1680 = ×50×l
Les B

2
 r 2 1 3 2 60º 2 1680
  1 = .....(i) = 8 –3× × (4)² h= = 67.2 m
r  2 4 360º 
50
 2
= (16 3 –8 ) cm² 414. (b) Let the sides of isosceles
If circle or bent in the form of
wa.th

triangle be 5x, 5x and 3x cm,


square, then 2 r1 = 4a1 412. (a) Area of rhombus
respectively.
r1 r2 1 1 By given condition,
 a1 = and a² = = ×d1×d2= ×55×48
2 2 2 2 Pe rime te r of isosc eles
triangle = Length of wire
wwM

r1 2 = 1320 cm
2   5x + 5x +3x = 78  13x = 78
A1 a 
 2 
 r 21 1 A B
= =
1
  x = 6 cm
 2 2 =
A2 a 2r2  r 22 2  Length of base = 3×6 = 18 cm

 

 2  O 415. (b) The angle made b y the
[from Eq. (i)] P minute hand in 20 min = 120º,
409. (b) (1 minute = 6º )
 The are a swep t by the
A B
D E C minute hand in 20 min
 Area of rhombus θ 120º
D = Base × Height = DC × AE = ×r2 = ×3.14×9×9
C 360º 360º
 DC×AE = 1320 = 84.78 cm²

Rakesh Yadav Readers Publication Pvt. Ltd. 356

For More Visit : www.LearnEngineering.in


For More Visit : www.LearnEngineering.in

416. (c) Given, AB = 2a = 7×11×2×3 = 462 cm² 1 49


R
Hence, radius of incircle Area of  BCD = ×7×7 = cm²
2 2
r .
O
r
=
 462
= =6 cm A D
Q s 77
P
A C M D B
419. (b) Width of the border = x m E
 AM = a D C
xm 7 cm
a H G
and AC = CM = MD = BD = F
2 3m xm
x
Now, OC = OP+PC = OP+CM
B 7 cm C
a E F

ir
xm
= r+ and  Re quir ed are a of s hade d
2 A B

v.iSn
4m region = 2 Area of curve BEDF
a Given, area of EFGH = 6m²
OD = OQ+QD = OQ+MD = r +  77 49   28 
2  (4–2x) (3–2x) =6 = 2  2 – 2  =2  2  =28 cm²
 12–8x – 6x +4x² = 6    
So,  OCD is an isosceles tri-

dnag
 4x²–14x+12 =6 423. (a) Let sides of a rectangle be l
angle. ( OC = OD)
 4x²–14x+6=0 and b.
  OMC = 90º Then, 2(l+b) = 18  l+b = 9
 2x²–7x+3=0
In  OMC,  Area of rectangle = l×b
 2x²–6x–x+3=0

eYrai
OC² = OM² + CM²  2x(x–3)–1(x–3)=0 F or m axim um , ar ea of
 a 2  a 2 rectangle, l=b
 (x–3)(2x–1) = 0
 r  
 = (a–r)²+     2l = 9  l = 4.5
 2 2  1
 x = 3,  Maximum area of rectangle =
a² 2
 r² +
4
+ar

= a² +r² – 2ar +


1
x=
2
snhe
= 0.5 ( x  3)
420. (b) Inner radius, r1 = 25 cm
l×b = (4.5)² = 20.25 cm²
424. (b) Let r = Radius of 3 smaller
laminas
In  ADC, (2r)² = r² + DC²
4
kgei
and external radius, r2
a 2
r = = 25+10 = 35 cm
3  DC = 3r  OC = 3 DC
Distance covered in 1 revolu-
417. (b) Let r be the radius of circle
ERna

and a be the side of square. 22 2 2r


By given condition, 2 r = 4a tion = 2 ×35 = 70× = × 3r =
7 3 3
r = 220 cm. Radius of larg er c ir cular
 a=
and distance covered in 10
aBryn

2 lamina = OE
r  2 revolutions = 2200 cm
 Area of square =  
2
 Speed of bicycle
OC + CE =
2r
+r= 2+ 3 r
Covered distance 3 3
2r 2 9.86r 2 =
= = = 2.46 r² Time Area of 3 laminas = 3 r²
4 4
Les

and area of circle =  r² 2200 22 Area of larger lamina


= cm/s = m/s
wa.th

= 3.14r² 5 5  2 3 
2

Hence, area of the circle is = 4.4 m/s 


= 
 
r 
larger than that of square. 421. (c) Length of wire = 36 cm  3 
 Perimeter of semi-circle  
418. (d) Let a = 35 cm, b = 44 cm and
=  r+2r 4  3  4 3  r²
wwM

c = 75cm
=
 22  3
a+b+c  36 = r   2

 s= 7 
2
{  Perimeter of semi-circle = = 7  4 3r²
Length of wire} 3
35+44+75
= = 77 D
2 36  7 A
Now, Area of D r = = 7 cm r r
36
r r
= s s–a s–bs–c Hence, radius of semi-circle=7 cm O
B r C
422. (c) Are a of c ur ve B CDE = r
= E
77×42×33×2 1 22 77
 (7)² = ×7×7= cm²
= 7×11×2×3×7×3×11×2 4 7 4 2 Residual area
7  4 3 
 – 3 357
Rakesh Yadav Readers Publication Pvt. Ltd. =  3 r2
 

For More Visit : www.LearnEngineering.in


For More Visit : www.LearnEngineering.in

 PQ = QR = RS = 4 cm
4 3 –2 1 3.14  20  20
=   r 2  Perimeter of shaded region = ( r²) =
3 4 4
= Perimeter of semi-circle PTS
 Required ratio = 314 m²
+ Perimeter of semi-circle QUS
4 3 – 2 2 + Perimeter of semi-circle PVQ 432. (a) Here width of sheet is 20
 r
 3  =  (6) +  (4) +  (2) = 12 cm cm, which is the maximum di-
  ameter of the circular sheet.
= 74 3 428. (b) Circumference of circular
r 2 path = 2 × 50 m  Remaining area of sheet
3
= 10000 cm = Area of rectangle sheet
4 3 –2 7–4 3 – Area of circular sheet
= × and circumference of wheel
74 3 7–4 3 = 2 × 50 = 100 cm = 25×20 –  (10)²

r
28 3 – 48 – 14  8 3 Distance covered in 60 min = = 500 – 314
= 
= 186 cm²

i
.iSn
49 – 48 10000 cm
433. (b) Let the other sides of a right
= 36 3 – 62 = 36 × 1.732 – 62 Distance covered in 15 min
isosceles triangle be a cm.
= 62.352 – 62 = 0.35 10000
=  ×15 = 2500 cm In  ABC,

agv
425. (a) Circ um fe re nc e of c ir cle 60 C
= 2 ×42  Number of revolutions

ridn
22 2500
= 2× ×42 = 264 cm = = 25 6 2 cm
7 100 a
429. (b) The distance covered by a

eeYa
Perimeter of square = 4x
 264 = 4x man diagonally is
 x = 66 cm 3 1000 A a B
426. (a) Ar ea of 2 bigg er s em i- d= ×1 = 50m
60 a²+a² = 6 2
r²
geisnh 1
circles = 2×  Area of field = d²  a 2 =6 2
2 2
 a = 6 cm
1
 0.5 2 1 0.25 = × (50)² = 1250 m² 1
= 2   × =
 cm² 2  Area of  ABC = × a²
 2  2 4 430. (c) Let the side of an square be 2
Enak

and area of 5 smaller semi – a cm. 1


By given condition, = ×6×6=18 cm²
5r² 1  0.5 2 2
circles = =5× × ×   Area of square – Area of an 434. (b) a = 9cm, b = 10 cm
R

 
2 2  4 
1 and c = 11 cm
5 0.25 1.25 equilateral triangle = 9  10  11
4
aryn

= × = cm² s= = 15 cm
2 16 32 2
3 1
Area of rectangle ABCD  a² – a² =
Les B

4 4 A = s s–a s–bs–c
= 2×0.5 = 1 cm²
 
3 1
Area of remaining portion  = 15 15–915–1015–11
 a² 1 – 4  =
0.25 1.25   4
= 1– – 1 = 15×6×5×4 =30 2 = 42.3 cm²
4 32
wa.th

 a² (4 – 3 )=1  a²=
4– 3 435. (a) As we know that, if the
 5
= 1– –  a = (4 – 3 )–1/2 cm length of square and rhombus
16 128
431. (d) Suppose a pole is fixed at a are same, then the area should
128 – 8 – 5 point C. be same.
wwM

=
128  Area of field in which the horse 436. (a) Area of circle = 4 cm²
128 – 13 can graze = Area of field in (given)
= cm² which the horse can graze =   r² = 4  r = 2 cm
128
427. (a) Given, OS = 6 cm Area of curve CFE
C
50 m
T F 20 m
D C
U
20 m

50 m
P QO
. .R S E
50 m
O
º m
30 2c
V 30º
A B A D B

Rakesh Yadav Readers Publication Pvt. Ltd. 358

For More Visit : www.LearnEngineering.in


For More Visit : www.LearnEngineering.in

OD 22 r²θ 3.14  20  20  90º


In  OAD, tan 30º = × 28 – × 21 × 21 = =
AD 7 4 360º 360º
51135 1 3.14  400
 AD = 2 3 cm = + (21×28–33×21) = = 314 cm²
4 2 4
Now, AB = 2 AD = 4 3 cm 1925 1  Area of minor segment
= + (–105) = Area of sector OACBO – Area
4 2
 Area of equilateral  ABC
= 481.25 – 52.50 = 428.75 cm² of  AOB = 314 – 200 = 114 cm²
3 3 439. (b) Let the sides of squares S1 442. (b) Angle made in 60 min by
= (AB)² = (4 3 )²
4 4 and S2 are a and b, respectively. minute hand of a clock = 360º
= 12 3 cm² So, perimeters of square S1 and and angle made in 15 min by
S2 are 4a and 4b, respectively.

ir
minute hand of a clock
437. (a) In  ABC, By given condition,

v.iSn
A 360º
4a = 4b + 12 = ×15º = 90º
a=b+3 ....(i) 60º
D E and a² = 3 (b)²–11  Required distance
x y  (b+3)² = 3 b²–11

dnag
2(14)90º 22 14  2
 b²+6b+9 = 3b² – 11 = = ×
360º 7 4
 2b² – 6b – 20 = 0 = 22 cm.
B C
 2b² – 10b + 4b – 20 = 0
BC = x ²  y² x

eYrai
 2b (b–5)+4(b–5) = 0 443. (c) Let the length of rectangle =
 Area of  ABC 2
 (b–5)(2b+4) = 0
1 1  b = 5m ( b  –2)
= × x × y = xy x
2 2 On putting the value of b in Eq. and breadth of rectangle =
6
Area of semi-circle BACB

=
x ²  y²
4
snhe
(i), we get
a = 5+3 = 8
 Perimeter of S1 = 4×8 = 32 m
 Area of rectangle

=
x x x²
× =
2 6 12
kgei
 Area of shaded portion 440. (b) From a rectangular sheet of
= Semi-circle ABDA cardboard of size 5×2 cm², a and area of square = x²
+ Area of semi-circle AECA- circle of radius 1 cm, can be Hence , ar ea of re maining
ERna

(Area of semi-circle BACB – cut-off. x² 11x ²


Area of  ABC) D C portion = x² – =
12 12
x ² y²   x ²  y²  444. (b) Remaining perimeter
2 cm

=  –  4  + Area 4 cm 4 cm
4 4  
aBryn

4 cm

4 cm
10 cm
of  ABC = Area of  ABC

2 cm
438. (d) In  ABC, A B
5 cm 2 cm 10 cm
AC² = 28²  21² = Area of rectangular sheet
4 cm

4 cm
784  441
= 5×2 = 10 cm² 18 cm
Les

= 1225  AC = 35 cm and area of circle 4 cm 4 cm


wa.th

A =  (1)² =  cm²  2r 


=  4  4+10+2+10+2
 R equired area = Area of  
D E
sheet – Area of circle = 2 × 3.14 × 4 + 24
28 cm

= (10 – ) cm² = 25.12 +24 = 49.12 cm


wwM

1 = 49.1 cm (approx)
441. (d) Area of  AOB = × OA × OB 445. (b) Distance travel in 1 revolu-
2
B C 440
21 cm tion = m
Area of shaded portion = Area O 1000
of Semi - circle ACE
cm

and circumference =  ×d
20

+ Area of  ABC – Area of quad- 90º


20

cm

44000
rant circle BCD A B = cm
1000
r² 1  2 C
= + ×BC×BA – ×r 1 44000  7
2 2 4 1  d= = 14 cm
= ×20×20 = 200 cm² 1000  22
22 1 35 35 1 2
= × × × + × 21 446. (c) Side of an e quilater al
7 2 2 2 2 and area of sector OACBO
triangle is a

Rakesh Yadav Readers Publication Pvt. Ltd. 359

For More Visit : www.LearnEngineering.in


For More Visit : www.LearnEngineering.in

then the altitude of equilateral a2 /4 2 1 324 = 36  h


   1: 2
3 a2 /2 4 2 324
triangle is a h   9cm
2 449.(a) Larger Radius (R) = 14 + 7 36
= 21cm
 Radius of incircle 453.(d) 2r  2(18  26)
Smaller Radius (S) = 7cm
a 3 1 a  Area of shaded portion 22
= × = 2 r  442  r  14cm
2 3 2 3   7
= R  r 2
2
C 360 360  Area of circle  r2
30
=  (21 21  7  7) 22
 1414 616sqcm
. .
360

r
aS a 7
R 22 1
  28 14 = 102.67cm2

i
454. (d) r  2r  36

.iSn
O =
7 12
22 
P x Q 450. (c)  r   2  36
30º  7 
A B

agv
D
 Diameter of incircle O 22 14
 r    36
 7 

ridn
 a  a A B
=2  
 = C
 2 3  3 367
OC = 12cm AC = CB = 5cm r   7 metre

eeYa
Let side of a square be x. 36
 a 2  Radius ‘OA’ = OC 2  AC 2 r 2 1 22
 
 
 = x² + x² Area    77
 3  122  52  144  25 2 2 7
 77sq .metre

geisnh
 = 2x²  169  13cm 455. (b) Let the radius of circle be ‘r’
3
 Diameter of circle  2  r = 24   r = 12
a²  Area of circle
 x² = = Area of square  2 13  26cm
6
Enak

451. (d) A =  (12)2 = 144 


447. (c) Radius of circle, r = 6 cm  Area of the rectangle = area
 Area of circle= r² =  ×6² of circle = 144 
456. (b)
R

= 36 cm² A
O
and area of sector subtending
an angle of 80º at O
aryn

B C
r²θ ×6²  80º D r
= = 8 cm² DB = DC = 3cm.
Les B

360º 360º
 Required difference AD  AB 2  BD 2  6 2  3 2 B
a=8cm
C
= 36 – 8 = 28 cm²
 36  9  27  3 3cm. 3 2
448.(a) Area of triangle  a
wa.th

D C  OD = In-radius 4
R 1
=  3 3  3cm. 3
r 3  88 16 3cm2
4
 Area of circle  r 2
wwM

radius of incircle (r)


A a B
   3  3  3 sq.cm a 8 4
=  
a Alternatively : 2 3 2 3 3
Radius of incircle  r  a 6
2   3 Area of inscribed circle =  r 2
Inradius(r)=
and Radius of circum-circle 2 3 2 3
2
 4 
   = 22 × 16
2
a  Area = r 2 =  3 = 3   
= = R  3 7 3
2 452.(d) Area of parallelogram
= base  height  Required area
r 2 r 2 = 27  12 = 324sq. cm.  22 16 
 
= 16 3 
 Ratio of area 
R 2 R 2 Again,  21 

Rakesh Yadav Readers Publication Pvt. Ltd. 360

For More Visit : www.LearnEngineering.in


For More Visit : www.LearnEngineering.in

16 86 24 1
= (211.732  22) =  Area of Δ OCD = × OD× OC
21 86 7 2

16  area of square 1 15 75
= 14.372 10.95cm2  10 
21 24 2
576 2 2 2 2
    cm
 7  49  Area of OAB
457.(b) height = h
Alternatively,
3 1 1
a (a ? side of ? ) let side of largest square = x  OB OA  43  6
= 2 2
2  AP = (8 - x)cm and

ir
a MC = (6 - x)cm 75
inradius =  Area of trapezium  6

v.iSn
2 3 In  ABC and  DMC, 2
incirle’s diameter (d)  B =  M = 90°
 C =  C = (common) 75 12 63
2a a    31.5sq.units
= =  ABC ~  DMC,
2 2

dnag
2 3 3
BC AB 6 8 460. (c) Side of the first square
a  =  
circum-radius = MC DM 6x x
3  Area

eYrai
 circumcircle’s diameter 24
3x  24 4x  x  cm
7  200  10 2metre
2a
(D) =  area of square
3 Its diagonal  2  side
 d:D:H

a 2a 3
= x2 =
49
snhe
576 2
cm

459.(b) For 3x + 4y = 12
 10 2  2
= : : a  2:4:3
kgei
3 3 2 By putting x = 0, y = 3  20metre
By puttting, y = 0, x = 4
458. (a)  Diagonal of new square
For 6x + 8y = 60,
ERna

15  2 20  20 2 metre
By putting x = 0, y 
2
1
diagonal 
2
By putting y = 0, x = 10  Its area 
aBryn

M Y 2
C (0,15
2
A
(0,3)
1
Side of maximum sized square X’
O
X
D (10,0)
 20 2 20 2
B (4,0) 2
AB×BC
Les

AB+BC  400sq.metre
wa.th

Y’
wwM

Rakesh Yadav Readers Publication Pvt. Ltd. 361

For More Visit : www.LearnEngineering.in


For More Visit : www.LearnEngineering.in

CHAPTER

MENSURATION
17
3–D (THREE DIMENSIONAL)
Mensuration is the branch of mathemat-
ics which deals with the study of differ-

r
ent geometrical shapes, their areas and

Sni
Volume. In the broadest sense, it is all
Face

gv.i
about the process of measurement. It is
based on the use of algebric equations
and geometric calculations to provide CUBOID SQUARE
CUBE

ridna
measurement data regarding the width, BASED
depth and volume of a given object or PYRAMID
group of objects. while the measurement

eeYa
results obtained by the use of mensura-
tion are estimates rather than actual
physical measurements, the caluclations
are usually considered very accurate.
geisnh
There are two types of geometric shapes:- TRIANGULAR PRISM
TRIANGULAR BASED CYLINDER
1. 2D 2. 3D PYRAMID
3D shapes: They have surface area and
Enak

volume.
(1) Cube
R

(2) Rectangular Prism (Cuboid)


(3) Cylinder (4) Cone
aryn

(5) Sphere and Hemisphere CONE SPHERE HEMISPHERE


(6) Prism (7) Pyramid
Les B

What is 3D....? surface area is the circumference of Cuboid (Parallelopiped)


the base of the solid and the face
A three-dimensional shape is a A cuboid is a 3 dimensional
parallel to it. The Total Surface area
wa. th

solid shape that has height and shape.


is the sum of both the curved sur-
depth. For example, a sphere and a It is a solid figure which has
cube are three-dimensional, but a face area and the area of the base
6-regular faces.
circle and a square are not. and top.
12 edges, 8 vertices and 4 di-
wwM

What is Volume.....? agonals.

Volume is the measure of the


amount of space inside of a solid fig-
Height

ure, like a cube, ball, cylinder or pyra-


mid. It's units are always "cubic",
that is the number of little element Le
ng
th dth
cubes that fit inside the figure. ea
br
Difference between Curved
surface area and Total surface area Look at this shape.
The area of all the curved There are 3 different measurements:
surfaces of any solid. The Curved Length, Breadth, Height

Rakesh Yadav Readers Publication Pvt. Ltd. 362

For More Visit : www.LearnEngineering.in


For More Visit : www.LearnEngineering.in

Formula (c) Diagonal (iii) Total surface area = Surface


= 2 2 2 area + Base area
(i) The volume is found using the l +b +h
formula: = 1248 + Length × Breadth
= 162 +182 + 242 = 1248 + 64 × 14 = 1248 + 896
Volume = Length × Breadth ×
Height = 2144 m²
= 1156 = 34 cm
Which is usually shortened to: 5. In swimming pool measuring
2. Find the length of the longest
V=l×b×h 90 m by 40 m, how much wa-
pole that can be placed in a
Or more simply: ter will be displaced by 150
room 30 m long, 24 m broad
V = lbh men, if the displacement of
and 18 m high.
(a) V = l  b  h cubic units water by one man is 8 cm³,
Sol. d= l 2  b2  h 2 what will be the rise in water

ir
(b) V = A1×A2 ×A 3 cubic units level?

v.iSn
d= 900 + 576 + 324
Where, Sol. Volume of water displaced by
A1 = area of base or top = l b = 1800 150 men = Volume of water
sq. units came out
d = 30 2 m
(Let the height raised in water

dnag
A2 = area of one side face =bh
3. A brick measures 20 cm × 10 = h)
sq. units
cm × 7.5 cm. How many  8 × 150 = 90 × 40 × h
A3 = area of other side face =
bricks will be required for a
hl sq. units 1
wall 20 m × 2 m × 0.75 m ?

eYari
(ii) Lateral surface Area /Curved  h= m = 33.33 cm
Sol. Number of bricks = 3
surface area/ Area of four
Total Volume of wall 6. A rectangular water reservoir
walls
is 15 m × 12 m at the base.
= Perimeter of Base  height Volume of one brick
= 2 (l + b)  h sq.units
(iii) Total surface Area = 2(lb + bh
+ hl) sq. units
=
snhe
20 × 2 × 0.75 ×100 ×100 ×100
20 ×10 × 7.5
Water flows into it through a
pipe whose cross section is 5
cm by 3 cm at the rate of 16
m/s. Find the height to which
= 20,000
kgei
(iv) Diagonal of cuboid water will rise in the reservoir
4. A rectangular sheet of metal is
= in 25 minutes.
l 2  b 2  h 2 units 80 m by 30 m. Equal squares Sol. Volume of water comes out
of side 8 m are cut off at the
ERna

(v) To find total surface area of a fro m pi pe in 1 sec


cuboid if the sum of all three corners and the remainder is 5 3
sides and diagonal are given. folded up to form an open rect- = × × 16 m³ = 0.0240 m³
100 100
Total surface area = (Sum of angular box. Find: (i) Volume
Volume of water comes out
(ii) Total surface area (iii) Sur-
aBryn

all three sides)² – (Diagonal)² from pipe in 25 min = 0.0240


Note : For painting the sur- face area of box.
× 25 × 60 = 36 m³
face area of a box or to know Sol. (i) When four square of 8 cm
 Volume of water pour into
how much tin sheet is re- are removed from four corners
tank = Volume of water comes
quired for making a box, we of rectangular sheet.
out from pipe.
Les

use formulae (iii) i.e. Total 8m 64 m 8m


 15 × 12 × h = 36
wa. th

surface Area. (  h = rise in level of water)


To find the length of the long-  h = 0.2 m
est pole to be placed in a room, 14 m 14 m
7. The sum of length, breadth
we use formulae (iv) i.e. Di- and height of a cuboid is 25
wwM

agonal. cm and its diagonal is 15 cm


8m 64 m 8m
EXAMPLES Length and Breadth of remaining long. Find the total surface
rectangular sheet will 64 cm and 14 area of the cuboid.
1. The dimensions of a cuboid cm & height of sheet will be 8 cm. Sol. We have the total surface area,
are 16 cm, 18 cm and 24 cm. = (25)² – (15)² = 625 – 225
Volume of open rectangular box
Find: = 400 sq. cm.
(a) Volume = Length × Breadth × Height
(b) Surface area (c) Diagonal = 64 × 14 × 8 = 7168 m³ CUBE
Sol. (a) Volume = l × b × h = 16 (ii) Surface area = 2(Le ngth +
×18 × 24 = 6912 cm³ A cube whose length, breadth and
Breadth) × Height
(b) Surface area = 2 (lb + bh + height are all equal is called a cube. A
= 2(64 + 14)×8 = 2 × 78 × 8 cube has 6 equal faces, 12 equal edges,
hl) = 2(16×18 + 18×24 +
24×16) = 2208 cm² = 1248 m² 8 vertices and 4 equal diagonals.

Rakesh Yadav Readers Publication Pvt. Ltd. 363

For More Visit : www.LearnEngineering.in


For More Visit : www.LearnEngineering.in

3. The surface area of a cube is A right circular cylinder is a cylin-


864 cm². Find the volume. der whose base is a circle and whose
a
Sol. 6a² = 864  a² = 144 elements are perpendicular to its
 a = 12 cm base.
r
a³ = (12)³ = 1728 cm³
a 2 r

a
4. The Cost of painting the whole
surface area of a cube at the
Consider a cube of edge a h h

units. rate of 13 paise per sq. cm is


Rs. 343.98. Then the volume
It is a special type of cuboid in Lateral surface (unrolled)

which l = b = h = a units i.e. of the cube is: d

Sol. Cost of painting the whole sur- Right Circlular Cylinder

r
d
each face is a square. Ab = r² = 0.25 d²
(i) Volume = a3 cubic units face are = Rs. 343.98 = 34398 r AL = 2 rh

Sni
(ii) Lateral surface Area paise V = r²h

Base (circle)

gv.i
= 4a2sq.units 34398 Properties of a Right Circu-
(iii) Total surface Area Total surface area =
13 lar Cylinder
= 6a2sq.units = 2646 cm² 1. The axis of a right circular

ridna
(iv) Diagonal of cube (d) 6a² = 2646 cylinder is the line joining the
a² = 441 centers of the bases.
= 3 a units
a = 21 cm 2. For any oblique or non-oblique
(v) Face diagonal of cube

eeYa
sections which do not pass any
 Volume of cube = a³
= 2 a units one base, the center of which
= (21)³ = 9261 cm³
is at the axis.
(vi) Volume of cube 5. A solid cube with an edge of
3. A right circular cylinder can be
10 cm is melted to form two equal
 su rface area 3
geisnh formed by revolving a rectangle
cubes. The edge of smaller
=   cubic units about one side as axis of revo-
 6 
 cube to the bigger cube is. lution.
Sol. Vo lume of larger cube = 4. Every section of a right circu-
EXAMPLES summati on o f vo lume of lar cylinder made by a cutting
Enak

smaller cubes plane containing two elements


1. Edge of a cube is 5 cm. Find:
 Let the edge of smaller cubes and parallel to the axis is a
(a) Volume be ‘a’ rectangle.
R

(b) Surface area


 (10)³ = (a)³ + (a)³ Fromulae for Right Circular
(c) Diagonal
 (10)³ = 2(a³) Cylinder
Sol. Volume = a³ = (5)³ = 125 cm³
aryn

10 Area of the base, Ab


Surface area = 6a² = 6 × (5)²  a= 1
2 3
Les B

= 150 cm2 A b  r ²
edge of smaller cube 10 
Diagonal = a 3 = 5 3 A b  d²
 edge of bigger cube = 3 2 4
= 8.660 = 8.66 cm
10 Lateral surface Area, AL
2. Three cubes of volume 1 cm³,
wa. th

1
216 cm³ and 512 cm³ are = 3 A L = 2rh
2
melted to form a new cube. A L = dh
What is the diagonal of the new Right Circular Cylinder
Volume, V
cube ?
wwM

A Right circular cylinder is a three-


Sol. Volume of new cube = 1 + 216 dimensional object with two congru- V = Ab h
+ 512 = 729 cm³ ent circles as parallel bases and a V = r²h
lateral surface consisting of a rect-
 Edge of new cube angle. Volume and surface area of 
V = d2 h
a Right Circular Cylinder. if ‘r’ is 4
= 3
729 = 9 cm the radius of a circular base of the Total surface Area, AT
 Surface area = 6a² cylinder and 'h' is the height of the
cylinder. Total surface area (open both ends),
= 6 × (9)² = 486 cm² Solid View AL = A
Wire-Frame
 Diagonal of the new cube Circular Base View Total surface Area (open one end), A
= Ab + AL
= a 3= 9 3 Lateral Surface h
Total surface Area (closed both
= 15.6 cm (approx) Circular Base r ends), A = 2Ab + AL

Rakesh Yadav Readers Publication Pvt. Ltd. 364

For More Visit : www.LearnEngineering.in


For More Visit : www.LearnEngineering.in

EXAMPLES 5. Th e radius of a wire is 9. The diameter of a cylindrical


decreased to one third. If tank is 24.5 metres and depth
1. Find the volume of an iron rod volume remains same, length 32 metres. How many metric
which is 7 cm long and whose will increase: tons of water will it hold ? (One
diameter is 1 cm. Sol. Let, radius = R and length = h, cubic metre of water weighs
Sol. Diameter = 1 cm. Volume = πR2h 1000 kg.)
1 1 Sol. Volume of the cylinder
Radius = cm New radius = R
2 3 22 24.5  24.5
Height = 7 cm Let, new length = H =   32
Volume of Cylinder =  r²h 2
7 22
1  = 15092 m³
22 1 1 11 Volume = π   R  × H
3

ir
=   7 Since 1 cubic metre = 1000 kg.
7 2 2 2 πR 2H

v.iSn
=  1 cubic metre = 1 metric ton
= 5.5 cubic cm 9
2. Water flows at 10 km per hour 1000kg 1metric ton
through a pipe with cross sec- πR 2H
 πR² h = or H = 9h  Volume of cylinder = 15092
tion a circle of radius 35 cm, 9

dnag
metric tones
into a cistern of dimensions 25 6. A cylindrical iron rod is 70 cm
m by 12 m by 10 m. By how long, and the diameter of its
Right Circular Cone
much will the water level rise end is 2 cm. What is its
in the cisten in 24 minutes ? weight, reckoning a cubic cm Co ne i s a three dimen sion al

eYari
Sol. Volume flown in 24 minutes of iron to weigh 10 grams ? geometric shape. If one end of a line
Sol. Volume of the iron rod = π r² h
is twisted about a second set line
22 while keeping the lines other end
22 35 35 10000  =  1  1  70 = 220 cm³
=  7 100 100  60  24 7

= 1540 cubic m
=
snhe
 weight of the cylinder
220  10
1000
= 2.2 kg.
fixed, we get a cone. The point about
which the line is curved is known
as the vertex and the base of the
cone is a circle. The vertex is
 1540 
kgei
Rise in level=  25  12  = 5.13 m 7. A cylinderical vessel, whose directly above the centre of the bot-
base is 14 dm in diameter tom.
3. A powder tin has a square base holds 2310 litres of water.
ERna

with side 8 cm and height 13 Taking a litre of water to oc-


cm. Another is cylindrical with Vertex
cupy 1000 cubic cm, what is
radius of its base 7 cm and the height of the vessel in dm? Semi Vertical
height 15 cm. Find the differ- θ Angle
ence in their capacities. 22 Height
 70  70  h = 2310 × 1000
aBryn

Sol.
Sol. Difference in capacities 7

22 1dm 10 cm Slant Height


=  7  7  15 – 8  8  13
7 2310  1000  7
 h=
= 2310 – 832 = 1478 cubic m 22  70  70
Les

Radius
4. A metallic sphere of radius 21 = 150 cm = 15 dm. Center
wa. th

cm is dropped into a 8. Find how man y pieces of


cylinderical vessel, which is Properties of a Cone
3
partially filled with water. The money cm in diameter and There are number of properties of a
4
diameter of the vessel is 1.68 cone. Some of them are as follows:
metres. If the sphere is com- 1
wwM

cm thick must be melted • Volume of a Cone


pletely submerged, find by how 8 • Lateral surface Area of a
much the surface of water will down to form a cube whose Cone
rise. edge is 3 cm long ? • Total Surface Area of a
4 Sol. Volume of one piece of money
Cone
Sol. Volume of sphere = πr ³ 22 3 3 1
3 = π r² h =    Definition of Right Circu-
7 8 8 8
 4 22  lar Cone
=  3  7  21  21  21 22 3 3 1
    n  3  3  3 A right circular cone is one whose
= 38808 cubic cm. 7 8 8 8 axis is perpendicular to the plane
22 3337888 of the base. We can generate a
  84  84  h = 38808 n =
7 22  3  3  1 right cone by revolving a right
 h = 1.75 cm n = 488.72 triangle about one of its legs.

Rakesh Yadav Readers Publication Pvt. Ltd. 365

For More Visit : www.LearnEngineering.in


For More Visit : www.LearnEngineering.in

Radius Where, Base area = π r²  Curved surface


1  22 
Volume of a Circular cone = =  7  7  25  = 550 cm²
3
×  ×Radius² × Height
4. A right cylindrical vessel is full
EXAMPLES with water. How many right
Slant Axis cones having same diameter
1. Find the volume of a cone and height as those of right
whose diameter of the base is cylinder will be needed to store
Vertex 21 cm and the slant height is that water ?
Formulae of Right Circular Cone 37.5 cm.
Sol. Volume of 1 cylinder = πr 2 h

r
2 2
For a right circular cone of radius r, Sol. h = 37.5 – 10.5 = 36 cm
1 2

Sni
height h and slant height l, we have Volume of 1 cone = πr h
1 2 3
Lateral surface area of a right circu-  volume = πr h

gv.i
lar cone =  rl 3 πr2 h
Total surface area of a right circular Number of cones = 1 =3
1 22 πr 2 h
cone =  (r + l)r =   10.5  10.5  36 3

ridna
3 7
Volume of a right circular cone= 5. A cylindrical piece of metal of
= 4158 cm³
1 radius 2 cm and height 6 cm
 r²h 2. The radius and height of a
3 right circular cone are in the is shaped into a cone of same

eeYa
Note:- ratio 5 : 12 and its volume is radius. The height of cone is:
Area is measured in square units 2512 cm³. Find the slant 1 2
and volume is measured in cubic Sol. Volume of Cone = πr h =
height, radius and curved sur- 3
units. face area of the cone. (Take
geisnh 1 2
Surface Area of a Right Circu- π = 3.14) π2 h
lar Cone:- Sol. Let radius = 5x, height = 12x 3
The surface area of a right circular Volume of cylinder = π × (2)² ×
cone is the sum of area of base and 6
Enak
1 2
lateral surface area of a cone. The  volume = πr h  Volume of cone = Volume of
3 cylinder
surface area is measured in terms
of square units. 1
R

1 22 h = 22 6
Surface area of a cone = Base Area  3.14  25x 2   12x  = 2512 3
+ Lateral surface Area of a cone 3  h = 18 cm
aryn

= πr 2 πrl = πr r l  x=2 6. From a solid right circular with


Surface Area of a Right Circular  Radius = 5x = 10 cm & height 10 cm and radius of the
Les B

Cone can be calculated by the Height = 12x = 24 cm base 6 cm, a right circular cone
following formula. 2 2 of the same height and base is
 l= r 2  h2 = 10  24
removed. Find the volume of
Area of a right circular cone
= r ( r +l ) = 26 cm the remaining solid.
Here, l= r2  h2
wa. th

Where, r = Radius  Curved surface area of cone Sol. Volume of remaining solid
h = Height and =  r l = 3.14 × 10 × 26 = 31.4 1 2
l = Slant height of cone × 26 = 816.4 cm² = πr 2 h – πr  h
3
Volume of a Right Circular 3. If a right circular cone of verti-
wwM

cal height 24 cm has a volume 22 1 22


Cone = × 6 × 6 × 10 – × ×
The volume of a cone is one third of of 1232 cm³, then the area of 7 3 7
the product of the area of base and its curved surface in cm² is: 6 × 6 × 10
the height of the cone. the volume 1 22 2
Sol.  Volume =  r2 h =  6  6  10 
of a right circular cone is measured 3 7 3
in terms of cubic units.
Volume of a right circular cone can 1 2 1 22 2 5280 2
 πr h =   r  24 = = 754 cm³
be calculated by the following for- 3 3 7 7 7
mula. = 1232 = r² = 49 = r = 7 7. The slant height of a conical
Volume of a right circular cone
Slant height = 24
2 2
 7  1
1 tomb is 17 metres. If its
=  Base area  height 2
3 = 25 cm

Rakesh Yadav Readers Publication Pvt. Ltd. 366

For More Visit : www.LearnEngineering.in


For More Visit : www.LearnEngineering.in

diameter be 28 metres, find the Lateral Area of a Trun-


cost of constructing it at Rs.
= 4 : 1  9 : 4 = 3 : 1
cated Cone
135 per cubic metre and also 11. If the heights and the curved
AL = π (R + r) l
find the cost of white-washing surface areas of two circular
its slant surface at Rs. 3.30 per cylinder are in the ratio 1 : 3 Surface Area of a Trun-
square metre. and 4 : 5 respectively. Find the cated Cone
Sol. Height of the cone ratio of their radii. AT = π [ l (R + r)+ R² +r² ]
2 Volume of a Truncated Cone
 35  21  1
= 
2
 – 14 = m Sol. Required ratio = (4 : 5) × 1 :  1
2   3
2 V= .π.h R 2  r2  R.r 
= (4 : 5) × (3 : 1) = 12 : 5 3
Volume of the cone

ir
1 22 21 Frustum of a right circular cone EXAMPLES

v.iSn
=   14  14  = 2156 m³
3 7 2 1. Calculate the lateral surface
A frustum of a cone or truncated
Cost of constructing the coni- cone is the result of cutting a cone area, surface area and volume
cal tomb = 2156 × 135 by a plane parallel to the base and of a truncated cone of radii 2

dnag
= Rs. 291060 removing the part containing the and 6 cm and height of 10 cm.
Curved surface area of the apex. Sol.
conical tomb = πr l Upper part
containing

eYari
22 35 the vertex
=  14  = 770 m³
7 2 10 cm l
 Cost of white washing = 770
× 3.30 = Rs. 2541
8. Radius of the base of a right
circular cone is 3 cm and the
height of the cone is 4 cm.
snhe Frustum of
a right cone
6 cm

Find the total surface area of l ² = 10² + (6 – 2)²


kgei
the cone. The height is the line segment that
2
Sol. Applying to the question, joins the two bases perpendicu- l = 102  6 – 2 = 10.77 cm
Total surface area= × r (l + r) larly.
ERna

AL = π (6 + 2)×10.77 = 270.78 cm²


=
22
7
3  2
4 3 3 2
 AT = 270.78 + π×6² + π×2² =396.35 cm²
1
V= π 10  6  2  6  2 =544.54 cm³
2 2

l  h2  r 2
 
aBryn

3
h l
2. Calculate the lateral surface
22  3  8 528 3 area, surface area and volume
= = = 75 sq. cm
7 7 7 of a truncated cone of radii 10
9. If the heights of two cones are R and 12 cm and a slant height
Les

in the ratio 1 : 4 and their of 15 cm.


The radii are of their bases are ‘r’
wa. th

diameters in the ratio 4 : 5


and ‘R’. Sol.
what is the ratio of their
volumes ? The slant height is the shortest
Sol. We have, possible distance between the
Ratio of Volumes = (4 : 5)² × (1 : edges of the two bases.
wwM

The slant height of the truncated h


15 cm
16 1 cone is obtained by applying the
4) =  Pythagoras theorem for the shaded
25 4
triangle:
= 4 : 25 l ² = h² + (R – r)²
[  ratio of diameters = ratio of radii] 2 12 cm
10. If the volumes of the two cones l= h2  R – r 
are in the ratio 4 : 1 and their AL = π (R + r) l
Unfold of a Truncated Cone
heights in the ratio 4 : 9, what =π(12 + 10)×15 = 1,036.73 cm²
is the ratio of their radii ? AT=1036.72 + π ×12² + π ×10²
= 1803.27 cm²
1 1
Sol. Ratio of radii = 4 : 1   :   l² = h² + (R – r)²
4 9 15² = h² + (12 – 10)²

Rakesh Yadav Readers Publication Pvt. Ltd. 367

For More Visit : www.LearnEngineering.in


For More Visit : www.LearnEngineering.in

(i) Surface area of a sphere = 4 3  1047.2


h = 152 – 22 = 14.866 cm
times the area of its g r e a t  h³ =
V = π
circle = 4πr2 = πd²  h³ = 1000
1
π 14.866 122  102  12  10 4  h = 10m
3 (ii) Volume of a sphere = πr 3
3 3. How many leaden ball of a
= 5,666.65 cm³
1
Sphere π 3 cm. in diameter can be cast
= d 4
6
A sphere is a solid bounded by a out of metal of a ball 3 cm in
closed surface every point of which (iii) For a spherical shell if R and r diameter supposing no waste.
is equidistant from a fixed point are outer and inner radii re-
Sol. Here, diameter of leaden ball

r
called the centre. Most familiar spectively, then the volume of
1

Sni
examples of a sphere are baseball, 4 = cm. = d1
a shell is = π(R 3 – r3 ) 4
tennis ball, bowl, and so forth. 3

gv.i
Terms such as radius, diameter, Diameter of metal ball
chord, and so forth, as applied to the π 3 = 3cm = d2
= (D – d3 )
sphere are defined in the same sense 6 Volume of leaden ball

ridna
as for the circle. 3
4  d1  π 3 π 1 
3
EXAMPLES =   = d =
Thus, a radius of a sphere is a    
3 2 6
1
6 4
straight line segment connecting its 1. The diameter of a sphere is
= 0.0082 cu. m.

eeYa
centre with any point on the sphere. 13.5m. Find its surface area
Obviously, all radii of the same and volume. π 3
sphere are equal. Volume of metal ball = d2
Sol. Here, d = 13.5 m 6
Diameter of the sphere is a straight
Surface area = 4πr2 = πd2 π 3
line drawn from the surface and
geisnh = 3 = 14.137 cu. m.
after passing through the centre = 6
ending at the surface. π(13.5)² = 572.56 sq. m.  Number of leaden ball =
The sphere may also be considered 3 4 π 3 14.137
as generated by the complete rota- Volume of sphere = πr = d
= 1728.00
3 6
Enak

0.0082
tion of a semicircle about a diameter.
π 4. A metal sphere of diameter 14
= (13.5)3 = 1288.25 cu. m.
6 cm is dropped into a cylindri-
R

2. Two spheres each of 10m di- cal vessel, which is partly filled
ameter are melted down and with water. The diameter of
the vessel is 1.68 metres. If
aryn

recast into a cone with a height


equal to the radius of its base, the sphere is completely sub-
merged, find by how much the
Les B

Find the height of the cone.


Sol. Here, d = 10m surface of water will rise.
Great and Small Circles: Sol. Radius of the sphere = 7 cm
 Radius of cone = height of
the cone (Given) Volume of sphere
Every section made by a plane
wa. th

passed through a sphere is a circle.  r=h  4 22 


=    7  7  7

If the plane passes through the cen- π 3 3 7
tre of a sphere, the plane section is Volume of sphere = (d)
6 1
a great circle; otherwise, the section = 1437 cu. m
3
wwM

is a small circle (Fig. 2). Clearly any π π


=  103 =  1000 Volume of water displaced by
plane through the centre of the 6 6
1
sphere contains a diameter. Hence, = 523.599 cu. m. sphere = 1437 cu cm.
all great circles of a sphere are Volume of two sphere 3
Let the water rise by h cm,
equals have for their common cen- = 1047.2 cu. m.
tre, the centre of the sphere and have Volume of the cone 22
Then,  28  28  h
for their radius, the radius of the = Volume of two spheres 7
sphere. 1
1 2 = 1437
Surface Area and Volume of a  πr h = 1047.2 3
3
Sphere: 4312  7 7
If ‘r’ is the radius and ‘d’ is the 1 or h = 
 πh3 = 1047.2 22  28  28  3 12
diameter of a great circle, then 3 = 0.58 cm

Rakesh Yadav Readers Publication Pvt. Ltd. 368

For More Visit : www.LearnEngineering.in


For More Visit : www.LearnEngineering.in

5. Find the weight of an iron r = 21 cm No Curves :


shell, the external and inter- A prism is a polyhedron, which
2
nal diameters of which are 13 Volume of hemisphere = r 3 means all faces are flat.
cm and 10 cm respectively, if 3
1 cubic cm of iron weighs 8 Volume of hemisphere
gms.  2 22 
Sol. Volume of iron shell =  3  7  21  21  21
3
4  13  3
 = 19404 cm³
= 3   2  – 5 
π
2. Find the curved surface area
 
of a hemisphere of radius 21

ir
4 22 1197 cm.
=   = 627 cu cm.
Sol. According to the question, We

v.iSn
3 7 8
have For example, a cylinder is not
Weight of iron shell = 627 × 8
Curved surface area = 2πr² prism, because it has curved
= 5016 gms. = 5.016 kg.
sides.
6. Find the surface area of a  22  Try drawing a shape on a piece

dnag
sph ere whose vo lume is =  2  7  21  21 cm²
of paper (using straight lines)
310464 cu cm.
= 2772 cm² Then imagine it extending up
4 3 from the sheet of paper ......
Sol. πr = 310464 3. A hemisphere of lead of radius

eYari
3 7 cm is cast into a right circu- that's a prism !
310464  3  7 lar cone of height 49 cm. Find
or r3 = = 74088 the radius of the base.
4  22
Sol. Volume of hemisphere
 r = 42 cm
 surface area = 4πr
22
2 1  4 22snhe 
= 2  3  7  7  7  7 
kgei
= 4  42  42
2156
7
= cubic cm
= 22176 sq. cm. 3
Let, the radius of the base of There are all Prism:
ERna

Hemisphere cone be ‘r’ cm. Square Cross-


A plane through the center of the 1 22 2 2156 Prism Section
Then,   r  49 =
sphere cuts it into two equal parts. 3 7 3
aBryn

Each part is called hemisphere.


2156  7  3
or r² = = 14
3  22  49
Cube Cross-Section
 Radius (r) = 14 = 3.74 cm
Les

Right Prism
wa. th

A prism is a solid object with:


(yes, a cube is a prism, because
2 3 • Identical ends it is a square all along its length)
1. Volume of hemispher = πr • Flat faces
3 Triangular Cross-
wwM

2. Curved surface area or Surface • Same cross section all Prism Section
along its length
area of hemisphere = 2πr2
The cross section of this ob-
3. Total surface area of solid
hemisphere ject is a triangle ..... it has the
same cross section all along its
= 2πr2 πr2  3πr2 length ... so it's a triangular Pentagonal Cross-
EXAMPLES prism. Prism Section

1. Find the volume of a hemisphere


of radius 21 cm.
Sol. According to the question, We
have

Rakesh Yadav Readers Publication Pvt. Ltd. 369

For More Visit : www.LearnEngineering.in


For More Visit : www.LearnEngineering.in

Regular and Irregular Prisms EXAMPLES regular pentagon. If the area


All the previous examples of the cross-section is 30 cm²,
1. What is the volume of this then what is the volume of the
are of Regular Prisms, because
prism ? prism ?
the cross section is regular (in
other words it is a shape with
9
equal edge lengths, and equal
angles.) 2
2 2 2
Here is an example of an Ir- 2 2 2 2
regular Prism: 2 2 2 5c
m
Irregular Pentagonal Prism: 12.
2
Sol. Volume = Base area × Height
Sol. Volume = Base area × height

r
= 30 × 12.5 = 375 cm³
There are 5 squares

Sni
5. The diagram shows a prism
 Base area = 5 × 2 × 2 = 20 whose corss-section is an equi-
Cross-Section

gv.i
Volume = 20 × 9 = 180 units. lateral triangle of lengths 10
It is "irregular" because the 2. The diagram show a prism cm. Given that its volume is
cross-section is not "regular" whose cross-section is a right 866 cm², what is the total sur-
triangle. What is the volume

ridna
in shape. face area of the prism ?
Surface Area of Prism of the prism ?

eeYa
cm

10
cm
10
4 in n
9i 10 cm
geisnh Sol. Volume = Base area × Height
3 in
Sol. Volume = base area × height. 3
Base area = × (10)2 = 43.3
1
4
Base area =  4  3 = 6 in² cm2
Enak

2
Height = Volume  Base area
Volume = 6 × 9 = 54 in³
Surface Area = 2 × Base Area 3. The diagram shows a prism 866
= = 20
R

+ Base Perimeter wh ose cross-se ctio n is a 43.3


× Length square. The length of the base C.S.A = Base perimeter × height
Volume of Prism of the prism is 19 cm and its
aryn

C.S.A = 3 × 10 × 20 = 600 cm²


volume is 1,539 cm³. T.S.A = C.S.A + 2 base area
Les B

What is the total surface area


T.S.A = 600 + 2 × 43.3
of the prism ?
= 600 + 86.6 = 686.6 cm²
6. There is a 10 cm long prism
whose cross-section is an isos-
wa. th

celes trapezoid:
19 cm

Sol. Volume = Base area × Height


1539 = Base area × 19
wwM

1539
Base area = = 81 6 cm 10 cm
19
The Volume of a prism is the Base area = a² = 81 5 cm 4 cm
product of its base area and a =9
length. 12 cm
C.S.A = Base perimeter × Height
Volume = Base Area × Length What is the total surface area
= 4 × 9 × 19 = 684 cm²
Volume = Base Area × Length of the prism ?
T.S.A = C.S.A + 2 Base area
Curved Surface Area = Base perim- = 684 + 2 × 81
eter × Height 1
= 684 + 162 = 846 cm² Sol. Base area = × Sum of 
Total Surface Area = CSA + 2 Base 2
4. The diagram shows a prism sides × height
Area
with a cross section that is a

Rakesh Yadav Readers Publication Pvt. Ltd. 370

For More Visit : www.LearnEngineering.in


For More Visit : www.LearnEngineering.in

1
= 1350 + 150×2.45= 1717.5 ft³ and they are named after the shape
= × (6 + 12) × 4 = 36 cm² 8. The diagram shows a prism of their base.
2
whose cross-section is an isos- 1. Triangular Pyramid
Base perimeter = 5 + 6 + 5 + celes trapezoid.
12 = 28 cm Pyramid Base
C.S.A. = Base perimeter × h
C.S.A. = 28 × 10 = 280 cm²
T.S.A. = C.S.A. + 2 Base area
T.S.A. = 280 + 2 × 36 = 352 cm² 6 cm 8 cm
7.
5 cm
A Notice these interesting things:

ir
7 ft 7 ft 10 cm • It has 4 Faces

v.iSn
B What is the volume of the • The 3 Side Faces are Tri-
10 ft
9 ft prism ? angles
15 ft Sol. Volume = Base area × Height • The Base is also a Triangle
D E • It has 4 Vertices (corner

dnag
The diagram shows a barn. 1
Base area = (S um of || points)
What is the volume of the 2
• It has 6 Edges
barn? sides) × height • It is also a Tetrahedron (if
(The length of the hypotenuse all triangles are equilateral
1

eYari
in the right triangle is rounded =  10  6  5
2 triangles)
to the nearest foot.) 2. Square Pyramid
Sol. Volume = Base area × height 1 Pyramid Base
=  16  5 = 40 cm²
Total base area = Area of 2
 BCED + Area of  ABC
Area of BCDE = 10 × 9 = 90 ft²
A
snhe
Volume = 40 × 8 = 320 cm³
Pyramids
kgei
Notice these interesting things:
• It has 5 Faces
7 7 • The 4 Side Faces are Tri-
ERna

angles
• The Base is a Square
• It has 5 Vertices (corner
C
B
N points)
aBryn

10 When we think of pyramids we think • It has 8 Edges


In  ABC, of the Great Pyramids of Egypt. 3. Pentagonal Pyramid
AB² = AN² + BN² They are actually Square Pyramids, Pyramid Base
because their base is a Square.
AN² = 49 – 25
AN² = 24
Les
wa. th

AN = 2 6 ft
 Area of  ABC Notice these interesting things:
1 • It has 6 Faces
=  BC  AN • Th e 5 Side Faces are
2
wwM

Triangles
1 • The Base is a Pentagon
=  10  2 6
2 Parts of Pyramid • It has 6 Vertices (corner
= 10 6 ft² Apex points)
• It has 10 Edges
 Total base area Right vs Oblique Pyramid:-
= 10 6  90  ft²
A pyramid is made by connecting a

Volume = 10 6  90  15  base to an apex. h
Types of Pyramid
= 1350 + 150 6
There are many types of Pyramids,

Rakesh Yadav Readers Publication Pvt. Ltd. 371

For More Visit : www.LearnEngineering.in


For More Visit : www.LearnEngineering.in

1
Sol. Volume = × Area of base ×
3
height
h 1
Volume =  42  7
3
Volume = 98 cm³
This tells us where the top (apex) of
2. The diagram shows a square-
the pyramid is. When the apex is
based pyramid with base
directly above the center of the base
lengths 6 in and height 8 in.
it is a Right Pyramid, otherwise it
What is the volume of the pyra- AB = 10 ft BC = 18 ft
is an Oblique Pyramid.

r
mid? VO = 12 ft VM = 15 ft
Regular vs Irregular Pyramid:-

Sni
This tells us about the shape VN = 13 ft
of the base. When the base is a regu- What is the total surface area

gv.i
lar polygon it is a Regular Pyramid, of the pyramid ?
otherwise it is an Irregular Pyramid. Sol. Total surface area = 2 × area

8 in

ridna
 ABV + 2 × area  VBC + Area
of ABCD
1 1
= 2   10  15  2   18  13  10  18

eeYa
2 2
= 150 +234 +180 = 564 ft²
5. The diagram shows a pyramid
Regular Irregular
Pyramid Pyramid 1 with a triangular base ABC.
geisnh
Sol. Volume × Area of base × The point D is vertically above
3
the point C. What is the vol-
height
ume of the pyramid ?
1
Volume =  6  6  8 = 96 in³
Base is Base is 3
Enak

Regular Irregular
3. Th e di agram shows a
rectangular-based pyramid
FORMULAE with base length 15 cm and
R

8 cm
(i) Volu me of pyramid width 8 cm. The height of the
1 pyramid is 20 cm. What is the
aryn

= × (area of base) × height 3 cm


3 volume of pyramid ?
Les B

(ii) Curved surface area


1 5 cm 4 cm
= × (perimeter of base) ×
2
20 cm

slant height
wa. th

(iii) Total surface area = curved Sol. Volume


8 cm 1
surface area + area of the
=  Area of base  height
base. 15 cm 3
wwM

EXAMPLES 1 1 1
Sol. Volume = × Area of base × =  348
1. The diagram shows a pyramid 3 3 2
whose base is a regular penta- Volume = 16 cm³
gon of area 42 cm² and whose 1
height =  15  8  20 Tetrahedron
height is 7 cm. What is the 3
volume of the pyramid? Volume = 800 cm³. Tetrahedron Facts
4. The diagram shows a pyramid
with vertex V and a rectangu-
7 cm lar base ABCD. M is the mid-
point of AB, N is the midpoint
42 cm²
of BC and O is the point at the
center of the base.

Rakesh Yadav Readers Publication Pvt. Ltd. 372

For More Visit : www.LearnEngineering.in


For More Visit : www.LearnEngineering.in

Notice these interesting things: Area of 4 triangles


(i) Volume (V) = π(r22  r12 )h
• It has 4 Faces
81  3
• Each face is an Equilateral = 4 = 81  3 (ii) Cu r ved su r face ar ea = 2 
4
Triangle (r1 + r2)h
• It has 6 Edges = 140.30
2. The length of one edge of a (iii) Total surface area = inner
• It has 4 Vertices (corner
points) and at each vertex regular tetrahedron is 9 units. curved surface area + Outer
3 edges meet What is its volume? curved surface area + area of
• It is one of the Platonic 2 Base an d To p =
3
Solids Sol. Volume =  Edge length
12 2πr1  r2   h  (r2  r1 )
The tetrahedron also has a beauti-

ir
ful and unique property ... all four 2 3 2 (iv) Thickness (t) = (r2 – r1)
=  9 =  729

v.iSn
vertices are the same distance from 12 12
each other! = 85.91 units³ EXAMPLES
And it is the only Platonic Solid with 3. The total length of the edges
no parallel faces. of a tetrahedron is 24 cm. 1. A hollow garden roller 63cm

dnag
When we say "tetrahedron" we often What is its surface area ? wide with a girth of 440cm is
mean "regular tetrahedron" (in other Sol. Note:- In tetrahedron has 6 made of iron 4cm thick. The
words all faces are the same size and edges. volume of iron is:
shape)

eYari
One edge length = 24 ÷ 6 Sol. Circumference = 440 cm
2
Surface Area = 3 × EdgeLength = 4 cm
 2πr = 440
Surface area
2 3
Volume = × Edge Length  = 3  Edge length
2
 440 
12

EXAMPLES
=
snhe
2
3  4 = 16 × 1.732
= 27.71 cm²
r =  2  22  7  = 70 cm

Inner radius = 70 – 4 = 66 cm
kgei
1. The length of one edge of a 4. The total length of the edges Volume of Iron
regular tetrahedron is 9 units. of a tetrahedron is 36cm. What 2 2
What is its surface area? is its volume? = π70  – 66   63
ERna

Sol. Surface area Sol. A tetrahedron has 6 edges


= 3  Edge length
2
So one Edge Length = 36cm ÷ 22
=  136  4  63 = 58752 cm³
6 = 6cm 7
2
= 3  9 = 81 × 1.732
aBryn

2 3 2. A hollow cylindrical tube open


= 140.30 Volume =  Edge length
12 at both ends is made of iron 2
Alternate:-
2 3 2 cm thick. If the e xternal
Calculate the area of a side, =  6 =  216 = 25.46 cm³
which is an equilateral tri- 12 12 diameter be 50 cm and the
Les

angle. length of tube be 140 cm, find


The base is 9, the height is
Hollow Cylinder the volume of iron in it.
wa. th

3 9 3 Sol. External diameter = 50 cm


= 9 =
2 2 50
 External radius = 2 = 25 cm
wwM

1
Area of triangle = bh
2
r1 22
1 3 Volume =  140 [(25)2 – (23)2]
= 99 h 7
2 2 r2
22
81 3 =  140  48  2 = 42240 cm³
= 7
4

Rakesh Yadav Readers Publication Pvt. Ltd. 373

For More Visit : www.LearnEngineering.in


For More Visit : www.LearnEngineering.in

EXERCISE

1. If diagonal of a cube is 12 cm, 10. The are a of thr ee adjac ent (a) 5 2 cm (b) 5 cm
faces of a cuboid are x, y, z
then its volume in cm3 is : square units respectively. If (c) 6 cm (d) 2 5 cm
(a) 8 (b) 12 (d) 3 2
(c) 24 the volume of the cuboid be v
18. The area of the four walls of a
cube units. then the correct
2. How m any c ub e s , e ac h of room is 660 m2 and its length
relation between v,x, y, z is
edge 3 cm, can be cut from a is twice of its breadth. If the
(a) v2 = xyz (b) v3 =xyz height of the room is 11 m,
cube of edge 15 cm ?
(c) v = x y z (d) v3 =x2y2z2
2 3 3 3
then area of its floor (in m2) is
(a) 25 (b) 27 (c) 125 (d) 144 11. The largest sphere is carved

r
3. What is the volume of a cube (a) 120 (b) 150
out of a cube of side 7 cm. The (c) 200 (d) 330

i
(in cubic cm) whose diagonal

.iSn
volume of the sphere (in cm 3) 19. If the length of the diagonal of
measures 4 3 cm? will be
(a) 16 (b) 27 (c) 64 (d) 8 (a) 718.66 (b) 543.72 a cube is 8 3 cm, then its to-
4. A cuboidal water tank has 216 (c) 481.34 (d) 179.67 tal surface area is

agv
1 12. The length (in meters) of the (a) 192 cm2 (b) 512 cm2
litres of water. Its depth is (c) 768 cm 2
(d) 384 cm2
3 longest rod that can be put in a

ridn
1 room of dimensions 10 m× 10 20. The maximum length of a
of its length and breadth is m × 5 m is pencil that can be kept in a
2
rectangular box of dimensions

eeYa
1 (a) 15 3 (b) 15
of of the difference of length 8cm × 6cm × 2cm is
3 (c) 10 2 (d) 5 3
and breadth. The length of the (a) 2 13 cm (b) 2 14 cm
tank is 13. A rectangular sheet of metal is
(c) 2 26 cm (d) 10 2 cm
(a) 72 dm (b) 18 dm 40 cm by 15 cm . equal squares
geisnh 21. The volume of a cubical box is
(d) 6 dm (d) 2 dm of side 4cm are cut off at the
5. The volume of cuboid is twice the corners and the remainder is 3.375 cubic metres. The length
volume of a cube. If the dimen- folded up to form an op en of edge of the box is
sions of the cuboid are 9 cm, 8 cm rectangular box The volume of (a) 75 m (b) 1.5 m
(c) 1.125 m (d) 2.5 m
Enak
and 6 cm, the total surface area of the box is
the cube is: (a) 896 cm3 (b) 986 cm3 22. Two cubes of sides 6 cm each
(a) 72 cm2 (b) 216 cm2 (c) 600 cm 3
(d) 916 cm3 are kept side by side to form a
(c) 432 cm2 (d) 108 cm2 14. The areas of three consecutive rectangular parallelopiped. The
R

6. The length, breadth and height faces of a cuboid are 12 cm 2, area (in sq. cm) of the whole sur-
of a room is 5m, 4 m and 3m then the volume (in cm3) of the face of the rectangular
aryn

respectively. Find the length of cuboid is parallelopiped is


the largest bamboo that can be (a) 3600 (b) 100 (a) 432 (b) 360
kept inside the room.
Les B

(c) 396 (d) 340


(c) 80 (d) 24 3
(a) 5 m (b) 60 m 23. 2 cm of rain has fallen on a
(c) 7 m (d) 5 2 m 15. The length of the longest rod square km of land. Assuming
that can be placed in a room that 50% of the raindrops could
7. A wooden box measures 20 cm
which is 12 m long, 9 m broad have been collected and con-
by 12 cm by 10 cm . Thickness
wa.th

and 8 m high is tained in a pool having a 100


of wood is 1 cm. Volume of wood
to make the box ( in cubic cm) is (a) 27m (b) 19m m× 10 m base, by what level
(c) 17m (d) 13m would the water level in the pool
(a) 960 (b) 519
(c) 2400 (d) 1120 16. The floor of a room is of size 4 have increased ?
wwM

8. A cuboidal block of 6 cm × 9 cm m × 3 m and its height is 3 m. (a) 1 km (b) 10 m


× 12 cm is cut up into exact The walls and celling of the (c) 10 cm (d) 1 m
number of equal cube. The room require painting. The 24. A parallelopiped whose sides
least posible number of cubes area to be painted is are in ratio 2 : 4 :8 have the
will be (a) 66 m2 (b) 54 m2 same volume as a cube. The
2
(a) 6 (b) 9 (c) 24 (d) 30 (c) 42 m (d) 33 m2 ratio of their surface area is:
9. A cistern of capacity 8000 litres 17. If the sum of three dimensions (a) 7 : 5 (b) 4 : 3
measures externally 3.3 m by and the total surface area of a (c) 8 : 5 (d) 7 :6
2.6 m by 1.1 m and its walls are rectangular box are 12 cm and 25. If two adjacent sid es of a
5 cm thick. The thickness of 94 cm 2 respectively, then the rectangular parallelopiped are
the bottom is: maximum length of a stick 1 cm and 2 cm and the total
(a) 1 m (b) 13.5 m that can be placed inside the surfac e ar ea of the
(c) 1 dm (d) 90 cm box is parallelopiped is 22 square cm,

Rakesh Yadav Readers Publication Pvt. Ltd. 374

For More Visit : www.LearnEngineering.in


For More Visit : www.LearnEngineering.in

then the d iagonal of the 33. Three cubes of sides 6 cm, 8 cm 41. A cuboidal shaped water tank,
parallelopiped is and 1 cm are melted to form a 2.1 m long and 1.5 m broad is half
(a) new cube. The surface area of filled with water. If 630 litres
10 cm (b) 2 3 cm
the new cube is more water is poured into tank,
(c) 14 cm (d) 4 cm (a) 486 cm2 (b) 496 cm2 the water level will rise
26. If the sum of the length, (c) 586 cm 2
(d) 658 cm2 (a) 2 cm (b) 0.15 cm
Br eadth and height of a 34. Some bricks are arranged in (c) 0.20 m (d) 0.18 cm
rectangular parallelopiped is 24 an area measuring 20 cu.m. If 42. A solid cuboid of dimensions 8
cm and the length of its diago- the length, breadth and height cm × 4 cm × 2 cm is melted and
nal is 15 cm, then its total of each brick is 25 cm, 12.5 cm cast into identical cubes of edge
surface area is and 8 cm respectively, then the 2 cm. Number of such identical
cubes is.

ir
(a) 256 cm2 (b) 265 cm2 number of bricks are (suppose
(c) 315 cm 2
(d) 351 cm2 there is no gap in between two (a) 16 (b) 4 (c) 10 (d) 8

v.iSn
27. If the total surface area of a bricks) 43. A metallic hem isphere is
cube is 96 cm2, its volume is (a) 6,000 (b) 8,000 melted and recast in the shape
(a) 56 cm3 (b) 16 cm3 (c) 4,000 (d) 10,000 of cone with the same base
(c) 64 cm 3
(d) 36 cm3 35. The whole surface of a cube is radius (R) as that of the hemi-

dnag
28. The length of the large st 150 sq. cm. Then the volume sphere. If H is the height of the
of the cube is cone, then:
possible rod that can be placed
in a cubical room is 35 3 m. (a) 125 cm3 (b) 216 cm3 2
(c) 343 cm 3
(d) 512 cm3 (a) H = 2 R (b) H = R

eYrai
The surface area of the largest 3
36. The ratio of the length and
possible sphere that fit within (c) H = 3R (d) B = 3R
br eadth of a r ectangular
the cub ical room (asuming 44. If the radius of a sphere is in-
parallelopiped is 5 : 3 and its
22 height is 6 cm. If the total sur- creased by 2 cm, its surface

7
(a) 3,500
(c) 2,450
) (in sq. m) is

(b) 3,850
(d) 4,250
snhe
face area of the parallelopiped
be 558 sq. cm, then its length
in dm is
area increased by 352 cm2. The
radius of sphere before change is
:
(a) 3 cm (b) 4 cm
kgei
(a) 9 (b) 1.5 (c) 10 (d) 15
29. The volume of air in a room is (c) 5 cm (d) 6 cm
204 m3. The height of the room 37. If the sum of the dimensions of
a rectangular parallelopiped is 24 45. The height of a conical tank is
is 6 m. What is the floor area of
cm and the length of the diago- 60 cm and the diameter of its
ERna

the room?
nal is 15 cm, then the total bas e is 64c m. The c ost of
(a) 32 m2 (b) 46 m2 painting it from outside at the
2 surface area of it is
(c) 44 m (d) 34 m2 rate of Rs. 35 per sq. m. is :
(a) 420 cm2 (b) 275 cm2
30. A square of side 3 cm is cut off 2 (a) Rs. 52.00 approx,
(c) 351 cm (d) 378 cm2
aBryn

from each corner of a rectan- (b) Rs. 39.20 approx,


gular sheet of length 24 cm and 38. The length, breadth and height
of a cuboid are in the ratio 3 : 4 (c) Rs. 35.20 approx,
breadth 18 cm and the remain- (d) Rs. 23.94 approx,
ing sheet is folded to form an : 6 and its volume is 576 cm3.
The whole surface area of the 46. A solid metallic cone of height
open rectangular box. The sur- 10 cm, radius of base 20 cm is
face area of the box is cuboid is
Les

melted to make spherical balls


(a) 468 cm2 (b) 396 cm2 (a) 216 cm2 (b) 324 cm2
each of 4 cm diameter. How
wa.th

2
(c) 615 cm 2
(d) 423 cm2 (c) 432 cm (d) 460 cm2 many such balls can be made?
31. Three solid iron cubes of edges 39. If the number of vertices, edges (a) 25 (b) 75 (c) 50 (d) 125
4 cm, 5 cm and 6 cm are melted and fac es of a rectangular 47. A cylindrical tank of diameter
together to make a new cube. parallelopiped are denoted by v, 35 cm is full of water. If 11 litres
wwM

62 cm3 of the melted material e and f respectively, the value of water is drawn off, the water
is lost due to improper handing. of (v – e + f) is level in the tank will drop by :
The area (in cm2) of the whole (a) 4 (b) 1 (c) 0 (d) 2 1 6
surface of the newly formed 40. A low land, 48 m long and 31.5m (a) 10 cm (b) 12 cm
2 7
cube is broad is raised to 6.5 dm. For
3
(a) 294(b) 343 (c) 125 (d) 216 this, earth is removed from a (c) 14 cm (d) 11 cm
32. Area of the floor of a cubical cuboidal hole, 27 m long and 7
room is 48 sq. m. The length of 18.2 m broad, dug by the side of 48. The volume of a right circular
cylinder whose height is 40cm,
the longest rod that can be kept the land. The depth of the hole and circumference of its base
in that room is will be. is 66 cm is:
(a) 9 metre (b) 12 metre (a) 3 m (b) 2 m (a) 55440 cm3 (b) 3465 cm3
(c) 18 metre (d) 6 metre (c) 2.2 m (d) 2.5 m (c) 7720 cm3 (d) 13860 cm3

Rakesh Yadav Readers Publication Pvt. Ltd. 375

For More Visit : www.LearnEngineering.in


For More Visit : www.LearnEngineering.in

49. The circumference of the base 56. A hollow cylinderical tube 20 is 1 cm and iron weights 8 g/
of a circular cylinder is 6 cm. cm long. is made of iron and its cm3, then the weight of the pipe
The height of the cylinder is external and internal diam- 22
is ( Take  ):
equal to the diameter of the eters are 8 cm and 6 cm respec- 7

base. How many litres of wa- tively. The volume of iron used (a) 3.696 kg (b) 3.6 kg
ter can it hold ? 22 (c) 36 kg (d) 36.9 kg
in making the tube is (  )
7 64. The volume of a right circular
(a) 54 cc (b) 36 cc (a) 1760 cu.cm (b) 880 cu.cm. cylinder, 14 cm in height, is
(c) 0.054 cc (d) 0.54 cc (c) 440 cu.cm (d) 220 cu.cm equal to that of a cube whose
50. The volume of a right circular 57. A sphere of radius 2 cm is put 22
into water contained in a cyl- edge is 11 cm Take   the
cylinder is equal to the volume 7
inder of base- radius 4 cm. If radius of the base of the cylin-

r
of that right cir cular cone
whose height is 108 cm and the sphere is completely im- der is

i
.iSn
diameter of base is 30 cm. If the mersed in the water, the water (a) 5.2 cm (b) 5.5 cm
height of the cylinder is 9 cm, level in the cylinder rise by (c) 11.0 cm (d) 22.0 cm
the diameter of its base is 1 1 65. If the volume of a right circu-
(a) cm (b) cm

agv
(a) 30 cm (b) 60 cm 3 2 lar cylinder is 9h m3, where h
(c) 50 cm (d) 40 cm 2 is its height (in metres) then the
(c) cm (d) 2 cm

ridn
51. Three solid metallic spheres of 3 diameter of the base of the cyl-
diameter 6 cm, 8 cm and 10 cm 58. A solid metallic spherical ball inder is equal to
are melted and recast into a of diameter 6 cm is melted and (a) 3 m (b) 6 m (c) 9 m (d) 12 m

eeYa
new soild sphere. The diameter recasted into a cone with diam- 66. Each of the measure of the ra-
of the new sphere is : eter of the base as 12 cm. The dius of base of a cone and that
(a) 4 cm (b) 6 cm height of the cone is of a sphere is 8 cm. Also, the
(a) 6 cm (b) 2 cm volume of these two solids are
(c) 8 cm (d) 12 cm
52. Find the volume of a prism
geisnh
(c) 4 cm (d) 3 cm equal. the slant height of the
59. The volume of a right circular cone is
which is based on a regular
cone is 1232 cm3 and its verti-
Hexagon & of height 10cm. If
cal height is 24 cm . Its curved (a) 8 17 cm (b) 4 17 cm
total S.A. is 156 3 cm2. surface area is (c) 34 2 cm (d) 34 cm
Enak

(a) 154 cm2 (b) 550 cm2 67. A well 20 m in diameter is dug
(a) 60 3 (b) 180 3 2
(c) 604 cm (d) 704 cm2 14 m deep and the earth taken
(c) 120 3 (d) 240 3 60. The volume of a sphere is out is spread all around it to a
R

53. Three solid spheres of a metal 88 3 width of 5 m to form an em-


 14  cm3 The curved sur-
whose radii are 1 cm, 6 cm and 21 bankment. The height of the
aryn

8 cm are melted to form an face area of the sphere is embankment is:


other solid sphere. The radius 22
(a) 10 m (b) 11 m
Les B

of this new sphere is ( Take   ) (c) 11.2 m (d) 11.5 m


7
(a) 10.5 cm (d) 9.5 cm (a) 2424 cm 2
(b) 2446 cm2 68. The diameter of the iron ball
(c) 10 cm (d) 9 cm (c) 2484 cm 2
(d) 2464 cm2 used for the shot-put game is
54. The slant height of a conical 61. The surface area of a sphere is 14 cm. It is melted and then a
wa.th

2
mountain is 2.5 km and the 6 4 cm Its diameter is equal to 1
solid cylinder of height 2
3
cm
area of its base is 1.54 km 2. (a) 16 cm (b) 8 cm
then find the height of coni- (c) 4 cm (d) 2 cm is made. What will be the diam-
cal mountain. 62. The diameter of the base of a eter of the base of the cylinder?
cylinderical drum is 35 dm. and (a) 14 cm (b) 28 cm
wwM

(a) 2.2 km (b) 2.4 km


(c) 3 km (d) 3.11 km the height is 24 dm. It is full of
14 28
kerosene. How many tins each (c) cm (d) cm
55. The base of a conical tent is 3 3
of size 25 cm × 22 cm × 35 cm
19.2 metres in diameter and 69. The sum of radii of two spheres
can be filled with kerosene from
the height is 2.8 metres . The the drum ? is 10 cm and the sum of their
area of the canvas required to volume is 880 cm3. What will be
22
put up such a tent ( in square (use   ) the product of their radii?
7
22 (a) 1200 (b) 1020
meters) (taking  ) is 1
7
(c) 600 (d) 120 (a) 21 (b) 26
nearly. 3
63. A hollow iron pipe is 21 cm long
(a) 3017.1 (b) 3170 and its exterior diameter is 8 1
(c) 301.7 (d) 30.17 (c) 33 (d) 70
cm. If the thickness of the pipe 3

Rakesh Yadav Readers Publication Pvt. Ltd. 376

For More Visit : www.LearnEngineering.in


For More Visit : www.LearnEngineering.in

70. A rectangular paper sheet of 78. If the area of the base of a cone 22
dimensions 22 cm × 12 cm is is 770 cm2 and the area of its ( Take   )
7
folded in the form of a cylinder curved surface is 814 cm². then (a) 1 cm (b) 5.2 cm
along its length. What will be find its volume. (c) 2.3 cm (d) 3.7 cm
the volume of this cylinder? (a) 2 1 3 5 cm³ (b) 392 5 cm³ 85. Two iron sphere each of
22 diameter 6 cm are immersed
(Take   ) (c) 5 5 0 5 cm³ (d) 6 1 6 5 cm³ in the water contained in a
7
(a) 460 cm 3
(b) 462 cm 3 79. The size of a rectangular piece cylinerical vessel of radius 6 cm.
of paper is 100 cm × 44 cm. A The level of the water in the
(c) 624 cm3 (d) 400 cm3
cylinder is formed by rolling the vessel will be raised by
71. A copper rod of 1 cm diameter (a) 1 cm (b) 2 cm
paper along its breadth. The
and 8 cm length is drawn into (c) 3 cm (d) 6 cm
volume of the c ylinder is

ir
a wire of uniform diameter and 86. The height of the cone is 30 cm.
18 m length. The radius ( in cm) 22

v.iSn
(Use  ) A small cone is cut off at the
of the wire is 7
top by a plane parallel to its
(a) 4400 cm3 (b) 15400 cm3
1 1 2 1
(a) (b) (c) (d) 15 (c) 35000 cm3 (d) 144 cm3 base. If its volume is of the
15 30 15 27
80. The radius of the base and

dnag
72. 12 spheres of the same size volume of the cone. at what
height of a metallic soild
are made by melting a solid cyl- height above the base, is the
cylinder are r cm and 6 cm section made ?
inder of 16 cm diameter and 2
respectively. It is melted and (a) 6 cm (b) 8 cm
cm height. The diameter of
recast into a solid cone of the

eYrai
each sphere is : (c) 10 cm (d) 20 cm
same r adius of bas e. The 87. The total surface area of a solid
(a) 2 cm (b) 4 cm
height of the cone is: hemisphere is 108 cm2. The
(c) 3 cm (d) 3 cm
(a) 54 cm (b) 27 cm volume of the hemisphere is
73. When the circumference of a toy (c) 18 cm (d) 9 cm (a) 72 cm3 (b) 144 cm3
ballon is increased from 20 cm to
25 cm its radius ( in cm) is in-
creased by :
snhe
81. The total surface ar ea of a
metallic hemisphere is 1848
cm2. The hemisphere is melted
(c) 108 6 cm (d) 54 6 cm3
3

88. A solid metallic sphere of radius


3 decimetres is melted to form a
kgei
5 5  to form a solid right circular circular sheet of 1 milimetre
(a) 5 (b) (c) 2 (d)
 5 cone. If the radius of the base thickness. The diameter of the
74. If the volume and surface area of the cone is the same as the sheet so formed is
radius of the hemisphere its (a) 26 metres (b) 24 metres
ERna

of a sphere are numerically the


same, then its radius is height is (c) 12 metres (d) 6 metres
(a) 42 cm (b) 26 cm 89. Water flows through a
(a) 1 unit (b) 2 units
(c) 28 cm (d) 30 cm cylinderical pipe. whose radius
(c) 3 units (d) 4 units is 7 cm, at 5 metre per second.
82. A right circular cylinder is
aBryn

75. In a right circular cone, the ra- The time, it takes to fill an empty
formed by rolling a rectangular
dius of its base is 7 cm and its water tank with height 1.54
paper 12 cm long and 3 cm wide metres and area of the base
height 24 cm. A cross- section is
along its length. The radius of (3 × 5) square metres, is
made through the midpoint of the
the base of the cylinder will be 22
height parallel to the base. The ( take   )
Les

3 6 7
volume of the upper portion is (a) cm (b) cm
2  (a) 6 minutes (b) 5 minutes
wa.th

(a) 169 cm3 (b) 154 cm3 (c) 10 minutes (d) 9 minutes
9
(c) 1078 cm 3
(d) 800 cm3 (c) cm (d) 2 cm 90. If S denotes the ar ea of the
2
76. Some solid metallic right cir- 83. What part of a ditch, 48 metres c u r v e d s u r f ac e o f a r i g h t
cular cones. each with radius long. 16.5 metres broad and 4 circular cone of height h
wwM

of the base 3 cm and height 4 metres deep can be filled by the a n d s e m i v e r ti c a l a n g le 


cm, are melted to form a solid ear th g ot b y digging a then S e quals
sphere of radius 6 cm. The num- cylinderical tunnel of diameter (a) h 2 tan2
ber of right circular cones is 4 m etre s and le ngth 56
1
(a) 12 (b) 24 (c) 48 (d) 6 metres? (Use  
22
)
(b) h 2 tan2
3
7
77. A right circular cylinder of (c) h sec tan
2

1 2 7 8
height 16 cm is covered by a (a) (b) (c) (d) 1
9 9 9 9
rectangular tin foil of size 16 cm (d) 2
sec tan
84. The volume of the metal of 3 h
× 22 cm, The volume of the cyl- cylinderical pipe is 748 cm 3. 91. The height and the radius of the
inder is The length of the pipe is 14 cm base of a right circular cone are
(a) 352 cm3 (b) 308 cm3 and its external radius is 9 cm. 12 cm and 6 cm respectively.
(c) 616 cm 3
(d) 176 cm3 its thickness is The rad ius of the c ircular

Rakesh Yadav Readers Publication Pvt. Ltd. 377

For More Visit : www.LearnEngineering.in


For More Visit : www.LearnEngineering.in

cross-section of the cone cut by 99. The diameter of the base of a 106. A s olid cylinde r has total
a plane parallel to its base at a right circular cone is 4 cm and surface area of 462 sq. cm. Its
distance of 3 cm from the base is its height 2 3 cm. The slant curved surface area is one third
(a) 4 cm (b) 5.5 cm height of the cone is of the total surface area. Then
(c) 4.5 cm (d) 3.5 cm (a) 5 cm (b) 4 cm the radius of the cylinder is
92. If S1 and S2 be the surface ar- (a) 7 cm (b) 3.5 cm
(c) 2 3 (d) 3 cm
eas of a sphere and the curved (c) 9 cm (d) 11 cm
surface area of the circum- 100. The rain water from a roof 22 107. The diameter of a cylinder is 7
scribed cylinder respectively, m × 20 m dr ains into a cm and its height is 16 cm. Us-
then S1 is equal to cylinderical vessel having a di-
22
3 1 ameter of 2 m and height 3.5 ing the value of   , the lat-
(a) S2 (b) S2 m, If the vessel is just full, then 7

r
4 2
2 the rainfall (in cm) is : eral surface area of the cylin-
(c) S2 (d) S2

i
.iSn
3 (a) 2 (b) 2.5 (c) 3 (d) 4.5 der is
93. The volume of a right circular 101. From a solid cylinder of height (a) 352 cm2 (b) 350 cm2
cylinder and that of a sphere 10 cm and radius of the base 6 (c) 355 cm 2
(d) 348 cm2
are equal and their radii are also cm, a cone of same height and 108.The height of a solid right cir-

agv
equal. If the height of the cyl- same base is removed. The vol- cular cylinder is 6 metres and
inder be h and the diameter of three times the sum of the area
ume of the remaining solid is :
the sphere d. then which of the

ridn
(a) 240 cu. cm of its two end faces is twice the
following relation is correct? area of its curved surface, The
(a) h =d (b) 2h =d (b) 5280 cu. cm
radius of its base (in meter) is

eeYa
(c) 2h = 3d (d) 3h =2d (c) 620  cu. cm
(a) 4 (b) 2 (c) 8 (d) 10
94. Water is being pumped out (d) 360 cu. cm 109. A semi-circular sheet of metal
through a circular pipe whose 102. Two solid right cones of equal of diameter 28 cm is bent into
internal diameter is 7cm. If the height and of radii r1 and r2 are an open conical cup. The depth
melted and made to form a solid
flow of water is 12 cm p er
geisnh of the cup is approximately
second, how many litres of sphere of radius R. Then the (a) 11 cm (b) 12 cm
water is being pumped out in height of the cone is (c) 13 cm (d) 14 cm
one hour? 110.A right angled sector of radius r
4R 2 R3 cm is rolled up into a cone in
(a) 1663.2 (b) 1500 (a) (b)
Enak

(c) 1747.6 (d) 2000 r12r22 r12r22 such a way that the two bind-
95. The lateral surface area of a ing radii are joined together .
cylinder is 1056 cm 2 and its 4R 3 R2 then the curved surface area
R

(c) (d) of the cone is


height is 16cm. Find its vol- r12
 r22 r12r22
ume. 103. The ratio of height and the di- r 2
(a) r 2 cm 2 (b) cm 2
aryn

(a) 4545 cm3 (b) 4455 cm3 ameter of a right circular cone 4
(c) 5445 cm3 (d) 5544 cm3 is 3 : 2 and its volume is 1078 r 2 2
Les B

96. The radius of the base and 22 (c) cm 2 (d) 2r cm 2


height of a right circular cone cc, then (taking   ) its 2
7 111.The radius of the base of a
are in the ratio 5 : 12. If the height is : conical tent is 16 metre. If
volume of the cone is 314
2 (a) 7 cm (b) 14 cm
7
3
wa.th

(c) 21 cm (d) 28 cm 427 sq. metre canvas is re-


3
cm , the slant height (in cm) of 104. From a right circular cylinder 7
the cone will be of radius 10 cm and height 21 quired to construct the tent,
(a) 12 (b) 13 (c) 15 (d) 17 cm. a right circular cone of then the slant height of the tent
97. A solid metallic cone is melted same base radius is removed. 22
wwM

If the volume of the remaining is : ( take   )


and recast into a solid cylinder 7
of the same base as that of the portion is 4400 cm 3, then the (a) 17 metre (b) 15 metre
cone. If the height of the cylin- height of the removed cone (
(c) 19 metre (d) 8.5 metre
der is 7 cm, the height of the 22 112. A circus tent is cylinderical up
cone was take   ) is : to a height of 3 m and conical
7
above it. If its diameter is 105m
(a) 20 cm (b) 21 cm (a) 15 cm (b) 18 cm and the slant height of the coni-
(c) 28 cm (d) 24 cm (c) 21 cm (d) 24 cm cal part is 63 m, then the total
98. A copper wire of length 36m and 105. A child reshapes a cone made area of the canvas required to
diameter 2mm is melted to up of clay of height 24 cm and 22
form a sphere. The radius of the radius 6 cm into a sphere. The make the tent is ( take   )
7
sphere (in cm) is radius (in cm) of the sphere is (a) 11385 m 2
(b) 10395 m 2

(a) 2.5 (b) 3 (c) 3.5 (d) 4 (a) 6 (b) 12 (c) 24 (d) 48 (c) 9900 m2 (d) 990 m2

Rakesh Yadav Readers Publication Pvt. Ltd. 378

For More Visit : www.LearnEngineering.in


For More Visit : www.LearnEngineering.in

113. A toy is in the form of a cone 119. The radius of the base of a th e n t he v ol um e of w at e r
mounte d on a hemis pher e. right circular cone is doubled needed to fill the empty space
The radius of the hemisphere keeping its height fixed. The is
and that of the cone is 3 cm and volume of the cone will be : (a) 3 cm3 (b) 18 cm3
height of the cone is 4 cm. The (a) Three times of the previous
total surface area of the toy volume (c) 54 cm3 (d) 81 cm3
22 (b) four times of the previous 126.A cylindrical can whose base
(takeing   ) is volume is horizontal and is of internal
7
r ad ius 3 . 5 c m c ontains
(a) 75.43 sq. cm, (c) 2 times of the previous sufficient water so that when
(b) 103.71 sq. cm, volume a s olid s p he r e is p lac e d
(c) 85.35 sq. cm, (d) double of the p ervious

ir
inside, water just covers the
(d) 120.71 sq. cm, volume sphere. The sphere fits in the

v.iSn
114. Marbles of diameter 1.4 cm are 120.The base of a right circular c an e xac tly . The d e p th of
dropped into a cylinderical bea- cone has the radius 'a' which
ker containing some water and water in the can before the
is same as that of a sphere. sphere was put, is
fully submerged. The diameter Both the sphere and the cone
of the beaker is 7 cm. Find how

dnag
have the same volume. Height 35 17
many marble s have b een (a) cm (b) cm
of the cone is 3 3
dropped in it if the water rises
(a) 3a (b) 4a
by 5.6 cm ? 7 14
(a) 50 (b) 150 (c) cm (d) cm

eYrai
7 7 3 3
(c) 250 (d) 350 (c) a (d) a
4 3 127.The radius and height of a
115.A cylindrical rod of iron whose cylinder are in the ratio 5 : 7
121. The circumference of the base
height is eight times its ra- and its volume is 550 cm 3 .
of a 16 cm high solid cone is
dius is melted and cast into Calculate it curved surface
spherical balls each of half
the radius of the c ylinde r.
The number of such spheri- (a) 1028
snhe
33 cm What is the volume of
the cone in cm3 ?
(b) 616
area in sq. cm.
(a) 110
(c) 220
(b) 444
(d) 616
cal balls is (c) 462 (d) 828
kgei
128.The area of the curved surface
(a) 12 (b) 16 (c) 24 (d) 48 122. A s olid s p h e r e of 6 c m and the area of the base of a
116.A cylinder has ‘r’ as the radius diameter is melted and recast right circular cylinder are a
of the bas e and ‘ h’ as the into 8 solid spheres of equal square cm and b square cm
ERna

height. The radius of base of volume. The radius (in cm) of respectively. The height of the
anothe r cy lind er, having each small sphere is cylinder is
double the volume b ut the (a) 1.5 (b) 3 (c) 2 (d) 2.5
same height as that of the first 2a a b
123. I n a c y lind e r i c al v e s s e l of (a) cm (b) cm
aBryn

cylinder must be equal to diameter 24 cm filled up with b 2


r sufficient quantity of water,
(a) (b) 2r (c) r 2 (d) 2r a a
2 a s o l i d s p h e r i c a l b a ll o f (c) cm (d) cm
r ad ius 6 c m is c om p le te ly 2 b 2 b
117. From a solid cylinder whose
immersed. Then the 129.The volume of a solid hemi-
height is 12 cm and diameter
Les

increase in he ight of water sphere is 19404 cm3. Its total


10 cm. a conical cavity of same surface area is
level is :
wa.th

height and same diameter of (a) 4158 cm2 (b) 2858 cm2
the base is hollowed out. The (a) 1.5 cm (b) 2 cm
(c) 3 cm (d) 4.2 cm (c) 1738 cm2 (d) 2038 cm2
volume of the remanining solid 130. A solid hemisphere is of radius 11
124. A solid wooden toy is in the
22 shape of a right circular cone cm. the curved surface area in sq.
is approximately (  )
wwM

cm is
7 mounted on a hemisphere. If
(a) 1140.85 (b) 1386.00
(a) 942.86 cm3 (b) 314.29 cm3 the radius of the hemisphere
(c) 760.57 (d) 860.57
(c) 628.57 cm3 (d) 450.76 cm3 is 4.2 cm and the total height
131. The base of a cone and a cylin-
118. The radius of a cylinder is 10 of the toy is 10.2 cm find the
der have the same radius 6 cm.
cm and height is 4 cm. The the volume of wood en toy
They have also the same height
number of centimetres that (nearly). 8 cm. The ratio of the curved
may be added either to the ra- (a) 104 cm3 (b) 162 cm3 surface area of the cylinder to
3
dius or to the height to get the (c) 421 cm (d) 266 cm3 that of the cone is
same increase in the volume 125. If a solid cone of volume 27 (a) 8 : 5 (b) 8 : 3
of the cylinder is cm 3 is kept inside a hollow (c) 4 : 3 (d) 5 : 3
(a) 5 cm (b) 4 cm c ylinde r whos e radius and 132. A right cylindrical vessel is full
(c) 25 cm (d) 16 cm height are that of the cone, with water. How many right

Rakesh Yadav Readers Publication Pvt. Ltd. 379

For More Visit : www.LearnEngineering.in


For More Visit : www.LearnEngineering.in

cones having the same diam- 139. What is the height of a cylin-
22
eter and height as that of the der that has the same volume ity of the cup ( Take  ) is
right cylinder will be needed to and radius as a sphere of diam- 7
store that water ? eter 12 cm ? (a) 624.26 cm3 (b) 622.36 cm3
22 (a) 7 cm (b) 10 cm (c) 622.56 cm3 (d) 623.20 cm3
(take   ) (c) 9 cm (d) 8 cm 147. A conical flask is full of wa-
7 140. The perimeter of the base of a
(a) 4 (b) 2 (c) 3 (d) 5 ter. The flask has base ra-
right circular cone is 8 cm. If the d i us r a nd he i g ht h. Th is
133. A spherical lead ball of radius
height of the cone is 21 cm, then water is poured into a cylin-
10 cm is melted and small lead
is volume is : drical flask of base radius m,
balls of radius 5mm are made
The total number of possible 112 height of cylindrical flask is
sm all lead balls is (a) 108  cm3 (b) cm3

r
 m h 2
(a) (b) m
22

i
108 2h 2

.iSn
(Take   ) (c) 112 cm3 (d) cm3
7 
(a) 8000 (b) 400 2h r²h
141 .If the volume of two right (c) (d)
(c) 800 (d) 125 cir cular cones are in the m 3m2

agv
134. The number of spherical bullets ratio 4 : 1 and their diameter 148.A solid spherical copper ball
that can be made out of solid are in the ratio 5 : 4, then the
cube of lead whose edge mea- whose diameter is 14 cm is

ridn
ratio of their height is : melted and converted into a
sures 44 cm each bullet being
of 4 cm diamete r, is (Take (a) 25 : 16 (b) 25 : 64 wire having diam eter equal
(c) 64 : 25 (d) 16 : 25

eeYa
22 to 14 cm. The length of the
 ) 142. The volume of a conical tent is wire is
7 1232 cu. m and the area of its
(a) 2541 (b) 2451 base is 154 sq. m. Find the 16
(a) 27 cm (b) cm
(c) 2514 (d) 2415 length of the canvas required 3
135. The radius of a metallic cylin-
geisnh
to build the tent, if the canvas
28
der is 3 cm and its height is 5 is 2m in width. (c) 15 cm (d) cm
3
cm. It is melted and moulded 22
into sm all cone s, e ach of ( Take  ) 149. A sphere of diameter 6 cm is
7 droped in a rig ht c ircular
Enak
height 1 cm and base radius 1
(a) 270 m (b) 272 m cylinderical vessel partly filled
mm. The num ber of s uch
(c) 276 m (d) 275 m with water. The diameter of the
cones formed is cylinderical vessel is 12 cm. If
143. If the ratio of the diameters of
(a) 450 (b) 1350
R

two r ight circular cones of the sphere is just completely


(c) 8500 (d) 13500 equal height be 3: 4, then the submerged in water, then the
136.A sector is formed by opening ratio of their volume will be ris e of water level in the
aryn

out a cone of base radius 8 cm (a) 3 : 4 (b) 9 : 16 cylindrical vessel is


and height 6 cm. Then the ra- (c) 16 : 9 (d) 27 : 64 (a) 2 cm (b) 1 cm
Les B

dius of the sector is (in cm) (c) 3 cm (d) 4 cm


144. The surface area of two spheres
150. A copper sphere of diameter 18
(a) 4 (b) 8 (c) 10 (d) 6 are in the ratio 4 : 9. Their vol-
cm is drawn into a wire of
137. A solid cone of height 9 cm umes will be in the ratio diameter 4 mm. The length of
with diameter of its base 18 (a) 2 : 3 (b) 4 : 9 the wire in metre is :
wa.th

cm is cut out from a wooden (c) 8 : 27 (d) 64 : 729 (a) 2.43 m (b) 243 m
solid sphere of radius 9 cm. 145. The total surface ar ea of a (c) 2430 m (d) 24.3 m
The p e r c e nt ag e of wood sphere is 8 square unit. The 151. A rectangular block of metal
wasted is : volume of the sphere is has dimensions 21 cm, 77 cm
and 24 cm. The block has been
wwM

(a) 25% (b) 30%


8 2 melted into a s pher e. The
(c) 50% (d) 75% (a)  cubic unit
radius of the sphere is (Take
3
138. The perimeter of the base of
8 22
a right circular cylinder is ‘a’  )
(b)  cubic unit 7
unit . If the volume of the cyl- 3
inder is V cubic unit, then (a) 21 cm (b) 7 cm
(c) 8 3 cubic unit
the height of the cylinder is (c) 14 cm (d) 28 cm
152. The radius of cross-section of
4a 2V 4a 2 (d)
8 3
 cubic unit
(a) unit (b) unit 5 a solid cylindrical rod of iron is
 V
146. A semicircular sheet of metal 50 cm. the cylinder is melted
a 2V 4V
of diameter 28 cm is bent into down and formed into 6 solid
(c) unit (d) 2 unit
4 a an open conical cup. The capac- spherical balls of the same

Rakesh Yadav Readers Publication Pvt. Ltd. 380

For More Visit : www.LearnEngineering.in


For More Visit : www.LearnEngineering.in

radius as that of the cylinder. into a right circular cone with (a) 1078 cubic cm
The le ngth of the r od (in radius of its base as 1.2 cm. (b) 1708 cubic cm
metres) is Then the height of the cone (in (c) 7108 cubic cm
cm) is (d) 7180 cubic cm
(a) 0.8 (b) 2 (c) 3 (d) 4
(a) 3.6 cm (b) 4.8 cm 166.A hollow sphere of internal and
153. Two right circular cones of
(c) 6.4 cm (d) 7.2 cm external diameter 6 cm and 10
equal height and radii of there
160. If surface area and volume of a cm respectively is melted into
respective base 3 cm and 4 cm
sphere are S and V a right circular c one of
are melted together and made
to a solid sphere of radius 5 cm. S3 diameter 8 cm. The height of
The height of a cone is respectively, then value of the cone is
V2
(a) 10 cm (b) 20 cm (a) 22.5 cm (b) 23.5 cm
is

ir
(c) 30 cm (d) 40 cm (c) 24.5 cm (d) 25.5 cm
(a) 36 units (b) 9 units
167.A flask in the shape of a right

v.iSn
154. The radius of the base and the (c) 18 units (d) 27 units
height of a right circular cone circular cone of height 24 cm
161. Assume that a drop of water is is filed with water. The water
are doubled. The volume of the spherical and its diameter is
cone will be is pour ed in right cirular
one-tenth of a cm. A conical
cylindrical flask whose radius

dnag
(a) 8 time s of the pr evious glass has a height equal to the
volume diameter of its rim. If 32,000 1
(b) three times of the previous drops of water fill the glass is rd of radius of the base of
3
volume completely. Then the height of the circular cone. Then the

eYrai
(c) 3 2 times of the preivous the glass (in cm) is
height of the water in the
(a) 1 (b) 2 (c) 3 (d) 4 cylindrical flask is
volume
162. A tank 40 m long, 30 m broad
(d) 6 time s of the pr evious (a) 32 cm (b) 24 cm
and 12 m deep is dug in a field
volume (c) 48 cm (d) 72 cm
155. If h, c, v are respecitvely the
height, curved surface area
and volume of a right circular
snhe
1000 m long and 30 m wide. By
how much will the level of the
field rise if the earth dug out of
the tank is evenly spread over
168.A solid metallic spherical ball of
diameter 6 cm is melted and
re cast into a cone with
diameter of the base as 12 cm.
kgei
cone then the v alue of the field ?
(a) 2 metre (b) 1.2 metre The height of the cone is
3vh 3 – c 2h 2  9v 2 is
(c) 0.5 metre (d) 5 metre (a) 2 cm (b) 3 cm
(a) 2 (b) – 1 (c) 1 (d) 0
(c) 4 cm (d) 6 cm
ERna

156. The total number of spherical 16 3.A sphe re is cut into two
hemispheres. One of them is 169. A he m i s p he r ic al b owl of
bullets, each of diameter 5
used as bowl. It takes 8 bowlfuls inte r nal r a d ius 15 cm
decimeter, that can be made by
utilizing the maximum of a of this to fill a conical vessel of contains a liquid. The liquid
height 12 cm and radius 6 cm. is to be filled into cylindrical
aBryn

rectangular block of lead with


11 metr e le ngth, 10 metre The radius of the sphere(in shaped bottles of diameter 5
breadth and 5 metre width is centimetre) will be c m and he ig ht 6 c m . The
(a) 3 (b) 2 (c) 4 (d) 6 number of bottles required to
(assume that   3 )
164.A ball of lead 4 cm in diameter empty the bowl is
(a) equal to 8800 (a) 30 (b) 40 (c) 50 (d) 60
is covered with gold. If the
Les

(b) less than 8800 170. If V1, V2 and V3 be the volumes


volume of the gold and lead are
(c) equal to 8400
wa.th

equal then the thickness of of a r ight cir cular c one. a


(d) greater than 9000 sphere and a right circular
157. If a metallic cone of radius 30 gold [g iven 3 2 = 1.25 9) is cy lind er having the same
cm and height 45 cm is melted approximately radius and same height then
and re cast into me tallic (a) 5.038 cm (b) 5.190 cm
wwM

spheres of radius 5 cm, find the V2 V 3 V1 V2


(c) 1.038 cm (d) 0.518 cm (a) V1= = (b)  = V3
number of spheres , 165.A conical cup is filled with 4 3 2 3
(a) 81 (b) 41 (c) 80 (d) 40 icecream. The ice-cream forms
158. A metallic sphere of radius V1 V2 V1 V3
a hemispherical shape on its (c)  = V3 (d) = V2 =
10.5 cm is melted and then 3 2 3 2
open top. The height of the
recast into small cones each of 171. If the surface area of a sphere
hemispherical part is 7 cm. the
radius 3.5 cm and height 3 cm. is 346.5 cm2, then its radius [
The number of cone s thus radius of the hemispherical
formed is part equals to the height of the 22
cone. Then the volume of the taking   ]
(a) 140(b) 132 (c) 112 (d) 126 7
159. A right circular cone is 3.6 cm  22  (a) 7 cm (b) 3.25 cm
high and radius of its base is ice-cream is   
(c) 5.25 cm (d) 9 cm
1.6 cm. It is melted and recast 
 7 

Rakesh Yadav Readers Publication Pvt. Ltd. 381

For More Visit : www.LearnEngineering.in


For More Visit : www.LearnEngineering.in

172. Deepali makes a model of a tion of a tunnel, cylindrical in 184. The total surface area of a right
cylindrical kaleidoscope for her shape, of diameter 4 m and circular cylinder with radius of
science project. She uses a the base 7 cm and heigtht 20
chart paper to make it. If the  22 
length 56 m is  Take  = cm is:
length of the kaleidoscope is 25  7  (a) 140 cm2 (b) 1000 cm2
cm and radius 35 cm, the area 2
1 (c) 900 cm (d) 1188 cm2
of the paper she used, in sq. 1
(a) Part (b) Part 185. The radius of base and curved
 22  9 2
   surface area of a right cylinder
cm, is  
 7  1 2 are 'r' units and 4rh square
(c) Part (d) Part
(a) 1100 (b) 5500 4 9 units respectively. The height
(c) 500 (d) 450 179. From a solid right circular cyl- of the cylinder is:

r
173. If the volume of a sphere is nu- inder of length 4 cm and diam-

i
.iSn
merically equal to its surface h
eter 6 cm, a conical cavity of (a) 4h units (b) units
area then its diameter is; the same height and base is 2
(a) 4 cm (b) 6 cm hollowed out. The whole surface (c) h units (d) 2h units
(c) 3 cm (d) 2 cm area of the remaining solid (in

agv
186. A hemi-spherical bowl has 3.5
174. 5 persons live in a tent. If square cm.) is cm radius. It is to be painted
each person requires 16 m²
(a) 48 (b) 63 inside as well as outside. The

ridn
of floor area and 100 m³ space
for air then the height of the (c) 15 (d) 24 cost of painting it at the rate of
c o ne o f s m al le s t s iz e to Rs. 5 per 10sq. cm. will be:
180. A spherical ball of radius 1 cm

eeYa
ac c o m od ate t he s e p e r s ons (a) Rs. 77 (b) Rs. 175
is dropped into a conical ves-
would be? sel of radius 3 cm and slant (c) Rs. 50 (d) Rs. 100
(a) 16 m (b) 18.75 m height 6 cm. The volume of 187. The volume of a right circular
(c) 10.25 m (d) 20 m water (in cm³), that can just cone which is obtained from a
175. The numerical values of the
geisnh
immerse the ball, is wooden cube of edge 4.2 dm
volume and the area of the wasting minimum amount of
lateral surface of a right cir- 5  4 wood is:
cular cone are equal. If the (a) (b) 3 (c) (d)
3 3 3 (a) 194.04 cu. dm
height of the cone be h and
181. If the height of a cylinder is 4 (b) 19.404 cu. dm
Enak

r a d i us b e r , th e v a lu e of
times its circumference, the (c) 1940.4 cu. dm
1 1 volume of the cylinder in terms (d) 1940.4 cu. dm
+ is
h2 r2 of its circumference, c is 188. If the radius of a sphere is in-
R

9 3 1 1 2c³ c³ creased by 2 cm, then its sur-


(a) (b) (c) (d) (a) (b) face area increases by 352cm².
1 1 3 9  
aryn

The radius of the sphere ini-


176.There is wooden sphere of ra- (c) 4 c³ (d) 2  c³  22 
Les B

dius 6 3 cm. The surface 182. The radii of a sphere and a tially was: u se 7 
 
right circular cylinder are 3 cm
area of the largest possible cube (a) 3cm (b) 5cm
eac h. I f their volumes are
cut out from the sphere will be (c) 4cm (d) 6cm
equal, then curved surface area
(a) 464 3 cm² (b) 646 3 cm² 189. A right triangle with sides 9 cm,
wa.th

of the cylinder is
(c) 864 cm² (d) 462 cm² 12 cm and 15 cm is rotated
 22  about the side of 9 cm to form a
177. I f a he m is p he r e i s m e l te d  Assume π 7 
  cone. The volume of the cone so
and f our s p he r e s of e qual
formed is:
wwM

volume are made, the radius 3 3


of each sphere will be equal (a) 75 cm2 (b) 65 cm2 (a) 432 p cm³ (b) 327 p cm³
to 7 7 (c) 334 p cm³ (d) 324 p cm³
(a) 1/4th of the hemisphere 190. The volume of the largest right
3 3
(b) radius of the hemisphere (c) 74 cm2 (d) 72 cm2 circular cone that can be cut
7 7 out of a cube of edge 7 cm ?
(c) 1/2 of the rad ius of the
hemisphere 183. The radius of a hemispherical
bowl is 6 cm. The capacity of
 22 
(d) 1/6 th of the radius of the u se
 

 7 
hemisphere
178. The portion of a ditch 48 m long. 
the bowl is: Take Π = 22 7  (a) 13.6 cm³ (b) 147.68 cm³
16.5 m wide and 4 m deep that (c) 89.9 cm³ (d) 121 cm³
(a) 452.57 cm³ (b) 452 cm³
can be filled with stones and 191. By melting two solid metallic
(c) 345.53 cm³ (d) 495.51 cm³
earth available during excava- spheres of radii 1 cm and 6 cm,

Rakesh Yadav Readers Publication Pvt. Ltd. 382

For More Visit : www.LearnEngineering.in


For More Visit : www.LearnEngineering.in

a hollow sphere of thickness 1 198. A plate of square base made tio of their total surface area is:
cm is made. The external ra- of brass is of length x cm and (a) 27 : 64 (b) 3 : 4
dius of the hollow sphere will thic k ne s s 1 m m . The p late (c) 9 : 16 (d) 3 : 8
be weighs 4725 gm. If 1 cubic cm 208. A h e m i s p h e r e a n d a c on e
(a) 8 cm (b) 9 cm of b r as s we ig hs 8 . 4 gr am , hav e e q ual b as e . I f th e ir
(c) 6 cm (d) 7 cm then the value of x is: heights are also equal, the
192. Water is flowing at the rate of 5 (a) 76 (b) 72 (c) 74 (d) 75 ratio of their curved surface
km/h through a pipe of diam- 199. The diameter of a 120 cm long will be :
eter 14 cm into a rectangular roller is 84 cm. It takes 500
complete revolutions of the (a) 1 : 2 (b) 2 :1
tank which is 50 m long 44m
wide, The time taken (in hours ) roller to level a ground. The (c) 1 : 2 (d) 2 : 1
for the rise in the level of water in cost of levelling the ground at 209. If the height of a given cone

ir
the tank to 7 cm is Rs. 1.50 sq. m. is: be doubled and radius of the
base remains the same the

v.iSn
(a) Rs. 5750 (b) Rs. 6000
1 1 (c) Rs. 3760 (d) Rs. 2376 r atio of the v olum e of the
(a) 2 (b) 1 (c) 3 (d) 2 given cone to that of the sec-
2 2 200. Two right circular cylinders of
ond cone will be
193.The volume (in m3) of rain wa- e qual v olume hav e their
(a) 2 : 1 (b) 1 : 8

dnag
ter that can be collected from heights in the ratio 1 : 2. The
(c) 1 : 2 (d) 8 : 1
1.5 hectares of ground in a ratio of their radii is :
210. Spheres A and B have their
rainfall of 5 cm is (a) 2 :1 (b) 2 : 1 radii 40 cm and 10 cm respec-
(a) 75 (b) 750 tively. Ratio of surface area of
(c) 1 : 2 (d) 1 : 4

eYrai
(c) 7500 (d) 75000 A to the surface area of B is :
201. If the volume of two cubes are
194. A river 3 m deep and 40 m (a) 1 : 16 (b) 4 : 1
in the ratio 27 : 1, the ratio of
wide is flowing at the rate (c) 1 : 4 (d) 16 : 1
their edge is :
of 2 km per hour, How much 211. If the radius of the base of a
(a) 3 : 1 (b) 27 : 1
water ( in-litres) will fall into
sea in a minute ?
(a) 4,00,000 m³ (b) 40,00,000 m³
(c) 40,000 m³ (d) 4,000 m³
(c) 1 : 3
snhe (d) 1 : 27
202. The edges of a cuboid are in
the ratio 1 : 2 : 3 and its sur-
cone be doubled and height is
left unchanged, then ratio of
the volume of new cone to that
of the original cone will be:
face area is 88 cm 2. The vol-
kgei
195. Water is flowing at the rate of (a) 1 : 4 (b) 2 : 1
ume of the cuboid is : (c) 1 : 2 (d) 4 : 1
3 km/hr through a circular
(a) 48 cm³ (b) 64 cm³ 212. A cube of edge 5 cm is cut into
pipe of 20 cm internal diam-
(c) 16 cm³ (d) 100 cm³ cubes each of edge of 1 cm. The
ERna

eter into a circular cistern of


diameter 10 m and depth 2m. 203. The volume of two spheres are ratio of the total surface area
in the ratio 8 : 27. The ratio of of one of the small cubes to that
In how much time will the cis- of the large cube is equal to:
tern be filled ? their surface area is:
(a) 4 : 9 (b) 2 : 3 (a) 1 : 125 (b) 1 : 5
(a) 1 hour
aBryn

(c) 4 : 5 (d) 5 : 6 (c) 1 : 625 (d) 1 : 25


(b) 1 hour 40 minutes 213. The diameter of two hollow
(c) 1 hour 20 minutes 204. The base radii of two cylinders
spheres made from the same
(d) 2 hours 40 minutes are in the ratio 2 : 3 and their
metal sheet are 21 cm and
heights are in the ratio 5 : 3. 17.5 cm respectively. The ra-
196. Water flows at the rate of 10
The ratio of their volumes is : tio of the area of metal sheets
metres per minute from
Les

cylindrcial pipe 5 mm in di- (a) 27 : 20 (b) 20 : 27 required for making the two
wa.th

ameter, How long it will take (c) 9 : 4 (d) 4 : 9 spheres is


to f ill up a c on ic al v e s s e l 205. The curved surface area of a (a) 6 : 5 (b) 36 : 25
whose diameter at the base cylinderical pillar is 264 m 2 (c) 3 : 2 (d) 18 : 25
is 30 cm and depth 24 cm? and its v olu m e is 9 2 4 m 3 214. B y m e l t i n g a s o l i d l e a d
(a) 28 mintues 48 seconds 22 sp he re of diame te r 12 c m,
wwM

( Taking  
7
). find the ratio three small spheres are
(b) 51 minutes 12 seconds made whose d iame te rs are
of its diameter to its height .
(c) 51 minutes 24 seconds in the ratio 3 : 4 : 5. The ra-
(a) 7 : 6 (b) 6 : 7
(d) 28 mintues 36 seconds dius (in cm) of the smallest
(c) 3 : 7 (d) 7 : 3
197. The radius of the base of coni- sphere is
206. The ratio of the volume of two
cal tent is 12 m. The tent is (a) 3 (b) 6 (c) 1.5 (d) 4
9 m high. Find the cost of can- cones is 2 : 3 and the ratio of 215. A cone is cut at mid point of
v a s r e qu ir e d t o m a k e t he radii of their base is 1 : 2. The i t s h e i g h t b y a f r us t u m
tent, if one square metre of ratio of their height is parallel to its base. The ratio
canvas costs Rs.120 (Take = (a) 3 : 8 (b) 8 : 3 between the volumes of two
3.14) (c) 4 : 3 (d) 3 : 4 parts of cone would be
(a) Rs. 67,830 (b) Rs. 67,800 207. If the volume of two cubes are (a) 1 : 1 (b) 1 : 8
(c) Rs. 67,820 (d) Rs. 67,824 in the ratio 27 : 64, then the ra- (c) 1 : 4 (d) 1 : 7

Rakesh Yadav Readers Publication Pvt. Ltd. 383

For More Visit : www.LearnEngineering.in


For More Visit : www.LearnEngineering.in

216.The ratio of the area of the in- 225. The diameter of two cylinders, 233. The radius and the height
circle and the circum-circle of whose volumes are equal, are of a c o ne a r e i n th e r a tio
a square is in the r atio 3 : 2 , The ir 4 : 3. The ratio of the curved
(a) 1 : 2 (b) 2 : 1 heights will be in the ratio. surface area and total sur-
(a) 4 : 9 (b) 5 : 6 face area of the cone is
(c) 1 : 2 (d) 2 : 1
(c) 5 : 8 (d) 8 : 9 (a) 5 : 9 (b) 3 : 7
(d) remains unchanged 226. The radius of base and slant (c) 5 : 4 (d) 16 : 9
217. The ratio of the surface area of height of a cone are in the 234. A sphere and a cylinder have
a sphere and the curved sur- ratio 4 : 7. If slant height is equal volume and equal ra-
face area of the cylinder cir- 1 4 c m the n the r adius (in dius. The ratio of the curved
cumscribing the sphere is surface area of the cylinder
22
(a) 1 : 2 (b) 1 : 1 cm) of its base is ( use ) to that of the sphere is

r
(c) 2 : 1 (d) 2 : 3 7
(a) 4 : 3 (b) 2 : 3

i
(a) 8 (b) 12 (c) 14 (d) 16

.iSn
218. The radii of two spheres are in (c) 3 : 2 (d) 3 : 4
the ratio 3 : 2. Their volume will 227. A right circular cylinder just
235. A right circular cylinder and a
be in the ratio : encloses a sphere of radius r.
cone have equal base radius
(a) 9 : 4 (b) 3 : 2 The ratio of the surface area
and equal he ig ht. If their

agv
of the sphere and the curved
(c) 8 :27 (d) 27 : 8 curved surfaces are in the ra-
surface area of the cylinder is
219. The volume of a sphere and a tio 8 : 5, then the radius of the
(a) 2 : 1 (b) 1 : 2

ridn
right circular cylinder having base to the height are in the
the same radius are equal. The (c) 1 : 3 (d) 1 : 1 ratio:
ratio of the diameter of the 228.The ratio of radii of two cone is (a) 2 : 3 (b) 4 : 3

eeYa
sphere to the height of the cyl- 3 : 4 and the ratio of their height
(c) 3 : 4 (d) 3 : 2
inder is is 4 : 3. Then the ratio of their
236. A right prism with trapezium
(a) 3 : 2 (b) 2 : 3 volume will be
base of parallel side 8 cm & 14
(a) 3 : 4 (b) 4 : 3
(c) 1 :2 (d) 2 : 1 cm. Height of prism is 12 cm &
(c) 9 : 16 (d) 16 : 9
220. A cone, a hemisphere and a
geisnh its volume is 1056 cm 3 then.
229.If a right circular cone is sepa-
cylinder stand on equal bases Find the distance two parallel
rated into solids of volumes V1,
and have the s ame he ight. lines.
V2, V3 by two planes parallel to
The ratio of their respective (a) 8 (b) 10 (c) 16 (d) 6
the base which also trisect the
volume is 237. The radii of the base of cylin-
Enak

altitude, then
(a) 1 : 2 : 3 (b) 2 : 1 : 3 der and a cone are in the ratio
V1 : V2 : V3 is
(c) 1 : 3 : 2 (d) 3 : 1 : 2 (a) 1 : 2 : 3 (b) 1 : 4 : 6 3 : 2 and their heights are
221. The radii of the base of two cyl- (c) 1 : 6 : 9 (d) 1 : 7 : 19
R

in the ratio 2 : 3 . Their vol-


inders are in the ratio 3 : 5 and
230. The total surf ace area of a umes are in the ratio of
their heights in the ratio 2 : 3. solid right circular cylinder is
aryn

The ratio of their curved sur- twice that of a solid sphere. If (a) 3: 2 (b) 3 3 : 2
face will be : they have the same radii, the (c) 3 : 2 2 (d) 2 : 6
(a) 2 : 5 (b) 2 : 3
Les B

ratio of the volume of the cyl-


(c) 3 : 5 (d) 5 :3 238. The heights of two cones are in
inder to that of the sphere is
222. If the radii of two spheres are the ratio 1 : 3 and the diameters
given by of their base are in the ratio 3 :
in the ratio 1 : 4, then their (a) 9 : 4 (b) 2 : 1 5, The ratio of their volume is
surface area are in the ratio : (c) 3 : 4 (d) 4 :9
wa.th

(a) 3 : 25 (b) 4 : 25
(a) 1 : 2 (b) 1 : 4 231. The respective height and vol- (c) 6 : 25 (d) 7 : 25
(c) 1 : 8 (d) 1 :16 ume of a hemisphere and a 239. A sphere and a hemisphere
223. The radii of the base of two right circ ular cy lind er are have the same volume . The
cylinders A and B are in the equal, then the ratio of their ratio of their radii is
wwM

ratio 3 : 2 and their height in radii is (a) 1 : 2 (b) 1 : 8


the ratio x : 1. If the volume
of cylinder A is 3 times that (a) 2: 3 (b) 3 :1 (c) 1 : 2 (d) 1 : 3 2
of cylinder B, the value of x is (c) 3 : 2 (d) 2 : 3 240. The diameter of the moon is as-
4 2 3 3 232. The ratio of the volume of a sumed to be one fourth of the di-
(a) 3
(b) 3
(c) (d) ameter of the earth. Then the ra-
4 2 cube and of a solid sphere is
tio of the volume of the earth to
224.A solid metallic sphere of radius 363 : 49. The ratio of an edge that of the moon is
8 cm is melted to form 64 equal of the cube and the radius of
(a) 64 : 1 (b) 1 : 64
small solid spheres. The ratio of
22 (c) 60 : 7 (d) 7 : 60
the surface area of this sphere the sphere is ( take  )
to that of a small sphere is 7 241. If A denotes the volume of a right
(a) 7 : 11 (b) 22 : 7 circular cylinder of same height
(a) 4 : 1 (b) 1 :16 as its diameter and B is the vol-
(c) 16 :1 (d) 1 : 4 (c) 11 :7 (d) 7 : 22

Rakesh Yadav Readers Publication Pvt. Ltd. 384

For More Visit : www.LearnEngineering.in


For More Visit : www.LearnEngineering.in

ume of a sphere of same radius will be 254.If the radius of a cylinder is


A (a) 8 : 3 (b) 3 : 8 de cr eased b y 50 % and the
then is: (c) 4 : 3 (d) 3 : 4 height is increased by 50% to
B
249. The volumes of a right circular form a new cylinder, the
4 3 2 3 cylinde r and a sphe re are volume will be decreased by
(a) (b) (c) (d)
3 2 3 4 equal. The radius of the cylin- (a) 0% (b) 25%
242. Diagonal of a cube is 6 3 cm. der and the diameter of the (c) 62.5% (d) 75%
sphere are equal. The ratio of 255. Each of the height and base
Ratio of its total surface area
and volume (numerically) is height and radius of the cylin- radius of a cone is increased by
(a) 2 : 1 (b) 1 : 6 der is
10 0%. The perc entage
(c) 1 : 1 (d) 1 : 2 (a) 3 : 1 (b) 1 : 3 increase in the volume of the

ir
243. A sphere and a hemisphere (c) 6 : 1 (d) 1 : 6 cone is
250. A large solid sphere is melted

v.iSn
have the same volume. The (a) 700% (b) 400%
ratio of their curved surface and moulded to form identical
(c) 300% (d) 100%
area is : right circular cones with base
3 2 radius and height same as 256. If both the radius and height
(a) 2 2 :1 (b) 2 3 :1 the radius of the sphere. One of a right circular cone are in-

dnag
of these cones is melted and creased by 20%, its volume will
2 1
(c) 4 3 :1 (d) 2 3 :1 moulde d to form a s maller be increased by
244. The volume of a cylinder and a solid sphere. Then the ratio of (a) 20% (b) 40%
the surface area of the smaller (c) 60% (d) 72.8%

eYrai
cone are in the ratio 3 : 1. Find
their diameters and then com- to the s ur f ac e ar e a of the 257. A cone of height 15 cm and
pare them when their heights larger sphere is b a s e d iam e t e r 3 0 c m is
are equal. 4 3 carved out of a wooden sphere
(a) 1 : 3 3 (b) 1 : 2 2
of radius 15 cm. The percent-
(a) Diameter of cylinder = 2
times of diameter of cone
(b) Diameter of cylinder = Di-
ameter of cone
(c) 1 : 23
2
snhe (d) 1 : 2 3
4

251. A plane divides a right circu-


age of used wood is :
(a) 75%
(c) 40%
(b) 50%
(d) 25%
kgei
lar cone into two parts of equal
(c) Diameter of cylinder > Di- 258. If the height of a right circular
ameter of cone volume. If the plane is paral-
lel to the base, then the ra- cone is increased by 200% and
(d) Diamete r of cylinde r < the radius of the base is re-
tio, in which the height of the
ERna

Diaameter of cone duced by 50%, the volume of the


cone is divided, is
245. A solid sphere is melted and re- cone
cast into a right circular cone (a) 1 : 2 (b) 1 : 3 2
(a) increases by 25%
with a base radius equal to the
(c) 1 : 3 2 – 1 (d) 1 : 3 2 1 (b) increases by 50%
aBryn

radius of sphere. What is the


ratio of the height and radius 252. A rectangle based pyramid, (c) remains unaltered
of the cone so formed ? length and width of the base is (d) decreases by 25%
(a) 4 : 3 (b) 2 : 3 18cm and 10cm respectively. 259. If the height and the radius of
(c) 3 : 4 (d) 4 : 1 Find the total surface area, if the base of a cone are each in-
its height is 12cm : creased by 100%, then the vol-
Les

246. Find the total surface area of a


(a) 267cm2 (b) 564cm2 ume of the cone becomes
prism which is based on  of pe-
wa.th

rimeter 45 cm & incircle radius (c) 516cm 2 (a) double that of the original
9cm, if its volume is 810 cm3. (d)None of these (b) three times that of the origi-
(a) 405 (b) 585 253. A cone of height 7 cm and nal
(c) 616 (d) 468 base radius 1 cm is carved (c) six times that of the original
wwM

247. The ratio of weights of two from a cuboidal block of wood (d) eight times that of the origi-
spheres of different materials 10 cm × 5 cm × 2 cm .[As- nal
is 8 : 17 and the ratio of weights 260. If the height of a cylinder is
per 1 cc of materials of each is 22 increased by 15 per cent and
suming  ] The percent-
289 : 64. The ratio of radii of 7 the radius of its base is de-
the two spheres is age of wood wasted in the pro- creased by 10 percent then by
(a) 8 : 17 (b) 4 : 17 cess is : what percent will its curved
(c) 17 : 4 (d) 17 : 8 surface area change ?
2 1
248. If the ratio of volumes of two (a) 92 % (b) 46 % (a) 3.5 percent decrease
3 3
cones is 2 : 3 and the ratio of (b) 3.5 percent increase
the radii of their bases is 1 : 2, 1 1 (c) 5 percent increase
then the ratio of their heights (c) 42 % (d) 41 %
3 3 (d) 5 percent decrease

Rakesh Yadav Readers Publication Pvt. Ltd. 385

For More Visit : www.LearnEngineering.in


For More Visit : www.LearnEngineering.in

261.If the radius of a sphere is 269. If each edge of a cube is in- the parallel sides is 8 cm, If the
doubled, its volume becomes creased by 50%, the percentage volume of the prism is 1056
(a) double (b) four times increase in its surface area is cm 3 , then the height of the
(c) six times (d) eight times (a) 150% (b) 75% prism is
262. If the radius of a right circular (c) 100% (d) 125% (a) 44 cm (b) 16.5 cm
cylinder is decreased by 50% 270. If the radius of a sphere be (c) 12 cm (d) 10.56 cm
and its height is increased by doubled, then the percentage 277. Each edge of a regular tetrahe-
60% its volume will be de- increase in volume is dron is 3 cm, then its volume is
creased by (a) 500% (b) 700% 9 2
(c) 600% (d) 800% (a) c.c. (b) 27 3 c.c.
(a) 10% (b) 60% 4
(c) 40% (d) 20% 271. Find the radius of maximum
4 2

r
263. The le ng th, b r e ad th and size sphere which can be in- (c) c.c. (d) 9 3 c.c.
scribed or put in a cone whose 9

i
.iSn
height of a cuboid are in the 278. The perimeter of the triangu-
ratio 1 : 2 : 3. If they are in- base radius and height are 6cm
and 8cm respectively? lar base of a right prism is 15
creased by 100%, 200% and cm and radius of the incircle of
200% respectively. Then com- (a) 4cm (b) 5cm
the triangular base is 3 cm. If

agv
pared to the original volume (c) 3cm
the volume of the prism be 270
the increase in the volume of (d) None of these
cm 3 then the he ight of the
272. If the length of each side of a

ridn
the cuboid will be prism is
(a) 5 times (b) 18 times regular tetrahedron is 12 cm,
then the volume of the tetra- (a) 6 cm (b) 7.5 cm
(c) 12 times (d) 17 times (c) 10 cm (d) 12 cm

eeYa
hedron is
264. Each of the radius of the base 279. The base of a solid right prism
and the height of a right circu- (a) 144 2 cu. cm, is a triangle whose sides are 9
lar cylinder is increased by (b) 72 2 cu. cm, cm, 12 cm and 15 cm, The
10%. The volume of the cylin- height of the prism is 5 cm. The
der is increased by
geisnh
(c) 8 2 cu. cm, the total surface area of the
(a) 3.31% (b) 14.5% prism is
(d) 12 2 cu. cm,
(c) 33.1% (d) 19.5% (a) 180 cm2 (b) 234 cm2
265. If the height of a cone is in- 273. If the radii of the circular ends 2
(c) 288 cm (d) 270 cm2
of a truncated conical bucket
Enak

crease by 100% then its volume 280.The base of a right prism is


which is 45cm high be 28 cm
is increased by : an e qu il ate r a l tr i an g l e of
and 7 cm then the capacity of
(a) 100% (d) 200% area 173 cm 2 and the volume
the bucket in cubic centimetre
R

(d) 300% (d) 400% of the prism is 10380 cm 3 .


266. A hemispherical cup of radius 22 The area of the lateral sur-
is (use  )
4 cm is filled to the brim with 7 face of the prism is
aryn

coffee. The coffee is then poured (a) 48510 (b) 45810 ( use 3 =1.73)
into a vertical cone of radius 8 (c) 48150 (d) 48051
Les B

(a) 1200 cm2 (b) 2400 cm2


cm and height 16 cm. The per- 274. There is a pyramid on a base 2
centage of the volume of the which is a regular hexagon of (c) 3600 cm (d) 4380 cm2
cone that remains empty is : side 2a cm. If every slant edge 281. The base of a right pyamid is a
square of side 16 cm long . If
(a) 87.5% (b) 80.5% 5a its height be 15 cm, then the
wa.th

(c) 81.6% (d) 88.2% of this pyramid is of length


2 area of the lateral surface in
267. The height of a circular cylinder
cm, then the volume of this square cm is :
is increased six times and the
pyramid is (a) 136 (b) 544
base area is decreased to one
(a) 3a3 cm3 (b) 3 2 a2 cm3 (c) 800 (d) 1280
wwM

ninth of its value. The factor by


which the lateral surface of the 282. Area of the base of a pyramid
(c) 3 3 a3 cm3 (d) 6a3 cm3 is 57 sq. cm. and height is 10
cylinder increases is
275. The base of a right pyramid is cm, then its volume (in cm 3),
1 2 3 a square of side 40 cm long. If is
(a) 2 (b) (c) (d)
2 3 2 the volume of the pyramid is (a) 570 (b) 390
268. If the radius of a sphere be 8000 cm3, then its height is : (c) 190 (d) 590
doubled. the area of its surface (a) 5 cm (b) 10 cm 283. The height of a right prism with
will become (c) 15 cm (d) 20 cm a square base is 15 cm. If the area
(a) Double 276.The base of a right prism is a of the total surface of the prism
(b) Three times trapezium. The length of the is 608 sq. cm, its volume is
(c) Four times parallel sides are 8 cm and 14 (a) 910 cm3 (b) 920 cm3
(d) None of the mentioned cm and the distance between (c) 960 cm3 (d) 980 cm3

Rakesh Yadav Readers Publication Pvt. Ltd. 386

For More Visit : www.LearnEngineering.in


For More Visit : www.LearnEngineering.in

284. The base of a right prism is an 291.A right prism stands on a base 300. The base of a prism is a right
equilateral triangle of side 8 cm of 6 cm side equilateral tri- angled triangle with two sides
and height of the prism is 10 angle and its volume is meeting at right angle are 5 cm
cm. Then the volume of the 81 3 cm 3. the height (in cm ) and 12 cm. The height of the
prism is of the prism is prism is 10 cm. The total sur-
(a) 320 3 cubic cm (a) 9 (b) 10 (c) 12 (d) 15 face area of the prism is
292.A right pyramid stands on a (a) 360 sq. cm (b) 300 sq. cm
(b) 160 3 cubic cm
(c) 330 sq. cm (d) 325 sq. cm
(c) 150 3 cubic cm square base of diagonal 10 2
301. The radius of a cylinder is 10
cm. If the height of the pyra- cm and height is 4 cm. The
(d) 300 3 cubic cm mid is 12 cm, the area (in cm2)
285. A prism has as the base a right number of centimetres that
of its slant surface is

ir
angled triangle whose sides ad- may be added either to the ra-
(a) 520(b) 420 (c) 360 (d) 260
jacent to the right angles are dius or to the height to get the

v.iSn
293. If the altitude of a right prism
10 cm and 12 cm long. The is 10 cm and its base is an same increase in the volume
height of the prism is 20 cm . equilateral triangle of side 12 of the cylinder is :
the density of the material of cm, then its total surface area (a) 25 (b) 4 (c) 5 (d) 16
the prism is 6gm/cubic cm. the (in cm2) is 302. If the area of the base, height

dnag
weight of the prism is and volume of a right prism be
(a) 6.4 kg (b) 7.2 kg (a) 5  3 3
  (b) 36 3
(c) 3.4 kg (d) 4.8 kg 3 3 
 
286. If the slant height of a right (c) 360 (d) 72 5  3
   2  p² cm², 10 3 cm and

eYrai
 
pyramid with square base is 4 294. A right pyramid stands on a
metre and the total slant sur- 7200 cm³ respectively, then
square base of side 16 cm and
fac e of the pyr amid is 12 the value of P (in cm) will be?
its height is 15 cm. The area
square metre, then the ratio of (in cm 2) of its slant surface is 2 3
total slant surface and area of
the base is :
(a) 16 : 3
(c) 32 : 9
(b) 24 : 5
(d) 12 : 3
snhe
(a) 514(b) 544 (c) 344 (d) 444
295. The base of a right prism is a
right angled triangle whose
sides are 5 cm, 12 cm and 13
(a) 4 (b)
3
(c) 3 (d)
2
303. If the base of right prism re-
mains same and the lateral
kgei
edges are halved, then its vol-
287. The length of each edge of a regu- cm. If the total surface area of
ume will be reduced by
lar tetrahedron is 12 cm. The the prism is 360 cm2, then its
area (in sq. cm) of the total sur- height (in cm) is (a) 33.33% (b) 50%
(c) 25% (d) 66%
ERna

face of the tetrahedron is (a) 10 (b) 12 (c) 9 (d) 11


296.A right pyramid 6 m high has a 304. The total surface ar ea of a
(a) 288 3 (b)144 2 reg ular triangular pyramid
square base of which the diago-
(c) 108 3 (d)144 3 with each edges of length 1 cm
nal is 1152 m. Volume of the
is?
aBryn

288. The base of right prism is a tri- pyramid is


angle whose perimeter is 28 (a) 144 m3 (b) 288 m3 4
(a) 2 cm² (b) 3 cm²
cm and the inradius of the tri- (c) 576 m3 (d) 1152 m3 2
angle is 4 cm. If the volume of 297. The height of the right pyramid (c) 4 cm² (d) 4 3 cm²
the prism is 366 cc, then its whose area of the base is 30 m2
305. Base of a right pyramid is a
height is and volume is 500 m2 is
Les

(a) 50 m (b) 60 m square of side 10 cm. If the


(a) 6 cm (b) 8 cm
wa.th

(c) 40 m (d) 20 m height of the pyramid is 12 cm,


(c) 4 cm
298.The base of a right. prism is an then its total surface area is
(d) None of these
equilateral triangle. If the lat- (a) 360 cm² (b)400 cm²
289.If the base of a right pyramid s eral surface area and volume (c)460 cm² (d) 260 cm²
triangle of sides 5 cm, 12 cm
is 120 cm2, 40 3 cm3 respec- 306. A right prism has a triangular
wwM

and 13 cm and its volume is


330 cm, then its height (in cm) tively then the side of base of base whose sides are 13 cm, 20
will be the prism is cm and 21 cm, If the altitude of
(a) 4 cm (b) 5 cm the prism is 9 cm, then its vol-
(a) 33 (b) 32 (c) 11 (d) 22
(c) 7 cm (d) 40 cm ume is
290.The base of a right pyramid is
299. Each edge of a regular tetrahe- (a) 1143 cm³ (b) 1314 cm³
equilateral tr iang le of side dron is 4 cm. its volume (in cu-
(c) 1413 cm³ (d) 1134 cm³
10 3 cm. If the total surface bic cm) is
307. Base of a prism of height 10 cm
are of the pyramid is 270 3 sq. 16 3 is square. Total surface area of
(a) (b) 16 3
cm. its height is 3 the prism is 192 sq. cm. The
volume of the prism is
(a) 12 3 cm (b) 10 cm 16 2
(c) (d) 16 2 (a) 120 cm3 (b) 640 cm3
(c) 10 3 cm (d) 12 cm 3 (c) 90 cm3 (d) 160 cm3

Rakesh Yadav Readers Publication Pvt. Ltd. 387

For More Visit : www.LearnEngineering.in


For More Visit : www.LearnEngineering.in

308. A right prism has triangular 316. Find the volue of a pyramid (a) 50 paise (b) 25 paise
base. If v be the number of ver- which is based on a square of (c) 75 paise (d) 1 paise
tices, e be the number of edges side 10cm & lateral edge of 322. The volume of a rectangular
and f be the number of faces of pyramid is 12 cm. block of stone is 10368 dm2 , its
the pr ism. The value of dimensions are in the ratio of
00 100 119
v e – f (a) (b) 3:2:1, If its entire surface is pol-
is 3 3 ished at 2 paise per dm2 , then
2
what is the total cost?
(a) 2 (b) 4 (c) 5 (d) 10 100 119
(c) (d) 100 119 (a) Rs. 31.68 (b) Rs. 31.50
309. The base of a right prism is a 9
(c) Rs. 63 (d) Rs. 63.36
trapezium whose lengths of 317. A rectangular water tank is 323. A rectangular water tank mea-
two parallel sides are 10 cm open at the top. Its capacity is

r
sure 15m × 6m at top and is
and 6 cm and distance be- 24 m3. Its length and breadth 10m deep. It is full of water. If

i
.iSn
tween them is 5 cm. If the are 4 m and 3 m respectively. water is drawn out lowering the
height of the prism is 8 cm, Ignoring the thickness of the level by 1 meter how much of
its volume is: material used for building the water has been drawn out?
(a) 300 cm³ (b) 300.5 cm³ tank, the total cost of painting

agv
(a) 90,000 litres
(c) 320 cm³ (d) 310 cm³ the inner and outer surfaces of
(b) 45,000 litres
310. Base of a right prism is a rect- the tank at the rate of Rs. 10

ridn
angle, the ratio of whose length per m2 is: (c) 80,000 litres
and breadth is 3 : 2. If the height (a) Rs. 400 (b) Rs. 500 (d) 40,000 litre
of the prism is 12 cm and total (c) Rs. 600 (d) Rs. 800 324. A rectangular tank is 45 m long

eeYa
surface area is 288 sq. cm, the and 26 m broad. Water flows
318. If V be the volume and S the
volume of the prism is: surface area of a cuboid of di- into it through a pipe whose
(a) 288 cm (b) 290 cm³ cross section is 13 cm2, at the
1 rate of 9 km/hour. How much
(c) 286 cm³ (d) 291 cm³ mensions a, b and c then is
geisnh V will the level of the water rise
311. Height of a prism-shaped part of
a machine is 8 cm and its base equal to: in the tank in 15 min?
is an isosceles triangle, whose (a) 0.0016m (b) 0.0020m
S
each of the equal sides is 5 cm (a) (a+b+c) (c) 0.0025m (d) 0.0018
2 325. The diagonals of the three faces
and remaining side is 6 cm. The
Enak

volume of the part is of a cuboid are x, y and z respec-


2  1 1 1
(a) 90 cm³ (b) 96 cm³ (b)  + +  tiv ely. Find the volume of
S a b c cuboid?
(c) 120 cm³ (d) 86 cm³
R

312. Find the total surface area of 2S xyz


pyramid which is based on a (c) (a)
a+b+c 2 2
aryn

equal  of side 18 3 & the (d) 2S(a + b + c)


height of pyramid is 12 cm. 319. An open box is made of wood 3 (b)  y +z –x z +x
2 2 2 2 2
–y2 x 2 +y2 –z2 
Les B

cm thick. Its external length is 2 2


(a) 124 3 (b) 624 3
1.46 m, breadth 1.16 m and
(c) 648 3 (d) 405 3 height 8.3 dm. The cost of paint-  y +z z +x x
2 2 2 2 2
+y2 
ing the inner surface of the box (c) 2 2
313. The base of a right prism is a
wa.th

at 50 paise per 100 cm2 is:


 ABCD. If the volume of prism (d) None of these
is 2070. Then find the lateral (a) Rs. 138.50 (b) Rs. 277 326. The same string, when wound
surface area. AB = 9, BC = 14, (c) Rs. 415.50 (d) Rs. 554 on the exterior four walls of a
CD = 13, DA = 12 DAB = 90°. 320. The areas of three adjacent cube of side n cm, starting at
wwM

(a) 720 (b) 540 faces of a cuboid are x, y & z point C and ending at point D,
(c) 920 (d) 960 square units respectively. If can given exactly one turn. The
314. Find the volue of pyramid which the volume of the cuboid be v length of the string, (in cm) is.
is base d on a e quilater al cubic units, then the correct
relation between v, x, y, z is: D
traingle of side 4 cm & height
(a) v2 = xyz (b) v3 = xyz
of pyramid is 20 3 cm.
(c) v2 = x3y3z3 (d) v3 = x2y2z2
(a) 100 (b) 160 321. 1 m3 piece of copper is melted
(c) 80 (d) 40 and recast into a square cross
C
315. Find the total surface area of section bar 36 m long. An ex-
pyramid of 4 cm height which act cube is cut off from this bar. (a) 2n (b) 17n
is based on a square of side 6cm. If 1 m3 of copper cost Rs. 108,
(a) 48 (b) 72 (c) 96 (d) 120 (c) n (d) 13n
then the cost of the cube is.

Rakesh Yadav Readers Publication Pvt. Ltd. 388

For More Visit : www.LearnEngineering.in


For More Visit : www.LearnEngineering.in

327. A reservoir is supplied water by 100 cm is rolled into cylinder 339. The base radius and height of
a pipe 6 cm in diameter. How of height 50 cm. If the cost of a c one is 5 cm and 2 5cm
many pipes of 1.5 cm diameter painting the cylinder is Rs. 50 respectively.if the cone is cut
would disc harg e the same per square meter, then what
will be the cost of painting the parallel to its base at a height
quantity, supposing the veloc-
surface of the cylinder? of h from the base. If the vol-
ity of water is same?
(a) 8 (b) 12 (c) 16 (d) 20 (a) Rs. 25.00 (b) Rs. 37.50 ume of this frustrum is 110 cm3
328. Given a solid cylinder of radius (c) Rs 75.00 (d) Rs. 87.50 find the radius of smaller cone?
334. Sixteen cylindrical cans, each (a) (104)1/3 cm (b) (104)1/2 cm
10 cm and length 1000 cm, a
with a radius of 1 unit, are
cylindrical hole is made into it (c) 5 cm
placed inside a cardboard box
to obtain a cylindrical shell of four in a row. If the cans touch (d) None of these
uniform thickness and having the adjacent cans and or the 340. A hemisphe reic al b owl is

ir
volume equal to one-fourth of walls of the box, then which of 176cm round the brim. suppos-

v.iSn
the original cy lind er. The the following could be the inte-
thickness of the cylindrical ing it to be half full, how many
rior area of the bottom of the
shell is: persons may be swerved from
box in square units?
(a) 16 (b) 32 (c) 64 (d) 128 it in hemispherical glasses 4
(a) 5  
5 – 2 cm cm in diameter at the top?

dnag
335. Find the volume of a rig ht
c i r c u l a r c o ne f o r m e d b y (a) 1372 (b) 1272
(b) 5(2 – 3) cm joining the edges of a sector of (c) 1172 (d) 1472
a circle of radius 4cm where the
(c) 5 cm (d) 5 2 cm angle of the sector is 90°. 341. A sphere of radius 3 cm is

eYrai
329. A well of radius 'r' is dug 20 m dropped into a cylindrical
2 3 2 2π vessel partly filled with water.
deep and the earth taken out (a) cm 3 (b) cm 3
is spread all around it to a width π 3 The radius of the vesasel is 6
of 1 m to form an embankment. π 5 3 cm.If the sphere is submerged
The height of the embankment
is 5 m then find the value of 'r':

(a)
1 5
(b)
1 5
(c)
3
cm 3
snhe
(d)
π
cm 3
336. A sector of circle of radius 3cm
has an angle of 120º. if it is
completely, then the surface of
the water is raised by
1 1
kgei
2 4 modulated into a cone, find the (a) cm (b) cm
4 2
5 –1 5 –1 volume of the cone.
(c) (d) (c) 1 cm (d) 2cm
2 4 π 2 2π 342. Let A and B be two solid spheres
ERna

330. A cylinder is filled to 4/5th of (a) cm 3 (b) cm 3 area of B is 300% higher than
3 3
volume. If is then tilted so that surface area of A.
level of water coincides with The volume of A is found to be
one edge of its bottom and top 2 3 3
(c) cm 3 (d) cm 3 k% lower than the volume of
edge of the opposite side. In the π π
aBryn

B.The value of k must be


process, 30 litre of the water is 337. If the slant height and the ra- (a) 85.5 (b) 92.5
spilled. What is the value of the dius of the base of a right cir- (c) 90.5 (d) 87.5
cylinder? cular cone are H and r respec-
(a) 75 litre (b) 96 litre 343. The base of a prism is a regu-
tively then the ratio of the ar- lar hexagon. If every edge of the
(c) Data insufficient eas of the lateral surface and prism measures 1 metre and
Les

(d) 100 litre


the base is: height is 1 metre, than volume
331. A monument has 50 cylindri-
wa.th

cal pillars each of diameter 50 (a) 2H :r (b) H : r of the prism is


cm and height 4 m. what will (c) H : 2r (d) H2 : r2
338. A sector of a circle of radius 3 2 3 3
be the labour charges for get- (a) cu m (b) cu m
ting these pillars cleared at the 15cm has the angle 120.It is 2 2
rate of 50 paise per m2 (Use π
wwM

rolled up so that two bounding


= 3.14). radii are joined toghether to 6 2 5 3
(c) cu m (d) cu m
(a) Rs. 237 (b) Rs. 157 form a cone.the volume of the 5 2
(c) Rs. 257 (d) Rs. 353 cone is. 344. The base of a right prism is a
332. A right circular cyindrical tank pentagon whose sides are in
has the storage capacity 38808
ml. If the radius of the base of
 
(a) 250 2 π cm 3
the ratio 1: 2 : 2 :1:2 and its
the cylinder is three fourth of
the height what is the radius (b) 100 2 π cm 3 height is 10 cm. If the longest
side of the base be 6 cm, the
of base? volume of the prism is
(a) 28 cm (b) 56 cm (c) 250 2 π /3 cm 3
(a) 270 cm3 (b) 360 cm3
(c) 21 cm (d) 42 cm
(c) 540 cm3
333. A rectangular piece of iron
sheet measuring 50 cm and
 
(d)  100 2 π / 3 cm 3 (d) None of these

Rakesh Yadav Readers Publication Pvt. Ltd. 389

For More Visit : www.LearnEngineering.in


For More Visit : www.LearnEngineering.in

345. There are two prism, one has 352. There is a pyramid on a base 359. A slab of ice 8 inches in length
equilateral triangle as a base which is a regular hexagon of 11 inches in breadth, and 2
and the other regular hexagon. side 2a. If every slant edge of inches thick was melted and
If both of the prisms have equal 5a resolidified in the form of a rod
heights and volumes, then find this pyramid is of length , of 8 inches diamete r. The
2
the ratio between the length of then the volume of this pyra- length of such a rod, in inches,
each side at their bases. mid is. is nearest to.
(a) 1 : 6 (b) (a) 3 (b) 3.5 (c) 4 (d) 4.5
6 :1 (a) 3a3 (b) 3a 2 2
360. A storage tank consists of a
(c) 3 :2 (d) 2 : 3 (c) 3a 3 3 (d) 6a3 circular c ylinder with a
346. The base of a right prism is a 353. If the area of the base of a regu- hemisphere adjoined on either
side. If the external diameter

r
trapezium. The lengths of the lar he xagonal pyramid is
parallel sides are 8 cm and 14 of the cylinder be 14 m and its

i
2
96 3 m and the area of one of

.iSn
cm and the distance between length be 50 m, then what will
the parallel sides is 8 cm. If the its side faces is 32 3 m3, then be the cost of painting it at the
volume of the prism is 1056 the volume of the pyramid is: rate of Rs. 10 per sq m?
cm 3, then the height of the (a) Rs. 38160 (b) Rs. 28160

agv
(a) 380 3 m 3 (b) 382 2 m 3
prism is (c) Rs. 39160 (d) None of these
(a) 44 cm (b) 16.5 cm (c) 384 3 m 3 (d) 386 3 m 3 361. The diameter of the iron ball

ridn
(c) 12 cm (d) 10.56 cm 354. What part of a ditch, 48 metres used for the shotput game is 14
347. If the base of a right rectangu- long 16.5 metres broad and 4 cm. It is melted and then a solid
lar prism is left unchanged and metres deep can be filled by the

eeYa
1
the measure of the late ral sand got by digging a cylindri- cylinder of height 2 cm is
edges are doubled, then its vol- cal tunnel of diameter 4 metres 3
ume will be and length 56 metres? made. What will be the diam-
(a) unchanged (b) tripled eter of the base of the cylinder?
22 
(c) doubled (d) quadrupled
geisnh

(a) 14 cm (b) 28 cm
 useπ  7 
348. Prism has as the base a right
14 28
angled triangle whose sides ad- 1 2 7 8 (c) cm (d) cm
jacent to the right angles are (a) (b) (c) (d) 3 3
9 9 9 9
10 cm and 12 cm long. The 355. A cylindrical rod of iron whose 362. If the area of the circular shell
Enak

height of the prism is 20 cm. height is eight times its radius having inner and outer radii of
The density of the material of is melted and cast into spheri- 8 cm and 12 cm respectively is
the prism is 6 gm. cubic cm. cal balls each of half the radius equal to the total surface area
R

The weight of the prism is. of the cylinder. The number of of cylinder of radius R 1 and
(a) 6.4 kg (b) 7.2 kg such pherical balls is height h, then h, in terms or
(c) 3.4 kg (d) 4.8 kg R1 will be.
aryn

(a) 12 (b) 16 (c) 24 (d) 48


349. The perimeter of the triangu-
356. Water flows at the rate of 10 R12 –40
lar base of a right prism is 15 3R12 –30
Les B

meters per minute from a cy- (a) (b)


cm and radius of the incircle of
lindrical pipe 5 mm in diam- 7R1 R 21
the triangular base is 3 cm. If eter. How long it take to fill up
the volume of the prism be 270 a conical vessel whose diam- 30 – R1 40 – R12
cm 3, then the height of the eter at the base is 30 cm and (c) R 21 (d)
R1
prism is-
wa.th

depth 24 cm?
(a) 6 cm (b) 7.5 cm 363. Two solid right cones of equal
(a) 28 minutes 48 seconds
(c) 10 cm (d) 12 cm heights are of radii r1 and r2 are
(b) 51 minutes 12 seconds
350. The base of a right prism is an melted and made to form a solid
(c) 51 minutes 24 seconds
equilateral tirang le. If its sphere of radius R. Then the
(d) 28 minutes 36 second
wwM

height is one-fourth and each height of the cone is:


side of the base is tripled, then 357. A semi-circular sheet of metal
of diameter 28 cm is bent into 4R 2 4R
the ratio of the volumes of the (a) (b)
an open conical cup. The depth r12  r22 r1  r2
old to the new prism is-
of the cup is aproximately :
(a) 4 : 3 (b) 1 : 4 4R 3 R2
(c) 1 : 2 (d) 4 : 9 (a) 11 cm (b) 12 cm
(c) 2 2 (d) 2
351. A right pyramid is on a regular (c) 13 cm (d) 14 cm r  r2
1 r1  r22
hexagonal base. Each side of 358. The height of a right prism with 364. A solid wooden toy is in the
the base is 10 m and the height a square base 15 cm. If the to- shape of a right circular cone
is 30 m. The volume of the tal S.A. of prism of 608 cm2. The mounted on a hemisphere. If
pyramid is find its volume. the radius of the hemisphere
(a) 2500 m3 (b) 2550 m3 (a) 480 (b) 460 is 4.2 cm and the total height
(c) 2598 m3 (d) 5196 m3 (c) 1500 (d) 960 of the toy is 10.2 cm, find the

Rakesh Yadav Readers Publication Pvt. Ltd. 390

For More Visit : www.LearnEngineering.in


For More Visit : www.LearnEngineering.in

volume of the wooden by 370. In a shower, 10 cm of rain falls. are of the largest area pos-
(nearly). What will be the volume of wa- sible?
(a) 104 cm3 (b) 162 cm3 ter that falls on 1 hectare area (a) 85.71 cm² (b) 257.14 cm²
(c) 427 cm3 (d) 266 cm3 of ground? (c) 514.28 cm² (d) 331.33 cm²
365. The volume of a cylinder and a (a) 500 m³ (b) 650 m³ 378.The areas of three adjacent
cone are in the ratio 3 : 1. Find (c) 1000 m³ (d) 750 m³ faces of a cuboid are x, y, z. If
their diameters and then com- 371. Seven equal cubes each of side the volume is V, then V² will
pare them when their heights 5 cm are joined end to end. Find be equal to
are equal. the surface area of the result- (a) xy/z (b) yz/x²
(a) Diameter of cylinder = 2 ing cuboid. (c) x²y²/z² (d) xyz
times diameter of cone (a) 750 cm² (b) 1500 cm² 379. The dimensions of a field are
(c) 2250 cm² (d) 700 cm²

ir
(b) Diameter of cylinder = Di- 20 m by 9m. A pit 10 m long,
ameter of cone 372. In a swimming pool measuring 4.5 m wide and 3m deep is dug

v.iSn
(c) Diameter of cylinder > Di- 90 m by 40m, 150 men take a in one corner of the field and
ameter of cone dip. If the average displace- the soil re move d has be en
(d) Diameter of cylinder < Di- ment of water by a man is 8 evenly spread over the remain-
ameter of cone cubic metres, what will be rise ing area of the field. What will

dnag
366. A oil funnel made of tin sheet in water level? be the rise in the height of field
consists of a 10 cm long cylindri- (a) 30 cm (b) 50 cm as a result of this operation?
cal portion attatched to 22 cm, di- (c) 20 cm (d) 33.33 cm (a) 1m (b) 2m (c) 3m (d) 4m
ameter of the cylindrical portion 373.A conical tent is to 380. A vessel is in the form of a hol-

eYrai
is 8 cm and the diameter of the accommodate 10 persons. Each low cylinde r mounte d on a
top of the funnel is 18 cm, find the person must have 6 m² space hemispherical bowl. The diam-
area of the tin sheet required to to sit and 30 m³ of air to breath. eter of the sphere is 14 cm and
make a funnel. What will be the height of the the total height of the vessel
18cm cone ?
(a) 37.5 m
(c) 75 m
snhe (b) 150 m
is 13cm. Find the capacity of
the vessel.
(Take = 22/7)
(d) None of these (a) 321.33 cm (b) 1642.67 cm³
kgei
22cm 374. A hollow spherical shell is made (c) 1232 cm³ (d) 1632.33 cm³
10cm of a metal of density 4.9 g/cm³. 381. A circular tent is cylindrical to
If its internal and external ra- a height of 3 metres and conical
ERna

8cm dii are 10 cm and 12 cm respec- above it. If its diameter is 105
(a) 728.57 cm3 (b) 782.57 cm3 tively, find the weight of the m and the slant height of the
(c) 872.57 cm3 (d) 827.57 cm3 shell. conical portion is 53 m,
367. A conical flask has radius a cm (Take = 3.1416) calculate the length of the
aBryn

and height h cm. It was com- (a) 5016 gm (b) 1416.8 gm canvas 6 m wide to make the
pletely filled with milk. The (c) 14942.28gm (d) 5667.1 gm required tent.
milk is poured into a cylindri- 375. A spherical cannon ball, 28 cm (a) 3894 m (b) 973.5 m
cal therefore flask whose base in diameter, is melted and cast (c) 1947 m (d) 1800 m
radius is p cm. What will be the into a right circular conical 382. A steel sphere of radius 4 cm
Les

height of the soultion level in mould the base of which is 35 is drawn into a wire of diam-
the flask? cm in diam eter . Find the eter 4 mm. Find the length of
wa.th

height of the cone correct up to wire.


a 2h 3hp 2 two places of decimals.
(a) cm (b) cm (a) 10,665 mm
3p2 a2 (a) 8.96 cm (b) 35.84 cm (b) 42.660 mm
p2 3a 2 (c) 5.97 cm (d) 17.9 cm (c) 21,333 mm
wwM

(c) cm (d) hp 2 cm 376. A rope is wound round the out- (d) 14,220 mm
3h 2
368. The perimeter of an equilateral side of a circular drum whose 383. A cylinder and a cone having
diameter is 70 cm and a bucket equal diameter of their bases
triangle is 72 3 cm. Find its is tied to the other end of the are placed in the Qutab Minar
height. rope. Find the number of revo- one on the other, with the cyl-
(a) 63 metres (b) 24 metres lutions made by the drum if the inder placed in the bottom. If
(c) 18 metres (d) 36 metres bucket is raised by 11 m. their curved surface area are
369. A pit 7.5 metre long, 6 metre (a) 10 (b) 2.5 (c) 5 (d) 5.5 in the ratio of 8 : 5, find the
wide and 1.5 metre deep is dug 377. A cube whose edge is 20 cm ratio of their heights. Assume
in a field. Find the volume of long has circle on each of its
soil removed in cubic metres. the height of the cylinder to be
faces painted black. What is the equal to the radius of Qutab
(a) 135 m³ (b) 101.25 m³ total area of the unpainted sur- Minar. (Assume Qutab Minar to
(c) 50.625 m³ (d) 67.5 m³ face of the cube if the circles

Rakesh Yadav Readers Publication Pvt. Ltd. 391

For More Visit : www.LearnEngineering.in


For More Visit : www.LearnEngineering.in

be hav ing same r ad ius sphere is 4.2 cm and the total mon diameter is 3.5 cm and
throughout). height of the toy is 10.2 cm, the heights of conical and cy-
(a) 1 : 4 (b) 3 : 4 find the volume of wood used lindrical portion are respec-
(c) 4 : 3 (d) 2 : 3 in the toy. tively 6 cm and 10 cm. Find the
384. If the curved surface area of a (a) 353.72 cm³(b) 266.11 cm³ volume of the solid.
cone is thrice that of another (c) 532.22 cm³(d) 133.55 cm³ (Use = 3.14)
cone and slant height of the 391.A cylindrical container whose (a) 117 cm² (b) 234 cm²
second cone is thrice that of diameter is12 cm and height is (c) 58.5 cm²
the first, find the ratio of the 15 cm, is filled with ice cream. (d) None of these
area of their base. The whole ice-cream is distrib-
(a) 81 : 1 (b) 9 : 1 397. A right elliptical cylinder full of
uted to 10 children in equal petrol has its widest elliptical
(c) 3 : 1 (d) 27 : 1 cones having hemispherical

r
side 2.4m and the shortest
385. A solid sphere of radius 6 cm is tops. If the height of the coni-
1.6m. Its height is 7m. Find the

i
.iSn
melted into a hollow cylinder of cal portion is twice the diam-
uniform thickness. If external time required to empty half the
eter of the cylindrical container
radius of the base of the cylin- its base, find the diameter of the tank through a hose of diam-
der is 5 cm and its height is 32 ice-cream cone. eter 4cm if the rate of flow of

agv
cm, find the uniform thickness (a) 6 cm (b) 13 cm petrol is 120 m/min
of the cylinder. (c) 3 cm (d) 18 cm (a) 60 min (b) 90 min

ridn
(a) 2 cm (b) 3 cm 392.A solid is in the form of a cylin- (c) 75 min (d) 70 min
(c) 1 cm (d) 3.5 cm der with hemispherical ends. 398. The radius of a right circular
386. A hollow sphere of external and The total height of the solid is cylinder is increased by 50%.

eeYa
internal radius 6 cm and 4 cm 19 cm and the diameter of the Find the percentage increase
respectively is melted into a cylinder is 7 cm. Find the total in volume
cone of base diameter 8cm. surface area of the solid. (Use (a) 120% (b) 75%
Find the height of the cone = 22/7). (c) 150% (d) 125%
(a) 25 cm (b) 35 cm
geisnh
(a) 398.75 cm²(b) 418 cm² 399. Water flows out at the rate of
(c) 30 cm (d) 38 cm (c) 444 cm² (d) 412 cm² 10m/min from a cylindrical
387. Three equal cubes are placed 393.A cone, a hemisphere and a
adjacently in a row. Find the pipe of diameter 5 mm. Find the
cylinder stand on equal bases time taken to fill a conical tank
ratio of total surface area of the and hav e the s am e height. whose diameter at the surface is
Enak

new cuboid to that of the sum


What is the ratio of their vol- 40 cm and depth 24 cm.
of the s urface areas of the
umes? (a) 50 min (b) 102.4 min
three cubes.
(a) 2 : 1 : 3 (b) 2.5 : 1 : 3
(a) 7 : 9 (b) 49 : 81 (c) 51.2 min (d) 25.6 min
R

(c) 1 : 2 : 3 (d) 1.5 : 2 : 3


(c) 9 : 7 (d) 27 : 23 400. The section of a solid right cir-
394.The internal and external di-
388. If V be the volume of a cuboid cular cone by a plane contain-
am eter s of a hollow he mi-
aryn

of dimension x, y, z and A is its ing vertex and perpendicular to


spherical vessel are 24 cm and
surface, then A/V will be equal base is an equilateral triangle
25 cm respectively. The cost of
Les B

to of side 12 cm. Find the volume


painting 1 cm² of the surface
(a) x²y²z² of the cone.
is ` 0.05. Find the total cost of
(b) 1/2(1/xy+1/xz+1/yz) (a) 72 cm³ (b) 144 cm³
painting the vessel all over.
(c) None of these
(Take  = 22/7)
(d) 1/xyz (c) 74 cm³ (d) 72 3 π cm³
wa.th

(a) ` 97.65 (b) ` 86.4


389. A toy is in the shape of a right 401. Iron weight 8 times the weight
circular cylinder with a hemi- (c) ` 184 (d) ` 96.28
395. A cyclindrical cane whose base of oak. Find the diameter of an
sphere on one end and a cone iron ball whose weight is equal
on the other. The height and is horizontal is of internal ra-
dius 3.5 cm contain sufficient to that of a ball of oak 18 cm in
wwM

radius of the cylindrical part are


water so that when a solid diameter.
13 cm and 5cm respectively.
The radii of the hemispherical sphere of max. size is placed, (a) 4.5 cm (b) 9 cm
and conical parts are the same water just immersed it. Calcu- (c) 12 cm (d) 15 cm
as that of the cylindrical part. late the de pth of 402. A piece of squared timber is 7
Calculate the surface area of water in the cane before the metres long and 0.1 metre both
the toy if the height of conical sphere was put. in width and thickness. What
part is 12 cm. 5 7 4 8 is its weight at the rate of 950
(a) 1440 cm² (b) 385 cm² (a) (b) (c) (d) kg per cubic metres?
2 3 3 3
(c) 1580 cm² (d) 770 cm² 396. A solid is in the form of a right (a) 66 kg (b) 67 kg
390. A solid wooden toy is in the form circular cylinder with a hemi- (c) 66.5 kg (d) 68.5 kg
of a cone mounted on a hemi- sphere at one end and a cone 403. How many c ubic metres of
sphere. If the radii of the hemi- at the other end. Their com- masonry are there in a wall 81

Rakesh Yadav Readers Publication Pvt. Ltd. 392

For More Visit : www.LearnEngineering.in


For More Visit : www.LearnEngineering.in

metres long, 4 metres high and (a) 4620 cu m (b) 4630 cu m 418. Find the weight of a lead pipe
0.2 metre thick. (c) 4520 cu m (d) 4830 cu m 3.5 metres long, if the external
(a) 64.8 cub m (b) 69 cub m 411. If the diameter of the base of a diameter of the pipe is 2.4 cm
(c) 68 cub m (d) 68.9 cub m and the thickness of the lead
closed right circular cylinder is
is 2 mm and 1 cc of lead weight
404. A river 10 metres deep and 200 equal to its height h, then its
11.4 gm.
metres wide is flowing at the whole surface area is: (a) 5.5 kg (b) 5 kg
1 4 (c) 8 kg (d) 10 kg
rate of 4 km/hr. Find how (a) 2ph2 (b) ph2 419. A closed rectangular box has
2 3
many cubic m of water run into inner dimensions 24 cm by 12
3 2 cm by 10 cm. Calculate its
the sea per second. (c) ph (d) ph 2
2 capacity and the area of tin foil
(a) 2500 cub metres

ir
needed to line its inner surface.
(b) 2000 cub metres 412. How many bullets can be made (a) 2680 cu cm, 1296 sq cm

v.iSn
(c) 2200 cub metres out of a cube of lead whose edge (b) 2880 cu cm, 1396 sq cm
(d) None of these measures 22 cm, each bullet (c) 2880 cu cm, 1296 sq cm
405. A cistern is constructed to hold being 2 cm in diameter? (d) 2860 cu cm, 1296 sq cm
200 litres, and the base of the (a) 5324 (b) 2662 420. The dimension of an open box

dnag
cistern is a square metre. what (c) 1347 (d) 2541 are 52 cm, 40 cm and 29 cm. Its
is the depth of the cistern? A 413. A cylindrical vessel 60 cm in thickness is 2 cm. If 1 cm3 of
cubic metre is 1000 litres. diameter is partially filled with metal used in the box weight 0.5
(a) 50 cm (b) 20 cm water. A sp here , 30 cm in gm, the weight of the box is:

eYrai
(c) 25 cm (d) 40 cm radius is gently dropped into the (a) 8.56 kg (b) 7.76 kg
406. A field is 500 metres long and vessle. To what further height (c) 7.756 kg (d) 6.832 kg
30 metres broad and a tank 50 will water in the cylinder rise? 421. Half cubic metre of gold sheet
metres long, 20 metres broad (a) 15 cm is extended by hammering so
as to cover an area of 1 hectare.
and 14 metres deep is dug in
the field, and the earth taken
out of it is spread evenly over
the field, How much is the level
(b) 30 cm
(c) 40 cm snhe
(d) Can't be determined
414. Th e d i f f er en ce b et w een t h e
Find the thickness of the gold.
(a) 0.05 cm
(c) 0.005 cm
(b) 0.5 cm
(d) 0.0005 cm
422. Two cubic metres of gold are
kgei
of the field raised? ou tside and inside su r face of a extended by hammering so as
(a) 0.5 m (b) 1.5 m cylindr ical m etallic pipe, 14cm to cove r an are a of twe lve
(c) 1 m (d) 2 m long, is 44cm 2. If the pipe is made hectares. Find the thickness of
ERna

407. Find the volume and surface of 99cm 3 of m etal. Find the ou ter gold.
area of a cube, whose each r adii of the pipe? (a) 0.017 cm (b) 0.0017 cm
edge measures 25 cm. (a) 2cm (b) 2.5cm (c) 1.7 cm (d) 0.17 cm
(a) 15265 cu cm, 3750 sq cm (c) 4cm (d) 5cm 423. A cub of silver is drawn into a
aBryn

(b) 15625 cu cm, 2500 sq cm 415. How many bullets can be made 1
(c) 15625 cu cm, 3850 sq cm out of a cube of lead whose edge wire mm in diameter, find
10
(d) Data inadequate measures 22 cm, each bullet the length of the wire.
408. The three co-terminus edges of being 2 cm in deameter? (p = 3.1416)
a rectangular solid are 36, 75 (a) 2341 (b) 2641 (a) 128 metres (b) 127.3 metres
Les

and 80 cm respectively. Find (c) 2541 (d) 2451 (c) 129.3 metres (d) 128.3 metres
the edge of a cube which will 416. A right cylindrical vessel is full
wa.th

424. A hollow cylindrical tube open


be of the same capacity? with water. How many right at both ends ins made of iron 4
(a) 70 cm (b) 36 cm cones having same diameter cm thic k. I f the internal
(c) 60 cm and height as those of right diameter be 40 cm and the
(d) Data inadequate cylinder will be needed to store length of the tube be 144 cm,
wwM

409. A cube of metal each edge of that water? find the volume of iron in it.
which measures 5 cm, weighs (a) 2 (b) 3 (c) 4 (d) 5 (a) 25344p (b) 23544p
0.625 kg. What is the length of 417. An open rectgangular cistern (c) 26344p (d) None of these
each edge of a cube of the same when measured from out side 425. A hollow cylindrical tube open
metal which weighs 40 kg? is 1 m 35 cm long; 1 m 8 cm at both ends is made of iron 2
broad and 90 cm deep, and is cm thic k. I f the internal
(a) 20 cm (b) 25 cm
made of iron 2.5 cm thick. Find diameter be 33 cm and the
(c) 15 cm (d) 30 cm (i) the capacity of the cistern, length of the tube be 70 cm, find
410. The sum of the radius of the (ii) the volume of the iron used. the volume of iron in it.
base and the height of a solid (a) 1171625 cu cm, 140575 cu cm (a) 12400 cu cm
cylinder is 37 m. If the total (b) 1711625 cu cm, 104575 cu cm (b) 15400 cu cm
surface area of the cylinder be (c) 1171625 cu cm, 145075 cu cm (c) 13800 cu cm
1628 sq m, find the volume. (d) None of these (d) 16400 cu cm

Rakesh Yadav Readers Publication Pvt. Ltd. 393

For More Visit : www.LearnEngineering.in


For More Visit : www.LearnEngineering.in

426. One cm of rain has fallen on 2 433. The external dimensions of a is increased by 1016 cm3. Find
square km of land. Assuming wooden box closed at both ends the side of the cube. If each side
that 25% of the raindrops could are 24 cm, 16 cm and 10 cm of it is decreased by 2 cm, by
have b een collected and respectively and thickness of how much will the volume
contained in a pool having a 50 decrease?
the wood is 5 mm. If the empty
m × 5 m base, by what level (a) 12 cm, 729 cm3
box weighs 7.35 kg, find the
would the water level in the pool (b) 8 cm, 512 cm3
weight of 1 cubic cm of wood:
have in creased? (c) 9 cm, 729 cm3
(a) 20 m (b) 40 m (a) 10 g (b) 12.5 g (d) 12 cm, 728 cm3
(c) 25 m (c) 27 g (d) 15 g 439. Three equal cubes are placed
(d) Data inadequate 434. The internal dimensions of a adjacently in a row. Find the
tank are 12 dm, 8 dm and 5 dm. ratio of total surface area of the

r
427. Two cm of rain has fallen on a
square km of land. Assuming How many cubes each of edge resulting cuboid to that of the

i
.iSn
that 40% of the raindrops could 7 cm can be placed in the tank total surface areas of the three
have b een collected and with faces parallel to the sides cubes:
contained in a pool having a of the tank. Find also, how (a) 5 : 7 (b) 7 : 9
(c) 9 : 7 (d) None of these

agv
200 m × 20 m base, by what much space is left unoccupied?
level would the water level in 440. A hollow square shaped tube
(a) 35 ; 113 dm3
the pool have incrased? open at both ends is made of

ridn
(b) 1313 ; 31.13 dm3 iron. The internal square is of
(a) 2 m (b) 1 m
(c) 35 ; 31.013 dm3 5 cm side and the length of the
(c) 4 m (d) 1.5 m
(d) 1309 ; 13.31 dm3 tube is 8 cm. There are 192 cm3

eeYa
428. The length of a tank is thrice
435. The length, breadth and height of iron in the tube. Find the
that of breadth, which is 256
of box are 2 m, 1.5 m and 80 cm thickness:
cm deep and holds 3000 L
respectively. What would be the (a) 2 cm (b) 0.5 cm
water. What is the base area of
cost of canvas to cover it up (c) 1 cm
the tank? (1000 L = 1 cubic
geisnh
(d) can't be determined
metre) fully, if one square metre of
441. A cube of 11 cm e dge is
(a) 111775 m2 (b) 1171.875 m2 canvas costs Rs. 25.00?
2
im mers ed c ompletely in a
(c) 1.171875 m (a) Rs. 260 (b) Rs. 290
rectangular vessel containing
(d) None of these (c) Rs. 285
Enak

water. If the dimesions of base


429. A lid of rectangular box of sides (d) None of these are 15 cm and 12 cm. Find the
39.5 cm by 9.35 cm by 9.35 cm 436. Three cubes each of edge 3 cm ris e in water level in the
is sealed all around with tape
long are placed together as vessel:
R

suc h that there is an


shown in the adjoining digure. (a) 6.85 cm (b) 7 cm
ovrelapping of 3.75 cm of the
Find the surface area of the (c) 7.31 cm (d) 7.39 cm
tape. What is the length of the
aryn

tape used? cuboid so formed: 442. A rectanular tank 25 cm long


and 20 cm wide contains water
(a) 111.54 cm (b) 101.45 cm
Les B

to a depth of 5 cm. A metal cube


(c) 110.45 cm (d) None of these
of side 10 cm is placed in the
430. A cistern from inside is 12.5 m 3cm m tank so that one face of the
long, 8.5 m broad and 4m high 3c
3cm 3cm 3cm cube rests on the bottom of the
and is open at top. Find the cost tank. Find how many litres of
wa.th

(a) 182 sq. cm (b) 162 sq. cm


of cementing the inside of a water must be poured into the
cistern at Rs. 24 per sq. m: (c) 126 sq. cm
(d) None of these tank so as to just cover the
(a) Rs. 6582 (b) Rs. 8256 cube?
437. A room is 36 m long, 12 m wide
(c) Rs. 7752 (d) Rs. 8752 (a) 1 L (b) 1.5 L
and 10 m high. It has 6 window,
wwM

431. 250 men took a dip in a water (c) 2 L (d) 2.5 L


each 3 m × 2.5 m; one door 9.5
tank at a time, which is 80m × m × 6 m and one fire chimney 443. A rectangular block has length
50m. What is the rise in the 4 m × 4 .5 m . Find the 10 cm, breadth 8 cm and height
water level if the average expenditure of papering its 2 cm. From this block, a cubical
displacement of 1 man is 4 m3? walls at the rate of 70 paise per hole of side 2 cm is drilled out.
(a) 22 cm (b) 25 cm metre, if the width of the paper Find the v olume and the
(c) 18 cm (d) 30 cm is 1.2 m: surface area of the remaining
432. The edge of a cube is increased (a) Rs. 490 (b) Rs. 690 solid:
by 100% the surface area of the (c) Rs. 1000 (a) 152 cm2, 512 cm2
cube is increased by: (d) None of these (b) 125 cm2, 215 cm2
(a) 100% (b) 200% 438. When each side of a cube is (c) 152 cm2, 240cm2
(c) 300% (d) 400% increased by 2 cm, the volume (d) 125 cm2, 512 cm2

Rakesh Yadav Readers Publication Pvt. Ltd. 394

For More Visit : www.LearnEngineering.in


For More Visit : www.LearnEngineering.in

444. How many bricks (number near 450. A conical vessel has a capacity cm, the height of the water
to next hundre d) will be of 15 L of milk. Its height is 50 level rises in it is:
required to build a wall 30 m cm and base radius is 25 cm. 2 2
long, 30 cm thick and 5m high How much m ilk can be (a) 2 cm (b) 3 cm
with a provision of 2doors, each 3 3
contained in a ves sel in
4 m × 2.5 m and each brick cylindrical form having the 2
being 20 cm × 16 cm × 8 cm (c) 5 cm (d) None of these
same dimensions as that cone? 3
when one-ninth of the wall is
(a) 15 L (b) 30 L 458. If h, c, v are respectively the
filled with lime?
(c) 45 L (d) None of these height, the curved surface area
(a) 13500 bricks
451. The height of a metric cylinder and the volume of a cone then
(b) 13600 bricks
is 14cm & the different of its in the value of 3  vh3 - c2h2 + 9v2
(c) 20050 bricks
curved S.A. is 44 cm2. If the cyl- is equal to :

ir
(d) 18500 bricks
445. A rectangular water reservoir inder is made up of 99 cm 3 (a) 1 (b) 2

v.iSn
is 15 m by 12 m at the base. metal the f ind the inne r & (c) 0
Water flows into it through a outer radius of cylinder. (d) None of these
pipe whose cross-section is 5 (a) 464 (b) 564 459. If P is the height of a tetra
cm by 3 cm at the rate of 16 m/ (c) 660 (d) 366 hedron & each side is of 2cm

dnag
s second. Find the height to 452. If the base rad ius and the the find the value of 3p2.
which the water will rise in the height of a right circular cone (a) 6a² (b) 8a² (c) 5a² (d) 7a²
reservoir in 25 minutes: are increased by 40% then the 460. If 'h' be the height of a pyramind
(a) 0.2 m (b) 2 cm percentage increase in volume standing on a base which is an

eYrai
(c) 0.5 m
(approx) is: equilateral trinagle of side 'a'
(d) None of these
(a) 175% (b) 120% units, then the slant edge is:
446. The volume of a wall, 3 times
as high as it is broad and 8 (c) 64% (d) 540%
453. From a circular sheet of paper (a) h 2  a 2 / 4 (b) h 2  a 2 / 8
times as long as it is high, is
36.864 m 3. The height of the
wall is:
(a) 1.8 m (b) 2.4 m
snhe
of radius 25 cm, a sector area
4% is r emov ed. If the
rem aining p art is used to
(c) h 2  a 2 / 3 (d)
461. Find the v olum e of a
h 2  a2

(c) 4.2 m make a conical surface, then tetrahedren whose height is


kgei
(d) None of these the ratio of the radius and
height of the cone is: 4 3 cm.
447. If the areas of 3 adjacent sides
of a cuboid ar e x, y, z (a) 16 : 25 (b) 9 : 25 (a) 72 (b) 108 (c) 54 (d) 36
ERna

respectively, then the voulum (c) 7 : 12 (d) 24 : 7 462. In a shower 10 cm of rain fall
of the cuboid is: 454. A conical tent has 60° angle at the volume of water that falls
the ve rtex. The ratio of its on 1.5 hectares of ground is:
(a) xyz (b) xyz
radius and slant height is: (a) 1500 m3 (b) 1400 m3
aBryn

(c) 3xyz (c) 1200 m 3


(d) 1000 m3
(a) 3 : 2 (b) 1 : 2
(d) None of these 463. Find the total surface area of
(c) 1 : 3
448. A cylindrical cistern whose pyramid of heights 12 which is
(d) can't be determined
diameter is 21 cm is partly based on a rectangle of length
455. Water flows at the rate of 5 m
filled with water . If a 18 and & breadth 10cm.
rectangular block of iron 14 cm per min from a cylindrical pipe
Les

8 mm in radius. How long will (a) 117 (b) 564 (c) 120 (d) 456
in length, 10.5 cm in breadth
464. If from a circular sheet of paper
wa.th

and 11 cm in thickness is it take to fill up a conical vessel


wholly immersed in water, by whose radius is 12 cm and of radius 15 cm, a sector of 144°
how many centimetres will the depth 35 cm? is removed and the remaining
water level rise? (a) 315 s (b) 365 s is used to make a c onic al
(a) 14 cm (b) 20 cm (c) 5 min (d) None of these surface, then the angle at the
wwM

456. A reservoir is in the shape of a vertex will be:


14
(c) cm (d) 12 cm frustum of a r ight circular 3  6 
3 cone. It is 8 m across at the top (a) sin–1 
   (b) sin–1 
 
449. A right circular c ylindric al 10  5 
and 4 m across at the bottom.
tunnel of diameter 4 m and 3  4
le ngth 10 m is to be It is 6 m deep its capacity is:
(c) 2sin–1 
   (d) 2sin–1  
constructed from a sheet of (a) 224 m3 (b) 176 m3 5
  5 
iron. The area of the iron sheet (c) 225 m3 (d) None of these 465. Find the length of the string
required. 457. A conical vessel whose internal bound on a cylindrical tank
radius is 10 cm and height 72 whose base diameter and
280
(a) (b) 40p cm is full of water. If this water
 1
is poured into a cylindrical height are 5 cm and 48cm
(c) 80p (d) None of these vessel with internal radius 30 11

Rakesh Yadav Readers Publication Pvt. Ltd. 395

For More Visit : www.LearnEngineering.in


For More Visit : www.LearnEngineering.in

respectively. The string makes 473. A cuboidal block of cr ease in the total s ur face
exactly four complete turns 6cm  9cm  12cm is cut into area of the cuboid over the to-
round the cylinder, while its exact number of equal cubes. tal surface area of the cube ?
two ends touch the tank’s top The least possible number of 1 2
and bottom : cubes will be : (a) 234 % (b) 234 %
3 3
(a) 75cm (b) 70cm (a) 6 (b) 9 (c) 24 (d) 30 (c) 117% (d) None of these
(c) 60cm (d) 80cm 474. If the cone is cut along its axis 480. What is the total surface area
466. A cone, a hemisphere and a from the middle, the new shape of the identical cubes of larg-
cylinder stand on equal bases we obtain after opening the pa- est possible size that are cut
of radius R and have equal per is : from a cuboid of size 75cm 
heights H. Their whole (a) isosceles triangle 15cm  4.5cm ?

r
surfaces are in the ratio: (b) right angle triangle (a) 20, 250cm2 (b) 20, 520cm2

i
.iSn
(a) ( 3 + 1) : 3 : 4 (d) equilateral triangle (c) 22, 250cm2 (d) None of these
(d) None of these 481. If the volume of a sphere, a
(b) ( 2 + 1) : 7 : 8 475. What is the height of the cone cube, a tetrahedron and a oc-
which is formed by joining the tahedron be same then which

agv
(c) ( 2 + 1) : 3 : 4 of the following has maximum
two ends of a sector of circle with
(d) None of these radius r and angle 60° : surface area ?

ridn
467. If l, b, p be the length, breadth (a) sphere (b) cube
and perimeter of a rectangle 35 25 (c) tetrahedron(d) octahedron
(a) r (b) r
and b, l, p are in GP (in order), 6 6 482. A spherical ball of lead 6cm in

eeYa
then l/b is: r2 35 radius is melted and recast
(a) 2 : 1 (c) (d) r into three spherical balls. The
3 6
radii of two of these balls are
(b) ( 3 – 1) : 1 476. A sphere of 20cm radius is 3cm and 4cm. What is the ra-
dropped into a cylindrical ves-
(c) ( 3 + 1) : 1
geisnh dius of the third sphere ?
sel of 60cm diameter, which is (a) 6cm (b) 6.5cm
(d) can't be determined partly filled with water, then its
(c) 5.5cm (d) 5cm
468. The height of a cir cu lar cylinder level rises by x cm. Find x:
i s i n cr eased b y 6 t i m es an d 483. The base of a right prism is a
21 triangle whose perimeter is
base ar ea is decr eased by 1/ 9th
Enak

(a) 11 cm (b) 12cm 45cm and the radius of incircle


tim es. By what factor its later al 27
su r face ar ea is incr eased ? 23 is 9cm. If the volume of the
(a) 2 (b) 3 (c) 6 (d) 1.5 (c) 22.5cm (d) 11 cm prism is 810cm3. Find its height
R

27 :
469. A pyramid with an equal  477. A right circular cone resting on
based of each side 4 cm while (a) 5cm (b) 4cm
(c) 6cm (d) 4.5cm
aryn

its slant height is twice the 4


its base is cut at th its height
height of pyramid. Find its vol- 5 484. What is the semi-vertical angle
ume- of a cone whose lateral surface
Les B

along a parallel to the circular


8 area is double the base area ?
(a) 8 3 (b) base. The height of original
3 3 (a) 30° (b) 45°
cone is 75cm and base diam-
(c) 4 3 (d) 3 3 (c) 60° (d) None of these
eter is 42cm.What is the base
485. What is the number of cones of
radius of cut out (top portion)
wa.th

470. A cube and a sphere have equal


semi-vertex angle  and hav-
surfac e ar eas. The ratio of cone ?
ing r as the radius of the mid-
their volume is : (a) 4.2cm (b) 8.4cm
section which can be moduled
(a) p : 3 (b) : 6 (c) 2.8cm (d) 3.5cm out of a cylinder of base radius
478. A solid sphere is melted and r and height 2r cot  :
wwM

(c) 6 :  (d) 6 : 
recast into a right circular cone (a) 5 (b) 7 (c) 6 (d) 4
471. A cone, a hemisphere and a cyl- with a base radius equal to the
inder stand on equal bases and 486. A water tank is 30cm long,
radius of the sphere. What is 20cm wide and 12m deep. It is
have the same height. The ratio the ratio of the height and made of iron sheet which is 3m
of their respective volumes is : radius of cone so formed ? wide. The tank is open at the
(a) 1 : 2 : 3 (b) 2 : 1 : 3 (a) 5 : 2 (b) 4 : 3 top. If the cost of iron sheet is
(c) 1 : 3 : 2 (d) 3 : 1 : 2 (c) 4 : 1 (d) 3 : 2
472. A cube of sides 3cm is melted ` 10per meter. Find the total
479. 125 identical cubes are cut cost of iron required to build the
and smaller cubes of sides 1cm from a big cube and all the
each are formed. How many tank ?
smaller cubes are arranged
such cubes are possible? (a) ` 6000 (b) ` 5000
in a row to form a long cuboid.
(a) 21 (b) 23 (c) 25 (d) 27 What is the per centage in- (c) ` 5500 (d) ` 5800

Rakesh Yadav Readers Publication Pvt. Ltd. 396

For More Visit : www.LearnEngineering.in


For More Visit : www.LearnEngineering.in

487. A trapezium based prism with (a) 942cm3 (b) 298cm3 500. The radius and the height of a
two parallel sides 8cm and (c) 314cm3 (d) 302cm3 cone are in the ratio 4 :3 . The
14cm respectively and distance 494. A hemisphere bowl V 1 and a ratio of the curved surface area
between two parallel sides is hollow right circular cylinder V2 and total surface area of the
8cm. Find the height of the (having length equal to its ra- cone is :
prism if the volume of the dius) have the same diameter (a) 5 : 9 (b) 3 : 7
equal to the length of a side of (c) 5 : 4 (d) 16 : 9
prism is 1056cm3 ?
a hollow cubical box V3. Water 501. From a right circular cylinder
(a) 11cm (b) 10cm is filled in all these vessels
(c) 9cm (d) 12cm of radius 10 cm and height 21
upto the same level and such
that hemispherical bowl is full cm, a right circular cone of
488. From a circular sheet of paper,
of water and the volumes of same base- radius is removed.
radius 10cm, A sector of area
If the volume of the remaining

ir
40% of the sheet is removed. If filled water are v1, v2 and v3 re-
spectively in V1, V2 and V3then : portion is 4400 cm 3 then the
the remaining part is used to

v.iSn
(a) V1<V2<V3 (b) V2<V3<V1 height of the removed cone is:
makde a conical surface. Then
(c) V3<V2<V1 (d) V3<V1<V2
the ratio of radius and height  22 
495. A vertical cone of volume V with  take   
will be : vertex downward is filled with  7 

dnag
(a) 4 : 3 (b) 3 : 4 water up to half of its height. (a) 15 cm (b) 18 cm
(c) 2 : 3 (d) 2 : 1 The volume of the water is : (c) 21 cm (d) 24 cm
489. If the volume of circular cell (a) V/16 (b) V/8
502. A right circular cylinder and a
having inner and outer radius (c) V/4 (d) V/2
cone have equal base radius

eYrai
8cm and 12cm respectively is 496. The heights of a cone, cylinder
and equal heig hts . If the ir
equal to the total surface area and hemisphere are equal. If
their radii are in the ratio 2 : 3 curved surfaces are in the ra-
of cylinder of radius R1 and tio 8 : 5, then the radius of the
: 1, then the ratio of their vol-
height h, then h in terms of R1 base to the height are in the
umes is :
will be :

(a)
40  R1
R1
(b)
40  R1
R1 2
snhe
(a) 2 : 9 : 2 (b) 4 : 9 : 1
(c) 4 : 27 : 2 (d) 2 : 3 : 1
497. The height of a right circular
ratio :
(a) 2 : 3
(c) 3 : 4
(b) 4 : 3
(d) 3 : 2
cone and the radius of its cir- 503. The curved surface area of a
kgei
40  R12 cular base are respectively 9 cylindrical pillar is 264 sq.m.
(c) (d) None of these cm and 3 cm. The cone is cut and its volume is 924 cu.m.
R1
by a plane parallel to its base The ratio of its diameter to
490. An iron pipe 20cm long has ex-
ERna

so as to divide it into two parts. height is :


terior diameter 25cm. If the The volume of the frustum (i.e.,
thickness of the pipe is 1cm, (a) 3: 7 (b) 7 : 3
the lower part) of the cone is 44
then the whole surface area of (b) 6 : 7 (d) 7 : 6
cubic cm. The radius of the up-
the pipe ? per circular surface of the frus- 504. A cube of edge 6 cm is painted
aBryn

(a) 3168cm2 (b) 3186cm2 on all sides and then cut into
 22  unit cubes. The number of unit
(c) 3200cm2 (d) 3150cm2 tum  take    is :
491. The cap acity of two hem i-  7  cubes with no sides painted is:
spherical bowls are 64 litre and 3
(a) 0 (b) 64 (c) 186 (d) 108
(a) 3 12 cm (b) 13 cm 505. There is a pyramid on a base
216 litre respectively. Then
Les

the ratio of their internal (c) 3 13 cm (d) 3 20 cm which is a regular hexagon of


side 2a cm. If every slant edge
wa.th

curved surface area will be: 498. A solid cylinder has total sur-
of this pyramid is of length
(a) 2 : 3 (b) 1 : 3 face area of 462 sq. cm. Curved
(c) 16 : 81 (d) 4 : 9 5a
1 cm, then the volume of this
492. If the length of a rectangular surface area is rd of its total 2
3
parallel pipe is three times of
wwM

pyramid is :
its breadth and five times of its surface area. The volume of
the cylinder is : (a) 3a 3 cm3 (b) 3 2 a 3 cm 3
he ight. If its volume is
14400cm3, then the total sur- (a) 530 cm3 (b) 536 cm3 (c) 3 3a 3cm3 (d) 6a 3cm3
3
face area will be : (c) 539 cm (d) 545 cm3 506. The base of a right prism is an
(a) 4230cm2 (b) 4320cm2 499. A solid is hemispherical at the equilateral triangle of area 173
bottom and conical above. If the cm 2 and the volume of the
(c) 4203cm2
surface areas of the two parts
(d) None of these prism is 10380 cm3. The area
are equal, then the ratio of ra-
493. A right angled triangle with its dius and height of its conical of the lateral surface of the
sides 5cm, 12cm and 13cm is
revolved about the side 12cm.
part is :
(a) 1 : 3 (b) 1 : 1

prism is use 3  1.73 
2
Find the volume of the solid (a) 1200 cm (b) 2400 cm2
formed ? (c) 3 :1 (d) 1 : 3 (c) 3600 cm2 (d) 4380 cm2

Rakesh Yadav Readers Publication Pvt. Ltd. 397

For More Visit : www.LearnEngineering.in


For More Visit : www.LearnEngineering.in

507. Three spherical balls of radii 1 The volume of the cone will be
cm, 2 cm and 3 cm are melted increased by : d3  d d3  1 
(a)     (b)   
to form a single spherical ball. (a) 700 % (b) 500 % 3  2 3 2 3
In the process, the loss of ma- (c) 300 % (d) 100 %
terial is 25 %. The radius of the 512. The height of a right prism with d2
new ball is : a square base is 15cm. If the (c)
4
 
2   (d) None of these
(a) 6 cm (b) 5 cm area of the total surfaces of the
516. A big cube of side 9cm is formed
(c) 3 cm (d) 2 cm prism is 608 sq.cm, its volume
by re arranging together 27
508.The height of a cone is 40cm. is :
small but identical cubes each
The cone is cut parllel to its (a) 910 cm3 (b) 920 cm3
of side 3cm. further, if the cor-
base such that the volume of (c) 960 cm 3
(d) 980 cm
ner cubes in the top most layer
513. The internal radius and thick-

r
1 of the big cube are removed,
the small cone is of the ness of a hollow metallic pipe
what is the change in total sur-

i
64

.iSn
are 24 cm and 1 cm respec-
cone. Find at which height from face area of the big cube?
tively. It is melted and recast
base the cone is cut ? into a solid cylinder of equal (a) 18cm2 , decreases
(a) 20cm (b) 30cm length. The diameter of the (b) 54cm 2, decreases

agv
(c) 25cm (d) 22.5cm solid cylinder will be : (c) 36cm 2, decreases
509. A cube of side 8 metre is re- (a) 7 cm (b) 14 cm (d) remains the same

ridn
duc ed 3 tim es in the ratio (c) 960 cm3 (d) 980 cm 517. The base radius and height of
2 : 1. The area of one face of 514. The radius of the base of a right a cone is 5cm and 25cm re-
the reduced cube to that of the circular cone is doubled. To spectively. If the cone is cut par-

eeYa
original cube is in the ratio : kee p the volume fixed, the allel to its base at a height of h
(a) 1 : 4 (b) 1 : 8 height of the cone will be from the base. If the volume of
(c) 1 : 16 (d) 1 : 64 (a) One-fourth of the previous this frustum is 110cm 3. Find
510. The volume of the largest cyl- height the radius of smaller cone ?®
inder formed, when a rectan-
geisnh 1 (a) 1041 / 3 cm (d) 1041 / 2 cm
gular sheet of paper of size 22 (b) times of the prevoious height
cm  15cm is rolled along its 2 (c) 5cm (d) None of these
(c) half of the previous height 518. A spherical steel ball was sil-
 22  ver polished then it was cut into
larger side, is  use   :
Enak

 7  (d) one - third of the previous height 4 similar pieces. What is the
(a) 288.75 cm3 (b) 577.50 cm3 515. If a cube maximum possible ratio of the polished area to the
volume is cut off from a solid non-polished area :
(c) 866.25 cm3 (d) 1155.00 cm3
sphere of diameter d, then the
R

511. Each of the height and radius (a) 1 : 1 (b) 1 : 2


volume of the rem aining (c) 2 : 1
of the base of a right circular (waste) material of the sphere
cone is increased by 100 %. (d) None of these
aryn

would be equal to :
Les B
wa.th
wwM

Rakesh Yadav Readers Publication Pvt. Ltd. 398

For More Visit : www.LearnEngineering.in


For More Visit : www.LearnEngineering.in

ANSWER KEY

1. (a) 53 . (d ) 1 05 . (a) 1 57 . (a) 2 09 . (c ) 2 61 .(d ) 3 13 . (a) 3 65 .(b ) 4 17 . (a) 4 69 .(b )


2. (c ) 54 . (b ) 1 06 . (a) 1 58 .(d ) 2 10 .(d ) 2 62 .(b ) 3 14 . (c ) 3 66 .(b ) 4 18 . (a) 4 70 .(b )
3. (c ) 55 . (c ) 1 07 . (a) 1 59 . (c ) 2 11 .(d ) 2 63 .(d ) 3 15 . (c ) 3 67 . (a) 4 19 . (c ) 4 71 . (a)
4. (b ) 56 . (c ) 1 08 . (a) 1 60 . (a) 2 12 .(d ) 2 64 . (c ) 3 16 .(b ) 3 68 .(d ) 4 20 .(d ) 4 72 .(d )
5. (b ) 57 . (c ) 1 09 .(b ) 1 61 .(d ) 2 13 .(b ) 2 65 . (a) 3 17 .(d ) 3 69 .(d ) 4 21 . (c ) 4 73 . (c )
6. (d ) 58 . (d ) 1 10 .(b ) 1 62 . (c ) 2 14 . (a) 2 66 . (a) 3 18 .(b ) 3 70 . (c ) 4 22 .(b ) 4 74 .(b )
7. (a) 59 . (b ) 1 11 .(d ) 1 63 . (a) 2 15 .(d ) 2 67 . (a) 3 19 .(b ) 3 71 . (a) 4 23 .(b ) 4 75 . (a)
8. (c ) 60 . (d ) 1 12 . (a) 1 64 .(d ) 2 16 . (a) 2 68 . (c ) 3 20 . (a) 3 72 .(d ) 4 24 . (a) 4 76 .(d )
9. (c ) 61 . (b ) 1 13 .(b ) 1 65 . (a) 2 17 .(b ) 2 69 .(d ) 3 21 . (a) 3 73 .(d ) 4 25 .(b ) 4 77 . (a)
10 . (a) 62 . (a) 1 14 .(b ) 1 66 . (c ) 2 18 .(d ) 2 70 .(b ) 3 22 .(d ) 3 74 . (c ) 4 26 . (a) 4 78 . (c )
11 . (d ) 63 . (a) 1 15 .(d ) 1 67 .(d ) 2 19 . (a) 2 71 . (c ) 3 23 . (a) 3 75 .(b ) 4 27 . (a) 4 79 .(b )

ir
12 . (b ) 64 . (b ) 1 16 . (c ) 1 68 .(b ) 2 20 . (a) 2 72 . (a) 3 24 . (c ) 3 76 . (c ) 4 28 . (c ) 4 80 . (a)
13 . (a) 65 . (b ) 1 17 . (c ) 1 69 .(d ) 2 21 . (a) 2 73 . (a) 3 25 .(b ) 3 77 . (c ) 4 29 .(b ) 4 81 . (c )

v.iSn
14 . (d ) 66 . (a) 1 18 . (a) 1 70 . (a) 2 22 .(d ) 2 74 . (c ) 3 26 .(b ) 3 78 .(d ) 4 30 . (a) 4 82 .(d )
15 . (c ) 67 . (c ) 1 19 .(b ) 1 71 . (c ) 2 23 . (a) 2 75 . (c ) 3 27 . (c ) 3 79 . (a) 4 31 .(b ) 4 83 .(b )
16 . (b ) 68 . (b ) 1 20 .(b ) 1 72 .(b ) 2 24 . (c ) 2 76 . (c ) 3 28 .(b ) 3 80 .(b ) 4 32 . (c ) 4 84 . (a)
17 . (a) 69 . (b ) 1 21 . (c ) 1 73 .(b ) 2 25 . (a) 2 77 . (a) 3 29 .(b ) 3 81 . (c ) 4 33 . (a) 4 85 . (c )
18 . (c ) 70 . (b ) 1 22 . (a) 1 74 .(b ) 2 26 . (a) 2 78 .(d ) 3 30 .(d ) 3 82 . (c ) 4 34 . (c ) 4 86 . (a)

dnag
19 . (d ) 71 . (b ) 1 23 .(b ) 1 75 .(d ) 2 27 .(d ) 2 79 . (c ) 3 31 .(b ) 3 83 .(b ) 4 35 .(b ) 4 87 .(d )
20 . (c ) 72 . (b ) 1 24 . (c ) 1 76 .(d ) 2 28 . (a) 2 80 . (c ) 3 32 . (c ) 3 84 . (a) 4 36 . (c ) 4 88 .(b )
21 . (b ) 73 . (c ) 1 25 . (c ) 1 77 . (c ) 2 29 .(d ) 2 81 .(b ) 3 33 . (a) 3 85 . (c ) 4 37 . (a) 4 89 . (c )
22 . (b ) 74 . (c ) 1 26 . (c ) 1 78 .(d ) 2 30 . (c ) 2 82 . (c ) 3 34 . (c ) 3 86 .(d ) 4 38 .(d ) 4 90 . (a)
23 . (b ) 75 . (b ) 1 27 . (c ) 1 79 . (a) 2 31 . (c ) 2 83 . (c ) 3 35 . (c ) 3 87 . (a) 4 39 .(b ) 4 91 .(d )

eYrai
24 . (d ) 76 . (b ) 1 28 . (c ) 1 80 . (a) 2 32 .(b ) 2 84 .(b ) 3 36 .(b ) 3 88 . (c ) 4 40 . (c ) 4 92 .(b )
25 . (c ) 77 . (c ) 1 29 . (a) 1 81 .(b ) 2 33 . (a) 2 85 .(b ) 3 37 .(b ) 3 89 .(d ) 4 41 .(d ) 4 93 . (c )
26 . (d ) 78 . (d ) 1 30 . (c ) 1 82 . (a) 2 34 .(b ) 2 86 . (a) 3 38 . (c ) 3 90 .(b ) 4 42 .(b ) 4 94 . (a)
27 . (c ) 79 . (b ) 1 31 . (a) 1 83 . (a) 2 35 . (c ) 2 87 .(d ) 3 39 . (a) 3 91 . (a) 4 43 . (c ) 4 95 .(b )
28 .
29 .
30 .
31 .
32 .
(b )
(d )
(b )
(a)
(b )
80 . (c )
81 . (c )
82 . (b )
83 . (b )
84 . (a)
1 32 . (c )
1 33 . (a)
1 34 . (a)
1 35 .(d )
1 36 . (c )
1 84 .(d )
1 85 .(d )
1 86 . (a)
1 87 .(b )
1 88 .(d )
snhe
2 36 . (a)
2 37 .(b )
2 38 . (a)
2 39 .(d )
2 40 . (a)
2 88 .(d )
2 89 . (a)
2 90 .(d )
2 91 . (a)
2 92 .(d )
3 40 . (a)
3 41 . (c )
3 42 .(d )
3 43 .(b )
3 44 . (a)
3 92 .(b )
3 93 . (c )
3 94 .(d )
3 95 .(b )
3 96 .(d )
4 44 .(b )
4 45 . (a)
4 46 .(b )
4 47 .(b )
4 48 . (c )
4 96 . (c )
4 97 .(b )
4 98 . (c )
4 99 .(d )
5 00 . (a)
kgei
33 . (a) 85 . (b ) 1 37 .(d ) 1 89 . (a) 2 41 .(b ) 2 93 .(d ) 3 45 .(b ) 3 97 .(d ) 4 49 .(b ) 5 01 . (c )
34 . (b ) 86 . (d ) 1 38 .(d ) 1 90 . (c ) 2 42 . (c ) 2 94 .(b ) 3 46 . (c ) 3 98 .(d ) 4 50 . (c ) 5 02 . (c )
35 . (a) 87 . (b ) 1 39 .(d ) 1 91 .(b ) 2 43 .(d ) 2 95 . (a) 3 47 . (c ) 3 99 . (c ) 4 51 .(b ) 5 03 .(b )
36 . (b ) 88 . (d ) 1 40 .(b ) 1 92 . (a) 2 44 .(b ) 2 96 .(d ) 3 48 .(b ) 4 00 .(d ) 4 52 . (a) 5 04 .(b )
ERna

37 . (c ) 89 . (b ) 1 41 . (c ) 1 93 .(b ) 2 45 .(d ) 2 97 . (a) 3 49 .(d ) 4 01 .(b ) 4 53 .(d ) 5 05 . (c )


38 . (c ) 90 . (c ) 1 42 .(d ) 1 94 .(d ) 2 46 .(b ) 2 98 . (a) 3 50 .(d ) 4 02 . (c ) 4 54 .(b ) 5 06 . (c )
39 . (d ) 91 . (c ) 1 43 .(b ) 1 95 .(b ) 2 47 . (a) 2 99 . (c ) 3 51 . (c ) 4 03 . (a) 4 55 . (a) 5 07 . (c )
40 . (b ) 92 . (d ) 1 44 .(c ) 1 96 . (a) 2 48 . (a) 3 00 . (a) 3 52 . (c ) 4 04 . (a) 4 56 .(d ) 5 08 .(b )
41 . (c ) 93 . (d ) 1 45 . (a) 1 97 .(d ) 2 49 .(d ) 3 01 . (c ) 3 53 . (c ) 4 05 .(b ) 4 57 . (a) 5 09 .(d )
aBryn

42 . (d ) 94 . (a) 1 46 .(b ) 1 98 .(d ) 2 50 .(d ) 3 02 . (a) 3 54 .(b ) 4 06 . (c ) 4 58 . (c ) 5 10 .(b )


43 . (a) 95 . (d ) 1 47 .(d ) 1 99 .(d ) 2 51 . (c ) 3 03 .(b ) 3 55 .(d ) 4 07 .(b ) 4 59 .(b ) 5 11 . (a)
44 . (d ) 96 . (b ) 1 48 .(d ) 2 00 . (a) 2 52 .(b ) 3 04 .(b ) 3 56 . (a) 4 08 . (c ) 4 60 . (c ) 5 12 . (c )
45 . (d ) 97 . (b ) 1 49 .(b ) 2 01 . (a) 2 53 . (a) 3 05 . (a) 3 57 .(b ) 4 09 . (a) 4 61 . (a) 5 13 .(b )
46 . (d ) 98 . (b ) 1 50 .(b ) 2 02 . (a) 2 54 . (c ) 3 06 .(d ) 3 58 .(d ) 4 10 . (a) 4 62 . (a) 5 14 . (a)
47 . (d ) 99 . (b ) 1 51 . (a) 2 03 . (a) 2 55 . (a) 3 07 .(d ) 3 59 .(b ) 4 11 . (c ) 4 63 .(b ) 5 15 .(b )
Les

48 . (d ) 1 00 .(b ) 1 52 .(d ) 2 04 .(b ) 2 56 .(d ) 3 08 . (c ) 3 60 .(d ) 4 12 .(d ) 4 64 . (c ) 5 16 .(d )


49 . (a) 1 01 . (a) 1 53 .(b ) 2 05 .(d ) 2 57 .(d ) 3 09 . (c ) 3 61 .(b ) 4 13 . (c ) 4 65 .(d ) 5 17 . (a)
wa.th

50 . (b ) 1 02 . (c ) 1 54 . (a) 2 06 .(b ) 2 58 .(d ) 3 10 . (a) 3 62 .(d ) 4 14 .(b ) 4 66 . (c ) 5 18 . (a)


51 . (d ) 1 03 . (c ) 1 55 .(d ) 2 07 . (c ) 2 59 .(d ) 3 11 .(b ) 3 63 . (c ) 4 15 . (c ) 4 67 . (c )
52 . (b ) 1 04 . (c ) 1 56 . (a) 2 08 .(b ) 2 60 .(b ) 3 12 . (c ) 3 64 .(d ) 4 16 .(b ) 4 68 . (a)
wwM

Rakesh Yadav Readers Publication Pvt. Ltd. 399

For More Visit : www.LearnEngineering.in


For More Visit : www.LearnEngineering.in

SOLUTION
1. (a) Let the side of cube = a cm 7 h = height = 3 m
a 3 Radius (r)= cm  Area of walls  2
2
(4 + 3) × 3 = 42 m²
4 Area of painted part
volume of sphere = r³
3 = 42 m2 + 12m2 = 54 m2
17. (a) Let
Diagonal of cube = a 3 cm 4 22 7 7 7 length = l,
= × × × ×
a 3 = 12 3 7 2 2 2 breadth = b
on squaring , a²(3) = 12 = 179.67 cm³ height = h

r
a² = 4 12. (b) Length of rod given that (l + b + h) = 12 cm

i
= total surface area of box

.iSn
a = 2 cm
= 102  102  52 = 225 = 2(lb + bh + hl) =94 m² (given)
volume of cube = a³ = 2³ = 8 cm
= 15cm (l + b + h)2 = l² + b² + h² + 2(lb + bh + hl)
15 ³ (12)2 = l² + b² + h² + 94
2. (c) Number of cubes = = 125

agv
3 ³ 40 cm 144 – 94  l² +b² + h²
13. (a)
32 cm 50 = l² + b² + h²
Diagonal

ridn
3. (c) Side of the cube = diagonal of box = l²b²h²
3
length of longest rod that can
7 cm

7 cm
15 cm

be put inside the box

eeYa
4 3
= = 4cm
3 = l²  b²  h² = 5 2 cm
Volume of the cube = (side)³ 32 cm 18. (c) Let breadth = b m
 (4)³ = 4×4×4 = 64 cm³ Volume of the box = l × b × h  length of room = 2b m
4. (b) Let l = 9x, h = 3x, b = x
geisnh (l = 2b)
= (40 – 8) × (15 – 8)× 4
l × b × h = 216 × 1000 height = 11 m
= 32 × 7 × 4 = 896 cm³
(1 litre = 1000 cm³) Area of four walls of room
14. (d) Let the three sides of the cuboid
9x × x × 3x = 216000 = 660 m² (given)
be l, b and h
27x³ = 216000 2(l + b) × h = 660
Enak

 lb = bh = hl = 12 2(2b +b) × 11 = 660


x³ = 8000
x = 20
 l² b²h² = 12 × 12 × 12 3b × 22 = 660
= 1728 b = 10
l = 180 cm = 18 dm
R

 Breadth = 10 m
Volume = 2 × volume  lbh = 1728 = 12 12 Length = 20 m
5. (b) of Cuboid Cube area of floor = l × b
= 24 3 cm³
aryn

l × b × h = 2 × (side)³ length × breadth =


15. (c) dimensions of room
986 20 × 10 = 200 m²
Les B

=(side)³ = 216 length(l) = 12 cm


2 breadth(b) = 9 cm 8 3
19. (d) Side of cube (a) = = 8 cm
side = 3 216 = 6 cm height(h) = 8 cm 3
Total surface area of cube  diagonal of cube = l 2 + b 2 + h2  Total surface area
wa.th

9  length of longest rod = 6(a)² = 6×8² = 384 cm²


. (c) Volume of the cistern  length of diagonal of cuboid 20. (c) Length of pencil
 (330 – 10) × (260 – 10)× (110 – x) == a 2 + b2 + c 2
= 8000 × 1000
 l22 + 92 + 82
wwM

(where x = thickness of bottom) = 8² + 6² + 2² = 64364


 1448164
x = 110 – 100 = 10 cm = 1 dm = 104 = 2 26 cm
10. (a) Le t the le ng th, b r e ad th  289 = 17 cm
21. (b) Edge of box
and height be l, b, h respectively 16. (b) area of floor  3 × 4 = 12 m²
lb = x = 3 3.375 = 1.5 m
height  3 m
bh = y  area of walls of room 22. (b) Whole surface area of cuboid
lh = z = 2(whole surface area of cube)
 (Perimeter × height of
 l ² b² h² = xyz – 2×area of one face
of floor) ( two faces of the two cubes
(l bh)² = xyz
 2(l + b) × h are not visible now)
 v² = xyz
11. (d) The diameter of sphere = side  l = length = 4 m  Required area = 12a² – 2a²
of cube = 7 cm b = breadth = 3 m = 10a² = 10×6² = 360 cm²

Rakesh Yadav Readers Publication Pvt. Ltd. 400

For More Visit : www.LearnEngineering.in


For More Visit : www.LearnEngineering.in

23. (b) Let the increase in level = x m 28.(b) = 405 cm ³


Now, 62 cm³ is lost
 2  1
 1000  1000    Volume of new cube = 405 – 62
100  2 = 343
= 100 × 10 × x (side of new cube)³ = 343
 x = 10m Diagonal = 35 3 side of new cube = 3
343
24. (d) Sides of parallelopiped are in =7
 The length of largest rod
ratio = 2 : 4 : 8 Total surface are a of new c ube
Let length = 2 units = Diagonal = side 3 = 6 (sid e)²
breadth = 4 units Side 3 = 35 3 = 6 ×(7)² = 6 ×49 = 294 cm²
Height = 8 units 32. (b) Area of cubical floor = 48

ir
Let the side of cube = a unit 35 3 Side² = 48
side = = 35

v.iSn
According to question, 3
side of cube = 35 side = 48 = 4 3
volume of c ube = volume of
parallelopiped Diagonal of cube = side 3
a³ = 2 × 4 × 8 = 4 3 × 3 = 12 m

dnag
a³ = 64 Length of longest rod = 12 m
a = 3 64 = 4 units 33. (a) Let side of new cube = a
Diameter of the sphere According to question,
Suface area of parallelopiped a³ = 6³ + 8³ + 1³

eYrai
= side of the cube
Surface area of cube = 2 × radius = side = 216 + 512 + 1
2(lb bh hl )
= 729
= 35
6a ² radius =
2
cm a= 3 729 = 9

=
28  32 16 
6 4 
2 =
7
6
=7:6 = 4r ²
snhe
Surface area of the sphere then surface area = 6(a)² = 6 × 9²
= 6 × 81 = 486 cm²
34. (b) Volume = 20 m³ = 20 × (100)³
cm³
kgei
25. (c) Let 22 35 35
= 4   Volume of one brick = (25 × 12.5 × 8) cm³
length = 1 cm 7 2 2
breadth = 2 cm,  Required number of bricks
= 3850 m²
ERna

height = h cm 29. (d) volume of air in room = 204 m³ 20 100 100 100
2(lb + bh + hl) = 22 (area of floor) × height = 204 = 25 12.5 8 = 8000
2(2 + 2h + h) = 22  volume = area of base × height
2 + 3h = 11 35. (a) Th e t ota l s ur f ac e ar e a of
(area of floor) × 6 = 204 cube
aBryn

3h = 9
204 6 (side)² = 150cm²
h = 3 cm area of floor = = 34 m²
6 6 (side)² = 150 cm²
Diognal = l ²b ²h ² 30. (b) 150
(side)² = = 25
1²  2²  3²  1  4  9  14 6
Les

26. (d) l ²b ²h ² = 15 side = 25 = 5 cm


wa.th

l² + b² + h² = 225...............(i)  volume of cube = (side)³


 l + b + h = 24 = (5)³ = 125 cm³
(l + b + h)² = 576 The box will be of cuboid shape 36. (b) Let Ratio of length : breadth
 l² + b² +h² + 2(lb +bh + hl) = 576 Length of the box, l = 24 – (2 ×3) = = 5x : 3x
wwM

24 – 6 = 18 cm Total surface area of parallelopiped


= 225 + 2(lb + bh + hl)= 576
breadth of the box, b = 558 cm²
2(lb + bh + hl) = 351 cm ²
= 18 – (2 × 3) = 18 – 6 = 12 cm 2 (lb + bh + hl) = 558
27. (c) Total surface area of cube
Height of the box ,h = 3 cm 2(5x × 3x + 3x × 6 + 6 × 5x) = 558
= 6(side)²
surface area of the box 2 (15x ² + 18x + 30x) = 558
6(side)² = 96
= 2(l + b)× h + l × b 15x² + 48x = 279
96 = 2(18 + 12)× 3 + 18 ×12 15x² + 48x – 279 = 0
(side)² = = 16 = 2 ×30 ×3 + 18 × 12
6 On solving, x= 3
= 180 + 216 = 396 cm²
side =
16 = 4 cm 15
31. (a) volume of all three cube  length = 5 × 3 = 15 cm =
Volume of the cube = (side)³ = 4³ + 5³ + 6³ 10
= (4)³ = 64 cm³ = 64 + 125 + 216 cm³ = 1.5 dm

Rakesh Yadav Readers Publication Pvt. Ltd. 401

For More Visit : www.LearnEngineering.in


For More Visit : www.LearnEngineering.in

37. (c) l + b + h = 24 cm  Volume Volume Let initial height = H


=
Length of diagonal = 15 cm Hamisphere of cone Final height = h
According to question ,
l² + b² + h² = 15 2 1
π R³ = π R2 h 2 2
l² + b² + h² = 225 cm 3 3  35   35 
(l + b + h)² – 2(lb + bh + hl) = 225 π   H – π  h = 11000 cm³
 2R= h 2  2
(24)² – 2(lb + bh + hl) = 225 44. (d) According to question
576 – 225 = 2(lb + bh + hl) Let the radius of sphere = r cm 2
351 = 2(lb + bh + hl)  35 
4 π (r + 2)² – 4 π r² = 352 π    H – h   11000
 Total surface area = 351 cm² 2
38. (c) Let length = 3x, 4π r + 2 2

– r 2 = 352
11000  2  2  7

r
breadth = 4x
4 π r² + 4 + 4r – r² = 352 H–h= 35  35  22
height = 6x

i
.iSn
3x × 4x × 6x = 576
352 80 3
576 π (1 + r) = = 22 = = 11 cm
16 7 7
x³ = 3 4 6 = 8

agv
22 48. (d)
(1 + r) = 22
x = 3 8 = 2 cm 7

ridn
 length = 3 × 2 = 6cm 1+r=7 h = 40 cm
breadth = 4 × 2 = 8 cm, r = 6 cm
height= 6 × 2 = 12 cm 45(d) .

eeYa
r
Total surface area= 2 (lb + bh + hl) H= 60 cm

= 2 (6 × 8 + 8 × 12 + 12 × 6)  circumference of its base = 66 cm


= 2 (48 + 96 + 72) 2π r = 66
= 2 × 216 = 432 cm² R = 32 cm
66 7
geisnh
We have to find the slant height 66 21
39. (d) As we know that r = = 2 22 = cm
Take ratio of H and R 2π 2
A parallelopiped has vertices (v) = 8
edge (e) = 12 = 60 : 32  volume = π r² h
face (f) = 6 15 : 8
22 21 21
Enak

Put into equation (v – e + f) L= 15²8² = 17 = ×  40


7 2 2
 8 – 12 + 6  2 = 17 × 4 = 68 cm = 13860 cm³
40. (b) According to the question. Cost of painting = Surface area of 49. (a) According to question,
R

1 cone × 35
1 dm = m = π R L × 35
10
aryn

Let depth of the hole = d 22 32  68 h


=   35
6.5 7 10000
Les B

48 m × 31.5 × m

10 = Rs. 23.94 (approx) . r
= 27 × 18.2 × d 46. (d)
2π r = 6 π
d = 2m
r = 3 cm
41. (c)
wa.th

H= 10cm
. height of cylinder = diameter
2.1 m × 1.5 m × h = 630 lt r = 2 cm
= 2×r = 2 × 3 = 6cm
21 15 630 Spherical balls volume of water
m × m ×h = m³ R = 20cm
10 10 1000 = π r²h= π (3)² × 6
Solid Cone
= 54 π cm³
wwM

 1 m³ = 1000lt  Let the spherical balls made = ‘x’ 50. (b) Volume of the cone
1000 cm³ = 1lt 
  According to question, 1
Volume of cone = x × volume of = π (15)² × 108 cm³
1 3
h = m = 0.20 metre sphere
5 Volume of the cylinder
1 4 = π × r² × 9 cm³
8 4 2 π R² H = x × π r³
42. (d) Number of cubes = 2 2 2
3 3 According to question,
(20)² ×10 = x × 4 × (2)³ 1
=8 x = 125 π × r² × 9 = π × 15 × 15 × 108
3
43. (a)when we change shape of a
35 5  15  108
solid figure, volume remains 47. (d) Radius of tank , r = cm r² = = 900
constant , 2 9

Rakesh Yadav Readers Publication Pvt. Ltd. 402

For More Visit : www.LearnEngineering.in


For More Visit : www.LearnEngineering.in

r = r² =7 ×7
900 = 30 diameter
55. (c) radius = r = 7  7 = 7 cm
Diameter of base = 2r 2
= 2 × 30 = 60 cm l = r ²h ²  7²24²  625
19.2
51. (d) Volume of new solid sphere = = 9.6 m
2 = 25
3 3 3
4 6 4 8 4  10  curved surface area = rl
         height = 2.8
3 2 3 2 3  2 l² = r² + h² = 9.6² + 2.8² 22
= 92.16 + 7.84 = 725 = 550 cm²
4 4  3 3 3 7
π r³ = π 3 4 5  
 = 100
3 3  60. (d) Volume of a sphere
r³ = 216, r = 6 cm l =
100 = 10 m 88

ir
 Diameter of the new sphere area of the canvas = π r l = × (14)³
21
= 2 × 6 = 12 cm

v.iSn
22
52. (b) Total surface area of =  9.6  10 = 301.7 4 22 4
 

7 =  × (14)³   r ³ 
prism (regular hexagon) 56. (c) External radius R = 4 cm 3 7 
3 
= Surface area + (base + top) area Internal radius r = 3cm Radius = 14

dnag
volume of iron used Curved surface area of sphere
156 3 = Perimeter of base ×
= π R² h – π r² h = 4(radius)²
height + 2 × Base area
= π h (R² – r²) 22
156 3 = 6 × a ×10 + 2 × 6× =4× × 14× 14= 2464 cm²

eYrai
= π h (R + r) (R – r) 7
3 22
×a2 =  20   4  3    4 – 3  61.(b)
4 7
r
156 3 = 60 a + 3 3a 2

3a 2 + 20 a = 52 3
=
22
7 snhe
 20  7  1 = 440 cu. cm

Volume surface area of sphere


a  
3a  20 = 52 3 57. (c) Volume =
kgei
of sphere of displaced = 64 
water
a= 2 3 4 (Radius)² = 64
Volume of prism = Base area × 4
π 2  2  2 = π  4 4  h 64
ERna

Height 3 (radius)² = = 16
4
3 2 2
=6×
4

× 2 3  ×10 h=
3
cm radius =
16 = 4 cm
diameter = 8 cm
aBryn

= 18 3 ×10 = 180 3 cm3 58.(d) 62. (a) 1 dm = 10 cm


53. (d) Volume of the new sphere
4 4 h
 π [r13 + r23 + r33] = π R³ =
3 3 r =3cm
Les

R³ = r13 + r23 + r33


wa.th

R³ = 1³ + 6³ + 8³ R =6cm
= 1 + 216 + 512 = 729
R = Volume of cone = volume of sphere
729 = 9 cm
54. (b) l = 2.5 km 1 4
R ²h = r ³ x × volume of 1 tin = volume of cylin-
area of base = 1.54 km²
wwM

3 3 der
π r² = 1.54
1 4  x × (25 × 22 × 35)
 6  6  h =  3  3  3
1.547 3 3
r² = 22 350 350
22 h = 3 cm =   240
7 2 2
1.547
1 x = 1200
59. (b) Volume of a cone = r ²h
r = = 0.7 km 3
22 63. (a)
We know that , l ² = r² + h² 1
r ² 24 = 1232 cm ²
h² = 3 h=21cm
l² – r ² = 2.5² – 0.7²
123237
= 5.76 = 2.4 km r² =
2422

Rakesh Yadav Readers Publication Pvt. Ltd. 403

For More Visit : www.LearnEngineering.in


For More Visit : www.LearnEngineering.in

Volume of hollow iron pipe 67.(c)  Cylinder is folded along the


length of rectangle
= R²–r²×h
 2R = 22
= { 4² – 3²}21
22 22  7 7
R= = = cm
22 2 2  22 2
= × 7 ×21 = 22 × 21 volume of well = volume of embank-
7 Volume of the cylinder = R² H
ment
= 462 cm³ (10)² × 14 = {15² – 10²} ×H 22 7 7
Now , =   × 12
10014 7 2 2
1 cm³ = 8g H= = 11 .2 m
462 cm³ = 8 × 462 g 125 = 22 × 7 × 3 = 462 cm³

r
= 3696 g = 3.696kg 68. (b) 71. (b)
R=1

i
.iSn
64. (b) R
2
r

h = 1800 cm
r= 7 cm = H =7

agv
h=14 cm 3 = H =8 cm
11cm

ridn
Volume of sphere = volume of cylinder (wire) (Rod)
Volume of the cylinder = volume of cube
4 7 Volume of wire = volume of Rod
r ²h = (side)³ 7 ³R ²

eeYa
3 3  r²h = R²h
22 R² = 4 × 7 × 7 = 2 × 2 × 7 × 7
× r²× 14 = 11 × 11× 11 1
7 r2 × 1800 = ×8=2
R=2277 = 2 × 7 = 14 cm 4
111111
r² =
geisnh
diameter of base of cylinder = 2R 2 1
222 = 2 × 14 = 28 cm r² = =
1800 900
121 69. (b)
= r 1 1
4 R r= 
900 30
Enak

11 72. (b)
r= cm = 5.5 cm
2
ATQ, 8 cm
R

65. (b) Let the radius = r R+r = 10


 r²h = 9h (R + r)² = 100 2 cm = 12× r
r² = 9
aryn

R² + r² + 2Rr = 100
r = R² + r² = 100 – 2Rr ...(i)
9 =3m
Volume of cylinder = 12 × volume of sphere
Les B

diameter = 3×2 = 6 m 4 4
R ³ r ³ = 880 4
66. (a) 3 3 (8)² × 2 = 12 × r ³
3
4
R ³r ³ = 880
8 823
wa.th

3
l r³ =
h 12  4
8 cm 880  3 880  3  7
R³ + r³ = = r=
 4 22  4 2  2  2 = 2 cm
8cm (R + r)(R² + r² – Rr) = 210 r = 2 cm
wwM

volume of cone = volume of sphere 10 × (100 –2Rr – Rr) = 210 d = 4cm


100 – 3Rr = 21 73. (c) 2R – 2r = 5
1 4 3Rr = 100 – 21 = 79
(8)² × h =  (8)³ 5
3 3 (R – r) =
79 1 2
8 × 8 × h = 4 × 8 × 8× 8 Rr = = 26
3 3 74. (c)
h = 32 cm
70. (b)
2pR=22 cm r
slant height = (l) = r ² h ² 22 cm

 8²  32² 12 cm 12 cm
4
= 64  1024  8 17 r³ = 4r²
3

Rakesh Yadav Readers Publication Pvt. Ltd. 404

For More Visit : www.LearnEngineering.in


For More Visit : www.LearnEngineering.in

radius (r) = 3 units 7707  2r = 12


75. (b)  r² =
22 12 6
 r = 2 = cm
12 =h  r = 7 5 cm 
83. (b)Volume of tunnel = × r² × H
H 24 cm and rl = 814
12 22 4 4
8147 37 = × × × 56 = 704 m³
 l= = 7 2 2
227 5 5
R = 7 cm Volume of ditch = 48 × 16.5 ×4
l² = h² + r²
= 3168 m³
Volume of bigger cone 3737
 = h² + 245 704 2
1 5 Required part = =

ir
= ×(7)²×24 3168 9
3

v.iSn
1369 144 84. (a) According to the question
 h² = – 245 =
1 22 5 5 h(R² – r²) = 748
=  × 7× 7 × 24 7487
3 7 12
 h= R² – r² =
= 22 ×7 × 8 = 1232 cm³ 5 2214

dnag
9² – r² = 17
volume of smaller cone h3 1  9² – r² = 17
= volume = r²h
volume of bigger cone 3 3
H  r² = 81 – 17 = 64

eYrai
1 22 12  r=8
Volume of smaller cone 123 = × × 7 5× 7 5×
5  Thickness = 9 – 8 = 1 cm
= 3 7
1232 243
= 616 5 cm³ 4 3
volume of smaller cone= 154 cm³ 85. (b) 2      r  = R² h
 When the cone is cut in be-
tween then the ratio of volume
of smaller cone to the bigger
snhe
79. (b) In this case the breadth be-
comes the circumference of the
base of the cylinder
 2r = 44
2×
3

4
3
× × 27 =  × 36 × h
kgei
one is always equal to the ratio
of the cubes of their heights 447 2742
 r= = 7 cm h=
76. (b) 222 363
ERna

New volume = r²h 827


h= = 2 cm
22 336
h = × 7 × 7 × 100 86. (d)
= 6 cm 7
= 15400 cm³ Ratio of height = 3 Ratio of volume
aBryn

1
3cm
80. (c) r²H = r²h 3
1
3 =
Volume of sphere 27
n=
volume of cone 1
H= h h 1
Les

4 3  =
6 ³ H 3
 h = 3H = 3 × 6 = 18 cm
wa.th

= 3 = 24 3 units  30
1 81. (c) 3r² = 1848
3 ²4 2 units  20
3
18487  The cut is made 20 cm above the
77. (c) Height of = Breadth r² = = 196
322 base
cylinder of tin foil
wwM

 r = 14 cm 87. (b) 3r² = 108


 Circumference of the base of cyl-
inder According to the question  r² = 36
= Length of the foil 2 1  r = 6 cm
= 22 cm r³ = r²h
3 3 2 2
 2r = 22 Volume = r³ = × 216 × 
 2r = h 3 3
227 7
r= = cm  h = 2 × 14 = 28 cm = 144
222 2 88. (d) Radius = 3 Decimeters
82. (b) The length of the paper be-
22 7 7
comes the circumference of the = 30 cm
Volume =r²h = × × × 16
7 2 2 base of cylinder when it is rolled Height of circular sheet = 1 mm
= 616cm³ = 0.1 cm
78. (d) r² = 770 along its length

Rakesh Yadav Readers Publication Pvt. Ltd. 405

For More Visit : www.LearnEngineering.in


For More Visit : www.LearnEngineering.in

4 1 h 4 = 124 = 4 cm
 × (30)³ = r² × =
3 10 r 3 100. (b) Volume of vessel
= Volume of roof
r = 1000094 h × r² × h = 22 × 20 × x
 r = 600 cm = 6 meters d = 4 (where x is rainfall in cm)
2 3
89. (b) Let no. of seconds required 22 100100350
to fill the tank = x 4d  × =x
7 2220100100
 (r²h)× x = 3 × 5× 1.54 h= , 3h= 2d
6  x = 2.5 cm
351.547100100 94. (a) Volume of water pumped out 101. (a) Volume of remaining solid
 x= in one hour = Volume of cylinder – Volume
22775

r
22 7 7
of cone
= 300 seconds

i
= × × × 12 × 3600

.iSn
 Time required = 5 minutes 7 2 2 1 2 2
pr2h – pr h = r²h
90. (c) = 1663200 cm³ 3 3
= 1663.2 ltr. 2 22

agv
95. (d) 2rh = 1056 × × 36 × 10 = 240 cm³
3 7
10567 21 102. (c) Let the height be H
r= =

ridn
22216 2 1 1
 r1² H + r2²H
22 21 21 3 3
Volume = r²h = × × ×16

eeYa
7 2 2 4
= 5544 cm³ = R³
r 3
96. (b) Let the radius and height be
= tan 1 4
h 5x and 12x
 H (r1² + r2²)= R³
 r = h tan
geisnh 3 3
1 2200
l  × × 25x² × 12x =
3 7 4R³
and = sec
h  H = r² r ²
220073 1 2
 l = h sec   x³ = =1
Enak
7222512 103. (c) Let height and diameter be
 S =× h tan × h sec  x =1 3x and 2x
= h²( tan × sec)
 slant height = 5²12² 1
R

 x² × 3x = 1078
91. (c) = 13 cm 3
1 10787
aryn

97. (b)r²H = r²h  x³ = = 49 × 7


3 22
 x=7
Les B

r ²h h
H= =  height = 7 × 3 = 21 cm
r ²3 3
 h = 3H = 3 × 7 104. (c)Radius of cylinder r = 10 cm
= 3 × 7 = 21 cm height of cylinder h = 21 cm
As DE ||BC,  ADE   ABC
wa.th

4 volume of cylinder = r2h


AD DE 98. (b) × r² × H = r³
= 3 r a d i u s o f c on e  r a d i u s of

AB BC cylinder =10 cm
1 1 4
12 – 3 DE 96 p× × × 36 × 100 = pr³ Let height of cone = h1
wwM

10 10 3
 =  = DE
12 6 12 r³ = 27 1
r = 3 cm  volume of cone =  r 2 h1
 DE = 4.5 cm 3
99. (b)
92. (d) Height of cylinder  volume of remaining portion
= Diameter of sphere  4400 cm³(given)
S 4r² 4r ² 1 (after removing cone)
1 2 3
 S = = =
2 2rh 4r ² 1 1 2
 S1 = S2 (h = 2r) r 2 h – 3  r h1 = 4400
r ²h  2 
4

Slant height =  2 3
   22   h1 
93. (d) r ³ = 1  r2 h –  = 4400
3   3

Rakesh Yadav Readers Publication Pvt. Ltd. 406

For More Visit : www.LearnEngineering.in


For More Visit : www.LearnEngineering.in

22  h1  108. (a) 110.(b)


× 10 × 10  21 –  = 4400
7 3
h mtr r r
 21 – 1 = 14 r 
3
r1
h r
21 – 14 = 1 r
3
height of cylinder h = 6 mtr  Circumference of sectors = 2
h1 = 21
Let radius of cylinder = r mtr
 curved surface area = 2rh  Circumference of base of cone
105. (a) area of end face = r² of radius r1 = 2r1

ir
 total area of two end faces =2 r² r
= 2r1

v.iSn
2
r
 r1 =
4
radius of cone = 6 cm

dnag
height of cone = 24 cm r
 given that  radius of cone =
1 2 4
 volume of cone = 6 × 24 cm³ 3 × 2r² = 2 × 2rh
3 3r = 2h curved surface area of cone = r1l

eYrai
cone is converted to sphere l = slant height
3 ×r = 2h
Let radius of sphere = r l=r
r = 4 mtr
4 3  radius of base = 4 mtr r
 Volume of sphere  r surface area of cone = r
3 109. (b) 
volume of sphere = volume of cone


4 3
3
1
r =  × 6 × 6 × 24
3
snhe 

r 2
4
4
kgei
1 6624 111.(d) radius of cone = r = 16 meter
 r³   ×3 (given)
3 4
 r³ = 3 × 3 × 24 Let slant height = l meter
ERna

radius of semi-circular sheet


= 3 × 3× 3 × 8 curved surface area = rl
r³ = (3)³ × (2)³ 28
=r  3 2
r = 3 × 2 = 6 cm 2 = 427 m (given)
7
 radius of sphere = 6 cm r = 14 cm
aBryn

106. (a) total surface area of cylinder Circumference of sheet = r 22 2992


 16  l 
 462 (given) = 14 cm 7 7
 (2rh + 2r² ) = 462 cm² Sheet is folded to form a cone 2992
Let radius of cone = r1 l = 8.5 meter
r = radius, h = height 2216
Les

2r ²  462 – 154  308 112. (a)


wa.th

r ²  154
154
r²   7  49
22
wwM

r = 7 cm  The circumference of base


107. (a) Diameter of cylinder = 7 cm of cone = circumference of sheet
 2r1 = 14
7
radius = cm, height = 16 cm r1 = 7 cm
2 105
 radius of cone = 7 cm
 lateral or curved surface area  radius of cone = m
slant height = radius of semi-cir- 2
 2rh cular sheet slant height of cone = 63 m
 r = radius l = 14 cm  curved surface area of cone = (rl)
h = height 22 105
 height= 2 2 = × ×63
22 7 14 –7 2
7
 2× × × 16  352 cm²
7 2 = 1 4 7 = 12 cm (approx) = 10395 m²

Rakesh Yadav Readers Publication Pvt. Ltd. 407

For More Visit : www.LearnEngineering.in


For More Visit : www.LearnEngineering.in

115. (d) Let radius of iron rod = r 33


105
 radius of cylinder = m  height = 8r 121. (c) Radius of base =
2 2
 volume of iron rod
height = 3 m (given) = × (r)² × 8r  8r³ 33  7 21
 curved surface area of cylinder =2rh = = cm
2  22 4
r
22 105  radius of spherical ball = 1
=2× × × 3 = 990 m² 2  Volume of cone = ××r² × h
7 2 3
volume of spherical ball
 Total curved area of structure 3 1 22 21 21
4 r  × = × × × 16
 curved area of cone + curved = ×   3 7 4 4
3 2
 
area of cylinder = 10395 + 990 = = 462 cm³
Let n balls are cast

r
11385 m² 122. (a) Let the radius of small
Total area of canvas 4  r³

i

.iSn
 n× 
   = 8r³ spheres be r cm
= 11385 m² 3 8  4  4
n   3r³  × 8 = ×(3)³
113. (b)   8  n = 48   3
6

agv
 8r³ = 3³
116. (c) Let the radius of base of sec-
ond cylinder = R 3
 r= = 1.5 cm

ridn
 2(r²h) = R²h 2
123. (b) 12
 2r² = R²

eeYa
Surface area of hemishpere R= r 2
6 cm
22 117. (c) Volume of remaining solid
= 2r² = 2 × × 9 = 56.57 cm² = Volume of cylinder – Volume
7
height of cone = 4 cm of cone
geisnh Let the increase in height= h cm
radius = 3 cm 1 2 2
pr2h – pr h = r²h 4
 slant height = 169 = 5 cm 3 3  R²h = r³
3
 surface area of cone = rl 2 22
Enak

= × × 5× 5 ×12 4
22 3 7 (12)² × h = × 6³
= ×3×5 3
7 = 628.57 cm³
 47.14 cm² 118. (a) Let the required increase 4 216
R

h × = = 2 cm
 total surface area of the toy area = x cm 3 144
of cone + area of hemisphere  (10 + x)² × 4 =× 10² × (4 + x) 124. (c) Height of the cone = 10.2 – 4.2
aryn

 47.14 + 56.57  103.71 cm² 100 + x² + 20x = 25(4 + x) = 6 cm


114. (b) diameter of beaker = 7 cm x² + 20x + 100 = 100 + 25x
Les B

7 x² – 5x = 0
radius = cm x–5=0
2
x=5
level of water rises = 5.6 cm
diameter of a marble = 1.4 cm  Required increase = 5 cm
wa.th

1.4 1
 radius = = 0.7 cm 119. (b) Let the old volume = r²h
2 3
Let n marbles are dropped so, 1  Volume of the toy
 New volume =  (2r)²h
wwM

Volume of n marbles 3
4 1 4
=n×  × (0.7)3 4 = r² h + r³
3 = r²h 3 3
3
4 7 7 1
 n×  × (0.7)3 = × × ×  New volume is four times the old = r² (h + 4r)
3 2 2 3
5.6 volume
120. (b) Let the height of cone be ‘h’ cm 1 22
4 7 7 7 = × × (4.2)² (4× 4.2 + 6)
 n× × × × 1 4 3 7
3 10 10 10 r²h = r³
7 7 56 3 3 1 22
= × × a² h = 4a³ = × × (4.2)² × 22. 8
2 2 10 3 7
h = 4a = 421 cm³ (approx.)
 n = 150

Rakesh Yadav Readers Publication Pvt. Ltd. 408

For More Visit : www.LearnEngineering.in


For More Visit : www.LearnEngineering.in

125. (c) Volume of water 136.(c) Slant height of cone


b
= Volume of cylinder – volume of 2 ×h= a
cone  = 82  62 = 10 cm
a Slant height of cone = radius of sector
1 2 2 h = cm
= pr2h – pr h = r²h 2 b = 10 cm
3 3 137. (d) Volume of sphere
1  129. (a) 2 π r³ = 19 40 4 4 4 3
= 2
 r²h 
 3 = r³ =  9
 3  19404  7  3 3 3
= 2 × 27 = 54 cm³ r³ = = 972 cm³
22  2
126. (c)  r = 21 cm 1 1

ir
volume of cone = r ²h   9²  9
 Total surface area = 3r² 3 3

v.iSn
22 = 243 cm³
= 3  21  21 = 4158 cm²  % of wasted wood
7
130. (c) Curved surface area 972 – 243100 = 75%

dnag
= 2r² =
Height of water after ball is im- 972
mersed = 3.5 × 2 = 7 cm 138. (d)
22
4 3 = 2  11  11 = 760.57 cm²
 Volume of water = r² h – r 7

eYrai
3 131. (a) Slant height of the cone (l)
 4 
 r² h – 3 r  = 62  82 = 10 cm
 
22  4  2rh 2h
 7  3.5  3.5 7 – 3  3.5  =

Volume of water before ball was
immersed

 Required ratio =

28
snhe rl
=
l
a
2r = a,r = 2
= = 8 : 5
kgei
10 Volume of cylinder = V
22  4 
 3.5  3.5 
7 –  3.5 
 132. (c) The volume of cone having  r ²h = V
7  3 
same height & diameter as that 2
ERna

= (3.5)² × h of a cylinder  a 
  h = V
7 1  2 
=h = cm n×  r2 h =  r2h
3 3 a 2
127. (c)Let height and radius be 4 =V
aBryn

1
=  volu m e of cylinder 4 2
= 7x and 5x respectively 3
 r² h= 550 V  4 4V
No. of cones required = 3 h= 
a² a²
(5x)²× 7x = 550 133. (a) Let the no. of small balls = x
12
22 3 139. (d) Radius of sphere = = 6 cm
4 3 41  2
Les

 25 x ²  7 x  550   10 = x ×  


 
7 3 3 2  Let the height of the cylinder = h
wa.th

3
x = 1 1 According to questions,
x = 1  1000 = x  Volume and radius are same
8
 height = 7 cm
 x = 8000 4
radius = 5 cm (6)² × h =  (6)³
134. (a) Let the no. of balls = x 3
wwM

 Curved surface are = 2rh


3 46
4 4 
h= = 8 cm
22  44 × 44 × 44 = x × ×π
  3
= 2  5  7 = 220 cm² 3 2 
7 140. (b)
44  44  44  7  3
128. (c) Let the height of the  =x
cylinder be ‘h’ cm and the radius 22  4  8
be r cm  x = 2541 h = 21 cm
 r² = b 135. (d) Let the no. of cones = x
2
b 2 1 1 
 r =   3  5 = x ×    1
 3 10  Perimeter of base = 8 cm
also, 2rh = a  x = 9×5×3×100 = 13500 2r = 8

Rakesh Yadav Readers Publication Pvt. Ltd. 409

For More Visit : www.LearnEngineering.in


For More Visit : www.LearnEngineering.in

143. (b) R atio of the v olum e of r = 7 cm


8
r = c on e s Slant = radius of
2
1 height,(I) of semicirular plate
h = 21 cm  r ²h  r 2  2
3 1  1   3   9 l = 14 cm
Volume of cone = 1 
r    
 r ²h  2  4  16 h² = l² – r²
3 2
1 4 4 112 = 14² – 7²
=    21 = cm³
3   π = 9 : 16 = 196 – 49
141. (c) 144. (c) Ratio of surface area of = 147
sphere
h = 147 = 7 3
4r ²
h1 h2 1 4
4r ² = 9 1 22

r
2 Volume of cone =  × 7
3 7

i
.iSn
r1  r 2
r2
 1  4 ×7× 7 3
1 
V r ²h r ²h r  9
1  3 1 1  1 1  4  2 = 622.36 cm³
1 2

agv
V
2 r ²h 2 r2 h 2
1
r 147. (d)
3 2 1 2 r

r 2 h 
4 r 3
 1  2

ridn
 1 

r 
 × 
h 
 = Ratio of their volume
 2   2  1 H
h
2r r
1  5 1  5 4

eeYa
 r ³ 3 3
2r 4 r 4 3 1 r  2 8
2 2  1
= 4 = r  =   = Volume of water in conical flask
 5 2 h1 4 r ³  2 3
  27
    = 3 2

4 h 1 1
2
= 8 : 27 = r ²h
3
25
 1 
4 h
geisnh
145. (a) If the height of water level in
16 h2 1

h r cylindrical flask = H units


1 64
h =
2 25 1
142. (d)  m²H = r ²h
Enak

3
Total surface area of sphere
= 8 squares 1 r ²h hr ²
H=  
3 m ² 3m ²
R

h 2m 4r² = 8
l r² = 2 148.(d) R=7cm
aryn

r= 2 units r
r h
7cm
r ² = 154 4
Les B

Volume of sphere = r ³
154  7 3
r² = = 49 4
22 4 8 2 volume of the solid sphere = r ³
r = 49 = 7 m
=
3
  2 ³ = 3
units 3
wa.th

also, volume = 1232 146. (b) 4


=  × 7 × 7 × 7 cm³
1
This part becomes the circumference of cone 3
r ²  h = 1232 r Let the length of wire = h cm
3 14
4
1232  3 1232  3 28 R²h =  ×7×7×7
wwM

h= = =8 3
r ² 154 radius will be
become slant height
l
h=8m 4
7×7×h= × 7 ×7 × 7
Area of canvas required = rl In this question just cut the 3
= r r ²  h ² semicircular paper and told it 28
to form cone h= cm
3
22 Circum fere nce of b ase of
=  7  24²  7²
7 cone= 2r 149. (b) 6
22
= × 7 × 25 = 550 m² 2 r
7  2
= circumference of semi r =3
length × 2 = 550 m² 3 cm
circlar Sheet
550
length(l) = = 275 m 2 r =  × 14
2

Rakesh Yadav Readers Publication Pvt. Ltd. 410

For More Visit : www.LearnEngineering.in


For More Visit : www.LearnEngineering.in

volume of both the cones will be According to question,


4
Volume of sphere = r ³ equal to the volume of sphere 500
3 n× = 550000
1 1 4 8
4  (3)2 h+  (4)2h =  (5)3
=  × 3× 3×3 = 36 cm³
3 3 3 550000  8
3 n= = 8800
1 4 500
If the water level rises by H cm h(32+42) = ×5×5×5
3 3 157. (a)Required number of spheres
R²H = 36
6 × 6 × h = 36 volume of metallic cone
1 4 =
h = 1 cm × h × 25 = ×5×5×5 volume of a sphere
3 3
150.(b)
1

ir
20   30  30  45
R =9 r = 0.2 cm
h= ×3 = 20 cm 3
3

v.iSn
h = 4 = 81
 555
1 3
4 154. (a) Volume of cone = r ²h
Volume of sphere = R³ 3 158. (d) Number of cones
3
Now, r1 = 2r, h1 = 2h

dnag
Volume of sphere
972 cm³  Volume of s econd cone = Volume of cone
Let the length of wire = h cm
1
(0.2)²×h = 972 = r1 ² h 1 4
3 10.5 ³

eYrai
972 3
h= = 24300 cm = 1
0.2  0.2 1 2 1 2 3.5 ²  3
= 2r 2h = r h × 8 3
= 243 metres 3 3
151. (a)Volume of = volume of = 8 times of the previous volume

4
sphere

 (radius)³
rectangular block Alternate: snhe
In the formula of volume of cone,
there is power 2 on radius and
=

159.(c)
4 10.5 10.5 10.5
3.5  3.5  3
= 126

3
kgei
power 1 on height
= length × breadth × height  (2)² × 2 = 8 times
= h
4 155. (d) 3.6 cm
(radius)³ = 21 × 77 × 24
ERna

3
h l
1.6 cm 1.2 cm
21  77  24  3  7
(radius)³= According to question
4  22
r 1
aBryn

(radius) = × 1.6 × 1.6 × 3.6


3 777333 C = rl 3
C² = ²r ²l ² 1
radius = 7 × 3 = 21 cm =  × 1.2 × 1.2 ×h
152. (d) 1 3
V =  r ²h
3 1.6  1.6  3.6 16 16  36
Les

1 h= 1.2  1.2 =
V² = ²r 4h ² 12 12 10
wa.th

r =50 9
3vh³ – c²h² + 9v² 64
= = 6.4 cm
1 1 10
3× r²h × h³ – ² r² l² h² + 9 × ²r4h² S³ 4 r ² ³
Volume of cylinder 3 9  4 ³ ³  r 6
160.(a) = × 3²
wwM

V ² 4 2
= 6 × volume of a sphere = ²r²h4 – ²r²h²(r² +h²) + ²r4h²  r ³
  4 ² ²  r 6
= ²r²h4 –²r4h² – ²r²h4 + ²r4h²  3 
2 4
50 h = 6  350 ³ =0 36
156. (a) volume of rectangular block =4 × ×9 = = 36 units
1
4 = 11 × 10 × 5= 550 m³
h = 6  50 1
3 = 550000 dm ³ (1 m = 10 dm) 161. (d)Radius of sphere = cm
20
= 400 cm = 4 m Volume of a sphere
153.(b) volume of a sphere
4 5 5 5
=  × × × dm3 4 1 1 1
+ h 5cm
3 2 2 2 =   
h = 3 20 20 20
500 Let the radius of cone = R
= dm3
3cm 4cm 8 height = 2R

Rakesh Yadav Readers Publication Pvt. Ltd. 411

For More Visit : www.LearnEngineering.in


For More Visit : www.LearnEngineering.in

According tio question, volume of cylinder = volume of


4 8 water
1 π R³ = π r³
 × R×R×2R 3 3 2
3 r
R³ = 2r³ π    H = 8 π r²
4 1 1 1 R³ = 2(2)³ 3
= ×   ×32000 H = 9 × 8 = 72 cm
3 20 20 20
R = 3
2  2 = 1.259 × 2 168. (b) volume of metallic sphere
2  32000 64000
R³ = = = 2.518 = volume of cone
20  20  20 20  20  20
 Thickness = R – r 4
40  40  40  π×3×3×3
R³ = = 2.518 – 2 3
20  20  20
= 0.518 cm 1

r
= π R² h
40 3

i
.iSn
R= =2 165. (a)
20 4
height of glass = 2R = 2 × 2 = 4 cm 7  π×3×3×3
7 3
162. (c) Volume of earth taken out
1

agv
= 40 × 30 × 12 = 14400 m³ = ×π×6×6×h
7 3
Area of rectangular field
108

ridn
= 1000×30 = 30000 m² h= = 3 cm
Area of region of tank = 40 × 30 = 1200 m² 66
In the que stion, R ad ius of
169. (d) Number of bottle
Remaining area = 30000 – 1200 hemisphere = Radius of cone =

eeYa
= 28800 m² height of cone = 7 cm volume of hemispherical bowl
( height of hemisphere =
14400
volume of cylindrical bottle
Increase in height = = 0.5 m = radius of hemisphere) volume
28800 of ic e cr eam = volume of 2
geisnh  15 15 15
hemisphere part + volume of 3
= = 60
163.(a)
.r R = 6 cm
. conical part 5 5
π×   6
2 2
2 22 1 22
H = 12 cm = × × (7)3 + × × 73 170. (a) volume of cone V1
3 7 3 7
Enak

1
22 = π r² h
= × 73 = 22 × 7² =1078 cm³ 3
According to question, 7 π
R

166. (c) = r³  h = r 
2 1 3
8× π r³ = π (6)² × 12 Volume of sphere , V2
3 3 . 4 cm
aryn

4
6  6  12 .r =
3
π r³
Les B

r³= R
82 Volume of cylinder V3
= 3 × 3× 3 = π r² h = π r³
r= 3
33 3 Volume of material of hollow sphere 1 4
= Volume of cone  V1 : V2 : V3 = : : 1
wa.th

= 3 cm 3 3
4 1 = 1:4:3
(53–33) = (4)2 × h
164. (d) 3  3  V2 V
= 3
V1 =
r 98 = 4 h 4 3
wwM

R 171. (c) 4 π radius2 = 346.5


98
h = = 24.5 cm 346.5  7
4
radius2 = = 5.25 cm
167.(d) Radius of the base of conical 4  22
4
π r³ shape = r cm 172. (b)
Volume of lead =
3
r
 Radius of base of cylinder =cm
4 4 3
Volume of Gold =  R3 –  r3
3 3 Volume of water = volume of cone
According to question, 1 1
=  r2 h =  r2 × 24
3 3
4 4 4 Height of kaleidoscope = 25 cm
π R³ – π r³ = π r³ = 8 π r² cm³
3 3 3

Rakesh Yadav Readers Publication Pvt. Ltd. 412

For More Visit : www.LearnEngineering.in


For More Visit : www.LearnEngineering.in

Radius of kaleidoscope = 35 cm OC = 2P =2(1) = 2 cm


paper used = curved surface area 3a = 2 × 6 3 then, CZ = OC + OZ
a = 12 = 2+1 = 3 cm
22 Surface area = 6a2 = 6 × 12 × 12
of cylinder = 2 ×  35  25 CZY, CZY  900
7  864 cm2
= 2 × 22 × 5 × 25 177. (c) Let hemisphere radius be = R CZ  3Q  3cm
= 5500 cm² & Sphere radius be = r
173. (b) According to the question, ATQ, YZ  1Q  3cm
Now, In cone XYC
Volume = Surface area 2 4
 of sphere of sphere R 3  4  r 3 r  ZY  3cm
3 3
h = CZ = 3 cm
4 3 r

ir
2R = 16r3
3
 3 r = 4  r2  3 = 1 Vol.of cone
R3

v.iSn
8 R 2 1 1
 r = 3 cm  r²h  ( 3 )²(3)
r 3
1 = r 1 3 3
 then diameter of sphere will be = 2r
= 2 × 3 = 6 cm 178. (d) Let part filled be 'x'  3cm²
ATQ,

dnag
174. (b) Let the height of cone 4
h metre) x × (48m × 16.5m × 4m) Vol. of sphere  rs³
= p (2)2 × 56 3
 Total area of ground will be
required 22  4  56  rs  1 cm

eYrai
= 5 × 16m2= 80 m2 x
 7  48 16.5  4 4
Total volume of air is needed   cm³
= 100 × 5 m3 = 500 m3 2 3
According to the question x Vol. of water that can immerse
9
 volume of cone = 500 m3


1
3 area of ground × height = 500
snhe
179. (a) According to the question,

5
4
the ball

= 
 4
3 3 

 cm³ 

5
3
cm³
3
kgei
1 181. (b) Here h = 4c,
 2
3 × r  h = 500 Volume of cylinder = r² h
Whole surface of remaining Solid
ERna

=  rl + 2  r h + r ² 4r²h
1 =
=  80 ×h = 500 4
3 Hence l = h²+r²
(Multiply 4 both in Numera-
l = 4²+3² tor & denominator )
 height = 500  3
aBryn

80 l=5 2
 height of cone = 18.75 metres =
2r  4c = c3
 = r  l+2h+r  4 
175. (d) Volume = Lateral 182. (a) According to the question,
of cone surface Area 22
= × 3 [5 + 2 × 4 + 3]
7 4 3
Les

1 2 Volume of sphere = πr
r h = rl [l = h +r ]
2 2 22 3
wa.th

3 = × 3 × 16= 48 
7 Volume of cyclinder = r²h
rh 180. (a) 4 3
2 2
3  h +r A P
B
πr = r²h
3
Squaring both sides
wwM

Z
4 4
1 h2  r2 Y X = r  h = h =  3 = 4cm
= 2 2 3 3
9 r h D O
C.S.A of cylinder = 2 π rh
1 h2 r2 22
= 2 2 + 2 2 = 2×  3 4
9 r h r h C
7
ΔABC = equilateral Δ
1 1 1 44  12 528 3
= 2 + 2 0 = = = 75 cm 2
9 r h ACB  600 &  BCP  30 7 7 7
176. (c) Diagonal of cube will be 183. (a) According to question,
equal to diameter of sphere CDO , CDO  900
and tangent is 90 R = 6 cm.
3a = 2 × r OD = 1P = 1 cm  The capacity of the

Rakesh Yadav Readers Publication Pvt. Ltd. 413

For More Visit : www.LearnEngineering.in


For More Visit : www.LearnEngineering.in

hemispheri-cal bowl
187. (b) 4.2 dm 1 22 7 7
=     7 = 89.8 cm³
2 2 22 3 7 2 2
=  r³ = × × 6³
3 3 7 191. (b) Radius of Ist solid metallic
= 452.57 cm³ spheres = R = 6 cm
r=2.1 dm
Radius of IInd solid metallic spheres
184. (d)
= r = 1 cm
Internal Radius of hollow sphere = x
20 cm External Radius of hollow sphere
r = 2.1 dm = x+1
h = 4.2 dm 4
So,   R 3 +r 3 

r
(for Max.) 3
7 cm

i
.iSn
1 4  3
According to the question, Volume of cone =  r ²h =   x  1 – x 3 
3 3  
 r = 7 cm 216 + 1 = x³ + 1 + 3x(x+1) – x³
 h = 20 cm 1 22 216 = 3x (x + 1)

agv
=   2.1  2.1  4.2
 Total surface Area of cylinder = 3 7 72 = x² + x
curved surface Area + 2 × area = 19.404 dm³  x² + x– 72 = 0

ridn
of base 188. (d) Let the intial radius = r After solving,
= 2rh + 2r 2 According to the question. x = 8 cm

eeYa
4(r + 2)² – 4r² = 352 so, the external radius of the hollow
= 2r [r + h] sphere
4[(r + 2)² – r² ] = 352
22 = x + 1 = 8 + 1 = 9 cm
=2× × 7 (7 + 20) = 44 × 27 352  7 192. (a) Let the time taken to fill
7 r² + 4 + 4r – r² =
geisnh 22  4 the tank = x hrs
 TSA of cylinder =1188 cm²
4r + 4 = 28 7
185. (d) According to question, (r²h) × x = 50 × 44 ×
4r =24 
100
Given:
r =6
 Radius of cylinder = r 50  44  7  7  100 100
Enak

189. (a)
 CSA of cylinder = 4  r h  x=
22  7  7  100  5000
 As we know = 2 hrs
R

 Curved surface area of cylinder 193. (b) The area of ground


= 2 R H  1.5 hectares =1.5 × 10000 m²
4  r h = 2  × r × Height
aryn

1  15000 m²
 Height = 2h unit Volume  r²h (\ 1 hectare = 10000 m²)
3
Les B

186. (a) According to the question,  level of rainfall


Radius = 3.5 cm 1 = height of water level
 12 ×12 × 9
 In the question it is given that A 3
5
hemi-shperical bowl is to be  144 × 3 = 5 cm = m
100
wa.th

painted inside as well as outside  432


Total area that is to be painted  volume of collected water
190. (c) 7 cm
= Inside area of bowl + outside area 5
of bowl  15000 × = 750 m³
100
= 2  r² + 2  r²
wwM

194. (d) Required quantity of water


= 4  r² r= 3.5 cm
3  40  2000
22 7 7 = = 4,000 m3
= 4   cm² 60
7 2 2
195. (b) Let r = Radius of pipe
 painting Rate = 10cm² in 5 Rs. Let R = Radius of cistern
According to the question,
5 1 1 Let H = Height of cistern
2
1 cm² will be painted = = Rs. Volume of cone =  r h Let the no. of hours be ‘x’
10 2 3
so total cost will be painted in Height = 7 cm x R ²H r ²h
 
22 7 7 1 7
Radius = cm 10 10
= 4    = Rs. 77 2 3000 ×  × × ×x
7 2 2 2 
100 100
 Volume of cone

Rakesh Yadav Readers Publication Pvt. Ltd. 414

For More Visit : www.LearnEngineering.in


For More Visit : www.LearnEngineering.in

3
10 10  r1  8
=× × ×2  2 =
2 2 r  27
6  r 2
  1   2 2
 ×x=1  2  =  
10  r  
3 

 2  
10 Cost of levelling the ground 4πr ²
x= = 1 hour 40 minutes 1
6 = 2rh × 500 × 1.5 Ratio of surface area = 4πr 2
196. (a) Diameter = 5 mm = 0.5 cm 2  22  0.42 1.2 1.5  500 2
radius = 0.25 cm = 2
7  r1   2 2 4
volume of water flowing from =   =   =
= 2376 3  9
 r2 

ir
the pipe in 1 minute
200. (a) since the volume of the
=  × 0.25 × 0.25 ×1000 cm³ H

v.iSn
two cylinders is same R
volume of conical vessel 204. (b) 1 = 2, 1 5
πr 21h1 3 H = 3
= 1 R 2
1  πr 2 h 2
=  × 15 × 15 × 24 cm³ 2 2
3 V
h1 1 1

dnag
1  Ratio of volumes = V
=  × 15 × 15 × 24 cm³ h2 2 2
3 2
1 r 
2
h πR 2 H R  H 
1 1 1
 15 15  24 1
  =
2 2
= =     1
 Time = 3 h = 1 πR 2 H

eYrai
r  R  H 
 0.25  0.25  1000  2 1 2 2  2  2
r 2
4 1 2 2 2 5 4 5 20
= 28 =     = × =
5 r = 1 = 1 3
  3
  9 3 27
2
= 28 minutes 48 second
197. (d) r = 12 m,h = 9 m
l= r ²  h ²  12²  9² = 15 m
 snhe
Ratio of their radii = 2 : 1
201. (a) We are given that volume
of two cube are in the ratio = on dividing:
π
205. (d) 2 r h = 264
π r² h =924
2π r h 264
.... (i)
....(ii)

=
kgei
cost of canvas = curved surface 27 :1 π r² h 924
area × cost of 1 m² 3 2 264
a  =
= rl × 120  1 27 r 924
ERna

= 3.14 × 12× 15 ×120 


a   =
 2 1 924  2
= Rs. 67824 r = = 7 cm
a 264
198. (d) 1 27 3
a = 3 = = 3 : 1 diametre = 2r = 2 × 7 = 14 cm
2 1 1 putting, r = 7 in (i)
aBryn

0.1 cm 202. (a) Ratio of edges of cuboid 2π r h = 264


= 1:2:3
264  7
x Let, h =
x l = x, b = 2x, h = 3x 2  22  7
= 6 cm
Surface area = 88 cm²
Les

8.4 gm = 1 cm³ 2 (lb + bh + hl ) = 88 Required ratio = 2r = 14 = 7


wa.th

4725 2 (2x² + 6x² + 3x²) = 88 h 6 3


4725 gm = cm3 1 2
8.4 11x² = 44  r1 h1 2
x² = 4, x = 2 3
206. (b) 1 2 2  3

4725
volume = x × x × 0.1= cm3  l = 2 cm, b = 4 cm, h = 6cm  r2 h
wwM

8.4 3
 Volume = l b h
 1  h1 2
4725 = 2×4×6  2  
= x² × 0.1 = cm3 
 2  h2 3
8.4 = 48 cm³
x = 75 cm V h1 8
1 8 
199. (d) According to the question. 203. (a) h2 3
V = 27 3
diameter = 84 cm 2
radius = 42 cm = 0.42 m 207. (c)
 a1  =
27
height = 120 cm = 1.2 m 4 3 64
 Curved surface area of cylinder 3
π (r )³
1
8
a2 
4 = a1 3
2  22  0.42 1.2 π (r )³ 27 =
= 2rh= 3 2 a2 4
7

Rakesh Yadav Readers Publication Pvt. Ltd. 415

For More Visit : www.LearnEngineering.in


For More Visit : www.LearnEngineering.in

Ratio of their total surface area ratio of smaller cone h1 ³


1 
2 r ²  h
6a 2 a  3 2 2 r2 ²  h 2 ratio of larger cone h ³
1 1 =  2
= =   1 r ² h
6a 2 a  r ²  h 1 1 1³ 1
2  2 3 1 1  
2³ 8
2
 3 9 area of part (ABCD) (i.e frus-
=   =
16
= 9 : 16 2r  ²  h
1 1

4
tum) = 8 – 1 =7
4
  = r12  h1 1
208. (b)Radius of both hemisphere  Required ratio = 1 : 7
and cone = R 4 : 1
216. (a)
Also height of hemisphere is 212. (d)
equal to its Radius = R

r
 height of both hemisphere 5 1

i
.iSn
and cone = R 2
6 1
Now, In cone Ratio of total surface area = area of incircle
2
6 5 
slant height, l = R²  R² area of circum circle

agv
1
= 2R  = = 1 : 25 a 2
25  
2  1

ridn
C.S.A of hemisphere 21 = = 1:2
213. (b) Let r1 = cm 2 2 
C.S.A of cone 2 a 2 
 2 

17.5

eeYa
2 π R² r2 = cm  
2 2 217. (b)
= = R
πR× 2 R 1
4r ² r ²
1 1
= 2:1  Required ratio = 4r ²  r ² H =2R

209. (c) Let height of cone = h


geisnh 2 2 R

radius of cone = r 21  21 36 (height of cylinder = 2 ×R)


=  = 36 : 25
1 17.5  17.5 25 Surface area of sphere
volume of cone = π r² h
3 214. (a) C.S.A of cylinder
Enak

Now height is doubled ,


4R ² 4R ²
volume of new cone 6 cm =
= + + 2R  H 2R (2R )
R

1 2 3x 4x 5x
= π r² (2h) = π r² h 4 R 2
3 3 1
= = =1:1
4 4 R 2
aryn

Required ratio = 1 : 2 {(3x)³ + (4x)³+ (5x)³} 1


210. (d) 3 218. (d) Ratio of volume = (Ratio of
Les B

r1 = 40 cm r2 = 10 cm
4 radius)³
= (6)³
3 4
A B
r13 3
x³ (27 + 64 +125)= 216 V1 3 r 
1 
= V = 4 
=  
4r ²  r 2 x³ × 216 = 216 r23 r2 
wa.th

surface area of A 1 1  2
= =
  3
Surface area of B 4r2 ² r2  216
x³ = =1 3
216 3 27
2 =   =
 40  16 3 2
  8
1 =1 x =
wwM

= 
   =  16 : 1
10  1 Radius of smallest sphere = 3x = 3 × 4
219. (a) r³ = r²h
211.(d) 1 = 3 cm 3
4
h1 h2 h = r
215. (d) 3
M
3
r1 2r1 r = h
4
h=
1 1
r2 = 2r1 A D 3 3
h2 =2  2r(diameter) = × 2h = h
h2 = h1 4 2
volume of new cone Diameter 3
B C  =
volume of old cone Height 2

Rakesh Yadav Readers Publication Pvt. Ltd. 416

For More Visit : www.LearnEngineering.in


For More Visit : www.LearnEngineering.in

220. (a) In this case height of cylin-


der and cone is equal to the ra- H 4 a ³ 363  22  4
 = 
dius of hemisphere h 9 r³ 49  7  3
 h = r Ratio of volumes 226.(a) Let the radius and slant
height be 4x and 7x a³  22 3
Cone : hemisphere : cylinder 
 

2
 7x = 14 cm r³ 7 
= 1
r ² r :  r ³ : r ²  r x = 2 cm
3 3 a 22
 Radius = 4 × 2 = 8 cm 
= 1:2:3 r 7
221.(a) Ratio of curved surface area = 227. (d) Height of cylinder
= Diameter of sphere 233. (a) cone  radius : height
Ratio of product of height and radius
 4r² : 2rh 4 : 3
 Required ratio

ir
Let 4x : 3x
C1 2r1h1 r1h1  4r² : 4r² (h=2r)
 curved surface area of cone =rl

v.iSn
C 2 = 2r2h 2 = r2h 2  Required ratio = 1 : 1 r = radius
32 V l = slant height
2 1 r²h 3²  4 3
= = 228. (a) V = 2 = = l =
53 5 R H 4² 3 4 h²  r²

dnag
2 
222. (d) Ratio of surface area
= (Ratio of radius)² 229. (d) Ratio of volume of bigger = 4x  ²  3x ² = 5x
2
cone and smaller cones
C1 4r12 r   curved surface area
 1 = (Ratio of altitude)3

eYrai
= C = 2 =   = (1 : 2 : 3)3   × 4x × 5x
2 4r2 r2 
= (1 : 8 : 27)  20x²
2
1  1  Ratio of parts = 1 : 8–1 : 27–8  total surface area  r l + r2
= 
  = 16

4 = 1 : 7 : 19  r (l + r)

223. (a)
R²H
r²h
=

33H
3
1
3
sphere be r snhe
230. (c)Let radii of cylinder and

 Volume of cylinder of height (h)


  × 4x (5x + 4x)
  × 4x × 9 x
 36 x2
kgei
 = = r2h
2 2 h 1  Curved area : total area
Total surface area of cylinder
H 4 20 x2 : 36 x2
  = 2rh + 2r² 5: 9
ERna

h 3
Total surface area of sphere 234. (b) Let radius of sphere = r
x 4
 = = 4r² radius of cylinder = r
1 3
 given that  let height of cylinder = h
4
given that volume of sphere
aBryn

x = 4r² = 2rh + 2r² 


3 = volume of cylinder
4 4 4r² = 2r (h + r)
224. (c) R 3  64  r 3  2r =h+r
4
3 3  r³ =r²h
3
4 r =h
R³  radius of sphere or cylinder's 4
Les

3  r=h
equal to height of cylinder 3
4 = 64
wa.th

r³  Ratio of volume of cylinder and  curved surface area


3 cylinder : sphere
sphere
3
8  4 4r
  = (4)³
  = r² ×r : r 3 = 3 : 4 2× × r × : 4r²
r  3 3
wwM

 r = 2 cm 4 8
231.(c) R ³ r ²H  : 4
C1 4R 2 R 2
3 3
  2 : 3
C 2 = 4r 2 =  r  4
R ³  r ²H 235. (c) radius of cone = radius of cyl-
Ratio of area =(Ratio of radius)² 3
= (8 : 2)² inder = r height of cone = height of
= 16 : 1 R² 3 cylinder = h
 (  H = R)
r² 4
R 2 H curved surface area of cylinder
1 R:r= 3 : 4  3 :2
225. (a) 2
=1 curved surface area of cone
R h
2 a³ 363
232. (b)  2rh 8
3²×H 4 49  =
=1 r ³ rl 5
2²  h 3

Rakesh Yadav Readers Publication Pvt. Ltd. 417

For More Visit : www.LearnEngineering.in


For More Visit : www.LearnEngineering.in

h 4 A 2r³  45 
 = = = 3 : 2 =  2  9  = cm
1 5  B 4  
r³
3 Volume of prism = Base area ×
h² 16
 = Height
l² 25 6 3
242. (c) Side of cube = 45
 l² = h² + r² 3 810 = × 9 × Height
= 6cm 2
 h² = 16
25 = 16 + r² 6  6² 810  2
Required rate =  1 : 1
Height = = 4 cm
r² = 9 6³ 45  9
r=3 243. (d) Let the radius of hemi-
sphere and sphere be ‘r’ and ‘R’ Total surface area = Lateral

r
 radius : height Surface Area + (Bottom + Top)

i
3 : 4 4 2 3

.iSn
Area
 R 3  r
236. (a) Volume of prism = Base 3 3 = (Perimeter of Base × Height)
Area × Height + (2 × Base area)
R3 1
1

agv
=  45 
1056 = (8 + 14) × h × 12 r 3 2 = (45 × 4) + 2 × 9  2 
2  

ridn
1056  2 R 1 = 180 + 405 = 585 cm²
h= = 8 cm 
22  12 r 32
247. (a)
237. (b) R atio of volume =

eeYa
 Ratio of curved surface area r1 r2
2
 3  2
  2 1
3 3 4R 2 2R 2 1
1 2 = = = 2 = 3 2 ² = 23 :1 Ratio of volume of sphere × ratio of
 2  3
  2 2r2 r   weight per 1 cc. of material of each
3
geisnh
Volume of cylinder 3 = Ratio of weight of two sphere
= 3 3: 2
244. (b) = 4
1 volume of cone 1 r ³
r12h1 2 r2 3 1  289  8
V1 3 r  h
r1 4

=   
1  1 64 17
Enak

238. (a) V = 1 2 r ³
2  r2 h 2
r
 2 h 2 3 2
3 h
2 h r ³ 8  64
3 1 1 88 8
R

Ratio of volume = 2 =3 r ³ = 17  289 = 17  17 17


2
5  3
: 25
aryn

 r12h  r
239. (d) Let the radius of sphere and  1  8
hemisphere be = R and r 1 2
 r2 h
1 r 17  8 : 17
Les B

2
4 2 3
 R³ = r³ R
3 3  r1 = r2 1 1
2R³ = r³ 248. (a) R =
Diameter of cylinder = Diam- 2 2
R³ 1 eter of cone
wa.th

=
r³ 2 245.(d) r V1 2
=
 R : r = 1 : 32 V2 3
240. (a) Ratio of radius of earth and r
moon = 4 : 1 h
1
πR 2 H
wwM

 Ratio of volume = 4³ : 1³ 3 1 1 2
=
4 Volume remains same 1 2 3
r13 3 πR
2
H
2
V1 3 r 
1  volume of sphere = volume of cone 3

V2 = 4r23 =  r2 
 = 64 : 1
4 1
3 r 3   r ²  h R 
2
H 
3 3
241. (b)Let the radius of cylinder  1    1 = 2
and sphere be = r cm 4r = h R  H  3
 2  2
 height of cylinder = 2r cm h 4
 =4:1
 A = r² × 2r = 2r³ r 1 2 H
1 1 2
4 246. (b) Area of  (Base Area) =    =
B = r³ Semiperimeter × in radius 2 H
2 3
3

Rakesh Yadav Readers Publication Pvt. Ltd. 418

For More Visit : www.LearnEngineering.in


For More Visit : www.LearnEngineering.in

1 H1 2 Surface area of smaller shpere =


2
 (5)2  13cm
 12
4 H = 3  Surface area of larger sphere
2 and slant height of PDA =
H 4 π r² r²
1 2 4  = = Slant height of RCB

H = 3 1 4 π R² R²
2
1 ² 1 ² 1  122  (9) 2  15cm
8  2 = =
=  8: 3 2 4
3  1  2  Area of  ABQ
 (4)3   (2)3  23
249. (d) .R     = Area of  SDC
R/2    
1
4 = 1813 117cm
2

ir
 1 : 23 2

v.iSn
Let the Radius of cylinder = R
251. (c) A Area of  PDA  Area of RCB
 Therefore, Radius of sphere
h1 1
R 1015  75cm2
= B C 2
2

dnag
Volume of Right circular cylinder h2 & Area of ABCD (Base area)
= π R² H D E = 10 × 18
= 180 cm2
3 1

eYrai
4 R  Volume of Cone ABC Total surface area = Base area
Volume of sphere = π
  =
3 2  Volum e of BCED 1 + area of (  ABQ +
Volu m e of Cone ABC 1  SDC +  PDA +  RCB)
3 πR³ = = 180 + 2  117 + 2  75 = 564cm2
4 R Volu m e of Cone A DE 11
= π
3 8
=

According to question,
Volume =
6

Volume
snhe =
1
2
253. (a) volume of cone =

1 22
1
3
22
π r² h
kgei
of Cylinder of sphere If a cone is cut in any parts parallel = × ×1² × 7 = cm³
3 7 3
to its base then the ratio of volume
π R³ Volume of cubical block
π R² H = of smaller cone to the volume of
= 10×5×2 cm³ =100 cm³
ERna

6 larger cone is equal to the ratio of


the cubes of their corresponding Wastage of wood
π R² H × 6
= 1 heights/ radii/slant height(it is  22 
π R³
proved by similarity) = 100 –  cm³
 3
aBryn

Η 1
=  1:6  height of Cone (h ) 3
R 6  1  1
=   = 2  300 – 22  278
250. (d) Radius of larger sphere = R units  height of Cone(h )
2  =  = cm³
 3  3
4 h1 1
Its volume = π R³ 278
3 h2 = 3 2
Les

Now cones are formed with base % wastag e = 3  100


 h1 : h2 – h1
wa.th

radius and height same as the 100


radius of larger sphere 32 –1

1
=1:   =
278 2
= 92 %
 Volume of smaller cone = π R³ 3 3 3
3 1 :  2 – 1
wwM

and one of the cone is converted 252. (b) Height (h) = 12cm (given) 1
into smaller sphere 254. (c) Decrease in radius = 50% =
 Slant height of 2
Therefore volume of smaller sphere
SDC  Slant height of QAB increase in height = 50%
1
=  R³ 1 Increment
3 =
2 Original
4 1
 r 3 =  R³ (Let)
3 3
Radius Height Volume
r3 1 Original 2 2 (2)² ×(2) = 8
=
R3 4 12 15 50%
decrease
50%
increase
–5

r 1 12 13 9 New 1 3 (1)² ×(3) = 3


= 34 5
R

Rakesh Yadav Readers Publication Pvt. Ltd. 419

For More Visit : www.LearnEngineering.in


For More Visit : www.LearnEngineering.in

Reduction in volume 259. (d) height = 100% R a d i u s = 264. (c)


5 1 100% Radius Height Volume
=  100 = 62 % Original 10 10 (10)²×(10) = 1000
8 2 1  Increment 1  Increment
331
255. (a) Increase in radius = 100% 1  Original 1  Original
New 11 11 (11)² ×(11) = 1331
1 Height Radius Volume
= 1 is
1 Original 1 1 (1) 21 = 1 1331 – 1000
3  % Increase = × 100
1 New 2 2 2
(2) 2 = 8 Constant 1000
Increase in height = 100% = = 33.1%
1 = eight times that of original
Radius Height Volume 265. (a) % Change in height = %
xy change in volume = 100%

r
Original 1 1 (1)² ×(1) = 1
7 260. (b) use x + y + 266. (a) Volume of coffee
100

i
.iSn
New 2 2 (2)² ×(2) = 8
percentage change in area 2 3 2 22 3
= r =   4
7 3 3 7
% increase =  100 = 700% 5   –10 
1 = 15 – 10 +

agv
100 128
= cm3
1  Increment = 5 – 1.5 = 3.5% 3
256. (d) 20% 
5  Original

ridn
(3.5 % increase)
1 2
Height Remember : when change in area Volume of cone = r  h
3
is asked in the question, then use

eeYa
this formula to save your valuable 1 2 1024
91 time. 8 16 =
= 
= × 100 = 72 .8% 3 3
125 261. (d) Let old radius = r  Required percentage
257.(d)
geisnh 4 10 24 128
 volume = r³ –
3 3 3  10 0
= = 87.5%
r =15 cm h =15 cm New radius = 2r 1 02 4
3
4
Enak

 New volume = (2r)³ 267. (a) Decrease in base radius =


R = 15 cm 3 (Decrease in base area)1/2
1
volume of cone = 15 ² 15 4 1
3 1
R

= ×8r³  1 2
1 3 =   =
 15 ³ 9  3
3  Volume becomes eight times
aryn

Let initi al rad ius and hei ght


4 3 262. (b)
Volume of sphere = 15 be 3 r and h
3
Les B

Radius Height Volume  New radius and height are r and


Required Percentage Original 2 5 (2)² ×(5) = 20 6h
volume of cone 12 old lateral surface area
= × 100 New = 2××3r×h
volume of sphere 1 8 (1)² ×(8) = 8
= 6rh
wa.th

1  Volume decreases New lateral surface area


 15 ³ = 2××r×6h
3 20 – 8
= 4 × 100 % decrease (% deh) = ×100 = 12rh
15 ³ 20
wwM

3 = 60% 12rh
Required factor = =2
6rh
1 263. (d) Length 1  2
= × 100 = 25% 268. (c) Let the original radius be ‘r’
4 Breadth 2  6  Area = 4r²
258. (d) Height 3  9 New area = 4(2r²) = 16r²
Radius Height Volume Volume 6  108
Original 2 1 (2)² ×(1) = 4  New area is 4 times the old area
1
 New volume = 18 times the 269. (d) Edge is increased by 50% =
New 1 3 (1)² ×(3) = 3
original volume 50 1  Increased
4–3

% decrease = × 100 =  Increase in volume 100 2  Original
4
25% = 18 – 1 = 17 times Let original edge =2

Rakesh Yadav Readers Publication Pvt. Ltd. 420

For More Visit : www.LearnEngineering.in


For More Visit : www.LearnEngineering.in

 increased edge =3 273. (a) Volume of bucket 21


edge surface area  Slant height  a
1 2
2 4 = h(R² +r² + Rr)
3 altitude = 3a
+ 5 , 6 is constant 1 22
= × × 45 (28² + 7² + 28 × 7)  height of the pyramid =h
3 7
3 9 2
22  21a  2
   –
% increase = 5
4
 1 00 = 125%
=
7
× 15 × 1029 = 48510 cm³  2 
 
 3a
274. (c) Side of regular hexagon =
270. (b) Let the inital radius = 1 unit 21 2 9a ²
2a cm
New radius = 2 unit (radius is = a – 3a 2 =
4 4
doubled) 3 2

ir
Radius : Volume area of hexagon = 6  × 2a  3
4 a =

v.iSn
1 (1)³ 2
2 (2)³
 6 3a 2 cm²  Volume of pyramide
4 
 π is constant  1
3  5a = area of base × height
slant edge of pyramid = cm 3

dnag
1 1 2
7 1 3
 = × 6 3a 2 × a
2 7 8 3 2
Percentage increase = 100 5a
= 3 3a ³ cm³
1 2

eYari
= 700% 275. (c)
271. (c)
A

(h-R) (l-r)

O
R
D

r
slant edge 
H
snhe 5a
2
(Given)
 area of base = 40 × 40
= 1600 cm²
kgei
B C
M r
5a
Let height of pyramid= h
Let the radius of cone = r 2 1
 Volume = × h × area of base
ERna

and height = h, slant height = l 3


and radius of sphere = R 1
F
=× h × 1600
hr 8 6 G a 3
 R = l  r  10  6  3cm  8000 (given)
5a
aBryn

 HF = (slant edge) = h = 15 cm
Detailed method 2
CD and CM are tangents at c, 276. (c) area of trapizium
 HG = slant height (l)
 CD = CM 1
 GF = base = × h (AB + CD)
Now in  ADO, 2
 (a) (given)
Les

(h - R)2 = (R)2 + (l - r)2 1


 2 = ×8 × (8 + 14)
wa. th

 h2 + R2 – 2Rh = R2 + l2 + r2 - 2lr  5a  2 2
slant height   2  – a  = 4 × 22 = 88 cm²
 h2 - 2rh = h2 + r2 + r2 – 2lr   = volume of prism
[ l2 = h2 + r2]
25a 2 21a =Height of prism × area of base
r l  r  rh (l  r ) = – a2 =  height × 88 = 1056 (given)
 R 
wwM

4 2
h h2 1056
rh l  r  rh (l  r ) rh  height =
R   88
l 2
r2 l  r l  r  l  r  12 cm
8 6 277. (a) Edge of regular tetra hadron
=  3cm = 3 cm
10  6
272. (a) Volume of tetrahedron  a = 3 cm
AOF is equilateral triangle of side 2a
a³ 12³ 3cm 3cm
= = 3
6 2 6 2  Altitude GO = × 2a
2 3cm
1728
= = 144 2 cm³ = 3 a
6 2

Rakesh Yadav Readers Publication Pvt. Ltd. 421

For More Visit : www.LearnEngineering.in


For More Visit : www.LearnEngineering.in

280. (c)  height of pyramid  15 cm


2
 volume  a³ cm³  base = 8 cm
12
2 9  slant height of pyramid
 × (3)³ = 2 cm³
12 4 2 2
278. (d) l = 15  8  17 cm

 Curved surface area of pyramid


Let side equilateral triangle be
= a 1
 × 64 × 17  544 cm²
3 2 2
r - inradius of incircle of triangle  area = a 282. (c) Volume of pyramid
4

r
Perimeter = 15 cm (given)
3 2 1
a  173 cm2 = × Area of base × height

Sni
15 = 3
 Semiperimeter (S) = cm 4
2

gv.i
173 1
Inradius of any triangle  a²= 3 ×4 = × 57 × 10 = 190 cm³
3
 ( 3  1.73 ) 283. (c) Let the side of square base
r 

ridna
s 173 = a cm
area  a²= 4
1.73  2a² + 4a × h = 608
r =
semiperimeter 173  2a² + 4a × 15 = 608
 4  100

eeYa
where  is the area of triangle =
173  a² + 30a = 304
 r = 3 cm given a² = 400  a² + 38a– 8a – 304 = 0
area of triangle a = 20 cm.
3  15  a(a + 38) – 8 (a + 38) = 0
Perimeter of base= 20 × 3 = 60 cm
2
geisnh
 Volume of prism = 10380  a = – 38,8
15 cm³  a = 8 cm
3× = area of traingle (given)  Volume of prism = 8 × 8 × 15
2
45 area of base × height = 10380 = 960 cm³
Enak

 cm = area of triangle
2 10380 3
 volume of prism  270 cm³ height = = 60 284. (b) Volume of prism= a² × h
173 4
(given)
LSA = 60 × 60 = 3600 cm²
R

45 3
 270 = h × =×(8)² × 10 = 160 3 cm³
2 4
281. (b)
aryn

 h = 12 cm 285. (b) Volume of prism


279. (c) 1
Les B

= 10 12  20  1200 cm ³


2
Perimeter of the base = 4 × 16
 Weight of prism = 1200 × 6
= 64 cm
= 7200 gm = 7.2 kg
Curved or lateral surface area of pyramid
wa. th

286. (a) Total slant surface area


9, 12, 15 is a trip let which 1
= × (perimeter of base) × Slant height 1
forms a right angle triangle 2
= 4  4  a  12
 area of base of prism 2
(where a is the side of the square base)
wwM

1
  9  12 = 54 cm² 12 3
2  a= = cm
8 2
Perimeter of triangle = 9 + 12 +
15 = 36 cm 9
 area of base = cm2
 total surface area of prism 4
= pe rime te r bas e × he ig ht + 12
2 × ar e a of b as e  Required ratio = 9
 height of prism = 5 cm given 4
= 16 : 3
 total surface area
287 (d) Total surface area of tetrahedron
= (36 × 5) + (2 × 54)
 180 + 108 = 288 cm² = 3a 2 = 3  122 = 144 3 cm²

Rakesh Yadav Readers Publication Pvt. Ltd. 422

For More Visit : www.LearnEngineering.in


For More Visit : www.LearnEngineering.in

288. (d) 294. (b)


= h ²  25
M
sl
Total surface area = 270 3 an
16 cm t
o he
1 M 15 cm ig
h
(p erim eter of base ×slant 8 t
2 E o E
16 8 cm
height) + Base area = 270 3 Slant height of pyramid
In radius of triangle
area of triangle 1 3 = 82  152 = 17
= 2

30 3  h ²  25  4 ( 8,15,17) is triplet)
semiperimeter
lateral surface area
10 3 ² = 270 3

ir
 ar ABC  =Inradius × semiperimeter 1
15 3 h ²  25  75 3 = 270 3 = × perimeter of base × slant

v.iSn
28 2
=4× = 4 × 14 = 56 cm h ²  25 = 13 height
2
h² +25 = 169 1
Volume of the prism =366 cm³ = × 64 ×17
h² = 169 – 25 = 144 2

dnag
(area of base)× height = 366 cm³
56 × height = 366 cm h = 144 = 12 cm = 32 × 17 = 544 cm²
295. (a) Perimeter of right 
366 291. (a) = (5 + 12 + 13) = 30
height = = 6.535 cm
total surface area = lateral sur

eYari
56
289. (a) 6 cm 6 cm face area + 2 × area of base
= (perimeter of base × height)+
2 × area of base
6 cm

Clearly the base triangle is the


snhe
Volume of prism = area of base × height

=
3
(6)2× height ATQ,
= (30 × height) + 2 ×
= (30 × height) + 60
1
2
× 5×12

4
kgei
right triangle 30 × height + 60 = 360
 area of triangle ABC 3 30 × height = 360 – 60 = 300
× 6 × 6× height = 81 3 height = 10 cm
1 4
ERna

=  5 12 = 30 cm²
2 81 3  4 296. (d) Height of pyramid = 6 m
Volume of the pyramid height = = 9 cm
1 3 66 Diagonal of square base = 24 2 m
=  base area  × height
3 292. (d) Side of square = 24 m
aBryn

1 Area of square= (24)² = 576 m²


× Base area × height = 330 2 Volume of the pyramid
3 10 slant height
12
1 1
× 30 × height = 330 5
=  area of base × height
3
1 3
330  3
Les

height = = 33 cm Side of square = 10 2 = 10 cm


30 2 1
 576  6
wa. th

=
A
3
290. (d) slant height = 5²  12² = 13 cm = 576 × 2 = 1152 m³
o
10 3
1 297. (a) Volume of pyramid
10 3 M M
lateral surface area = × perimeter 1
2
wwM

slant height
(h) =  area of base × height
E of base × slant height 3
B C o E
10 3 1
1 500=  30  height
Base is equilateral triangle = × 40 × 13 = 260 cm ² 3
2
In radius of equilateral triangle 500  3
293. (d) Total surface area of prism height= = 50 m
side of equilateral  30
= (perimeter of base × height 298. (a) Lateral s urfac e area of
= OE = + 2 × base area)
2 3 prism = 120
3 base perimeter × height = 120
10 3 = (3 × 12 × 10) + 2 ×  12 ²
= = 5 cm 4 L. S. A of p rism = (B as e
2 3 = 360 + 72 3 perimeter × height)
3 × (side) × height = 120
slant length, l = h ²  OE ² 
= 72 5  3 cm²  (perimeter of eq. Δ = 3× side)

Rakesh Yadav Readers Publication Pvt. Ltd. 423

For More Visit : www.LearnEngineering.in


For More Visit : www.LearnEngineering.in

120 302. (a) As we know,  9327723


side × height = = 40 .... (i) Volume of Right Prism = Area
3
of the base × Height  997722
volume of prism = 40 3
 972
3 3 2
area of base × height = 40 3  7200  P  100 3 Volume of Prism
2
= (9  7  2)  9  1134cm 3
3  72×2 = 9P2
side  ²  height = 40 3  P2 = 16 307. (d) Let the side of the square
4
 P=4 = a cm
(side)² × height
303. (b) Half of its lateral edges ATQ
40 3  4 Half of its edges T.S.A = C.S.A + 2 base area

r
= = 160 .. (ii) C.S.A = base perimeter × h
3 Half of its volume
Volume = base area × h

Sni
Dividing (ii) by (i) Then, volume reduced by = 50%
304. (b) Total surface area  T.SA = (base perimeter × h) +

gv.i
side  ² × height 160 (2 base area)
=  3 2 192= 4a × 10 +2a2
side × height 40  4  1 
2a2 + 40a – 192 = 0
side = 4 cm  4 

ridna
a2 + 20a – 96 = 0
299.(c) Volume of te tr ahed ron
= 3 cm
2 a2 + 24a– 4a – 96 = 0
2 2 305. (a) a (a+ 24)–4 (a+24) = 0
= side ³ = (4)³ (a+24) (a–4) = 0

eeYa
12 12
10  a = 4, (–24)
2444 16 2 10 10
 a = 4 (Side can never be in – ve)
 = cm³ 10
Volume = base area × h
12 3
Volume = 16×10
300. (a) Area of the base of prism (a
geisnh
right triangle) Area of base = 10 × 10 = 100 cm2 Volume = 160 cm3
Area of 4 Phase 308. (c) According to the question,
1 V = num be r of v ertic es of
=  5  12 = 30 cm²
2 1 prism = 6
=( ×Base× slant height) × 4
Enak

Third side of the triangle 2 e = edges of prism = 9


= 12²  5² = 13 cm f= faces of the prism = 5
1
 ( × 10 × 13) × 4 ATQ,
Pe rime te r of the triangle 2
R

= 5 + 12 + 13 = 30 cm v e – f 69–5
= 65 × 4 = 260 =
Total surface area 2 2
aryn

= lateral surface + 2 × (base area) [slant height = 122  52  169 = 13]


= (perimeter of base × height) + 10
= =5
Les B

2× (base area) Total Surface area 2


= (30 × 10) + 2 × 30  260 + 100 309. (c) ATQ
= 300 + 60 = 360 cm² Volume of prism = Area of base
 360 m2
301. (c) Let radius be increased × height
306.(d) Volume of prism = (area of
wa. th

= trapezium area × height


by x cm. base × height
Area of base (i.e area of triangle) 1
Volume of cylinder = (10 + 6) × 5 × 8
2
 Area of base
 102 4  x = 16 × 5 × 4 = 320 cm³
310. (a)
wwM

= s s –a
 s – bs – c 
  10  x 4   10 4  x 
2 2

= (By Hero's formula) 2x


 10  x  254  x
2
13  20  21 54
So, S=   27
100  20x  x2  100  25x 2 2 3x
Base of prism
x2 5x 0  27 27 – 1327 – 2027 – 21  length : breadth
 x x  5  0 3x : 2x
 27 14  7  6 Perimeter of base
 x  5 cm = 2 (3x + 2x ) = 10x

Rakesh Yadav Readers Publication Pvt. Ltd. 424

For More Visit : www.LearnEngineering.in


For More Visit : www.LearnEngineering.in

area of base 1 317(d) Length of tank (l) = 4m


 2x  3x = 6x² Area of  DAB = × AB × AD Breadth of tank (b) = 3m
2
height of Prism = 12 cm (given) Depth of tank (d) = h
1
total surface area of prism = × 9 × 12 = 54 cm²  Volume = lbh
2
= (Perimeter of base × height) + 24 = 4 × 3 × h
In  BDC
(2 × area of base)  h = 2m
14  13  15 (As tank is open from upper
288 = 10 x × 12 +12 x² Semi perimeter =
12 x² +120x – 288 = 0 2 sides remaining five sides of the
x² + 10x – 24 = 0 = 21 cm tank will be painted)
x=2 Area of  Area of 5 sides = 2(hb + lh) + lb
 area of base  6 × 4 = 2(2 × 3 + 2 × 4) + 4 × 3
= 21   21 – 14    21 – 13    21 – 15 

ir
= 28 + 12 = 40m2
 24 cm²

v.iSn
= 21  7  8  6 = 84 cm  Total area to be painted inner
 volume of prism  24 × 12 and outer side.
Volume of prism = Base area ×
 288 cm³ Height 40 m2 + 40 m2 = 80 m2
311. (b) Volume of the part (prism) = 2070 = (54 + 84) × Height = 80m2 cost of painting = 80 × 10

dnag
Area of base × height = 800 Rs.
Area of base (Isoscales  ) 2070 318. (b) V = abc
Height = = 15 cm
138  S = 2 (ab + bc + ca)
b
= 4a ² – b ² Lateral surface area = Perimeter
1 1 1 

eYari
4 of base × Height
 S = 2abc  c  a  b 
6 = (9 + 14 + 13 + 12) × 15
= 4  5  ² –  6  ² = 12 cm² = 48 × 15 = 720 cm²
4 1 1 1 
1  S = 2V  c  a  b 
Volume of prism = 12 × 8
= 96 cm³
312.c)
A D
snhe
314.(c) Volume of pyramid = ×Area
of base × Heights
3
1 2 1 1 1 
 V = S  c  a  b 
1
kgei
3
= × × 4 × 4 × 20 3 319. (b) External length (L) = 1.46 =
3 4
146 cm
F = 80cm³
External breadth (B) = 1.16 m
ERna

315.(c) Slant height of pyramid


= 116 cm
B E C
2 External Height (H) = 8.3 dm
6
a  42    = 5 cm = 83 cm
In circle radius (EF) = 2  Internal length (l) = 146 – 6
2 3
aBryn

Total surface area = Lateral Sur- = 140 cm


18 3 face area + Base area  Internal breadth (b) = 116 –
= = 9 cm 6 = 110 cm
2 3 1
= × Perimeter of base × Slant  Internal Height (h) = 83 – 3
2 = 80 cm
Slant height (DE) = 12 2  92
Les

height + Base area Total surface area of inner walls


wa. th

= 15 cm 1  = 2h (l + b) + lb
Total surface area   24  5 
=   + (6 × 6) = 2 × 80 (140 + 110) + 140 × 110
2  = 40,000 + 1540
1  3 = 60 + 36 = 96 cm²
= 3 ×  18 3 15  +
2  4
18 3  ² 2
= 55400 cm2
Cost of painting 100 cm2 = 50
10 
132 – 
wwM

316.(b) slant height =  paise = 0.5 Rs.


= 405 3 + 243 3 = 648 3cm2  2 
cost of painting 55400 cm2 = 554
A 9 B Height of pyramid
313.(a) × 0.5 Rs = Rs. 277 Rs.
= 12 2 – 52 = 119 cm 320. (a)
12 14 1
Volume of pyramid = × Area
3 x b y
D 13 C of base × Height z h
In  DAB  l
BD2 = AD2 +AB2 = × 10×10 × 119 Volume = l × b × h
3
BD2 = (12)2 + 92 x = hb
100
BD = 15cm = 119 cm³ y=lh
3

Rakesh Yadav Readers Publication Pvt. Ltd. 425

For More Visit : www.LearnEngineering.in


For More Visit : www.LearnEngineering.in

z=lb x 2  y2  z 2 2 2
 6  1.5 
xyz = (l b h)²  l2 + b2 + h2 = 2
...(iv)  π   2  = n × π ×  2 
V² = xyz
subtracting (i) from (iv) we get, (n = no. of pipes)
321. (a)Volume of bar = Volume of
piece of copper y2  z2 – x 2  n = 16
area of cross section of square × h2  328. (b) Volume of cylindrical shell
2
length = 1m3 1
a2 × 36 = 1 y2  z 2 – x 2 = × volume of solid cylinder
4
 h
1 2
 a m 1
6 Subtracting (ii) from (iv) we get,  (R2 – r2) × 1000 =  R2 ×1000
4
then volume of cube

r
x 2  z2 – y 2 x 2  z2 – y 2 R2
l2  l 
1 1 1 1 2 2 = R2 – r 2 =

Sni
= × × = m3 4
6 6 6 216 subtracting (iii) from (iv) we get
R2

gv.i
Cost of cube = Volume × cost of 1m3 R2 – = r2
2 x 2  y 2 – z2 4
1 b 
= ×108 = Rs. 50 paise 2 3R2 = 4r2
216

ridna
322. (d) Let the length, breadth and x 2  y2 – z2 3 102
 b r2 =
height of block be 3x, 2x and x 2 4
 Volume of block = lbh r2 = 75
 Volume = lbh

eeYa
3x × 2x × x = 10368 r= 5 3
x = 12 dm

x 2
 y 2 – z 2  y 2  z 2 – x 2  x 2  z 2 – y 2  thickness of cylinder = radius of
Now, length breadth and height 8 solid cylinder – Inner radius of
of block will be 36 dm, 24 dm
cylinderical hole = 10 – 5 3
and 12 dm respectively. x  y 2 – z 2  y 2  z 2 – x 2 x 2  z 2 – y 2 
2
geisnh
Total surface area of block 
 =5 × 2– 3
= 2 (lb + bh + hl)
2 2   cm
326. (b) The string of minimum 329. (b) Let the radius of well = r
= 3168 dm2
length, if starting from C, Volume of embankment (hollow
 cost of polishing entire sur- touches next corner at height n/
Enak

face = 3168 × 0.02 = 63.36 Rs. cylinder)


4 on the completion of one turn, = volume of earth taken out
323.(a) Volume of water drawn out =
starting from height n/4 touches
(Volume of water initially in
next corner at height n/2 in the  2

 π r  1 – r  5  πr  20
2 2
R

tank) – (Volum e of water


second turn, and so on.
remained in tank)
 r  1  r r  1 – r   r  4
2
= 15 × 6 × 10 – 15 × 6 × 9 m3 D
aryn

= 90,000 litre (1m3 = 1000 litre)  (2r + 1) × 1 = 4r2


324. (c) Volume of water will come  4r2 – 2r – 1 = 0
Les B

x
out of pipe in 15 min 2  4  16 5 1
n/4
= Cross section area × Length C r  = m
n 8 4
per 15 min 330. (d)
2
wa. th

13 9000 n  17  n
 × ×15 = 2.925m3. CX  n 2    =
10000 60  4 4
 Volume of water filled in tank in  Le ngth of string
15 min
17  n
wwM

= Volume of water came out of  4 = 17n .


pipe in 15 min 4
 45 × 26 × h = 2.925 Alternate:-
( h = level of water rise in tank ) Opening up the four vertical
sides of the cubes of side n,
 h = 0.0025m
325. (b) Let the length, breadth and Length of string = 4n 2  n 2  Let total volume of cylinder
height of cuboid be l, b and h = 5x litre
 x2 = l2 + b2 .......(i) = 17  n 4
y2 = b2 + h2 .......(ii) 327. (c) Volume of supplied water by Volume of water = ×5x = 4x litre
5
z2 + l2 + h2 ........(iii) diameter of 6 cm = n × volume After tilting, there will be water
adding (i), (ii), (iii) we get, of water supplied by diameter 5x
x2 + y2 + z2 = 2(l2+b2+h2) 1.5 cm half of total volume 4x –
2

Rakesh Yadav Readers Publication Pvt. Ltd. 426

For More Visit : www.LearnEngineering.in


For More Visit : www.LearnEngineering.in

3x Slant height (l) = radius of circle 120º


 30  = 4 cm  × 2π (1 5) = 2πr
2 360º
r = radius of cone
 x = 20 litre Per imeter of base of cone (where, r is radius. of cone

Hence, volume of cylinder  r = 5cm
= 2rπ
= 5x = 100 litre.
2 2
331. (b) Curved surface area 2πrh 90º h= l2 – r2 = 15 – 5
Curved surface area of 50 pil-  2rπ = × (perimeter of circle)
360º
= 200 = 10 2
lars = 50 × 2πrh 1
22 50  2rπ = × 2π × 4 1 2
4  Volume = πr h
= 50 × 2× × × 4 = 314m2  r = 1 cm 3
7 2  100

ir
Labout charge for getting these height of cone (h) = l 2 – r 2 1

v.iSn
pillars cleared = 314 × 0.5 = 16 – 1 = 15 ×π × 5 × 5 × 10 2
3
= Rs. 157 1 2
3  Volume of cone = πr h = [(250 2 )π /3] cm3
3
332. (c) r  h ........(given)
4 1 22 1 2

dnag
= × ×(1)2× 15 339. (a) Volume of cone = πr h
4r 3 7 3
 h π 5
3  1 22
cm3
3 = × × 5 × 5 × 25 = 644.761
 Volume = πr 2h = 38808 ml

eYari
3 7
= 38.808 l θ
336. (b) Arc of Circle = π × r Volume of smaller cone
= 38.808 × 1000 cm3 180
= 654.76 – 110 = 544.761
22  4r  120 2
 7
× r2 ×   = 38808

 r3 = 9261
 r = 21 cm
3
=
180 snhe
 π ×3 =
3
 π ×3 = 2 π

Base of cone = Arc of Circle


2πr = 2π
Volume of smaller cone
 Volume of larger cone

 radius of smaller cone 


kgei
333. (a) r = 1 cm =  radius of large cone 
50 cm  h² = l ² – b²
50 cm
10 cm = (3)² – (1)² = 8 3
544.761  r 
ERna

 = 
2πr = 100 cm h = 2 2 cm 654.761  5 
Then, curved surface area 1  r = (104)1/3 cm
Volume of Cone = π r 2h
= 2πrh 3 340. (a) Circum fere nce of
aBryn

100 × 50 = 5000 cm2 1 hesmispherical bowl = 176


= ×π×1 h
Cost of painting 3  2πr = 176
50 1 2 2π  r = 28 cm
= 5000× = ×π×1 2 2 = Whe n bowl is half f ull
100  100 3 3
Les

= Rs. 25.00 337. (b) Slant height = H, and radius 2 1 1 3


= 3× = π28 
=r 3 πr
wa. th

334. (c) D C 2 3
curved surface area πrl Volume of hemispherical glass
 area of base
= 2
πr 2 3
= π2
rH 3
H
wwM

= = =H:r No. of persons may be served =


r 2 r
A B 338.(c) Radius of cone = Slant height 1 3
Diameter of each circle = 2cm π 28
of Cone = 15 cm. 3 = 1372
 AB = 2 × 4 = 8 2 3
π 2
Similarly, CD = 8 3
Area of Bottom = AB × CD = 64 cm2
341. (c) Volume of cylindrical vessel
335. (c) = Volume of sphere
1200 l = R
4
π ×(6)2 × h = ×π × (3)3
0 l 3
90  Perimeter of base of sector = pe-
rimeter of base of cone  Surface of water rise (h) = 1 cm

Rakesh Yadav Readers Publication Pvt. Ltd. 427

For More Visit : www.LearnEngineering.in


For More Visit : www.LearnEngineering.in

342. (d)Surface area of B = surface  Area of ABCDE (base) = (18  Volume of prism = Area of base
area of A + 300% of surface of A + 9) = 27 cm2 × height
= 4 (Surface of area of A) Volume of prism = Area of the = 60 × 20 = 1200 cm3
Let the radius of A be a and ra- base × Height Material used for 1 cubic cm.
dius of B be b. = (27 × 10) = 6 gm
 4π b2 = 4 × 4π a2 = 270 cm3 Material used for 1200 cm3
 b = 2a 345. (b) Let the height of each prism = 1200 × 6 = 7200 gm = 7.2 kg
4 3 be h units and the length of each 349. (d) Perimeter of triangle = 15 cm
Volume of A = πa side of equilateral triangle at the
3 15
base of first prism be a units and semiperimeters = cm
2
4 3 4 3 that the second prism having
Volume of B = πb = π 2a  inradius r = 3 cm

r
3 3 regular hexagon as base be b
units. 15 45

Sni
4 (See the figures given below) Δ = r.s = 2 ×3 = 2 cm
= π8a 3

gv.i
3
Volume of prism = area of base
% of volume of A lower than B a
× height
= K%
45

ridna
b
4 4 270  h
 π8a 3 – πa 3 2
II
3 3
4
I h = 12 cm
3
π8a According to the question,

eeYa
3 3 2
Volume of first prism = Volume a1 .h1 2
7 of second prism V1 = 4  a1  h1
  100 = 87.5% 350. (d) V 3 2 2 =  a  . h

8 3 2 3 2 2 a1 .h 2 2
a h  6 b h 4
 k = 87.5 4
geisnh4
343. (b)Given prism is a solid with 2

regular hexagonal base  2 3 2  a   h  4


a  b  a2 = 6b2   1  . 1  
Its volume = Area of the base ×   3a 2   h 2 / 4  9
 4 2
Height
Enak

a  6b 
a 6  a2  3a1 & 4h1  h 2 
3 3 3 3 =
 ×1=  cu m b 1 351. (c) Volume of a pyramid
2 2
R

 a:b= 6:1 1
Since the area of regular  = × Area of the base × Height
 
3
1
 hexagon withsidel m= 3 3 346. (c) Area of trapezium = ×
m2 
aryn

 2  2 1 3 3
height × (sum of parallel sides) = × × (10)2× 30 = 2598m3
3 2
Les B

344. (a)
1 [ Area of the regular hexagon
= × 8 (8 + 14) = 88 cm2
2 3 3 2
A  Volume = area of traperium of side a  a sq units ]
2
wa. th

0
90 3 × height 1056 = 88 × h 352. (c)
B P

3   3
O
3
E h = 12 cm
347. (c) The base of the prism is rect-
E h D
C D angular and we are not chang-
a a 3
ing the base so length & breadth
wwM

AB = 3 2
will be remain same. If we F
O
C
2a
AE = 3 2 double the lateral edges it means
we are doing double its height A B
 ABE = 45° so Volume of the prism will be AB = BC = CD = EF = FA = 2a
 AEB = 45° doubled.
5a
Area of  BCDE = 18cm2 PE= and OE = 2a
1 2
348. (b) Area of base = × base ×
1 2 2
Area of Δ OAB = ×3×3 height  5a  2 3a
2  h = OP =   – 4a =
2  2
= 4.5 cm2 1
= × 10 × 12 = 60 cm2 1
 Area of Δ ABE = 9 cm 2 2 Volume of the pyramid = ×
3

Rakesh Yadav Readers Publication Pvt. Ltd. 428

For More Visit : www.LearnEngineering.in


For More Visit : www.LearnEngineering.in

Area of the base × Height vol. passed by pipe = vol of cone


1 1 3a 1
= × 6 × × 2az × a 3  π r2 ×1000 ×T = π ×15×15 × 24
3 2 2 3
[ speed of water = 1000 cm/ minute] 50
= 3a 3 3 43
353. (c) Area of regular hexagon of 28800 7
T= minute = 28 minute 48 sec 7
3 3 2 1000
side a  a
2 357. (b) C = 2π rh + 2π r2
3 3 2 = 2π r (h + r) = 2π r (43 + 7)
 a = 96 3  a = 8 m
2 O 22
Let h be the height of the pyra- A B = 2 × 7 × 50 = 2200 m2

ir
14 cm 7
mid. Then area of one side face

v.iSn
361. (b) Diameter of ball = 14 cm
r1
1 Radius = 7 cm
a  l , where l

m
of the pyramid =
2

c
h 7

14
is the slant height of the face. Height of solid cylinder = cm
3

dnag
l=
1 A.T.Q.
 a  l = 32 3  l = 8 3
2 Length (ACB) of semi-circular Volume of ball = Volume of
3a 2 sheet =  r cylinder
 h2  l2

eYari
4 4
22  π r3 = π r2h
3  64  14  44 cm. 3
  h 2  64  3 7
4
Slant height of the cone  14cm. 4 7
 1  ×π × 7 × 7 × 7 = π × r2×
 h2 = 64 × 3 1 – 4  = 144


 h = 12 m

Volume of the pyramid


 2πr1 
snhe
Circumference of the base of thecone
44
r1
3
2
 r = 49 × 4
 r = 7 × 2 = 14 cm
3

Diameter of the base of the cylinder


7
kgei
1
= × Area of the base × h 44  D = 2r = 2 × 14 = 28 cm
3 44  r1  r1  7cm. 362. (d) Inner radius r = 8 cm
1 7
  96 3  12  384 3m
3
Outer Radius R = 8 cm
ERna

3  h  142  72 7 3 cm. Area of Circular shell


354. (b) Volume of tunnel = πr 2h  71.732 12 cm. = π (R2 –r2) = π (144–64) = 80π
358. (d) Total surface area of prism A.T.Q.
22
= ×2 ×2 × 56 = 704m3 = Surface area + (base + top) area Total surface area of cylinder
aBryn

7 = area of shell
608 = Perimeter of base × height
Volume of the ditch = 48 × 16.5  2π R1 (h + R1) = 80π
+ 2 × base area
× 4 = 3168 m3
608 = 4 × a × 15 + 2 × a × a 40
704 2 608 = 60 a + 2a2  h + R1 =
Part of the ditch filled 
 R1
3168 9 a2 + 30 a = 304
Les

355. (d) Height of cylindrical rod a(a + 30) = 304 40


 h= R – R1
wa. th

(h) = 8r a = 8 cm 1

r Volume of prism = Base area × 40 – R12


Radius of spherical ball = Height  h R1
2
= 8 × 8 × 15 = 960 cm3 363. (c)
Num ber of spheric al balls
wwM

2
359. (b) A.T.Q.
πr h
 l × b × h = πr 2h1 +
4 r 
3
h h =
π 
3 2 22
2
8 × 11 × 2 = × 4 × 4 × h1 r1 r2
π  r  8r 7
= 3 1 1 4
4 r  7 π r12 h + π r22 h = π R3
π    h1  3 3 3
3 2 2
= 16 × 3 = 48 1 4
h1 = 3.5 π h (r12 + r22) = π R3
1 3 3
356. (a) Radius of pipe (r) = cm 360. (d) C.S.A. of storage tank =
4 C.S.A. of cylinder + C.S.A. of 4R 3
h 
Radius of cone (R) = 15 cm hemisphere r  r22
1
2

Rakesh Yadav Readers Publication Pvt. Ltd. 429

For More Visit : www.LearnEngineering.in


For More Visit : www.LearnEngineering.in

364. (d) height of cone 368.(d) A 4


10.2 – 4.2 = 6 = (12³ – 10³) = 3047.89 cm³
3
Volume of woods
Weight = volume × density
a a
1 2 2 3 πr 2  4.9 × 3047.89
= πr h + πr = (h+2r)  14942.28 gm
3 3 3
375. (b) The volume in both the
πr 2 B a/2 C a/2 D cases will be equal. Let the
 (6 + 8.4) height of cone be = h
3 Let one side of  be = a
2

22 4.2  4.2 Perimeter of equilateral triangle = 3a 4 22 22  35  h


  ×14.4 × × (14)³ = ×   ×
7 3 3a = 72 3 = a = 24 3 cm 3 7 7 2  3

r
 22 × 0.6 × 1.4 × 14.4 Height = AC; by Pythagoras theorem  35 
2

Sni
 266.112  266 (nearly)  4(14)³ = h   =h
a 
2
2 
365. (b) h1 = h2 AC² = a² –  

gv.i
2 4  14  14  14  2  2
πr12h1 3 =
 AC = 36 cm 35  35
1 2 1
πr2 h 2 369. (d) Volume of soil removed = h = 35.84 cm

ridna
3
=l×b×h 376. (c) C ir cumf er ence of the
r12 = r22 = 7.5 ×6×1.5 = 67.5 m³ circular face of the cylinder =
r1 = r2 370. (c) I hectare = 10000 m² 2r
D1 = D2

eeYa
1 22 35
(b) Diameter of cylindar = diam- Height = 10 cm = m 2 × × =2.2 m
10 7 100
eter of cone
18cm Number of revolutions required
366. (b) 1
Volume = 10000× =1000 m³ 11
geisnh 10 to lift the bucket by = =5
2.2
12cm

371. (a) Total surface are of 7 cubes


377. (c) Surface area of the cube 6a²
22cm  7×6a² = 1050
= 6×(20)² = 2400
10cm But on joining end to end, 12
Area of 6 circles of radius 10 cm
sides will be covered.
Enak

= 6 r²
So there area = 12×a²
8cm = 6 ×  × 100 =1885.71
R=9  12×25 = 300
Remaining area = 2400 – 1884
So the surface area of the
R

r=4 = 514.28
resulting figure = 1050 – 300 =
2 2
750 378. (d) x . y. z = l b × bh × lh
l 12  5 
= (lbh)²
aryn

372. (d)Let the rise in height be = h


l = 13 (V) Volume of a cuboid = lbh
Then, as per the question, the
Area of funnel So V² = (lbh)² = xyz
Les B

volume of water should be equal


= 2π rh × π (R + r)l 379. (a) Volume of mud dug out
in both the cases.
= π [2 × 4 × 10 + (9 + 4) × 13] = 10×4.5 × 3 = 135 m³
Now, 90×40×h = 150×8
= 249π Let the remaining ground rise
150  8 1 100 by = h m
wa. th

22 h= = m= cm
= 249 × = 782.57 cm3 90  40 3 3 Then {(20×9) – (10×4.5)} h = 135
7
= 33.33 cm 135h = 135  h = 1 m
367. (a) 373. (d) Area of base = 6×10 = 60m² 380. (b) Height of the cylinder
Volume of tent = 30×10=300m³ = 13 – 7 = 6 cm
wwM

h2 Let the radius be = r, height = Radius of the cylinder and the


h h, slant height= l hemisphere = 7 cm
a 60 Volume of the vessel = volume
r² = 60  r = of c ylinde r + volume of
P 
hemisphere
1 r2 h 60
π r2h2 = π r2h 300 =  900 =  . . 4r 3
3 3   r²h +
32
1 h  h = 15 m
πp 2h1  πa 2h 374. (c) Volume of metal used 4  3.14  (7)²
3  3.14×(7)²×6+
4R3
4r 3
32
a 2h = –
h1  3 3  1642.6 cm²
3P 2

Rakesh Yadav Readers Publication Pvt. Ltd. 430

For More Visit : www.LearnEngineering.in


For More Visit : www.LearnEngineering.in

381. (c) 391. (a) Volume of ice cream = r²h


864
 = 32 (25 – r²) = 3.14 × (6)² × 15
3
= 1695.6 cm²
 r² = 16 = r = 4 cm
r 2  24
So thickness of the cylinder Volume of 1 cone =
3
= 5 – 4 = 1 cm
Then  10×3.14 × r² × 8 =
105 386. (d) The volume in both the
Radius = =52.5 cm 1695.6
2 cases would be the same.
 r = 3cm (approx.)
Area of the entire canvas, used for the Let the height of the cone = h
Then, external radius = 6cm So diameter = 2 × 3 = 6 cm
tent
Internal radius = 4cm 392. (b) Rad ius of cylinder and
= Area of cylinder = area of cone

ir
7
= 2 rh+ rl 4. 63  43  .42.h hemispheres = =3.5cm

v.iSn
 = 2
=  r (2h + l) = 3. 14 × 52 .5 3 3 Height of cylinder = 19 – (3.5 × 2)
= 12 cm
(2 53  52.5 +53)
2 2
63  43 Total surface area of solid=
= 5 × l (because area of canvas = l
 h=
4 2 rh + 4 r²

dnag
× b also)  2 × 3.14 × 3.5 × 12 + 4 ×
216  64
= l  1947 m  h= = 38 cm 3.14 × (3.5)²
4
382. (c) The volume in both the  418 cm²
387. (a) Let arc side of the cube be =

eYari
cases would be the same. 393 (c) As they stand on the same
a units
4r3 base so their radius is also
Total surface area of 3 cubes
Therefore = =r²h same.
3 = 3 × 6a² = 18 a²
Total surface area of cuboid r2 h
4  3.14  4  10
3
64000
3

=3.14 × 2²×h

Ratio =
snhe
= 18a² – 4a² = 14a²
14a 2
7:9
Then; volume of cone =

Volume of hemisphere =
3
2r 2
3
18a 2
kgei
 h= = 21333.33 mm Volume of cylinder = r²h
3 388. (c) A = 2 (xy + yz + zx)
r 2h 2r 3
383. (b) As the cylinder and cone V = xyz Ratio = : :r²h
have equal diameters. So they 3 3
ERna

have equal area. Let cone's 2  xy  yz  zx  h 2r


A/V =  : : h  h : 2r : 3h
h ei gh t be h 2 and as pe r xyz 3 3
question, cylinder's height be Radius of a hemisphere = Its
h1. 2 2 2 1 1 1 height
+ + y  2 x  y  z
aBryn

= So h : 2h : 3h  1 : 2 : 3
2rh1 z x  
8
394. (d) Total surface to be painted
r h  r = 5
2
2
2 389. (d) Radius of cy lind er ,
hemisphere and cone = 5cm = exte rnal s ur face are a +
On solving we get the desired Height of cone = 12 cm internal surface + surface area
ratio as 3 : 4 of right area
Les

Surface area of toy = 2rh +


384. (a) Let the slant height of 1st = 2 (R² + r²) + 2 (R² – r²)
wa. th

cone = L 4r2 Cost of painting


+  rL
Then the slant height of 2nd cone 2
 25  2  24  2  25  2
= 3L
L= h2  r 2 = 122  52 =13  2× 3.14 ×   2    2    2 
Let the radius of 1st cone = r1 
wwM

And let the radius of 2nd cone = Then  (2 ×3 .1 4×5×13 ) + 2


 24  
r2  
(2×3.14×25) + (3.14×5×13)  2   × 0.05
Then, r1 L = 3 × r2 × 3L  770 cm²
 2 × 6.28 × (12.5)² × 0.05 × ` 96.28
 r1 L = 9r2 L  r1 = 9r2 390. (b)Height of cone = 10.2 – 4.2
395. (b)
Ratio of area of the base 81 : 1 = 6cm
385. (c) Let the internal radius of the r2h 4r 3
Volume of wood = + 3.5
cylinder = r
Then, the volume of sphere =
2
3 32
3
h
. 7
Volume of sphere cylinder 3.14  4.2  6 4  3.14  4.2
 +
4.63 3 32
 =  h (5² – r²)
3 3.5
 266 cm²

Rakesh Yadav Readers Publication Pvt. Ltd. 431

For More Visit : www.LearnEngineering.in


For More Visit : www.LearnEngineering.in

Time taken to fill the tank 407. (b) Volume of cube = a3 = 253
4
 (3.5)²×7–  (3.5)³ 10048 = 15625 cm3
3 = =51.2 mins Surface = 4a2
196.25
=  (3.5)²×h = 4 × 25 × 25
400. (d) One side of the equilateral
4 35 triangle = diameter of cone. = 2500 cm2
7– × =h 408. (c) Volume of the rectangular
3 10
12 solid
14 7 Therefore radius of cone = =6
7– =h= 2 = 36 × 75 × 80 = 21600 cu cm
3 3 Height of cone = Height of \ Edge of the cube = 3 216000
3.5 equilateral triangle be = 60 cm
396. (d) Radius = =1.75 cm
2 3a 409. (a) Volume of the cube = 5 × 5
 Height of cone =6 3

r
r2 h 2 × 5 = 125 cu cm
Volume of solid =  r²h +

Sni
3 r 2 h 0.625 kg  125 cu cm
3 Volume of cone =
2r 3 125

gv.i
+ \ 40 kg  × 40
3 2
 6  6 3 0.625
6  = 72 3 cm³
3 = 8000 cu cm
 3.14 × (1.75)² × (10 + +
3 401. (b) Let the radius of iron ball = r1

ridna
Let the radius of ball = r0 \ edge = 8000 = 20 cm.
2(1.75)
)  121 cm³ Then, as iron weights 8 times 410. (a) r + h = 37 and 2pr (r + h) =
3
oak 1628
397. (d )Volum e of the e llip tic al

eeYa
cylinder 4r03 8  4r13 r0 1628
 = = r =2  r0 = 2r1 or, pr = = 22
2.4 1.6 3 3 1 74
= × × ×7 1 \ r = 7 cm and h = 37 – 7 = 30 cm
2 2 So d iame te r of iron=
2
= 3.14 × 1.2 × 0.8 × 7  9 m³
geisnh 22
diameter of oak \ Volume = π r2h = ×7×7
Amount of water emptied per 7
1 × 30 = 4620 cu cm.
 2 
2
 ×18 = 9 cm
2
minute = 120 × 3.14 ×   1
100  402. (c) Volume of the timber = 7 ×
Enak

411. (c)Radius = h and height = h


0.1 × 0.1 = 0.07 cu m 2
Time required to empty half the
\ Weight of the timber = 0.07 × \ Whole surface
4.5 950 = 66.5 kg
R

2
tank = 120  3.14  (0.02)² 1  1 
403. (a) Volume of masonary =
 h
= 2p ×   × h + 2p × 2 h 
 
Length × Breadth × Height 2   
=70 min = 81 × 0.2 × 4 = 64.8 m3
aryn

398. (d) Let initial radius = r 3 2


9 = ph
2
Les B

Then volume = r² h 404. (a) Speed of the river = km/hr


2 412. (d) Volume of cube
r 3r = (22 × 22 × 22) cm3
New radius = r + r  9 5 5
2 2 = × = m/sec. Volume of 1 bullet
2 18 4
9 \ required answer
wa. th

New volume =  r²h  4 22 


4 = 
  111
 cm3
5 3 7 
5 = 10 × 200 × = 2500 cub m.
Increased volume = r²h 4 \ Number of bullets
4
5r 2h 200  22  22  22  3  7 
wwM

Percent increase = ×100 405. (b) 1 sq m × depth =   = 2541


4r2h 1000 =  
 4  22 
= 125%
1 1
399. (c) Volume of the cone \ depth = m= × 100 = 20 cm. 413. (c) Let h and H be the heights
5 5
2
r h 3.14  20  20  24 of water level before and after
=  406. (c) Area of field = 500 × 30
3 3 = 15000 m2 dropping the sphere into it.
 10048 cm³ Area of field after constrauction Then, [p × (30)2 × H] – [ π × (30)2
Diameter of pipe = 5m of tank = 15000 – (50 × 20) 4
= 1400 m2 × h] = p × (30)3
Volume of water flowing out of 3
the pipe per minute Volume of tank = Area of field
after constraction of tank × h 4
 10 × (2.5)² × 3.14  196.25 or p × 900 × (H – h) = p × 27000
50 × 20 × 14 = 14000 × h 3
cm³
 h = 1m or, (H – h) = 40 cm.

Rakesh Yadav Readers Publication Pvt. Ltd. 432

For More Visit : www.LearnEngineering.in


For More Visit : www.LearnEngineering.in

414.(b) Let outer radii = R1 and inner 419. (c) Capacity = Volume = (24 × = 25344p
radii = R2 12 × 10) = 2880 cu cm 425. (b) Volume of Hollow cylindrical
 2  R1h - 2  R2h = 44 Area of tin foil neede = Total Tube
[Where, h = height of pipe] surface area = 2( lb + bh + hl ) = p(R2 – r2)h
= 2 (288 +120 +240)
2 2
 2
22
 14[ R 1  R 2 ]  44 = 1296 cm² 22   37  –  33  
7 =      × 70
420. (d) Volume of metal 7  2   2  
1 = (52 × 40 × 29 – 48 × 36 × 27) =
 R1  R2   0.5    (i ) 13664 cm3
2  37 33   37 33 
\ Weight of metal =22 × 10 ×  2  2  ×  2 – 2 
   
and  ( R12  R2 2 )  h  99 (given) = 220 × 35 × 2
 1 

ir
13664  0.5 
=  
 = 15400 cm3
22  1000 
R1 R2R1 R2 14  99

v.iSn
 426. (a) Volume of collected Rain =
7 = 6.832 kg. Volume of swimming pool
421. (c) Volume of Sheet
 4  0.5 R1  R2   9 25  1 
1   2  1000  1000 
R1 + R2 = 4.5 ----------(ii) 100 100 

dnag
= cu m
On adding (i) and (ii) :- 2 = 50 ×5×h
2R1= 5  R1 = 2.5cm 1  25  2  1000  1000
415. (c) Required answer  100 100 100 
=   cu cm h=
2  100  100  50  5

eYari
22  22  22  3  7 Area of sheet = 1 hectare h = 20 m
= = 2541
4  22 111 = 10000 sqr. metres 427. (a) Volume of collected Rain =
416. (b) Volume of 1 cylinder = pr2h = (10000 × 100 × 100) sq cm. Volume of swimming pool
40  2 
Volume of 1 cone =
1 2
3
pr h Thickness =snheVolume
Area
1100 100 100 1

100  100
 1  1000  1000 
= 200 × 20 × h

r 2h 40  2  1000  1000
kgei
Number of cones = 1 =3 = = h=
2 10000 100 100 200 100  100  200  20
r 2h
3 = 0.005 cm h=2m
422. (b) Volume of sheet = 2 cm3 = 2 428. (c) Volume of the tank = 3 m3
ERna

417. (a) (i) Capacity = (135 – 5) (108


× 100 × 100 × 100 cm3 \ Base area × height = 3 m3
– 5) (90 – 2.5) [Since cistern is
open] = 1171625 Area of sheet = 12 hectare 3
(ii) Volume of iron = [(135 × 108 = 12 × 10000  Base area =
2.56
× 90) – (1171625)] = 120000 m2 = 120000 × 100 ×
aBryn

= 1.171875 m2
= 140575 cu cm 100 cm2
[\ Volume of cuboid = (l × b) × h
418. (a) External radius of the pipe Volume = (base area) × height]
= 1.2 cm Thickness = 429. (b) Total length of tape
Area
Internal radius of the pipe = 2(l + b) + 3.75
2  100  100  100
Les

= (1.2 – 0.2) = 1 cm = 2(39.5 + 9.35) + 3.75


=
External volume 120000  100  100 = 101.45 cm
wa. th

430. (a) Are a of sur face to be


 22  1 cemented = 2 × (l + b) × h + (l × b)
=  1.2 1.2  3.5 100  = = 0.0017 cm
7  600 i.e, area of four walls + area of
= 1584 cu cm floor
0.01 0.01
wwM

Internal voulme 423. (b) 3.1416 × × ×h=1 = 2 × (21) × 4 + (106.25)


2 2 = 274.25 m2
 22 
4 \ Cost of cementing = 24 ×
 11 3.5 100 
=   \ h= 274.25 = Rs. 6582
7  3.1416  0.01 0.01
431. (b) Total v olum e of water
= 1100 cu cm = 12732.365 cm = 127.3 m displaced by 250 men
Volume of lead = (External 424. (a) Volume of hollow cylindrical = 250 × 4 = 1000 m3
Volume) – (Internal Volume) tube = p(R2 – r2)h \ Rise in water level (h)
= (1584 – 1100) = 484 cu cm
22 Volume
Weight of the pipe = ((24)2 – (20)2) × 144 = Base area
7
 484 11.4  22
= 
 
 = 5.5176 kg. = × 44 × 4 × 144 1000
 1000  7 = = 25 cm
80  50

Rakesh Yadav Readers Publication Pvt. Ltd. 433

For More Visit : www.LearnEngineering.in


For More Visit : www.LearnEngineering.in

432. (c)Let each edge of smaller cube \ Total volume occupied by


7–5
=1m these cubes = (7)3 × 17 × 11 × 7 = = 1 cm
2
\ Each edge of larger cube = 2m = 448987 cm3
and surface area of smaller cube Total volume of the tank = 120 441. (d) Base area of vessel × rise in
= 6 × (1)2 = 6m2 × 80 × 50 = 480000 cm3 water level = Volume of cube
\ Surface area of larger cube = \ Area of unoccuppied space = 15 × 12 × h = 11 × 11 × 11
6 × (2)2 = 24 m2 480000 – 448987  h = 7.39 cm
\ % increase in surface area = = 31013 cm3 = 31.013 dm3 442. (b) (Initial volume of water +
435. (b) Surface area of the cuboid = required volume of water +
24 – 6 volume of cube)
× 100 = 300% 2(l b + bh + h l) = 11.6 m3
6 \ Cost of canvas = 11.6 × 25 = Base area of vessel × 10
Alternatively: = Rs. 290 \ 25 × 20 × 5 + required volume

r
2 436. (c) Required surface area of water + 1000 = 25 × 20 × 10

Sni
S2 e  S2 4  Required volume of water =
2
=    = = 2(9 × 3 + 3 × 3 + 3 × 9) = 126 cm2
S1 e S1 1 437. (a) Area of 4 walls 1500 cm3 = 1.5 litre

gv.i
 1
443. (c) Net volume = (10 × 8 × 2) –
\ percnetage increase in surface = 2(36 + 12) × 10 = 960 m2
(2 × 2 × 2) = 152 cm3
Total area of (windows + door +
4 –1 Net surface area = 2(10 × 8 + 8 ×
chimney) = 120 m2

ridna
area = ×100= 300% 2 + 2 × 10) + 4 (2 × 2) – 2(2 × 2)
1 \ Net area for papering = 960 –
= 240 cm2
where S = surface area, e = edge 120 = 840 m2
444. (b) Net volume of the wall
of cube. \ Length of required paper =
= Total volume – Volume taken

eeYa
433. (a) External volume of the bos 840 away due to doors
= 24 × 16 × 10 = 3484 cm3 = 700 m
1.2 = (30 × 0.3 × 5) – 2(4 × 2.5 × 0.3)
Thickness of the wood = 5mm = 39 m3
= 0.5 cm Hence, cost of papering = 700 ×
\ Internal breadth of box
geisnh
0.7 = Rs. 490 8
438. (d) (x + 2)3 – x3 = 1016 Volume of the bricks = 39 ×
= 24 – 2 × 0.5 = 23 cm 9
 x = 12 cm
Internal breadth of box 1
and x3 – (x – 2)3 = (12)2 – (10)2 (Since part is lime in the wall)
= 16 – 2 × 0.5 = 9 cm
= 728 9
\ Internal volume of the box
Enak

439. (b) Now let us consider that \ Number of bricks


= 23 × 15 × 9 = 3105 cm3
surface area of each face of the
\ Volume of the wood = 3840 – 39  8
cube 1 cm3. =
R

3105 = 735 cm3 9  0.2  016  0.08


\ Total surface area of the
Now, total weight of wood = cuboid = 14 cm2 = 13541.66 = 13600
Volume × weight of 1 cm3 wood 445. (a) Volume of water which flow
aryn

and Total surface area of the 3


7350 = 735 × weight of 1 cm3 cubes = 18 cm2 in 25 minutes
wood = 25 × 60 × 0.05 × 0.03 × 16
Les B

Hence, required ratio = 14 : 18


\ Weight of 1 cm3 wood = 10 gm =7:9 = 36 m3
434. (c) Length of tank = 120 m 440. (c) Iron used in the tube 36
= Difference in external and \ Rise in water level =
120 1 15 12
But since, = 17 , hence internal volumes of the tube
wa. th

7 7
1
17 cubes can be placed along = m = 0.2 m
5
length and breadth of tank = 80
cm 446. (b)h : b = 3 : 1 and l : h = 8 : 1
 l : h : b = 24 : 3 : 1
wwM

80 3 \ 24x × 3x × x = 36.846
But since, = 11 , hence 11
7 7 5 cm cm  x3 = 0.512  x = 0.8
8
cubes can be placed along \ h = 3x = 2.4 m
7 cm
breadth and height of tank 447. (b) lb = x, bh = y, hl = z
= 50 cm. \ 192 = 8x2 – 8(5)2 \ lb × bh × hl = xyz
 x = 7 cm
50 1  (lbh)2 = xyz  lbh = xyz
But since, = 7 , hence only Hence, the thickness of the tube
7 7 448. (c) Volume of the block = 14 ×
7 cubes can be placed along 10.5 × 11 cm3
height of the tank.

Rakesh Yadav Readers Publication Pvt. Ltd. 434

For More Visit : www.LearnEngineering.in


For More Visit : www.LearnEngineering.in

to the circumference of the base 458.(c) Let slant height = l and


21
Radius of the tank = = 10.5 cm of circular cone. radius = r
2
\ 2pR = 48p  R = 24 cm 1
Volume of the cylinder = pr2h v  r 2h 3v  r 2h
Now, since the slant height of 3
22 21 21 cone is equal to the radius of the
= × × ×h  9v2  2r4h2
7 2 2 original circular sheet. and C =  rl
Hence, l = 25 cm  C2 =  2r2l2 =  2r2 (h2 + r2)
22 21 21
\ × × × h = 14 × 10.5 × 11 [ l2 = h2 + r2]
7 2 2 h = 7 cm (\ l = r2 h2 )
 C =  r h +  2r4
2 2 2 2

14 2 Radius 24  3  vh3- c2h2 + 9v2


h= = 4 cm \
3 3 Height = 7 = (  r2h)  h3-(  2r2h2 +  2r4) h2

ir
449. (b) 2prh = 2 × p × 2 × 10 = 40p m2 +  2r4h2

v.iSn
450. (c) Sicne radius and height of BD =  2r 2h 4 -  2r 2h 4 -  2r 4h 2 +
454. (b) = cos 60°
the cylinder are same as that of AB  2r4h2 = 0
cone. Therefore cylinder can 3
BD 1
contain 15 × 3 = 45 litre of milk. = 459. (b)Side of Tetrahedren = H

dnag
AB 2 2
451.(b)
A 3
2a = P
2

eYari
30°30° 2 2
P= a
3
8
B 60° 60° C P² = a²
3
snhe
D
BD = CD , are the radii of the
base and AB = AC are the slant
heights of the cone. A is vertex
8
3P² = a² × 3
3
3P² = 8a²
kgei
Height of  AFB (FI) and BC is the base. 460. (c) A B
= 12 ²  5² = 13 cm 1
455. (a) Volume of cone = p × 144 × 35
3 . a
l
ERna

Height of  AED (EJ) h


Volume of water flowing per E
= = 15cm D C
12 ²  9² second = p × (0.8)2 ×
500
BE = height of pyramid = hm
Total surface Area = area of 60 BC = slant edge of pyramid = l
aBryn

(  AFB +  BGC +  CDH +  a


  144  35
 3  EC = base =
 AED + (  ABCD )   3
\ Required time = 500 =
 0.64  In  BEC,
1 1 60
=2× × 13 × 18 + 2 × × 15 BC² = EC² + BE²
4 4 315 seconds
Les

× 10 + 18 × 10 456. (d) Volume of frustum  a 2


l² =  
 + h²
wa. th

= 234 + 150 + 180 = 564 1  3


= π h (R² + Rr + r2)
V2 (1.4)3 2.744 3 a2
452. (a) = 3 = l=  h²
V1 (1) 1 1 22 3
= ×  6  16  8  4
\ % increase in volume
wwM

3 7 3
2.744 – 1  461.(a) Side of tetrahydren = H
1 22 528 2
=   × 100 = 174.4%
= ×  6  28 =

 1  3 7 3 3
453. (d) Area of circular sheet = 176 cm³ = ×4 3 =6 2
= 625p 2
Since length of arc and area of 1 Volume of terahydren
457. (a) p × (10)2 × 72 = p × (30)2 × h
sector are directly proportional 3
2 3
to the central angle. = × 6 2
 
8 12
Therefore, length of remaining  h=
3
96 2
arc = × 2 × p × 25 = 48p 2
= ×6 2 ×6 2 ×6 2
100 12
= 2 cm
But the remaining arc is equal 3 = 72cm³

Rakesh Yadav Readers Publication Pvt. Ltd. 435

For More Visit : www.LearnEngineering.in


For More Visit : www.LearnEngineering.in

462. (a) Volum e of r ain = 1. 5 AB² = 225 – 81 468. (a) Lateral sur face are a c
hectare×10cm AB = 12 cm ylinder (A) = 2  rh
1 BC 9  Base area is decreased by 1/9th
= 1.5×100×100× = 1500 cm³ sin = = times
10 AC 15
F
 side (radius) will decrease by 1/
463. (b) 3 time
3
A
18 B  = sin–1 = [ area d(side)2]
5
G
 new radius = r   r 3
J H 10 E
3
D C 2 = 2sin–1
5 and new height = h  6 h
I 465. (d) According to the given infor-
mation, the string will b e  new lateral surface area of cylin-

r
Area of base (ABCD) = 18 × 10
bounded on a cylindrical tank
= 180 r

Sni
as shown in the figure,  A/  2    6h  4rh
In  AFB  3
der

gv.i
GF² = (12)² + 5²
 A  22rh  2 A
/
GF = 13 cm
48cm
1 i.e. A /  2times of A.
Area of  AFB = 13 18

ridna
2 469. (b)
= 117 cm²
In  BEC 51
11

eeYa
EH² = (12)² + 9² The above figure, will look like the
EH = 15 cm figure (below), when we open it.
D C 4 2
1 12cm Inradius = =
Area of  BEC = 15 10 2 3 3
2
geisnh 12cm l = 2h
= 75 cm² 2
12cm  2 
Total surface area of Pyramid M (2h)² = h² +  
12cm  3
= 180 + 2 × 117 + 2 × 75
A 16cm B
= 180 + 234 + 150 = 564 cm² 4
Enak

The base circumference 4h² = h² +


464. (c) 15 3
144° 22 56 1 4
 2r  2    = 16cm 3h² =
7 11 2 3
R

 AM = length of one complete 4


h² =
9
turn  16 2  12 2
aryn

Remaining portion of circle of 2


= 20cm h=
 3
Les B

circumference = ×2r  Total length = 4  20 = 80cm


360º 1
466.(c) Whole surface area = cone : Volume of pyramid = × Area
3
216 22 hemisphere : cylinder of base × Height
= ×2× ×15 = 18
360 7 = r(l + r) : 3r² : 2r(r + h)
wa. th

1 3 2 8
Circumference of base of cone × × 4 × 4× = cm³
= r  2r  r : 3r² : 2r(r + r)
 3 4 3 3 3
= 18
2 r = 18 470. (b) Let the side of the cube and
= ( 2 + 1)r : 3r : 4r radius of the sphere be a and r
r=9
wwM

= ( 2 + 1) : 3 : 4 respectively
A 1
467. (c) Since, b l and 2(l + b) are in  2 2
GP, therefore  6a  4r  a  r   
2 2
 
 3 
l 2(l +b)
15 = 3
b l
 2 2
l r  
3

Suppose = x, V1 a3  3 
C b  =
B 9 V2 4 3 4 3
 1 r r
then x = 2  1   3 3
In  ABC,  x
AB² + BC² = AC²
2
 x – 2x – 2 = 0 2 2 3/2 3
=  = : 6
AB² + 9² = 15²  x= 3 +1 3 3 4

Rakesh Yadav Readers Publication Pvt. Ltd. 436

For More Visit : www.LearnEngineering.in


For More Visit : www.LearnEngineering.in

471. (a) V(cone) : V(hemisphere) :


OP 1 PC 810
V(cylinder)    h= 2  4cm
OQ 5 BQ 945
1 2 2 3 484. (a)
 r .r : r : r 2 .r Since, the ratio in radii of the
3 3 two cones is 1 : 5. 
 1: 2 : 3 There fore the radius of smaller
l
472. (d) Volume of original cube =
21
n  volume of smaller cubes cone ODC is = 4.2 cm.
5
3 3 3 r
n  27 4 3 1 2 Lateral surface area = 2  Base
1 1  1 478. (c) r  r h area
473. (c) H.C.F of 6,9 and 12 = 3 3 3

ir
(since radii of sphere and cone  rl  2r 2  l  2r
 Least possible number of

v.iSn
are same) r r 1
6  9  12 Sin   
cubes   24  4r = h l 2r 2 z
33 3    30
h 4

dnag
[ Note :- The cuboid is cut into    h : r  4 :1 h
smaller cubes of equal s i z e r 1 485. (c) cot    h  r cot .
i.e. size should be maximum] 479. (b) Area of large cube = 6(5)2 r
474. (b) It will be in the form of a right =150 (unit)2 let no. of cones  n

eYari
angled triangle. Area of cuboid = 2(1  1 + 1  125
r r
+ 125  1) = 502sq. units 
90° 90°  % increase in surface area=
a
h
a 502  150 2

475. (a) Arc of sector  2r


60 2r

150 snhe
 100  234 %

 No. of cubes
3
480. (a) H.C.F of 75, 15, 4.5 = 1.5 r
kgei
360 6
75154.5 1 
1500  n  r 2 r cot   r 2(2r cot )
=
1.51.51.5  3 
n  6
ERna

r Area of each cube = 6 (1.5) 2 486. (a)


Area of all the 1500 cubes
This arc of sector will be equal to = 1500  6  (1.5)2 = 20,250cm2 12 12
aBryn

the perimeter of cone. Let the radius of 481. (c) The solid with the least num-
cone be R, then: 30
ber of sides will have maximum
2r r surface area. So, tetrahedron 20
2R   R  , and slant
6 6 will have maximum surface 30
height of cone (l ) = radius of sec- area. Notice that in a sphere Total surface area of tank
Les

tor = r there are infinite number of TS A = 30  20+2(12  20) +


wa. th

2(30  12) =1800


r2 35 sides with least possible length.
h  l2  R2  r2   r So, the surface area of the  area of iron sheet = T.S.A
36 6
476. (d) Volume of water displaced = sphere will be least.  Length  width = 1800
volume of sphere 4 4 1800
 600 m
wwM

482. (d)  (r13  r2 3  r 3 3 )   (6) 3  Length =


4 3 3 3
 (30) 2  x   203
3  Cost = 600  10 = ` 6000
 27 +64 + r33 = 216
320 23 487.(d) Volume of prism = (Base area)
x  11 cm  r33 = 125  r = 5 cm  height.
27 27
483. (b) In radius (r) = 9cm Base Area = Area of trapezium
477. (a) ODC ~ OAB
Area (A) 1
O
= = 8 148  88
x x semiperimeter (s) 2
D
P C
h
45  1056 = 88 × height
4x
 A= 9  cm 2
2 1056
height = = 12cm
A Q B V= Area of base (A)  h 88

Rakesh Yadav Readers Publication Pvt. Ltd. 437

For More Visit : www.LearnEngineering.in


For More Visit : www.LearnEngineering.in

488. (b)  R3 = 64 4 2
 R=4  : 9:  4: 27: 2
3 3
 l = 60cm, b = 20cm, h = 12cm 497. (b)
 Total surface area A
= 2(lb + bh + hl)
= 2(1200 + 240 + 720) O’
Let radius of conical surface = r = 4320cm2 D E
 2r  60 % of 2R 493. (c)
A A
3
r  10  6cm B O C
5

r
12cm 13cm
and slant height of cone = l = R 13 12 13 Let DO` = r cm and OO` = h cm,

Sni
=10cm From similar triangles ADO` and
ABO,

gv.i
height(h)  l2 r2  (10)2 (6)2  8cm D C
B 5cm C B 5 A O  DO  9h r
r : h  6:8  3: 4 i.e. after revolution, a cone of    
AO BO 9 3
489. (c) we have , radius 5cm & height 12cm is

ridna
formed  9  h  3r  h  9  3r
2R1(R1  h )   (12 2  82 )
 volume of the cone 1
volume of frustum  h (r12  r1r2  r 2 )
80 40 1 3

eeYa
 R1  h   =  5 12 = 100
2

2R1 R1 3 1 22
40 40  R12  44   (9  3r )(9  r 2  3r )
h   R1  = 100  3.14 = 314cm3 3 7
R1 R1 2
 r 3 , V  r 2 .r  r 3
494. (a) V1 
geisnh 22
490. (a) Total surface area of pipe 3 2 44  3r 32 3r r 2
(hollow cylinder) 7
 2 ( R  r )h  ( R  r ) V3  2r 3  8r 3 447
  33  r 3
 V1  V2  V3 22
Enak

here  R  r  thickness  1cm, h  20cm


495. (b)Let r be the radius and h be 14 27r 3 r 3  2714 13
25 the height of the cone.
R= = 12.5cm
2 r  3 13 cm
R

Since,  AGC ~  AFE


 r  12.5-1  11.5 F E
498. (c)Let the height of cylinder be
D h cm and radius of base= r cm
aryn

22
Area = 2  (12.5 + 11.5)(20 + 1)  2  r2 + 2  rh = 462 (i)
7
Area of curved surface = 2π rh
Les B

= 44 × 72 = 3168 cm2 G
B C 1
491. (d) v 1  64 ltr v 2  216 ltr =  462 = 154
3
1
r1  64  3 4 2  2r 2 154  462
wa. th

     A
r2  216  6 3
AG GC h/ 2 r  2r 2  462 154  308
   GC  r 
 v  r 3  AF FE h 2 22 2
 
 volume of water filled in cone  2 r  308
7
wwM

2 2
A1  r1  4 1  r  h 1 1  V 3087
          r 2h   r 2   49  r  7cm
A2  r2  9 3 2  2 8 3  8 222
496. (c) Height of cone = height of
22
 v  r 2  cylinder = radius of hemisphere  2rh 154 2 7h 154
  = r units 7
492. (b) Let lenght (l ) = 15R  Ratio of the volumes of cone, 154 7
 breadth (b) = 5R cylinder and hemisphere h   cm
222 2
and height (h) = 3R 1 2
 volume (V) = lbh  r12 h :  r2 2 h : r 3  Volume of cylinder  r2h
3 3
= 15R  5R  3R
1 2 3 2 22 7
 14400 = 225R3 = 2 r : 32 r 3 : r 3  77  539 cm3
3 3 7 2

Rakesh Yadav Readers Publication Pvt. Ltd. 438

For More Visit : www.LearnEngineering.in


For More Visit : www.LearnEngineering.in

 2rh  264 507. (c) Volume of the new ball


499. (d)  2r 2  r r 2 h2
3 4
2
22
7h  264 =   r13  r2 3  r3 3 
 2r  r  h  4r  r h
2 2 2 2 2
7 4 3

 3r 2  h2  3r  h 
r

1
h 
264
6

  13  2 3  33 
h 3 222   1  8  27  36 cubic cm
r 4 r h Diameter 2  7 7
   k    4 36  3
500. (a)
h 3 4 3 Height 6 3  r 3  36  r 3   27
3 4
504. (b) Volume of bigger cube
r  4k;h  3k
= 6  6  6 = 216 cu. cm  r  3 27  3 cm

ir
 l  r2 h2  16k2  9k2 Volume of unit cube = 1  1  1 508. (b)

v.iSn
A
= 1 cu.cm
Number of uncoloured cubes
 25k 2  5k D M r E 40cm
= 4 4 4
= 64, because edge of uncoloured h

dnag
Curved surface area
 cube = 4 cm
Total surface area B C
N R
3
505. (c) Area of the base  6  2a 
2

π rl l 5k 5 4 Let the radius of small cone = r


    and height = h from the base,

eYari
π r(r  l ) r  l 4k  5k 9
3
 6 4a2  6 3a2sq.cm. 1 1 2 1
501. (c) Volume of the c ylinder 4   R h  r 2 40 h
64 3 3
 πr 2 h
 5a 
2

22
 101021  6600 cu.cm
7
Volume of the cone =6600 –
Height  snhe   2a 2
 2  
R2
r 2
8
 40 h  (i )
5

25 2 9a2 3  AME ~  ANC


kgei
4400 = 2200 cu.cm.  a 4a2   a cm.
4 4 2 40 h r
1   (ii )
 2200   102 h 40 R
 volume of pyramid
ERna

3
 from (i) and (ii)
2200 1
 2200  h  21 cm.  area of base  height 2
3  40  8
21     40  h  
 40  h  5
502. (c)Radius of the base = r units
aBryn

1 3
and height = h units  6 3a2  a  3 3a3cm3
 40  h   25  40  125  8
3
3 2
Curved surface of cylinder 2rh 506. (c)Volume of right prism =Area
   40  h  5  2  10
Curved surface of cone rl of the base  height
 10380 = 173  h  h  40  10  30cm
8 2h 4 h
Les

    Short Trick:-
5 l 5 h r 2
2 10380
wa. th

h   60 cm Bigger Smaller conecone


173 Ratio of vol. 64 1
16 h2 h2 r 2 25
  2 2  Ratio of (height
25 h r h2 16 3
3
64 = 4
Side
2
2 2
Now, Area of triangle 
r 25 r 25 9 4 1 =1 3
wwM

1   2  1 
h2 16 h 16 16 /radius / slant height)
3
Side
2

r 3

173  i.e. 4 represent = 40
4
h 4  4  40 cm
503. (b) Curved surface area of cyl- 173  4 173  4
inder = 2  rh  Side    20 cm 40
3 1.73  1 
= 10cm
and volume =  r2h 4
 Perimeter  320  60 cm  Required height = h
r 2h 924 r 924 = 40 – 10 = 30 cm
     Area of the lateral surface
2rh 264 2 264 509. (d) Let Edge reduced from 2  1
 Perimeter base  height
924×2 2 1
r = = 7metre  60 60  3600 sq.cm. 2 1
264

Rakesh Yadav Readers Publication Pvt. Ltd. 439

For More Visit : www.LearnEngineering.in


For More Visit : www.LearnEngineering.in

2 1  r2h  49h 25 h r
23  1  
 r 2  49 25 5
8  1
 r  49  7cm  h  25  5r (i)
Edge of the smaller cube  1 metre
 Didameter  14cm Volume of frustum (V)
 Required ratio  1: 64
1 2
514. (a) New volume  r h 1  2 2
510. (b) 2r  22 3  5 r 5r  h
3 
= 
22 227 7 1 1
 2r  h   r 2h
2
 2 r  22 r   cm
7 222 2 3 3 1 
 110   25  r 2  5r  25  5r 
3 

r
h  15 cm h
 4h  h h  

Sni
4
 Volume of the cylinder  r 2 h 515. (b) The diagonal of cube will be 5 
 5  r  (5 2  r 2  5r )  110
3 

gv.i
equal to the diameter of the
22 7 7 sphere.
   15  577.5cm3
7 2 2 5  3
 5  r 3   110
3
4 d   d3
3 

ridna
 volume of sphere     
511. (a) Volume of the original cone 3 2 6

1 d 110  3
 r 2h
 5 r 
3 3
and each side of cube = a =

eeYa
3 3 5
Volume of the new cone d3
 volume of cube  a3  110  3
1 1  3 3  125  r 3 
  4r 2h  2h  8  r 2h  22
3 
3 
geisnh
 Remaining volume 5
7
 Percentage increase  d3 d3
= –
 r  104 3 cm
1
6 3 3
1 
7  r 2h 
Enak

 3  d3   Short Trick :-
  1 
=  2 
= 1 ×100 = 700% 3 3  Volume of frustrum × 3
r 2h R3 – r 3 =
3
R

R
516. (d) 518. (a)
512. (c) Total surface area of prism
aryn

= Curved surface area + 2  Area


of base
Les B

 608 = P erim eter of


base  height + 2  Area of base
(Front view) (Side View)
 608 = 4x  15 + 2x2 Since, there are three faces
(Where x = side of square) which are visible in a corner Polished area = 4 π (r)2
wa. th

 x2 + 30x - 304 = 0 cube. When the cube of corner Non-polished area


 x2 + 38x - 8x - 304 = 0 is removed then the 3 faces of
other cubes will be visible from   r2 
 x (x + 38) - 8(x + 38) = 0 = 4   2    4r 2
outside. So, there will not be any
wwM

2 
 (x - 8) (x + 38) = 0 change in the surface area of  
 x=8 this solid figure. In the adjoining figure one of the
Volume = Base area × Height 517. (a) four parts of the sphere is
= 8² × 15 = 960 cm³ A
shown.(To understand it prop-
513. (b) If the length of the pipe be h erly, take an apple and cut it in
cm, then the four parts one across hori-
D r E 25cm
Its volume  r12 h  r2 2 h M zontal and another cut make
h vertical to it then you will notice
= h (r12  r22 ) = h (252  242 ) N that in a piece there are 2 semi-
D C
= 49h cu.cm. 5 circles.)
 AME ~  ANC Therefore required ratio = 1 : 1
 Volume of new cylinder

Rakesh Yadav Readers Publication Pvt. Ltd. 440

For More Visit : www.LearnEngineering.in


For More Visit : www.LearnEngineering.in

CHAPTER

QUADRILATERAL
18

QUADRILATERAL Add all 1


×13.2×64 = 422.4 cm²

ir
A plane figure bounded by four line 2(OA² + OB² + OC² + OD²) 2
= AB² + BC² + DC² + AD²

v.iSn
segments AB, BC, CD and DA is 1
called a quadrilateral. It is denoted 2(AB² + DC²) = AB² + BC² + DC² Area of DBCD = × CM×BD =
2
by symbol ‘ ’ i.e. ABCD. + AD² 1
AB² + DC² = BC² + AD² ×16.8 ×64 = 537.6 m²
2

dnag
2. Area of a quadrilateral  Area of quadrilateral ABCD
1 = Area of  ABD + Area of
= × One diagonal × sum of
2  BCD = 422.4 + 537.6 = 960

eYari
the perpendiculars drawn to cm²
the diagonals from the oppo- Alternate
site vertices. 1
Pairs of consecutive (adjacent) Area of ABCD = × BD × (h1 +
B 2
angles:
( A , B ), ( B , C ), ( C , D ),
( D , A )
A
h1
snhe h2 1
h 2) =
2
× 64 (13.2 + 16.8) = 32 × 30

Pairs of adjacent sides : = 960 cm2


kgei
(AB,BC), (BC,CD), (CD,DA) and 2. In a quadrilateral ABCD, it is
D C
given that BD = 16 cm. If AL
(DA,AB) Area (  ABCD) = Area
 ABD) + Area (  BDC)  BD and CM  BD such that AL
ERna

( = 9 cm and CM = 7 cm, then ar


Properties 1 1 (quad.ABCD)= ?
 Sum of four interior angles is = ×BD × h1   BD × h2 = D
2 2
360°. i.e.
aBryn

1
 A +  B +  C +  D = 360º ×BD×h1  h2  L

The figure formed by joining 2 A


M C
the mid-points of a quadrilat-
eral is a parallelogram. B
EXAMPLES
Les

1. If the diagonals intersect at (a) 256 cm² (b) 128 cm² (c) 64
1. ABCD is a quadrilateral in cm² (d) 96 cm²
wa. th

right angle then sum of square


which diagonal BD Sol. (b) Area of quadrilateral ABCD
of opposite sides are equal
AB² + DC² = AD² + BC² = 64 cm.AL  BD and CM  BD, 1
such that AL = ×BD(AL + CM)
2
B = 13.2 cm and CM = 16.8 cm.
wwM

A The area of the quadrilateral 1


ABCD in square centimeters is: = × 16 × (9 + 7) = 128 cm²
2
(a) 422.4 (b) 690.0 (c)
O
537.6 (d) 960.0
C 3. ABCD is a quadrilateral such
D Sol. (d)
D C that  D= 90º. A circle C (o,r)
Proof:- Apply Pythagoras theo- L touches the sides AB, BC, CD
rem in all right angled  and DA at P, Q, R and S respec-
OA² + OB² = AB² M tively. If BC = 38 cm, CD = 25
OB² + OC² = BC² A B cm and BP = 27 cm. Find r.
OD² + OC² = DC² 1 (a) 7 cm (b) 14 cm
Are a of DAB D = ×AL×BD = (c) 13 cm (d) 11 cm
OA² + OD² = AD² 2

Rakesh Yadav Readers Publication Pvt. Ltd. 441

For More Visit : www.LearnEngineering.in


For More Visit : www.LearnEngineering.in

Sol. (b) C Q B Properties if l || m, th en ar( ABCD )=


(i) The opposite sides are equal ar( ABPQ)

R .
O r P
and parallel.
(ii) Opposite angles are equal.
(A =  C) and
(xiv) if l || m, and ||gm ABCD
and  APB made on the same
base AB then,
(  B =  D)
D A (iii) Sum of any two adjacent
S angles are 180°.
(Two tangents drawn from an
(iv) Diagonals bisect each-other.
external point to a circle are
(v) Diagonals need not be equal
equal)
in length.
BP = BQ = 27 cm

r

(vi) Diagonals need not bisect at 1
CQ = BC – BQ=38–27=11cm

.iSn

i
right angle. ar(  APB)  ar ( ABCD)
 CQ = CR =11cm 2
(vii) Each diagonal divides a ||gm
 DR = CD – CR = 25–11=14 cm into two congruent triangles. (xv) Sum of square of diagonals is
 OR  CD and OS  AD (line

nagv
i.e .  ABC   AD C and equal to sum of square of all
drawn from centre to tangent
wi ll be pe rpen dicu lar to  ABD   BCD. sides
tangent) (viii) Bisectors of the angles of a AC² + BD² = AB² + BC² + CD² +

eYarid
 Radius (r) = DR = 14 cm ||gm form a rectangle. AD² = 2(AB² + BC²)
4. ABCD is a cyclic quadrilateral. i.e. PQRS is a rectangle.
If AB = 5 cm, CD = 15 cm, DA A B
4 0
180–0 0
= 13 cm. If diagonals intersect 4 3
at right angle. Find BC?

Sol. A 5 B
1
isnhe
1 2
2 D
0 18
0

You might also like